From henk at ripe.net Sat Jan 1 01:01:01 2011 From: henk at ripe.net (Henk Uijterwaal) Date: Sat, 01 Jan 2011 01:01:01 +0100 Subject: [BLML] List of BLML Abbreviations Message-ID: (Automated, regular posting) Usenet Bridge Abbreviations ABF Australian Bridge Federation AC Appeals committee ACBL American Contract Bridge League AI Authorised information ArtAS Artificial adjusted score AssAS Assigned adjusted score ATF Across-the-field [matchpointing] ATTNA Appeal to the National Authority BBL British Bridge League [now defunct] BGB Bridge Great Britain BIT Break in Tempo BLML Bridge-laws mailing list BoD Board of directors [ACBL] BoG Board of governors [ACBL] BOOT Bid-Out-Of-Turn CD Convention Disruption C&E Conduct and ethics [often hearings] CC Convention card CHO Center Hand Opponent [ie partner] CoC Conditions of contest COOT Call-Out-Of-Turn CoP Code of practice CPU Concealed partnership understanding CTD Chief Tournament director DBF Danish Bridge Federation DIC Director in charge DP Disciplinary penalty EBL European Bridge League EBU English Bridge Union EHAA Every Hand an Adventure [a system] F2F Face-to-face [to distinguish from Online bridge] FNJ Fit-Non-Jump (A non-jump bid in a new suit that implies a fit for partner's suit). FOLOOT Faced Opening-Lead-Out-Of-Turn FSF Fourth Suit Forcing GCC General Convention Chart [ACBL] HUM Highly Unusual Method IB Insufficient Bid IBLF International Bridge Laws Forum LA Logical alternative L&EC Laws & Ethics Committee [English, Welsh or Scottish] LHO Left hand Opponent Lnn Law number nn LOL Little old lady [may be of either sex] LOOT Lead-Out-Of-Turn MB Misbid ME Misexplanation MI Misinformation MPC Major penalty card mPC Minor penalty card MSC Master Solvers' Club [The Bridge World] NA National Authority NABC ACBL North American Bridge Championships NBB Nederlandse Bridge Bond [Dutch Bridge League] NBO National Bridge organisation NCBO National Contract Bridge organisation NIBU Northern Ireland Bridge Union NO Non-offender NOs Non-offenders NOS Non-offending side OBM Old Black Magic OBOOT Opening-Bid-Out-Of-Turn OKB OKBridge OLB Online bridge [to distinguish from Face-to-face bridge] OLOOT Opening-Lead-Out-Of-Turn OOT Out-Of-Turn Os Offenders OS Offending side pd Partner PLOOT Play-Out-Of-Turn POOT Pass-Out-Of-Turn PP Procedural Penalty PH Passed Hand RA Regulating Authority RGB rec.games.bridge [newsgroup] RGBO rec.games.bridge.okbridge [newsgroup] RHO Right Hand Opponent RLB Real Life Bridge [to distinguish from Online bridge] RoC Rule of coincidence RoW Rest of World [apart from North America] RTFLB Read the [fabulous] Law book! SAYC Standard American Yellow Card SBU Scottish Bridge Union SO Sponsoring organisation TBW The Bridge World [magazine] TD Tournament director TDic Tournament director in charge TFLB The [fabulous] Law book! UI Unauthorised information UPH UnPassed Hand WBF World Bridge Federation WBFLC WBF Laws Committee WBU Welsh Bridge Union YC Young Chelsea ZO Zonal organisation ZT Zero Tolerance [for unacceptable behaviour] Hand diagrams: 3m 3C or 3D [minor] 3M 3H or 3S [Major] ..3H 3H after a hesitation 3H! 3H alerted Cards and bids: H3 A card (3 of hearts) 3H A bid (3 hearts. The above may also be found on David Stevenson's Bridgepage at http://blakjak.com/usenet_br.htm From grandaeval at tiscali.co.uk Sat Jan 1 10:31:43 2011 From: grandaeval at tiscali.co.uk (Grattan) Date: Sat, 1 Jan 2011 09:31:43 -0000 Subject: [BLML] 1.1.11 Message-ID: Grattan Endicott References: <184B4C9981744CF4BC3A20BF59F0D334@Mildred> <4D1C4068.9070905@skynet.be> <1957070655.97086.1293699676312.JavaMail.ngmail@webmail18.arcor-online.net> <98830332.101890.1293710782116.JavaMail.ngmail@webmail13.arcor-online.net> <4D1C9FB6.9000808@gmail.com> <4D1DE110.9000607@skynet.be> <4D1E3DB5.6060709@gmail.com> Message-ID: <4D1F1252.60504@skynet.be> Hirsch Davis wrote: > > > I am not rectifying the failure to alert if there was no damage, nor did > I ever say that I would. > No, and I never said you wouldn't. I was merely pointing out that our rullings up to here are the same. Alain did not give UI, so there is no need to penalize Patrick for such. >>> We've been here before. In a real bridge game, the PP Alain received >>> for his intentional infraction would wipe out his above average on the >>> hand. Repeated intentional infractions would result in ejection and a >>> Conduct and Ethics hearing. >>> >> Indeed, there you go: a PP because a player infracted something his >> opponents were not damaged by. A PP because the player knew what he >> should be doing. > > NO! The PP is because the player knew what he should be doing and did > something else. > Indeed, I understand that. But who else but Alain and myself are liable for such a PP? Don't you see that there will be quite innocent players out there, who will act as Alain, and who will not get a PP. That is what I am trying to make you see. This regulation will only hurt those who know it exists, not those who don't know about it! And of course also not those who know about it, but keep schtum about that fact. >> So now we move to the next table, and the same happens. Are you going to >> give the same PP at that table? When the player in question simplu tells >> you "I thought my partner was right when he did not alert - I have misbid". >> > > If I believe that there was no intent to infract a Law, 72.B.1 does not > apply. > So this is what it has become to: an infraction (only because the WBF says so) that causes no damage and is without any penalty except a PP for those who are foolish enough to admit that they knew what they did was wrong. >>> My comments do not apply, of course, if the game in question is not >>> bridge as defined by the Laws as written and interpreted by the WBFLC. >>> However, it is my belief that such social games are not on topic for a >>> bridge laws mailing list, since said laws are not applicable to the game >>> in question. >>> >> I resent this. >> > Herman, > > You have my deepest apologies. I had believed that you were directing > games sponsored by the Belgian Bridge Federation/European Bridge > League. If in fact you simply direct social games with no WBF > affiliations, then of course you can impose dWS on your game, even if it > is not duplicate bridge as defined by the Laws, and I can see how you > would feel resentment at my suggestion of your social game being > off-topic. However, please bear in mind that any comments that you make > suggesting that the Laws not be followed as written apply to your game > only and do not apply to any games where the 2007 Laws of Duplicate > Bridge is the governing document. > You misunderstand my position. I am, on blml, argueing against the WBF interpretation. I will, as director, apply the laws that are put before me. Alain broke one of those laws, and his opponents did not call me - why should they? the were not damaged. If the opponents had called me, I had of course told Alain that I would have to give him a PP, and then it is up to me to choose the size of that PP. I happen to believe this infraction does not merit a high PP, and we shall disagree about that. But I do not see, not even after the Beijing pronouncement (which does not impose an automatic PP, does it?), that I am going against the laws and regulations. And I wish to return to my first question. What is so wrong about Alain's actions. To put it another way: if the Beijing interpretation were reversed, would the game of bridge suffer? > Hirsch > Best wishes for 2011 for everyone! -- Herman De Wael Wilrijk Antwerpen Belgium From swillner at nhcc.net Sat Jan 1 21:27:44 2011 From: swillner at nhcc.net (Steve Willner) Date: Sat, 01 Jan 2011 15:27:44 -0500 Subject: [BLML] The Final Answer for Serious Error Adjustments. Message-ID: <4D1F8E40.50005@nhcc.net> On Dec 25, Marvin French wrote: > (2)(a) None vulnerable. The table result is +980 for the OS and the > cost of the revoke > was 1030 points. Normal compensation for the NOS is 980+980=1860 > points. This is reduced by the cost of the revoke, 1030 points, > yielding 830 points of compensation, thereby improving the NOS table > result by 830 points. -980+830= -150 points, the proper adjusted > score for the NOS. The OS gets -980, a worse score than the -50 of > variant (1). Marvin has (temporarily?) unsubscribed, but he asked me to post a correction to the above. The "normal compensation" is 1960 points, so the adjusted score for the NOS should be -50. You could get the same answer by starting with the normal adjusted score (what you would give if no serious error), then subtracting the cost of the serious error (but never making the final result worse than the table score). My comments: I believe Marvin's "final answer" is what the Laws text actually says, but Ton's commentary disagrees. ACBL practice also seems to disagree, though I don't think it's entirely consistent. I don't see why we can't have consistent guidance on this subject, at least within each RA. (Does anyone happen to know whether the EBU White Book says anything on the subject?) Happy New Year to all! From nigelguthrie at yahoo.co.uk Sat Jan 1 22:41:59 2011 From: nigelguthrie at yahoo.co.uk (Nigel Guthrie) Date: Sat, 1 Jan 2011 21:41:59 +0000 (GMT) Subject: [BLML] The Final Answer for Serious Error Adjustments. In-Reply-To: <4D1F8E40.50005@nhcc.net> References: <4D1F8E40.50005@nhcc.net> Message-ID: <131798.20247.qm@web28505.mail.ukl.yahoo.com> [Steve Willner] My comments: I believe Marvin's "final answer" is what the Laws text actually says, but Ton's commentary disagrees. ACBL practice also seems to disagree, though I don't think it's entirely consistent. I don't see why we can't have consistent guidance on this subject, at least within each RA. (Does anyone happen to know whether the EBU White Book says anything on the subject?) [Nigel] Expert guidance may deflect BLML feeding-frenzy to another legal anomaly; but is of little use to the average club director unless inserted, in-place, in the law-book. From grandaeval at tiscali.co.uk Sun Jan 2 14:02:30 2011 From: grandaeval at tiscali.co.uk (Grattan) Date: Sun, 2 Jan 2011 13:02:30 -0000 Subject: [BLML] Decennial First Sunday. Message-ID: Sent: Sunday, January 02, 2011 > > Grattan Endicott **************************************************** > Skype directory: grattan.endicott > **************************************************** > "It is good to be merry and wise, > It is good to be honest and true, > It is best to be off with the old love > Before you are on with the new." > [Song - anon.] > ++++++++++++++++++++++++++++++++++++ (This is a copy of a contribution I have made to an in-house EBU topic. It may interest some.) ....................................................................................... +=+ In the edition of the 2007 laws published for the United Kingdom by the English Bridge Union Limited, in line with the page number at the top of each page, there are signposts to the first law that appears on the page*. These have been entered when publishing this edition. There is then a line across the page. The Laws of Duplicate Bridge are printed below that line and do not include whatever is printed above it. I have no comment upon the arrangement if it is thought convenient for users of the book, provided there is no confusion of extraneous material with the actual laws as promulgated by the WBF. It was the decision of the drafting committee that the 2007 Laws should run on and not be divided into Chapters, Parts and Sections, as were the Laws of Duplicate Contract Bridge previously. Largely this was triggered by the perception that the previous arrangement, confusingly, had statements in the laws in certain places that had effect or wholly belonged in other places outside of the Chapter/Part/Section in which they appeared. > ~ Grattan ~ +=+ (* i.e. no part of which appears on the previous page) From henk at ripe.net Mon Jan 3 09:49:28 2011 From: henk at ripe.net (Henk Uijterwaal) Date: Mon, 3 Jan 2011 09:49:28 +0100 Subject: [BLML] BLML Usage statistics Message-ID: <201101030849.p038nSpI022226@dog.ripe.net> BLML usage statistics for December 2010 Posts From ----- ---- 59 agot (at) ulb.ac.be 49 richard.hills (at) immi.gov.au 38 ehaa (at) starpower.net 31 nigelguthrie (at) yahoo.co.uk 26 blml (at) arcor.de 21 jfusselman (at) gmail.com 19 Hermandw (at) skynet.be 17 svenpran (at) online.no 14 PeterEidt (at) t-online.de 13 mfrench1 (at) san.rr.com 10 sater (at) xs4all.nl 10 harald.skjaran (at) gmail.com 9 jean-pierre.rocafort (at) meteo.fr 9 grandaeval (at) tiscali.co.uk 8 swillner (at) nhcc.net 8 Ron.Johnson (at) NRCan-RNCan.gc.ca 5 bpark56 (at) comcast.net 5 JffEstrsn (at) aol.com 4 adam (at) tameware.com 3 t.kooyman (at) worldonline.nl 3 richard.willey (at) gmail.com 3 rfrick (at) rfrick.info 3 petrus (at) stift-kremsmuenster.at 3 lavaldubreuil (at) xplornet.com 3 jeff.ford (at) gmail.com 3 hirsch9000 (at) gmail.com 3 grabiner (at) alumni.princeton.edu 3 gampas (at) aol.com 3 ardelm (at) optusnet.com.au 2 jrhind (at) therock.bm 2 info (at) honorsbridgeclub.org 2 henk (at) ripe.net 2 gordonrainsford (at) btinternet.com 2 adam (at) irvine.com 1 roger-eymard (at) orange.fr 1 mikopera (at) nyc.rr.com 1 mikeamostd (at) btinternet.com 1 madam (at) civilradio.hu 1 lumenco (at) ono.com 1 larry (at) charmschool.orangehome.co.uk 1 jmmgc1 (at) hotmail.com 1 diggadog (at) iinet.net.au 1 craigstamps (at) comcast.net 1 bridge (at) vwalther.de From agot at ulb.ac.be Mon Jan 3 16:08:20 2011 From: agot at ulb.ac.be (Alain Gottcheiner) Date: Mon, 03 Jan 2011 16:08:20 +0100 Subject: [BLML] interesting dWS case In-Reply-To: <116755.85439.qm@web28516.mail.ukl.yahoo.com> References: <4D18BD6E.2020106@ulb.ac.be> <574858.6894.qm@web28513.mail.ukl.yahoo.com> <4D19B21A.9000005@skynet.be> <116755.85439.qm@web28516.mail.ukl.yahoo.com> Message-ID: <4D21E664.7090708@ulb.ac.be> Le 28/12/2010 16:06, Nigel Guthrie a ?crit : > {Herman de Wael] > [SNIP] > Please inform us, Nigel, what terrible act Alain has done. He has not > > informed, in due time, that 2S showed, according to system, womething > that Patrick did not in fact have. Nevertheless, Alain submits to > rectification for any damage that such an omission might have caused. > Please tell us Nigel, why you believe that the act that Alain has done > would be regarded as so much worse than the infraction that Patrick has > comitted, in not alerting the 1S bid and explaining it as showing > diamonds? Now there's an infraction if ever I saw one. > {SNIP] > > [Nigel] > I agree with Herman that current disclosure rules are improvable. On BLML I, > too, have suggested radical simplification. Nevertheless, until the rules are > changed, I think we should comply with them. > > AG : I'd like to add something here. In Brussels, there are quite a few dWSists and "extra-alerters" of doubles etc. Some are considered the most ethical players around (names given on request). And some who, for other reasons, aren't (no names, of course), keep complying with "main school" requirements. Knowing them, I'm wondering whether it could be because it helps them in their shenanigans. (don't ask me why they're still around after their infractions have been duly noted. Something is roten in this kingdom too) Best regards Alain From agot at ulb.ac.be Mon Jan 3 16:17:20 2011 From: agot at ulb.ac.be (Alain Gottcheiner) Date: Mon, 03 Jan 2011 16:17:20 +0100 Subject: [BLML] interesting dWS case [SEC=UNOFFICIAL] In-Reply-To: References: Message-ID: <4D21E880.205@ulb.ac.be> Le 30/12/2010 7:05, richard.hills at immi.gov.au a ?crit : > > Richard Hills: > > Those who have read Victor Mollo's stories about Bridge in the > Menagerie know that Charlie the Chimp is a character with grey > ethics. Really ? I seem to remember that Charlie's main problem is that he always focuses on some other deal, and that makes his bidding a bit chaotic. And that is what I intended to mean. From agot at ulb.ac.be Mon Jan 3 16:26:52 2011 From: agot at ulb.ac.be (Alain Gottcheiner) Date: Mon, 03 Jan 2011 16:26:52 +0100 Subject: [BLML] interesting dWS case [SEC=UNOFFICIAL] In-Reply-To: <98830332.101890.1293710782116.JavaMail.ngmail@webmail13.arcor-online.net> References: <184B4C9981744CF4BC3A20BF59F0D334@Mildred> <4D1C4068.9070905@skynet.be> <1957070655.97086.1293699676312.JavaMail.ngmail@webmail18.arcor-online.net> <98830332.101890.1293710782116.JavaMail.ngmail@webmail13.arcor-online.net> Message-ID: <4D21EABC.90501@ulb.ac.be> Le 30/12/2010 13:06, Thomas Dehn a ?crit : > Grattan wrote: >> Grattan Endicott> **************************************************** >> Skype directory: grattan.endicott >> **************************************************** >> ----- Original Message ----- >> From: "Thomas Dehn" >> To: >> Sent: Thursday, December 30, 2010 9:01 AM >> Subject: Re: [BLML] interesting dWS case [SEC=UNOFFICIAL] >> >> >>> I still think that the dWS is wrong. Not because of >>> what TFLB says, but in practical terms. >>> >> +=+ I am in doubt as to what dWS is or its implications. >> However, from what (little) I have read of this topic >> the reference to TFLB is misplaced since wherever this >> episode occurred it was not in a competition played >> under the Laws of Duplicate Bridge. > There exist two philosophies on how to deal with defects > in a law. > > 1. "This law is a bad law, but nevertheless I will follow it literally" > > 2. "This law is a bad law. I will violate it, and do something sensible instead". > AG : I would like to point out that saving the normal course of the board, as was the case here, is more that merely doing something sensible ; it is fair to other competitors, especially in Butler scoring, where your possible absurd score in 6S doubled will influence theirs. And in my opinion that's the biggest argument in favor of using dWS when obvious. but you indeed described my state of mind correctly. From agot at ulb.ac.be Mon Jan 3 16:31:51 2011 From: agot at ulb.ac.be (Alain Gottcheiner) Date: Mon, 03 Jan 2011 16:31:51 +0100 Subject: [BLML] interesting dWS case In-Reply-To: <4D1C96FD.3060802@meteo.fr> References: <4D18BD6E.2020106@ulb.ac.be> <4D1C96FD.3060802@meteo.fr> Message-ID: <4D21EBE7.6080302@ulb.ac.be> Le 30/12/2010 15:28, Jean-Pierre Rocafort a ?crit : > Alain Gottcheiner a ?crit : >> Hi all, >> >> Here is what happened yesterday in one of the few Belgian torunaments >> that count, directed by a Mr. hDW. >> >> You're North, 3rd-in-hand, and hold : >> >> xxx >> KJx >> AKJxxx >> J >> >> South opens 1C. You answer 1S (T-Walsh - usually diamonds but might be a >> balanced club raise). >> Partner does NOT alert and bids 2S. What do you do ? >> >> I did nothing. >> I mean, 2S is alertable (it is a reverse, similar to 1C-1NT-2S), yet I >> didn't alert. > i tend to sympathise with your attitude and, after rethinking, it's > because i don't understand why 2S should be alertable. it's obviously a > reverse, what need to alert that a reverse is a reverse? if partner had > alerted 1S, opponent had asked and partner explained, would you have > alerted 2S? AG : what if partner alerted 1S and they didn't ask ? I would have to alert, because they wouldn't know what the alert was about (subject formerly discussed a couple of times on blml). Some alert "Walsh" 1S , and "double alerts", alas, aren't used yet in my country. They could think that was the case ... So, to which of those two case should I refer ? (and please notice that some TDs would require an alert of 2S after the explanation of 1S, too, because "opponents are not expected to be able to work out your system's subtle points) From agot at ulb.ac.be Mon Jan 3 16:36:06 2011 From: agot at ulb.ac.be (Alain Gottcheiner) Date: Mon, 03 Jan 2011 16:36:06 +0100 Subject: [BLML] interesting dWS case [SEC=UNOFFICIAL] In-Reply-To: <4D1C9FB6.9000808@gmail.com> References: <184B4C9981744CF4BC3A20BF59F0D334@Mildred> <4D1C4068.9070905@skynet.be> <1957070655.97086.1293699676312.JavaMail.ngmail@webmail18.arcor-online.net> <98830332.101890.1293710782116.JavaMail.ngmail@webmail13.arcor-online.net> <4D1C9FB6.9000808@gmail.com> Message-ID: <4D21ECE6.1030708@ulb.ac.be> Le 30/12/2010 16:05, Hirsch Davis a ?crit : > On 12/30/2010 7:06 AM, Thomas Dehn wrote: >> Grattan wrote: >>> Grattan Endicott>> **************************************************** >>> Skype directory: grattan.endicott >>> **************************************************** >>> ----- Original Message ----- >>> From: "Thomas Dehn" >>> To: >>> Sent: Thursday, December 30, 2010 9:01 AM >>> Subject: Re: [BLML] interesting dWS case [SEC=UNOFFICIAL] >>> >>> >>>> I still think that the dWS is wrong. Not because of >>>> what TFLB says, but in practical terms. >>>> >>> +=+ I am in doubt as to what dWS is or its implications. >>> However, from what (little) I have read of this topic >>> the reference to TFLB is misplaced since wherever this >>> episode occurred it was not in a competition played >>> under the Laws of Duplicate Bridge. >> There exist two philosophies on how to deal with defects >> in a law. >> >> 1. "This law is a bad law, but nevertheless I will follow it literally" > HD: Since the game is defined by its Laws, this is the only viable > option in play, whatever your philosophical leanings. > AG : you're playing soccer. A player gets seriously wounded, and the referee doesn't realize this.. What would you do ? Me, I would find some way to interrupt the game, perhaps taking the ball into my hands. And, with all due respect to Levi-Strauss and Huizinga, those who would say to themselves "I can't pick the ball, it's against the rules" are wrong. From svenpran at online.no Mon Jan 3 17:32:42 2011 From: svenpran at online.no (Sven Pran) Date: Mon, 3 Jan 2011 17:32:42 +0100 Subject: [BLML] interesting dWS case [SEC=UNOFFICIAL] In-Reply-To: <4D21ECE6.1030708@ulb.ac.be> References: <184B4C9981744CF4BC3A20BF59F0D334@Mildred> <4D1C4068.9070905@skynet.be> <1957070655.97086.1293699676312.JavaMail.ngmail@webmail18.arcor-online.net> <98830332.101890.1293710782116.JavaMail.ngmail@webmail13.arcor-online.net> <4D1C9FB6.9000808@gmail.com> <4D21ECE6.1030708@ulb.ac.be> Message-ID: <000901cbab63$d866f850$8934e8f0$@no> On Behalf Of Alain Gottcheiner ............... > AG : you're playing soccer. A player gets seriously wounded, and the referee > doesn't realize this.. > > What would you do ? Me, I would find some way to interrupt the game, perhaps > taking the ball into my hands. And, with all due respect to Levi-Strauss and > Huizinga, those who would say to themselves "I can't pick the ball, it's against the > rules" are wrong. I have noticed a major difference between soccer and (duplicate) bridge: In soccer the players are supposed to violate the rules if their team can benefit. For instance if the only way to prevent opponents score a goal is to tackle in such a way that it will result in a free kick, or maybe even a penalty kick, you tackle. If you don't then you are considered too chicken for this game. And as for the player who is down without the referee noticing I believe there is a well known universal practice for that situation: The ball is kicked out over the sideline; and when eventually the game is continued the throw in is made towards an opposing team player so that the team that interrupted the game doesn't lose. But bridge is a game where you do not violate the rules even if you are willing to "pay the penalty"; that makes a comparison between soccer and bridge completely irrelevant. From agot at ulb.ac.be Mon Jan 3 18:18:21 2011 From: agot at ulb.ac.be (Alain Gottcheiner) Date: Mon, 03 Jan 2011 18:18:21 +0100 Subject: [BLML] interesting dWS case [SEC=UNOFFICIAL] In-Reply-To: <000901cbab63$d866f850$8934e8f0$@no> References: <184B4C9981744CF4BC3A20BF59F0D334@Mildred> <4D1C4068.9070905@skynet.be> <1957070655.97086.1293699676312.JavaMail.ngmail@webmail18.arcor-online.net> <98830332.101890.1293710782116.JavaMail.ngmail@webmail13.arcor-online.net> <4D1C9FB6.9000808@gmail.com> <4D21ECE6.1030708@ulb.ac.be> <000901cbab63$d866f850$8934e8f0$@no> Message-ID: <4D2204DD.6010506@ulb.ac.be> Le 3/01/2011 17:32, Sven Pran a ?crit : > On Behalf Of Alain Gottcheiner > ............... >> AG : you're playing soccer. A player gets seriously wounded, and the > referee >> doesn't realize this.. >> >> What would you do ? Me, I would find some way to interrupt the game, > perhaps >> taking the ball into my hands. And, with all due respect to Levi-Strauss > and >> Huizinga, those who would say to themselves "I can't pick the ball, it's > against the >> rules" are wrong. > I have noticed a major difference between soccer and (duplicate) bridge: > > In soccer the players are supposed to violate the rules if their team can > benefit. For instance if the only way to prevent opponents score a goal is > to tackle in such a way that it will result in a free kick, or maybe even a > penalty kick, you tackle. If you don't then you are considered too chicken > for this game. AG : OK, perhaps bad example. Football is no game anymore, perhaps. So let's stick to bridge. Here is a problem for you (happened more than once) : LHO makes some (alerted) bid you're familiar with. RHO makes some (alerted) response you're unfamiliar with. The bad-but-to-be-obeyed rules of this competition say that : a) you may not ask just about the last bid, but only about all alerted, yet non-explained bids b) you may not ask about a bid whose meaning you know How do you intend to follow the rules ? Perhaps b) is less important if partner is awake, so you forget b). Aren't you deliberately violating the game's rules ? If you consider it normal that, because you know the meaning of LHO's bid, you aren't entitled to know the meaning of RHO's bid (strict interpretation of the rules), than you are more than a masochist in my view. Or, other real-life example, partner overcalls 2H over 1NT, meaning hearts and a minor, alerted and explained. The LOL on your right acts accordingly. On the next deal, partner, a passed hand, overcalls 2H over 1NT, meaning in this case 5S+4H. The LOL nods. You tell her "you'd better ask". You've broken a rule. So what ? (in fact, this is a good comparison. The aim is to avoid opponents' feeling that they might have been robbed and to spare the TD headaches) Best regards Alain From jean-pierre.rocafort at meteo.fr Mon Jan 3 19:28:24 2011 From: jean-pierre.rocafort at meteo.fr (Jean-Pierre Rocafort) Date: Mon, 03 Jan 2011 19:28:24 +0100 Subject: [BLML] interesting dWS case In-Reply-To: <4D21EBE7.6080302@ulb.ac.be> References: <4D18BD6E.2020106@ulb.ac.be> <4D1C96FD.3060802@meteo.fr> <4D21EBE7.6080302@ulb.ac.be> Message-ID: <4D221548.50205@meteo.fr> Alain Gottcheiner a ?crit : > Le 30/12/2010 15:28, Jean-Pierre Rocafort a ?crit : >> Alain Gottcheiner a ?crit : >>> Hi all, >>> >>> Here is what happened yesterday in one of the few Belgian torunaments >>> that count, directed by a Mr. hDW. >>> >>> You're North, 3rd-in-hand, and hold : >>> >>> xxx >>> KJx >>> AKJxxx >>> J >>> >>> South opens 1C. You answer 1S (T-Walsh - usually diamonds but might be a >>> balanced club raise). >>> Partner does NOT alert and bids 2S. What do you do ? >>> >>> I did nothing. >>> I mean, 2S is alertable (it is a reverse, similar to 1C-1NT-2S), yet I >>> didn't alert. >> i tend to sympathise with your attitude and, after rethinking, it's >> because i don't understand why 2S should be alertable. it's obviously a >> reverse, what need to alert that a reverse is a reverse? if partner had >> alerted 1S, opponent had asked and partner explained, would you have >> alerted 2S? > AG : what if partner alerted 1S and they didn't ask ? I would have to > alert, because they wouldn't know what the alert was about (subject > formerly discussed a couple of times on blml). Some alert "Walsh" 1S , > and "double alerts", alas, aren't used yet in my country. They could > think that was the case ... i am strongly opposed to the notion of multi-level alerts. if you institute 2 levels, players will assume something for 1st level, another thing for 2nd level and you will soon need to have a 3rd level, and so on. it would only make players less attentive and more lawyer. i think the worst alert regulations are those with a list of calls to be alerted and the best ones those only telling to alert when you know from agreement or partnership experience something the opponents don't know and would better ask. if they don't ask when they are told it would be in their interest, it's at their own risk, particularly if this prevents them from understanding further bids which would be obvious if they had inquired in the first place. ultimately, in order to reduce useless alerts, it may be better not to alert artificial calls when you are certain your present opponents won't be mistaken. and more, it's less serious to forget to alert a bid that can't have an obvious meaning (maybe a 2NT overcall of a 1NT opening) than an insidious bid such as 1S transfer response to 1C opening. > > So, to which of those two cases should I refer ? i am not sure to understand your question. it would be nice to educate players to alert "useful" (for opponents) and to educate them never to assume. jpr > > (and please notice that some TDs would require an alert of 2S after the > explanation of 1S, too, because "opponents are not expected to be able > to work out your system's subtle points) > > -- _______________________________________________ Jean-Pierre Rocafort METEO-FRANCE DSI/CM 42 Avenue Gaspard Coriolis 31057 Toulouse CEDEX Tph: 05 61 07 81 02 (33 5 61 07 81 02) Fax: 05 61 07 81 09 (33 5 61 07 81 09) e-mail: jean-pierre.rocafort at meteo.fr Serveur WWW METEO-France: http://www.meteo.fr _______________________________________________ From richard.hills at immi.gov.au Mon Jan 3 23:26:50 2011 From: richard.hills at immi.gov.au (richard.hills at immi.gov.au) Date: Tue, 4 Jan 2011 09:26:50 +1100 Subject: [BLML] uninteresting dWikiS case [SEC=UNOFFICIAL] In-Reply-To: <4D2204DD.6010506@ulb.ac.be> Message-ID: Hirsch Davis: >>..... >>NO! The PP is because the player knew what he should be >>doing and did something else. >>..... >>please bear in mind that any comments that you make >>suggesting that the Laws not be followed as written apply to >>your game only and do not apply to any games where the 2007 >>Laws of Duplicate Bridge is the governing document. >> >>Hirsch Alain Gottcheiner: >..... >Or, other real-life example, partner overcalls 2H over 1NT, >meaning hearts and a minor, alerted and explained. The LOL on >your right acts accordingly. >On the next deal, partner, a passed hand, overcalls 2H over >1NT, meaning in this case 5S+4H. The LOL nods. You tell her >"you'd better ask". >You've broken a rule. >..... Richard Hills: No, Alain broke two rules. And his second infraction of the local Alert regulation would have been unnecessary if Alain had initially explained in accordance with Law 40B6(a), first phrase: "When explaining the significance of partner's call or play in reply to opponent's enquiry (see Law 20) a player shall disclose all special information conveyed to him through partnership agreement or partnership experience ..... " Richard Hills: Thus Alain's explanation of his partner's first 2H overcall over 1NT should have disclosed _all_ the special information conveyed to Alain by partnership agreement: "Because partner is not a passed hand, 2H promises hearts and a minor. But whenever partner is a passed hand, an overcall of 2H over 1NT would now promise five spades and four hearts." Best wishes Richard Hills Recruitment Section Specialist Recruitment Team Level 5 Aqua, workstation W569, 6223 8453 DIAC Social Club movie tickets -------------------------------------------------------------------- Important Notice: If you have received this email by mistake, please advise the sender and delete the message and attachments immediately. This email, including attachments, may contain confidential, sensitive, legally privileged and/or copyright information. Any review, retransmission, dissemination or other use of this information by persons or entities other than the intended recipient is prohibited. DIAC respects your privacy and has obligations under the Privacy Act 1988. The official departmental privacy policy can be viewed on the department's website at www.immi.gov.au. See: http://www.immi.gov.au/functional/privacy.htm --------------------------------------------------------------------- From richard.hills at immi.gov.au Tue Jan 4 00:07:49 2011 From: richard.hills at immi.gov.au (richard.hills at immi.gov.au) Date: Tue, 4 Jan 2011 10:07:49 +1100 Subject: [BLML] uninteresting dWikiS case [SEC=UNOFFICIAL] In-Reply-To: Message-ID: Thomas Dehn: >>>There exist two philosophies on how to deal with defects in >>>a law. >>> >>>1. "This law is a bad law, but nevertheless I will follow it >>> literally" >>> >>>2. "This law is a bad law. I will violate it, and do >>> something sensible instead". Richard Hills: What is (or should be) the only correct philosophy on how to deal with a defect in a Duplicate Bridge Law (or regulation) -> 3. This Law is a bad Law, I will follow it literally unless and until it can be amended, and I will actively strive for its amendment through appropriate channels. Grattan Endicott: >>+=+ If a tournament is specified to be played under the Laws >>of Duplicate Bridge there is a contract between the entrant >>and the tournament organizer that the specified laws will be >>applied. >> As to your item (2.) you are surely not suggesting that >>there are Directors so arrogant and bumptious as to replace >>the appointed law with a substitute of their own devising? >> ~ Grattan ~ +=+ Robert Frick: >I know maybe you are just tongue-in-cheek here. But in case >you are not -- as far as I know, the claim laws and UI laws >are too incompetently written to be actually followed. Richard Hills: The foundation issue is whether the rules of a mind game (for example, Duplicate Bridge) should be obeyed. Merely a secondary meta-issue is whether the Laws of Duplicate Bridge can be improved in clarity (for example by commentary from the EBU White Book). Understanding the Internet: Model, Metaphor and Analogy T.G. McFadden, page 96: "Or we might be very confused, as was the tourist in Oxford who, after seeing all of the colleges and the Bodleian Library, still asked 'But where is the University?' Gilbert Ryle (1949) famously called this error a 'category mistake.' Our tourist was mistakenly allocating the university to the same category as that to which the other institutions belong" Richard Hills: It is a category mistake for an arrogant and bumptious Director to argue that he is entitled to violate _any_ Law merely because he lacks the diligence and/or inclination to seek an understanding of a few poorly written Laws. Best wishes Richard Hills Recruitment Section Specialist Recruitment Team Level 5 Aqua, workstation W569, 6223 8453 DIAC Social Club movie tickets -------------------------------------------------------------------- Important Notice: If you have received this email by mistake, please advise the sender and delete the message and attachments immediately. This email, including attachments, may contain confidential, sensitive, legally privileged and/or copyright information. Any review, retransmission, dissemination or other use of this information by persons or entities other than the intended recipient is prohibited. DIAC respects your privacy and has obligations under the Privacy Act 1988. The official departmental privacy policy can be viewed on the department's website at www.immi.gov.au. See: http://www.immi.gov.au/functional/privacy.htm --------------------------------------------------------------------- From adam at irvine.com Tue Jan 4 00:51:06 2011 From: adam at irvine.com (Adam Beneschan) Date: Mon, 03 Jan 2011 15:51:06 -0800 Subject: [BLML] Still out of order In-Reply-To: Your message of "Sat, 25 Dec 2010 09:32:38 GMT." <4B27E00E623D47D2A2BAE4C21B48C409@Mildred> Message-ID: <20110103235109.EE29BA8C837@mailhub.irvine.com> Grattan wrote: > Christmas wishes to all. > ++++++++++++++++++++++++++++++++++++ > +=+ I have not read this topic in detail. I can offer > two pieces of information. My files show > (i) that the drafting committee intended > no change in the effect of Law 57 between 1997 > and 2007, merely tidying up the consistency. > (ii) there was discussion whether to include > a reference to Law 23. Those who said 'no' won > the day. They made the case that Law 23 applies > generally, in respect of any irregularity, and does > not need a special mention. > ~ Grattan ~ +=+ > ........................................................... Thanks, Grattan, that was the sort of information I was looking for. I still believe that when the drafting committee changed the language, they accidentally confused the issue. To me, the clear and simple reading of the new wording is that, when a singleton is involved, Law 57C1 is supposed to apply only when there has been instruction from the declarer. To come to the other (correct) conclusion, that it also applies if dummy plays the singleton without declarer's instruction, requires some mental gymnastics, logical analysis, and maybe research into other Laws, especially since that conclusion *seems* to contradict the new language in the second sentence of 57C1. The right answer is not at all obvious, no matter what anyone says. Some of the posters here have made some decent arguments about why 57C1 should still apply in that case, but a director shouldn't have to figure all that out. I think the committee erred. Not that I think there's any urgency that this be fixed, since this must be a very rare situation if it occurs at all. -- Adam From richard.hills at immi.gov.au Tue Jan 4 01:19:07 2011 From: richard.hills at immi.gov.au (richard.hills at immi.gov.au) Date: Tue, 4 Jan 2011 11:19:07 +1100 Subject: [BLML] uninteresting dWikiS case [SEC=UNOFFICIAL] In-Reply-To: <1955550077.152466.1293814067338.JavaMail.ngmail@webmail07.arcor-online.net> Message-ID: Law 72B1: "A player MUST NOT infringe a law intentionally, even if there is a prescribed rectification he is willing to accept." Law 81C: "The Director's duties and powers normally include also the following: [...] 5. to waive rectification for cause, in his discretion, upon the request of the non-offending side." Richard Hills: But to "waive rectification" is NOT synonymous with "cancel infraction". A Law 72B1 intentional infraction remains a Law 72B1 intentional infraction whether or not Law 81C happens to be subsequently invoked. Thus an ethical player who is playing Duplicate Bridge, not playing social bridge, is obliged to obey Law 72B1. Alain Gottcheiner: >AG : I'd like to add something here. >In Brussels, there are quite a few dWSists and "extra- >alerters" of doubles etc. >Some are considered the most ethical players around (names >given on request). Richard Hills: No, those players are unintentionally unethical. They fondly but incorrectly believe that obeying The Unwritten Laws of Duplicate Bridge is ethical. However, it is the actual Laws of Duplicate Bridge which necessarily define bridge ethics. Alain Gottcheiner: >And some who, for other reasons, aren't (no names, of >course), keep complying with "main school" requirements. >Knowing them, I'm wondering whether it could be because it >helps them in their shenanigans. Richard Hills: It is not "main school" to obey the Lawbook and regulations, it is the "ONLY school". And if a Secretary Bird does so, that is not an "unethical shenanigan" (as stated by The Unwritten Laws of Duplicate Bridge) but rather "complying with the lawful procedures and ethical standards set out in these laws" (as stated by Law 72A). Best wishes Richard Hills Recruitment Section Specialist Recruitment Team Level 5 Aqua, workstation W569, 6223 8453 DIAC Social Club movie tickets -------------------------------------------------------------------- Important Notice: If you have received this email by mistake, please advise the sender and delete the message and attachments immediately. This email, including attachments, may contain confidential, sensitive, legally privileged and/or copyright information. Any review, retransmission, dissemination or other use of this information by persons or entities other than the intended recipient is prohibited. DIAC respects your privacy and has obligations under the Privacy Act 1988. The official departmental privacy policy can be viewed on the department's website at www.immi.gov.au. See: http://www.immi.gov.au/functional/privacy.htm --------------------------------------------------------------------- From grandaeval at tiscali.co.uk Mon Jan 3 23:29:58 2011 From: grandaeval at tiscali.co.uk (Grattan) Date: Mon, 3 Jan 2011 22:29:58 -0000 Subject: [BLML] interesting dWS case [SEC=UNOFFICIAL] References: <184B4C9981744CF4BC3A20BF59F0D334@Mildred> <4D1C4068.9070905@skynet.be> <1957070655.97086.1293699676312.JavaMail.ngmail@webmail18.arcor-online.net> <98830332.101890.1293710782116.JavaMail.ngmail@webmail13.arcor-online.net> <4D1C9FB6.9000808@gmail.com> <4D21ECE6.1030708@ulb.ac.be><000901cbab63$d866f850$8934e8f0$@no> <4D2204DD.6010506@ulb.ac.be> Message-ID: Grattan Endicott To: "Bridge Laws Mailing List" Sent: Monday, January 03, 2011 5:18 PM Subject: Re: [BLML] interesting dWS case [SEC=UNOFFICIAL] Le 3/01/2011 17:32, Sven Pran a ?crit : > On Behalf Of Alain Gottcheiner The bad-but-to-be-obeyed rules of this competition say that : a) you may not ask just about the last bid, but only about all alerted, yet non-explained bids b) you may not ask about a bid whose meaning you know +=+ This is, then, a competition in which Law 20F3 does not apply? ~ G ~ +=+ From richard.hills at immi.gov.au Tue Jan 4 05:20:02 2011 From: richard.hills at immi.gov.au (richard.hills at immi.gov.au) Date: Tue, 4 Jan 2011 15:20:02 +1100 Subject: [BLML] uninteresting dWikiS case [SEC=UNOFFICIAL] In-Reply-To: Message-ID: Alain Gottcheiner: >>The bad-but-to-be-obeyed rules of this competition say that: >> >>a) you may not ask just about the last bid, but only about >>all alerted, yet non-explained bids >> >>b) you may not ask about a bid whose meaning you know Grattan Endicott: >+=+ This is, then, a competition in which Law 20F3 does not >apply? ~ G ~ +=+ > >"It is good to be merry and wise, >It is good to be honest and true, >It is best to be off with the old love >Before you are on with the new." > [Song - anon.] Richard Hills: Under the old love of the 1997 Lawbook it was an infraction to ask about a single call. Under the new love of the 2007 Law 20F3 it is a player's right to ask about a single call (at her own risk of thereby placing Law 16B1 restrictions upon her partner). So it seems that Alain's Regulating Authority created its alert regulation under the ancien regime of the 1997 Lawbook, then the RA indolently failed to revise its alert regulation once the 2007 Lawbook took effect. Therefore, does this contradiction between Law and regulation justify Alain ignoring _any_ Law and _any_ regulation which Alain personally deems bad? No. Rather, a sensible Brussels Director has powers under the sensible Lawbook to sensibly resolve this silly paradox which has been carelessly created by a silly Belgian Regulating Authority. Law 80B2(f): "The Tournament Organizer's powers and duties include: to announce regulations supplementary to, but not in conflict with, these Laws." Law 81A: "The Director is the official representative of the Tournament Organizer." Law 81B2: "The Director applies, and is bound by, these Laws and supplementary regulations announced under authority given in these Laws." Law 81C, clauses 2 and 7: "The Director (not the players) has the responsibility for rectifying irregularities and redressing damage. The Director's duties and powers normally include also the following: 2. to administer and interpret these Laws and to advise the players of their rights and responsibilities thereunder. 7. to refer any matter to an appropriate committee." Richard Hills: Note the key word "supplementary" in Laws 80B2(f) and 81B2, which makes it abundantly clear that a regulation is subordinate to a Law if a paradox needs to be paradoctored. Put it all together, and the sensible Brussels Director: (x) announces Law 20F3 to her players, (y) advises that the contradictory alert regulation clause is invalid, but that the rest of the bad-but-to-be-obeyed alert regulation is still applicable, and (z) refers the alert regulation to the relevant sub-committee of the Regulating Authority, with a recommendation that the alert regulation have Sense and Sensibility (pointing to the ABF alert regulation as a good-and-joyfully-obeyed model). Best wishes Richard Hills Recruitment Section Specialist Recruitment Team Level 5 Aqua, workstation W569, 6223 8453 DIAC Social Club movie tickets -------------------------------------------------------------------- Important Notice: If you have received this email by mistake, please advise the sender and delete the message and attachments immediately. This email, including attachments, may contain confidential, sensitive, legally privileged and/or copyright information. Any review, retransmission, dissemination or other use of this information by persons or entities other than the intended recipient is prohibited. DIAC respects your privacy and has obligations under the Privacy Act 1988. The official departmental privacy policy can be viewed on the department's website at www.immi.gov.au. See: http://www.immi.gov.au/functional/privacy.htm --------------------------------------------------------------------- From blml at arcor.de Tue Jan 4 05:28:33 2011 From: blml at arcor.de (Thomas Dehn) Date: Tue, 4 Jan 2011 05:28:33 +0100 (CET) Subject: [BLML] interesting dWS case [SEC=UNOFFICIAL] In-Reply-To: <000901cbab63$d866f850$8934e8f0$@no> References: <000901cbab63$d866f850$8934e8f0$@no> <184B4C9981744CF4BC3A20BF59F0D334@Mildred> <4D1C4068.9070905@skynet.be> <1957070655.97086.1293699676312.JavaMail.ngmail@webmail18.arcor-online.net> <98830332.101890.1293710782116.JavaMail.ngmail@webmail13.arcor-online.net> <4D1C9FB6.9000808@gmail.com> <4D21ECE6.1030708@ulb.ac.be> Message-ID: <1668949171.272724.1294115313586.JavaMail.ngmail@webmail10.arcor-online.net> Sven Pran wrote: > On Behalf Of Alain Gottcheiner > ............... > > AG : you're playing soccer. A player gets seriously wounded, and the referee > > doesn't realize this.. > > > > What would you do ? Me, I would find some way to interrupt the game, perhaps > > taking the ball into my hands. And, with all due respect to Levi-Strauss and > > Huizinga, those who would say to themselves "I can't pick the ball, it's against the > > rules" are wrong. > > I have noticed a major difference between soccer and (duplicate) bridge: > > In soccer the players are supposed to violate the rules if their team can > benefit. For instance if the only way to prevent opponents score a goal is > to tackle in such a way that it will result in a free kick, or maybe even a > penalty kick, you tackle. If you don't then you are considered too chicken > for this game. > > And as for the player who is down without the referee noticing I believe > there is a well known universal practice for that situation: The ball is > kicked out over the sideline; and when eventually the game is continued the > throw in is made towards an opposing team player so that the team that > interrupted the game doesn't lose. > > But bridge is a game where you do not violate the rules even if you are > willing to "pay the penalty"; that makes a comparison between soccer and > bridge completely irrelevant. That is a fiction. It is not a reality. I have observed many incidents where bridge players quite deliberately broke the law and regulations. From little things such as not having a correctly filled in convention card to not so little things like deliberately using UI or claiming when they notice that they have only 12 cards to actual cheating such as "reverse hesitations". A rulebook does not create a culture of not violating the rules. Pretty much the only game or sport that actually has such a culture is golf. With everything else, quite a few players will deliberately break the rules once in a while. Thomas From richard.hills at immi.gov.au Tue Jan 4 06:11:36 2011 From: richard.hills at immi.gov.au (richard.hills at immi.gov.au) Date: Tue, 4 Jan 2011 16:11:36 +1100 Subject: [BLML] The Monty Hall trap [SEC=UNOFFICIAL] In-Reply-To: <1774182614.49514.1292477043379.JavaMail.ngmail@webmail15.arcor-online.net> Message-ID: ACBL syllogism: >>1. we may not vary our methods after an insufficient bid >>2. we may not vary our methods after an insufficient bid >>3. ergo we may not vary our methods after an insufficient bid Thomas Dehn: >I believe such a regulation to conflict with TFLB. Law 40B3, obviously in conflict with The Fabulous Law Book :-) "The Regulating Authority may disallow prior agreement by a partnership to vary its understandings during the auction or play following a question asked, a response to a question, or any irregularity." Thomas Dehn: >L27A1 gives LHO the choice between accepting and not accepting >an IB. Such regulations take away the right to accept the IB >through the back door. > >Example: > >Assume I play that over > >1D pass 4NT(RKCB) 5H > >double shows an even number of aces, and pass shows an odd >number of aces. > >1S pass 4NT(RKCB) 4H(IB, accepted). > >Is opener now supposed to double with an even number of aces? Richard Hills: For events in Oz, the ABF merely prohibits the offending side from varying their methods after an irregularity. The non- offending side is permitted by the ABF to arrange prior creation of as many new methods as it likes. (As a boring semi-expert I have not bothered. I believe that imps gained by superior varied methods are comprehensively outweighed by imps lost due to partner or myself forgetting those superior -- but highly infrequent -- varied methods.) Best wishes Richard Hills Recruitment Section Specialist Recruitment Team Level 5 Aqua, workstation W569, 6223 8453 DIAC Social Club movie tickets -------------------------------------------------------------------- Important Notice: If you have received this email by mistake, please advise the sender and delete the message and attachments immediately. This email, including attachments, may contain confidential, sensitive, legally privileged and/or copyright information. Any review, retransmission, dissemination or other use of this information by persons or entities other than the intended recipient is prohibited. DIAC respects your privacy and has obligations under the Privacy Act 1988. The official departmental privacy policy can be viewed on the department's website at www.immi.gov.au. See: http://www.immi.gov.au/functional/privacy.htm --------------------------------------------------------------------- From richard.hills at immi.gov.au Tue Jan 4 06:25:04 2011 From: richard.hills at immi.gov.au (richard.hills at immi.gov.au) Date: Tue, 4 Jan 2011 16:25:04 +1100 Subject: [BLML] The Monty Hall trap [SEC=UNOFFICIAL] In-Reply-To: Message-ID: >double shows an even number of aces, and pass shows an odd >number of aces. > >1S pass 4NT(RKCB) 4H(IB, accepted). > >Is opener now supposed to double with an even number of aces? The advantage of an invariant method of doubling the IB of 4H with an even number of aces is that the final contract can be 4S instead of 5S when two aces are missing, a safety play in the auction. :-) :-) Best wishes Richard Hills Recruitment Section Specialist Recruitment Team Level 5 Aqua, workstation W569, 6223 8453 DIAC Social Club movie tickets -------------------------------------------------------------------- Important Notice: If you have received this email by mistake, please advise the sender and delete the message and attachments immediately. This email, including attachments, may contain confidential, sensitive, legally privileged and/or copyright information. Any review, retransmission, dissemination or other use of this information by persons or entities other than the intended recipient is prohibited. DIAC respects your privacy and has obligations under the Privacy Act 1988. The official departmental privacy policy can be viewed on the department's website at www.immi.gov.au. See: http://www.immi.gov.au/functional/privacy.htm --------------------------------------------------------------------- From blml at arcor.de Tue Jan 4 06:33:48 2011 From: blml at arcor.de (Thomas Dehn) Date: Tue, 4 Jan 2011 06:33:48 +0100 (CET) Subject: [BLML] The Monty Hall trap [SEC=UNOFFICIAL] In-Reply-To: References: Message-ID: <969343849.272976.1294119228597.JavaMail.ngmail@webmail10.arcor-online.net> richard.hills at immi.gov.au wrote: > ACBL syllogism: > > >>1. we may not vary our methods after an insufficient bid > >>2. we may not vary our methods after an insufficient bid > >>3. ergo we may not vary our methods after an insufficient bid > > Thomas Dehn: > > >I believe such a regulation to conflict with TFLB. > > Law 40B3, obviously in conflict with The Fabulous Law Book :-) > > "The Regulating Authority may disallow prior agreement by a > partnership to vary its understandings during the auction or > play following a question asked, a response to a question, or > any irregularity." I don't think that applies here. Haying a different agreement after an IB is not a variation of understandings following an irregularity, it is a different auction. Thomas From Hermandw at skynet.be Tue Jan 4 09:08:20 2011 From: Hermandw at skynet.be (Herman De Wael) Date: Tue, 04 Jan 2011 09:08:20 +0100 Subject: [BLML] uninteresting dWikiS case [SEC=UNOFFICIAL] In-Reply-To: References: Message-ID: <4D22D574.9070208@skynet.be> richard.hills at immi.gov.au wrote: > Richard Hills: > > What is (or should be) the only correct philosophy on how to > deal with a defect in a Duplicate Bridge Law (or regulation) -> > > 3. This Law is a bad Law, I will follow it literally unless > and until it can be amended, and I will actively strive for > its amendment through appropriate channels. > Isn't that what I'm doing? There is only one problem here - the "literally". The law (and the added regulation) does not literally say that I should give Alain a PP, not that I should throw him out of the BBF. -- Herman De Wael Wilrijk Antwerpen Belgium From Hermandw at skynet.be Tue Jan 4 09:10:34 2011 From: Hermandw at skynet.be (Herman De Wael) Date: Tue, 04 Jan 2011 09:10:34 +0100 Subject: [BLML] Still out of order In-Reply-To: <20110103235109.EE29BA8C837@mailhub.irvine.com> References: <20110103235109.EE29BA8C837@mailhub.irvine.com> Message-ID: <4D22D5FA.5080204@skynet.be> Adam Beneschan wrote: > > Grattan wrote: > >> Christmas wishes to all. >> ++++++++++++++++++++++++++++++++++++ >> +=+ I have not read this topic in detail. I can offer >> two pieces of information. My files show >> (i) that the drafting committee intended >> no change in the effect of Law 57 between 1997 >> and 2007, merely tidying up the consistency. >> (ii) there was discussion whether to include >> a reference to Law 23. Those who said 'no' won >> the day. They made the case that Law 23 applies >> generally, in respect of any irregularity, and does >> not need a special mention. >> ~ Grattan ~ +=+ >> ........................................................... > > Thanks, Grattan, that was the sort of information I was looking for. > > I still believe that when the drafting committee changed the language, > they accidentally confused the issue. To me, the clear and simple > reading of the new wording is that, when a singleton is involved, Law > 57C1 is supposed to apply only when there has been instruction from > the declarer. To come to the other (correct) conclusion, that it also > applies if dummy plays the singleton without declarer's instruction, > requires some mental gymnastics, logical analysis, and maybe research > into other Laws, especially since that conclusion *seems* to > contradict the new language in the second sentence of 57C1. The right > answer is not at all obvious, no matter what anyone says. Some of the > posters here have made some decent arguments about why 57C1 should > still apply in that case, but a director shouldn't have to figure all > that out. I think the committee erred. Not that I think there's any > urgency that this be fixed, since this must be a very rare situation > if it occurs at all. > Of course not, it happens all the time! I will always, as Declarer, instruct my dummy never to touch the cards until my LHO has played. And I will always, fourth in hand, play when dummy touches the cards. This does happen, all the time. > -- Adam > -- Herman De Wael Wilrijk Antwerpen Belgium From blml at arcor.de Tue Jan 4 09:20:33 2011 From: blml at arcor.de (Thomas Dehn) Date: Tue, 4 Jan 2011 09:20:33 +0100 (CET) Subject: [BLML] uninteresting dWikiS case [SEC=UNOFFICIAL] In-Reply-To: <4D22D574.9070208@skynet.be> References: <4D22D574.9070208@skynet.be> Message-ID: <1915818315.281546.1294129233085.JavaMail.ngmail@webmail07.arcor-online.net> Herman De Wael wrote: > richard.hills at immi.gov.au wrote: > > Richard Hills: > > > > What is (or should be) the only correct philosophy on how to > > deal with a defect in a Duplicate Bridge Law (or regulation) -> > > > > 3. This Law is a bad Law, I will follow it literally unless > > and until it can be amended, and I will actively strive for > > its amendment through appropriate channels. > > > > Isn't that what I'm doing? > There is only one problem here - the "literally". The law (and the added > regulation) does not literally say that I should give Alain a PP, not > that I should throw him out of the BBF. It isn't possible to "literally" apply TFLB. TFLB is too self contradictory and incomplete for that. Sure, you can "literally" apply some of the laws, such as computation of scores, or leads out of turn, or revokes. But most everything else requires the TD to use his brain rather than merely apply a fixed ruling from some law. Was that a BIT? What did it "demonstrably" suggest? What are the logical alternatives? What was "likely" had the irregularity not occurred? Some of those rulings are obvious, but many are not. Moreover, there are at least the following: A) What would Kaplan have done? B) The intent of the lawmakers C) The literal text of the original english version of the law D) The literal text of the translation E) Any additional instructions, regulations, and communications. F) The "spirit" of TFLB, such as trying to create equity and rectification of damage, rather than severe penalties. The average player and TD can, with the best intentions, only try to implement a combination of D) and F). Thomas From blml at arcor.de Tue Jan 4 09:24:12 2011 From: blml at arcor.de (Thomas Dehn) Date: Tue, 4 Jan 2011 09:24:12 +0100 (CET) Subject: [BLML] uninteresting dWikiS case [SEC=UNOFFICIAL] In-Reply-To: <4D22D574.9070208@skynet.be> References: <4D22D574.9070208@skynet.be> Message-ID: <893414029.281697.1294129452617.JavaMail.ngmail@webmail07.arcor-online.net> Herman De Wael wrote: > richard.hills at immi.gov.au wrote: > > Richard Hills: > > > > What is (or should be) the only correct philosophy on how to > > deal with a defect in a Duplicate Bridge Law (or regulation) -> > > > > 3. This Law is a bad Law, I will follow it literally unless > > and until it can be amended, and I will actively strive for > > its amendment through appropriate channels. > > > > Isn't that what I'm doing? > There is only one problem here - the "literally". The law (and the added > regulation) does not literally say that I should give Alain a PP, not > that I should throw him out of the BBF. It isn't possible to "literally" apply TFLB. TFLB is too self contradictory, ambiguously worded, and incomplete for that. Sure, you can "literally" apply some of the laws, such as those addressing computation of scores, or leads out of turn, or revokes. But most everything else requires the TD to use his brain rather than merely apply a fixed ruling from some law. Was that a BIT? What did it "demonstrably" suggest? What are the logical alternatives? What was "likely" had the irregularity not occurred? Some of those rulings are obvious, but many are not. Moreover, there are at least the following: A) What would Kaplan have done? B) The intent of the lawmakers C) The literal text of the original english version of the law D) The literal text of the translation E) Any additional instructions, regulations, and communications. F) The "spirit" of TFLB, such as trying to create equity and rectification of damage, rather than severe penalties. The average player and TD can, with the best intentions, only try to implement a combination of D) and F). Thomas From svenpran at online.no Tue Jan 4 10:28:47 2011 From: svenpran at online.no (Sven Pran) Date: Tue, 4 Jan 2011 10:28:47 +0100 Subject: [BLML] interesting dWS case [SEC=UNOFFICIAL] In-Reply-To: <1668949171.272724.1294115313586.JavaMail.ngmail@webmail10.arcor-online.net> References: <000901cbab63$d866f850$8934e8f0$@no> <184B4C9981744CF4BC3A20BF59F0D334@Mildred> <4D1C4068.9070905@skynet.be> <1957070655.97086.1293699676312.JavaMail.ngmail@webmail18.arcor-online.net> <98830332.101890.1293710782116.JavaMail.ngmail@webmail13.arcor-online.net> <4D1C9FB6.9000808@gmail.com> <4D21ECE6.1030708@ulb.ac.be> <1668949171.272724.1294115313586.JavaMail.ngmail@webmail10.arcor-online.net> Message-ID: <000701cbabf1$cb590cf0$620b26d0$@no> On Behalf Of Thomas Dehn > Sven Pran wrote: > .................. > > But bridge is a game where you do not violate the rules even if you > > are willing to "pay the penalty"; that makes a comparison between > > soccer and bridge completely irrelevant. > > That is a fiction. It is not a reality. I have observed many incidents where bridge > players quite deliberately broke the law and regulations. From little things such as > not having a correctly filled in convention card to not so little things like > deliberately using UI or claiming when they notice that they have only 12 cards to > actual cheating such as "reverse hesitations". I (like many if not all competent directors) assume that a player obeys Law 72B1. This is a matter of confidence, and whenever it becomes clear that a player has violated this law then the reaction will be so much more severe. > > A rulebook does not create a culture of not violating the rules. > Pretty much the only game or sport that actually has such a culture is golf. With > everything else, quite a few players will deliberately break the rules once in a > while. Have you ever watched Snooker at high level? I have seen players calling attention to their own irregularity when nobody else, and in particular not the referee could notice it. From agot at ulb.ac.be Tue Jan 4 13:54:49 2011 From: agot at ulb.ac.be (Alain Gottcheiner) Date: Tue, 04 Jan 2011 13:54:49 +0100 Subject: [BLML] uninteresting dWikiS case [SEC=UNOFFICIAL] In-Reply-To: References: Message-ID: <4D231899.20407@ulb.ac.be> Le 3/01/2011 23:26, richard.hills at immi.gov.au a ?crit : > Hirsch Davis: > >>> ..... >>> NO! The PP is because the player knew what he should be >>> doing and did something else. >>> ..... >>> please bear in mind that any comments that you make >>> suggesting that the Laws not be followed as written apply to >>> your game only and do not apply to any games where the 2007 >>> Laws of Duplicate Bridge is the governing document. >>> >>> Hirsch > Alain Gottcheiner: > >> ..... >> Or, other real-life example, partner overcalls 2H over 1NT, >> meaning hearts and a minor, alerted and explained. The LOL on >> your right acts accordingly. >> On the next deal, partner, a passed hand, overcalls 2H over >> 1NT, meaning in this case 5S+4H. The LOL nods. You tell her >> "you'd better ask". >> You've broken a rule. >> ..... > Richard Hills: > > No, Alain broke two rules. And his second infraction of the > local Alert regulation would have been unnecessary if Alain > had initially explained in accordance with > > Law 40B6(a), first phrase: > > "When explaining the significance of partner's call or play > in reply to opponent's enquiry (see Law 20) a player shall > disclose all special information conveyed to him through > partnership agreement or partnership experience ..... " > > Richard Hills: > > Thus Alain's explanation of his partner's first 2H overcall > over 1NT should have disclosed _all_ the special information > conveyed to Alain by partnership agreement: > > "Because partner is not a passed hand, 2H promises hearts and > a minor. But whenever partner is a passed hand, an overcall > of 2H over 1NT would now promise five spades and four hearts." Sorry, but that doesn't hold water. To explain what the bid would have meant in other contexts has never been par of one's alert-explain duty. From agot at ulb.ac.be Tue Jan 4 13:58:17 2011 From: agot at ulb.ac.be (Alain Gottcheiner) Date: Tue, 04 Jan 2011 13:58:17 +0100 Subject: [BLML] uninteresting dWikiS case [SEC=UNOFFICIAL] In-Reply-To: References: Message-ID: <4D231969.2080609@ulb.ac.be> Le 4/01/2011 1:19, richard.hills at immi.gov.au a ?crit : > Law 72B1: > > "A player MUST NOT infringe a law intentionally, even if there > is a prescribed rectification he is willing to accept." > > Law 81C: > > "The Director's duties and powers normally include also the > following: > [...] > 5. to waive rectification for cause, in his discretion, upon > the request of the non-offending side." > > Richard Hills: > > But to "waive rectification" is NOT synonymous with "cancel > infraction". A Law 72B1 intentional infraction remains a Law > 72B1 intentional infraction whether or not Law 81C happens to > be subsequently invoked. > > Thus an ethical player who is playing Duplicate Bridge, not > playing social bridge, is obliged to obey Law 72B1. > > Alain Gottcheiner: > >> AG : I'd like to add something here. >> In Brussels, there are quite a few dWSists and "extra- >> alerters" of doubles etc. >> Some are considered the most ethical players around (names >> given on request). > Richard Hills: > > No, those players are unintentionally unethical. AG : reread, please. They are *considered* the most ethical players around, which means in this case that opponents appreciate their efforts to avoid damage to their opponents. > > Richard Hills: > > It is not "main school" to obey the Lawbook and regulations, > it is the "ONLY school". And if a Secretary Bird does so, > that is not an "unethical shenanigan" (as stated by The > Unwritten Laws of Duplicate Bridge) but rather "complying > with the lawful procedures and ethical standards set out in > these laws" (as stated by Law 72A). We'd better go on with this in private, if you wish. I can show you a case where the creation of UI was intended, without any doubt. From agot at ulb.ac.be Tue Jan 4 14:00:39 2011 From: agot at ulb.ac.be (Alain Gottcheiner) Date: Tue, 04 Jan 2011 14:00:39 +0100 Subject: [BLML] uninteresting dWikiS case [SEC=UNOFFICIAL] In-Reply-To: References: Message-ID: <4D2319F7.2090908@ulb.ac.be> Le > Richard Hills: > > Under the old love of the 1997 Lawbook it was an infraction to > ask about a single call. > > Under the new love of the 2007 Law 20F3 it is a player's right > to ask about a single call (at her own risk of thereby placing > Law 16B1 restrictions upon her partner). > > So it seems that Alain's Regulating Authority created its > alert regulation under the ancien regime of the 1997 Lawbook, > then the RA indolently failed to revise its alert regulation > once the 2007 Lawbook took effect. The problem remains the same : the fact that you know everything about LHO's bid gives your partnership a disadvantage. In this case, it will put UI pressure on your partner. That's bad. From ehaa at starpower.net Tue Jan 4 15:52:50 2011 From: ehaa at starpower.net (Eric Landau) Date: Tue, 4 Jan 2011 09:52:50 -0500 Subject: [BLML] uninteresting dWikiS case In-Reply-To: References: Message-ID: <22EBC8A8-58AB-48E6-9BB7-25CB97D4A5B9@starpower.net> On Jan 3, 2011, at 7:19 PM, richard.hills at immi.gov.au wrote: > Law 72B1: > > "A player MUST NOT infringe a law intentionally, even if there > is a prescribed rectification he is willing to accept." > > Law 81C: > > "The Director's duties and powers normally include also the > following: > [...] > 5. to waive rectification for cause, in his discretion, upon > the request of the non-offending side." > > Richard Hills: > > But to "waive rectification" is NOT synonymous with "cancel > infraction". A Law 72B1 intentional infraction remains a Law > 72B1 intentional infraction whether or not Law 81C happens to > be subsequently invoked. > > Thus an ethical player who is playing Duplicate Bridge, not > playing social bridge, is obliged to obey Law 72B1. > > Alain Gottcheiner: > >> AG : I'd like to add something here. >> In Brussels, there are quite a few dWSists and "extra- >> alerters" of doubles etc. >> Some are considered the most ethical players around (names >> given on request). > > Richard Hills: > > No, those players are unintentionally unethical. They fondly > but incorrectly believe that obeying The Unwritten Laws of > Duplicate Bridge is ethical. However, it is the actual Laws > of Duplicate Bridge which necessarily define bridge ethics. > > Alain Gottcheiner: > >> And some who, for other reasons, aren't (no names, of >> course), keep complying with "main school" requirements. >> Knowing them, I'm wondering whether it could be because it >> helps them in their shenanigans. > > Richard Hills: > > It is not "main school" to obey the Lawbook and regulations, > it is the "ONLY school". And if a Secretary Bird does so, > that is not an "unethical shenanigan" (as stated by The > Unwritten Laws of Duplicate Bridge) but rather "complying > with the lawful procedures and ethical standards set out in > these laws" (as stated by Law 72A). The ACBL takes a very different attitude. They officially encourage players to go beyond what TFLB and the alert regulations specifically require in the area of disclosure, with the objective of making sure that they are as forthcoming and helpful as possible in making sure that their opponents understand their methods. The ACBL calls this "active ethics", and would agree with Alain in describing those who follow its precepts as "the most ethical players around". Eric Landau 1107 Dale Drive Silver Spring MD 20910 ehaa at starpower.net From ehaa at starpower.net Tue Jan 4 16:31:46 2011 From: ehaa at starpower.net (Eric Landau) Date: Tue, 4 Jan 2011 10:31:46 -0500 Subject: [BLML] The Monty Hall trap In-Reply-To: References: Message-ID: <95857DFE-B801-4A9C-B54A-84A71A5EE674@starpower.net> On Jan 4, 2011, at 12:11 AM, richard.hills at immi.gov.au wrote: > ACBL syllogism: > >>> 1. we may not vary our methods after an insufficient bid >>> 2. we may not vary our methods after an insufficient bid >>> 3. ergo we may not vary our methods after an insufficient bid > > Thomas Dehn: > >> I believe such a regulation to conflict with TFLB. > > Law 40B3, obviously in conflict with The Fabulous Law Book :-) > > "The Regulating Authority may disallow prior agreement by a > partnership to vary its understandings during the auction or > play following a question asked, a response to a question, or > any irregularity." > > Thomas Dehn: > >> L27A1 gives LHO the choice between accepting and not accepting >> an IB. Such regulations take away the right to accept the IB >> through the back door. >> >> Example: >> >> Assume I play that over >> >> 1D pass 4NT(RKCB) 5H >> >> double shows an even number of aces, and pass shows an odd >> number of aces. >> >> 1S pass 4NT(RKCB) 4H(IB, accepted). >> >> Is opener now supposed to double with an even number of aces? > > Richard Hills: > > For events in Oz, the ABF merely prohibits the offending side > from varying their methods after an irregularity. The non- > offending side is permitted by the ABF to arrange prior > creation of as many new methods as it likes. > > (As a boring semi-expert I have not bothered. I believe that > imps gained by superior varied methods are comprehensively > outweighed by imps lost due to partner or myself forgetting > those superior -- but highly infrequent -- varied methods.) It may well not be worthwhile for Richard and his partner to construct legal-in-Australia "superior varied methods" for use specifically over insufficient bids, but they would nevertheless probably benefit from a discussion of how their existing "meta- agreements" would actually apply in such situations. Problems arise when regulators fail to distinguish between making explicit agreements as to how one's agreed methods apply in previously unencountered situations and "varying" those agreements for those situations. Adopting method X in place of method Y is "varying [your] understandings"; adopting method X in place of having no prior understanding is not. How could you have the legal auction 1S-1H-1S if responder's 1S call must have the same meaning as would a 1S response to an opening 1S bid absent the opponent's irregularity? Eric Landau 1107 Dale Drive Silver Spring MD 20910 ehaa at starpower.net From richard.hills at immi.gov.au Tue Jan 4 22:47:14 2011 From: richard.hills at immi.gov.au (richard.hills at immi.gov.au) Date: Wed, 5 Jan 2011 08:47:14 +1100 Subject: [BLML] The Monty Hall trap [SEC=UNOFFICIAL] In-Reply-To: <969343849.272976.1294119228597.JavaMail.ngmail@webmail10.arcor-online.net> Message-ID: Law 40B3: "The Regulating Authority may disallow prior agreement by a partnership to vary its understandings during the auction or play following a question asked, a response to a question, or any irregularity." Thomas Dehn: >I don't think that applies here. > >Having a different agreement after an IB is not a variation >of understandings following an irregularity, it is a >different auction. Eric Landau: [snip] >How could you have the legal auction 1S-1H-1S if responder's >1S call must have the same meaning as would a 1S response to >an opening 1S bid absent the opponent's irregularity? The Fabulous Law Book's advice on the meaning of "meaning". Law 17 footnote: "* For example, a substituted call differs if its meaning is much different or if it is psychic." Law 27 footnote: "* the meaning of (information available from) a call is the knowledge of what it shows and what it excludes." Law 40B1(a): "In its discretion the Regulating Authority may designate certain partnership understandings as 'special partnership understandings'. A special partnership understanding is one whose meaning, in the opinion of the Regulating Authority, may not be readily understood and anticipated by a significant number of players in the tournament." Richard Hills: So if I was an ACBL Director, I would tell the Dehn-Landau partnership that Law 40B3 does indeed apply after an accepted insufficient bid (advising them that their sea- lawyering, if they wish to appeal, cannot be upheld by an ordinary ACBL Appeals Committee, due to Law 93B3, although of course they still have an undisputed right to appeal to an ordinary ACBL Appeals Committee), and for this auction WEST NORTH EAST 1S 1H 1S the meaning of East's 1S must be not be an ACBL Special Partnership Understanding, that is not much different nor psychic when compared to the meaning of East's 2S in this auction WEST NORTH EAST 1S Pass 2S What's the problem? Best wishes Richard Hills not an ACBL Director, but a (retired) ABF Club Director, the lowest possible rank of ABF Directors, hence my worthless postings should be ignored by all sea-lawyers -------------------------------------------------------------------- Important Notice: If you have received this email by mistake, please advise the sender and delete the message and attachments immediately. This email, including attachments, may contain confidential, sensitive, legally privileged and/or copyright information. Any review, retransmission, dissemination or other use of this information by persons or entities other than the intended recipient is prohibited. DIAC respects your privacy and has obligations under the Privacy Act 1988. The official departmental privacy policy can be viewed on the department's website at www.immi.gov.au. See: http://www.immi.gov.au/functional/privacy.htm --------------------------------------------------------------------- From richard.hills at immi.gov.au Tue Jan 4 23:16:27 2011 From: richard.hills at immi.gov.au (richard.hills at immi.gov.au) Date: Wed, 5 Jan 2011 09:16:27 +1100 Subject: [BLML] ABF National Authority 4th March 2010 [SEC=UNOFFICIAL] Message-ID: http://www.abf.com.au/about/minutes/namin1004.pdf [snip] For the purposes of this discussion the following facts are assumed: a. On a hand played in Round 8 of the Summer Festival, with the North/South pair as defenders and East/West as declarer, the result recorded using the electronic scorer was 2H making 7 tricks (E/W -100 ). The "accept" button was punched. This result was not challenged while the E/W pair remained at the table. b. Subsequently declarer claimed that the correct result was 2H making 9 tricks (E/W +140). The matter was brought to the attention of a director and who discussed the matter with a player from both teams. Neither player could state conclusively how the hand was played. The player who claimed that 2H had gone down agreed that if the line suggested had been used, the contract appeared unbeatable. c. The director involved in this discussion decided the match should be scored on the basis that the declarer made 9 tricks in 2H. d. The result was that the team disputing the score lost the match by a bigger margin. e. The team disputing the score subsequently contacted the acting CTD who promised to look into the matter. f. At no stage did the acting CTD tell the team disputing the score that the decision was final. g. When the team that was disadvantaged by the decision attempted to appeal at the end of Round 14, the Tournament Committee decided that the appeal was out of time. [snip] How should (and did) the ABF National Authority rule? Best wishes Richard Hills Recruitment Section Specialist Recruitment Team Level 5 Aqua, workstation W569, 6223 8453 DIAC Social Club movie tickets -------------------------------------------------------------------- Important Notice: If you have received this email by mistake, please advise the sender and delete the message and attachments immediately. This email, including attachments, may contain confidential, sensitive, legally privileged and/or copyright information. Any review, retransmission, dissemination or other use of this information by persons or entities other than the intended recipient is prohibited. DIAC respects your privacy and has obligations under the Privacy Act 1988. The official departmental privacy policy can be viewed on the department's website at www.immi.gov.au. See: http://www.immi.gov.au/functional/privacy.htm --------------------------------------------------------------------- From bpark56 at comcast.net Tue Jan 4 23:54:30 2011 From: bpark56 at comcast.net (Robert Park) Date: Tue, 04 Jan 2011 17:54:30 -0500 Subject: [BLML] The Monty Hall trap [SEC=UNOFFICIAL] In-Reply-To: References: Message-ID: <4D23A526.1040509@comcast.net> On 1/4/11 4:47 PM, richard.hills at immi.gov.au wrote: > Law 40B3: > > "The Regulating Authority may disallow prior agreement by a > partnership to vary its understandings during the auction or > play following a question asked, a response to a question, or > any irregularity." > > Thomas Dehn: > >> I don't think that applies here. >> >> Having a different agreement after an IB is not a variation >> of understandings following an irregularity, it is a >> different auction. > Eric Landau: > > [snip] > >> How could you have the legal auction 1S-1H-1S if responder's >> 1S call must have the same meaning as would a 1S response to >> an opening 1S bid absent the opponent's irregularity? > The Fabulous Law Book's advice on the meaning of "meaning". > > Law 17 footnote: > > "* For example, a substituted call differs if its meaning is > much different or if it is psychic." > > Law 27 footnote: > > "* the meaning of (information available from) a call is the > knowledge of what it shows and what it excludes." > > Law 40B1(a): > > "In its discretion the Regulating Authority may designate > certain partnership understandings as 'special partnership > understandings'. A special partnership understanding is one > whose meaning, in the opinion of the Regulating Authority, > may not be readily understood and anticipated by a > significant number of players in the tournament." > > Richard Hills: > > So if I was an ACBL Director, I would tell the Dehn-Landau > partnership that Law 40B3 does indeed apply after an > accepted insufficient bid (advising them that their sea- > lawyering, if they wish to appeal, cannot be upheld by an > ordinary ACBL Appeals Committee, due to Law 93B3, although > of course they still have an undisputed right to appeal to > an ordinary ACBL Appeals Committee), and for this auction > > WEST NORTH EAST > 1S 1H 1S > > the meaning of East's 1S must be not be an ACBL Special > Partnership Understanding, that is not much different nor > psychic when compared to the meaning of East's 2S in this > auction > > WEST NORTH EAST > 1S Pass 2S > > What's the problem? You seem to be saying that the partnership is required to have 1S and 2S bid mean exactly the same thing (or nearly so, whatever 'nearly' means). I fail to see why. We have two different auctions here. Why can't 1S-(1H)-2S show better values or better shape than 1S-(1H)-1S...or the other way around if you would prefer a jump to 2S to be preemptive? I think one way or the other is how most players would interpret the situation. Why can't they have an agreement to be on the same page? If you believe your statement to be true, please define what "not much different" means, so all of us (directors included) and be consistent in our behavior and rulings. --Bob Park > Best wishes > > Richard Hills > not an ACBL Director, but a (retired) ABF Club Director, > the lowest possible rank of ABF Directors, hence my > worthless postings should be ignored by all sea-lawyers > > > > -------------------------------------------------------------------- > Important Notice: If you have received this email by mistake, please advise > the sender and delete the message and attachments immediately. This email, > including attachments, may contain confidential, sensitive, legally privileged > and/or copyright information. Any review, retransmission, dissemination > or other use of this information by persons or entities other than the > intended recipient is prohibited. DIAC respects your privacy and has > obligations under the Privacy Act 1988. The official departmental privacy > policy can be viewed on the department's website at www.immi.gov.au. See: > http://www.immi.gov.au/functional/privacy.htm > > > --------------------------------------------------------------------- > > _______________________________________________ > Blml mailing list > Blml at rtflb.org > http://lists.rtflb.org/mailman/listinfo/blml > From nigelguthrie at yahoo.co.uk Wed Jan 5 00:12:31 2011 From: nigelguthrie at yahoo.co.uk (Nigel Guthrie) Date: Tue, 4 Jan 2011 23:12:31 +0000 (GMT) Subject: [BLML] The Monty Hall trap In-Reply-To: <95857DFE-B801-4A9C-B54A-84A71A5EE674@starpower.net> References: <95857DFE-B801-4A9C-B54A-84A71A5EE674@starpower.net> Message-ID: <821566.28383.qm@web28515.mail.ukl.yahoo.com> [Eric Landau] How could you have the legal auction 1S-1H-1S if responder's 1S call must have the same meaning as would a 1S response to an opening 1S bid absent the opponent's irregularity? {Nige1] Yet another legal anomaly, for which director and partnerships will have to devise work-rounds until 2018. By 2018, I hope that law-makers and players accept what is the root of the problem: the anomalies are created is by giving players (offenders and victims) *options* after an illegal call (insufficient bid, call out of turn, or whatever). IMO, an illegal call should be cancelled with no other option for the offender. The illegal call should be UI to the offender's partner, who should be silenced for the rest of the auction. The LHO of the offender should not have the option to accept the illegal call. If he attempts to condone it, that should be treated as a subsequent infraction. These rules are simple, clear and fair. Justice is done and seen to be done. They also allow the deal to be bid and played in a far more normal and understandable way than current legislation permits. An obvious consequence is that it deters illegal bids. If such deterrence is considered to be against hallowed "Equity" Principles, so be it. From richard.hills at immi.gov.au Wed Jan 5 00:18:22 2011 From: richard.hills at immi.gov.au (richard.hills at immi.gov.au) Date: Wed, 5 Jan 2011 10:18:22 +1100 Subject: [BLML] uninteresting dWikiS case [SEC=UNOFFICIAL] In-Reply-To: <893414029.281697.1294129452617.JavaMail.ngmail@webmail07.arcor-online.net> Message-ID: Thomas Dehn: >..... >Moreover, there are at least the following: >A) What would Kaplan have done? Richard Hills: What Kaplan advocated in the 1960s was wildly different to what Kaplan advocated in the 1990s. Thomas Dehn: >B) The intent of the lawmakers Richard Hills: A standard principle of jurisprudence is that the intent of the lawmakers is relevant only to a carelessly written ambiguous Law (for example, the second sentence of the 2007 Law 55A). But if there is a carelessly written unambiguous Law (for example, the notorious 1997 Law 25B), then the actual text of the unambiguous Law prevails no matter what the original intent of the careless Drafting Committee was. Thomas Dehn: >C) The literal text of the original English version of the > law >D) The literal text of the translation Richard Hills: The WBF has specifically ruled that the English version of the Laws prevails over a translation if the two versions have an inconsistency. Thomas Dehn: >E) Any additional instructions, regulations, and > communications. >F) The "spirit" of TFLB, such as trying to create equity > and rectification of damage, rather than severe > penalties. >..... Grattan Endicott, 12th June 2005: >>..... >>I think also that the letter of the law should embody >>the spirit of the law. >> ~ Grattan ~ +=+ Richard Hills: I agree with Grattan, since otherwise players would be disciplined and/or ostracised for "infracting" the spirit of The Unwritten Laws of Duplicate Bridge. WBF Code of Practice, "Ethics", page 6: "A contestant may be penalized only for a lapse of ethics where a player is in breach of the provisions of the laws in respect of conduct. A player who has conformed to the laws and regulations is not subject to criticism. This does not preclude encouragement of a generous attitude to opponents, especially in the exchange of information behind screens." Alain Gottcheiner: >>>Sorry, but that doesn't hold water. To explain what >>>the bid would have meant in other contexts has never >>>been par of one's alert-explain duty. Richard Hills: Maybe not part, but definitely par. The Pocket Oxford Dictionary defines par as "equal footing", and it is important under Law 40B6(a) that the opposing declarer has an equal footing to my partnership in this and subsequent defences (given that the opposing declarer will naturally assume that the opening lead of the same honour has the same meaning). So as par for my generous attitude to opponents in the exchange of information, I am meticulous in informing an opposing declarer that our leads and signals are very substantially different between our current defence of declarer's notrump contract versus our possible future defence against a suit contract. Best wishes Richard Hills Recruitment Section Specialist Recruitment Team Level 5 Aqua, workstation W569, 6223 8453 DIAC Social Club movie tickets -------------------------------------------------------------------- Important Notice: If you have received this email by mistake, please advise the sender and delete the message and attachments immediately. This email, including attachments, may contain confidential, sensitive, legally privileged and/or copyright information. Any review, retransmission, dissemination or other use of this information by persons or entities other than the intended recipient is prohibited. DIAC respects your privacy and has obligations under the Privacy Act 1988. The official departmental privacy policy can be viewed on the department's website at www.immi.gov.au. See: http://www.immi.gov.au/functional/privacy.htm --------------------------------------------------------------------- From richard.hills at immi.gov.au Wed Jan 5 00:30:03 2011 From: richard.hills at immi.gov.au (richard.hills at immi.gov.au) Date: Wed, 5 Jan 2011 10:30:03 +1100 Subject: [BLML] The Monty Hall trap [SEC=UNOFFICIAL] In-Reply-To: <4D23A526.1040509@comcast.net> Message-ID: [snip] >If you believe your statement to be true, please define what >"not much different" means, so all of us (directors included) >can be consistent in our behavior and rulings. > > --Bob Park >>Best wishes >> >>Richard Hills >>not an ACBL Director, but a (retired) ABF Club Director, >>the lowest possible rank of ABF Directors, hence my >>worthless postings should be ignored by all sea-lawyers The footnote to Law 17D3 does not define "much different", so a Law 17D3 ruling will involve judgement by the Director in charge and hence involve inconsistency. In my opinion (but not the opinion of Nigel Guthrie) the idea of "consistency" is a will o' the wisp, not only impractical but also undesirable. Of course my opinion (and Nigel Guthrie's opinion) is totally and utterly irrelevant for ACBL Directors. The appropriate body to give guidance on "much different" in ACBL-land is the ACBL Laws Commission. Best wishes Richard Hills Recruitment Section Specialist Recruitment Team Level 5 Aqua, workstation W569, 6223 8453 DIAC Social Club movie tickets -------------------------------------------------------------------- Important Notice: If you have received this email by mistake, please advise the sender and delete the message and attachments immediately. This email, including attachments, may contain confidential, sensitive, legally privileged and/or copyright information. Any review, retransmission, dissemination or other use of this information by persons or entities other than the intended recipient is prohibited. DIAC respects your privacy and has obligations under the Privacy Act 1988. The official departmental privacy policy can be viewed on the department's website at www.immi.gov.au. See: http://www.immi.gov.au/functional/privacy.htm --------------------------------------------------------------------- From nigelguthrie at yahoo.co.uk Wed Jan 5 00:32:28 2011 From: nigelguthrie at yahoo.co.uk (Nigel Guthrie) Date: Tue, 4 Jan 2011 23:32:28 +0000 (GMT) Subject: [BLML] ABF National Authority 4th March 2010 [SEC=UNOFFICIAL] In-Reply-To: References: Message-ID: <764142.92719.qm@web28501.mail.ukl.yahoo.com> [Richard Hills] http://www.abf.com.au/about/minutes/namin1004.pdf For the purposes of this discussion the following facts are assumed: a. On a hand played in Round 8 of the Summer Festival, with the North/South pair as defenders and East/West as declarer, the result recorded using the electronic scorer was 2H making 7 tricks (E/W -100 ). The "accept" button was punched. This result was not challenged while the E/W pair remained at the table. b. Subsequently declarer claimed that the correct result was 2H making 9 tricks (E/W +140). The matter was brought to the attention of a director and who discussed the matter with a player from both teams. Neither player could state conclusively how the hand was played. The player who claimed that 2H had gone down agreed that if the line suggested had been used, the contract appeared unbeatable. c. The director involved in this discussion decided the match should be scored on the basis that the declarer made 9 tricks in 2H. d. The result was that the team disputing the score lost the match by a bigger margin. e. The team disputing the score subsequently contacted the acting CTD who promised to look into the matter. f. At no stage did the acting CTD tell the team disputing the score that the decision was final. g. When the team that was disadvantaged by the decision attempted to appeal at the end of Round 14, the Tournament Committee decided that the appeal was out of time. [snip] How should (and did) the ABF National Authority rule? [Nigel] In the UK, North is the player responsible for entering the score and East or West is meant to check it. Is that the Australian protocol? One side thought the result was 2H-1. The other 2H+1. But if none of the players could remember what happened, who came up with the 9 trick line? and why? Why did the team appeal if they would lose the match, no matter what the result of the appeal? I don't know how the ABFNA should rule -- except insofar that, given what appears to be procedural anomalies, the appeal was not "out of time". From richard.hills at immi.gov.au Wed Jan 5 01:07:27 2011 From: richard.hills at immi.gov.au (richard.hills at immi.gov.au) Date: Wed, 5 Jan 2011 11:07:27 +1100 Subject: [BLML] ABF National Authority 4th March 2010 [SEC=UNOFFICIAL] In-Reply-To: <764142.92719.qm@web28501.mail.ukl.yahoo.com> Message-ID: Nigel Guthrie: >Why did the team appeal if they would lose the match, no matter >what the result of the appeal? Richard Hills: While it is ACBL and EBU tradition for their major imp teams events to be run as a knockout throughout, in Australia only the final stage of our January National Open Teams is knockout. The initial stage is 200-odd teams playing a giant Swiss of 14 x 20-board matches, using the WBF victory point scale. So the appellants were therefore attempting to scrounge 1 vp. Best wishes Richard Hills Recruitment Section Specialist Recruitment Team Level 5 Aqua, workstation W569, 6223 8453 DIAC Social Club movie tickets -------------------------------------------------------------------- Important Notice: If you have received this email by mistake, please advise the sender and delete the message and attachments immediately. This email, including attachments, may contain confidential, sensitive, legally privileged and/or copyright information. Any review, retransmission, dissemination or other use of this information by persons or entities other than the intended recipient is prohibited. DIAC respects your privacy and has obligations under the Privacy Act 1988. The official departmental privacy policy can be viewed on the department's website at www.immi.gov.au. See: http://www.immi.gov.au/functional/privacy.htm --------------------------------------------------------------------- From richard.hills at immi.gov.au Wed Jan 5 02:45:02 2011 From: richard.hills at immi.gov.au (richard.hills at immi.gov.au) Date: Wed, 5 Jan 2011 12:45:02 +1100 Subject: [BLML] uninteresting dWikiS case [SEC=UNOFFICIAL] In-Reply-To: <22EBC8A8-58AB-48E6-9BB7-25CB97D4A5B9@starpower.net> Message-ID: Eric Landau: >The ACBL takes a very different attitude. They officially >encourage players to go beyond what TFLB and the alert >regulations specifically require in the area of disclosure, >with the objective of making sure that they are as >forthcoming and helpful as possible in making sure that >their opponents understand their methods. > >The ACBL calls this "active ethics", and would agree with >Alain in describing those who follow its precepts as "the >most ethical players around". Richard Hills: The ABF takes a very different attitude. The ABF officially encourages players to obey the ABF Alert Regulation, not flaunt their flouting of the ABF Alert Regulation and then misname such flouting as "active ethics". But ..... The ABF carefully designed its Alert Regulation to _require_ Aussie players to be "as forthcoming and helpful as possible in making sure that their opponents understand their methods". For example, the ABF Alert Regulation clause 13.5: "Your policy should be to alert any call by partner (other than a self-alerting call) that the opponents could reasonably misunderstand." Which differs from the dWikiS Alert Regulation clause 13.5: "Your policy should be to fail to alert any call by partner (thus intentionally creating MI) whenever it is reasonably likely that choosing to obey Law 21 and Law 40 will cause partner to be constrained by Law 75A, thus masochistically disadvantaging your side." Peter Eidt: >>I'm coming out to be a masochist. Best wishes Richard Hills Recruitment Section Specialist Recruitment Team Level 5 Aqua, workstation W569, 6223 8453 DIAC Social Club movie tickets -------------------------------------------------------------------- Important Notice: If you have received this email by mistake, please advise the sender and delete the message and attachments immediately. This email, including attachments, may contain confidential, sensitive, legally privileged and/or copyright information. Any review, retransmission, dissemination or other use of this information by persons or entities other than the intended recipient is prohibited. DIAC respects your privacy and has obligations under the Privacy Act 1988. The official departmental privacy policy can be viewed on the department's website at www.immi.gov.au. See: http://www.immi.gov.au/functional/privacy.htm --------------------------------------------------------------------- From richard.hills at immi.gov.au Wed Jan 5 04:20:56 2011 From: richard.hills at immi.gov.au (richard.hills at immi.gov.au) Date: Wed, 5 Jan 2011 14:20:56 +1100 Subject: [BLML] uninteresting dWikiS case [SEC=UNOFFICIAL] In-Reply-To: <000701cbabf1$cb590cf0$620b26d0$@no> Message-ID: Sven Pran: >I (like many if not all competent directors) assume that a >player obeys Law 72B1. This is a matter of confidence, and >whenever it becomes clear that a player has violated this >law then the reaction will be so much more severe. Richard Hills: In 1986 I was unaware of the existence of what is now Law 72B1. Thus I intentionally perpetrated an insufficient bid because I believed that I would retain a residual advantage after the consequential rectification. Luckily(1) my then partner was better versed in the Laws, so explained my violation of Propriety(2) to me. Sven Pran: >Have you ever watched Snooker at high level? I have seen >players calling attention to their own irregularity when >nobody else, and in particular not the referee could >notice it. Law 79A2: "A player must not knowingly accept either the score for a trick that his side did not win or the concession of a trick that his opponents could not lose." Richard Hills: Because of the word "knowingly", often nobody else and particularly not the Director could notice an infraction of Law 79A2. However, on numerous occasions I have seen an ethical but slightly incompetent semi-expert claim, with the claim then agreed by the opponents, but a few boards later the semi- expert summoned the Director pursuant to Law 79A2 because upon reconsideration he realised that he had incompetently claimed too many tricks. Best wishes Richard Hills (1) It seems that Brussels players are less lucky in their available mentors. (2) It is contrary to the official definition of "must not" when a Brussels mentor argues that an intentional Law 72B1 infraction is not a heinous crime. Alain Gottcheiner: >>..... >>I can show you a case where the creation of UI was >>intended, without any doubt. Richard Hills: When a Brussels mentor strongly and repeatedly advocates intentional infractions of the creation-of-MI Laws 21 and 40, it is not at all surprising that some Brussels players believe that an intentional infraction of the unnecessary- creation-of-UI Law 73A is not a heinous crime. -------------------------------------------------------------------- Important Notice: If you have received this email by mistake, please advise the sender and delete the message and attachments immediately. This email, including attachments, may contain confidential, sensitive, legally privileged and/or copyright information. Any review, retransmission, dissemination or other use of this information by persons or entities other than the intended recipient is prohibited. DIAC respects your privacy and has obligations under the Privacy Act 1988. The official departmental privacy policy can be viewed on the department's website at www.immi.gov.au. See: http://www.immi.gov.au/functional/privacy.htm --------------------------------------------------------------------- From richard.hills at immi.gov.au Wed Jan 5 07:10:12 2011 From: richard.hills at immi.gov.au (richard.hills at immi.gov.au) Date: Wed, 5 Jan 2011 17:10:12 +1100 Subject: [BLML] uninteresting dWikiS case [SEC=UNOFFICIAL] In-Reply-To: <4D1F1252.60504@skynet.be> Message-ID: Herman De Wael: >You misunderstand my position. >I am, on blml, arguing against the WBF interpretation. Saint Augustine: Roma locuta est; causa finita est. Rome has spoken; the case is concluded. WBF Laws Committee minutes (as ratified by the supreme WBF authority, not Rome, but the WBF Executive), page 5, 10th October 2008, the official interpretation of Law 20: "There is no infraction when a correct explanation discloses that partner's prior explanation was mistaken. The words 'nor may he indicate in any manner that a mistake has been made' (in Law 20F5(a)) do not refer to compliance with the overriding requirement of the laws always to respond to enquiries under Law 20F with correct explanations of the partnership understandings." Herman De Wael: >But I do not see, not even after the Beijing >pronouncement (which does not impose an automatic PP, >does it?), that I am going against the laws and >regulations. Richard Hills: Ridiculous. Combining the logical fallacy of "moving the goalposts" (Herman has previously admitted that his pet theory infracted the Laws) with the logical fallacy of "special pleading". (1) An infraction remains an infraction whether or not a procedural penalty is mentioned in that specific Law, and (2) Law 40C3(b): "Repeated violations of requirements to disclose partnership understandings may be PENALIZED." What's the problem? The problem is that Herman De Wael needs an optometrist, since his blurred vision prevents him reading "no infraction when a correct explanation discloses that partner's prior explanation was mistaken" and "over- riding requirement of the laws ... correct explanations". Best wishes Richard Hills Recruitment Section Specialist Recruitment Team Level 5 Aqua, workstation W569, 6223 8453 DIAC Social Club movie tickets -------------------------------------------------------------------- Important Notice: If you have received this email by mistake, please advise the sender and delete the message and attachments immediately. This email, including attachments, may contain confidential, sensitive, legally privileged and/or copyright information. Any review, retransmission, dissemination or other use of this information by persons or entities other than the intended recipient is prohibited. DIAC respects your privacy and has obligations under the Privacy Act 1988. The official departmental privacy policy can be viewed on the department's website at www.immi.gov.au. See: http://www.immi.gov.au/functional/privacy.htm --------------------------------------------------------------------- From Hermandw at skynet.be Wed Jan 5 08:36:50 2011 From: Hermandw at skynet.be (Herman De Wael) Date: Wed, 05 Jan 2011 08:36:50 +0100 Subject: [BLML] uninteresting dWikiS case [SEC=UNOFFICIAL] In-Reply-To: References: Message-ID: <4D241F92.5080900@skynet.be> richard.hills at immi.gov.au wrote: > Herman De Wael: > >> You misunderstand my position. >> I am, on blml, arguing against the WBF interpretation. > > Saint Augustine: > > Roma locuta est; causa finita est. > Rome has spoken; the case is concluded. > > WBF Laws Committee minutes (as ratified by the supreme > WBF authority, not Rome, but the WBF Executive), page 5, > 10th October 2008, the official interpretation of Law 20: > > "There is no infraction when a correct explanation > discloses that partner's prior explanation was mistaken. > The words 'nor may he indicate in any manner that a > mistake has been made' (in Law 20F5(a)) do not refer to > compliance with the overriding requirement of the laws > always to respond to enquiries under Law 20F with correct > explanations of the partnership understandings." > The fact that Rome has spoken does not make it right. I can still argue against Rome, can't I? It's not because you are infallible that you must be right. Pope Urban VIII condemned Galileo - did that make him right? -- Herman De Wael Wilrijk Antwerpen Belgium From blml at arcor.de Wed Jan 5 08:57:21 2011 From: blml at arcor.de (Thomas Dehn) Date: Wed, 5 Jan 2011 08:57:21 +0100 (CET) Subject: [BLML] uninteresting dWikiS case [SEC=UNOFFICIAL] In-Reply-To: References: Message-ID: <1707395149.304158.1294214241521.JavaMail.ngmail@webmail12.arcor-online.net> richard.hills at immi.gov.au wrote: > Thomas Dehn: > > >..... > >Moreover, there are at least the following: > >A) What would Kaplan have done? > > Richard Hills: > > What Kaplan advocated in the 1960s was wildly different to > what Kaplan advocated in the 1990s. > > Thomas Dehn: > > >B) The intent of the lawmakers > > Richard Hills: > > A standard principle of jurisprudence is that the intent of > the lawmakers is relevant only to a carelessly written > ambiguous Law (for example, the second sentence of the 2007 > Law 55A). But if there is a carelessly written unambiguous > Law (for example, the notorious 1997 Law 25B), then the > actual text of the unambiguous Law prevails no matter what > the original intent of the careless Drafting Committee was. > > Thomas Dehn: > > >C) The literal text of the original English version of the > > law > >D) The literal text of the translation > > Richard Hills: > > The WBF has specifically ruled that the English version of > the Laws prevails over a translation if the two versions > have an inconsistency. It is natural for the WBF to make such a ruling, but that ruling does not remove the practical limitation that most TDs and players will nevertheless have to base their own interpretation of TFLB upon the translated text. That is what they have, and what they have the necessary language skills to understand. Thomas From blml at arcor.de Wed Jan 5 09:01:33 2011 From: blml at arcor.de (Thomas Dehn) Date: Wed, 5 Jan 2011 09:01:33 +0100 (CET) Subject: [BLML] uninteresting dWikiS case [SEC=UNOFFICIAL] In-Reply-To: <4D241F92.5080900@skynet.be> References: <4D241F92.5080900@skynet.be> Message-ID: <1383433544.304248.1294214493902.JavaMail.ngmail@webmail12.arcor-online.net> Herman De Wael wrote: > richard.hills at immi.gov.au wrote: > > Herman De Wael: > > > >> You misunderstand my position. > >> I am, on blml, arguing against the WBF interpretation. > > > > Saint Augustine: > > > > Roma locuta est; causa finita est. > > Rome has spoken; the case is concluded. > > > > WBF Laws Committee minutes (as ratified by the supreme > > WBF authority, not Rome, but the WBF Executive), page 5, > > 10th October 2008, the official interpretation of Law 20: > > > > "There is no infraction when a correct explanation > > discloses that partner's prior explanation was mistaken. > > The words 'nor may he indicate in any manner that a > > mistake has been made' (in Law 20F5(a)) do not refer to > > compliance with the overriding requirement of the laws > > always to respond to enquiries under Law 20F with correct > > explanations of the partnership understandings." > > > > The fact that Rome has spoken does not make it right. I can still argue > against Rome, can't I? You can. And Rome then can have you burned at the stake. Rome could also have you burned at the stake if you don't argue against Rome. But they are more likely to do so if you argue against Rome. Thomas From harald.skjaran at gmail.com Wed Jan 5 09:37:45 2011 From: harald.skjaran at gmail.com (=?UTF-8?Q?Harald_Skj=C3=A6ran?=) Date: Wed, 5 Jan 2011 09:37:45 +0100 Subject: [BLML] The Monty Hall trap In-Reply-To: <821566.28383.qm@web28515.mail.ukl.yahoo.com> References: <95857DFE-B801-4A9C-B54A-84A71A5EE674@starpower.net> <821566.28383.qm@web28515.mail.ukl.yahoo.com> Message-ID: 2011/1/5 Nigel Guthrie : > [Eric Landau] > > How could you have the legal auction 1S-1H-1S if responder's 1S call > must have the same meaning as would a 1S response to an opening 1S > bid absent the opponent's irregularity? > > {Nige1] > Yet another legal anomaly, for which director and partnerships will have to > devise work-rounds until 2018. > > > By 2018, I hope that law-makers and players accept what is the root of the > problem: the anomalies are created is by giving players (offenders and victims) > *options* after an illegal call (insufficient bid, call out of turn, or > whatever). > > IMO, an illegal call should be cancelled with no other option for the offender. > The illegal call should be UI to the offender's partner, who should be silenced > for the rest of the auction. The LHO of the offender should not have the option > to accept the illegal call. If he attempts to condone it, that should be treated > as a subsequent infraction. These rules are simple, clear and fair. Justice is > done and seen to be done. They also allow the deal to be bid and played in a far > more normal and understandable way than current legislation permits. An obvious > consequence is that it deters illegal bids. If such deterrence is considered to > be against hallowed "Equity" Principles, so be it. So the partner of an unsufficient bidder will have UI constrictions as a defender, in addition to being silenced for the rest of the auction. Those draconical rules would not be considered fair at all by most players. And absolutely prevent the board from being played in as normal as possible way - as today's lawbook allows. How can silencing a player allow the deal to be bid and played in a normal way??? It's true that the new L27 is somewhat murky. But it's obvious to me that it's far better than any of it's predecessors. > _______________________________________________ > Blml mailing list > Blml at rtflb.org > http://lists.rtflb.org/mailman/listinfo/blml > -- Kind regards, Harald Skj?ran From blml at arcor.de Wed Jan 5 10:09:32 2011 From: blml at arcor.de (Thomas Dehn) Date: Wed, 5 Jan 2011 10:09:32 +0100 (CET) Subject: [BLML] The Monty Hall trap [SEC=UNOFFICIAL] In-Reply-To: References: Message-ID: <1697843461.306431.1294218572993.JavaMail.ngmail@webmail12.arcor-online.net> richard.hills at immi.gov.au wrote: > Law 40B3: > > "The Regulating Authority may disallow prior agreement by a > partnership to vary its understandings during the auction or > play following a question asked, a response to a question, or > any irregularity." > > Thomas Dehn: > > >I don't think that applies here. > > > >Having a different agreement after an IB is not a variation > >of understandings following an irregularity, it is a > >different auction. > > Eric Landau: > > [snip] > > >How could you have the legal auction 1S-1H-1S if responder's > >1S call must have the same meaning as would a 1S response to > >an opening 1S bid absent the opponent's irregularity? > > The Fabulous Law Book's advice on the meaning of "meaning". > > Law 17 footnote: > > "* For example, a substituted call differs if its meaning is > much different or if it is psychic." > > Law 27 footnote: > > "* the meaning of (information available from) a call is the > knowledge of what it shows and what it excludes." > > Law 40B1(a): > > "In its discretion the Regulating Authority may designate > certain partnership understandings as 'special partnership > understandings'. A special partnership understanding is one > whose meaning, in the opinion of the Regulating Authority, > may not be readily understood and anticipated by a > significant number of players in the tournament." > > Richard Hills: > > So if I was an ACBL Director, I would tell the Dehn-Landau > partnership that Law 40B3 does indeed apply after an > accepted insufficient bid (advising them that their sea- > lawyering, if they wish to appeal, cannot be upheld by an > ordinary ACBL Appeals Committee, due to Law 93B3, although > of course they still have an undisputed right to appeal to > an ordinary ACBL Appeals Committee), and for this auction > > WEST NORTH EAST > 1S 1H 1S > > the meaning of East's 1S must be not be an ACBL Special > Partnership Understanding, that is not much different nor > psychic when compared to the meaning of East's 2S in this > auction > > WEST NORTH EAST > 1S Pass 2S > > What's the problem? I fear indeed that thus far Eric and me have failed at getting across the problem. You are playing with an unknown partner, especially you have no experience how this partner handles insufficient bids. The following auction develops: Partner LHO YOU RHO 1S 2H 4S 4H (IB) 4NT pass Do you take partner's 4NT as RKCB (what 4NT would have meant without RHO's insufficient bid), or as Josephine ("not much different nor psychic when compared to the meaning of 5NT over a possible 5H bid")? Thomas From blml at arcor.de Wed Jan 5 10:18:17 2011 From: blml at arcor.de (Thomas Dehn) Date: Wed, 5 Jan 2011 10:18:17 +0100 (CET) Subject: [BLML] interesting dWS case [SEC=UNOFFICIAL] In-Reply-To: <000701cbabf1$cb590cf0$620b26d0$@no> References: <000701cbabf1$cb590cf0$620b26d0$@no> <000901cbab63$d866f850$8934e8f0$@no> <184B4C9981744CF4BC3A20BF59F0D334@Mildred> <4D1C4068.9070905@skynet.be> <1957070655.97086.1293699676312.JavaMail.ngmail@webmail18.arcor-online.net> <98830332.101890.1293710782116.JavaMail.ngmail@webmail13.arcor-online.net> <4D1C9FB6.9000808@gmail.com> <4D21ECE6.1030708@ulb.ac.be> <1668949171.272724.1294115313586.JavaMail.ngmail@webmail10.arcor-online.net> Message-ID: <1436151551.306629.1294219097444.JavaMail.ngmail@webmail12.arcor-online.net> Sven Pran wrote: > On Behalf Of Thomas Dehn > > Sven Pran wrote: > > > .................. > > > But bridge is a game where you do not violate the rules even if you > > > are willing to "pay the penalty"; that makes a comparison between > > > soccer and bridge completely irrelevant. > > > > That is a fiction. It is not a reality. I have observed many incidents where bridge > > players quite deliberately broke the law and regulations. From little things such as > > not having a correctly filled in convention card to not so little things like > > deliberately using UI or claiming when they notice that they have only 12 cards to > > actual cheating such as "reverse hesitations". > > I (like many if not all competent directors) assume that a player obeys Law > 72B1. This is a matter of confidence, and whenever it becomes clear that a > player has violated this law then the reaction will be so much more severe. I am pretty sure that the percentage of tournament bridge players which are aware of L72B1 is less than 50%. > > A rulebook does not create a culture of not violating the rules. > > Pretty much the only game or sport that actually has such a culture is golf. With > > everything else, quite a few players will deliberately break the rules once in a > > while. > > Have you ever watched Snooker at high level? I have seen players calling > attention to their own irregularity when nobody else, and in particular not > the referee could notice it. Such things happen in most games and sports. But what is the percentage? In football it is pretty low. In bridge it is higher than in football, but clearly nowhere near 100%. Just think about what happens when you get called to the table because there was an alleged hesitation. Somehow, the NOS' estimation of how long the hesitation took is (almost) never lower than the OS' estimation. Thomas From nigelguthrie at yahoo.co.uk Wed Jan 5 11:33:33 2011 From: nigelguthrie at yahoo.co.uk (Nigel Guthrie) Date: Wed, 5 Jan 2011 10:33:33 +0000 (GMT) Subject: [BLML] uninteresting dWikiS case [SEC=UNOFFICIAL] In-Reply-To: <4D241F92.5080900@skynet.be> References: <4D241F92.5080900@skynet.be> Message-ID: <901893.41253.qm@web28514.mail.ukl.yahoo.com> [Herman de Wael] The fact that Rome has spoken does not make it right. I can still argue against Rome, can't I? It's not because you are infallible that you must be right. Pope Urban VIII condemned Galileo - did that make him right? [Nigel] Herman is not defending Civil rights or Astronomical truth. We are discussing the rules of a *game*. Their main requirement is that players enjoy the resulting game. To that end, the rules should be simple enough to be consistently enforced and for players to understand them and to comply with them. In that sense they need to be fair. Game-rules define an artificial and arbitrary world, however. That ethos of that world need not reflect real-world morality: Liar-dice rewards lying. Diplomacy rewards duplicity. The winner of Hangman executes his opponent. If you don't like the rules of a game, there is no need to break them. If you break them deliberately to gain advantage, you are simply a cheat; you spoil the enjoyment of the game for others. Instead, you may try to persuade the rule-makers to change them. If that fails you can just invent a new game with your own rules. From nigelguthrie at yahoo.co.uk Wed Jan 5 11:54:21 2011 From: nigelguthrie at yahoo.co.uk (Nigel Guthrie) Date: Wed, 5 Jan 2011 10:54:21 +0000 (GMT) Subject: [BLML] The Monty Hall trap In-Reply-To: References: <95857DFE-B801-4A9C-B54A-84A71A5EE674@starpower.net> <821566.28383.qm@web28515.mail.ukl.yahoo.com> Message-ID: <804455.33281.qm@web28513.mail.ukl.yahoo.com> [Harald Skj?ran] So the partner of an unsufficient bidder will have UI constrictions as a defender, in addition to being silenced for the rest of the auction. Those draconical rules would not be considered fair at all by most players. And absolutely prevent the board from being played in as normal as possible way - as today's lawbook allows. How can silencing a player allow the deal to be bid and played in a normal way??? It's true that the new L27 is somewhat murky. But it's obvious to me that it's far better than any of it's predecessors. {Nigel] Currently few players have adjusted their system to take advantage of the options offered to offenders and victims by insufficient-bid and bid-out-of-turn laws; but some secretary-birds have do so. Also, as explained here and elsewhere, some adjustments are obvious to an intelligent player, without discussion. When and where such adjustments are legal is a moot point. For example, Richard Hills and Gratton Endicott have different views from David Stevenson and others. (I hope I'm not misrepresenting anybody's position. I don't do so intentionally). Anyway, I disagree with Harald: I feel that players distrust sophisticated rules that they don't understand and are inconsistently enforced. I think they prefer simple fair comprehensible rules. The suggested rule is just a suggestion. There have been many other suggested simplifications. My point is that law-makers should adopt a rule, preferably a simpler rule, that offers no options to offenders or their victims. From Hermandw at skynet.be Wed Jan 5 13:10:18 2011 From: Hermandw at skynet.be (Herman De Wael) Date: Wed, 05 Jan 2011 13:10:18 +0100 Subject: [BLML] uninteresting dWikiS case [SEC=UNOFFICIAL] In-Reply-To: <901893.41253.qm@web28514.mail.ukl.yahoo.com> References: <4D241F92.5080900@skynet.be> <901893.41253.qm@web28514.mail.ukl.yahoo.com> Message-ID: <4D245FAA.5020208@skynet.be> Nigel Guthrie wrote: > [Herman de Wael] > > The fact that Rome has spoken does not make it right. I can still argue > against Rome, can't I? It's not because you are infallible that you must be > right. > Pope Urban VIII condemned Galileo - did that make him right? > > [Nigel] > > Herman is not defending Civil rights or Astronomical truth. We are discussing > the rules of a *game*. Their main requirement is that players enjoy the > resulting game. To that end, the rules should be simple enough to be > consistently enforced and for players to understand them and to comply with > them. In that sense they need to be fair. > > > Game-rules define an artificial and arbitrary world, however. That ethos of that > world need not reflect real-world morality: Liar-dice rewards lying. Diplomacy > rewards duplicity. The winner of Hangman executes his opponent. > > > If you don't like the rules of a game, there is no need to break them. If you > break them deliberately to gain advantage, you are simply a cheat; you spoil the > enjoyment of the game for others. > > Instead, you may try to persuade the rule-makers to change them. If that fails > you can just invent a new game with your own rules. > And ij what way do you think I should do that? Write on blml perhaps? Isn't that what I am doing? You talk about "the rules should be simple enough". Have you tried explaining the Beijing interpretation to a player? Did he understand it? Is it simple? "you are not allowed to correct partner's wrong explanation in any manner, but you must correctly=wrongly explain his next bid, even if that corrects his previous explanation." Simple? -- Herman De Wael Wilrijk Antwerpen Belgium From Hermandw at skynet.be Wed Jan 5 13:11:29 2011 From: Hermandw at skynet.be (Herman De Wael) Date: Wed, 05 Jan 2011 13:11:29 +0100 Subject: [BLML] uninteresting dWikiS case [SEC=UNOFFICIAL] In-Reply-To: <1383433544.304248.1294214493902.JavaMail.ngmail@webmail12.arcor-online.net> References: <4D241F92.5080900@skynet.be> <1383433544.304248.1294214493902.JavaMail.ngmail@webmail12.arcor-online.net> Message-ID: <4D245FF1.9000000@skynet.be> Thomas Dehn wrote: >> >> The fact that Rome has spoken does not make it right. I can still argue >> against Rome, can't I? > > You can. > And Rome then can have you burned at the stake. > > Rome could also have you burned at the stake if > you don't argue against Rome. But they are more > likely to do so if you argue against Rome. > Is that what it has come to. We have spoken, and if Herman does not agree, we shall burn him at the stake? > > Thomas -- Herman De Wael Wilrijk Antwerpen Belgium From svenpran at online.no Wed Jan 5 13:41:09 2011 From: svenpran at online.no (Sven Pran) Date: Wed, 5 Jan 2011 13:41:09 +0100 Subject: [BLML] uninteresting dWikiS case [SEC=UNOFFICIAL] In-Reply-To: <4D245FAA.5020208@skynet.be> References: <4D241F92.5080900@skynet.be> <901893.41253.qm@web28514.mail.ukl.yahoo.com> <4D245FAA.5020208@skynet.be> Message-ID: <000d01cbacd5$d536eff0$7fa4cfd0$@no> On Behalf Of Herman De Wael ............... > You talk about "the rules should be simple enough". Have you tried explaining the > Beijing interpretation to a player? Did he understand it? > Is it simple? "you are not allowed to correct partner's wrong explanation in any > manner, but you must correctly=wrongly explain his next bid, even if that corrects > his previous explanation." Simple? I have many times had to tell players that they are NEVER entitled to be told what cards a player actually holds, they are entitled to be told what agreements and understandings the partnership has. This is particularly relevant when a player eventually discovers that an opponent did not hold cards that correspond to the explanation given. Then I have to explain the difference between misinformation which may result in a rectification, and misbid which presumably is as surprising to his partner as to his opponents and then does not result in any rectification. I don't know what experience you have in this respect, but I most often succeed in making the player understand (and accept) my ruling. From blml at arcor.de Wed Jan 5 13:43:57 2011 From: blml at arcor.de (Thomas Dehn) Date: Wed, 5 Jan 2011 13:43:57 +0100 (CET) Subject: [BLML] uninteresting dWikiS case [SEC=UNOFFICIAL] In-Reply-To: <4D245FF1.9000000@skynet.be> References: <4D245FF1.9000000@skynet.be> <4D241F92.5080900@skynet.be> <1383433544.304248.1294214493902.JavaMail.ngmail@webmail12.arcor-online.net> Message-ID: <16543374.313152.1294231437117.JavaMail.ngmail@webmail12.arcor-online.net> Herman De Wael wrote: > Thomas Dehn wrote: > >> > >> The fact that Rome has spoken does not make it right. I can still argue > >> against Rome, can't I? > > > > You can. > > And Rome then can have you burned at the stake. > > > > Rome could also have you burned at the stake if > > you don't argue against Rome. But they are more > > likely to do so if you argue against Rome. > > > > Is that what it has come to. We have spoken, and if Herman does not > agree, we shall burn him at the stake? Modern times are not that violent. Still, it seems that quite some people expect you to simply shut up. Thomas From richard.willey at gmail.com Wed Jan 5 14:09:48 2011 From: richard.willey at gmail.com (richard willey) Date: Wed, 5 Jan 2011 08:09:48 -0500 Subject: [BLML] uninteresting dWikiS case [SEC=UNOFFICIAL] In-Reply-To: <16543374.313152.1294231437117.JavaMail.ngmail@webmail12.arcor-online.net> References: <4D241F92.5080900@skynet.be> <1383433544.304248.1294214493902.JavaMail.ngmail@webmail12.arcor-online.net> <4D245FF1.9000000@skynet.be> <16543374.313152.1294231437117.JavaMail.ngmail@webmail12.arcor-online.net> Message-ID: On Wed, Jan 5, 2011 at 7:43 AM, Thomas Dehn wrote: Herman De Wael wrote: >> Is that what it has come to. We have spoken, and if Herman does not >> agree, we shall burn him at the stake? > Modern times are not that violent. > Still, it seems that quite some people > expect you to simply shut up. I think a metaphorical "witch burning" is more than reasonable. I would define "witch burning" as 1. Yanking your license to direct games 2. Not allowing you to volunteer your services as scribe or serve in an official or semi official capacity at WBF events. 3. Making it damn clear to everyone just why this al happened > -- > I think back to the halcyon dates of my youth, when indeterminate Hessians > had something to do with the Revolutionary War, where conjugate priors were > monks who had broken their vows, and the expression (X'X)^-1(X'Y) was greek > > Those were simpler times > -------------- next part -------------- An HTML attachment was scrubbed... URL: http://lists.rtflb.org/pipermail/blml/attachments/20110105/1d693a4c/attachment.html From ehaa at starpower.net Wed Jan 5 14:52:06 2011 From: ehaa at starpower.net (Eric Landau) Date: Wed, 5 Jan 2011 08:52:06 -0500 Subject: [BLML] The Monty Hall trap In-Reply-To: References: Message-ID: On Jan 4, 2011, at 4:47 PM, richard.hills at immi.gov.au wrote: > Law 40B3: > > "The Regulating Authority may disallow prior agreement by a > partnership to vary its understandings during the auction or > play following a question asked, a response to a question, or > any irregularity." > > Thomas Dehn: > >> I don't think that applies here. >> >> Having a different agreement after an IB is not a variation >> of understandings following an irregularity, it is a >> different auction. > > Eric Landau: > > [snip] > >> How could you have the legal auction 1S-1H-1S if responder's >> 1S call must have the same meaning as would a 1S response to >> an opening 1S bid absent the opponent's irregularity? > > The Fabulous Law Book's advice on the meaning of "meaning". > > Law 17 footnote: > > "* For example, a substituted call differs if its meaning is > much different or if it is psychic." > > Law 27 footnote: > > "* the meaning of (information available from) a call is the > knowledge of what it shows and what it excludes." > > Law 40B1(a): > > "In its discretion the Regulating Authority may designate > certain partnership understandings as 'special partnership > understandings'. A special partnership understanding is one > whose meaning, in the opinion of the Regulating Authority, > may not be readily understood and anticipated by a > significant number of players in the tournament." > > Richard Hills: > > So if I was an ACBL Director, I would tell the Dehn-Landau > partnership that Law 40B3 does indeed apply after an > accepted insufficient bid (advising them that their sea- > lawyering, if they wish to appeal, cannot be upheld by an > ordinary ACBL Appeals Committee, due to Law 93B3, although > of course they still have an undisputed right to appeal to > an ordinary ACBL Appeals Committee), and for this auction > > WEST NORTH EAST > 1S 1H 1S > > the meaning of East's 1S must be not be an ACBL Special > Partnership Understanding, that is not much different nor > psychic when compared to the meaning of East's 2S in this > auction > > WEST NORTH EAST > 1S Pass 2S > > What's the problem? The problem is that you are not only requiring the meaning of 1S-1H-1S to be "not much different" from the meaning of 1S-P-2S, but would require the same of 1S-1H-2S or of 1S-(1H)/2H-2S. So that would mean that you are required to ascribe essentially the same meaning to three different auctions, when common sense and straightforward "bridge logic" demands that they can't all mean the same thing. Eric Landau 1107 Dale Drive Silver Spring MD 20910 ehaa at starpower.net From richard.hills at immi.gov.au Wed Jan 5 22:12:31 2011 From: richard.hills at immi.gov.au (richard.hills at immi.gov.au) Date: Thu, 6 Jan 2011 08:12:31 +1100 Subject: [BLML] ABF National Authority 4th March 2010 [SEC=UNOFFICIAL] In-Reply-To: Message-ID: Herman De Wael: >The fact that Rome has spoken does not make it right. I can >still argue against Rome, can't I? >..... Thomas Cathcart and Daniel Klein, Plato And A Platypus Walk Into A Bar, page 45 (with bonus RH footnotes added): Four rabbis used to argue theology together, and three(1) were always in accord against the fourth(2). One day, the odd rabbi out, after losing three to one again, decided to appeal to a higher authority(3). "O, God!" he cried. "I know in my heart that I am right and they are wrong! Please give me a sign to prove it to them!" ..... the sky turned pitch-black, the earth shook, and a deep, booming voice intoned, "HEEEEEEEE'S RIIIIIIIGHT!" The rabbi put his hand on his hips, turned to the other three, and said, "Well?" "So," shrugged one of the other rabbis, "now it's three to two." (1) e.g. rabbis Herman De Wael, Herman De Wael & Herman De Wael (2) e.g. rabbi Richard Willey (3) e.g. the WBF Laws Committee +=+=+=+=+=+=+=+=+=+=+=+=+=+=+=+=+=+=+=+=+=+=+=+=+=+=+=+=+=+=+=+= http://www.abf.com.au/about/minutes/namin1004.pdf [information from original post not repeated here] The adjustment to the score: As arising out of the facts set out at a-d,the applicable law is Law 79 which reads as follows: Law 79 TRICKS WON A. Agreement on Tricks Won 1. The number of tricks won shall be agreed upon before all four hands have been returned to the board. 2. A player must not knowingly accept either the score for a trick that his side did not win or the concession of a trick that his opponents could not lose. B. Disagreement on Tricks Won If a subsequent disagreement arises, the Director must be called, then: 1. The Director determines whether there has been a claim or concession and, if so, applies Law 69. 2. If 1 does not apply the Director rules what score is to be recorded. If the Director is not called before the round ends he rules in accordance with C below or Law 87, as applicable, but there shall be no obligation to increase a side's score. C. Error in Score 1. An error in computing or tabulating the agreed-upon score, whether made by a player or scorer, may be corrected until the expiration of the period specified by the Tournament Organizer. Unless the Tournament Organizer specifies a later* time, this Correction Period expires 30 minutes after the official score has been made available for inspection. 2. Regulations may provide for circumstances in which a scoring error may be corrected after expiry of the Correction Period if the Director and the Tournament Organizer are both satisfied beyond reasonable doubt that the record is wrong. * An earlier time may be specified when required by the special nature of a contest. The National Authority noted that if an electronic scoring machine is used according to the instructions the fact that the score has been entered by North/South and agreed by East/West provides prima facie evidence that the score has been agreed. The National Authority further noted that there was a common law doctrine (non est factum) which states that if a person does not apply his/her mind to what is being done then their purported agreement to a score is not binding. The National Authority considered whether the words "computing or tabulating" as used in Law 79C were apt to catch the situation where an error arose because a player did not apply his/her mind to the question of the score to be recorded. The National Authority was not satisfied that such an error fell within this definition but noted that it has long been the practice where a score is queried for the director to discuss the matter with representatives from both sides and to adjust the score after discussion with representatives from both pairs who were present at the table. Law 79B is applicable to this situation. A disagreement has subsequently arisen. Law 79B2 clearly gives the director the power to rule "what score is to be recorded". There was no significant difference between the account of the facts provided to the National Authority by Laurie Kelso and by the Hinge team. A director had clearly made a ruling that the result would be recorded as 2H making 9 tricks. Appeal rights: The National Authority next considered the facts set out at e and f above. The applicable law is Law 92 which reads as follows: Law 92 RIGHT TO APPEAL A. Contestant's Right A contestant or his captain may appeal for a review of any ruling made at his table by the Director. Any such appeal, if deemed to lack merit, may be the subject of a sanction imposed by regulation. B. Time of Appeal The right to request or appeal a Director's ruling expires 30 minutes after the official score has been made available for inspection unless the Tournament Organizer has specified a different time period. C. How to Appeal All appeals shall be made through the Director. D. Concurrence of Appellants An appeal shall not be heard unless 1. in a pairs event both members of the partnership concur in making the appeal (but in an individual contest an appellant does not require his partner's concurrence). 2. in a team event the team captain concurs in making the appeal. The National Authority noted that the Hinge team had a right of appeal against the decision made by the Director. The phrase "at his table" should be interpreted a) to mean at his or her table; and b) to mean applying to the match in question. The National Authority noted that the actions of the Acting Chief Tournament director in promising to investigate the matter had the effect of delaying the appeal against the decision. The National Authority noted that an inquiry as to whether the board had been fouled had been set in train but was fruitless. The National Authority expressed the view that the Acting Chief Tournament Director should have indicated that a ruling had been made and that the ruling would apply unless either an appeal against the ruling was successful or the Acting Chief Tournament Director managed to ascertain facts that led to the ruling being set aside. The National Authority noted that the regulations specified an appeal period and that the appeal period had expired long before the end of Round 14. The National Authority noted that it was open to the Regulations Committee to adopt a regulation designed to encourage players to pay more attention to the use of the electronic scorer. Best wishes Richard Hills Recruitment Section Specialist Recruitment Team Level 5 Aqua, workstation W569, 6223 8453 DIAC Social Club movie tickets -------------------------------------------------------------------- Important Notice: If you have received this email by mistake, please advise the sender and delete the message and attachments immediately. This email, including attachments, may contain confidential, sensitive, legally privileged and/or copyright information. Any review, retransmission, dissemination or other use of this information by persons or entities other than the intended recipient is prohibited. DIAC respects your privacy and has obligations under the Privacy Act 1988. The official departmental privacy policy can be viewed on the department's website at www.immi.gov.au. See: http://www.immi.gov.au/functional/privacy.htm --------------------------------------------------------------------- From richard.hills at immi.gov.au Wed Jan 5 23:18:40 2011 From: richard.hills at immi.gov.au (richard.hills at immi.gov.au) Date: Thu, 6 Jan 2011 09:18:40 +1100 Subject: [BLML] ABF National Authority 4th March 2010 [SEC=UNOFFICIAL] In-Reply-To: <764142.92719.qm@web28501.mail.ukl.yahoo.com> Message-ID: Nigel Guthrie: >>One side thought the result was 2H-1. The other 2H+1. But if >>none of the players could remember what happened, who came >>up with the 9 trick line? Richard Hills: The Director, after discussion with both sides, pursuant to the Director's powers under Law 79B2. Nigel Guthrie: >>and why? Richard Hills: According to the Convenor, Sean Mullamphy, the Director's reason for "why?" was due to the hand record indicating nine easy tricks on any normal line of play. Thus under the Law 85A1 "balance of probabilities" it was deemed that the most likely reason for the anomaly was one player erring by pressing 7 instead of 9 on the electronic scorer, and an opposing player erring by then pressing the "accept" button. Best wishes Richard Hills Recruitment Section Specialist Recruitment Team Level 5 Aqua, workstation W569, 6223 8453 DIAC Social Club movie tickets -------------------------------------------------------------------- Important Notice: If you have received this email by mistake, please advise the sender and delete the message and attachments immediately. This email, including attachments, may contain confidential, sensitive, legally privileged and/or copyright information. Any review, retransmission, dissemination or other use of this information by persons or entities other than the intended recipient is prohibited. DIAC respects your privacy and has obligations under the Privacy Act 1988. The official departmental privacy policy can be viewed on the department's website at www.immi.gov.au. See: http://www.immi.gov.au/functional/privacy.htm --------------------------------------------------------------------- From richard.hills at immi.gov.au Thu Jan 6 00:05:53 2011 From: richard.hills at immi.gov.au (richard.hills at immi.gov.au) Date: Thu, 6 Jan 2011 10:05:53 +1100 Subject: [BLML] The Monty Hall trap [SEC=UNOFFICIAL] In-Reply-To: Message-ID: Eric Landau: >..... >when common sense and straightforward "bridge logic" demands >that they can't all mean the same thing. Richard Hills: Back in 1973, during the Watergate scandal, a popular bumper sticker for ACBL-land cars was, "Don't blame me, I'm from Massachusetts" which was a reference to Massachusetts being the only state which did not vote for President Nixon's re-election.(1) Don't blame me, I'm from Canberra. It is not my fault that an ACBL prohibition is contrary to a Eric Landau's common sense and contrary to Eric Landau's straightforward logic. Although the ACBL prohibition is straightforwardly logical. An ACBL little old lady would be distressed by her insufficient bid. But that ACBL little old lady would be outraged if a cunning smarty-pants opponent was then allowed to rub salt in her wounds by an esoteric variation of system. (ABF little old ladies are not outraged by esoteric methods, since in ABF-land little old ladies employ all sorts of esoteric methods themselves.) Best wishes Richard Hills (1) Nixon's re-election slogan in 1972 was "Four more years." Updated in 1973, "With time off for good behaviour." And Nixon's 1968 slogan did not need updating in 1973, "Nixon's the one." -------------------------------------------------------------------- Important Notice: If you have received this email by mistake, please advise the sender and delete the message and attachments immediately. This email, including attachments, may contain confidential, sensitive, legally privileged and/or copyright information. Any review, retransmission, dissemination or other use of this information by persons or entities other than the intended recipient is prohibited. DIAC respects your privacy and has obligations under the Privacy Act 1988. The official departmental privacy policy can be viewed on the department's website at www.immi.gov.au. See: http://www.immi.gov.au/functional/privacy.htm --------------------------------------------------------------------- From JffEstrsn at aol.com Thu Jan 6 00:44:26 2011 From: JffEstrsn at aol.com (Jeff Easterson) Date: Thu, 06 Jan 2011 00:44:26 +0100 Subject: [BLML] Herman Message-ID: <4D25025A.8020209@aol.com> May I defend Herman? I often disagree with him and think that he often exaggerates or (at least in the past) attempts to provoke dicussion and seems to me to occasionally argue unreasonably. (But this was much more frequent a few years ago.) But even if I disagree and even occasionally suspect him of sophism, I'd like to defend his right to express his opinions. No one is forced to respond to his postings, or even read them. You have the option of simply ignoring him if you think he is introducing a red herring. But often (at least, in my opinion, lately) he has made interesting and constructive suggestions and argued quite convincingly for his position. And, he seems to draw a line between his postings on blml and his work as a TD. (At least I hope he does.) Ciao, JE From JffEstrsn at aol.com Thu Jan 6 01:12:51 2011 From: JffEstrsn at aol.com (Jeff Easterson) Date: Thu, 06 Jan 2011 01:12:51 +0100 Subject: [BLML] Herman In-Reply-To: <4D25025A.8020209@aol.com> References: <4D25025A.8020209@aol.com> Message-ID: <4D250903.8010909@aol.com> Pardon, I meant sophistry. Am 06.01.2011 00:44, schrieb Jeff Easterson: > May I defend Herman? I often disagree with him and think that he often > exaggerates or (at least in the past) attempts to provoke dicussion and > seems to me to occasionally argue unreasonably. (But this was much more > frequent a few years ago.) But even if I disagree and even occasionally > suspect him of sophism, I'd like to defend his right to express his > opinions. No one is forced to respond to his postings, or even read > them. You have the option of simply ignoring him if you think he is > introducing a red herring. But often (at least, in my opinion, lately) > he has made interesting and constructive suggestions and argued quite > convincingly for his position. And, he seems to draw a line between his > postings on blml and his work as a TD. (At least I hope he does.) > Ciao, JE > _______________________________________________ > Blml mailing list > Blml at rtflb.org > http://lists.rtflb.org/mailman/listinfo/blml > From richard.hills at immi.gov.au Thu Jan 6 02:05:29 2011 From: richard.hills at immi.gov.au (richard.hills at immi.gov.au) Date: Thu, 6 Jan 2011 12:05:29 +1100 Subject: [BLML] uninteresting dWikiS case [SEC=UNOFFICIAL] In-Reply-To: <901893.41253.qm@web28514.mail.ukl.yahoo.com> Message-ID: Nigel Guthrie: >.....If you don't like the rules of a game, there is no need to >break them. If you break them deliberately to gain advantage, >you are simply a cheat..... Richard Hills: In my opinion, if you break the rules of a game deliberately AND SECRETLY to gain an advantage, you are simply a cheat. In my opinion, if you break the rules of a game deliberately AND OPENLY to gain an advantage (e.g. by bragging about your non- masochistic methods on blml), you are simply a simpleton. Best wishes Richard Hills Recruitment Section Specialist Recruitment Team Level 5 Aqua, workstation W569, 6223 8453 DIAC Social Club movie tickets -------------------------------------------------------------------- Important Notice: If you have received this email by mistake, please advise the sender and delete the message and attachments immediately. This email, including attachments, may contain confidential, sensitive, legally privileged and/or copyright information. Any review, retransmission, dissemination or other use of this information by persons or entities other than the intended recipient is prohibited. DIAC respects your privacy and has obligations under the Privacy Act 1988. The official departmental privacy policy can be viewed on the department's website at www.immi.gov.au. See: http://www.immi.gov.au/functional/privacy.htm --------------------------------------------------------------------- From richard.hills at immi.gov.au Thu Jan 6 03:06:44 2011 From: richard.hills at immi.gov.au (richard.hills at immi.gov.au) Date: Thu, 6 Jan 2011 13:06:44 +1100 Subject: [BLML] Herman [SEC=UNOFFICIAL] In-Reply-To: <4D250903.8010909@aol.com> Message-ID: Henk Uijterwaal, 24th April 2009: >I never thought I'd see the words "excellent" and "The Lone >Wolff" in one sentence. I'd put this book in the all time >top 3 of worst bridge books ever written. John (MadDog) Probst, 28th April 2009: Bah, Do you mean The all time bottom three of worst bridge books ever written or do we get Herman to explain that it means something else entirely? Best wishes Richard Hills Recruitment Section Specialist Recruitment Team Level 5 Aqua, workstation W569, 6223 8453 DIAC Social Club movie tickets -------------------------------------------------------------------- Important Notice: If you have received this email by mistake, please advise the sender and delete the message and attachments immediately. This email, including attachments, may contain confidential, sensitive, legally privileged and/or copyright information. Any review, retransmission, dissemination or other use of this information by persons or entities other than the intended recipient is prohibited. DIAC respects your privacy and has obligations under the Privacy Act 1988. The official departmental privacy policy can be viewed on the department's website at www.immi.gov.au. See: http://www.immi.gov.au/functional/privacy.htm --------------------------------------------------------------------- From richard.hills at immi.gov.au Thu Jan 6 03:17:23 2011 From: richard.hills at immi.gov.au (richard.hills at immi.gov.au) Date: Thu, 6 Jan 2011 13:17:23 +1100 Subject: [BLML] Precision's up to date in Kansas City [SEC=UNOFFICIAL] Message-ID: Rodgers and Hammerstein, Oklahoma! (1943): Precision's up to date in Kansas City They gone about as fer as they can go They went an' built some asking bids seven stories high About as high as an auction' orta grow. Matchpoint pairs Board 5 Dlr: North Vul: North-South The bidding has gone: WEST NORTH EAST SOUTH --- 1C (1) Pass 1H (2) Pass 2H (3) Pass 3D (4) Pass 4C (5) Pass 4H (6) Pass Pass(7) Pass (1) Alerted; Precision (2) Alerted; 5+ hearts, game forcing (3) Alerted; trump asking (4) Alerted; 6+ hearts, one honour (5) Alerted; asks about club control (6) Alerted; third-round club control (7) 100% forcing pass, as can easily be deduced from the auction You, West, hold: Q932 5 J9875 642 What opening lead do you make? Best wishes Richard Hills Recruitment Section Specialist Recruitment Team Level 5 Aqua, workstation W569, 6223 8453 DIAC Social Club movie tickets -------------------------------------------------------------------- Important Notice: If you have received this email by mistake, please advise the sender and delete the message and attachments immediately. This email, including attachments, may contain confidential, sensitive, legally privileged and/or copyright information. Any review, retransmission, dissemination or other use of this information by persons or entities other than the intended recipient is prohibited. DIAC respects your privacy and has obligations under the Privacy Act 1988. The official departmental privacy policy can be viewed on the department's website at www.immi.gov.au. See: http://www.immi.gov.au/functional/privacy.htm --------------------------------------------------------------------- From swillner at nhcc.net Thu Jan 6 04:21:36 2011 From: swillner at nhcc.net (Steve Willner) Date: Wed, 05 Jan 2011 22:21:36 -0500 Subject: [BLML] interesting dWS case In-Reply-To: <4D21EBE7.6080302@ulb.ac.be> References: <4D18BD6E.2020106@ulb.ac.be> <4D1C96FD.3060802@meteo.fr> <4D21EBE7.6080302@ulb.ac.be> Message-ID: <4D253540.4040006@nhcc.net> On 1/3/2011 10:31 AM, Alain Gottcheiner wrote: > AG : what if partner alerted 1S and they didn't ask ? I would have to > alert, Is there really any jurisdiction that makes alertability contingent on whether opponents ask about a previous alert? I realize thing are done differently outside the ACBL, but this seems especially strange to me. From richard.hills at immi.gov.au Thu Jan 6 05:22:20 2011 From: richard.hills at immi.gov.au (richard.hills at immi.gov.au) Date: Thu, 6 Jan 2011 15:22:20 +1100 Subject: [BLML] Herman [SEC=UNOFFICIAL] In-Reply-To: Message-ID: David Burn, 13th February 2008: [snip] >I think that Herman has been unresponsive to the arguments >advanced, especially by those in positions of authority, [snip] >I have said many times, that he is wrong to rely on the >"principle" that creation of UI is more to be avoided than >creation of MI; and that he is wrong to say that the Laws are >somehow "hierarchically" based on that principle. > >But if the anti-dWS amendment, or appendix, or whatever, >actually makes its way into the Laws, then Herman will have >done the game a great service by recognising that there was a >difficulty in the first place. [snip] Jeff Easterson: [snip] >>I'd like to defend his right to express his opinions. No >>one is forced to respond to his postings, or even read >>them. You have the option of simply ignoring him if you >>think he is introducing a red herring. Richard Hills: Neither David Burn nor myself dispute Herman De Wael's right to hold an opinion that the WBF Laws Committee should reverse its interpretation in favour of the obviously better De Wael worldview. Jeff Easterson: [snip] >>And, he seems to draw a line between his postings on blml >>and his work as a TD. (At least I hope he does.) Richard Hills: Neither David Burn nor myself dispute Herman De Wael's integrity and sincerity as a Director. The flashing neon red herring is this -> 2007 Introduction, third sentence, slightly modified: "Herman De Wael should be ready to accept gracefully any official interpretation awarded by the WBF Laws Committee." When Herman De Wael has doffed his Director's hat to wear a player's hat Herman-as-a-player intentionally ignores the WBF Laws Committee's official interpretation of Law 20. Then Herman-as-a-player compounds his very serious "must not" infraction by advising many other gullible Belgian players to likewise intentionally lie about their mutual pre-existing partnership understandings. So the flashing neon red herring is whether a player who might prima facie receive a Law 40C3(b) Disciplinary Penalty might prima facie be deemed by the WBF Credentials Committee to longer match the "fit and proper person" criterion for being a WBF Director. Of course, a happy resolution for all concerned would be for Herman-as-a-player to admit that his decade-long Quixotic crusade now lacks any skerrick of Lawful justification, so therefore switch his at-the-table mode to the "masochism" mandated by the WBF Laws Committee (unless and until the WBF Laws Committee should reverse its interpretation in favour of the obviously better De Wael worldview). Best wishes Richard Hills -------------------------------------------------------------------- Important Notice: If you have received this email by mistake, please advise the sender and delete the message and attachments immediately. This email, including attachments, may contain confidential, sensitive, legally privileged and/or copyright information. Any review, retransmission, dissemination or other use of this information by persons or entities other than the intended recipient is prohibited. DIAC respects your privacy and has obligations under the Privacy Act 1988. The official departmental privacy policy can be viewed on the department's website at www.immi.gov.au. See: http://www.immi.gov.au/functional/privacy.htm --------------------------------------------------------------------- From richard.hills at immi.gov.au Thu Jan 6 05:40:16 2011 From: richard.hills at immi.gov.au (richard.hills at immi.gov.au) Date: Thu, 6 Jan 2011 15:40:16 +1100 Subject: [BLML] interesting dWS case [SEC=UNOFFICIAL] In-Reply-To: <4D253540.4040006@nhcc.net> Message-ID: Book of Daniel 6:8 Now, O king, establish the decree, and sign the writing, that it be not changed, according to the law of the Medes and Persians, which altereth not. Alain Gottcheiner: >>AG : what if partner alerted 1S and they didn't ask ? I would >>have to alert, >>..... Steve Willner: >Is there really any jurisdiction that makes alertability >contingent on whether opponents ask about a previous alert? I >realize thing are done differently outside the ACBL, but this >seems especially strange to me. Richard Hills: The jurisdiction is that of the De Wael School, according to the law of the Alain and Herman, which altereth not. In that strange land only expected alerts are made. Unexpected alerts are avoided, thus gaining a non-masochistic advantage as partner cannot be constrained by the coils of Law 75A. Best wishes Richard Hills Recruitment Section Specialist Recruitment Team Level 5 Aqua, workstation W569, 6223 8453 DIAC Social Club movie tickets -------------------------------------------------------------------- Important Notice: If you have received this email by mistake, please advise the sender and delete the message and attachments immediately. This email, including attachments, may contain confidential, sensitive, legally privileged and/or copyright information. Any review, retransmission, dissemination or other use of this information by persons or entities other than the intended recipient is prohibited. DIAC respects your privacy and has obligations under the Privacy Act 1988. The official departmental privacy policy can be viewed on the department's website at www.immi.gov.au. See: http://www.immi.gov.au/functional/privacy.htm --------------------------------------------------------------------- From Hermandw at skynet.be Thu Jan 6 09:00:24 2011 From: Hermandw at skynet.be (Herman De Wael) Date: Thu, 06 Jan 2011 09:00:24 +0100 Subject: [BLML] Alain's case revisited Message-ID: <4D257698.6010201@skynet.be> Alain told you his case. He did not alert 2S, and did not give UI to Patrick. Patrick then realized his mistake, and passed 3Di, landing them in a sensible contract. They score +110 (I presume). What would have happened if Alain had followed the Beijing interpretation? So, he alerts. Presumably the opponents ask, but even if they don't, Patrick has UI. Patrick again realizes he was wrong, and he's in problems, because he has (at least partially) learnt this from UI. He tries to find a non-suggested alternative. If he doesn't succeed, in the eyes of the TD (which can easily be me, since we have no differences of opinion here), the TD will immpose that alternative on him. it seems reasonable to assume that the contract will be 4Sp. The opponents have AI telling them of the misunderstanding, so this is doubled. It goes three off. -800. (and this was a Butler tournament as well!). Now among all the talk in this thread, I have heard about PPs, but no-one has suggested that I award -800 to Alain after his infraction. What is a player to do from now on? Follow the Beijing interpretation and write down -800, or hide behind not knowing this interpretation and write +110? Remember that even the PP was suggested only on Alain, because he should have known better! What good is an interpretation if it isn't backed up with sensible policing? -- Herman De Wael Wilrijk Antwerpen Belgium From blml at arcor.de Thu Jan 6 09:24:41 2011 From: blml at arcor.de (Thomas Dehn) Date: Thu, 6 Jan 2011 09:24:41 +0100 (CET) Subject: [BLML] Alain's case revisited In-Reply-To: <4D257698.6010201@skynet.be> References: <4D257698.6010201@skynet.be> Message-ID: <9647244.332577.1294302281035.JavaMail.ngmail@webmail17.arcor-online.net> Herman De Wael wrote: > Alain told you his case. He did not alert 2S, and did not give UI to > Patrick. Patrick then realized his mistake, and passed 3Di, landing them > in a sensible contract. They score +110 (I presume). > > What would have happened if Alain had followed the Beijing > interpretation? So, he alerts. Presumably the opponents ask, but even if > they don't, Patrick has UI. Patrick again realizes he was wrong, and > he's in problems, because he has (at least partially) learnt this from > UI. He tries to find a non-suggested alternative. If he doesn't succeed, > in the eyes of the TD (which can easily be me, since we have no > differences of opinion here), the TD will immpose that alternative on > him. it seems reasonable to assume that the contract will be 4Sp. The > opponents have AI telling them of the misunderstanding, so this is > doubled. It goes three off. -800. (and this was a Butler tournament as > well!). -1600, not -800. Surely with his GF hand opposite Patrick's "reverse" Alain would redouble. If you consider 4SX down 3 a "likely" score had the irregularity not occured, then 4SXX down 3 is "likely", too. > Now among all the talk in this thread, I have heard about PPs, but > no-one has suggested that I award -800 to Alain after his infraction. > > What is a player to do from now on? Follow the Beijing interpretation > and write down -800, or hide behind not knowing this interpretation and > write +110? Remember that even the PP was suggested only on Alain, > because he should have known better! I have done that in the past. Here an example where we went down -4000 in 4C redoubled in such an UI situation. Me LHO Partner RHO 1H 2D 4C(1) p p p XX(2) All pass (1) Partner had wanted to try out a mix of fit showing jumps and splinter. We agreed that 4C was a fit showing jump, and that system would be on after interference. So I alerted 4C as a fit showing jump, but partner had forgotten that system is on after intervention. (2) In the splinter version of the system, this shows a C void. In the fit showing jump version of the system, it is natural. Comments of other players on such auctions were along the lines of "you are taking being ethical too far. You are punishing yourself and your team mates. He should not redouble 4C, that is silly". Overall, I had the impression that many other players were taking their obligations less seriously. And the TDs were not likely to assign those horror scores. It was frustrating. In the end, this was one out of several reasons I pretty much stopped playing bridge other than when played with screens. Thomas From svenpran at online.no Thu Jan 6 09:40:49 2011 From: svenpran at online.no (Sven Pran) Date: Thu, 6 Jan 2011 09:40:49 +0100 Subject: [BLML] ABF National Authority 4th March 2010 [SEC=UNOFFICIAL] In-Reply-To: References: <764142.92719.qm@web28501.mail.ukl.yahoo.com> Message-ID: <000601cbad7d$6bb851b0$4328f510$@no> On Behalf Of richard.hills at immi.gov.au > Nigel Guthrie: > > >>One side thought the result was 2H-1. The other 2H+1. But if none of > >>the players could remember what happened, who came up with the 9 trick > >>line? > > Richard Hills: > > The Director, after discussion with both sides, pursuant to the Director's powers > under Law 79B2. > > Nigel Guthrie: > > >>and why? > > Richard Hills: > > According to the Convenor, Sean Mullamphy, the Director's reason for "why?" was > due to the hand record indicating nine easy tricks on any normal line of play. > Thus under the Law > 85A1 "balance of probabilities" it was deemed that the most likely reason for the > anomaly was one player erring by pressing 7 instead of 9 on the electronic scorer, > and an opposing player erring by then pressing the "accept" button. In my opinion the side claiming 2H-1 (as I understand was registered) should be given the opportunity to show a "normal" line that would give this result, and the registered result should only be changed if they were unable to show such line. Did the director's procedure include this? I had a similar case some time ago; the two sides could not afterwards agree on what had really happened and my initial impression was that the registered result must have been incorrect. However, when asking the declaring side (who "defended" the registered result) how play had gone the declarer stated a line that although definitely not optimal for the defending side was certainly still "normal" within the definition of "Normal" for Laws 70 and 71. Consequently no correction was made to the registered result. (And I expect the players will be more observant when registering scores in the future). From Hermandw at skynet.be Thu Jan 6 09:51:03 2011 From: Hermandw at skynet.be (Herman De Wael) Date: Thu, 06 Jan 2011 09:51:03 +0100 Subject: [BLML] Alain's case revisited In-Reply-To: <9647244.332577.1294302281035.JavaMail.ngmail@webmail17.arcor-online.net> References: <4D257698.6010201@skynet.be> <9647244.332577.1294302281035.JavaMail.ngmail@webmail17.arcor-online.net> Message-ID: <4D258277.5030000@skynet.be> Thomas Dehn wrote: >> >> What is a player to do from now on? Follow the Beijing interpretation >> and write down -800, or hide behind not knowing this interpretation and >> write +110? Remember that even the PP was suggested only on Alain, >> because he should have known better! > > I have done that in the past. Here an example where > we went down -4000 in 4C redoubled in such > an UI situation. > > Me LHO Partner RHO > 1H 2D 4C(1) p I presume there's a line missing here - who doubled? > p p XX(2) All pass > > (1) Partner had wanted to try out a mix of fit showing jumps > and splinter. We agreed that 4C was a fit showing jump, > and that system would be on after interference. > So I alerted 4C as a fit showing jump, but partner > had forgotten that system is on after intervention. > (2) In the splinter version of the system, this shows a C void. > In the fit showing jump version of the system, it is natural. > > Comments of other players on such auctions were along > the lines of "you are taking being ethical too far. You > are punishing yourself and your team mates. He should > not redouble 4C, that is silly". > And indeed that is a sentiment that is often heard. But you have misunderstood the question. The problem is not about passing a redouble. Patrick and Alain are very ethical players, who know their responsabilities regarding UI. I have no question that, in the presence of UI, Patrick would have gone for -800. The problem is what Alain should do. Give the UI, and land in -800, or NOT give the UI, and land in +110? The problem here is not about not using the UI, but about not giving it. The Beijing interpretation obliges a player to give UI, but does not describe how the TD should react if the player refuses to do so! > Overall, I had the impression that many other > players were taking their obligations less seriously. > And the TDs were not likely to assign > those horror scores. > > It was frustrating. In the end, this was one out > of several reasons I pretty much stopped > playing bridge other than when played with screens. > > > Thomas > -- Herman De Wael Wilrijk Antwerpen Belgium From harald.skjaran at gmail.com Thu Jan 6 09:56:03 2011 From: harald.skjaran at gmail.com (=?UTF-8?Q?Harald_Skj=C3=A6ran?=) Date: Thu, 6 Jan 2011 09:56:03 +0100 Subject: [BLML] Precision's up to date in Kansas City [SEC=UNOFFICIAL] In-Reply-To: References: Message-ID: 2011/1/6 : > > Rodgers and Hammerstein, Oklahoma! (1943): > > Precision's up to date in Kansas City > They gone about as fer as they can go > They went an' built some asking bids seven stories high > About as high as an auction' orta grow. > > Matchpoint pairs > Board 5 > Dlr: North > Vul: North-South > > The bidding has gone: > > WEST ? ? ?NORTH ? ? EAST ? ? ?SOUTH > --- ? ? ? 1C ?(1) ? Pass ? ? ?1H ?(2) > Pass ? ? ?2H ?(3) ? Pass ? ? ?3D ?(4) > Pass ? ? ?4C ?(5) ? Pass ? ? ?4H ?(6) > Pass ? ? ?Pass(7) ? Pass > > (1) Alerted; Precision > (2) Alerted; 5+ hearts, game forcing > (3) Alerted; trump asking > (4) Alerted; 6+ hearts, one honour > (5) Alerted; asks about club control > (6) Alerted; third-round club control > (7) 100% forcing pass, as can easily be deduced from the auction > > You, West, hold: > > Q932 > 5 > J9875 > 642 > > What opening lead do you make? The NS bidding indicates a club lead, eventhough partner "failed" to double 4C to ask for a club lead. Partner presumably can stand other leads, else he should have doubled. I'll lead the systemic spade. For me it's the trey. Second choice: Low club. > > Best wishes > > Richard Hills > Recruitment Section > Specialist Recruitment Team > Level 5 Aqua, workstation W569, 6223 8453 > DIAC Social Club movie tickets > > > > > -------------------------------------------------------------------- > Important Notice: If you have received this email by mistake, please advise > the sender and delete the message and attachments immediately. ?This email, > including attachments, may contain confidential, sensitive, legally privileged > and/or copyright information. ?Any review, retransmission, dissemination > or other use of this information by persons or entities other than the > intended recipient is prohibited. ?DIAC respects your privacy and has > obligations under the Privacy Act 1988. ?The official departmental privacy > policy can be viewed on the department's website at www.immi.gov.au. ?See: > http://www.immi.gov.au/functional/privacy.htm > > > --------------------------------------------------------------------- > > _______________________________________________ > Blml mailing list > Blml at rtflb.org > http://lists.rtflb.org/mailman/listinfo/blml > -- Kind regards, Harald Skj?ran From nigelguthrie at yahoo.co.uk Thu Jan 6 10:30:27 2011 From: nigelguthrie at yahoo.co.uk (Nigel Guthrie) Date: Thu, 6 Jan 2011 09:30:27 +0000 (GMT) Subject: [BLML] uninteresting dWikiS case [SEC=UNOFFICIAL] In-Reply-To: References: Message-ID: <690962.52185.qm@web28504.mail.ukl.yahoo.com> [Richard Hills] In my opinion, if you break the rules of a game deliberately AND SECRETLY to gain an advantage, you are simply a cheat. In my opinion, if you break the rules of a game deliberately AND OPENLY to gain an advantage (e.g. by bragging about your non- masochistic methods on blml), you are simply a simpleton. [Nigel] Herman De Wael breaks the rules to demonstrate that his rules work better (any advantage gained is trivial and incidental). Nevertheless, he gives a bad example to others with less pure motives. From nigelguthrie at yahoo.co.uk Thu Jan 6 10:41:31 2011 From: nigelguthrie at yahoo.co.uk (Nigel Guthrie) Date: Thu, 6 Jan 2011 09:41:31 +0000 (GMT) Subject: [BLML] Precision's up to date in Kansas City [SEC=UNOFFICIAL] In-Reply-To: References: Message-ID: <422178.22508.qm@web28507.mail.ukl.yahoo.com> {Richard Hills] [Rodgers and Hammerstein, Oklahoma! (1943)] Precision's up to date in Kansas City They gone about as fer as they can go They went an' built some asking bids seven stories high About as high as an auction' orta grow. Matchpoint pairs Board 5 Dlr: North Vul: North-South The bidding has gone: WEST NORTH EAST SOUTH --- 1C (1) Pass 1H (2) Pass 2H (3) Pass 3D (4) Pass 4C (5) Pass 4H (6) Pass Pass(7) Pass (1) Alerted; Precision (2) Alerted; 5+ hearts, game forcing (3) Alerted; trump asking (4) Alerted; 6+ hearts, one honour (5) Alerted; asks about club control (6) Alerted; third-round club control (7) 100% forcing pass, as can easily be deduced from the auction You, West, hold: Q932 5 J9875 642 What opening lead do you make? {Nigel] Whether or not North's pass is forcing, it is tempting to bid 4S. Anyway Spade = 10, Diamond = 9, Club = 8 The Dog opposite did not bark in the night. Declarer seems to have club *length*. So a club lead seems rather speculative. From richard.hills at immi.gov.au Fri Jan 7 03:56:47 2011 From: richard.hills at immi.gov.au (richard.hills at immi.gov.au) Date: Fri, 7 Jan 2011 13:56:47 +1100 Subject: [BLML] ABF National Authority 4th March 2010 [SEC=UNOFFICIAL] In-Reply-To: <000601cbad7d$6bb851b0$4328f510$@no> Message-ID: Richard Hills: >> ... nine easy tricks on any normal line of play. ... Sven Pran: >In my opinion the side claiming 2H-1 (as I understand was >registered) should be given the opportunity to show a "normal" >line that would give this result, and the registered result >should only be changed if they were unable to show such line. Richard Hills: Sorry, my earlier statement was ambiguous. I should have written "nine easy tricks on ALL normal lines of play". Sven Pran: >Did the director's procedure include this? Richard Hills: Yes. ABF National Authority, paragraph b: b. Subsequently declarer claimed that the correct result was 2H making 9 tricks (E/W +140). The matter was brought to the attention of a director and who discussed the matter with a player from both teams. Neither player could state conclusively how the hand was played. ***The player who claimed that 2H had gone down agreed that if the line suggested had been used, the contract appeared unbeatable.*** Best wishes Richard Hills Recruitment Section Specialist Recruitment Team Level 5 Aqua, workstation W569, 6223 8453 DIAC Social Club movie tickets -------------------------------------------------------------------- Important Notice: If you have received this email by mistake, please advise the sender and delete the message and attachments immediately. This email, including attachments, may contain confidential, sensitive, legally privileged and/or copyright information. Any review, retransmission, dissemination or other use of this information by persons or entities other than the intended recipient is prohibited. DIAC respects your privacy and has obligations under the Privacy Act 1988. The official departmental privacy policy can be viewed on the department's website at www.immi.gov.au. See: http://www.immi.gov.au/functional/privacy.htm --------------------------------------------------------------------- From richard.hills at immi.gov.au Fri Jan 7 04:43:37 2011 From: richard.hills at immi.gov.au (richard.hills at immi.gov.au) Date: Fri, 7 Jan 2011 14:43:37 +1100 Subject: [BLML] Alain's case revisited [SEC=UNOFFICIAL] In-Reply-To: <4D258277.5030000@skynet.be> Message-ID: Herman De Wael: [snip] >Patrick and Alain are very ethical players, who know their >responsibilities regarding UI. I have no question that, in >the presence of UI, Patrick would have gone for -800. > >The problem is what Alain should do. Give the UI, and land >in -800, or NOT give the UI, and land in +110? [snip] Richard Hills: What's the problem? See the first sentence of Law 12B1: "The objective of score adjustment is to redress damage to a non-offending side and to take away any advantage gained by an offending side through its infraction." If one ethical partner obeys Law 21B1(a), so executes an unexpected Alert, but the other ethical partner then unintentionally infracts Law 75A, the Director takes away any advantage by adjusting the offending side's score from +110 to -800. If one ethical partner unintentionally infracts Law 21B1(a), thus preventing the later application of Law 75A, the Director takes any advantage by adjusting the offending side's score from +110 to -800. Best wishes Richard Hills Recruitment Section Specialist Recruitment Team Level 5 Aqua, workstation W569, 6223 8453 DIAC Social Club movie tickets -------------------------------------------------------------------- Important Notice: If you have received this email by mistake, please advise the sender and delete the message and attachments immediately. This email, including attachments, may contain confidential, sensitive, legally privileged and/or copyright information. Any review, retransmission, dissemination or other use of this information by persons or entities other than the intended recipient is prohibited. DIAC respects your privacy and has obligations under the Privacy Act 1988. The official departmental privacy policy can be viewed on the department's website at www.immi.gov.au. See: http://www.immi.gov.au/functional/privacy.htm --------------------------------------------------------------------- From ardelm at optusnet.com.au Fri Jan 7 06:39:14 2011 From: ardelm at optusnet.com.au (Tony Musgrove) Date: Fri, 07 Jan 2011 16:39:14 +1100 Subject: [BLML] ABF National Authority 4th March 2010 [SEC=UNOFFICIAL] In-Reply-To: References: <000601cbad7d$6bb851b0$4328f510$@no> Message-ID: <201101070539.p075dd1s027608@mail03.syd.optusnet.com.au> At 01:56 PM 7/01/2011, you wrote: >Richard Hills: > > >> ... nine easy tricks on any normal line of play. ... > >Sven Pran: > > >In my opinion the side claiming 2H-1 (as I understand was > >registered) should be given the opportunity to show a "normal" > >line that would give this result, and the registered result > >should only be changed if they were unable to show such line. > >Richard Hills: > >Sorry, my earlier statement was ambiguous. I should have >written "nine easy tricks on ALL normal lines of play". > >Sven Pran: > > >Did the director's procedure include this? > >Richard Hills: > >Yes. > >ABF National Authority, paragraph b: > >b. Subsequently declarer claimed that the correct result was >2H making 9 tricks (E/W +140). The matter was brought to the >attention of a director and who discussed the matter with a >player from both teams. Neither player could state >conclusively how the hand was played. ***The player who >claimed that 2H had gone down agreed that if the line >suggested had been used, the contract appeared unbeatable.*** It pains me to hear this sort of carry on in Oz bridge. Sounds more like US to me. Shoot them both I say, Tony (Sydney) From richard.hills at immi.gov.au Fri Jan 7 06:41:30 2011 From: richard.hills at immi.gov.au (richard.hills at immi.gov.au) Date: Fri, 7 Jan 2011 16:41:30 +1100 Subject: [BLML] Precision's up to date in Kansas City [SEC=UNOFFICIAL] In-Reply-To: Message-ID: >>Matchpoint pairs >>Board 5 >>Dlr: North >>Vul: North-South >> >>The bidding has gone: >> >>WEST ? ? ?NORTH ? ? EAST ? ? ?SOUTH >>--- ? ? ? 1C ?(1) ? Pass ? ? ?1H ?(2) >>Pass ? ? ?2H ?(3) ? Pass ? ? ?3D ?(4) >>Pass ? ? ?4C ?(5) ? Pass ? ? ?4H ?(6) >>Pass ? ? ?Pass(7) ? Pass >> >>(1) Alerted; Precision >>(2) Alerted; 5+ hearts, game forcing >>(3) Alerted; trump asking >>(4) Alerted; 6+ hearts, one honour >>(5) Alerted; asks about club control >>(6) Alerted; third-round club control >>(7) 100% forcing pass, as can easily be deduced from the auction >> >>You, West, hold: >> >>Q932 >>5 >>J9875 >>642 >> >>What opening lead do you make? Harald Skj?ran >The NS bidding indicates a club lead, even though partner >"failed" to double 4C to ask for a club lead. >Partner presumably can stand other leads, else he should have >doubled. > >I'll lead the systemic spade. Richard Hills: Partner should have doubled??? The auction is entirely consistent with pard holding AK doubleton in clubs and no other honours, in which case East-West defending a hypothetical 4Cx for -910 might just barely possibly be an inadequate score. And the auction is entirely consistent (due to North's apparent slam try club ask) with a score of -680 or -650, depending upon whether a club is led at trick one. Of course, the opening lead decision is for a trivial one imp at real bridge, but in this problem the unreal bridge of matchpoint pairs applies. Best wishes Richard Hills Recruitment Section Specialist Recruitment Team Level 5 Aqua, workstation W569, 6223 8453 DIAC Social Club movie tickets -------------------------------------------------------------------- Important Notice: If you have received this email by mistake, please advise the sender and delete the message and attachments immediately. This email, including attachments, may contain confidential, sensitive, legally privileged and/or copyright information. Any review, retransmission, dissemination or other use of this information by persons or entities other than the intended recipient is prohibited. DIAC respects your privacy and has obligations under the Privacy Act 1988. The official departmental privacy policy can be viewed on the department's website at www.immi.gov.au. See: http://www.immi.gov.au/functional/privacy.htm --------------------------------------------------------------------- From richard.hills at immi.gov.au Fri Jan 7 07:29:50 2011 From: richard.hills at immi.gov.au (richard.hills at immi.gov.au) Date: Fri, 7 Jan 2011 17:29:50 +1100 Subject: [BLML] ABF National Authority 4th March 2010 [SEC=UNOFFICIAL] In-Reply-To: <201101070539.p075dd1s027608@mail03.syd.optusnet.com.au> Message-ID: Tony Musgrove: It pains me to hear this sort of carry on in Oz bridge. Sounds more like US to me. Shoot them both I say, Tony (Sydney) Law 81C2: The Director (not the players) has the responsibility for rectifying irregularities and redressing damage. The Director's duties and powers normally include also the following: ..... 2. to administer and interpret these Laws and to advise the players of their rights and responsibilities thereunder. Grattan Endicott, 6th March 2009: +=+ In duplicate bridge, however, the flaw is patched to a degree by Law 81C2. In one's early bridge life, in particular, every encounter with a wise tournament director who explains matters is a contribution to a future grasp of the essentials. ~ Grattan ~ +=+ Best wishes Richard Hills Recruitment Section Specialist Recruitment Team Level 5 Aqua, workstation W569, 6223 8453 DIAC Social Club movie tickets -------------------------------------------------------------------- Important Notice: If you have received this email by mistake, please advise the sender and delete the message and attachments immediately. This email, including attachments, may contain confidential, sensitive, legally privileged and/or copyright information. Any review, retransmission, dissemination or other use of this information by persons or entities other than the intended recipient is prohibited. DIAC respects your privacy and has obligations under the Privacy Act 1988. The official departmental privacy policy can be viewed on the department's website at www.immi.gov.au. See: http://www.immi.gov.au/functional/privacy.htm --------------------------------------------------------------------- From Hermandw at skynet.be Fri Jan 7 09:12:01 2011 From: Hermandw at skynet.be (Herman De Wael) Date: Fri, 07 Jan 2011 09:12:01 +0100 Subject: [BLML] Alain's case revisited [SEC=UNOFFICIAL] In-Reply-To: References: Message-ID: <4D26CAD1.3040305@skynet.be> richard.hills at immi.gov.au wrote: > > If one ethical partner unintentionally infracts Law 21B1(a), > thus preventing the later application of Law 75A, the > Director takes any advantage by adjusting the offending > side's score from +110 to -800. > Richard, do me a favour. Please write the sentence explaining to Alain and Patrick your ruling in giving them -800, including that you are ruling on the use of UI which has never been given at the table. Just write the sentence and then think to yourself "do I really want to ask TDs to make such a ruling?". > Best wishes > > Richard Hills > Recruitment Section > Specialist Recruitment Team > Level 5 Aqua, workstation W569, 6223 8453 > DIAC Social Club movie tickets > > > > -------------------------------------------------------------------- > Important Notice: If you have received this email by mistake, please advise > the sender and delete the message and attachments immediately. This email, > including attachments, may contain confidential, sensitive, legally privileged > and/or copyright information. Any review, retransmission, dissemination > or other use of this information by persons or entities other than the > intended recipient is prohibited. DIAC respects your privacy and has > obligations under the Privacy Act 1988. The official departmental privacy > policy can be viewed on the department's website at www.immi.gov.au. See: > http://www.immi.gov.au/functional/privacy.htm > > > --------------------------------------------------------------------- > > _______________________________________________ > Blml mailing list > Blml at rtflb.org > http://lists.rtflb.org/mailman/listinfo/blml > > > > > No virus found in this incoming message. > Checked by AVG - www.avg.com > Version: 9.0.872 / Virus Database: 271.1.1/3364 - Release Date: 01/06/11 20:34:00 > -- Herman De Wael Wilrijk Antwerpen Belgium From nigelguthrie at yahoo.co.uk Fri Jan 7 10:53:33 2011 From: nigelguthrie at yahoo.co.uk (Nigel Guthrie) Date: Fri, 7 Jan 2011 09:53:33 +0000 (GMT) Subject: [BLML] ABF National Authority 4th March 2010 [SEC=UNOFFICIAL] In-Reply-To: References: Message-ID: <954938.41905.qm@web28513.mail.ukl.yahoo.com> [Tony Musgrove] It pains me to hear this sort of carry on in Oz bridge. Sounds more like US to me. Shoot them both I say, {Nigel] The wisdom of Solomon! What that the actual ruling? From agot at ulb.ac.be Fri Jan 7 12:50:57 2011 From: agot at ulb.ac.be (Alain Gottcheiner) Date: Fri, 07 Jan 2011 12:50:57 +0100 Subject: [BLML] uninteresting dWikiS case [SEC=UNOFFICIAL] In-Reply-To: References: Message-ID: <4D26FE21.3000200@ulb.ac.be> Le 5/01/2011 0:18, richard.hills at immi.gov.au a ?crit : > > Richard Hills: > > The WBF has specifically ruled that the English version of > the Laws prevails over a translation if the two versions > have an inconsistency. Fair enough, but to be able to react in that way, one would need to understand that there is indeed something else in the English version, and the discrepancy in the traduction is a strong hint that it isn't that obvious. Whence I find it difficult to apply. [apart, of course, from the fact that translation might be impossible. How do you translate into Russian "the sea and the pure heavens composed a wonderful camaieu" ?] > Alain Gottcheiner: > >>>> Sorry, but that doesn't hold water. To explain what >>>> the bid would have meant in other contexts has never >>>> been par of one's alert-explain duty. > Richard Hills: > > Maybe not part, but definitely par. AG : everybody understood that I meant "part", so thanks for pointing that I was right. And of course most would agree that in the original discussion, "should one explain what the PH 2H overcall means when partner makes a non-PH overcall ?", the response in an obvious negative one, and that the problem about attitude isn't solved by your remark. > So as par for my generous attitude to opponents in the > exchange of information, I am meticulous in informing an > opposing declarer that our leads and signals are very > substantially different between our current defence of > declarer's notrump contract versus our possible future > defence against a suit contract. AG : this I think fair ; the main element here is the frequence of the possible ensuing change ; but pre-alerts should govern those "basic changing elements" information, and BTW tpeople will be much more aware of the possibility of such a difference than of the esoteric one mentioned above. NB : our CC mentioned "meaning of overcalls may vary by PH" in the "specials" section. Best regards Alain From blml at arcor.de Fri Jan 7 14:04:55 2011 From: blml at arcor.de (Thomas Dehn) Date: Fri, 7 Jan 2011 14:04:55 +0100 (CET) Subject: [BLML] ABF National Authority 4th March 2010 [SEC=UNOFFICIAL] In-Reply-To: References: Message-ID: <783196594.22429.1294405495431.JavaMail.ngmail@webmail07.arcor-online.net> richard.hills at immi.gov.au wrote: > Richard Hills: > > >> ... nine easy tricks on any normal line of play. ... > > Sven Pran: > > >In my opinion the side claiming 2H-1 (as I understand was > >registered) should be given the opportunity to show a "normal" > >line that would give this result, and the registered result > >should only be changed if they were unable to show such line. > > Richard Hills: > > Sorry, my earlier statement was ambiguous. I should have > written "nine easy tricks on ALL normal lines of play". > > Sven Pran: > > >Did the director's procedure include this? > > Richard Hills: > > Yes. > > ABF National Authority, paragraph b: > > b. Subsequently declarer claimed that the correct result was > 2H making 9 tricks (E/W +140). The matter was brought to the > attention of a director and who discussed the matter with a > player from both teams. Neither player could state > conclusively how the hand was played. ***The player who > claimed that 2H had gone down agreed that if the line > suggested had been used, the contract appeared unbeatable.*** Richard, that just states that there exists one line that makes at least 8 tricks. It does not state that this line is a normal line. It does not state that 9 (nine, not eight) tricks are made on ALL normal lines. Thomas From agot at ulb.ac.be Fri Jan 7 14:36:37 2011 From: agot at ulb.ac.be (Alain Gottcheiner) Date: Fri, 07 Jan 2011 14:36:37 +0100 Subject: [BLML] uninteresting dWikiS case [SEC=UNOFFICIAL] In-Reply-To: <901893.41253.qm@web28514.mail.ukl.yahoo.com> References: <4D241F92.5080900@skynet.be> <901893.41253.qm@web28514.mail.ukl.yahoo.com> Message-ID: <4D2716E5.4090606@ulb.ac.be> Le 5/01/2011 11:33, Nigel Guthrie a ?crit : > [Herman de Wael] > > The fact that Rome has spoken does not make it right. I can still argue > against Rome, can't I? It's not because you are infallible that you must be > right. > Pope Urban VIII condemned Galileo - did that make him right? > > [Nigel] > > Herman is not defending Civil rights or Astronomical truth. We are discussing > the rules of a *game*. Their main requirement is that players enjoy the > resulting game. To that end, the rules should be simple enough to be > consistently enforced and for players to understand them and to comply with > them. In that sense they need to be fair. > > > Game-rules define an artificial and arbitrary world, however. That ethos of that > world need not reflect real-world morality: Liar-dice rewards lying. Diplomacy > rewards duplicity. The winner of Hangman executes his opponent. AG : you have to make the difference between role actions and person actions. The winner of Hangman doesn't earn the right to kill one's opponent. Diplomacy allows you to lie to your opponents, but when revealing your orders (a time when you aren't role-playing anymore), you can't lie. And may I add : at Illuminati you may steal money from the bank, but not your neighbour's watch. Bridge is indeed a difficult case, as it puts so much emphasis on controlling the players' state of mind, which is so difficult. This mixes role actions and personal actions in a way that makes defining "fair", "honest" and other words more difficult that at other games. > > If you don't like the rules of a game, there is no need to break them. If you > break them deliberately to gain advantage, you are simply a cheat; you spoil the > enjoyment of the game for others. AG : of course, this has to be reconsidered when the rules actually spoil the enjoyment and a reversed rule would not - something bridge instances don't understand. Actually, I think that this is the biggest argument for dWS : in the rare cases where I have seen it in action, the opponents always seemed comfortable with what happened, and charmed that their opponents took steps to allow them a fair board. Just compare this with the "enjoyment" from UI rulings. Of course, if you use it at the wrong time, you'll just end up being called a cheat, and perhaps you'll be. Apparently this hasn't happened up to now. Best regards Alain From agot at ulb.ac.be Fri Jan 7 14:55:46 2011 From: agot at ulb.ac.be (Alain Gottcheiner) Date: Fri, 07 Jan 2011 14:55:46 +0100 Subject: [BLML] Alain's case revisited In-Reply-To: <4D257698.6010201@skynet.be> References: <4D257698.6010201@skynet.be> Message-ID: <4D271B62.6080509@ulb.ac.be> Le 6/01/2011 9:00, Herman De Wael a ?crit : > Alain told you his case. He did not alert 2S, and did not give UI to > Patrick. Patrick then realized his mistake, and passed 3Di, landing them > in a sensible contract. AG : by the way, it wasn't, 3NT was, but somehow the score was good for us. 3NT should have won. > They score +110 (I presume). > > What would have happened if Alain had followed the Beijing > interpretation? So, he alerts. Presumably the opponents ask, but even if > they don't, Patrick has UI. Patrick again realizes he was wrong, and > he's in problems, because he has (at least partially) learnt this from > UI. He tries to find a non-suggested alternative. If he doesn't succeed, > in the eyes of the TD (which can easily be me, since we have no > differences of opinion here), the TD will immpose that alternative on > him. it seems reasonable to assume that the contract will be 4Sp. The > opponents have AI telling them of the misunderstanding, so this is > doubled. It goes three off. -800. (and this was a Butler tournament as > well!). > > Now among all the talk in this thread, I have heard about PPs, but > no-one has suggested that I award -800 to Alain after his infraction. > > What is a player to do from now on? Follow the Beijing interpretation > and write down -800, or hide behind not knowing this interpretation and > write +110? Remember that even the PP was suggested only on Alain, > because he should have known better! AG : well, that doesn't constitue a good argument, given L72. What certainly does is the effect this 800 would have had on other competitiors' scores. From agot at ulb.ac.be Fri Jan 7 15:01:39 2011 From: agot at ulb.ac.be (Alain Gottcheiner) Date: Fri, 07 Jan 2011 15:01:39 +0100 Subject: [BLML] Alain's case revisited In-Reply-To: <9647244.332577.1294302281035.JavaMail.ngmail@webmail17.arcor-online.net> References: <4D257698.6010201@skynet.be> <9647244.332577.1294302281035.JavaMail.ngmail@webmail17.arcor-online.net> Message-ID: <4D271CC3.6030605@ulb.ac.be> Le 6/01/2011 9:24, Thomas Dehn a ?crit : > Herman De Wael wrote: >> Alain told you his case. He did not alert 2S, and did not give UI to >> Patrick. Patrick then realized his mistake, and passed 3Di, landing them >> in a sensible contract. They score +110 (I presume). >> >> What would have happened if Alain had followed the Beijing >> interpretation? So, he alerts. Presumably the opponents ask, but even if >> they don't, Patrick has UI. Patrick again realizes he was wrong, and >> he's in problems, because he has (at least partially) learnt this from >> UI. He tries to find a non-suggested alternative. If he doesn't succeed, >> in the eyes of the TD (which can easily be me, since we have no >> differences of opinion here), the TD will immpose that alternative on >> him. it seems reasonable to assume that the contract will be 4Sp. The >> opponents have AI telling them of the misunderstanding, so this is >> doubled. It goes three off. -800. (and this was a Butler tournament as >> well!). > -1600, not -800. Surely with his GF hand opposite > Patrick's "reverse" Alain would redouble. AG : well, I would never accept playing in 4S on a non-fit (partner can't have more than 4 spades and I don't own 4). I suppose the end result would be 4NT -1 or -2 After 1C-1S-2S-3D (long-suit try)-3H-3NT-4S-(X), sure 4NT is to play, for any reason. From agot at ulb.ac.be Fri Jan 7 15:04:59 2011 From: agot at ulb.ac.be (Alain Gottcheiner) Date: Fri, 07 Jan 2011 15:04:59 +0100 Subject: [BLML] Precision's up to date in Kansas City [SEC=UNOFFICIAL] In-Reply-To: References: Message-ID: <4D271D8B.6080506@ulb.ac.be> Le 6/01/2011 9:56, Harald Skj?ran a ?crit : > 2011/1/6: >> Rodgers and Hammerstein, Oklahoma! (1943): >> >> Precision's up to date in Kansas City >> They gone about as fer as they can go >> They went an' built some asking bids seven stories high >> About as high as an auction' orta grow. >> >> Matchpoint pairs >> Board 5 >> Dlr: North >> Vul: North-South >> >> The bidding has gone: >> >> WEST NORTH EAST SOUTH >> --- 1C (1) Pass 1H (2) >> Pass 2H (3) Pass 3D (4) >> Pass 4C (5) Pass 4H (6) >> Pass Pass(7) Pass >> >> (1) Alerted; Precision >> (2) Alerted; 5+ hearts, game forcing >> (3) Alerted; trump asking >> (4) Alerted; 6+ hearts, one honour >> (5) Alerted; asks about club control >> (6) Alerted; third-round club control >> (7) 100% forcing pass, as can easily be deduced from the auction AG : there seems to be missing something here. Was there a double ? And why should a pass be forcing, when one just discovered that one was lacking an important control ? From agot at ulb.ac.be Fri Jan 7 15:06:44 2011 From: agot at ulb.ac.be (Alain Gottcheiner) Date: Fri, 07 Jan 2011 15:06:44 +0100 Subject: [BLML] uninteresting dWikiS case [SEC=UNOFFICIAL] In-Reply-To: <690962.52185.qm@web28504.mail.ukl.yahoo.com> References: <690962.52185.qm@web28504.mail.ukl.yahoo.com> Message-ID: <4D271DF4.4090600@ulb.ac.be> Le 6/01/2011 10:30, Nigel Guthrie a ?crit : > [Richard Hills] > > In my opinion, if you break the rules of a game deliberately AND > SECRETLY to gain an advantage, you are simply a cheat. > > In my opinion, if you break the rules of a game deliberately AND > OPENLY to gain an advantage (e.g. by bragging about your non- > masochistic methods on blml), you are simply a simpleton. > AG : in my opinion, if you break the rules of a game deliberately, without profit in mind, and opponents say you did right, then there is a problem with the rules. From blml at arcor.de Fri Jan 7 17:13:25 2011 From: blml at arcor.de (Thomas Dehn) Date: Fri, 7 Jan 2011 17:13:25 +0100 (CET) Subject: [BLML] Alain's case revisited In-Reply-To: <4D271CC3.6030605@ulb.ac.be> References: <4D271CC3.6030605@ulb.ac.be> <4D257698.6010201@skynet.be> <9647244.332577.1294302281035.JavaMail.ngmail@webmail17.arcor-online.net> Message-ID: <1426441734.16537.1294416805974.JavaMail.ngmail@webmail14.arcor-online.net> Somehow Alain's mails give me trouble with the reply-to settings ... Alain Gottcheiner wrote: > Le 6/01/2011 9:24, Thomas Dehn a ?crit : > > Herman De Wael wrote: > >> Alain told you his case. He did not alert 2S, and did not give UI to > >> Patrick. Patrick then realized his mistake, and passed 3Di, landing them > >> in a sensible contract. They score +110 (I presume). > >> > >> What would have happened if Alain had followed the Beijing > >> interpretation? So, he alerts. Presumably the opponents ask, but even if > >> they don't, Patrick has UI. Patrick again realizes he was wrong, and > >> he's in problems, because he has (at least partially) learnt this from > >> UI. He tries to find a non-suggested alternative. If he doesn't succeed, > >> in the eyes of the TD (which can easily be me, since we have no > >> differences of opinion here), the TD will immpose that alternative on > >> him. it seems reasonable to assume that the contract will be 4Sp. The > >> opponents have AI telling them of the misunderstanding, so this is > >> doubled. It goes three off. -800. (and this was a Butler tournament as > >> well!). > > -1600, not -800. Surely with his GF hand opposite > > Patrick's "reverse" Alain would redouble. > AG : well, I would never accept playing in 4S on a non-fit (partner > can't have more than 4 spades and I don't own 4). > I suppose the end result would be 4NT -1 or -2 > After 1C-1S-2S-3D (long-suit try)-3H-3NT-4S-(X), sure 4NT is to play, > for any reason. Sorry, I am not familiar with the details of your system, but I understood the auction differently. Something like this, from your point of view, where you have to bend over backwards to avoid the UI of partner's failure to alert 1S: 1C 1S (T-Walsh) 2S (reverse) 3D (forcing) 3H - 4NT (natural) Surely from your partner's PoV, who assumes a S fit, this 4NT bid is not natural. I can easily construct some horror end contract from there. Thomas From jfusselman at gmail.com Fri Jan 7 20:39:55 2011 From: jfusselman at gmail.com (Jerry Fusselman) Date: Fri, 7 Jan 2011 13:39:55 -0600 Subject: [BLML] Alain's case revisited In-Reply-To: <9647244.332577.1294302281035.JavaMail.ngmail@webmail17.arcor-online.net> References: <4D257698.6010201@skynet.be> <9647244.332577.1294302281035.JavaMail.ngmail@webmail17.arcor-online.net> Message-ID: On Thu, Jan 6, 2011 at 2:24 AM, Thomas Dehn wrote: > Herman De Wael wrote: >> Now among all the talk in this thread, I have heard about PPs, but >> no-one has suggested that I award -800 to Alain after his infraction. >> >> What is a player to do from now on? Follow the Beijing interpretation >> and write down -800, or hide behind not knowing this interpretation and >> write +110? Remember that even the PP was suggested only on Alain, >> because he should have known better! > > I have done that in the past. Here an example where > we went down -4000 in 4C redoubled in such > an UI situation. > > Me ? LHO ? Partner ? RHO > 1H ? 2D ? ? ?4C(1) ? ? ?p > p ? ? ?p ? ? ? ?XX(2) ? ? ?All pass > > (1) Partner had wanted to try out a mix of fit showing jumps > ? ? ?and splinter. We agreed that 4C was a fit showing jump, > ? ? ?and that system would be on after interference. > ? ? ?So I alerted 4C as a fit showing jump, but partner > ? ? ?had forgotten that system is on after intervention. > (2) In the splinter version of the system, this shows a C void. > ? ? ?In the fit showing jump version of the system, it is natural. > > Comments of other players on such auctions were along > the lines of "you are taking being ethical too far. You > are punishing yourself and your team mates. He should > not redouble 4C, that is silly". > > Overall, I had the impression that many other > players were taking their obligations less seriously. > And the TDs were not likely to assign > those horror scores. > > It was frustrating. In the end, this was one out > of several reasons I pretty much stopped > playing bridge other than when played with screens. > I sympathize with Thomas's frustration at directors' frequent avoidance of assigning the score from the kind of horror contracts that are likely after UI involving artificial bids. Sometimes it seems to me that directors work out the initial ramifications of the UI well, but it is difficult for them to appreciate how likely horror contracts are. I sympathize with the directors too, for handling this kind of UI is not easy. But this consideration sounds a like a good argument against rules that force the creation of UI. It sounds to me like an argument against the Beijing interpretation and for making dWS permissible. The other advantage of allowing dWS is it avoids the requirement for mind reading of the kind that Hirsch recommended in this exchange with Herman: [Hirsch] The PP is because the player knew what he should be doing and did something else. [Herman] So now we move to the next table, and the same happens. Are you going to give the same PP at that table? When the player in question simplu tells you "I thought partner was right when he did not alert - I have misbid". [Hirsch] If I believe that there was no intent to infract a Law, 72.B.1 does not apply. Hirsch, as well as (I am almost sure) most of the posters on this board, believe that mind reading is necessary when assessing dWS explanations. Apparently mind reading is what the Beijing interpretation calls for. This requirement seems to me a backwards step in the laws. Among our best bridge laws are the ones that avoid requiring the director to read minds. For example, many of our best bridge laws contain phrases like "could have known" and "could have seen" instead of "did know" and "did see." The director does not have to read minds in these cases. And in a case where UI suggested 4S, the director does not ask "Would you have bid 4S had there been no UI?" and the read the player's mind to decide if he is truly sincere. Instead, we have a fine procedure in place where we construct LAs and determine what is demonstrably shown so that the state of the bidder's mind does not have to be ascertained. Admittedly, in a few situations, directors need to divine state of mind, but we should try to keep those cases to a minimum when writing and interpreting bridge laws. I can't tell what I dislike more about the Beijing interpretation: It's requirement to read minds or its high likelihood of ruining the board. I would prefer my opponents to use dWS when it helps to save the board. Jerry Fusselman From ehaa at starpower.net Fri Jan 7 23:11:30 2011 From: ehaa at starpower.net (Eric Landau) Date: Fri, 7 Jan 2011 17:11:30 -0500 Subject: [BLML] Alain's case revisited In-Reply-To: References: <4D257698.6010201@skynet.be> <9647244.332577.1294302281035.JavaMail.ngmail@webmail17.arcor-online.net> Message-ID: <0F0A4E0E-3319-4F02-8EC6-F49CA1492683@starpower.net> On Jan 7, 2011, at 2:39 PM, Jerry Fusselman wrote: > I sympathize with Thomas's frustration at directors' frequent > avoidance of assigning the score from the kind of horror contracts > that are likely after UI involving artificial bids. Sometimes it > seems to me that directors work out the initial ramifications of the > UI well, but it is difficult for them to appreciate how likely horror > contracts are. I sympathize with the directors too, for handling this > kind of UI is not easy. > > But this consideration sounds a like a good argument against rules > that force the creation of UI. It sounds to me like an argument > against the Beijing interpretation and for making dWS permissible. > > The other advantage of allowing dWS is it avoids the requirement for > mind reading of the kind that Hirsch recommended in this exchange with > Herman: > > [Hirsch] The PP is because the player knew what he should be doing > and did something else. > > [Herman] So now we move to the next table, and the same happens. Are > you going to give the same PP at that table? When the player in > question simplu tells you "I thought partner was right when he did not > alert - I have misbid". > > [Hirsch] If I believe that there was no intent to infract a Law, > 72.B.1 does not apply. > > Hirsch, as well as (I am almost sure) most of the posters on this > board, believe that mind reading is necessary when assessing dWS > explanations. Apparently mind reading is what the Beijing > interpretation calls for. > > This requirement seems to me a backwards step in the laws. Among our > best bridge laws are the ones that avoid requiring the director to > read minds. For example, many of our best bridge laws contain phrases > like "could have known" and "could have seen" instead of "did know" > and "did see." The director does not have to read minds in these > cases. And in a case where UI suggested 4S, the director does not ask > "Would you have bid 4S had there been no UI?" and the read the > player's mind to decide if he is truly sincere. Instead, we have a > fine procedure in place where we construct LAs and determine what is > demonstrably shown so that the state of the bidder's mind does not > have to be ascertained. > > Admittedly, in a few situations, directors need to divine state of > mind, but we should try to keep those cases to a minimum when writing > and interpreting bridge laws. > > I can't tell what I dislike more about the Beijing interpretation: > It's requirement to read minds or its high likelihood of ruining the > board. I would prefer my opponents to use dWS when it helps to save > the board. What some people call "reading minds", others call "good communication skills", and still others, bridge club directors especially, call "knowing their customers". Anyone not totally oblivious to others "reads minds" with a great deal better than random accuracy. What Jerry calls "our best bridge laws" are precisely those that require us to give identical rulings to those whom we know to be innocent newbies as to confirmed cheaters. And because our clubs (and our SOs) are businesses, that means being far too tolerant to the cheaters rather than driving the innocent infractors away (and having to listen to those in this forum who constantly carp about how our laws let people get away with things they shouldn't). Only the real-life lawyers among our players seem to actually understand that subtle bit about being punished for doing just what a cheater would do but, of course, we're not calling you cheaters, we'd never ever do that, no sirree, so do please come back next week. If we had genuinely psychic directors who actually could read players' minds, wouldn't we want to write our laws to take advantage of those skills, and expect our game to be better for it? If so, shouldn't we work to hone our communications skills and our knowledge of our customers, write laws that take advantage of those (admittedly somewhat less impressive, but not entirely useless) skills, and do our best to come as close as we can to our ideal of perfection? Eric Landau 1107 Dale Drive Silver Spring MD 20910 ehaa at starpower.net From jfusselman at gmail.com Sat Jan 8 23:00:55 2011 From: jfusselman at gmail.com (Jerry Fusselman) Date: Sat, 8 Jan 2011 16:00:55 -0600 Subject: [BLML] Alain's case revisited In-Reply-To: <0F0A4E0E-3319-4F02-8EC6-F49CA1492683@starpower.net> References: <4D257698.6010201@skynet.be> <9647244.332577.1294302281035.JavaMail.ngmail@webmail17.arcor-online.net> <0F0A4E0E-3319-4F02-8EC6-F49CA1492683@starpower.net> Message-ID: I want to give a clarification and request a clarification. In brief, I am ignoring clubs and penalty points right now, and I want to know how Eric (or someone who agrees with Eric's post) would rule if his innocent newbie unconsciously used the same UI to take the same improper action as a conscious cheat. On Fri, Jan 7, 2011 at 4:11 PM, Eric Landau wrote: > What some people call "reading minds", others call "good > communication skills", and still others, bridge club directors > especially, call "knowing their customers". Here, and later, Eric refers to bridge club directors. I would rather leave clubs entirely out of the discussion: For my post, I was thinking only of events run in (using ACBL names) sectionals, regionals, and nationals. Almost anything can happen in clubs, and frequently does. Lets face it: Clubs run by their own rules. I want to hear if what I said is right in the domain of sectionals, regionals, and nationals, for I consider clubs a separate issue. > What Jerry calls "our best bridge laws" are precisely those that > require us to give identical rulings to those whom we know to be > innocent newbies as to confirmed cheaters. Eric, are you thinking that the requirement you mention here is a bad thing? For example, please consider the noncompetitive auction 1S-2S 3C-...3S 4S. I.e., there was a slow 3S signoff after the 3C game try. It gets raised to 4S anyway by a hand that some players would have taken straight to game (instead of 3C), but no one would tried for a slam. Please also assume that the slow signoff demonstrably shows extras. In the play, it easily makes ten tricks, as it should in just about all cases, and both sides play it well. This happens at two tables---one with a confirmed cheater bidding 4S, and another with a confirmed innocent newbie bidding 4S. Do you want the rectifications in these two cases to be the same or different? (I am not asking about penalty points here, only the rectification.) If it helps to reach a decision, please assume that the confirmed cheater consciously used the delay to upgrade his hand, but the confirmed innocent newbie's upgrade caused by the pause was entirely unconscious. And please assume also that the director knows all of this perfectly. How should he rule in the two cases if you had the laws arranged the way you think best? Does my choice, +170/-170 in both cases, match yours? > ?And because our clubs > (and our SOs) are businesses, that means being far too tolerant to > the cheaters rather than driving the innocent infractors away (and > having to listen to those in this forum who constantly carp about how > our laws let people get away with things they shouldn't). ?Only the > real-life lawyers among our players seem to actually understand that > subtle bit about being punished for doing just what a cheater would > do but, of course, we're not calling you cheaters, we'd never ever do > that, no sirree, so do please come back next week. > > If we had genuinely psychic directors who actually could read > players' minds, wouldn't we want to write our laws to take advantage > of those skills, and expect our game to be better for it? ?If so, > shouldn't we work to hone our communications skills and our knowledge > of our customers, write laws that take advantage of those (admittedly > somewhat less impressive, but not entirely useless) skills, and do > our best to come as close as we can to our ideal of perfection? > I plan to respond to this later. I suspect that we have a conflict of visions about directors and players. Jerry Fusselman From nigelguthrie at yahoo.co.uk Sun Jan 9 18:49:44 2011 From: nigelguthrie at yahoo.co.uk (Nigel Guthrie) Date: Sun, 9 Jan 2011 17:49:44 +0000 (GMT) Subject: [BLML] Alain's case revisited In-Reply-To: References: <4D257698.6010201@skynet.be> <9647244.332577.1294302281035.JavaMail.ngmail@webmail17.arcor-online.net> <0F0A4E0E-3319-4F02-8EC6-F49CA1492683@starpower.net> Message-ID: <936302.47101.qm@web28514.mail.ukl.yahoo.com> I agree with Jerry Fusselmann, that *wherever possible*, the laws should avoid reliance on the director's mind-reading skills. I also agree with Eric Landau that some directors pride themselves on their mind-reading abilities. Such directors may welcome laws that allow them to hone their skills. There are problems, however. For example... 1. Honest Law-abiding players will admit to their motives, so they are unfairly targeted. Even a wizard telepath can't suss out all the rationalisers and prevaricators, so some get away with law-breaking. 2. Suppose you suspect that a director dislikes you. Exercising his mind-reading skills, he rules against you. Whatever the truth of the matter, If you believe you are innocent, you may feel aggrieved to be penalised on such nebulous evidence. 3. Even unbiased third-parties judge laws to be unfair, when seemingly identical cases give rise to contradictory rulings. Subjectivity can't be expunged from the laws but it can be minimised. IMO there is a long way to go in that direction. From richard.hills at immi.gov.au Sun Jan 9 21:33:17 2011 From: richard.hills at immi.gov.au (richard.hills at immi.gov.au) Date: Mon, 10 Jan 2011 07:33:17 +1100 Subject: [BLML] ABF National Authority 4th March 2010 [SEC=UNOFFICIAL] In-Reply-To: <783196594.22429.1294405495431.JavaMail.ngmail@webmail07.arcor-online.net> Message-ID: ABF National Authority, paragraph b: >>b. Subsequently declarer claimed that the correct result was >>2H making 9 tricks (E/W +140). The matter was brought to the >>attention of a director and who discussed the matter with a >>player from both teams. Neither player could state >>conclusively how the hand was played. ***The player who >>claimed that 2H had gone down agreed that if the line >>suggested had been used, the contract appeared unbeatable.*** Thomas Dehn: >Richard, that just states that there exists one line that >makes at least 8 tricks. > >It does not state that this line is a normal line. >It does not state that 9 (nine, not eight) tricks are made on >ALL normal lines. Richard Hills, earlier post, 6th January 2011: According to the Convenor, Sean Mullamphy, the Director's reason for "why?" was due to the hand record indicating nine easy tricks on any normal line [typo, should have written ALL normal lines] of play. ABF National Authority, paragraph c: c. The director involved in this discussion decided the match should be scored on the basis that the declarer made 9 tricks in 2H. Best wishes Richard Hills Recruitment Section Specialist Recruitment Team Level 5 Aqua, workstation W569, 6223 8453 DIAC Social Club movie tickets -------------------------------------------------------------------- Important Notice: If you have received this email by mistake, please advise the sender and delete the message and attachments immediately. This email, including attachments, may contain confidential, sensitive, legally privileged and/or copyright information. Any review, retransmission, dissemination or other use of this information by persons or entities other than the intended recipient is prohibited. DIAC respects your privacy and has obligations under the Privacy Act 1988. The official departmental privacy policy can be viewed on the department's website at www.immi.gov.au. See: http://www.immi.gov.au/functional/privacy.htm --------------------------------------------------------------------- From richard.hills at immi.gov.au Sun Jan 9 23:38:12 2011 From: richard.hills at immi.gov.au (richard.hills at immi.gov.au) Date: Mon, 10 Jan 2011 09:38:12 +1100 Subject: [BLML] Precision's up to date in Kansas City [SEC=UNOFFICIAL] In-Reply-To: <4D271D8B.6080506@ulb.ac.be> Message-ID: Rodgers and Hammerstein, Oklahoma! (1943): Precision's up to date in Kansas City They gone about as fer as they can go They went an' built some asking bids seven stories high About as high as an auction' orta grow. Matchpoint pairs Board 5 Dlr: North Vul: North-South The bidding has gone: WEST NORTH EAST SOUTH --- 1C (1) Pass 1H (2) Pass 2H (3) Pass 3D (4) Pass 4C (5) Pass 4H (6) Pass Pass(7) Pass (1) Alerted; Precision (2) Alerted; 5+ hearts, game forcing (3) Alerted; trump asking (4) Alerted; 6+ hearts, one honour (5) Alerted; asks about club control (6) Alerted; third-round club control (7) 100% forcing pass, as can easily be deduced from the auction Alain Gottcheiner, 8th January 2011: >>>AG : there seems to be missing something here. Was there a >>>double ? And why should a pass be forcing, when one just >>>discovered that one was lacking an important control ? Konrad Ciborowski, 8th September 2005: >>I have always ridiculed the use of smileys because I have >>always thought that people who need to a special way to mark >>the jokes in the text (one might almost say "to have jokes >>alerted") are the same ones who need the laughter from the >>background in the sitcoms. >>..... >>I think we got to the point where one might reproduce one of >>David Burn's wonderful contributions when someone made some >>obviously light-hearted statement and was severely reprimanded >>by David Stevenson. David Burn countered with: >> >>"These are known as jokes. Of course such frivolity has little >>place on as august mailing list as this one but I am sure our >>contributor will soon see the error of his ways under your >>helpful guidance." You, West, hold: Q932 5 J9875 642 What opening lead do you make? John (MadDog) Probst, 6th March 2009: >my systemic club. Richard Hills, 6th March 2009: Yes, in my opinion at matchpoint pairs a club lead is automatic on this auction. Why get an equal bottom for -680? The excessively descriptive auction gives us a chance that other pairs might not take, establishing partner's marked club winner(s) for an equal top of -650. But..... Kansas City NABC 2001 Appeals Casebook, Case 12 Open Pairs First Qualifying Session Brd: 5 Allen Rew Dlr: North K85 Vul: N/S QJ98 A2 AQ85 Paul O'Hara Roisin O'Hara Q932 AJT7 5 K J9875 K43 642 JT973 Thomas Mori 64 AT76432 QT6 K The Facts: 4H made six, +680 for N/S. The opening lead was a club. The Director was called at the conclusion of play. North gave the correct explanation of the 4H bid according to N/S's partnership agreement: The first step showed no control, the second step showed third-round control, etc. South said that when he bid 4H he thought he was showing second-round club control and only later realized that his partner's explanation was correct. The Director ruled that there had been a mistaken bid ([1997] Law 75D2 [now the 2007 Law 75C]) and allowed the table result to stand. The Appeal: E/W appealed the Director's ruling. E/W thought that N/S had an obligation to know their conventions. Since South had forgotten the correct response to the asking bid, E/W believed they were damaged in their selection of the opening lead: a spade might have been led had they been informed that South had misbid. N/S agreed with the Director's ruling and presented system notes which clearly confirmed that the 4H bid showed third-round club control. The Committee Decision: The Committee reviewed [1997] Laws 40A [now the 2007 Law 40A3] and [1997 Law] 75D2 [now the 2007 Law 75C] and could find no basis for an adjustment. The Committee recalled Edgar Kaplan's statement that if the laws permit an intentional deviation from partnership agreements, then certainly an unintentional one is not cause for an adjustment. In this case it was clear that there had been a misbid rather than a misexplanation. Therefore, there was no infraction. The Committee was informed that the Screening Director had reviewed [1997] Laws 75D2 [now the 2007 Law 75C] and [1997 Law] 40A [now the 2007 Law 40A3] with E/W. Since no issue was presented to the Committee beyond the Director's correct review of the laws, the Committee allowed the table result to stand and issued both E/W players an AWMW. DIC of Event: Henry Cukoff Committee: Michael Huston (chair), Mark Bartusek, Simon Kantor, Ellen Siebert, Adam Wildavsky Grattan Endicott (casebook panellist): "Oh, yes indeed. And not only Kaplan. I quote from the minutes of the WBF Laws Committee, Albuquerque, September 27, 1994: 'It was pointed out that one cannot devise a law which says deliberate infringement of partnership agreement is acceptable but accidental infringement is punishable.' There is no thought that the right to violate one's announced agreements, so long as there is no concealed understanding about it with partner, should be taken away. The meeting noted that players can be disciplined if their forgetting of agreements and consequent convention disruptions interfere with the orderly progress of the game and/or indicate the player's inattention to the game, or interfere with the enjoyment of the game. The action (then) taken can include prohibition of the use of the convention, etc.'" Best wishes Richard Hills Recruitment Section Specialist Recruitment Team Level 5 Aqua, workstation W569, 6223 8453 DIAC Social Club movie tickets -------------------------------------------------------------------- Important Notice: If you have received this email by mistake, please advise the sender and delete the message and attachments immediately. This email, including attachments, may contain confidential, sensitive, legally privileged and/or copyright information. Any review, retransmission, dissemination or other use of this information by persons or entities other than the intended recipient is prohibited. DIAC respects your privacy and has obligations under the Privacy Act 1988. The official departmental privacy policy can be viewed on the department's website at www.immi.gov.au. See: http://www.immi.gov.au/functional/privacy.htm --------------------------------------------------------------------- From richard.hills at immi.gov.au Mon Jan 10 00:47:27 2011 From: richard.hills at immi.gov.au (richard.hills at immi.gov.au) Date: Mon, 10 Jan 2011 10:47:27 +1100 Subject: [BLML] uninteresting dWikiS case [SEC=UNOFFICIAL] In-Reply-To: <4D271DF4.4090600@ulb.ac.be> Message-ID: >>In my opinion, if you break the rules of a game deliberately AND >>OPENLY to gain an advantage (e.g. by bragging about your non- >>masochistic methods on blml), then you are simply a simpleton. >AG : in my opinion, if you break the rules of a game deliberately, >without profit in mind, and opponents say you did right, then there >is a problem with the rules. Begging the question, petitio principii. Without profit in mind??? Herman De Wael has long advocated that his (now closed) loophole in the rules was designed with profit in mind, hence its superiority to the (now one and only) traditional interpretation of the Laws of Duplicate Bridge. In my opinion, if you break the rules of a game deliberately and openly TO GAIN AN ADVANTAGE (e.g. by infracting Law 21B1(a) to prevent constraint of partner's logical alternatives pursuant to Law 16B1(a)'s criterion of "unexpected alert"), then you are simply a simpleton. And if you and your opponents assert that "you did right" because there is a problem with the overly-complex Duplicate Bridge rules, then you and your opponents should abandon Duplicate Bridge for the simpletons' simple game of X|X|? ----- |?|X ----- ?| |X Of course, neither Herman De Wael nor Alain Gottcheiner are simpletons. This is because genius-level blmlers are much better at creating specious clutching-at-straws moving-the-goalposts begging-the- question (petitio principii) new arguments than simpletons could ever achieve. Best wishes Richard Hills Recruitment Section Specialist Recruitment Team Level 5 Aqua, workstation W569, 6223 8453 DIAC Social Club movie tickets -------------------------------------------------------------------- Important Notice: If you have received this email by mistake, please advise the sender and delete the message and attachments immediately. This email, including attachments, may contain confidential, sensitive, legally privileged and/or copyright information. Any review, retransmission, dissemination or other use of this information by persons or entities other than the intended recipient is prohibited. DIAC respects your privacy and has obligations under the Privacy Act 1988. The official departmental privacy policy can be viewed on the department's website at www.immi.gov.au. See: http://www.immi.gov.au/functional/privacy.htm --------------------------------------------------------------------- From grandaeval at tiscali.co.uk Mon Jan 10 01:23:16 2011 From: grandaeval at tiscali.co.uk (Grattan) Date: Mon, 10 Jan 2011 00:23:16 -0000 Subject: [BLML] Alain's case revisited References: <4D257698.6010201@skynet.be><9647244.332577.1294302281035.JavaMail.ngmail@webmail17.arcor-online.net><0F0A4E0E-3319-4F02-8EC6-F49CA1492683@starpower.net> Message-ID: <3C29C3CA12094D2390FF156A7DCFFC90@Mildred> Grattan Endicott To: "Bridge Laws Mailing List" Sent: Saturday, January 08, 2011 10:00 PM Subject: Re: [BLML] Alain's case revisited This happens at two tables---one with a confirmed cheater bidding 4S, and another with a confirmed innocent newbie bidding 4S. Do you want the rectifications in these two cases to be the same or different? (Iam not asking about penalty points here, only the rectification.) +=+ Could I enquire whether the two players have entered under the same Conditions of Contest? If so, should one be advantaged by comparison with the other against the field? ~ G ~ +=+ ~ Grattan ~ +=+ From jfusselman at gmail.com Mon Jan 10 02:17:12 2011 From: jfusselman at gmail.com (Jerry Fusselman) Date: Sun, 9 Jan 2011 19:17:12 -0600 Subject: [BLML] Alain's case revisited In-Reply-To: <3C29C3CA12094D2390FF156A7DCFFC90@Mildred> References: <4D257698.6010201@skynet.be> <9647244.332577.1294302281035.JavaMail.ngmail@webmail17.arcor-online.net> <0F0A4E0E-3319-4F02-8EC6-F49CA1492683@starpower.net> <3C29C3CA12094D2390FF156A7DCFFC90@Mildred> Message-ID: On Sun, Jan 9, 2011 at 6:23 PM, Grattan wrote: > > > Grattan Endicott **************************************************** > Skype directory: ?grattan.endicott > **************************************************** > "Sir, we are a nest of singing birds." > ? ? ? ? ? ? ? [Dr. Samuel Johnson, 1730] > ++++++++++++++++++++++++++++++++++ > ----- Original Message ----- > From: "Jerry Fusselman" > To: "Bridge Laws Mailing List" > Sent: Saturday, January 08, 2011 10:00 PM > Subject: Re: [BLML] Alain's case revisited > > > This happens at two tables---one with a confirmed > cheater bidding 4S, and another with a confirmed > innocent newbie bidding 4S. Do you want the > rectifications in these two cases to be the same or > different? ?(Iam not asking about penalty points here, > only the rectification.) > > +=+ Could I enquire whether the two players have > entered under the same Conditions of Contest? > ? ? ?If so, should one be advantaged by comparison > with the other against the field? > ? ? ? ? ? ? ? ? ? ? ? ? ? ? ? ? ? ? ? ? ? ? ?~ G ~ ? +=+ > ? ? ? ? ? ? ? ? ? ? ? ? ? ? ? ? ? ? ?~ Grattan ~ ? +=+ Thanks; sure! My answer is yes to the first question. My personal answer to the second question is no, but I have a feeling that the thrust of Eric's argument is ..., well perhaps we should wait until Eric responds. In his post, he considered the difference between an innocent newbie and a confirmed cheat to be of vital importance, so maybe it matters for my hypothetical case. Jerry Fusselman From richard.hills at immi.gov.au Mon Jan 10 02:33:11 2011 From: richard.hills at immi.gov.au (richard.hills at immi.gov.au) Date: Mon, 10 Jan 2011 12:33:11 +1100 Subject: [BLML] Alain's case revisited [SEC=UNOFFICIAL] In-Reply-To: <4D26CAD1.3040305@skynet.be> Message-ID: Herman De Wael >Richard, do me a favour. > >Please write the sentence explaining to Alain and Patrick your >ruling in giving them -800, including that you are ruling on the >use of UI which has never been given at the table. > >Just write the sentence and then think to yourself "do I really >want to ask TDs to make such a ruling?". Richard Hills: Herman, do me a favour. Consider this knockout teams grand final, in which the seats occupied by Alain and Patrick in the Open Room are occupied by the experts Niala and Kcirtap in the Closed Room. Niala and Kcirtap play the same methods as Alain and Patrick, and Kcirtap is identical in forgetfulness to Patrick. The only difference is that Niala invariably obeys Law 21B1(a), so on the board in question Niala and Kcirtap inevitably score -800. Should Alain and Patrick gain a match-winning 14 imps due to Alain's infraction and Herman's directing??? On a similar topic, the offending side partially retaining its gains from use-of-UI, the WBF Laws Committee 12th October 2010: [snip] "such adjustments may encourage players to infract the law by allowing them some proportion of their infractive result. For this reason such adjustments should in general be avoided." Best wishes Richard Hills Recruitment Section Specialist Recruitment Team Level 5 Aqua, workstation W569, 6223 8453 DIAC Social Club movie tickets -------------------------------------------------------------------- Important Notice: If you have received this email by mistake, please advise the sender and delete the message and attachments immediately. This email, including attachments, may contain confidential, sensitive, legally privileged and/or copyright information. Any review, retransmission, dissemination or other use of this information by persons or entities other than the intended recipient is prohibited. DIAC respects your privacy and has obligations under the Privacy Act 1988. The official departmental privacy policy can be viewed on the department's website at www.immi.gov.au. See: http://www.immi.gov.au/functional/privacy.htm --------------------------------------------------------------------- From richard.hills at immi.gov.au Mon Jan 10 06:12:44 2011 From: richard.hills at immi.gov.au (richard.hills at immi.gov.au) Date: Mon, 10 Jan 2011 16:12:44 +1100 Subject: [BLML] Precision's up to date in Kansas City [SEC=UNOFFICIAL] In-Reply-To: Message-ID: Konrad Ciborowski quoting David Burn: "These are known as jokes. Of course such frivolity has little place on an august mailing list as this one but I am sure our contributor will soon see the error of his ways under your helpful guidance." Gordon Bower verifying Grattan Endicott's use of "autochthonous": Indeed a real word, though not one you hear very often. Its antonym, "allochthonous," on the other hand, you will hear very frequently -- if you are a geologist in Alaska, and we have at least one of those on this mailing list. (It was news to me to discover, after the spelling bee, that the words also have currency in other fields.) Let's see: maybe we can use this to simplify the next edition of the alert rules: "Autochthonous conventions are part of one's general bridge knowledge and experience, but all allochthons are allocated alerts." And anyone who takes that suggestion seriously ought be allochtrocuted. GRB -------------------------------------------------------------------- Important Notice: If you have received this email by mistake, please advise the sender and delete the message and attachments immediately. This email, including attachments, may contain confidential, sensitive, legally privileged and/or copyright information. Any review, retransmission, dissemination or other use of this information by persons or entities other than the intended recipient is prohibited. DIAC respects your privacy and has obligations under the Privacy Act 1988. The official departmental privacy policy can be viewed on the department's website at www.immi.gov.au. See: http://www.immi.gov.au/functional/privacy.htm --------------------------------------------------------------------- From richard.hills at immi.gov.au Mon Jan 10 07:58:07 2011 From: richard.hills at immi.gov.au (richard.hills at immi.gov.au) Date: Mon, 10 Jan 2011 17:58:07 +1100 Subject: [BLML] Alain's case revisited [SEC=UNOFFICIAL] In-Reply-To: <936302.47101.qm@web28514.mail.ukl.yahoo.com> Message-ID: Jerry Fusselman: >>This happens at two tables---one with a confirmed >>cheater bidding 4S, and another with a confirmed >>innocent newbie bidding 4S. Do you want the >>rectifications in these two cases to be the same or >>different? (I am not asking about penalty points here, >>only the rectification.) Grattan Endicott: >+=+ Could I enquire whether the two players have entered >under the same Conditions of Contest? > If so, should one be advantaged by comparison with >the other against the field? ~ G ~ +=+ Richard Hills (male chauvinist pig): I can understand why an innocent newbie would be playing in the Venice Cup. :-) :-) But could I enquire why the confirmed cheater has also been allowed to enter under the same Conditions of Contest in this Venice Cup? WBF General Conditions of Contest 2009, clause 5.1: " ... must notably respect the spirit of fair play and ethics, and behave accordingly." WBF General Conditions of Contest 2009, clause 5.2(a): " ... taking into account the reports (if any) received from Zonal Conferences and NBOs regarding previous proceedings involving suspension of or disciplinary proceedings against players under their jurisdiction ... " WBF General Conditions of Contest 2009, clause 5.2(b): " ... In any such case of refusal, no reason shall be given by the Credentials Committee." Best wishes Richard Hills Recruitment Section Specialist Recruitment Team Level 5 Aqua, workstation W569, 6223 8453 DIAC Social Club movie tickets -------------------------------------------------------------------- Important Notice: If you have received this email by mistake, please advise the sender and delete the message and attachments immediately. This email, including attachments, may contain confidential, sensitive, legally privileged and/or copyright information. Any review, retransmission, dissemination or other use of this information by persons or entities other than the intended recipient is prohibited. DIAC respects your privacy and has obligations under the Privacy Act 1988. The official departmental privacy policy can be viewed on the department's website at www.immi.gov.au. See: http://www.immi.gov.au/functional/privacy.htm --------------------------------------------------------------------- From Hermandw at skynet.be Mon Jan 10 08:59:12 2011 From: Hermandw at skynet.be (Herman De Wael) Date: Mon, 10 Jan 2011 08:59:12 +0100 Subject: [BLML] uninteresting dWikiS case [SEC=UNOFFICIAL] In-Reply-To: References: Message-ID: <4D2ABC50.1090203@skynet.be> richard.hills at immi.gov.au wrote: >>> In my opinion, if you break the rules of a game deliberately AND >>> OPENLY to gain an advantage (e.g. by bragging about your non- >>> masochistic methods on blml), then you are simply a simpleton. > >> AG : in my opinion, if you break the rules of a game deliberately, >> without profit in mind, and opponents say you did right, then there >> is a problem with the rules. > > Begging the question, petitio principii. > > Without profit in mind??? Herman De Wael has long advocated that > his (now closed) loophole in the rules was designed with profit in > mind, hence its superiority to the (now one and only) traditional > interpretation of the Laws of Duplicate Bridge. > > In my opinion, if you break the rules of a game deliberately and > openly TO GAIN AN ADVANTAGE (e.g. by infracting Law 21B1(a) to > prevent constraint of partner's logical alternatives pursuant to > Law 16B1(a)'s criterion of "unexpected alert"), then you are simply > a simpleton. > But Richard - what is this? You are being masochistic and I, who prefer not to be hurt, am not. And I am wrong because I suffer less pain than you? Under your logic, a player is forced to revoke because not revoking is OPENLY GAINING AN ADVANTAGE. Yes, I know this is silly - but so is your argument. And I concede that indeed, the laws and regulation, by the Beijing interpretation force me to revoke, and to suffer the penalty for the revoke. But I am merely stating that I believe the WBF was wrong in issueing that interpretation. Indeed, in the whole world there is only one player who is able to knowingly break this interpretation (all the others do so unknowingly), and that one player's opponents do not even call the director. > And if you and your opponents assert that "you did right" because > there is a problem with the overly-complex Duplicate Bridge rules, > then you and your opponents should abandon Duplicate Bridge for > the simpletons' simple game of > > X|X|? > ----- > |?|X > ----- > ?| |X > > Of course, neither Herman De Wael nor Alain Gottcheiner are > simpletons. > > This is because genius-level blmlers are much better at creating > specious clutching-at-straws moving-the-goalposts begging-the- > question (petitio principii) new arguments than simpletons could > ever achieve. > And you are very good at absurd arguments without touching the true nature of the discussion. Is the Beijing interpretation the best one for the game of bridge? > Best wishes > > Richard Hills > Recruitment Section > Specialist Recruitment Team > Level 5 Aqua, workstation W569, 6223 8453 > DIAC Social Club movie tickets > > > > -------------------------------------------------------------------- > Important Notice: If you have received this email by mistake, please advise > the sender and delete the message and attachments immediately. This email, > including attachments, may contain confidential, sensitive, legally privileged > and/or copyright information. Any review, retransmission, dissemination > or other use of this information by persons or entities other than the > intended recipient is prohibited. DIAC respects your privacy and has > obligations under the Privacy Act 1988. The official departmental privacy > policy can be viewed on the department's website at www.immi.gov.au. See: > http://www.immi.gov.au/functional/privacy.htm > > > --------------------------------------------------------------------- > > _______________________________________________ > Blml mailing list > Blml at rtflb.org > http://lists.rtflb.org/mailman/listinfo/blml > > > > > No virus found in this incoming message. > Checked by AVG - www.avg.com > Version: 9.0.872 / Virus Database: 271.1.1/3370 - Release Date: 01/09/11 20:34:00 > -- Herman De Wael Wilrijk Antwerpen Belgium From Hermandw at skynet.be Mon Jan 10 09:09:01 2011 From: Hermandw at skynet.be (Herman De Wael) Date: Mon, 10 Jan 2011 09:09:01 +0100 Subject: [BLML] Alain's case revisited [SEC=UNOFFICIAL] In-Reply-To: References: Message-ID: <4D2ABE9D.1040102@skynet.be> richard.hills at immi.gov.au wrote: > Herman De Wael > >> Richard, do me a favour. >> >> Please write the sentence explaining to Alain and Patrick your >> ruling in giving them -800, including that you are ruling on the >> use of UI which has never been given at the table. >> >> Just write the sentence and then think to yourself "do I really >> want to ask TDs to make such a ruling?". > > Richard Hills: > > Herman, do me a favour. > > Consider this knockout teams grand final, in which the seats > occupied by Alain and Patrick in the Open Room are occupied by > the experts Niala and Kcirtap in the Closed Room. Niala and > Kcirtap play the same methods as Alain and Patrick, and Kcirtap > is identical in forgetfulness to Patrick. > > The only difference is that Niala invariably obeys Law 21B1(a), > so on the board in question Niala and Kcirtap inevitably score > -800. Should Alain and Patrick gain a match-winning 14 imps due > to Alain's infraction and Herman's directing??? > Richard, you are using the same argument again and again, even after I have told you it is wrong: masochism is no virtue! Of course under the Beijing interpretation it would be wrong for Alain to act as he did - but my point is that the Beijing interpretation is bad for bridge. I believe that it should be Niala who would be acting wrongly. Let me change the story one more time: There is no under the rules of the competition no need to alert 2Sp. So alain does not break the Beijing interpretation. OTOH, Niala says at the table "stupid Kcirtap - 1Sp showed diamonds". Now Kcirtap has UI, and they reach 4SpX -800. Do you still believe that Alain has done wrong? After all, he is +110, the other table is -800, and he has gained +14. Your argument proves just one thing: If the Beijing interpretation is right, then it is right. So I ask you again: in the second example, Niala has acted wrongly. He has broken L20F5. Do you consider Alain's actions correct in those circumstances? Then please tell me why he should necessarily score -800 with the alert rule in place. And please don't answer "because the WBF says so". We are both intelligent persons who are free to discuss whether the WBF rules are good for bridge or not. > On a similar topic, the offending side partially retaining its > gains from use-of-UI, the WBF Laws Committee 12th October 2010: > > [snip] > > "such adjustments may encourage players to infract the law by > allowing them some proportion of their infractive result. For > this reason such adjustments should in general be avoided." > > Best wishes > > Richard Hills > Recruitment Section > Specialist Recruitment Team > Level 5 Aqua, workstation W569, 6223 8453 > DIAC Social Club movie tickets > > > > -------------------------------------------------------------------- > Important Notice: If you have received this email by mistake, please advise > the sender and delete the message and attachments immediately. This email, > including attachments, may contain confidential, sensitive, legally privileged > and/or copyright information. Any review, retransmission, dissemination > or other use of this information by persons or entities other than the > intended recipient is prohibited. DIAC respects your privacy and has > obligations under the Privacy Act 1988. The official departmental privacy > policy can be viewed on the department's website at www.immi.gov.au. See: > http://www.immi.gov.au/functional/privacy.htm > > > --------------------------------------------------------------------- > > _______________________________________________ > Blml mailing list > Blml at rtflb.org > http://lists.rtflb.org/mailman/listinfo/blml > > > > > No virus found in this incoming message. > Checked by AVG - www.avg.com > Version: 9.0.872 / Virus Database: 271.1.1/3370 - Release Date: 01/09/11 20:34:00 > -- Herman De Wael Wilrijk Antwerpen Belgium From Hermandw at skynet.be Mon Jan 10 09:37:42 2011 From: Hermandw at skynet.be (Herman De Wael) Date: Mon, 10 Jan 2011 09:37:42 +0100 Subject: [BLML] Alain's case revisited [SEC=UNOFFICIAL] In-Reply-To: References: Message-ID: <4D2AC556.9070509@skynet.be> richard.hills at immi.gov.au wrote: > Herman De Wael > >> Richard, do me a favour. >> Did anyone else notice Richard not doing me (and us) this favour? I am trying to argue a point, and all Richard is doing is insult me and repeat points made previously. Richard is not trying to listen. -- Herman De Wael Wilrijk Antwerpen Belgium From JffEstrsn at aol.com Mon Jan 10 09:57:11 2011 From: JffEstrsn at aol.com (Jeff Easterson) Date: Mon, 10 Jan 2011 09:57:11 +0100 Subject: [BLML] Alain's case revisited [SEC=UNOFFICIAL] In-Reply-To: <4D2AC556.9070509@skynet.be> References: <4D2AC556.9070509@skynet.be> Message-ID: <4D2AC9E7.6060208@aol.com> Yes, I noticed but I didn't notice that he insulted you. At least no more than the two of you "insult" each other regularly in controversies. Are you perhaps a bit over-sensitive? Ciao, JE Am 10.01.2011 09:37, schrieb Herman De Wael: > richard.hills at immi.gov.au wrote: >> Herman De Wael >> >>> Richard, do me a favour. >>> > Did anyone else notice Richard not doing me (and us) this favour? > > I am trying to argue a point, and all Richard is doing is insult me and > repeat points made previously. > Richard is not trying to listen. > > From Hermandw at skynet.be Mon Jan 10 11:36:43 2011 From: Hermandw at skynet.be (Herman De Wael) Date: Mon, 10 Jan 2011 11:36:43 +0100 Subject: [BLML] Alain's case revisited [SEC=UNOFFICIAL] In-Reply-To: <4D2AC9E7.6060208@aol.com> References: <4D2AC556.9070509@skynet.be> <4D2AC9E7.6060208@aol.com> Message-ID: <4D2AE13B.10605@skynet.be> Maybe I am being oversensitive, but I was not commenting on me insulting Richard, but on him insulting me. which he does, sometimes. In any case, more often than responding to valid arguments that I try to put to him - and the rest of the list. I would like to see someone try, anyone, actually: You've heard the case of Patrick and Alain, and you believe Alain did something wrong. You wish to punish Alain by an AS of -800 for 4Sp doubled. Please write down the sentence you shall use to explain to them how you arrive at this score. Feel free to complete the following sentence: "Alain, you broke the Beijing interpretation by not alerting 2Sp. If you had alerted 2 Sp ..." Jeff Easterson wrote: > Yes, I noticed but I didn't notice that he insulted you. At least no > more than the two of you "insult" each other regularly in > controversies. Are you perhaps a bit over-sensitive? Ciao, JE > > Am 10.01.2011 09:37, schrieb Herman De Wael: >> richard.hills at immi.gov.au wrote: >>> Herman De Wael >>> >>>> Richard, do me a favour. >>>> >> Did anyone else notice Richard not doing me (and us) this favour? >> >> I am trying to argue a point, and all Richard is doing is insult me and >> repeat points made previously. >> Richard is not trying to listen. >> >> > > _______________________________________________ > Blml mailing list > Blml at rtflb.org > http://lists.rtflb.org/mailman/listinfo/blml > > > > > No virus found in this incoming message. > Checked by AVG - www.avg.com > Version: 9.0.872 / Virus Database: 271.1.1/3370 - Release Date: 01/09/11 20:34:00 > -- Herman De Wael Wilrijk Antwerpen Belgium From nigelguthrie at yahoo.co.uk Mon Jan 10 13:00:47 2011 From: nigelguthrie at yahoo.co.uk (Nigel Guthrie) Date: Mon, 10 Jan 2011 12:00:47 +0000 (GMT) Subject: [BLML] Alain's case revisited [SEC=UNOFFICIAL] In-Reply-To: <4D2ABE9D.1040102@skynet.be> References: <4D2ABE9D.1040102@skynet.be> Message-ID: <838975.24149.qm@web28514.mail.ukl.yahoo.com> [Herman De Wael] Of course under the Beijing interpretation it would be wrong for Alain to act as he did - but my point is that the Beijing interpretation is bad for bridge. [SNIP] And please don't answer "because the WBF says so". We are both intelligent persons who are free to discuss whether the WBF rules are good for bridge or not. [Nigel] It is sensible to *argue* about how to improve the laws of Bridge. But players, especially directors, should not *break* them or advocate breaking them. Think of it this way: Suppose you you suggest changes and they become law. Some would dislike the changes and would say so. That is their right and to be expected. But what would be your attitude - to players, who deliberately and systematically flouted the new laws and - to directors who condoned infractions and openly persuaded others to break the law? From blml at arcor.de Mon Jan 10 13:20:24 2011 From: blml at arcor.de (Thomas Dehn) Date: Mon, 10 Jan 2011 13:20:24 +0100 (CET) Subject: [BLML] Alain's case revisited [SEC=UNOFFICIAL] In-Reply-To: <838975.24149.qm@web28514.mail.ukl.yahoo.com> References: <838975.24149.qm@web28514.mail.ukl.yahoo.com> <4D2ABE9D.1040102@skynet.be> Message-ID: <1502928923.25544.1294662024180.JavaMail.ngmail@webmail08.arcor-online.net> Nigel Guthrie > [Herman De Wael] > Of course under the Beijing interpretation it would be wrong for Alain to > act as > he did - but my point is that the Beijing interpretation is > > > bad for bridge. > > [SNIP] > And please don't answer "because the WBF says so". We are both intelligent > persons who are free to discuss whether the WBF rules are > > good for bridge or not. > > [Nigel] > It is sensible to *argue* about how to improve the laws of Bridge. > > But players, especially directors, should not *break* them or advocate > breaking > them. > > Think of it this way: > > Suppose you you suggest changes and they become law. Some would dislike the > changes and would say so. That is their right and to be expected. > > > But what would be your attitude > - to players, who deliberately and systematically flouted the new laws and > - to directors who condoned infractions and openly persuaded others to break > the law? Nigel, I think you only write that because you do not consider the Beijing interpretation to be that horrible. Consider this hypothetical zero tolerance L99: "If a player makes any scoring error, such as by entering the wrong score, number of tricks, or opening lead on the traveler, or by reporting the wrong IMP score in a teams match, the player is immediately disqualified, reported as a cheater to the regulating authority, and banned for one year. If it cannot be identified which player entered the wrong score, all players at that table are disqualified, and banned for one year." The new laws come out, and they contain this L99. Would you then implement this L99 at your local club, and close the club a few weeks later because pretty much everybody has been banned for one year? Or would you look for some way to avoid applying those overly harsh penalties? Thomas From nigelguthrie at yahoo.co.uk Mon Jan 10 13:21:41 2011 From: nigelguthrie at yahoo.co.uk (Nigel Guthrie) Date: Mon, 10 Jan 2011 12:21:41 +0000 (GMT) Subject: [BLML] Alain's case revisited [SEC=UNOFFICIAL] In-Reply-To: <4D2AE13B.10605@skynet.be> References: <4D2AC556.9070509@skynet.be> <4D2AC9E7.6060208@aol.com> <4D2AE13B.10605@skynet.be> Message-ID: <334994.97415.qm@web28503.mail.ukl.yahoo.com> {Herman De Wael] You've heard the case of Patrick and Alain, and you believe Alain did something wrong. You wish to punish Alain by an AS of -800 for 4Sp doubled. Please write down the sentence you shall use to explain to them how you arrive at this score. Feel free to complete the following sentence: "Alain, you broke the Beijing interpretation by not alerting 2Sp. If you had alerted 2 Sp ..." [Nigel] "You broke the law by failing to alert 2S. (Yes the Beijing interpretation is law, even if few know about it). You admit you did this knowingly and deliberately and thereby gained 110. "To restore equity" (great phrase that!) I am adjusting your score to -800 instead. That is the result that those in your predicament might expect if they complied with the law. (You protest that such a law-abiding person would be a "masochist" but...) it is often against your interests to abide by the law. (e.g. to hand in a case full of money that you find) That is no excuse for breaking a law. From Hermandw at skynet.be Mon Jan 10 14:19:05 2011 From: Hermandw at skynet.be (Herman De Wael) Date: Mon, 10 Jan 2011 14:19:05 +0100 Subject: [BLML] Alain's case revisited [SEC=UNOFFICIAL] In-Reply-To: <838975.24149.qm@web28514.mail.ukl.yahoo.com> References: <4D2ABE9D.1040102@skynet.be> <838975.24149.qm@web28514.mail.ukl.yahoo.com> Message-ID: <4D2B0749.3090907@skynet.be> Nigel, I know all this - please react to my questions, not to my (perhaps) actions. Why is the Beijing interpretation good for bridge - if you believe it is? If all you can say is "the law is the law", don't bother. Herman. Nigel Guthrie wrote: > [Herman De Wael] > Of course under the Beijing interpretation it would be wrong for Alain to act as > he did - but my point is that the Beijing interpretation is > > > bad for bridge. > > [SNIP] > And please don't answer "because the WBF says so". We are both intelligent > persons who are free to discuss whether the WBF rules are > > good for bridge or not. > > [Nigel] > It is sensible to *argue* about how to improve the laws of Bridge. > > But players, especially directors, should not *break* them or advocate breaking > them. > > Think of it this way: > > Suppose you you suggest changes and they become law. Some would dislike the > changes and would say so. That is their right and to be expected. > > > But what would be your attitude > - to players, who deliberately and systematically flouted the new laws and > - to directors who condoned infractions and openly persuaded others to break the > law? > _______________________________________________ > Blml mailing list > Blml at rtflb.org > http://lists.rtflb.org/mailman/listinfo/blml > > > > > No virus found in this incoming message. > Checked by AVG - www.avg.com > Version: 9.0.872 / Virus Database: 271.1.1/3370 - Release Date: 01/09/11 20:34:00 > -- Herman De Wael Wilrijk Antwerpen Belgium From Hermandw at skynet.be Mon Jan 10 14:23:35 2011 From: Hermandw at skynet.be (Herman De Wael) Date: Mon, 10 Jan 2011 14:23:35 +0100 Subject: [BLML] Alain's case revisited [SEC=UNOFFICIAL] In-Reply-To: <334994.97415.qm@web28503.mail.ukl.yahoo.com> References: <4D2AC556.9070509@skynet.be> <4D2AC9E7.6060208@aol.com> <4D2AE13B.10605@skynet.be> <334994.97415.qm@web28503.mail.ukl.yahoo.com> Message-ID: <4D2B0857.7050804@skynet.be> OK Nigel, thanks for trying to go along with me, but I have a comment: Nigel Guthrie wrote: > > [Nigel] > "You broke the law by failing to alert 2S. (Yes the Beijing interpretation is > law, even if few know about it). You admit you did this knowingly and > deliberately and thereby gained 110. "To restore equity" (great phrase that!) I > am adjusting your score to -800 instead. That is the result that those in your > predicament might expect if they complied with the law. (You protest that such a > law-abiding person would be a "masochist" but...) it is often against your > interests to abide by the law. (e.g. to hand in a case full of money that you > find) That is no excuse for breaking a law. You did not answer this the way I believed you should, so I shall simply ask the next question the player is going to ask you, as the TD at the table: "How do you arrive at a score of -800?" and I'll ask his next question if your answer is nothing more than "4SpX-3": "How can we ever arrive in 4Sp? - I know he does not have spades, and he knows I have only 4 of them - so how could we ever pass out 4SpX, how could we even bid them?" Please answer that question from a player. -- Herman De Wael Wilrijk Antwerpen Belgium From nigelguthrie at yahoo.co.uk Mon Jan 10 14:44:42 2011 From: nigelguthrie at yahoo.co.uk (Nigel Guthrie) Date: Mon, 10 Jan 2011 13:44:42 +0000 (GMT) Subject: [BLML] Alain's case revisited [SEC=UNOFFICIAL] In-Reply-To: <1502928923.25544.1294662024180.JavaMail.ngmail@webmail08.arcor-online.net> References: <838975.24149.qm@web28514.mail.ukl.yahoo.com> <4D2ABE9D.1040102@skynet.be> <1502928923.25544.1294662024180.JavaMail.ngmail@webmail08.arcor-online.net> Message-ID: <99661.50541.qm@web28501.mail.ukl.yahoo.com> [Thomas Dehn] I think you only write that because you do not consider the Beijing interpretation to be that horrible. Consider this hypothetical zero tolerance L99: "If a player makes any scoring error, such as by entering the wrong score, number of tricks, or opening lead on the traveler, or by reporting the wrong IMP score in a teams match, the player is immediately disqualified, reported as a cheater to the regulating authority, and banned for one year. If it cannot be identified which player entered the wrong score, all players at that table are disqualified, and banned for one year." The new laws come out, and they contain this L99. Would you then implement this L99 at your local club, and close the club a few weeks later because pretty much everybody has been banned for one year? Or would you look for some way to avoid applying those overly harsh penalties? [Nigel] Thomas asks what if a law-change is so disliked by players that they stop playing or are banned for breaking it. As a law-maker, I would keep changing it until they did like it. Or I would change it back to what it was. IMO, that is what the law-makers of a game should do. A game is pointless unless players enjoy it. For universally hated laws and for ambiguities and anomalies (eg where the law-maker's intention is unclear) law-changes should be *immediate*, and in place. That policy might have nipped DWS in the bud. For most changes, however, you need a decent trial-period --- but a *decade* is far too long, IMO. Anyway the answer is not to break the laws but to persuade law-makers to change them. If a group dislike the law but law-makers are intransigent, then that group can just invent a new game with their own rules -- as the ACBL seem to have already done :) But I don't think they should call that game "Bridge". In practice, many players regularly break many of the laws. One example: many players in receipt of UI, "make the bid that they would have made anyway". Some do so because the are incapable of the feats of imagination required by law. But most, because they don't like the law and feel that it is unfair. Most uses of UI are not ruled against. Few are even noticed or reported. As far as I know, nobody has been banned. Arguably laws that are misunderstood and rarely enforced should be changed or dropped; but that is no excuse to break them. I've suggested many radical law-simplifications that mitigate this sort of thing :) BLMLers consider most of them "horrible" :( From blml at arcor.de Mon Jan 10 14:46:27 2011 From: blml at arcor.de (Thomas Dehn) Date: Mon, 10 Jan 2011 14:46:27 +0100 (CET) Subject: [BLML] Alain's case revisited [SEC=UNOFFICIAL] In-Reply-To: <4D2B0857.7050804@skynet.be> References: <4D2B0857.7050804@skynet.be> <4D2AC556.9070509@skynet.be> <4D2AC9E7.6060208@aol.com> <4D2AE13B.10605@skynet.be> <334994.97415.qm@web28503.mail.ukl.yahoo.com> Message-ID: <1391468851.32037.1294667187404.JavaMail.ngmail@webmail08.arcor-online.net> Herman De Wael wrote: > OK Nigel, thanks for trying to go along with me, but I have a comment: > > Nigel Guthrie wrote: > > > > [Nigel] > > "You broke the law by failing to alert 2S. (Yes the Beijing interpretation is > > law, even if few know about it). You admit you did this knowingly and > > deliberately and thereby gained 110. "To restore equity" (great phrase that!) I > > am adjusting your score to -800 instead. That is the result that those in your > > predicament might expect if they complied with the law. (You protest that such a > > law-abiding person would be a "masochist" but...) it is often against your > > interests to abide by the law. (e.g. to hand in a case full of money that you > > find) That is no excuse for breaking a law. > > You did not answer this the way I believed you should, so I shall simply > ask the next question the player is going to ask you, as the TD at the > table: > > "How do you arrive at a score of -800?" and I'll ask his next question > if your answer is nothing more than "4SpX-3": > "How can we ever arrive in 4Sp? - I know he does not have spades, and he > knows I have only 4 of them - so how could we ever pass out 4SpX, how > could we even bid them?" That flaw is in the ruling you considered, Herman, not in the "Bejing interpretation". Indeed 4SpX-3 looks not "at all probable had the irregularity not occured". Nobody in this thread understood how you arrived at 4SX. 6NTX or 6DX, though ... Thomas From nigelguthrie at yahoo.co.uk Mon Jan 10 14:58:26 2011 From: nigelguthrie at yahoo.co.uk (Nigel Guthrie) Date: Mon, 10 Jan 2011 13:58:26 +0000 (GMT) Subject: [BLML] Alain's case revisited [SEC=UNOFFICIAL] In-Reply-To: <4D2B0749.3090907@skynet.be> References: <4D2ABE9D.1040102@skynet.be> <838975.24149.qm@web28514.mail.ukl.yahoo.com> <4D2B0749.3090907@skynet.be> Message-ID: <726101.67398.qm@web28515.mail.ukl.yahoo.com> [Herman De Wael] Nigel, I know all this - please react to my questions, not to my (perhaps) actions. Why is the Beijing interpretation good for bridge - if you believe it is? If all you can say is "the law is the law", don't bother. [Nigel] I will bother although I feel that you should obey a law you don't like (but continue to campaign for change). Your questions have been answered many times by me and others. BTW, If you read my posts, you will realize that I think current disclosure laws are ultra-sophisticated, over-subjective, unenforceable, and incomplete; so I too have suggested radical simplification. From nigelguthrie at yahoo.co.uk Mon Jan 10 15:06:44 2011 From: nigelguthrie at yahoo.co.uk (Nigel Guthrie) Date: Mon, 10 Jan 2011 14:06:44 +0000 (GMT) Subject: [BLML] Alain's case revisited [SEC=UNOFFICIAL] In-Reply-To: <4D2B0857.7050804@skynet.be> References: <4D2AC556.9070509@skynet.be> <4D2AC9E7.6060208@aol.com> <4D2AE13B.10605@skynet.be> <334994.97415.qm@web28503.mail.ukl.yahoo.com> <4D2B0857.7050804@skynet.be> Message-ID: <683970.34112.qm@web28508.mail.ukl.yahoo.com> [Herman] You did not answer this the way I believed you should, so I shall simply ask the next question the player is going to ask you, as the TD at the table: "How do you arrive at a score of -800?" and I'll ask his next question if your answer is nothing more than "4SpX-3": "How can we ever arrive in 4Sp? - I know he does not have spades, and he knows I have only 4 of them - so how could we ever pass out 4SpX, how could we even bid them?" Please answer that question from a player. [Nigel] Now, you are quibbling, Herman. It is a question of director-judgement what the score would have been, had you not broken the law. If you think some other score would have been more likely, then you can explain why. But in the end, the director has the right to impose his own judgement, subject to appeal. From Hermandw at skynet.be Mon Jan 10 15:50:37 2011 From: Hermandw at skynet.be (Herman De Wael) Date: Mon, 10 Jan 2011 15:50:37 +0100 Subject: [BLML] Alain's case revisited [SEC=UNOFFICIAL] In-Reply-To: <683970.34112.qm@web28508.mail.ukl.yahoo.com> References: <4D2AC556.9070509@skynet.be> <4D2AC9E7.6060208@aol.com> <4D2AE13B.10605@skynet.be> <334994.97415.qm@web28503.mail.ukl.yahoo.com> <4D2B0857.7050804@skynet.be> <683970.34112.qm@web28508.mail.ukl.yahoo.com> Message-ID: <4D2B1CBD.6030301@skynet.be> Nigel Guthrie wrote: > [Herman] > You did not answer this the way I believed you should, so I shall simply > > ask the next question the player is going to ask you, as the TD at the > table: > > "How do you arrive at a score of -800?" and I'll ask his next question > if your answer is nothing more than "4SpX-3": > "How can we ever arrive in 4Sp? - I know he does not have spades, and he > knows I have only 4 of them - so how could we ever pass out 4SpX, how > could we even bid them?" > Please answer that question from a player. > > [Nigel] > Now, you are quibbling, Herman. > Now you are avoiding, Nigel. I am not asking to defend the ruling (which neither of us can be certain about) Pleaase write down the sentence. You will se what I mean when you have written a list of convoluted "would have"s. I would write down the sentence myself, but then you would not be convinced that it cannot be said shorter and more understandable. It really is an atrocious piece of literature, which no player (and few directors) will ever be able to follow. Try it, please. > It is a question of director-judgement what the score would have been, had you > not broken the law. If you think some other score would have been more likely, > then you can explain why. But in the end, the director has the right to impose > his own judgement, subject to appeal. -- Herman De Wael Wilrijk Antwerpen Belgium From Hermandw at skynet.be Mon Jan 10 15:51:56 2011 From: Hermandw at skynet.be (Herman De Wael) Date: Mon, 10 Jan 2011 15:51:56 +0100 Subject: [BLML] Alain's case revisited [SEC=UNOFFICIAL] In-Reply-To: <1391468851.32037.1294667187404.JavaMail.ngmail@webmail08.arcor-online.net> References: <4D2B0857.7050804@skynet.be> <4D2AC556.9070509@skynet.be> <4D2AC9E7.6060208@aol.com> <4D2AE13B.10605@skynet.be> <334994.97415.qm@web28503.mail.ukl.yahoo.com> <1391468851.32037.1294667187404.JavaMail.ngmail@webmail08.arcor-online.net> Message-ID: <4D2B1D0C.7090801@skynet.be> Thomas Dehn wrote: > Herman De Wael wrote: >> OK Nigel, thanks for trying to go along with me, but I have a comment: >> >> Nigel Guthrie wrote: >>> >>> [Nigel] >>> "You broke the law by failing to alert 2S. (Yes the Beijing interpretation is >>> law, even if few know about it). You admit you did this knowingly and >>> deliberately and thereby gained 110. "To restore equity" (great phrase that!) I >>> am adjusting your score to -800 instead. That is the result that those in your >>> predicament might expect if they complied with the law. (You protest that such a >>> law-abiding person would be a "masochist" but...) it is often against your >>> interests to abide by the law. (e.g. to hand in a case full of money that you >>> find) That is no excuse for breaking a law. >> >> You did not answer this the way I believed you should, so I shall simply >> ask the next question the player is going to ask you, as the TD at the >> table: >> >> "How do you arrive at a score of -800?" and I'll ask his next question >> if your answer is nothing more than "4SpX-3": >> "How can we ever arrive in 4Sp? - I know he does not have spades, and he >> knows I have only 4 of them - so how could we ever pass out 4SpX, how >> could we even bid them?" > > That flaw is in the ruling you considered, Herman, not in the "Bejing interpretation". > Indeed 4SpX-3 looks not "at all probable had the irregularity not occured". > Nobody in this thread understood how you arrived at 4SX. > > 6NTX or 6DX, though ... > Well Thomas, for all our sakes - please enlighten us, and write down the sentence. > > Thomas The mere fact that you don't answer the question shows me why you still fail to understand my point. Really, try it. -- Herman De Wael Wilrijk Antwerpen Belgium From agot at ulb.ac.be Mon Jan 10 16:38:20 2011 From: agot at ulb.ac.be (Alain Gottcheiner) Date: Mon, 10 Jan 2011 16:38:20 +0100 Subject: [BLML] Alain's case revisited [SEC=UNOFFICIAL] In-Reply-To: <838975.24149.qm@web28514.mail.ukl.yahoo.com> References: <4D2ABE9D.1040102@skynet.be> <838975.24149.qm@web28514.mail.ukl.yahoo.com> Message-ID: <4D2B27EC.5050507@ulb.ac.be> Le 10/01/2011 13:00, Nigel Guthrie a ?crit : > [Herman De Wael] > Of course under the Beijing interpretation it would be wrong for Alain to act as > he did - but my point is that the Beijing interpretation is > > > bad for bridge. > > [SNIP] > And please don't answer "because the WBF says so". We are both intelligent > persons who are free to discuss whether the WBF rules are > > good for bridge or not. > > [Nigel] > It is sensible to *argue* about how to improve the laws of Bridge. > > But players, especially directors, should not *break* them or advocate breaking > them. > > Think of it this way: > > Suppose you you suggest changes and they become law. Some would dislike the > changes and would say so. That is their right and to be expected. > > > But what would be your attitude > - to players, who deliberately and systematically flouted the new laws and > - to directors who condoned infractions and openly persuaded others to break the > law? AG : you're right, but fail to consider one thing. If and when dWS were implemented, no one would think about doing otherwise. Because it's so reasonable. Up to now, I've heard comments like "unlawful", "eccentric", "ch***ing", but never "illogical". Best regards Alain From nigelguthrie at yahoo.co.uk Mon Jan 10 16:48:41 2011 From: nigelguthrie at yahoo.co.uk (Nigel Guthrie) Date: Mon, 10 Jan 2011 15:48:41 +0000 (GMT) Subject: [BLML] Alain's case revisited [SEC=UNOFFICIAL] In-Reply-To: <4D2B1CBD.6030301@skynet.be> References: <4D2AC556.9070509@skynet.be> <4D2AC9E7.6060208@aol.com> <4D2AE13B.10605@skynet.be> <334994.97415.qm@web28503.mail.ukl.yahoo.com> <4D2B0857.7050804@skynet.be> <683970.34112.qm@web28508.mail.ukl.yahoo.com> <4D2B1CBD.6030301@skynet.be> Message-ID: <596816.91301.qm@web28509.mail.ukl.yahoo.com> [Herman] Now you are avoiding, Nigel. I am not asking to defend the ruling (which neither of us can be certain about). Please write down the sentence. You will see what I mean when you have written a list of convoluted "would have"s. I would write down the sentence myself, but then you would not be convinced that it cannot be said shorter and more understandable. It really is an atrocious piece of literature, which no player (and few directors) will ever be able to follow. Try it, please. [Nigel] I'm a player and few of us understand the laws. FWIW, IMO disclosure laws are already to complex. But I've already tried to explain this ruling. Twice. I've no idea how the bidding would go, had there been no infraction. That depends on your agreements. The assumption is that *having established your agreements*, the director judges that a law-abiding pair in your predicament would reach 4SX-3 (or whatever). That is disputable, and you can challenge it. The same kind of reasoning is used, elsewhere, in the law, so that (if this ruling is correct) sophists versed in equity law have no trouble explaining it. Few players will understand but that is true of rulings based on many other laws. I think what I write is clear. If not, try rewriting it in a form, clearer to you, yourself From Hermandw at skynet.be Mon Jan 10 16:55:40 2011 From: Hermandw at skynet.be (Herman De Wael) Date: Mon, 10 Jan 2011 16:55:40 +0100 Subject: [BLML] Alain's case revisited [SEC=UNOFFICIAL] In-Reply-To: <596816.91301.qm@web28509.mail.ukl.yahoo.com> References: <4D2AC556.9070509@skynet.be> <4D2AC9E7.6060208@aol.com> <4D2AE13B.10605@skynet.be> <334994.97415.qm@web28503.mail.ukl.yahoo.com> <4D2B0857.7050804@skynet.be> <683970.34112.qm@web28508.mail.ukl.yahoo.com> <4D2B1CBD.6030301@skynet.be> <596816.91301.qm@web28509.mail.ukl.yahoo.com> Message-ID: <4D2B2BFC.9000505@skynet.be> Nigel Guthrie wrote: > [Herman] > Now you are avoiding, Nigel. I am not asking to defend the ruling (which neither > of us can be certain about). > > Please write down the sentence. > You will see what I mean when you have written a list of convoluted > "would have"s. > I would write down the sentence myself, but then you would not be > convinced that it cannot be said shorter and more understandable. > It really is an atrocious piece of literature, which no player (and few > directors) will ever be able to follow. > Try it, please. > [Nigel] > I'm a player and few of us understand the laws. FWIW, IMO disclosure laws are > already to complex. But I've already tried to explain this ruling. > > Twice. > > I've no idea how the bidding would go, had there been no infraction. That > depends on your agreements. The assumption is that *having established your > agreements*, the director judges that a law-abiding pair in your predicament > would reach 4SX-3 (or whatever). That is disputable, and you can challenge it. > No they would not! By not writing down the sentence, after whichever piece of auction, you still have not seen what I mean. The problem is that, after whichever actual auction, the pair would not arrive in 4Sp. Only the Director can arrive in 4Sp, and he would need a reasoning like this: "If Alain had alerted, Patrick would have had UI, which would not allow him to realize that passing 3Di might work to his advantage, so the TD would have had to rule 4Sp". Try it, and try to avoid the question: how can I be judged not to have found the solution if I hadn't had the UI, when I actually found the solution without the UI?" The whole thing is crazy, and no player will ever understand it. > > The same kind of reasoning is used, elsewhere, in the law, so that (if this > ruling is correct) sophists versed in equity law have no trouble explaining it. > Few players will understand but that is true of rulings based on many other > laws. > > I think what I write is clear. > If not, try rewriting it in a form, clearer to you, yourself No, try it for yourself. What you write is clear, but it doesn't address the difficulty in explaining a double negative. Remember that Patrick found out he had forgotten the meaning of 1Sp by himself, without the use of any UI. How can you then explain ruling against him for a piece of UI that he never received? -- Herman De Wael Wilrijk Antwerpen Belgium From agot at ulb.ac.be Mon Jan 10 16:59:04 2011 From: agot at ulb.ac.be (Alain Gottcheiner) Date: Mon, 10 Jan 2011 16:59:04 +0100 Subject: [BLML] Alain's case revisited [SEC=UNOFFICIAL] In-Reply-To: <4D2B1D0C.7090801@skynet.be> References: <4D2B0857.7050804@skynet.be> <4D2AC556.9070509@skynet.be> <4D2AC9E7.6060208@aol.com> <4D2AE13B.10605@skynet.be> <334994.97415.qm@web28503.mail.ukl.yahoo.com> <1391468851.32037.1294667187404.JavaMail.ngmail@webmail08.arcor-online.net> <4D2B1D0C.7090801@skynet.be> Message-ID: <4D2B2CC8.2060308@ulb.ac.be> Le 10/01/2011 15:51, Herman De Wael a ?crit : > Thomas Dehn wrote: >> Herman De Wael wrote: >>> OK Nigel, thanks for trying to go along with me, but I have a comment: >>> >>> Nigel Guthrie wrote: >>>> [Nigel] >>>> "You broke the law by failing to alert 2S. (Yes the Beijing interpretation is >>>> law, even if few know about it). You admit you did this knowingly and >>>> deliberately and thereby gained 110. "To restore equity" (great phrase that!) I >>>> am adjusting your score to -800 instead. That is the result that those in your >>>> predicament might expect if they complied with the law. (You protest that such a >>>> law-abiding person would be a "masochist" but...) it is often against your >>>> interests to abide by the law. (e.g. to hand in a case full of money that you >>>> find) That is no excuse for breaking a law. >>> You did not answer this the way I believed you should, so I shall simply >>> ask the next question the player is going to ask you, as the TD at the >>> table: >>> >>> "How do you arrive at a score of -800?" and I'll ask his next question >>> if your answer is nothing more than "4SpX-3": >>> "How can we ever arrive in 4Sp? - I know he does not have spades, and he >>> knows I have only 4 of them - so how could we ever pass out 4SpX, how >>> could we even bid them?" >> That flaw is in the ruling you considered, Herman, not in the "Bejing interpretation". >> Indeed 4SpX-3 looks not "at all probable had the irregularity not occured". >> Nobody in this thread understood how you arrived at 4SX. >> >> 6NTX or 6DX, though ... >> > AG : do you really think that, facing a normal reverse, a complete misfit (revealed after 3D-3H) and holding a 13-count without hope of quickly establishing my suit, I would have bid anything other than 3NT or 4NT ? Of course, I would have been compelled to bid the latter, "bending backwards", but why would we reach six ? Furthermore, the 4NT bid would be so inconsistent with the 3D bid (because all game forces over a major raise begin with 2NT or a jump) that partner would now have AI matching UI. This shows well the whole problem with applying L12 : it compels the TD to twist normal bidding to a great extent. which lead to absurd positions like imagining that we could reach 6NT. Now, using dWS would avoid such problems, of course. From svenpran at online.no Mon Jan 10 17:07:16 2011 From: svenpran at online.no (Sven Pran) Date: Mon, 10 Jan 2011 17:07:16 +0100 Subject: [BLML] Alain's case revisited [SEC=UNOFFICIAL] In-Reply-To: <4D2B27EC.5050507@ulb.ac.be> References: <4D2ABE9D.1040102@skynet.be> <838975.24149.qm@web28514.mail.ukl.yahoo.com> <4D2B27EC.5050507@ulb.ac.be> Message-ID: <000801cbb0e0$7461cb00$5d256100$@no> On Behalf Of Alain Gottcheiner > AG : you're right, but fail to consider one thing. > > If and when dWS were implemented, no one would think about doing otherwise. > Because it's so reasonable. > Up to now, I've heard comments like "unlawful", "eccentric", "ch***ing", but never > "illogical". Why then not consider the "best" law to be that the player making a call explains it him- (or her-) self? That would ensure that opponents always receive correct information. This reminds me of a tale from an American bridgeclub back in Culbertson's days: A drop-in player asked if they used asking bids (recently added to Culbertson's system) and the answer was "yes, we use asking bids". A little later was heard the following 'asking bid': "Do you have the King of Diamonds, partner?" Why not allow players to ask "how many aces do you hold partner?" instead of bidding 4NT? NO. The communication between partners in bridge is limited to the "bridge language" comprising a total of 38 different (possible) calls to be treated as codes with predefined meanings defined within the relevant partnership agreements and understandings. The only logical consequence is that these decoded meanings alone are legal information and as such shall be disclosed to opponents. Cause for adjustment exists if opponents are "damaged" from misinformation (but not from misbids so long as we allow psyches etc.). From agot at ulb.ac.be Mon Jan 10 17:58:17 2011 From: agot at ulb.ac.be (Alain Gottcheiner) Date: Mon, 10 Jan 2011 17:58:17 +0100 Subject: [BLML] Alain's case revisited [SEC=UNOFFICIAL] In-Reply-To: <000801cbb0e0$7461cb00$5d256100$@no> References: <4D2ABE9D.1040102@skynet.be> <838975.24149.qm@web28514.mail.ukl.yahoo.com> <4D2B27EC.5050507@ulb.ac.be> <000801cbb0e0$7461cb00$5d256100$@no> Message-ID: <4D2B3AA9.7050100@ulb.ac.be> Le 10/01/2011 17:07, Sven Pran a ?crit : > On Behalf Of Alain Gottcheiner >> AG : you're right, but fail to consider one thing. >> >> If and when dWS were implemented, no one would think about doing > otherwise. >> Because it's so reasonable. >> Up to now, I've heard comments like "unlawful", "eccentric", "ch***ing", > but never >> "illogical". > Why then not consider the "best" law to be that the player making a call > explains it him- (or her-) self? That would ensure that opponents always > receive correct information. AG : of course ! You only need 4 little notebooks. The player making the bid writes the meaning down on one's own notebook, then shows it to both opponents in turn if they want to know. Kind of generalizing the behind-screen procedure. I suppose that the reason why the procedure wasn't implemented is that it would be ratehr slow. Because, to avoid giving away UI that your bid is artificial, you'd need to scribble something in all cases. But indeed, in a world not governed by time, this would be by far the best method. From blml at arcor.de Mon Jan 10 18:06:34 2011 From: blml at arcor.de (Thomas Dehn) Date: Mon, 10 Jan 2011 18:06:34 +0100 (CET) Subject: [BLML] Alain's case revisited [SEC=UNOFFICIAL] In-Reply-To: <4D2B2CC8.2060308@ulb.ac.be> References: <4D2B2CC8.2060308@ulb.ac.be> <4D2B0857.7050804@skynet.be> <4D2AC556.9070509@skynet.be> <4D2AC9E7.6060208@aol.com> <4D2AE13B.10605@skynet.be> <334994.97415.qm@web28503.mail.ukl.yahoo.com> <1391468851.32037.1294667187404.JavaMail.ngmail@webmail08.arcor-online.net> <4D2B1D0C.7090801@skynet.be> Message-ID: <1786675819.19695.1294679194425.JavaMail.ngmail@webmail17.arcor-online.net> Alain Gottcheiner wrote: > Le 10/01/2011 15:51, Herman De Wael a ?crit : > > Thomas Dehn wrote: > >> Herman De Wael wrote: > >>> OK Nigel, thanks for trying to go along with me, but I have a comment: > >>> > >>> Nigel Guthrie wrote: > >>>> [Nigel] > >>>> "You broke the law by failing to alert 2S. (Yes the Beijing interpretation is > >>>> law, even if few know about it). You admit you did this knowingly and > >>>> deliberately and thereby gained 110. "To restore equity" (great phrase that!) I > >>>> am adjusting your score to -800 instead. That is the result that those in your > >>>> predicament might expect if they complied with the law. (You protest that such a > >>>> law-abiding person would be a "masochist" but...) it is often against your > >>>> interests to abide by the law. (e.g. to hand in a case full of money that you > >>>> find) That is no excuse for breaking a law. > >>> You did not answer this the way I believed you should, so I shall simply > >>> ask the next question the player is going to ask you, as the TD at the > >>> table: > >>> > >>> "How do you arrive at a score of -800?" and I'll ask his next question > >>> if your answer is nothing more than "4SpX-3": > >>> "How can we ever arrive in 4Sp? - I know he does not have spades, and he > >>> knows I have only 4 of them - so how could we ever pass out 4SpX, how > >>> could we even bid them?" > >> That flaw is in the ruling you considered, Herman, not in the "Bejing interpretation". > >> Indeed 4SpX-3 looks not "at all probable had the irregularity not occured". > >> Nobody in this thread understood how you arrived at 4SX. > >> > >> 6NTX or 6DX, though ... > >> > > > AG : do you really think that, facing a normal reverse, a complete > misfit (revealed after 3D-3H) and holding a 13-count without hope of > quickly establishing my suit, I would have bid anything other than 3NT > or 4NT ? Of course, I would have been compelled to bid the latter, > "bending backwards", but why would we reach six ? Because your partner, who thinks that 1S - 2S established a S fit, will interpret 4NT as some sort of slam try for spades (RKCB in many systems), rather than as quantitative. > Furthermore, the 4NT bid would be so inconsistent with the 3D bid > (because all game forces over a major raise begin with 2NT or a jump) > that partner would now have AI matching UI. That is a totally different argument. Of course depending on the details of the system and the auction, the subsequent auction can become so inconsistent that the TD will assume that the player who received UI from an alert or a failure to alert would have worked out the problem anyways at some point. More precisely, that it is not even "at all probably" that the player had not worked out the problem absent the UI. Thomas From blml at arcor.de Mon Jan 10 19:00:12 2011 From: blml at arcor.de (Thomas Dehn) Date: Mon, 10 Jan 2011 19:00:12 +0100 (CET) Subject: [BLML] Alain's case revisited [SEC=UNOFFICIAL] Message-ID: <742678624.21558.1294682412469.JavaMail.ngmail@webmail17.arcor-online.net> Herman De Wael wrote: > Thomas Dehn wrote: > > Herman De Wael wrote: > >> OK Nigel, thanks for trying to go along with me, but I have a comment: > >> > >> Nigel Guthrie wrote: > >>> > >>> [Nigel] > >>> "You broke the law by failing to alert 2S. (Yes the Beijing interpretation is > >>> law, even if few know about it). You admit you did this knowingly and > >>> deliberately and thereby gained 110. "To restore equity" (great phrase that!) I > >>> am adjusting your score to -800 instead. That is the result that those in your > >>> predicament might expect if they complied with the law. (You protest that such a > >>> law-abiding person would be a "masochist" but...) it is often against your > >>> interests to abide by the law. (e.g. to hand in a case full of money that you > >>> find) That is no excuse for breaking a law. > >> > >> You did not answer this the way I believed you should, so I shall simply > >> ask the next question the player is going to ask you, as the TD at the > >> table: > >> > >> "How do you arrive at a score of -800?" and I'll ask his next question > >> if your answer is nothing more than "4SpX-3": > >> "How can we ever arrive in 4Sp? - I know he does not have spades, and he > >> knows I have only 4 of them - so how could we ever pass out 4SpX, how > >> could we even bid them?" > > > > That flaw is in the ruling you considered, Herman, not in the "Bejing interpretation". > > Indeed 4SpX-3 looks not "at all probable had the irregularity not occured". > > Nobody in this thread understood how you arrived at 4SX. > > > > 6NTX or 6DX, though ... > > > > Well Thomas, for all our sakes - please enlighten us, and write down the > sentence. > > > > > Thomas > > The mere fact that you don't answer the question shows me why you still > fail to understand my point. > Really, try it. I don't see the problem, other than that it will be more than one sentence. Just like a lower court references the binding decision of a national court, you reference the binding decision of the WBFLC. And just like a lower court you should feel free to state in your ruling that you disagree with that binding decision. There were two irregularities in this board o the failure to alert 1S o the failure to alert 2S (1) Furthermore, from the 2S bid we know that the player who failed to alert 1S at that point did not correctly remember system, rather than merely failed to alert. (1) L20F5(a) states that "A player whose partner has given a mistaken explanation may not correct the error during the auction, nor may he indicate in any manner that a mistake has been made." The WBFLC has made a binding decision that L20F5(a) does not apply in this situation, 2S must be alerted even though that tells partner that 1S did not show spades. http://www.ecatsbridge.com/Documents/files/WBFInformation/Reports_Minutes/LawsCommitteeMinutes/2008-Beijing.pdf, page 5. "There is no infraction when a correct explanation discloses that partner?s prior explanation was mistaken. The words ?nor may he indicate in any manner that a mistake has been made? (in Law 20F5(a)) do not refer to compliance with the overriding requirement of the laws always to respond to enquiries under Law 20F with correct explanations of the partnership understandings. " This is an unusual new interpretation of TFLB, but nevertheless, until the WBFLC has reversed this, I am bound by this decision. If you want to complain about this interpretation, please complain to the WBFLC, here is how to contact them. Furthermore, the WBFLC also has decided that WBFLC decisions that are hidden in some pdfs on their web page are binding even though they were never communicated to you in any official publication, such as an update to the local translation of TFLB. The score for the offensive side is the most unfavorable result that was at all probable (L12C1). Note that there is no "had the irregularity not occurred". Here, the NOS will get the worst out of o both 1S and 1S alerted o only 1S alerted o neither 1S nor 2S alerted Furthermore, assessment of probable auctions and plays is not limited to what occurred at the table. Had 2S been alerted as required by the alert regulations, opener would not have been allowed to use the UI from the unexpected alert, and thus would not have been allowed to "wake up" and remember that 1S was T-Walsh (see L16 B1(a) and the footnote to L16). He would only have been allowed to "wake up" when the auction itself (rather than the unexpected alert) shows that something is wrong. This holds regardless of whether opener actually remembered the system. Do you have your system notes with you so that I can work out which subsequent auctions were "at all probable" respectively "likely had the irregularity not occurred"? I'm pretty sure I won't arrive at Herman's 4SX-3. If I then deem the result to be far away from equity, I might be inclined to grant a request by the NOS to waive the rectification, L81C5. Thomas From JffEstrsn at aol.com Mon Jan 10 21:10:33 2011 From: JffEstrsn at aol.com (Jeff Easterson) Date: Mon, 10 Jan 2011 21:10:33 +0100 Subject: [BLML] Alain's case revisited [SEC=UNOFFICIAL] In-Reply-To: <596816.91301.qm@web28509.mail.ukl.yahoo.com> References: <4D2AC556.9070509@skynet.be> <4D2AC9E7.6060208@aol.com> <4D2AE13B.10605@skynet.be> <334994.97415.qm@web28503.mail.ukl.yahoo.com> <4D2B0857.7050804@skynet.be> <683970.34112.qm@web28508.mail.ukl.yahoo.com> <4D2B1CBD.6030301@skynet.be> <596816.91301.qm@web28509.mail.ukl.yahoo.com> Message-ID: <4D2B67B9.2030802@aol.com> Test Am 10.01.2011 16:48, schrieb Nigel Guthrie: > [Herman] > Now you are avoiding, Nigel. I am not asking to defend the ruling (which neither > of us can be certain about). > > Please write down the sentence. > You will see what I mean when you have written a list of convoluted > "would have"s. > I would write down the sentence myself, but then you would not be > convinced that it cannot be said shorter and more understandable. > It really is an atrocious piece of literature, which no player (and few > directors) will ever be able to follow. > Try it, please. > [Nigel] > I'm a player and few of us understand the laws. FWIW, IMO disclosure laws are > already to complex. But I've already tried to explain this ruling. > > Twice. > > I've no idea how the bidding would go, had there been no infraction. That > depends on your agreements. The assumption is that *having established your > agreements*, the director judges that a law-abiding pair in your predicament > would reach 4SX-3 (or whatever). That is disputable, and you can challenge it. > > > The same kind of reasoning is used, elsewhere, in the law, so that (if this > ruling is correct) sophists versed in equity law have no trouble explaining it. > Few players will understand but that is true of rulings based on many other > laws. > > I think what I write is clear. > If not, try rewriting it in a form, clearer to you, yourself > _______________________________________________ > Blml mailing list > Blml at rtflb.org > http://lists.rtflb.org/mailman/listinfo/blml > From JffEstrsn at aol.com Mon Jan 10 21:13:25 2011 From: JffEstrsn at aol.com (Jeff Easterson) Date: Mon, 10 Jan 2011 21:13:25 +0100 Subject: [BLML] Alain's case revisited [SEC=UNOFFICIAL] In-Reply-To: <596816.91301.qm@web28509.mail.ukl.yahoo.com> References: <4D2AC556.9070509@skynet.be> <4D2AC9E7.6060208@aol.com> <4D2AE13B.10605@skynet.be> <334994.97415.qm@web28503.mail.ukl.yahoo.com> <4D2B0857.7050804@skynet.be> <683970.34112.qm@web28508.mail.ukl.yahoo.com> <4D2B1CBD.6030301@skynet.be> <596816.91301.qm@web28509.mail.ukl.yahoo.com> Message-ID: <4D2B6865.5090205@aol.com> Please excuse the previous email (test). I kept trying to send this email and it kept coming back. I haven't been following this thread closely so don't understand the ominous "sentence" to which they keep referring. ("Please write down the sentence.") What sentence is this? Ciao, JE Am 10.01.2011 16:48, schrieb Nigel Guthrie: > [Herman] > Now you are avoiding, Nigel. I am not asking to defend the ruling (which neither > of us can be certain about). > > Please write down the sentence. > You will see what I mean when you have written a list of convoluted > "would have"s. > I would write down the sentence myself, but then you would not be > convinced that it cannot be said shorter and more understandable. > It really is an atrocious piece of literature, which no player (and few > directors) will ever be able to follow. > Try it, please. > [Nigel] > I'm a player and few of us understand the laws. FWIW, IMO disclosure laws are > already to complex. But I've already tried to explain this ruling. > > Twice. > > I've no idea how the bidding would go, had there been no infraction. That > depends on your agreements. The assumption is that *having established your > agreements*, the director judges that a law-abiding pair in your predicament > would reach 4SX-3 (or whatever). That is disputable, and you can challenge it. > > > The same kind of reasoning is used, elsewhere, in the law, so that (if this > ruling is correct) sophists versed in equity law have no trouble explaining it. > Few players will understand but that is true of rulings based on many other > laws. > > I think what I write is clear. > If not, try rewriting it in a form, clearer to you, yourself > _______________________________________________ > Blml mailing list > Blml at rtflb.org > http://lists.rtflb.org/mailman/listinfo/blml > From jfusselman at gmail.com Mon Jan 10 21:23:31 2011 From: jfusselman at gmail.com (Jerry Fusselman) Date: Mon, 10 Jan 2011 14:23:31 -0600 Subject: [BLML] Alain's case revisited [SEC=UNOFFICIAL] In-Reply-To: <4D2B6865.5090205@aol.com> References: <4D2AC556.9070509@skynet.be> <4D2AC9E7.6060208@aol.com> <4D2AE13B.10605@skynet.be> <334994.97415.qm@web28503.mail.ukl.yahoo.com> <4D2B0857.7050804@skynet.be> <683970.34112.qm@web28508.mail.ukl.yahoo.com> <4D2B1CBD.6030301@skynet.be> <596816.91301.qm@web28509.mail.ukl.yahoo.com> <4D2B6865.5090205@aol.com> Message-ID: On Mon, Jan 10, 2011 at 2:13 PM, Jeff Easterson wrote: > > I haven't been following this thread closely so don't understand the > ominous "sentence" to which they keep referring. ("Please write down the > sentence.") ?What sentence is this? > Here you go, Jeff: [Richard] If one ethical partner unintentionally infracts Law 21B1(a), thus preventing the later application of Law 75A, the Director takes any advantage by adjusting the offending side's score from +110 to -800. [Herman] Richard, do me a favour. Please write the sentence explaining to Alain and Patrick your ruling in giving them -800, including that you are ruling on the use of UI which has never been given at the table. Just write the sentence and then think to yourself "do I really want to ask TDs to make such a ruling?". From richard.hills at immi.gov.au Mon Jan 10 21:40:03 2011 From: richard.hills at immi.gov.au (richard.hills at immi.gov.au) Date: Tue, 11 Jan 2011 07:40:03 +1100 Subject: [BLML] Alain's case revisited [SEC=UNOFFICIAL] In-Reply-To: <000801cbb0e0$7461cb00$5d256100$@no> Message-ID: Alain Gottcheiner: [snip] >>Up to now, I've heard comments like "unlawful", "eccentric", >>"ch***ing", but never "illogical". Sven Pran: >Why then not consider the "best" law to be that the player >making a call explains it him- (or her-) self? That would >ensure that opponents always receive correct information. > >This reminds me of a tale from an American bridge club back >in Culbertson's days: A drop-in player asked if they used >asking bids (recently added to Culbertson's system) and the >answer was "yes, we use asking bids". > >A little later was heard the following 'asking bid': "Do you >have the King of Diamonds, partner?" > >Why not allow players to ask "how many aces do you hold >partner?" instead of bidding 4NT? [snip] You, South, hold: 9873 K32 873 J32 Very logical convention described by George S. Kaufman: "North opens with a club, and South responds with one no- trump. North now bids two no-trump, which in my system calls for the showing of deuces and treys. So South bids three diamonds, showing two deuces. North now bids three no-trump, which is an invitation bid asking for treys. South, having all four treys, bids four no-trump, and the contract is doubled and set eleven hundred. This is the only system under which this result can be obtained." Jack Olsen, The Mad World of Bridge, page 82: "Taking the Kaufman convention as a starter and adding to it, one can easily see how one's holdings of fours and fives could then be shown, then sixes and sevens, and so on up to kings and aces, simply by a series of asking bids. Five no- trump would be an invitation bid asking for fours, six no- trump would ask for fives, and so on up to fourteen no- trump, which would be the asking bid for aces. Who could deny the accuracy with which one could determine one's partner's holding after such a series of Kaufman bids?" Best wishes Richard Hills Recruitment Section Specialist Recruitment Team Level 5 Aqua, workstation W569, 6223 8453 DIAC Social Club movie tickets -------------------------------------------------------------------- Important Notice: If you have received this email by mistake, please advise the sender and delete the message and attachments immediately. This email, including attachments, may contain confidential, sensitive, legally privileged and/or copyright information. Any review, retransmission, dissemination or other use of this information by persons or entities other than the intended recipient is prohibited. DIAC respects your privacy and has obligations under the Privacy Act 1988. The official departmental privacy policy can be viewed on the department's website at www.immi.gov.au. See: http://www.immi.gov.au/functional/privacy.htm --------------------------------------------------------------------- From richard.hills at immi.gov.au Mon Jan 10 21:56:40 2011 From: richard.hills at immi.gov.au (richard.hills at immi.gov.au) Date: Tue, 11 Jan 2011 07:56:40 +1100 Subject: [BLML] A Linnaean Treatise on Kibitzers [SEC=UNOFFICIAL] Message-ID: William Dodd, 1934 Bridge World, A Linnaean Treatise on Kibitzers: [snip] "At a recent tournament there were 53 tables in play with 246 kibitzers to be taken care of. Four a while the situation looked hopeless, but finally the tournament manager showed why he was a tournament manager by announcing that four official kibitzers would be assigned to each table. In order to be absolutely fair, credentials were demanded and assignments made on a length-of- service basis. Not to be outdone, the remaining 34 kibitzers got together and prevailed upon four of their number to make up another table. This arrangement left 30 of them to kibitz the special table. This tournament occurred about a week ago and I just learned today that the 34 kibitzers were still there, that they had played one hand and hoped to make a final post-mortem on it by tomorrow at the latest." Best wishes Richard Hills Recruitment Section Specialist Recruitment Team Level 5 Aqua, workstation W569, 6223 8453 DIAC Social Club movie tickets -------------------------------------------------------------------- Important Notice: If you have received this email by mistake, please advise the sender and delete the message and attachments immediately. This email, including attachments, may contain confidential, sensitive, legally privileged and/or copyright information. Any review, retransmission, dissemination or other use of this information by persons or entities other than the intended recipient is prohibited. DIAC respects your privacy and has obligations under the Privacy Act 1988. The official departmental privacy policy can be viewed on the department's website at www.immi.gov.au. See: http://www.immi.gov.au/functional/privacy.htm --------------------------------------------------------------------- From ehaa at starpower.net Mon Jan 10 22:56:11 2011 From: ehaa at starpower.net (Eric Landau) Date: Mon, 10 Jan 2011 16:56:11 -0500 Subject: [BLML] Alain's case revisited In-Reply-To: References: <4D257698.6010201@skynet.be> <9647244.332577.1294302281035.JavaMail.ngmail@webmail17.arcor-online.net> <0F0A4E0E-3319-4F02-8EC6-F49CA1492683@starpower.net> Message-ID: On Jan 8, 2011, at 5:00 PM, Jerry Fusselman wrote: > I want to give a clarification and request a clarification. In brief, > I am ignoring clubs and penalty points right now, and I want to know > how Eric (or someone who agrees with Eric's post) would rule if his > innocent newbie unconsciously used the same UI to take the same > improper action as a conscious cheat. > > On Fri, Jan 7, 2011 at 4:11 PM, Eric Landau wrote: > >> What some people call "reading minds", others call "good >> communication skills", and still others, bridge club directors >> especially, call "knowing their customers". > > Here, and later, Eric refers to bridge club directors. I would rather > leave clubs entirely out of the discussion: For my post, I was > thinking only of events run in (using ACBL names) sectionals, > regionals, and nationals. Almost anything can happen in clubs, and > frequently does. Lets face it: Clubs run by their own rules. I want > to hear if what I said is right in the domain of sectionals, > regionals, and nationals, for I consider clubs a separate issue. > >> What Jerry calls "our best bridge laws" are precisely those that >> require us to give identical rulings to those whom we know to be >> innocent newbies as to confirmed cheaters. > > Eric, are you thinking that the requirement you mention here is a bad > thing? For example, please consider the noncompetitive auction > > 1S-2S > 3C-...3S > 4S. > > I.e., there was a slow 3S signoff after the 3C game try. It gets > raised to 4S anyway by a hand that some players would have taken > straight to game (instead of 3C), but no one would tried for a slam. > Please also assume that the slow signoff demonstrably shows extras. > In the play, it easily makes ten tricks, as it should in just about > all cases, and both sides play it well. > > This happens at two tables---one with a confirmed cheater bidding 4S, > and another with a confirmed innocent newbie bidding 4S. Do you want > the rectifications in these two cases to be the same or different? I would hope to see vast differences in the processes by which the rectification is determined, notwithstanding that I would expect the resulting rectifications to be the same in almost every case. > (I > am not asking about penalty points here, only the rectification.) If > it helps to reach a decision, please assume that the confirmed cheater > consciously used the delay to upgrade his hand, but the confirmed > innocent newbie's upgrade caused by the pause was entirely > unconscious. And please assume also that the director knows all of > this perfectly. How should he rule in the two cases if you had the > laws arranged the way you think best? Does my choice, +170/-170 in > both cases, match yours? Yes. But note that you have forced me to this position by stipuation: "the confirmed innocent newbie's upgrade caused by the pause was entirely unconscious." It is, after all, the "upgrade caused by the pause" that is to be rectified, "entirely unconscious" or not. But when it comes to the sort of understanding of the game of bridge that Jerry and I and I'd assume all of us take for granted, things are often not so clear; I have discovered in my half-century of competitive bridge playing that "confirmed innocent newbies" think and act in all manner of odd and amazing ways. So upon being confronted with Jerry's scenario, I would use my not insignificant (albeit far from perfect) "mind reading" skills by interrogating the newbie as to why he bid 3C, and then 4S, and listening to their answers, before accepting the conlusion that there was an "upgrade cause by the pause" that happened to be, irrelevantly, as Jerry is "not asking about penalty points here, only the rectification", "entirely unconscious". Then, having done so, I give him +170 and a gentle education on his responsibilities after his partner pauses at a critical point in the auction. But it is not beyond the realm of possibility, or even of my own experience, that this innocent newbie, unlike the confirmed cheater, could wind up convincing me that he was never ever stopping short of game no matter what, had his own, not necessarily sensible (to Jerry or me), reason for his 3C bid, and that, *for him*, not bidding game was not an LA. >> And because our clubs >> (and our SOs) are businesses, that means being far too tolerant to >> the cheaters rather than driving the innocent infractors away (and >> having to listen to those in this forum who constantly carp about how >> our laws let people get away with things they shouldn't). Only the >> real-life lawyers among our players seem to actually understand that >> subtle bit about being punished for doing just what a cheater would >> do but, of course, we're not calling you cheaters, we'd never ever do >> that, no sirree, so do please come back next week. >> >> If we had genuinely psychic directors who actually could read >> players' minds, wouldn't we want to write our laws to take advantage >> of those skills, and expect our game to be better for it? If so, >> shouldn't we work to hone our communications skills and our knowledge >> of our customers, write laws that take advantage of those (admittedly >> somewhat less impressive, but not entirely useless) skills, and do >> our best to come as close as we can to our ideal of perfection? > > I plan to respond to this later. I suspect that we have a conflict of > visions about directors and players. I suspect more a conflict of interpretation of all of those "could have"s and "might have"s scattered throughout TFLB. Do we take those as determinations of conceivability (possibly leading to, "He didn't, but he could have, so we rule against him") or as determinations of finite probability (possibly leading to, "He could have, but he didn't, so we rule in his favor")? Jerry seems to favor the former interpretation, while I would prefer the latter, notwithstanding the difficulty of operationalizing "he didn't" for us non-psychics. Eric Landau 1107 Dale Drive Silver Spring MD 20910 ehaa at starpower.net From richard.hills at immi.gov.au Mon Jan 10 23:50:23 2011 From: richard.hills at immi.gov.au (richard.hills at immi.gov.au) Date: Tue, 11 Jan 2011 09:50:23 +1100 Subject: [BLML] Ecclesiastes [SEC=UNOFFICIAL] Message-ID: Ecclesiastes 4:12 A threefold cord is not quickly broken. Richard Hills: >>>..... >>>This is because genius-level blmlers are much better at >>>creating ..... moving-the-goalposts ..... arguments than >>>simpletons could ever achieve. Herman De Wael: >>..... >>I am trying to argue a point, and all Richard is doing is >>insult me and repeat points made previously. >>Richard is not trying to listen. Richard Hills: Of course I am not listening to Herman's attempt to move the goalposts to a new argument when Herman has not yet broken the threefold cord of my previous points: (1) The relevant authority of the WBF Laws Committee as ratified by the WBF Executive. (2) The relevant equity that an abider-by-the-rules should not often be disadvantaged by a breaker-of-the-rules. (3) The relevant necessary complexity of the rules because Duplicate Bridge is "a thinking game" (i.e. a complex game whose attraction rests upon it being a partnership game of imperfect information, hence its rules have to be more detailed and nuanced than 99% of the other mind games listed upon the website BoardGameGeek). Jeff Easterson: >Yes, I noticed but I didn't notice that he insulted you. >At least no more than the two of you "insult" each other >regularly in controversies. Are you perhaps a bit over- >sensitive? Ciao, JE Richard Hills: I apologise for implying Herman is a genius-level blmler. :-) :-) Ciao, RH -------------------------------------------------------------------- Important Notice: If you have received this email by mistake, please advise the sender and delete the message and attachments immediately. This email, including attachments, may contain confidential, sensitive, legally privileged and/or copyright information. Any review, retransmission, dissemination or other use of this information by persons or entities other than the intended recipient is prohibited. DIAC respects your privacy and has obligations under the Privacy Act 1988. The official departmental privacy policy can be viewed on the department's website at www.immi.gov.au. See: http://www.immi.gov.au/functional/privacy.htm --------------------------------------------------------------------- From richard.hills at immi.gov.au Tue Jan 11 00:49:16 2011 From: richard.hills at immi.gov.au (richard.hills at immi.gov.au) Date: Tue, 11 Jan 2011 10:49:16 +1100 Subject: [BLML] Ecclesiastes [SEC=UNOFFICIAL] In-Reply-To: Message-ID: Ecclesiastes 9:11 I returned and saw under the sun, that the race is not to the swift, nor the battle to the strong, neither yet bread to the wise, nor yet riches to men of understanding, nor yet favour to men of skill; but time and chance happeneth to them all. George Orwell, Politics and the English Language (1946) [snip] Here it is in modern English: Objective considerations of contemporary phenomena compel the conclusion that success or failure in competitive activities exhibits no tendency to be commensurate with innate capacity, but that a considerable element of the unpredictable must invariably be taken into account. [snip] Now analyse these two sentences a little more closely. The first contains forty-nine words but only sixty syllables, and all its words are those of everyday life. The second contains thirty-eight words of ninety syllables: eighteen of those words are from Latin roots, and one from Greek. The first sentence contains six vivid images, and only one phrase ("time and chance") that could be called vague. The second contains not a single fresh, arresting phrase, and in spite of its ninety syllables it gives only a shortened version of the meaning contained in the first. [snip] David Burn, 13th February 2008: [snip] Of one thing I am sure: it is not good enough for those who make the Laws to say, ex cathedra, that it is obvious what those Laws mean. In truth, that wasn't good enough even when Kaplan was Archbishop; as recent correspondence has made clear, if bridge really is to be a global game, the Lawmakers ought to listen very hard to people such as Robert Geller, who does not know how to translate the Laws into Japanese because he does not know what they mean in English. And the Lawmakers ought also to listen to Herman de Wael. David Burn London, England -------------------------------------------------------------------- Important Notice: If you have received this email by mistake, please advise the sender and delete the message and attachments immediately. This email, including attachments, may contain confidential, sensitive, legally privileged and/or copyright information. Any review, retransmission, dissemination or other use of this information by persons or entities other than the intended recipient is prohibited. DIAC respects your privacy and has obligations under the Privacy Act 1988. The official departmental privacy policy can be viewed on the department's website at www.immi.gov.au. See: http://www.immi.gov.au/functional/privacy.htm --------------------------------------------------------------------- From richard.hills at immi.gov.au Tue Jan 11 05:33:47 2011 From: richard.hills at immi.gov.au (richard.hills at immi.gov.au) Date: Tue, 11 Jan 2011 15:33:47 +1100 Subject: [BLML] EBU L&EC meeting 3rd November 2010 [SEC=UNOFFICIAL] Message-ID: Felicitations to blmler Gordon Rainsford on his twin promotions. Best wishes Richard Hills Recruitment Section Specialist Recruitment Team Level 5 Aqua, workstation W569, 6223 8453 DIAC Social Club movie tickets http://www.ebu.co.uk/general/frontpage/minutes.htm 1C1 The Chairman proposed that David Burn be co-opted to the committee for the year 2010-2011. He would not have voting rights. Carried nem con. 1C2 The Chairman proposed that Gordon Rainsford be invited to attend L&E meetings from now on in his new role as Assistant Chief TD elect. Carried nem con. Gordon was welcomed to the meeting and congratulated on his appointment to the post from April 1st 2011. 3.2 Appeal from Sheffield Bridge Club The committee considered an Appeal to National Authority from a judgement ruling at Sheffield Bridge Club. The case centred around whether there had been an "unmistakable hesitation" and what constituted one. In the case one player thought there had been and an opponent thought there had not. The other two were non- committal. The TD had made a judgement and ruled accordingly. The club had held an appeal and confirmed the TD's ruling. The member asserted that a) there was a question of principle and b) an error in tournament direction and therefore the appeal should be allowed to be heard. The Committee considered the four grounds for hearing the appeal as outlined in 8B4 of the Orange Book and declined to hear it because: In their opinion There was no question of principle involved. The word "unmistakable" in the phrase unmistakable hesitation means one that has happened in the view of the tournament director. He may obtain evidence as to this from the comments of the players and/or the actions taken at the table. When the director decides that there has been a hesitation he must also take a view as to whether it was unmistakable. In other words if he concluded it was no more than a flicker it might fail to meet the test but if he determines that it did take place and was of a significant length then it is unmistakable. There was neither an error in the application of the law nor any error of tournament direction. The director asked the appropriate questions and used the correct Law in rendering his judgement. His judgement was not grossly inappropriate. The deposit was therefore forfeited and the appellant would be informed of the decision. ACTION JD 5.10 WBF Minutes The committee noted the minutes of the WBF Laws Committee from two meetings held in Philadelphia. They are attached to these minutes (Appendix A) The Committee noted discussion about "Reveley Rulings" and confirmed the EBU's position that "Reveley Rulings" are not permitted in England. The Committee also considered the minute about Insufficient Bids (Law 27) but declined to make any further comment. 5.11 New code of Laws Mr Endicott said that the first preliminary discussions about the 2017 code of laws was scheduled for about six months time. Any suggestions for law amendments should be sent directly to Grattan. -------------------------------------------------------------------- Important Notice: If you have received this email by mistake, please advise the sender and delete the message and attachments immediately. This email, including attachments, may contain confidential, sensitive, legally privileged and/or copyright information. Any review, retransmission, dissemination or other use of this information by persons or entities other than the intended recipient is prohibited. DIAC respects your privacy and has obligations under the Privacy Act 1988. The official departmental privacy policy can be viewed on the department's website at www.immi.gov.au. See: http://www.immi.gov.au/functional/privacy.htm --------------------------------------------------------------------- From ardelm at optusnet.com.au Tue Jan 11 05:41:17 2011 From: ardelm at optusnet.com.au (Tony Musgrove) Date: Tue, 11 Jan 2011 15:41:17 +1100 Subject: [BLML] EBU L&EC meeting 3rd November 2010 [SEC=UNOFFICIAL] In-Reply-To: References: Message-ID: <201101110441.p0B4fQFs030798@mail01.syd.optusnet.com.au> At 03:33 PM 11/01/2011, you wrote: >Felicitations to blmler Gordon Rainsford on his twin promotions. > >Best wishes > >Richard Hills >Recruitment Section >Specialist Recruitment Team >Level 5 Aqua, workstation W569, 6223 8453 >DIAC Social Club movie tickets > >http://www.ebu.co.uk/general/frontpage/minutes.htm > >1C1 The Chairman proposed that David Burn be co-opted to the >committee for the year 2010-2011. He would not have voting rights. >Carried nem con. > >1C2 The Chairman proposed that Gordon Rainsford be invited to >attend L&E meetings from now on in his new role as Assistant Chief >TD elect. Carried nem con. Gordon was welcomed to the meeting and >congratulated on his appointment to the post from April 1st 2011. cut I hope all those nem cons are not just fooling about, Cheers. Tony (Sydney) From ardelm at optusnet.com.au Tue Jan 11 06:04:22 2011 From: ardelm at optusnet.com.au (Tony Musgrove) Date: Tue, 11 Jan 2011 16:04:22 +1100 Subject: [BLML] Alain's case revisited [SEC=UNOFFICIAL] In-Reply-To: <742678624.21558.1294682412469.JavaMail.ngmail@webmail17.ar cor-online.net> References: <742678624.21558.1294682412469.JavaMail.ngmail@webmail17.arcor-online.net> Message-ID: <201101110504.p0B54VcA029812@mail05.syd.optusnet.com.au> At 05:00 AM 11/01/2011, you wrote: >Herman De Wael wrote: > > Thomas Dehn wrote: > > > Herman De Wael wrote: > > >> OK Nigel, thanks for trying to go along with me, but I have a comment: > > >> > > >> Nigel Guthrie wrote: > > >>> > > >>> [Nigel] > > >>> "You broke the law by failing to alert 2S. (Yes the Beijing > interpretation is > > >>> law, even if few know about it). You admit you did this knowingly and > > >>> deliberately and thereby gained 110. "To restore equity" > (great phrase that!) I > > >>> am adjusting your score to -800 instead. That is the result > that those in your > > >>> predicament might expect if they complied with the law. (You > protest that such a > > >>> law-abiding person would be a "masochist" but...) it is often > against your > > >>> interests to abide by the law. (e.g. to hand in a case full > of money that you > > >>> find) That is no excuse for breaking a law. > > >> > > >> You did not answer this the way I believed you should, so I shall simply > > >> ask the next question the player is going to ask you, as the TD at the > > >> table: > > >> > > >> "How do you arrive at a score of -800?" and I'll ask his next question > > >> if your answer is nothing more than "4SpX-3": > > >> "How can we ever arrive in 4Sp? - I know he does not have spades, and he > > >> knows I have only 4 of them - so how could we ever pass out 4SpX, how > > >> could we even bid them?" > > > > > > That flaw is in the ruling you considered, Herman, not in the > "Bejing interpretation". > > > Indeed 4SpX-3 looks not "at all probable had the irregularity > not occured". > > > Nobody in this thread understood how you arrived at 4SX. > > > > > > 6NTX or 6DX, though ... > > > > > > > Well Thomas, for all our sakes - please enlighten us, and write down the > > sentence. > > > > > > > > Thomas > > > > The mere fact that you don't answer the question shows me why you still > > fail to understand my point. > > Really, try it. > >I don't see the problem, other than that it will >be more than one sentence. Just like a lower court references >the binding decision of a national court, you reference the >binding decision of the WBFLC. And just like a lower court >you should feel free to state in your ruling that you >disagree with that binding decision. > > >There were two irregularities in this board >o the failure to alert 1S >o the failure to alert 2S (1) >Furthermore, from the 2S bid we know that the player >who failed to alert 1S at that point did not correctly >remember system, rather than merely failed to alert. > >(1) L20F5(a) states that "A player whose partner has given a >mistaken explanation may not >correct the error during the auction, nor may he indicate in any manner >that a mistake has been made." >The WBFLC has made a binding decision that L20F5(a) does not apply >in this situation, 2S must be alerted even though that tells partner >that 1S did >not show spades. > >http://www.ecatsbridge.com/Documents/files/WBFInformation/Reports_Minutes/LawsCommitteeMinutes/2008-Beijing.pdf, >page 5. >"There is no infraction when a correct explanation discloses that >partner's prior explanation >was mistaken. The words "nor may he indicate in any manner that a >mistake has been >made" (in Law 20F5(a)) do not refer to compliance with the >overriding requirement of the >laws always to respond to enquiries under Law 20F with correct >explanations of the >partnership understandings. " > >This is an unusual new interpretation of TFLB, but nevertheless, >until the WBFLC has reversed this, I am bound by this decision. >If you want to complain about this interpretation, please complain >to the WBFLC, here is how to contact them. addresses here> > >Furthermore, the WBFLC also has decided that WBFLC decisions >that are hidden in some pdfs on their web page are binding >even though they were never communicated to you >in any official publication, such as an update to the local >translation of TFLB. > > >The score for the offensive side is the most unfavorable >result that was at all probable (L12C1). Note that there is >no "had the irregularity not occurred". Here, the NOS will get the >worst out of >o both 1S and 1S alerted >o only 1S alerted >o neither 1S nor 2S alerted >Furthermore, assessment of probable auctions and plays >is not limited to what occurred at the table. > >Had 2S been alerted as required by the alert regulations, >opener would not have been allowed to use the UI from the unexpected >alert, and >thus would not have been allowed to "wake up" and remember that 1S was T-Walsh >(see L16 B1(a) and the footnote to L16). He would only have been allowed to >"wake up" when the auction itself (rather than the unexpected alert) shows >that something is wrong. This holds regardless of whether opener >actually remembered >the system. I know that it is not 'done' to simply say I agree, but I *do* agree with this, and particularly this paragraph. This is how I rule and I am glad to see it so well stated. Cheers, Tony (Sydney) >Do you have your system notes with you so that I can work out >which subsequent auctions were "at all probable" respectively >"likely had the irregularity not occurred"? > > > >I'm pretty sure I won't arrive at Herman's 4SX-3. > >If I then deem the result to be far away from >equity, I might be inclined to grant a request >by the NOS to waive the rectification, L81C5. > > > >Thomas > >_______________________________________________ >Blml mailing list >Blml at rtflb.org >http://lists.rtflb.org/mailman/listinfo/blml From richard.hills at immi.gov.au Tue Jan 11 06:24:30 2011 From: richard.hills at immi.gov.au (richard.hills at immi.gov.au) Date: Tue, 11 Jan 2011 16:24:30 +1100 Subject: [BLML] EBU L&EC meeting 3rd November 2010 [SEC=UNOFFICIAL] In-Reply-To: Message-ID: 5.11 New code of Laws Mr Endicott said that the first preliminary discussions about the 2017 code of laws was scheduled for about six months time. Any suggestions for law amendments should be sent directly to Grattan. +=+=+=+=+=+=+=+=+=+=+=+=+=+=+=+=+=+=+=+=+=+=+=+=+=+=+=+=+=+=+=+=+ INTRODUCTION TO THE 2018 LAWS OF DUPLICATE BRIDGE The Laws are designed to define correct procedure and to provide an adequate remedy when there is a departure from correct procedure. They are primarily designed not as punishment for irregularities but rather for the rectification of situations where non-offenders may otherwise be damaged. Players should be ready to accept gracefully any rectification or adjusted score awarded by the Director. There have been many developments in Duplicate Bridge over the last ten years and there are no signs that these changes have stopped. The task that confronted the Drafting Committee was to ensure the Laws were clarified so as to fix past ambiguities; and also to establish principles that can give guidance, rather than to retain prescriptive details (see, for example, the abolition of Penalty Cards). For the avoidance of doubt, this Introduction and the Definitions that follow form part of the Laws. For the further avoidance of further doubt, these are the principles regarding the creation of misinformation (MI), and the use of unauthorised information (use-of-UI) -> 1. It is an over-riding principle there should not be MI. It does not matter if there is consequential creation of UI. For further information on MI, see Law 40. 2. Bridge is a thinking game, so therefore unintentional creation of UI is not an infraction. 3. On the other hand, use-of-UI (selection from amongst logical alternatives one that is demonstrably suggested by the unauthorised information) is always an infraction. For the definition of "logical alternative" see Law 16. Example: For what would have been a Penalty Card ruling under the previous 2007 Lawbook, the partner of the defender who exposed the now non-penalty card must carefully avoid taking any advantage from that unauthorised information. Directors again have been given considerably more discretionary powers. There are fewer automatic rectifications (in addition to the abolition of Penalty Cards there are no longer any other automatic cardplay restrictions). Bridge is played in different ways in different countries so the Laws give more power to Regulating Authorities to make controlling regulations. Artificial bidding is a fact of life so an attempt has been made to solve problems, or to allow Regulating Authorities to solve problems, that arise when something goes wrong. We have tried to clarify the ruling idea that "a specific Law over-rides a more general Law" by including cross-references when it is unclear which of two Laws is more specific. The Table of Contents which was absent in the 2007 Lawbook has been restored in this 2018 Lawbook. Many thanks to Rick Assad for preparing the Index (but the Index lacks any Lawful power). Where headings remain they do not limit the application of any law, nor indeed does the omission of a cross-reference. Established usage has been retained in regard to "may" do (failure to do it is not wrong), "does" (establishes correct procedure without suggesting that violation be penalised) "should" do (failure to do it is an infraction jeopardising the infractor's rights but not often penalised), "shall" do (a violation will incur a procedural penalty more often than not), "must" do (the strongest word, a serious matter indeed). Again, "must not" is the strongest prohibition, "shall not" is strong but "may not" is stronger ? just short of "must not". Finally, unless the context clearly dictates otherwise, the singular includes the plural and the feminine includes the masculine, and vice versa. Best wishes Richard Hills Recruitment Section Specialist Recruitment Team Level 5 Aqua, workstation W569, 6223 8453 DIAC Social Club movie tickets -------------------------------------------------------------------- Important Notice: If you have received this email by mistake, please advise the sender and delete the message and attachments immediately. This email, including attachments, may contain confidential, sensitive, legally privileged and/or copyright information. Any review, retransmission, dissemination or other use of this information by persons or entities other than the intended recipient is prohibited. DIAC respects your privacy and has obligations under the Privacy Act 1988. The official departmental privacy policy can be viewed on the department's website at www.immi.gov.au. See: http://www.immi.gov.au/functional/privacy.htm --------------------------------------------------------------------- From richard.hills at immi.gov.au Tue Jan 11 07:13:34 2011 From: richard.hills at immi.gov.au (richard.hills at immi.gov.au) Date: Tue, 11 Jan 2011 17:13:34 +1100 Subject: [BLML] Nigel's case supported [SEC=UNOFFICIAL] Message-ID: Nigel Guthrie: >..... >Anyway the answer is not to break the laws but to persuade law- >makers to change them. > >If a group dislike the law but law-makers are intransigent, Richard Hills: Not "intransigent" but "cognisant of agamous precentor De Wael". :-) :-) All joking/insults aside, there is a 0.00% chance that the De Wael loophole will be restored by the "intransigent" WBF LC. Nigel Guthrie: >then that group can just invent a new game with their own rules >-- as the ACBL seem to have already done :) > >But I don't think they should call that game "Bridge". >..... Richard Hills: Harold Vanderbilt in 1925 carefully drew a distinction between the progenitor game of Auction Bridge and his descendant game of Contract Bridge. What's the problem? Is the problem the unprecedented fact Nigel and Richard agree? :-) :-) Best wishes Richard Hills Recruitment Section Specialist Recruitment Team Level 5 Aqua, workstation W569, 6223 8453 DIAC Social Club movie tickets -------------------------------------------------------------------- Important Notice: If you have received this email by mistake, please advise the sender and delete the message and attachments immediately. This email, including attachments, may contain confidential, sensitive, legally privileged and/or copyright information. Any review, retransmission, dissemination or other use of this information by persons or entities other than the intended recipient is prohibited. DIAC respects your privacy and has obligations under the Privacy Act 1988. The official departmental privacy policy can be viewed on the department's website at www.immi.gov.au. See: http://www.immi.gov.au/functional/privacy.htm --------------------------------------------------------------------- From blml at arcor.de Tue Jan 11 08:03:18 2011 From: blml at arcor.de (Thomas Dehn) Date: Tue, 11 Jan 2011 08:03:18 +0100 (CET) Subject: [BLML] Ecclesiastes [SEC=UNOFFICIAL] Message-ID: <1740619554.32853.1294729398892.JavaMail.ngmail@webmail18.arcor-online.net> richard.hills at immi.gov.au wrote: > Richard Hills: > > >>>..... > >>>This is because genius-level blmlers are much better at > >>>creating ..... moving-the-goalposts ..... arguments than > >>>simpletons could ever achieve. > > Herman De Wael: > > >>..... > >>I am trying to argue a point, and all Richard is doing is > >>insult me and repeat points made previously. > >>Richard is not trying to listen. > > Richard Hills: > > Of course I am not listening to Herman's attempt to move the > goalposts to a new argument when Herman has not yet broken > the threefold cord of my previous points: > > (1) The relevant authority of the WBF Laws Committee as > ratified by the WBF Executive. > > (2) The relevant equity that an abider-by-the-rules should > not often be disadvantaged by a breaker-of-the-rules. > > (3) The relevant necessary complexity of the rules because > Duplicate Bridge is "a thinking game" (i.e. a complex > game whose attraction rests upon it being a partnership > game of imperfect information, hence its rules have to > be more detailed and nuanced than 99% of the other mind > games listed upon the website BoardGameGeek). Richard, with respect to (1), in post World War II Europe a person is not accepted as an authority merely because he or she was appointed to some post. An entity like the WBFLC gains authority by making good decisions in a timely fashion, if it communicates those decisions such that the majority of players understands and approves of the reasoning behind those decisions. As long as the WBFLC just states that "the WBFLC is the ultimate authority and our decisions are binding", many Europeans will just conclude that the decisions by the WBFLC was not based on any reasoning that can be communicated to other people. With respect to (2), that is the grief. An abider-by-the-rules is put into a very bad situation by this. Thomas From bmeadows666 at gmail.com Tue Jan 11 09:11:15 2011 From: bmeadows666 at gmail.com (Brian) Date: Tue, 11 Jan 2011 03:11:15 -0500 Subject: [BLML] Ecclesiastes [SEC=UNOFFICIAL] In-Reply-To: <1740619554.32853.1294729398892.JavaMail.ngmail@webmail18.arcor-online.net> References: <1740619554.32853.1294729398892.JavaMail.ngmail@webmail18.arcor-online.net> Message-ID: <4D2C10A3.2070408@gmail.com> On 01/11/2011 02:03 AM, Thomas Dehn wrote: > > with respect to (1), in post World War II Europe a person > is not accepted as an authority merely because he or she > was appointed to some post. An entity like the > WBFLC gains authority by making good decisions > in a timely fashion, if it communicates those decisions > such that the majority of players understands and > approves of the reasoning behind those decisions. > I couldn't agree more - and the part about communication goes beyond the WBFLC. As long as the game's hierarchy persist in their view that there is effective communication between the different levels of the game, from the grass roots players and humble club TD right the way through national and zonal levels to the WBFLC, the aims stated in your second sentence will never be achieved. Before Grattan protests that it's the zonal organizations who are the constituent members of the WBF, *it doesn't matter*. If the lines of communication are broken at some point, the level at which the break occurs is largely irrelevant to those at the lowest levels. The WBF will continue to be seen as some aloof and uncontactable body handing down their ex-cathedra judgments. Of course I don't believe that I know enough about bridge law to teach the WBFLC how to make the next set of laws. I sure as hell DO believe that I know how to improve communications in the game, though. Regrettably, I don't see the changes happening any time soon. Unless and until they do, nothing will change, and the WBFLC will never gain the type of authority you suggest with the players. Brian. From Hermandw at skynet.be Tue Jan 11 09:52:17 2011 From: Hermandw at skynet.be (Herman De Wael) Date: Tue, 11 Jan 2011 09:52:17 +0100 Subject: [BLML] Alain's case revisited [SEC=UNOFFICIAL] In-Reply-To: <4D2B3AA9.7050100@ulb.ac.be> References: <4D2ABE9D.1040102@skynet.be> <838975.24149.qm@web28514.mail.ukl.yahoo.com> <4D2B27EC.5050507@ulb.ac.be> <000801cbb0e0$7461cb00$5d256100$@no> <4D2B3AA9.7050100@ulb.ac.be> Message-ID: <4D2C1A41.7050203@skynet.be> Alain Gottcheiner wrote: >> Why then not consider the "best" law to be that the player making a call >> explains it him- (or her-) self? That would ensure that opponents always >> receive correct information. > AG : of course ! You only need 4 little notebooks. The player making the > bid writes the meaning down on one's own notebook, then shows it to both > opponents in turn if they want to know. Kind of generalizing the > behind-screen procedure. > > I suppose that the reason why the procedure wasn't implemented is that > it would be ratehr slow. Because, to avoid giving away UI that your bid > is artificial, you'd need to scribble something in all cases. > But indeed, in a world not governed by time, this would be by far the > best method. > No it wouldn't, because this method would make misunderstandings available to opponents. -- Herman De Wael Wilrijk Antwerpen Belgium From Hermandw at skynet.be Tue Jan 11 09:58:50 2011 From: Hermandw at skynet.be (Herman De Wael) Date: Tue, 11 Jan 2011 09:58:50 +0100 Subject: [BLML] Alain's case revisited [SEC=UNOFFICIAL] In-Reply-To: References: <4D2AC556.9070509@skynet.be> <4D2AC9E7.6060208@aol.com> <4D2AE13B.10605@skynet.be> <334994.97415.qm@web28503.mail.ukl.yahoo.com> <4D2B0857.7050804@skynet.be> <683970.34112.qm@web28508.mail.ukl.yahoo.com> <4D2B1CBD.6030301@skynet.be> <596816.91301.qm@web28509.mail.ukl.yahoo.com> <4D2B6865.5090205@aol.com> Message-ID: <4D2C1BCA.6020404@skynet.be> Jerry Fusselman wrote: > On Mon, Jan 10, 2011 at 2:13 PM, Jeff Easterson wrote: >> >> I haven't been following this thread closely so don't understand the >> ominous "sentence" to which they keep referring. ("Please write down the >> sentence.") What sentence is this? >> > > Here you go, Jeff: > > [Richard] > > If one ethical partner unintentionally infracts Law 21B1(a), > thus preventing the later application of Law 75A, the > Director takes any advantage by adjusting the offending > side's score from +110 to -800. > Really Richard? Do you think it is that simple? Why -800, why not -100 or -37625? -- Herman De Wael Wilrijk Antwerpen Belgium From Hermandw at skynet.be Tue Jan 11 10:01:19 2011 From: Hermandw at skynet.be (Herman De Wael) Date: Tue, 11 Jan 2011 10:01:19 +0100 Subject: [BLML] Ecclesiastes [SEC=UNOFFICIAL] In-Reply-To: References: Message-ID: <4D2C1C5F.7030405@skynet.be> Richard is deliberately being obtuse. richard.hills at immi.gov.au wrote: > > Ecclesiastes 4:12 > > A threefold cord is not quickly broken. > > Richard Hills: > >>>> ..... >>>> This is because genius-level blmlers are much better at >>>> creating ..... moving-the-goalposts ..... arguments than >>>> simpletons could ever achieve. > > Herman De Wael: > >>> ..... >>> I am trying to argue a point, and all Richard is doing is >>> insult me and repeat points made previously. >>> Richard is not trying to listen. > > Richard Hills: > > Of course I am not listening to Herman's attempt to move the > goalposts to a new argument when Herman has not yet broken > the threefold cord of my previous points: > > (1) The relevant authority of the WBF Laws Committee as > ratified by the WBF Executive. > Yes. > (2) The relevant equity that an abider-by-the-rules should > not often be disadvantaged by a breaker-of-the-rules. > Yes. > (3) The relevant necessary complexity of the rules because > Duplicate Bridge is "a thinking game" (i.e. a complex > game whose attraction rests upon it being a partnership > game of imperfect information, hence its rules have to > be more detailed and nuanced than 99% of the other mind > games listed upon the website BoardGameGeek). > What? I am trying to tell Richard that the laws as the WBF promulgate them are less "right" than the ones I would propose. And Richard answers that the WBF sets the laws. Why do I bother? -- Herman De Wael Wilrijk Antwerpen Belgium From Hermandw at skynet.be Tue Jan 11 10:06:54 2011 From: Hermandw at skynet.be (Herman De Wael) Date: Tue, 11 Jan 2011 10:06:54 +0100 Subject: [BLML] Alain's case revisited [SEC=UNOFFICIAL] In-Reply-To: <742678624.21558.1294682412469.JavaMail.ngmail@webmail17.arcor-online.net> References: <742678624.21558.1294682412469.JavaMail.ngmail@webmail17.arcor-online.net> Message-ID: <4D2C1DAE.8060000@skynet.be> Thomas does what I asked: Thomas Dehn wrote: > > Had 2S been alerted as required by the alert regulations, > opener would not have been allowed to use the UI from the unexpected alert, and > thus would not have been allowed to "wake up" and remember that 1S was T-Walsh > (see L16 B1(a) and the footnote to L16). He would only have been allowed to > "wake up" when the auction itself (rather than the unexpected alert) shows > that something is wrong. This holds regardless of whether opener actually remembered > the system. > > Do you have your system notes with you so that I can work out > which subsequent auctions were "at all probable" respectively > "likely had the irregularity not occurred"? > > > > I'm pretty sure I won't arrive at Herman's 4SX-3. > That doesn't matter. I said this might be a result and someone then suggested the AS ought to be -800. I am merely asking to write down the verdict that arrives at that score. Thomas writes down the sentence using "(Patrick) would not have been allowed to wake up" as the core phrase. Now please tell me Thomas, how you intend to explain to Patrick that he would not be allowed to wake up because of UI, when he in fact woke up without it? > If I then deem the result to be far away from > equity, I might be inclined to grant a request > by the NOS to waive the rectification, L81C5. > > whyever would you do that? To spare innocent infractors and only leave Alain with a bad score? > > Thomas -- Herman De Wael Wilrijk Antwerpen Belgium From Hermandw at skynet.be Tue Jan 11 10:10:09 2011 From: Hermandw at skynet.be (Herman De Wael) Date: Tue, 11 Jan 2011 10:10:09 +0100 Subject: [BLML] Nigel's case supported [SEC=UNOFFICIAL] In-Reply-To: References: Message-ID: <4D2C1E71.7060004@skynet.be> richard.hills at immi.gov.au wrote: > > Nigel Guthrie: > >> ..... >> Anyway the answer is not to break the laws but to persuade law- >> makers to change them. >> >> If a group dislike the law but law-makers are intransigent, > > Richard Hills: > > Not "intransigent" but "cognisant of agamous precentor De Wael". > > :-) :-) > > All joking/insults aside, there is a 0.00% chance that the De > Wael loophole will be restored by the "intransigent" WBF LC. > And why do you so categorically state this? 0.00% is very little. Make it 0.001% and I'll wager 1 whatever on it, 100000 to 1. > Nigel Guthrie: > >> then that group can just invent a new game with their own rules >> -- as the ACBL seem to have already done :) >> >> But I don't think they should call that game "Bridge". >> ..... > > Richard Hills: > > Harold Vanderbilt in 1925 carefully drew a distinction between > the progenitor game of Auction Bridge and his descendant game > of Contract Bridge. > > What's the problem? > Is the problem the unprecedented fact Nigel and Richard agree? > > :-) :-) > > Best wishes > > Richard Hills > Recruitment Section > Specialist Recruitment Team > Level 5 Aqua, workstation W569, 6223 8453 > DIAC Social Club movie tickets > > > > > -------------------------------------------------------------------- > Important Notice: If you have received this email by mistake, please advise > the sender and delete the message and attachments immediately. This email, > including attachments, may contain confidential, sensitive, legally privileged > and/or copyright information. Any review, retransmission, dissemination > or other use of this information by persons or entities other than the > intended recipient is prohibited. DIAC respects your privacy and has > obligations under the Privacy Act 1988. The official departmental privacy > policy can be viewed on the department's website at www.immi.gov.au. See: > http://www.immi.gov.au/functional/privacy.htm > > > --------------------------------------------------------------------- > > _______________________________________________ > Blml mailing list > Blml at rtflb.org > http://lists.rtflb.org/mailman/listinfo/blml > > > > > No virus found in this incoming message. > Checked by AVG - www.avg.com > Version: 9.0.872 / Virus Database: 271.1.1/3371 - Release Date: 01/10/11 08:34:00 > -- Herman De Wael Wilrijk Antwerpen Belgium From Hermandw at skynet.be Tue Jan 11 10:12:33 2011 From: Hermandw at skynet.be (Herman De Wael) Date: Tue, 11 Jan 2011 10:12:33 +0100 Subject: [BLML] Alain's case revisited [SEC=UNOFFICIAL] In-Reply-To: <4D2B6865.5090205@aol.com> References: <4D2AC556.9070509@skynet.be> <4D2AC9E7.6060208@aol.com> <4D2AE13B.10605@skynet.be> <334994.97415.qm@web28503.mail.ukl.yahoo.com> <4D2B0857.7050804@skynet.be> <683970.34112.qm@web28508.mail.ukl.yahoo.com> <4D2B1CBD.6030301@skynet.be> <596816.91301.qm@web28509.mail.ukl.yahoo.com> <4D2B6865.5090205@aol.com> Message-ID: <4D2C1F01.9090505@skynet.be> Jeff Easterson wrote: > Please excuse the previous email (test). I kept trying to send this > email and it kept coming back. > > I haven't been following this thread closely so don't understand the > ominous "sentence" to which they keep referring. ("Please write down the > sentence.") What sentence is this? > The sentence (in its grammatical sense) that describes to the players the way the director arrives at an AS of 4Sp doubled -3 (or any other contract that one would care to give in this situation). I believe many readers on this list still haven't grasped this problem since they can't even see how convoluted the ruling actually is! And I would like to seem someone else try - otherwise you will simply tell me that it's me who's making it convoluted. > Ciao, JE > -- Herman De Wael Wilrijk Antwerpen Belgium From svenpran at online.no Tue Jan 11 10:56:25 2011 From: svenpran at online.no (Sven Pran) Date: Tue, 11 Jan 2011 10:56:25 +0100 Subject: [BLML] Alain's case revisited [SEC=UNOFFICIAL] In-Reply-To: <4D2C1A41.7050203@skynet.be> References: <4D2ABE9D.1040102@skynet.be> <838975.24149.qm@web28514.mail.ukl.yahoo.com> <4D2B27EC.5050507@ulb.ac.be> <000801cbb0e0$7461cb00$5d256100$@no> <4D2B3AA9.7050100@ulb.ac.be> <4D2C1A41.7050203@skynet.be> Message-ID: <000301cbb175$cf9acdb0$6ed06910$@no> On Behalf Of Herman De Wael > > I suppose that the reason why the procedure wasn't implemented is > > that it would be ratehr slow. Because, to avoid giving away UI that > > your bid is artificial, you'd need to scribble something in all cases. > > But indeed, in a world not governed by time, this would be by far the > > best method. > > > > No it wouldn't, because this method would make misunderstandings available to > opponents. And what is so wrong about opponents being informed when you have misunderstandings? From Hermandw at skynet.be Tue Jan 11 10:58:04 2011 From: Hermandw at skynet.be (Herman De Wael) Date: Tue, 11 Jan 2011 10:58:04 +0100 Subject: [BLML] Alain's case revisited [SEC=UNOFFICIAL] In-Reply-To: <000301cbb175$cf9acdb0$6ed06910$@no> References: <4D2ABE9D.1040102@skynet.be> <838975.24149.qm@web28514.mail.ukl.yahoo.com> <4D2B27EC.5050507@ulb.ac.be> <000801cbb0e0$7461cb00$5d256100$@no> <4D2B3AA9.7050100@ulb.ac.be> <4D2C1A41.7050203@skynet.be> <000301cbb175$cf9acdb0$6ed06910$@no> Message-ID: <4D2C29AC.6040701@skynet.be> Sven Pran wrote: > On Behalf Of Herman De Wael >>> I suppose that the reason why the procedure wasn't implemented is >>> that it would be ratehr slow. Because, to avoid giving away UI that >>> your bid is artificial, you'd need to scribble something in all cases. >>> But indeed, in a world not governed by time, this would be by far the >>> best method. >>> >> >> No it wouldn't, because this method would make misunderstandings available > to >> opponents. > > And what is so wrong about opponents being informed when you have > misunderstandings? > Nothing if you're a masochist. This is a topic under discussion. Let's just say that the current laws do NOT provide for an entitlement that there has been a misunderstanding. See L20F5. -- Herman De Wael Wilrijk Antwerpen Belgium From blml at arcor.de Tue Jan 11 11:04:35 2011 From: blml at arcor.de (Thomas Dehn) Date: Tue, 11 Jan 2011 11:04:35 +0100 (CET) Subject: [BLML] Alain's case revisited [SEC=UNOFFICIAL] Message-ID: <1528745143.92703.1294740275052.JavaMail.ngmail@webmail11.arcor-online.net> Herman De Wael wrote: > Thomas does what I asked: > > Thomas Dehn wrote: > > > > Had 2S been alerted as required by the alert regulations, > > opener would not have been allowed to use the UI from the unexpected alert, and > > thus would not have been allowed to "wake up" and remember that 1S was T-Walsh > > (see L16 B1(a) and the footnote to L16). He would only have been allowed to > > "wake up" when the auction itself (rather than the unexpected alert) shows > > that something is wrong. This holds regardless of whether opener actually remembered > > the system. > > > > Do you have your system notes with you so that I can work out > > which subsequent auctions were "at all probable" respectively > > "likely had the irregularity not occurred"? > > > > > > > > I'm pretty sure I won't arrive at Herman's 4SX-3. > > > > That doesn't matter. I said this might be a result and someone then > suggested the AS ought to be -800. I am merely asking to write down the > verdict that arrives at that score. > > Thomas writes down the sentence using "(Patrick) would not have been > allowed to wake up" as the core phrase. > Now please tell me Thomas, how you intend to explain to Patrick that he > would not be allowed to wake up because of UI, when he in fact woke up > without it? I agree that there is a problem there. Sometimes the player wakes up on his own. Sometimes he doesn't. The fact that he woke up this one time does not imply that he always would wake up. Generally, the TD has to evaluate how likely it was that the player would have woken up in this scenario on his own. But players are there to get scores by playing bridge, rather than random gifts and punishment dished out by the TD on behalf of the WBF. I consider this aspect to be an area that touches pretty much anything where the TD assigns a score. It needs to be clarified overall, rather than merely for the dWS scenario. The OS reaches 6H via hesitation blackwood, and makes six when declarer makes a 50-50 guess. It is "at all probable", and maybe even "likely", that he would have guessed wrongly had the hand been played again in 5H or 6H. One can easily read TFLB in many different ways: o both sides get 5H+1 o OS gets 5H just made o OS gets 6H down one I think TFLB fails to state clearly what the expected ruling is in this simple scenario. And as long as that isn't stated clearly in TFLB itself in a way that an average player can understand it without consulting a TD or BLML, it doesn't make much sense to debate how more complex such cases should be handled. Thomas From Hermandw at skynet.be Tue Jan 11 11:16:28 2011 From: Hermandw at skynet.be (Herman De Wael) Date: Tue, 11 Jan 2011 11:16:28 +0100 Subject: [BLML] EBU L&EC meeting 3rd November 2010 [SEC=UNOFFICIAL] In-Reply-To: References: Message-ID: <4D2C2DFC.8030502@skynet.be> richard.hills at immi.gov.au wrote: > > 5.11 New code of Laws > Mr Endicott said that the first preliminary discussions about the > 2017 code of laws was scheduled for about six months time. Any > suggestions for law amendments should be sent directly to Grattan. > I believe that one of the most important discussions the WBF should start with is the question of whether or not the opponents are entitled to the knowledge that there has been a misunderstanding. -- Herman De Wael Wilrijk Antwerpen Belgium From agot at ulb.ac.be Tue Jan 11 11:44:49 2011 From: agot at ulb.ac.be (Alain Gottcheiner) Date: Tue, 11 Jan 2011 11:44:49 +0100 Subject: [BLML] Alain's case revisited [SEC=UNOFFICIAL] In-Reply-To: <4D2C1A41.7050203@skynet.be> References: <4D2ABE9D.1040102@skynet.be> <838975.24149.qm@web28514.mail.ukl.yahoo.com> <4D2B27EC.5050507@ulb.ac.be> <000801cbb0e0$7461cb00$5d256100$@no> <4D2B3AA9.7050100@ulb.ac.be> <4D2C1A41.7050203@skynet.be> Message-ID: <4D2C34A1.2060409@ulb.ac.be> Le 11/01/2011 9:52, Herman De Wael a ?crit : > Alain Gottcheiner wrote: >>> Why then not consider the "best" law to be that the player making a call >>> explains it him- (or her-) self? That would ensure that opponents always >>> receive correct information. >> AG : of course ! You only need 4 little notebooks. The player making the >> bid writes the meaning down on one's own notebook, then shows it to both >> opponents in turn if they want to know. Kind of generalizing the >> behind-screen procedure. >> >> I suppose that the reason why the procedure wasn't implemented is that >> it would be ratehr slow. Because, to avoid giving away UI that your bid >> is artificial, you'd need to scribble something in all cases. >> But indeed, in a world not governed by time, this would be by far the >> best method. >> > No it wouldn't, because this method would make misunderstandings > available to opponents. > I don't understand. I remember that you said yourself that one has the right to know this. From svenpran at online.no Tue Jan 11 13:14:26 2011 From: svenpran at online.no (Sven Pran) Date: Tue, 11 Jan 2011 13:14:26 +0100 Subject: [BLML] Alain's case revisited [SEC=UNOFFICIAL] In-Reply-To: <4D2C29AC.6040701@skynet.be> References: <4D2ABE9D.1040102@skynet.be> <838975.24149.qm@web28514.mail.ukl.yahoo.com> <4D2B27EC.5050507@ulb.ac.be> <000801cbb0e0$7461cb00$5d256100$@no> <4D2B3AA9.7050100@ulb.ac.be> <4D2C1A41.7050203@skynet.be> <000301cbb175$cf9acdb0$6ed06910$@no> <4D2C29AC.6040701@skynet.be> Message-ID: <000701cbb189$173953e0$45abfba0$@no> On Behalf Of Herman De Wael > > And what is so wrong about opponents being informed when you have > > misunderstandings? > > > > Nothing if you're a masochist. > > This is a topic under discussion. > > Let's just say that the current laws do NOT provide for an entitlement that there > has been a misunderstanding. See L20F5. The current laws do indeed provide for an entitlement that there has been a misunderstanding in the form of an initially incorrect explanation, see L20F5(b). (And they do of course provide for redress if such misunderstanding is not revealed in time to avoid opponents becoming damaged.). From blml at arcor.de Tue Jan 11 13:50:48 2011 From: blml at arcor.de (Thomas Dehn) Date: Tue, 11 Jan 2011 13:50:48 +0100 (CET) Subject: [BLML] Alain's case revisited [SEC=UNOFFICIAL] Message-ID: <647204170.104776.1294750248997.JavaMail.ngmail@webmail11.arcor-online.net> Herman De Wael wrote: > Jeff Easterson wrote: > > Please excuse the previous email (test). I kept trying to send this > > email and it kept coming back. > > > > I haven't been following this thread closely so don't understand the > > ominous "sentence" to which they keep referring. ("Please write down the > > sentence.") What sentence is this? > > > > The sentence (in its grammatical sense) that describes to the players > the way the director arrives at an AS of 4Sp doubled -3 (or any other > contract that one would care to give in this situation). > I believe many readers on this list still haven't grasped this problem > since they can't even see how convoluted the ruling actually is! > And I would like to seem someone else try - otherwise you will simply > tell me that it's me who's making it convoluted. While I agree that there are several problems, I don't agree that the dWS solves those problems. In particular, assuming the dWS becomes law, then the second alert is disallowed. You still can get convoluted and confusing ruling in case the player alerts. Joe Jim 1C 1S(1) 2S(2) (1) T-Walsh, alertable per local alert regulations. Not alerted. (2) A reverse. Alertable per local alert regulations, but not alertable per L101 "dWS". Alerted anyways. Subsequently, Jim and Joe bend over backwards to avoid using any UI. However, this is one of those hands where they would have gotten a worse score if Joe had "woken up", which he would have been allowed to do if Jim had not alerted 2S. You judge it "likely" that Joe would have woken up. I wouldn't want to rule on that one. Thomas From richard.willey at gmail.com Tue Jan 11 14:19:22 2011 From: richard.willey at gmail.com (richard willey) Date: Tue, 11 Jan 2011 08:19:22 -0500 Subject: [BLML] Ecclesiastes [SEC=UNOFFICIAL] In-Reply-To: <4D2C1C5F.7030405@skynet.be> References: <4D2C1C5F.7030405@skynet.be> Message-ID: On Tue, Jan 11, 2011 at 4:01 AM, Herman De Wael wrote: > > I am trying to tell Richard that the laws as the WBF promulgate them are > less "right" than the ones I would propose. And Richard answers that the > WBF sets the laws. > > Why do I bother? Because you're an egotistical twit who is more interested in showing off than doing the right thing... > -- > I think back to the halcyon dates of my youth, when indeterminate Hessians > had something to do with the Revolutionary War, where conjugate priors were > monks who had broken their vows, and the expression (X'X)^-1(X'Y) was greek > > Those were simpler times > -------------- next part -------------- An HTML attachment was scrubbed... URL: http://lists.rtflb.org/pipermail/blml/attachments/20110111/7450d81d/attachment.html From agot at ulb.ac.be Tue Jan 11 14:25:35 2011 From: agot at ulb.ac.be (Alain Gottcheiner) Date: Tue, 11 Jan 2011 14:25:35 +0100 Subject: [BLML] Alain's case revisited [SEC=UNOFFICIAL] In-Reply-To: <647204170.104776.1294750248997.JavaMail.ngmail@webmail11.arcor-online.net> References: <647204170.104776.1294750248997.JavaMail.ngmail@webmail11.arcor-online.net> Message-ID: <4D2C5A4F.3070408@ulb.ac.be> Le 11/01/2011 13:50, Thomas Dehn a ?crit : > Herman De Wael wrote: >> Jeff Easterson wrote: >>> Please excuse the previous email (test). I kept trying to send this >>> email and it kept coming back. >>> >>> I haven't been following this thread closely so don't understand the >>> ominous "sentence" to which they keep referring. ("Please write down the >>> sentence.") What sentence is this? >>> >> The sentence (in its grammatical sense) that describes to the players >> the way the director arrives at an AS of 4Sp doubled -3 (or any other >> contract that one would care to give in this situation). >> I believe many readers on this list still haven't grasped this problem >> since they can't even see how convoluted the ruling actually is! >> And I would like to seem someone else try - otherwise you will simply >> tell me that it's me who's making it convoluted. > While I agree that there are several problems, I don't agree > that the dWS solves those problems. In particular, > assuming the dWS becomes law, then the second alert > is disallowed. You still can get convoluted and confusing > ruling in case the player alerts. AG : you're right. While in favor of using dWS when it solves the problem of keeping the deal alive, I don't think it can ever become law. What it can become is a posibility offered to the player, at one's own risks. The new rules about corrected bids act a bit in the same way : avoiding an absurd result on the board that would have strange consequences to other competitors. I would even accept the idea that, by using dWS principles, the player agrees to play for some limited amount of matchpoints, as in this case. The important part is that thir opponents, and everybody else in the room, are playing for the normal amount, and will usually get the normal amount. Best regards Alain From harald.skjaran at gmail.com Tue Jan 11 15:52:38 2011 From: harald.skjaran at gmail.com (=?UTF-8?Q?Harald_Skj=C3=A6ran?=) Date: Tue, 11 Jan 2011 15:52:38 +0100 Subject: [BLML] Ecclesiastes [SEC=UNOFFICIAL] In-Reply-To: References: <4D2C1C5F.7030405@skynet.be> Message-ID: 2011/1/11 richard willey : > On Tue, Jan 11, 2011 at 4:01 AM, Herman De Wael wrote: >> >> I am trying to tell Richard that the laws as the WBF promulgate them are >> less "right" than the ones I would propose. And Richard answers that the >> WBF sets the laws. >> >> Why do I bother? > > > Because you're an egotistical twit who is more interested in showing off > than doing the right thing... Why on earth did you say such a thing? Herman is trying to argue for what he believes is best for bridge. It's a real possibility that he is right about that aspect. The way Richard H. responds, it's like fighting windmills. > > >> >> -- >> I think back to the halcyon dates of my youth, when indeterminate Hessians >> had something to do with the Revolutionary War, where conjugate priors were >> monks who had broken their vows, and the expression (X'X)^-1(X'Y) was greek >> >> Those were simpler times > > _______________________________________________ > Blml mailing list > Blml at rtflb.org > http://lists.rtflb.org/mailman/listinfo/blml > > -- Kind regards, Harald Skj?ran From ehaa at starpower.net Tue Jan 11 16:39:06 2011 From: ehaa at starpower.net (Eric Landau) Date: Tue, 11 Jan 2011 10:39:06 -0500 Subject: [BLML] EBU L&EC meeting 3rd November 2010 In-Reply-To: <4D2C2DFC.8030502@skynet.be> References: <4D2C2DFC.8030502@skynet.be> Message-ID: <99220844-B551-4B55-B5F1-E27A4E76E32B@starpower.net> On Jan 11, 2011, at 5:16 AM, Herman De Wael wrote: > I believe that one of the most important discussions the WBF should > start with is the question of whether or not the opponents are > entitled > to the knowledge that there has been a misunderstanding. I don't think "entitled" is the word Herman is looking for. Nobody (I hope) would propose that partnerships have a positive obligation to inform their opponents that they are having a misunderstanding. Of course, there may be laws that have the effect of making such revelation unavoidable in the specific situations in which they apply (as well as situations in which it which it will be avoidable but will occur nevertheless), and the information is perforce authorized unless the opposing pair has committed some infraction. But if we're writing laws, we need to be careful to distinguished information that is merely "authorized" from that which is "entitled". Eric Landau 1107 Dale Drive Silver Spring MD 20910 ehaa at starpower.net From Hermandw at skynet.be Tue Jan 11 18:12:29 2011 From: Hermandw at skynet.be (Herman De Wael) Date: Tue, 11 Jan 2011 18:12:29 +0100 Subject: [BLML] Alain's case revisited [SEC=UNOFFICIAL] In-Reply-To: <000701cbb189$173953e0$45abfba0$@no> References: <4D2ABE9D.1040102@skynet.be> <838975.24149.qm@web28514.mail.ukl.yahoo.com> <4D2B27EC.5050507@ulb.ac.be> <000801cbb0e0$7461cb00$5d256100$@no> <4D2B3AA9.7050100@ulb.ac.be> <4D2C1A41.7050203@skynet.be> <000301cbb175$cf9acdb0$6ed06910$@no> <4D2C29AC.6040701@skynet.be> <000701cbb189$173953e0$45abfba0$@no> Message-ID: <4D2C8F7D.7040800@skynet.be> Sven Pran wrote: > On Behalf Of Herman De Wael >>> And what is so wrong about opponents being informed when you have >>> misunderstandings? >>> >> >> Nothing if you're a masochist. >> >> This is a topic under discussion. >> >> Let's just say that the current laws do NOT provide for an entitlement > that there >> has been a misunderstanding. See L20F5. > > The current laws do indeed provide for an entitlement that there has been a > misunderstanding in the form of an initially incorrect explanation, see > L20F5(b). > (And they do of course provide for redress if such misunderstanding is not > revealed in time to avoid opponents becoming damaged.). > How can you read this in that manner? The only entitlement that L20F5 gives is the fact that opponents get it before they see the dummy. Don't forget that when the misunderstanding pair does not become declaring side, there is no entitlement towards declarer. OTOH, L20F5 explicitely forbids a misunderstanding side of revealing that misunderstanding to opponents. Talk about non-entitlement! Herman. -- Herman De Wael Wilrijk Antwerpen Belgium From Hermandw at skynet.be Tue Jan 11 18:13:36 2011 From: Hermandw at skynet.be (Herman De Wael) Date: Tue, 11 Jan 2011 18:13:36 +0100 Subject: [BLML] Ecclesiastes [SEC=UNOFFICIAL] In-Reply-To: References: <4D2C1C5F.7030405@skynet.be> Message-ID: <4D2C8FC0.7090107@skynet.be> I demand an apology, or some other action. I do not like being called a twit. Please! richard willey wrote: > On Tue, Jan 11, 2011 at 4:01 AM, Herman De Wael > wrote: > > > I am trying to tell Richard that the laws as the WBF promulgate them are > less "right" than the ones I would propose. And Richard answers that the > WBF sets the laws. > > Why do I bother? > > Because you're an egotistical twit who is more interested in showing off > than doing the right thing... > > -- > I think back to the halcyon dates of my youth, when indeterminate > Hessians had something to do with the Revolutionary War, where > conjugate priors were monks who had broken their vows, and the > expression (X'X)^-1(X'Y) was greek > > Those were simpler times > > > > _______________________________________________ > Blml mailing list > Blml at rtflb.org > http://lists.rtflb.org/mailman/listinfo/blml > > > > > No virus found in this incoming message. > Checked by AVG - www.avg.com > Version: 9.0.872 / Virus Database: 271.1.1/3372 - Release Date: 01/10/11 20:34:00 > -- Herman De Wael Wilrijk Antwerpen Belgium From Hermandw at skynet.be Tue Jan 11 18:18:41 2011 From: Hermandw at skynet.be (Herman De Wael) Date: Tue, 11 Jan 2011 18:18:41 +0100 Subject: [BLML] Alain's case revisited [SEC=UNOFFICIAL] In-Reply-To: <647204170.104776.1294750248997.JavaMail.ngmail@webmail11.arcor-online.net> References: <647204170.104776.1294750248997.JavaMail.ngmail@webmail11.arcor-online.net> Message-ID: <4D2C90F1.40408@skynet.be> Thomas Dehn wrote: > Herman De Wael wrote: >> Jeff Easterson wrote: >>> Please excuse the previous email (test). I kept trying to send this >>> email and it kept coming back. >>> >>> I haven't been following this thread closely so don't understand the >>> ominous "sentence" to which they keep referring. ("Please write down the >>> sentence.") What sentence is this? >>> >> >> The sentence (in its grammatical sense) that describes to the players >> the way the director arrives at an AS of 4Sp doubled -3 (or any other >> contract that one would care to give in this situation). >> I believe many readers on this list still haven't grasped this problem >> since they can't even see how convoluted the ruling actually is! >> And I would like to seem someone else try - otherwise you will simply >> tell me that it's me who's making it convoluted. > > While I agree that there are several problems, I don't agree > that the dWS solves those problems. In particular, > assuming the dWS becomes law, then the second alert > is disallowed. You still can get convoluted and confusing > ruling in case the player alerts. > > Joe Jim > 1C 1S(1) > 2S(2) > > (1) T-Walsh, alertable per local alert regulations. Not alerted. > (2) A reverse. Alertable per local alert regulations, but not alertable per L101 "dWS". Alerted anyways. > > Subsequently, Jim and Joe bend over backwards to avoid using any UI. However, > this is one of those hands where they would have gotten a worse score if Joe had > "woken up", which he would have been allowed to do if Jim had not > alerted 2S. You judge it "likely" that Joe would have woken up. > > I wouldn't want to rule on that one. > > > Thomas > Thomas, you are deliberately making this more difficult than it needs to be. Even in your scenario, there is a simple ruling: UI, per the "disallowed" alert. Now you maintain that, because of UI, a player bends over backwards in a situation in which he would not have needed to? Isn't that something which can happen in any normal UI case as well? After all, this "dWS" rule, is just one about not giving UI. Just like any other one! There is no need to call this more difficult than anything we already know. It is precisely the Beijing rule which is odd - because it obliges a player to give UI to his partner, necessitating a ruling by a Director about "missing UI". Which I have never seen anyone rule, ever! But the "dWS" rule is just the same one as the one you would need if a player broke L20F5. Which is a story we have all seen before: a wrong alert, and a partner saying "you should alert this!". -- Herman De Wael Wilrijk Antwerpen Belgium From nigelguthrie at yahoo.co.uk Tue Jan 11 19:19:49 2011 From: nigelguthrie at yahoo.co.uk (Nigel Guthrie) Date: Tue, 11 Jan 2011 18:19:49 +0000 (GMT) Subject: [BLML] Ecclesiastes [SEC=UNOFFICIAL] In-Reply-To: References: <4D2C1C5F.7030405@skynet.be> Message-ID: <505171.31412.qm@web28513.mail.ukl.yahoo.com> [richard willey] Because you're an egotistical twit who is more interested in showing off than doing the right thing... I think back to the halcyon dates of my youth, when indeterminate Hessians had something to do with the Revolutionary War, where conjugate priors were monks who had broken their vows, and the expression (X'X)^-1(X'Y) was greek. Those were simpler times [Nigel] Most of us web wanderers seem to be on an ego-trip :) From richard.willey at gmail.com Tue Jan 11 20:09:31 2011 From: richard.willey at gmail.com (richard willey) Date: Tue, 11 Jan 2011 14:09:31 -0500 Subject: [BLML] Ecclesiastes [SEC=UNOFFICIAL] In-Reply-To: <4D2C8FC0.7090107@skynet.be> References: <4D2C1C5F.7030405@skynet.be> <4D2C8FC0.7090107@skynet.be> Message-ID: On Tue, Jan 11, 2011 at 12:13 PM, Herman De Wael wrote: > I demand an apology, or some other action. > I do not like being called a twit. > Please! > As I said, an egotistical little twit. If you were actually interested in moving forward, a private email would have sufficed. Instead, you prefer to grandstand, demanding an apology, and praying that the community pays more attention to poor aggrieved little Herman... Maybe, someday, you'll grow up enough to learn that there is "good" attention and "bad" attention... > -- > I think back to the halcyon dates of my youth, when indeterminate Hessians > had something to do with the Revolutionary War, where conjugate priors were > monks who had broken their vows, and the expression (X'X)^-1(X'Y) was greek > > Those were simpler times > -------------- next part -------------- An HTML attachment was scrubbed... URL: http://lists.rtflb.org/pipermail/blml/attachments/20110111/ee7441c2/attachment.html From jfusselman at gmail.com Tue Jan 11 20:19:02 2011 From: jfusselman at gmail.com (Jerry Fusselman) Date: Tue, 11 Jan 2011 13:19:02 -0600 Subject: [BLML] Ecclesiastes [SEC=UNOFFICIAL] In-Reply-To: References: <4D2C1C5F.7030405@skynet.be> <4D2C8FC0.7090107@skynet.be> Message-ID: I find Richard Willey's behavior today appalling. It detracts from BLML. Jerry Fusselman From richard.hills at immi.gov.au Tue Jan 11 22:28:52 2011 From: richard.hills at immi.gov.au (richard.hills at immi.gov.au) Date: Wed, 12 Jan 2011 08:28:52 +1100 Subject: [BLML] Ecclesiastes [SEC=UNOFFICIAL] In-Reply-To: <505171.31412.qm@web28513.mail.ukl.yahoo.com> Message-ID: Ecclesiastes 1:2 Vanity of vanities; all is vanity. Nigel Guthrie: >>>Most of us web wanderers seem to be on an ego-trip :) Richard Hills: I have been on a massive ego-trip for my entire blml career. No joke. But the question is whether the outcome of a blml ego-trip will benefit the bridge community in general. >(3) The relevant necessary complexity of the rules because > Duplicate Bridge is "a thinking game" (i.e. a complex > game whose attraction rests upon it being a partnership > game of imperfect information, hence its rules have to > be more detailed and nuanced than 99% of the other mind > games listed upon the website BoardGameGeek). Herman De Wael >>What? >> >>I am trying to tell Richard that the laws as the WBF >>promulgate them are less "right" than the ones I would >>propose. And Richard answers that the WBF sets the laws. Richard Hills: No, my point (1) was stating that the WBF sets the Laws. My point (3) was stating that simplifying the Laws a la De Wael would destroy the nature of the game. That is, lying to the opponents is part of Poker's nature, but not part of Duplicate Bridge's nature. So once one accepts that the traditional nature of Duplicate Bridge requires that all questions be honestly answered and all alerts invariably displayed, the Laws and Regulations of Duplicate Bridge are necessarily complex. Herman De Wael: >>Why do I bother? Richard Hills: Why indeed? As Jeff Easterson correctly noted, Herman De Wael has constructively posted on other topics in the recent past. Best wishes Richard Hills Recruitment Section Specialist Recruitment Team Level 5 Aqua, workstation W569, 6223 8453 DIAC Social Club movie tickets -------------------------------------------------------------------- Important Notice: If you have received this email by mistake, please advise the sender and delete the message and attachments immediately. This email, including attachments, may contain confidential, sensitive, legally privileged and/or copyright information. Any review, retransmission, dissemination or other use of this information by persons or entities other than the intended recipient is prohibited. DIAC respects your privacy and has obligations under the Privacy Act 1988. The official departmental privacy policy can be viewed on the department's website at www.immi.gov.au. See: http://www.immi.gov.au/functional/privacy.htm --------------------------------------------------------------------- From svenpran at online.no Tue Jan 11 23:37:49 2011 From: svenpran at online.no (Sven Pran) Date: Tue, 11 Jan 2011 23:37:49 +0100 Subject: [BLML] Alain's case revisited [SEC=UNOFFICIAL] In-Reply-To: <4D2C8F7D.7040800@skynet.be> References: <4D2ABE9D.1040102@skynet.be> <838975.24149.qm@web28514.mail.ukl.yahoo.com> <4D2B27EC.5050507@ulb.ac.be> <000801cbb0e0$7461cb00$5d256100$@no> <4D2B3AA9.7050100@ulb.ac.be> <4D2C1A41.7050203@skynet.be> <000301cbb175$cf9acdb0$6ed06910$@no> <4D2C29AC.6040701@skynet.be> <000701cbb189$173953e0$45abfba0$@no> <4D2C8F7D.7040800@skynet.be> Message-ID: <001601cbb1e0$2d3d64e0$87b82ea0$@no> On Behalf Of Herman De Wael .............. > > > > The current laws do indeed provide for an entitlement that there has > > been a misunderstanding in the form of an initially incorrect > > explanation, see L20F5(b). > > (And they do of course provide for redress if such misunderstanding is > > not revealed in time to avoid opponents becoming damaged.). > > > > How can you read this in that manner? > The only entitlement that L20F5 gives is the fact that opponents get it before they > see the dummy. Inaccurate: The declaring side must unasked provide opponents with correction of any incorrect explanation during the auction before presumed declarer's LHO selects his opening lead. > Don't forget that when the misunderstanding pair does not become declaring side, > there is no entitlement towards declarer. I am fully aware of that > > OTOH, L20F5 explicitely forbids a misunderstanding side of revealing that > misunderstanding to opponents. Talk about non-entitlement! Not if such revelation is an inevitable consequence of providing opponents with a corrected explanation (in due time). From richard.hills at immi.gov.au Tue Jan 11 23:53:12 2011 From: richard.hills at immi.gov.au (richard.hills at immi.gov.au) Date: Wed, 12 Jan 2011 09:53:12 +1100 Subject: [BLML] Nigel's case supported [SEC=UNOFFICIAL] In-Reply-To: <4D2C1E71.7060004@skynet.be> Message-ID: Richard Hills: >>All joking/insults aside, there is a 0.00% chance that the De >>Wael loophole will be restored by the "intransigent" WBF LC. Herman De Wael: >And why do you so categorically state this? > >0.00% is very little. Make it 0.001% and I'll wager 1 whatever >on it, 100000 to 1. Stephen Fry, host of the BBC television quiz show QI: "One in thirty million people risk dying by being murdered, the risk of choking to death is one in a hundred and twenty million, the risk of dying by tea cosy is one in twenty billion." Best wishes Richard Hills Recruitment Section Specialist Recruitment Team Level 5 Aqua, workstation W569, 6223 8453 DIAC Social Club movie tickets -------------------------------------------------------------------- Important Notice: If you have received this email by mistake, please advise the sender and delete the message and attachments immediately. This email, including attachments, may contain confidential, sensitive, legally privileged and/or copyright information. Any review, retransmission, dissemination or other use of this information by persons or entities other than the intended recipient is prohibited. DIAC respects your privacy and has obligations under the Privacy Act 1988. The official departmental privacy policy can be viewed on the department's website at www.immi.gov.au. See: http://www.immi.gov.au/functional/privacy.htm --------------------------------------------------------------------- From Hermandw at skynet.be Wed Jan 12 00:02:17 2011 From: Hermandw at skynet.be (Herman De Wael) Date: Wed, 12 Jan 2011 00:02:17 +0100 Subject: [BLML] Ecclesiastes [SEC=UNOFFICIAL] In-Reply-To: References: Message-ID: <4D2CE179.6080200@skynet.be> richard.hills at immi.gov.au wrote: > > Richard Hills: > > No, my point (1) was stating that the WBF sets the Laws. My > point (3) was stating that simplifying the Laws a la De Wael > would destroy the nature of the game. That is, lying to the > opponents is part of Poker's nature, but not part of > Duplicate Bridge's nature. So once one accepts that the > traditional nature of Duplicate Bridge requires that all > questions be honestly answered and all alerts invariably > displayed, the Laws and Regulations of Duplicate Bridge are > necessarily complex. > Please tell me, Richard, if you cheat on your wife. I am convinced you will answer no to this, whether that is true or not. My point being that lying about a crime is not in any way or form worse than the crime itself. When Patrick failed to alert 1Sp, Alain did not speak out. As such, he "lied" about the meaning of 1Sp. If the WBF thinks it is OK about lying about 1Sp (by L20F5) then why should they bother so much about lying about 2Sp? The fact that people are bothered about lying about 2Sp and not about lying about 1Sp tells me something about the way many people have been brought up. Active lies are considered worse than passive lies, perhaps. Well, I don't believe that is true. Alain and Patrick are playing together, and Alain and Patrick conspired to keep the meaning of 1Sp hidden from their opponents. If you believe that Full Information is the highest good, then you should argue against L20F5. But indeed, that is not what you are saying. But it does paint a strange picture. It's OK to cheat on your wife, as long as you don't admit to it. Lying is bad, Misinformation is just an irregularity. Well, I don't ascribe to that filosophy. Now if, Richard, you believe that lying about 1Sp is right, and lying about 2Sp is wrong, because the WBF says so, then we are back at square one. So tell me Richard, why you believe the WBF is correct in maintaining L20F5 and at the same time defending alerting 2Sp. And, just to be complete, please note that in some other decision in Beijing, the WBF explicitely reaffirmed their stance over L20F5. -- Herman De Wael Wilrijk Antwerpen Belgium From JffEstrsn at aol.com Wed Jan 12 00:17:42 2011 From: JffEstrsn at aol.com (Jeff Easterson) Date: Wed, 12 Jan 2011 00:17:42 +0100 Subject: [BLML] Ecclesiastes [SEC=UNOFFICIAL] In-Reply-To: <4D2CE179.6080200@skynet.be> References: <4D2CE179.6080200@skynet.be> Message-ID: <4D2CE516.3070200@aol.com> Is Richard married? Am 12.01.2011 00:02, schrieb Herman De Wael: > richard.hills at immi.gov.au wrote: >> Richard Hills: >> >> No, my point (1) was stating that the WBF sets the Laws. My >> point (3) was stating that simplifying the Laws a la De Wael >> would destroy the nature of the game. That is, lying to the >> opponents is part of Poker's nature, but not part of >> Duplicate Bridge's nature. So once one accepts that the >> traditional nature of Duplicate Bridge requires that all >> questions be honestly answered and all alerts invariably >> displayed, the Laws and Regulations of Duplicate Bridge are >> necessarily complex. >> > Please tell me, Richard, if you cheat on your wife. > I am convinced you will answer no to this, whether that is true or not. > My point being that lying about a crime is not in any way or form worse > than the crime itself. > > When Patrick failed to alert 1Sp, Alain did not speak out. As such, he > "lied" about the meaning of 1Sp. > If the WBF thinks it is OK about lying about 1Sp (by L20F5) then why > should they bother so much about lying about 2Sp? > The fact that people are bothered about lying about 2Sp and not about > lying about 1Sp tells me something about the way many people have been > brought up. Active lies are considered worse than passive lies, perhaps. > Well, I don't believe that is true. Alain and Patrick are playing > together, and Alain and Patrick conspired to keep the meaning of 1Sp > hidden from their opponents. If you believe that Full Information is the > highest good, then you should argue against L20F5. > But indeed, that is not what you are saying. But it does paint a strange > picture. It's OK to cheat on your wife, as long as you don't admit to > it. Lying is bad, Misinformation is just an irregularity. > Well, I don't ascribe to that filosophy. > > Now if, Richard, you believe that lying about 1Sp is right, and lying > about 2Sp is wrong, because the WBF says so, then we are back at square one. > > So tell me Richard, why you believe the WBF is correct in maintaining > L20F5 and at the same time defending alerting 2Sp. > And, just to be complete, please note that in some other decision in > Beijing, the WBF explicitely reaffirmed their stance over L20F5. > From richard.hills at immi.gov.au Wed Jan 12 00:37:50 2011 From: richard.hills at immi.gov.au (richard.hills at immi.gov.au) Date: Wed, 12 Jan 2011 10:37:50 +1100 Subject: [BLML] Ecclesiastes [SEC=UNOFFICIAL] In-Reply-To: <4D2CE179.6080200@skynet.be> Message-ID: Herman De Wael: >Please tell me, Richard, if you cheat on your wife. >I am convinced you will answer no to this, whether that is >true or not. Richard Hills: A false premise implies any conclusion. Therefore it is equally valid for me to answer either "Yes" or "No", because I cannot cheat on my wife, since I have never been married. Herman De Wael: >My point being that lying about a crime is not in any way or >form worse than the crime itself. Richard Hills: Likewise, it is false to imply that I would intentionally lie to cover up my partner's unintentional MI. A secondary crime is much "worse" than the original crime when the first was unintentional but the second is intentional. See "must not" in Law 72B1. Ecclesiastes 10:8 He that diggeth a pit shall fall into it. Grattan Endicott, 13th June 2007: +=+ The sadness is that the whole is tainted by the rottenness of a percentage of the ingredients. Blml allows of an infinite variety of opinion. This is good. But when a long-winded contributor makes repetitive assertions as to the Laws of the game, not as opinions of what is desirable but rather professing an interpretation of Law at variance with the proper - authorized - interpretation, the value and credibility of the whole discussion becomes diminished, corrupted, noisome. People turn aside from the stench, not persevering with research of the wholesome parts for what golden truths and pearls of wisdom may lie there. ~ G ~ +=+ Best wishes Richard Hills Recruitment Section Specialist Recruitment Team Level 5 Aqua, workstation W569, 6223 8453 DIAC Social Club movie tickets -------------------------------------------------------------------- Important Notice: If you have received this email by mistake, please advise the sender and delete the message and attachments immediately. This email, including attachments, may contain confidential, sensitive, legally privileged and/or copyright information. Any review, retransmission, dissemination or other use of this information by persons or entities other than the intended recipient is prohibited. DIAC respects your privacy and has obligations under the Privacy Act 1988. The official departmental privacy policy can be viewed on the department's website at www.immi.gov.au. See: http://www.immi.gov.au/functional/privacy.htm --------------------------------------------------------------------- From richard.hills at immi.gov.au Wed Jan 12 01:00:43 2011 From: richard.hills at immi.gov.au (richard.hills at immi.gov.au) Date: Wed, 12 Jan 2011 11:00:43 +1100 Subject: [BLML] Ecclesiastes [SEC=UNOFFICIAL] In-Reply-To: Message-ID: Harald Skjaran: >Why on earth did you say such a thing? Richard Hills: Because I did not say such a thing. It seems that Harald is confusing Richard H(ills) with the less prolific blmler Richard W(illey). Harald Skjaran: >Herman is trying to argue for what he believes is best for bridge. Richard Hills: So? The brilliant Sir Isaac Newton believed in alchemy. Harald Skjaran: >It's a real possibility that he is right about that aspect. Richard Hills: No. Indeed, definitely not. As Nigel Guthrie has pointed out, the changes proposed by Herman De Wael are so fundamental that if ever enacted (which fortunately will never happen) bridge could no longer be called bridge. Harald Skjaran: >The way Richard H. responds, it's like fighting windmills. Richard Hills: Yes, I agree that Herman De Wael can aptly be described as Don Quixote tilting at windmills, thanks to his futile war for a (now closed) loophole in the Lawbook. Ecclesiastes 8:8 There is no discharge in that war. -------------------------------------------------------------------- Important Notice: If you have received this email by mistake, please advise the sender and delete the message and attachments immediately. This email, including attachments, may contain confidential, sensitive, legally privileged and/or copyright information. Any review, retransmission, dissemination or other use of this information by persons or entities other than the intended recipient is prohibited. DIAC respects your privacy and has obligations under the Privacy Act 1988. The official departmental privacy policy can be viewed on the department's website at www.immi.gov.au. See: http://www.immi.gov.au/functional/privacy.htm --------------------------------------------------------------------- From richard.hills at immi.gov.au Wed Jan 12 03:45:20 2011 From: richard.hills at immi.gov.au (richard.hills at immi.gov.au) Date: Wed, 12 Jan 2011 13:45:20 +1100 Subject: [BLML] Hesitation Blackwood [SEC=UNOFFICIAL] Message-ID: Thomas Dehn: The OS reaches 6H via hesitation blackwood, and makes six when declarer makes a 50-50 guess. It is "at all probable", and maybe even "likely", that he would have guessed wrongly had the hand been played again in 5H or 6H. One can easily read TFLB in many different ways: o both sides get 5H+1 o OS gets 5H just made o OS gets 6H down one I think TFLB fails to state clearly what the expected ruling is in this simple scenario. And as long as that isn't stated clearly in TFLB itself in a way that an average player can understand it without consulting a TD or BLML, it doesn't make much sense to debate how more complex such cases should be handled. Richard Hills: The contract is adjusted to 5H for both sides. The play in 5H may well have been different to the play in 6H. If, for example, the Conditions of Contest use imps, the OS may well have taken a safety play to restrict themselves to just 11 tricks in 5H. And of course comprehensive training of Directors cannot be achieved merely by the Lawbook, which is merely a reference manual. It is up to Regulating Authorities to provide added guidance. The English Bridge Union is particularly thoughtful in providing guidance, as it makes its White Book freely available to non-EBU Directors. http://www.ebu.co.uk/lawsandethics/misc/whitebook.htm Best wishes Richard Hills Recruitment Section Specialist Recruitment Team Level 5 Aqua, workstation W569, 6223 8453 DIAC Social Club movie tickets -------------------------------------------------------------------- Important Notice: If you have received this email by mistake, please advise the sender and delete the message and attachments immediately. This email, including attachments, may contain confidential, sensitive, legally privileged and/or copyright information. Any review, retransmission, dissemination or other use of this information by persons or entities other than the intended recipient is prohibited. DIAC respects your privacy and has obligations under the Privacy Act 1988. The official departmental privacy policy can be viewed on the department's website at www.immi.gov.au. See: http://www.immi.gov.au/functional/privacy.htm --------------------------------------------------------------------- From richard.hills at immi.gov.au Wed Jan 12 04:17:51 2011 From: richard.hills at immi.gov.au (richard.hills at immi.gov.au) Date: Wed, 12 Jan 2011 14:17:51 +1100 Subject: [BLML] EBU L&EC meeting 3rd November 2010 [SEC=UNOFFICIAL] In-Reply-To: <4D2C2DFC.8030502@skynet.be> Message-ID: 5.11 New code of Laws Mr Endicott said that the first preliminary discussions about the 2017 code of laws was scheduled for about six months time. Any suggestions for law amendments should be sent directly to Grattan. +=+=+=+=+=+=+=+=+=+=+=+=+=+=+=+=+=+=+=+=+=+=+=+=+=+=+=+=+=+=+=+=+ 2018 LAW 20 - REVIEW AND EXPLANATION OF CALLS A. Call Not Clearly Recognised A player may require clarification forthwith if she is in doubt what call has been made. B. Review of Auction during Auction Period During the auction period, a player is entitled to have all previous calls restated* when it is her turn to call, unless she is required by law to pass. Alerts should be included when responding to the request. A player may not ask for a partial review of previous calls and may not halt the review before it is completed. C. Review after Final Pass 1. After the final pass either defender has the right to ask if it is her opening lead (see Laws 47E and 41). 2. Declarer** or either defender may, at her first turn to play, require all previous calls to be restated*. (See Laws 41B and 41C). As in B the player may not ask for only a partial restatement or halt the review. D. Who May Review the Auction A request to have calls restated* shall be responded to only by an opponent (unless the Director at the table rules otherwise). E. Correction of Error in Review All players, including dummy or a player required by law to pass, are responsible for prompt correction of errors in restatement* (see Law 12C1 when an uncorrected review causes damage). F. Explanation of Calls 1. (a) During the auction before the final pass, any player may request, but only at her own turn to call, honest*** explanations of the opponents' prior auction. She is entitled to know about calls actually made, about relevant alternative calls available that were not made, and about relevant inferences from the choice of action where these are matters of partnership understanding. She is not entitled to know about calls that might be made in an entirely different auction (for example, if the pre-existing mutual partnership understanding of the opponents in a particular auction is that 4NT promises 5/5 in the minors, the player is not entitled at that time to know the opponents' responses to their 4NT Keycard Blackwood convention). (b) Except on the instruction of the Director (who may, in accordance with guidance from her Regulating Authority, direct a player to temporarily leave the table) replies should be given by the partner of the player who made the call in question. The partner of a player who asks a question may not ask a supplementary question until her turn to call or play. Law 16 may apply and the Regulating Authority may establish regulations for written explanations. 2. After the final pass and throughout the play period, either defender at her own turn to play may request honest*** explanations of the opposing auction. At her turn to play from her hand or from dummy declarer may request honest*** explanations of a defender's call or card play understandings. Honest*** explanations should be given on a like basis to 1 and by the partner of the player whose action is explained. 3. Under 1 and 2 above a player may ask concerning a single call but Law 16B1 may apply. 4. If a player subsequently realises that her own explanation was erroneous or incomplete she must call the Director immediately. The Director applies Law 21B or Law 40B4. 5. (a) A player whose partner has given a mistaken explanation may not correct the error during the auction, nor may she indicate by any mannerism (as in a shrug, glare, frown etc.) that a mistake has been made. "Mistaken explanation" here includes failure to alert or announce as regulations require or an alert (or an announcement) that regulations do not require. (b) Nevertheless, the player must still give honest*** explanations of her partner's subsequent calls whether or not those honest*** explanations indirectly indicate to partner her partner's error. Such an indirect indication is unauthorised information, so Law 73C and/or Law 75A may apply to partner. (c) The player must call the Director and inform her opponents that, in her opinion, her partner's explanation was erroneous (see Law 75) but only at her first legal opportunity, which is (i) for a defender, at the end of the play. (ii) for declarer or dummy, after the final pass of the auction (but before the opening lead is faced, and preferably before the opening lead is selected). 6. If the Director judges that a player has based an action on misinformation given to him by an opponent see, as appropriate, Law 21 or Law 47E. G. Incorrect Procedure 1. (a) Asking a question solely for partner's benefit is an infraction. The Director applies Law 90 or Law 91. (b) Asking a question solely for the purpose of making either side create unauthorised information is an infraction. The Director applies Law 90 or Law 91. 2. Except as the Regulating Authority allows a player may not consult her own System Card and notes during the auction and play periods, but see Law 40B2(b). * When the calls are not spoken responders must ensure that it is clear to an enquiring opponent what calls have been made. ** Declarer's first turn to play is from dummy unless accepting an opening lead out of turn. *** Unintentional errors in explanation are honest. Dishonest intentional misexplanations could well receive Law 72B1 and Law 91 Disciplinary Penalties. 2018 LAW 21 - CALL BASED ON MISINFORMATION A. Call Based on Caller's Misunderstanding No rectification or redress is due to a player who acts on the basis of her own misunderstanding. B. Call Based on Misinformation from an Opponent 1. (a) The Director is to presume Mistaken Explanation rather than Mistaken Call in the absence of evidence to the contrary. See Law 75 for a detailed indicative example. (b) Failure to alert promptly where an alert is required by the Regulating Authority is deemed misinformation. 2. When it is too late to change a particular call (for example, in a belated Law 20F4 correction of misinformation) the Director requires the auction and play to continue. At the end of play, if the Director judges that the offending side gained an advantage from the irregularity, then the Director awards an adjusted score. 3. Until the end of the auction period and provided that her partner has not subsequently called, a player may change a call without other rectification for her side when the Director judges that the decision to make the call could well have been influenced by misinformation given to the player by an opponent (see Law 17E). At the end of play, if the Director judges that the offending side gained an advantage from the irregularity, then the Director awards an adjusted score. 4. When a player elects to change a call because of misinformation (as in 3 preceding), her LHO may then in turn change any subsequent call she may have made, without other rectification unless at the end of the hand the Director judges the LHO's withdrawn call to have conveyed such information as to damage the non-offending side in which case Law 16D applies. Best wishes Richard Hills Recruitment Section Specialist Recruitment Team Level 5 Aqua, workstation W569, 6223 8453 DIAC Social Club movie tickets -------------------------------------------------------------------- Important Notice: If you have received this email by mistake, please advise the sender and delete the message and attachments immediately. This email, including attachments, may contain confidential, sensitive, legally privileged and/or copyright information. Any review, retransmission, dissemination or other use of this information by persons or entities other than the intended recipient is prohibited. DIAC respects your privacy and has obligations under the Privacy Act 1988. The official departmental privacy policy can be viewed on the department's website at www.immi.gov.au. See: http://www.immi.gov.au/functional/privacy.htm --------------------------------------------------------------------- From jfusselman at gmail.com Wed Jan 12 05:19:20 2011 From: jfusselman at gmail.com (Jerry Fusselman) Date: Tue, 11 Jan 2011 22:19:20 -0600 Subject: [BLML] Ecclesiastes [SEC=UNOFFICIAL] In-Reply-To: References: Message-ID: On Tue, Jan 11, 2011 at 6:00 PM, Richard Hills wrote: > Harald Skjaran: > >>Why on earth did you say such a thing? > > Richard Hills: > > Because I did not say such a thing. > > It seems that Harald is confusing Richard H(ills) with the less > prolific blmler Richard W(illey). > False. Reread Harald's first line---it properly names richard willey. The names Harald used in his post today were 100% accurate, and I think that his post was entirely fair in all respects. > Harald Skjaran: > >>Herman is trying to argue for what he believes is best for bridge. > > Richard Hills: > > So? ?The brilliant Sir Isaac Newton believed in alchemy. Richard H misses the point. Richard W's posts today detract from BLML. Carefully exploring a minority opinion is what BLML is all about; ad hominem attacks are not---at least I hope not. Jerry Fusselman From richard.hills at immi.gov.au Wed Jan 12 05:36:33 2011 From: richard.hills at immi.gov.au (richard.hills at immi.gov.au) Date: Wed, 12 Jan 2011 15:36:33 +1100 Subject: [BLML] EBU L&EC meeting 3rd November 2010 [SEC=UNOFFICIAL] In-Reply-To: Message-ID: 5.11 New code of Laws Mr Endicott said that the first preliminary discussions about the 2017 code of laws was scheduled for about six months time. Any suggestions for law amendments should be sent directly to Grattan. +=+=+=+=+=+=+=+=+=+=+=+=+=+=+=+=+=+=+=+=+=+=+=+=+=+=+=+=+=+=+=+=+ 2018 LAW 12 - SCORE ADJUSTMENT, DIRECTOR'S DISCRETIONARY POWERS A. Power to Award an Adjusted Score On the application of a player within the period established under Law 92B, or on his own initiative, the Director may award an adjusted score when these Laws empower her to do so (in team play see Law 86). This includes: 1. (a) The Director may award an adjusted score when she judges that a rectification provided in these Laws does not redress damage (see B below) to the non-offending side for the particular type of infraction committed at the same table by the offending side. (b) The Director may not award an adjusted score on the ground that the rectification provided in these Laws is either unduly severe to the offending side or unduly advantageous to the non- offending side. 2. The Director awards an artificial adjusted score if no rectification can be made that will permit normal play of the board (see C2 below). 3. The Director may award an adjusted score if there has been an incorrect rectification of an irregularity. B. Objective of Score Adjustment The objective of score adjustment is to redress damage to a non-offending side and to remove all advantages gained by an offending side through its infraction. Damage exists when, because of an infraction, a non-offending side obtains a table result less favourable than would have been the expectation of the non-offending side at the instant before the infraction ? but see C1(b). C. Awarding an Adjusted Score 1. (a) When after an irregularity the Director is empowered by these laws to adjust a score and is able to award an assigned adjusted score, she does so. Such a score replaces the score obtained in play. (b) If subsequent to the irregularity the non-offending side has contributed to its own damage by a ridiculous* error (unrelated to the infraction), or by wild or gambling action, then the non-offending side receives partial rectification only, ignoring such part of the damage as is self-inflicted. The offending side, however, receives all of the score that was its expectation at the instant before its infraction (i.e. the non- offending side's subsequent error is irrelevant for the calculation of the score awarded to the offending side by the Director). (c) In order to do equity, and unless the Regulating Authority forbids it, an assigned adjusted score may be weighted to reflect the probabilities of a number of potential results. (But an illegally obtained result, potential or actual, must not be included in the weightings.) (d) If the possibilities are numerous or not obvious, the Director may award an artificial adjusted score. (e) In its discretion the Regulating Authority may apply all or part of the following procedure in place of (c): (i) The score assigned in place of the actual score for a non-offending side is the most favourable result that was likely at the instant before the infraction. (ii) For an offending side the score assigned is the most unfavourable result that was at all probable at the instant before the infraction. (f) The scores awarded to the two sides need not balance. 2. (a) When owing to an irregularity no result can be obtained [and see C1(d)] the Director awards an artificial adjusted score according to responsibility for the irregularity: average minus (at most 40% of the available matchpoints in pairs) to a contestant directly at fault, average (50% in pairs) to a contestant only partly at fault, and average plus (at least 60% in pairs) to a contestant in no way at fault. (b) When the Director awards an artificial adjusted score of average plus or minus at international match points that score is normally plus or minus 3 imps, but this may be varied as Law 86A allows. (c) The foregoing is modified for a non-offending contestant that obtains a session score exceeding 60% of the available matchpoints or for an offending contestant that obtains a session score that is less than 40% of the available matchpoints (or the equivalent in imps). Such contestants are awarded the percentage obtained (or the equivalent in imps) on the other boards of that session. This clause is merely a default, and may be freely amended by the Regulating Authority. (For example, a regulation may be created capping the number of average plus scores that a contestant may gain in a single session.) 3. In individual events the Director enforces the rectifications in these Laws, and the provisions requiring the award of adjusted scores, equally against both members of the offending side even though only one of them may be responsible for the irregularity. But the Director shall not award a procedural penalty against the offender's partner if of the opinion that she is in no way to blame. 4. When the Director awards non-balancing adjusted scores in knockout play, each contestant's score on the board is calculated separately and the average of them is assigned to each. * "Ridiculous" is assessed by the class of player involved. For a novice player a "ridiculous" error would be extremely rare (e.g. the rare major mechanical error of revoking). Best wishes Richard Hills Recruitment Section Specialist Recruitment Team Level 5 Aqua, workstation W569, 6223 8453 DIAC Social Club movie tickets -------------------------------------------------------------------- Important Notice: If you have received this email by mistake, please advise the sender and delete the message and attachments immediately. This email, including attachments, may contain confidential, sensitive, legally privileged and/or copyright information. Any review, retransmission, dissemination or other use of this information by persons or entities other than the intended recipient is prohibited. DIAC respects your privacy and has obligations under the Privacy Act 1988. The official departmental privacy policy can be viewed on the department's website at www.immi.gov.au. See: http://www.immi.gov.au/functional/privacy.htm --------------------------------------------------------------------- From jfusselman at gmail.com Wed Jan 12 05:47:33 2011 From: jfusselman at gmail.com (Jerry Fusselman) Date: Tue, 11 Jan 2011 22:47:33 -0600 Subject: [BLML] EBU L&EC meeting 3rd November 2010 [SEC=UNOFFICIAL] In-Reply-To: <4D2C2DFC.8030502@skynet.be> References: <4D2C2DFC.8030502@skynet.be> Message-ID: On Tue, Jan 11, 2011 at 4:16 AM, Herman De Wael wrote: > > I believe that one of the most important discussions the WBF should > start with is the question of whether or not the opponents are entitled > to the knowledge that there has been a misunderstanding. > I am probably going to echo what Eric said here, though perhaps in a different way. I feel that Herman's question is a little vague. Let me try to give answer with a little bit of nuance to see what people think. Perhaps this answer will help eliminate some of the vagueness: 1. The opponents are not entitled to hear statements during or after the auction listing all personal errors in bidding if there is no way the opponents could deduce them with perfect knowledge of your partnership understandings. 2. An opponent is entitled to know each of your misunderstandings that he could have deduced using the contents of his own hand, the auction, and a complete description of your partnership understandings. Does this seem like a good way to answer Herman's question? It is just one possibility, and actually, I share Herman's question. Jerry Fusselman From richard.hills at immi.gov.au Wed Jan 12 06:40:32 2011 From: richard.hills at immi.gov.au (richard.hills at immi.gov.au) Date: Wed, 12 Jan 2011 16:40:32 +1100 Subject: [BLML] Ecclesiastes [SEC=UNOFFICIAL] In-Reply-To: Message-ID: "Carefully exploring a minority opinion is what BLML is all about" No. There is an idiosyncratic German bridge player who has argued for a decade that a literal interpretation of Law 77 means that scores are non-reciprocal. For example, if North-South score +140, the correct score for East-West is not the majority opinion -140, but rather a zero score. There is not any merit arguing whether that interpretation of Law 77 is correct. There is not any merit arguing that interpretation must be correct, because the WBF Laws Committee has not issued a contrary interpretation of Law 77. There is not any merit arguing that interpretation should be correct, because that would be a "more logical" Law 77. There is not any merit arguing, because the opponents had no objection to being ripped off, that an intentional Law 77 infraction at the table to gain a score advantage is correct. "Sufficiently exploring a minority opinion is what blml is all about." More than sufficient time has been given to the De Wael loophole on blml over the past decade. Ecclesiastes 3:7 a time to rend, and a time to sew; a time to keep silence, and a time to speak Sew I will follow Jeff Easterson's sewber advice, and now keep silence in response to future Rend Herring postings by the acolytes of Alain or by the hetaerae of Herman. Best wishes Richard Hills Recruitment Section Specialist Recruitment Team Level 5 Aqua, workstation W569, 6223 8453 DIAC Social Club movie tickets -------------------------------------------------------------------- Important Notice: If you have received this email by mistake, please advise the sender and delete the message and attachments immediately. This email, including attachments, may contain confidential, sensitive, legally privileged and/or copyright information. Any review, retransmission, dissemination or other use of this information by persons or entities other than the intended recipient is prohibited. DIAC respects your privacy and has obligations under the Privacy Act 1988. The official departmental privacy policy can be viewed on the department's website at www.immi.gov.au. See: http://www.immi.gov.au/functional/privacy.htm --------------------------------------------------------------------- From Hermandw at skynet.be Wed Jan 12 09:18:35 2011 From: Hermandw at skynet.be (Herman De Wael) Date: Wed, 12 Jan 2011 09:18:35 +0100 Subject: [BLML] Alain's case revisited [SEC=UNOFFICIAL] In-Reply-To: <001601cbb1e0$2d3d64e0$87b82ea0$@no> References: <4D2ABE9D.1040102@skynet.be> <838975.24149.qm@web28514.mail.ukl.yahoo.com> <4D2B27EC.5050507@ulb.ac.be> <000801cbb0e0$7461cb00$5d256100$@no> <4D2B3AA9.7050100@ulb.ac.be> <4D2C1A41.7050203@skynet.be> <000301cbb175$cf9acdb0$6ed06910$@no> <4D2C29AC.6040701@skynet.be> <000701cbb189$173953e0$45abfba0$@no> <4D2C8F7D.7040800@skynet.be> <001601cbb1e0$2d3d64e0$87b82ea0$@no> Message-ID: <4D2D63DB.6080400@skynet.be> Sven Pran wrote: > On Behalf Of Herman De Wael > .............. >>> >>> The current laws do indeed provide for an entitlement that there has >>> been a misunderstanding in the form of an initially incorrect >>> explanation, see L20F5(b). >>> (And they do of course provide for redress if such misunderstanding is >>> not revealed in time to avoid opponents becoming damaged.). >>> >> >> How can you read this in that manner? >> The only entitlement that L20F5 gives is the fact that opponents get it > before they >> see the dummy. > > Inaccurate: The declaring side must unasked provide opponents with > correction of any incorrect explanation during the auction before presumed > declarer's LHO selects his opening lead. > And how does that differ from what I wrote? >> Don't forget that when the misunderstanding pair does not become declaring > side, >> there is no entitlement towards declarer. > > I am fully aware of that > >> >> OTOH, L20F5 explicitely forbids a misunderstanding side of revealing that >> misunderstanding to opponents. Talk about non-entitlement! > > Not if such revelation is an inevitable consequence of providing opponents > with a corrected explanation (in due time). > And when is due time? After the hand in the case of misunderstanders being defenders! So declarer is NEVER entitled to the knowledge of a misunderstanding by the side that ends up defending (unless you believe an entitlement after trick 13 is still worth something). The defending side are indeed entitled to this knowledge from the declaring side, but they would very often see this anyway when dummy comes down, so their useful entitlement lasts only one card (the opening lead). Quite the contrary from the sentence Sen wrote in trying to argue against something I said - I don't even remember why he wrote it. But a nice example of how argumentation on blml often goes: You use all sorts of arguments, I refute them, and then they are forgotten. Until the next time someone uses them, and I refute them ... -- Herman De Wael Wilrijk Antwerpen Belgium From Hermandw at skynet.be Wed Jan 12 09:22:42 2011 From: Hermandw at skynet.be (Herman De Wael) Date: Wed, 12 Jan 2011 09:22:42 +0100 Subject: [BLML] Ecclesiastes [SEC=UNOFFICIAL] In-Reply-To: References: Message-ID: <4D2D64D2.9030705@skynet.be> richard.hills at immi.gov.au wrote: > >> It's a real possibility that he is right about that aspect. > > Richard Hills: > > No. Indeed, definitely not. As Nigel Guthrie has pointed out, the > changes proposed by Herman De Wael are so fundamental that if ever > enacted (which fortunately will never happen) bridge could no > longer be called bridge. > Richard Hills not only knows what is best for bridge - he also knows when a change would make it no longer bridge. Richard Hills has not understood a word of what has been said in the past 15 years. Richard Hills has not seen that in the past 15 years, only one true example of a dWS ruling has been seen. That ruling has gone unnoticed by the opponents, who certainly thought they were playing bridge. -- Herman De Wael Wilrijk Antwerpen Belgium From Hermandw at skynet.be Wed Jan 12 09:23:59 2011 From: Hermandw at skynet.be (Herman De Wael) Date: Wed, 12 Jan 2011 09:23:59 +0100 Subject: [BLML] Ecclesiastes [SEC=UNOFFICIAL] In-Reply-To: References: Message-ID: <4D2D651F.4070000@skynet.be> richard.hills at immi.gov.au wrote: > "Carefully exploring a minority opinion is what BLML is all > about" > > No. > Not only does Richard know what is best for bridge, he also knows what is best for blml. Why do we bother listening to such a tyrant? -- Herman De Wael Wilrijk Antwerpen Belgium From Hermandw at skynet.be Wed Jan 12 09:25:28 2011 From: Hermandw at skynet.be (Herman De Wael) Date: Wed, 12 Jan 2011 09:25:28 +0100 Subject: [BLML] Ecclesiastes [SEC=UNOFFICIAL] In-Reply-To: References: Message-ID: <4D2D6578.9020106@skynet.be> richard.hills at immi.gov.au wrote: > > Sew I will follow Jeff Easterson's sewber advice, and now > keep silence in response to future Rend Herring postings > by the acolytes of Alain or by the hetaerae of Herman. > Considering that Richard merely repeats everything he has already said three times (in a pleasant manner, usually), there is much to be said for this intention of his. -- Herman De Wael Wilrijk Antwerpen Belgium From Hermandw at skynet.be Wed Jan 12 09:36:51 2011 From: Hermandw at skynet.be (Herman De Wael) Date: Wed, 12 Jan 2011 09:36:51 +0100 Subject: [BLML] Ecclesiastes [SEC=UNOFFICIAL] In-Reply-To: References: Message-ID: <4D2D6823.6040802@skynet.be> richard.hills at immi.gov.au wrote: > Herman De Wael: > >> Please tell me, Richard, if you cheat on your wife. >> I am convinced you will answer no to this, whether that is >> true or not. > > Richard Hills: > > A false premise implies any conclusion. Therefore it is > equally valid for me to answer either "Yes" or "No", because > I cannot cheat on my wife, since I have never been married. > Alter the question: "Did you ever allow an illegal immigrant into Australia, Richard?" > Herman De Wael: > >> My point being that lying about a crime is not in any way or >> form worse than the crime itself. > > Richard Hills: > > Likewise, it is false to imply that I would intentionally lie > to cover up my partner's unintentional MI. A secondary crime > is much "worse" than the original crime when the first was > unintentional but the second is intentional. See "must not" > in Law 72B1. > Well, Richard, then what do you call it when you follow L20F5: "not revealing that your partner has given a mistaken explanation" Suppose after your partner has given a wrong explanation, and you keep silent, as L20F5 tells you to do, your RHO turns to you and asks "is that true?". What do you do? A - follow L20F5 and lie to your opponents; or B - break L20F5 and speak the truth. I don't care what you do, Richard, but you must admit that in either case there is a problem. - If you do B, then there is a very easy way for your opponents to gain an advantage: whenever your partner explains something, they ask you "is that true?". Since you will tell them the truth, you will give your partner a piece of UI, and you will get a lesser score than I do, who will gladly say "of course it is!". Surely the Lawmakers cannot have intended you to be so disadvantaged against me, and so the interpretation that must be given to L20F5 should include lying to the opponents in response to the direct question "is this true?" - If you do A, then there is apparently no qualm for you to "lie to your opponents". You apparently accept that the WBFLC is authorized to write laws that condone your lying to your opponents. And so it should be. But in that second case, your assertion two posts ago that "lying to the opponents is part of Poker's nature, but not part of Duplicate Bridge's nature". is a non-valid argument. I again ask you, what is so wrong about lying about a part of your system that is not equally wrong about keeping silent about partners (unintentional) lie? -- Herman De Wael Wilrijk Antwerpen Belgium From svenpran at online.no Wed Jan 12 10:21:48 2011 From: svenpran at online.no (Sven Pran) Date: Wed, 12 Jan 2011 10:21:48 +0100 Subject: [BLML] Alain's case revisited [SEC=UNOFFICIAL] In-Reply-To: <4D2D63DB.6080400@skynet.be> References: <4D2ABE9D.1040102@skynet.be> <838975.24149.qm@web28514.mail.ukl.yahoo.com> <4D2B27EC.5050507@ulb.ac.be> <000801cbb0e0$7461cb00$5d256100$@no> <4D2B3AA9.7050100@ulb.ac.be> <4D2C1A41.7050203@skynet.be> <000301cbb175$cf9acdb0$6ed06910$@no> <4D2C29AC.6040701@skynet.be> <000701cbb189$173953e0$45abfba0$@no> <4D2C8F7D.7040800@skynet.be> <001601cbb1e0$2d3d64e0$87b82ea0$@no> <4D2D63DB.6080400@skynet.be> Message-ID: <000601cbb23a$23e88780$6bb99680$@no> On Behalf Of Herman De Wael > Sven Pran wrote: > > On Behalf Of Herman De Wael > > .............. > >>> > >>> The current laws do indeed provide for an entitlement that there has > >>> been a misunderstanding in the form of an initially incorrect > >>> explanation, see L20F5(b). > >>> (And they do of course provide for redress if such misunderstanding > >>> is not revealed in time to avoid opponents becoming damaged.). > >>> > >> > >> How can you read this in that manner? > >> The only entitlement that L20F5 gives is the fact that opponents get > >> it > > before they > >> see the dummy. > > > > Inaccurate: The declaring side must unasked provide opponents with > > correction of any incorrect explanation during the auction before > > presumed declarer's LHO selects his opening lead. > > > > And how does that differ from what I wrote? I honestly believe you are aware that I refer to the start of the clarification period while you apparently refer to the end of the clarification period. The distinction is significant. > > >> Don't forget that when the misunderstanding pair does not become > >> declaring > > side, > >> there is no entitlement towards declarer. > > > > I am fully aware of that > > > >> > >> OTOH, L20F5 explicitely forbids a misunderstanding side of revealing > >> that misunderstanding to opponents. Talk about non-entitlement! > > > > Not if such revelation is an inevitable consequence of providing > > opponents with a corrected explanation (in due time). > > > > And when is due time? After the hand in the case of misunderstanders being > defenders! > So declarer is NEVER entitled to the knowledge of a misunderstanding by the side > that ends up defending (unless you believe an entitlement after trick 13 is still > worth something). Entitlement after trick 13 is indeed worth an assigned adjusted score if relevant. > > The defending side are indeed entitled to this knowledge from the declaring side, > but they would very often see this anyway when dummy comes down, so their > useful entitlement lasts only one card (the opening lead). For the presumed defending side the entitlement is worth the right to withdraw their last pass and replace it with a different call, causing the auction to continue and even possibly resulting in the originally presumed defending side becoming the declaring side. But this can no longer be ruled after dummy is faced. > > Quite the contrary from the sentence Sen wrote in trying to argue against > something I said - I don't even remember why he wrote it. > But a nice example of how argumentation on blml often goes: You use all sorts of > arguments, I refute them, and then they are forgotten. Until the next time someone > uses them, and I refute them ... Due time for presumed declaring side is at the start of the clarification period. Due time for the defending side is at the end of the play. From Hermandw at skynet.be Wed Jan 12 10:49:31 2011 From: Hermandw at skynet.be (Herman De Wael) Date: Wed, 12 Jan 2011 10:49:31 +0100 Subject: [BLML] Alain's case revisited [SEC=UNOFFICIAL] In-Reply-To: <000601cbb23a$23e88780$6bb99680$@no> References: <4D2ABE9D.1040102@skynet.be> <838975.24149.qm@web28514.mail.ukl.yahoo.com> <4D2B27EC.5050507@ulb.ac.be> <000801cbb0e0$7461cb00$5d256100$@no> <4D2B3AA9.7050100@ulb.ac.be> <4D2C1A41.7050203@skynet.be> <000301cbb175$cf9acdb0$6ed06910$@no> <4D2C29AC.6040701@skynet.be> <000701cbb189$173953e0$45abfba0$@no> <4D2C8F7D.7040800@skynet.be> <001601cbb1e0$2d3d64e0$87b82ea0$@no> <4D2D63DB.6080400@skynet.be> <000601cbb23a$23e88780$6bb99680$@no> Message-ID: <4D2D792B.2090302@skynet.be> Sven Pran wrote: > On Behalf Of Herman De Wael >> Sven Pran wrote: >>> On Behalf Of Herman De Wael >>> .............. >>>>> >>>>> The current laws do indeed provide for an entitlement that there has >>>>> been a misunderstanding in the form of an initially incorrect >>>>> explanation, see L20F5(b). >>>>> (And they do of course provide for redress if such misunderstanding >>>>> is not revealed in time to avoid opponents becoming damaged.). >>>>> >>>> >>>> How can you read this in that manner? >>>> The only entitlement that L20F5 gives is the fact that opponents get >>>> it >>> before they >>>> see the dummy. >>> >>> Inaccurate: The declaring side must unasked provide opponents with >>> correction of any incorrect explanation during the auction before >>> presumed declarer's LHO selects his opening lead. >>> >> >> And how does that differ from what I wrote? > > I honestly believe you are aware that I refer to the start of the > clarification period while you apparently refer to the end of the > clarification period. > The distinction is significant. > Why? The clarification perios is a period during which nothing happens, except that clarification. The opening leader has to make his opening lead after the clarification period has ended. What distinction is there, then? And anyway, you were saying something which was totally untrue - completely opposite in fact to the truth. I pointed that out, and accepted that there was one instant - the clarification period - where you were right. I fail to see why this discussion is continueing. -- Herman De Wael Wilrijk Antwerpen Belgium From svenpran at online.no Wed Jan 12 11:09:29 2011 From: svenpran at online.no (Sven Pran) Date: Wed, 12 Jan 2011 11:09:29 +0100 Subject: [BLML] Alain's case revisited [SEC=UNOFFICIAL] In-Reply-To: <4D2D792B.2090302@skynet.be> References: <4D2ABE9D.1040102@skynet.be> <838975.24149.qm@web28514.mail.ukl.yahoo.com> <4D2B27EC.5050507@ulb.ac.be> <000801cbb0e0$7461cb00$5d256100$@no> <4D2B3AA9.7050100@ulb.ac.be> <4D2C1A41.7050203@skynet.be> <000301cbb175$cf9acdb0$6ed06910$@no> <4D2C29AC.6040701@skynet.be> <000701cbb189$173953e0$45abfba0$@no> <4D2C8F7D.7040800@skynet.be> <001601cbb1e0$2d3d64e0$87b82ea0$@no> <4D2D63DB.6080400@skynet.be> <000601cbb23a$23e88780$6bb99680$@no> <4D2D792B.2090302@skynet.be> Message-ID: <000b01cbb240$cd2ab3d0$67801b70$@no> On Behalf Of Herman De Wael > >> And how does that differ from what I wrote? > > > > I honestly believe you are aware that I refer to the start of the > > clarification period while you apparently refer to the end of the > > clarification period. > > The distinction is significant. > > > > Why? I described "why" in detail further down in my comment, the part that you apparently just snipped away. > > The clarification perios is a period during which nothing happens, except that > clarification. The opening leader has to make his opening lead after the > clarification period has ended. THIS is totally untrue: The clarification period can be temporarily terminated and the auction continued if misinformation is revealed, possibly even ending with the opposite side becoming declaring. > What distinction is there, then? > > And anyway, you were saying something which was totally untrue - completely > opposite in fact to the truth. I pointed that out, and accepted that there was one > instant - the clarification period - where you were right. I fail to see why this > discussion is continueing. Frankly I have no idea what I shall have said that was totally untrue and I really did not understand that part of your comment in your previous entry or this part here. From nigelguthrie at yahoo.co.uk Wed Jan 12 12:17:15 2011 From: nigelguthrie at yahoo.co.uk (Nigel Guthrie) Date: Wed, 12 Jan 2011 11:17:15 +0000 (GMT) Subject: [BLML] Ecclesiastes [SEC=UNOFFICIAL] In-Reply-To: <4D2D64D2.9030705@skynet.be> References: <4D2D64D2.9030705@skynet.be> Message-ID: <981233.77121.qm@web28511.mail.ukl.yahoo.com> [Richard Hills] No. Indeed, definitely not. As Nigel Guthrie has pointed out, the changes proposed by Herman De Wael are so fundamental that if ever enacted (which fortunately will never happen) bridge could no longer be called bridge. [Nigel] I didn't point that out because I don't believe it's true. Bridge is what official law-makers say it is. I don't like Herman's suggestion. I don't think it makes the game simpler or more fun. But if the WBFLC adopted it, then the game would still be "Bridge". My stated view is that if a group of ordinary players unilaterally decide to play a new game with their own rules, then they shouldn't call that game "Bridge" From nigelguthrie at yahoo.co.uk Wed Jan 12 12:32:43 2011 From: nigelguthrie at yahoo.co.uk (Nigel Guthrie) Date: Wed, 12 Jan 2011 11:32:43 +0000 (GMT) Subject: [BLML] Ecclesiastes [SEC=UNOFFICIAL] In-Reply-To: <4D2D6823.6040802@skynet.be> References: <4D2D6823.6040802@skynet.be> Message-ID: <139528.76055.qm@web28503.mail.ukl.yahoo.com> [Herman de Wael] I again ask you, what is so wrong about lying about a part of your system that is not equally wrong about keeping silent about partners (unintentional) lie? [Nigel] That question has been answered many times. Neither is intrinsically right. A game is an artificial world where ordinary rules need not apply. In the artifical context of a game, the rules of the game tell you what is right and wrong. The rule-makers of a game are free to repeal the the Geneva Convention or the law of Gravity, if they feel it makes the game more enjoyable. Have you ever played GTA? From sater at xs4all.nl Wed Jan 12 14:20:07 2011 From: sater at xs4all.nl (Hans van Staveren) Date: Wed, 12 Jan 2011 14:20:07 +0100 Subject: [BLML] EBU L&EC meeting 3rd November 2010 [SEC=UNOFFICIAL] In-Reply-To: References: Message-ID: <001201cbb25b$6e69d9f0$4b3d8dd0$@nl> [RH, new law 12] (b) If subsequent to the irregularity the non-offending side has contributed to its own damage by a ridiculous* error (unrelated to the infraction), or by wild or gambling action, then the non-offending side receives partial rectification only, ignoring such part of the damage as is self-inflicted. The offending side, however, receives all of the score that was its expectation at the instant before its infraction (i.e. the non- offending side's subsequent error is irrelevant for the calculation of the score awarded to the offending side by the Director). (c) In order to do equity, and unless the Regulating Authority forbids it, an assigned adjusted score may be weighted to reflect the probabilities of a number of potential results. (But an illegally obtained result, potential or actual, must not be included in the weightings.) [HvS] 12C1b is just restated using a word unlikely to make it into the law. 12C1c is modified to expressly forbid Reveley rulings, as the English call them. I want to point out that the wording here is ambiguous(or wrong). The result obtained illegally might be present in the weighting, if the result could also have been reached legally, through a different auction. Example, someone shoots a game, using UI. His normal call is an invite, which his partner would probably accept. Now a weighted score of two results, including the game is possible and legal. Hans From Hermandw at skynet.be Wed Jan 12 14:33:21 2011 From: Hermandw at skynet.be (Herman De Wael) Date: Wed, 12 Jan 2011 14:33:21 +0100 Subject: [BLML] Ecclesiastes [SEC=UNOFFICIAL] In-Reply-To: <139528.76055.qm@web28503.mail.ukl.yahoo.com> References: <4D2D6823.6040802@skynet.be> <139528.76055.qm@web28503.mail.ukl.yahoo.com> Message-ID: <4D2DADA1.3040903@skynet.be> Nigel Guthrie wrote: > > > [Herman de Wael] > I again ask you, what is so wrong about lying about a part of your system that > is not equally wrong about keeping silent about partners > > (unintentional) lie? > > [Nigel] > That question has been answered many times. Never really satisfactorily. > Neither is intrinsically right. This seems to agree with me. > A > game is an artificial world where ordinary rules need not apply. In the > artifical context of a game, the rules of the game tell you what is right and > wrong. The rule-makers of a game are free to repeal the the Geneva Convention or > the law of Gravity, if they feel it makes the game more enjoyable. > Indeed, the rule-makers are free to make the rules, and if they wish to make the one thing right and the other wrong, they are free to do so. But we are free to discuss those decisions, and one way in which we are discussing them is by saying that one is "more wrong" than another, and that the one makes the game less enjoyable. But when we are discussing them in that manner, we should be allowed to discuss them freely, and not merely repeat, "because that is how it is". Richard continues to argue this problem, but usually his response is "because that is how it is". For just once, he gave a different reply, "because it is lying". So I ask him "why is one form of lying more wrong than another", and here you are with the reply "because the WBF say so". This is circular reasoning, even when it is not done by just one reasoner. So I ask you, Nigel, why you agree with the WBF that the second form of lying is "wronger" than the first. > Have you ever played GTA? > _______________________________________________ > Blml mailing list > Blml at rtflb.org > http://lists.rtflb.org/mailman/listinfo/blml > > > > > No virus found in this incoming message. > Checked by AVG - www.avg.com > Version: 9.0.872 / Virus Database: 271.1.1/3374 - Release Date: 01/11/11 20:34:00 > -- Herman De Wael Wilrijk Antwerpen Belgium From ehaa at starpower.net Wed Jan 12 15:16:23 2011 From: ehaa at starpower.net (Eric Landau) Date: Wed, 12 Jan 2011 09:16:23 -0500 Subject: [BLML] EBU L&EC meeting 3rd November 2010 In-Reply-To: References: Message-ID: A very thoughtful job, but I have one quibble... On Jan 11, 2011, at 10:17 PM, richard.hills at immi.gov.au wrote: > 5.11 New code of Laws > Mr Endicott said that the first preliminary discussions about the > 2017 code of laws was scheduled for about six months time. Any > suggestions for law amendments should be sent directly to Grattan. > > +=+=+=+=+=+=+=+=+=+=+=+=+=+=+=+=+=+=+=+=+=+=+=+=+=+=+=+=+=+=+=+=+ > > 2018 LAW 20 - REVIEW AND EXPLANATION OF CALLS > > A. Call Not Clearly Recognised > > A player may require clarification forthwith if she is in doubt > what call has been made. > > B. Review of Auction during Auction Period > > During the auction period, a player is entitled to have all > previous calls restated* when it is her turn to call, unless she > is required by law to pass. Alerts should be included when > responding to the request. A player may not ask for a partial > review of previous calls and may not halt the review before it is > completed. > > C. Review after Final Pass > > 1. After the final pass either defender has the right to ask if > it is her opening lead (see Laws 47E and 41). > > 2. Declarer** or either defender may, at her first turn to play, > require all previous calls to be restated*. (See Laws 41B and > 41C). As in B the player may not ask for only a partial > restatement or halt the review. > > D. Who May Review the Auction > > A request to have calls restated* shall be responded to only by > an opponent (unless the Director at the table rules otherwise). > > E. Correction of Error in Review > > All players, including dummy or a player required by law to > pass, are responsible for prompt correction of errors in > restatement* (see Law 12C1 when an uncorrected review causes > damage). > > F. Explanation of Calls > > 1. (a) During the auction before the final pass, any player may > request, but only at her own turn to call, honest*** explanations > of the opponents' prior auction. She is entitled to know about > calls actually made, about relevant alternative calls available > that were not made, and about relevant inferences from the choice > of action where these are matters of partnership understanding. ************************************************************************ *********** > She is not entitled to know about calls that might be made in an > entirely different auction (for example, if the pre-existing > mutual partnership understanding of the opponents in a particular > auction is that 4NT promises 5/5 in the minors, the player is not > entitled at that time to know the opponents' responses to their > 4NT Keycard Blackwood convention). Why not? This seems unnecessary and problematic. It would mean either abandoning the established right of a player to peruse his opponents' convention card freely at his turn to bid, or introducing for the first time the notion that one's right to disclosure depends on what the opponents happen to have written on their CC. It would also explicitly repudiate the "Kaplan paradigm" of full disclosure, leaving us without an overarching set of principles to guide us in those inevitable situations not explicity and unambiguously covered by the rules. ************************************************************************ ************* > (b) Except on the instruction of the Director (who may, in > accordance with guidance from her Regulating Authority, direct a > player to temporarily leave the table) replies should be given by > the partner of the player who made the call in question. The > partner of a player who asks a question may not ask a > supplementary question until her turn to call or play. Law 16 may > apply and the Regulating Authority may establish regulations for > written explanations. > > 2. After the final pass and throughout the play period, either > defender at her own turn to play may request honest*** > explanations of the opposing auction. At her turn to play from > her hand or from dummy declarer may request honest*** > explanations of a defender's call or card play understandings. > Honest*** explanations should be given on a like basis to 1 and > by the partner of the player whose action is explained. > > 3. Under 1 and 2 above a player may ask concerning a single call > but Law 16B1 may apply. > > 4. If a player subsequently realises that her own explanation was > erroneous or incomplete she must call the Director immediately. > The Director applies Law 21B or Law 40B4. > > 5. (a) A player whose partner has given a mistaken explanation > may not correct the error during the auction, nor may she > indicate by any mannerism (as in a shrug, glare, frown etc.) > that a mistake has been made. "Mistaken explanation" here > includes failure to alert or announce as regulations require or > an alert (or an announcement) that regulations do not require. > > (b) Nevertheless, the player must still give honest*** > explanations of her partner's subsequent calls whether or not > those honest*** explanations indirectly indicate to partner her > partner's error. Such an indirect indication is unauthorised > information, so Law 73C and/or Law 75A may apply to partner. > > (c) The player must call the Director and inform her > opponents that, in her opinion, her partner's explanation was > erroneous (see Law 75) but only at her first legal opportunity, > which is > (i) for a defender, at the end of the play. > (ii) for declarer or dummy, after the final pass of the > auction (but before the opening lead is faced, and > preferably before the opening lead is selected). > > 6. If the Director judges that a player has based an action on > misinformation given to him by an opponent see, as appropriate, > Law 21 or Law 47E. > > G. Incorrect Procedure > > 1. (a) Asking a question solely for partner's benefit is an > infraction. The Director applies Law 90 or Law 91. > > (b) Asking a question solely for the purpose of making > either side create unauthorised information is an infraction. > The Director applies Law 90 or Law 91. > > 2. Except as the Regulating Authority allows a player may not > consult her own System Card and notes during the auction and play > periods, but see Law 40B2(b). > > * When the calls are not spoken responders must ensure that it is > clear to an enquiring opponent what calls have been made. > > ** Declarer's first turn to play is from dummy unless accepting an > opening lead out of turn. > > *** Unintentional errors in explanation are honest. Dishonest > intentional misexplanations could well receive Law 72B1 and Law 91 > Disciplinary Penalties. > > 2018 LAW 21 - CALL BASED ON MISINFORMATION > > A. Call Based on Caller's Misunderstanding > > No rectification or redress is due to a player who acts on the > basis of her own misunderstanding. > > B. Call Based on Misinformation from an Opponent > > 1. (a) The Director is to presume Mistaken Explanation rather > than Mistaken Call in the absence of evidence to the contrary. > See Law 75 for a detailed indicative example. > > (b) Failure to alert promptly where an alert is required > by the Regulating Authority is deemed misinformation. > > 2. When it is too late to change a particular call (for > example, in a belated Law 20F4 correction of misinformation) > the Director requires the auction and play to continue. At the > end of play, if the Director judges that the offending side > gained an advantage from the irregularity, then the Director > awards an adjusted score. > > 3. Until the end of the auction period and provided that her > partner has not subsequently called, a player may change a call > without other rectification for her side when the Director judges > that the decision to make the call could well have been influenced > by misinformation given to the player by an opponent (see Law > 17E). At the end of play, if the Director judges that the > offending side gained an advantage from the irregularity, then the > Director awards an adjusted score. > > 4. When a player elects to change a call because of misinformation > (as in 3 preceding), her LHO may then in turn change any > subsequent call she may have made, without other rectification > unless at the end of the hand the Director judges the LHO's > withdrawn call to have conveyed such information as to damage the > non-offending side in which case Law 16D applies. Eric Landau 1107 Dale Drive Silver Spring MD 20910 ehaa at starpower.net From ehaa at starpower.net Wed Jan 12 15:30:31 2011 From: ehaa at starpower.net (Eric Landau) Date: Wed, 12 Jan 2011 09:30:31 -0500 Subject: [BLML] EBU L&EC meeting 3rd November 2010 In-Reply-To: References: Message-ID: On Jan 11, 2011, at 11:36 PM, richard.hills at immi.gov.au wrote: > 5.11 New code of Laws > Mr Endicott said that the first preliminary discussions about the > 2017 code of laws was scheduled for about six months time. Any > suggestions for law amendments should be sent directly to Grattan. > > +=+=+=+=+=+=+=+=+=+=+=+=+=+=+=+=+=+=+=+=+=+=+=+=+=+=+=+=+=+=+=+=+ > > 2018 LAW 12 - SCORE ADJUSTMENT, DIRECTOR'S DISCRETIONARY POWERS > > A. Power to Award an Adjusted Score > > On the application of a player within the period established > under Law 92B, or on his own initiative, the Director may award > an adjusted score when these Laws empower her to do so (in team > play see Law 86). This includes: > > 1. (a) The Director may award an adjusted score when she judges > that a rectification provided in these Laws does not redress > damage (see B below) to the non-offending side for the particular > type of infraction committed at the same table by the offending > side. > > (b) The Director may not award an adjusted score on the ground > that the rectification provided in these Laws is either unduly > severe to the offending side or unduly advantageous to the non- > offending side. > > 2. The Director awards an artificial adjusted score if no > rectification can be made that will permit normal play of the > board (see C2 below). > > 3. The Director may award an adjusted score if there has been an > incorrect rectification of an irregularity. > > B. Objective of Score Adjustment > > The objective of score adjustment is to redress damage to a > non-offending side and to remove all advantages gained by an > offending side through its infraction. Damage exists when, > because of an infraction, a non-offending side obtains a table > result less favourable than would have been the expectation of > the non-offending side at the instant before the infraction ? but > see C1(b). > > C. Awarding an Adjusted Score > > 1. (a) When after an irregularity the Director is empowered by > these laws to adjust a score and is able to award an assigned > adjusted score, she does so. Such a score replaces the score > obtained in play. > > (b) If subsequent to the irregularity the non-offending > side has contributed to its own damage by a ridiculous* error > (unrelated to the infraction), or by wild or gambling action, > then the non-offending side receives partial rectification only, > ignoring such part of the damage as is self-inflicted. The > offending side, however, receives all of the score that was its > expectation at the instant before its infraction (i.e. the non- > offending side's subsequent error is irrelevant for the > calculation of the score awarded to the offending side by the > Director). > > (c) In order to do equity, and unless the Regulating > Authority forbids it, an assigned adjusted score may be weighted > to reflect the probabilities of a number of potential results. > (But an illegally obtained result, potential or actual, must not > be included in the weightings.) > > (d) If the possibilities are numerous or not obvious, the > Director may award an artificial adjusted score. > > (e) In its discretion the Regulating Authority may apply all > or part of the following procedure in place of (c): > > (i) The score assigned in place of the actual score for a > non-offending side is the most favourable result that was likely > at the instant before the infraction. > > (ii) For an offending side the score assigned is the most > unfavourable result that was at all probable at the instant > before the infraction. > > (f) The scores awarded to the two sides need not balance. > > 2. (a) When owing to an irregularity no result can be obtained > [and see C1(d)] the Director awards an artificial adjusted score > according to responsibility for the irregularity: average minus > (at most 40% of the available matchpoints in pairs) to a > contestant directly at fault, average (50% in pairs) to a > contestant only partly at fault, and average plus (at least 60% > in pairs) to a contestant in no way at fault. > > (b) When the Director awards an artificial adjusted score of > average plus or minus at international match points that score > is normally plus or minus 3 imps, but this may be varied as Law > 86A allows. > > (c) The foregoing is modified for a non-offending contestant > that obtains a session score exceeding 60% of the available > matchpoints or for an offending contestant that obtains a > session score that is less than 40% of the available matchpoints > (or the equivalent in imps). Such contestants are awarded the > percentage obtained (or the equivalent in imps) on the other > boards of that session. This clause is merely a default, and > may be freely amended by the Regulating Authority. (For example, > a regulation may be created capping the number of average plus > scores that a contestant may gain in a single session.) > > 3. In individual events the Director enforces the rectifications > in these Laws, and the provisions requiring the award of > adjusted scores, equally against both members of the offending > side even though only one of them may be responsible for the > irregularity. But the Director shall not award a procedural > penalty against the offender's partner if of the opinion that > she is in no way to blame. > > 4. When the Director awards non-balancing adjusted scores in > knockout play, each contestant's score on the board is > calculated separately and the average of them is assigned to > each. > > * "Ridiculous" is assessed by the class of player involved. > For a novice player a "ridiculous" error would be extremely > rare (e.g. the rare major mechanical error of revoking). We've devoted a lot of virtual ink to debating the moral and practical aspects of having laws by which the outcome of an adjudication depends on the adjudicators assessment of the "class of player involved". I am firmly in the camp which deprecates such laws. I would argue for substituting something like "an irrational action" for "a ridiculous error" in L12C1(b) above, and omitting the footnote. Eric Landau 1107 Dale Drive Silver Spring MD 20910 ehaa at starpower.net From ehaa at starpower.net Wed Jan 12 15:44:18 2011 From: ehaa at starpower.net (Eric Landau) Date: Wed, 12 Jan 2011 09:44:18 -0500 Subject: [BLML] EBU L&EC meeting 3rd November 2010 In-Reply-To: References: <4D2C2DFC.8030502@skynet.be> Message-ID: <6EA476DD-1155-4C74-81F1-B046D3EC8E26@starpower.net> On Jan 11, 2011, at 11:47 PM, Jerry Fusselman wrote: > On Tue, Jan 11, 2011 at 4:16 AM, Herman De Wael wrote: > >> I believe that one of the most important discussions the WBF should >> start with is the question of whether or not the opponents are >> entitled >> to the knowledge that there has been a misunderstanding. > > I am probably going to echo what Eric said here, though perhaps in a > different way. I feel that Herman's question is a little vague. Let > me try to give answer with a little bit of nuance to see what people > think. Perhaps this answer will help eliminate some of the vagueness: > > 1. The opponents are not entitled to hear statements during or after > the auction listing all personal errors in bidding if there is no way > the opponents could deduce them with perfect knowledge of your > partnership understandings. > > 2. An opponent is entitled to know each of your misunderstandings > that he could have deduced using the contents of his own hand, the > auction, and a complete description of your partnership > understandings. > > Does this seem like a good way to answer Herman's question? It is > just one possibility, and actually, I share Herman's question. I don't support the notion of a positive "entitlement" to knowledge of an opponents' misunderstanding, even if limited. I'd answer the question as follows: 1. A player is entitled to a complete description of his opponents' partnership understandings. 2. A player who may deduce, from the contents of his own hand, the auction, and the complete description of his opponents' partnership understandings to which he is entitled, that his opponents have had a bidding misunderstanding may freely use that knowledge to his own advantage. Eric Landau 1107 Dale Drive Silver Spring MD 20910 ehaa at starpower.net From gordonrainsford at btinternet.com Wed Jan 12 15:47:59 2011 From: gordonrainsford at btinternet.com (Gordon Rainsford) Date: Wed, 12 Jan 2011 14:47:59 +0000 Subject: [BLML] EBU L&EC meeting 3rd November 2010 In-Reply-To: <99220844-B551-4B55-B5F1-E27A4E76E32B@starpower.net> References: <4D2C2DFC.8030502@skynet.be> <99220844-B551-4B55-B5F1-E27A4E76E32B@starpower.net> Message-ID: On 11 Jan 2011, at 15:39, Eric Landau wrote: > On Jan 11, 2011, at 5:16 AM, Herman De Wael wrote: > >> I believe that one of the most important discussions the WBF should >> start with is the question of whether or not the opponents are >> entitled >> to the knowledge that there has been a misunderstanding. > > I don't think "entitled" is the word Herman is looking for. I think it probably is. > Nobody (I hope) would propose that partnerships have a positive > obligation > to inform their opponents that they are having a misunderstanding. The question as it arises is whether, when assigning an adjusted score on the basis of misinformation, we assume the NOS know their opponents have had a misunderstanding. This was discussed at some length at San Remo, and the impression I have is that many countries assign on the basis of the misunderstanding being known, while the view of the WBF CTD (and the view of the EBU) seems to be that there is no such entitlement. The problem with this latter view is that those who fail to correct misinformation in time may well be better off than they would have been if they had corrected in time for their opponents to change a call. Imagine an uncontested auction 2H-4H. The 2H bid is described as strong, but their agreement is in fact that it is weak. If this is corrected in time, the member of the NOS in the pass-out seat may well come in with a double or a bid. If it is not corrected, redress will be made by way of an adjustment, and it may well be that neither member of the NOS would have a bid or double to make if they had the correct information, unless they were entitled to the knowledge of the misinformation. Gordon Rainsford From svenpran at online.no Wed Jan 12 16:06:07 2011 From: svenpran at online.no (Sven Pran) Date: Wed, 12 Jan 2011 16:06:07 +0100 Subject: [BLML] EBU L&EC meeting 3rd November 2010 In-Reply-To: References: <4D2C2DFC.8030502@skynet.be> <99220844-B551-4B55-B5F1-E27A4E76E32B@starpower.net> Message-ID: <000301cbb26a$3e7873f0$bb695bd0$@no> On Behalf Of Gordon Rainsford > > Imagine an uncontested auction 2H-4H. The 2H bid is described as strong, but > their agreement is in fact that it is weak. If this is corrected in time, the member of > the NOS in the pass-out seat may well come in with a double or a bid. If it is not > corrected, redress will be made by way of an adjustment, and it may well be that > neither member of the NOS would have a bid or double to make if they had the > correct information, unless they were entitled to the knowledge of the > misinformation. Such adjustment should be assessed on the presumption what NOS would have done had they had correct information in time. So NOS should not under any circumstances be worse off with an assigned adjusted score than had they had the correction in time to possibly change call during the auction. This has nothing to do with whether they are entitled to knowledge of a misunderstanding. From ehaa at starpower.net Wed Jan 12 16:13:40 2011 From: ehaa at starpower.net (Eric Landau) Date: Wed, 12 Jan 2011 10:13:40 -0500 Subject: [BLML] EBU L&EC meeting 3rd November 2010 In-Reply-To: References: <4D2C2DFC.8030502@skynet.be> <99220844-B551-4B55-B5F1-E27A4E76E32B@starpower.net> Message-ID: On Jan 12, 2011, at 9:47 AM, Gordon Rainsford wrote: > On 11 Jan 2011, at 15:39, Eric Landau wrote: > >> On Jan 11, 2011, at 5:16 AM, Herman De Wael wrote: >> >>> I believe that one of the most important discussions the WBF should >>> start with is the question of whether or not the opponents are >>> entitled >>> to the knowledge that there has been a misunderstanding. >> >> I don't think "entitled" is the word Herman is looking for. > > I think it probably is. I (and others might well) read "entitled" (unmodified) to mean that if a player becomes aware of the fact that he is having a bidding misunderstanding he is obliged to inform his opponents. >> Nobody (I hope) would propose that partnerships have a positive >> obligation >> to inform their opponents that they are having a misunderstanding. > > The question as it arises is whether, when assigning an adjusted > score on the basis of misinformation, we assume the NOS know their > opponents have had a misunderstanding. I'd call that "presumptively entitled for purposes of adjudication". > This was discussed at some length at San Remo, and the impression I > have is that many countries assign on the basis of the > misunderstanding being known, while the view of the WBF CTD (and the > view of the EBU) seems to be that there is no such entitlement. > > The problem with this latter view is that those who fail to correct > misinformation in time may well be better off than they would have > been if they had corrected in time for their opponents to change a > call. > > Imagine an uncontested auction 2H-4H. The 2H bid is described as > strong, but their agreement is in fact that it is weak. If this is > corrected in time, the member of the NOS in the pass-out seat may > well come in with a double or a bid. If it is not corrected, redress > will be made by way of an adjustment, and it may well be that neither > member of the NOS would have a bid or double to make if they had the > correct information, unless they were entitled to the knowledge of > the misinformation. The key words here are "if they had the correct information". It should be the responsibility of the adjudicators to make a positive determination as to whether, if they had the correct information, it is "likely" that they would have been able to deduce the misunderstanding, and rule accordingly. Eric Landau 1107 Dale Drive Silver Spring MD 20910 ehaa at starpower.net From nigelguthrie at yahoo.co.uk Wed Jan 12 16:29:51 2011 From: nigelguthrie at yahoo.co.uk (Nigel Guthrie) Date: Wed, 12 Jan 2011 15:29:51 +0000 (GMT) Subject: [BLML] Ecclesiastes [SEC=UNOFFICIAL] In-Reply-To: <4D2DADA1.3040903@skynet.be> References: <4D2D6823.6040802@skynet.be> <139528.76055.qm@web28503.mail.ukl.yahoo.com> <4D2DADA1.3040903@skynet.be> Message-ID: <844406.67262.qm@web28501.mail.ukl.yahoo.com> [Herman de Wael] So I ask you, Nigel, why you agree with the WBF that the second form of lying is "wronger" than the first. [Nigel] Richard misrepresents me. Now Herman. Richard and Herman should get together to design a new war-game: pitching their straw-men against each other in a fight to the death. They have vast armies, so the game won't end before 2018 :) Anyway, I've answered Herman's question several times. *Neither* is intrinsically *wrong*, IMO. Which would be *more fun* is arguable. But that is not the primary bone of contention between us. We simply disagree whether: - you should use argument to persuade law-makers to change the rules of a game. OR - you should resort to infraction or advocating law-breaking to that end. From agot at ulb.ac.be Wed Jan 12 16:43:36 2011 From: agot at ulb.ac.be (Alain Gottcheiner) Date: Wed, 12 Jan 2011 16:43:36 +0100 Subject: [BLML] EBU L&EC meeting 3rd November 2010 In-Reply-To: <6EA476DD-1155-4C74-81F1-B046D3EC8E26@starpower.net> References: <4D2C2DFC.8030502@skynet.be> <6EA476DD-1155-4C74-81F1-B046D3EC8E26@starpower.net> Message-ID: <4D2DCC28.3050700@ulb.ac.be> Le 12/01/2011 15:44, Eric Landau a ?crit : > On Jan 11, 2011, at 11:47 PM, Jerry Fusselman wrote: > >> On Tue, Jan 11, 2011 at 4:16 AM, Herman De Wael wrote: >> >>> I believe that one of the most important discussions the WBF should >>> start with is the question of whether or not the opponents are >>> entitled >>> to the knowledge that there has been a misunderstanding. >> I am probably going to echo what Eric said here, though perhaps in a >> different way. I feel that Herman's question is a little vague. Let >> me try to give answer with a little bit of nuance to see what people >> think. Perhaps this answer will help eliminate some of the vagueness: >> >> 1. The opponents are not entitled to hear statements during or after >> the auction listing all personal errors in bidding if there is no way >> the opponents could deduce them with perfect knowledge of your >> partnership understandings. >> >> 2. An opponent is entitled to know each of your misunderstandings >> that he could have deduced using the contents of his own hand, the >> auction, and a complete description of your partnership >> understandings. >> >> Does this seem like a good way to answer Herman's question? It is >> just one possibility, and actually, I share Herman's question. > I don't support the notion of a positive "entitlement" to knowledge > of an opponents' misunderstanding, even if limited. I'd answer the > question as follows: > > 1. A player is entitled to a complete description of his opponents' > partnership understandings. > > 2. A player who may deduce, from the contents of his own hand, the > auction, and the complete description of his opponents' partnership > understandings to which he is entitled, that his opponents have had a > bidding misunderstanding may freely use that knowledge to his own > advantage. AG : basically agree, but please add to the sources 'the explanations he recieved, even if corrected', i.e. he's allowed to deduce that, since North explained South's bid as something else, North bid the hand under these conditions. From agot at ulb.ac.be Wed Jan 12 16:49:38 2011 From: agot at ulb.ac.be (Alain Gottcheiner) Date: Wed, 12 Jan 2011 16:49:38 +0100 Subject: [BLML] Ecclesiastes [SEC=UNOFFICIAL] In-Reply-To: <844406.67262.qm@web28501.mail.ukl.yahoo.com> References: <4D2D6823.6040802@skynet.be> <139528.76055.qm@web28503.mail.ukl.yahoo.com> <4D2DADA1.3040903@skynet.be> <844406.67262.qm@web28501.mail.ukl.yahoo.com> Message-ID: <4D2DCD92.50908@ulb.ac.be> Le 12/01/2011 16:29, Nigel Guthrie a ?crit : > [Herman de Wael] > So I ask you, Nigel, why you agree with the WBF that the second form of lying is > "wronger" than the first. > > > [Nigel] > Richard misrepresents me. Now Herman. Richard and Herman should get together to > design a new war-game: pitching their straw-men against each other in a fight to > the death. They have vast armies, so the game won't end before 2018 :) > > Anyway, I've answered Herman's question several times. *Neither* is > intrinsically *wrong*, IMO. > > Which would be *more fun* is arguable. But that is not the primary bone of > contention between us. We simply disagree whether: > > - you should use argument to persuade law-makers to change the rules of a game. > OR > - you should resort to infraction or advocating law-breaking to that end. AG : please allow me to suggest a third way : you should experiment with law-breaking, at your own risks, not to win but to see whether it would be a good idea to make it non-law-breaking anymore. (e.g. testing opponents' fellings, the global effect on the tournament's equity, on partnership harmony, time won or lost, ...) Kind of in-vivo test. From richard.willey at gmail.com Wed Jan 12 16:50:24 2011 From: richard.willey at gmail.com (richard willey) Date: Wed, 12 Jan 2011 10:50:24 -0500 Subject: [BLML] EBU L&EC meeting 3rd November 2010 In-Reply-To: <4D2DCC28.3050700@ulb.ac.be> References: <4D2C2DFC.8030502@skynet.be> <6EA476DD-1155-4C74-81F1-B046D3EC8E26@starpower.net> <4D2DCC28.3050700@ulb.ac.be> Message-ID: >From my own perspective, I think that the Laws for Online Bridge are the ones the deserve/require the most serious attention. -- I think back to the halcyon dates of my youth, when indeterminate Hessians had something to do with the Revolutionary War, where conjugate priors were monks who had broken their vows, and the expression (X'X)^-1(X'Y) was greek Those were simpler times -------------- next part -------------- An HTML attachment was scrubbed... URL: http://lists.rtflb.org/pipermail/blml/attachments/20110112/215e07d7/attachment-0001.html From Hermandw at skynet.be Wed Jan 12 16:55:16 2011 From: Hermandw at skynet.be (Herman De Wael) Date: Wed, 12 Jan 2011 16:55:16 +0100 Subject: [BLML] EBU L&EC meeting 3rd November 2010 In-Reply-To: <000301cbb26a$3e7873f0$bb695bd0$@no> References: <4D2C2DFC.8030502@skynet.be> <99220844-B551-4B55-B5F1-E27A4E76E32B@starpower.net> <000301cbb26a$3e7873f0$bb695bd0$@no> Message-ID: <4D2DCEE4.2040606@skynet.be> Sven Pran wrote: > On Behalf Of Gordon Rainsford > > Such adjustment should be assessed on the presumption what NOS would have > done had they had correct information in time. > > So NOS should not under any circumstances be worse off with an assigned > adjusted score than had they had the correction in time to possibly change > call during the auction. This has nothing to do with whether they are > entitled to knowledge of a misunderstanding. > The problem with your view here, Sven, is that you appear to be saying they ARE entitled to the knowledge of the misunderstanding, while in fact you are saying just the opposite. Since the current laws envisage that a correction need only be given after the bidding is over, it turns out that, during the bidding, the opponents do not get this information. The case is clearest when screens are in use. -- Herman De Wael Wilrijk Antwerpen Belgium From Hermandw at skynet.be Wed Jan 12 16:59:57 2011 From: Hermandw at skynet.be (Herman De Wael) Date: Wed, 12 Jan 2011 16:59:57 +0100 Subject: [BLML] EBU L&EC meeting 3rd November 2010 In-Reply-To: <4D2DCC28.3050700@ulb.ac.be> References: <4D2C2DFC.8030502@skynet.be> <6EA476DD-1155-4C74-81F1-B046D3EC8E26@starpower.net> <4D2DCC28.3050700@ulb.ac.be> Message-ID: <4D2DCFFD.20808@skynet.be> Alain Gottcheiner wrote: > AG : basically agree, but please add to the sources 'the explanations he > recieved, even if corrected', i.e. he's allowed to deduce that, since > North explained South's bid as something else, North bid the hand under > these conditions. No, I don't think that is true. South bids 2He, weak, as is their system. North bids 4He, but explains 2He as strong. Opponents are entitled to know the system, which includes that North's 4He is based on a weakish hand, but they are not (IMO) entitled to know that North misbid. They are not entitled to know what North thought the system was. But, that is my opinion. Alains should not go around agreeing with Eric and me, and then go spouting the exact opposite opinion. Agree, Eric? -- Herman De Wael Wilrijk Antwerpen Belgium From PeterEidt at t-online.de Wed Jan 12 17:00:40 2011 From: PeterEidt at t-online.de (Peter Eidt) Date: Wed, 12 Jan 2011 17:00:40 +0100 Subject: [BLML] =?utf-8?q?Ecclesiastes_=5BSEC=3DUNOFFICIAL=5D?= In-Reply-To: <4D2DCD92.50908@ulb.ac.be> References: <4D2DCD92.50908@ulb.ac.be> Message-ID: <1Pd37p-1j8CMC0@fwd07.aul.t-online.de> From: Alain Gottcheiner > Le 12/01/2011 16:29, Nigel Guthrie a ?crit : > > [Herman de Wael] > > So I ask you, Nigel, why you agree with the WBF that the second form > > of lying is "wronger" than the first. > > > > > > > > [Nigel] > > Richard misrepresents me. Now Herman. Richard and Herman should get > > together to design a new war-game: pitching their straw-men against > > each other in a fight to the death. They have vast armies, so the > > game won't end before 2018 :) > > > > Anyway, I've answered Herman's question several times. *Neither* is > > intrinsically *wrong*, IMO. > > > > > > Which would be *more fun* is arguable. But that is not the primary > > bone of contention between us. We simply disagree whether: > > > > > > - you should use argument to persuade law-makers to change the rules > > of a game. > > OR > > - you should resort to infraction or advocating law-breaking to that > > end. > > > AG : please allow me to suggest a third way : > > you should experiment with law-breaking, at your own risks, not to win > but to see whether it would be a good idea to make it non-law-breaking > anymore. > (e.g. testing opponents' fellings, the global effect on the > tournament's equity, on partnership harmony, time won or lost, ...) > Kind of in-vivo test. [Peter] But you shall never do this within a game or tournament which is played under the Rules of Duplicated Bridge 2007. Im my game there is Law 72 B1: "A player must not infringe a law intentionally, even if there is a prescribed rectification he is willing to accept." From Hermandw at skynet.be Wed Jan 12 17:02:33 2011 From: Hermandw at skynet.be (Herman De Wael) Date: Wed, 12 Jan 2011 17:02:33 +0100 Subject: [BLML] Ecclesiastes [SEC=UNOFFICIAL] In-Reply-To: <844406.67262.qm@web28501.mail.ukl.yahoo.com> References: <4D2D6823.6040802@skynet.be> <139528.76055.qm@web28503.mail.ukl.yahoo.com> <4D2DADA1.3040903@skynet.be> <844406.67262.qm@web28501.mail.ukl.yahoo.com> Message-ID: <4D2DD099.3000907@skynet.be> Nigel Guthrie wrote: > [Herman de Wael] > So I ask you, Nigel, why you agree with the WBF that the second form of lying is > "wronger" than the first. > > > [Nigel] > Richard misrepresents me. Now Herman. Richard and Herman should get together to > design a new war-game: pitching their straw-men against each other in a fight to > the death. They have vast armies, so the game won't end before 2018 :) > > Anyway, I've answered Herman's question several times. *Neither* is > intrinsically *wrong*, IMO. > If that is your opinion, then please stay out of our discussion. > Which would be *more fun* is arguable. But that is not the primary bone of > contention between us. We simply disagree whether: It is the primary bone of contention between Richard and myself. And you stating that it doesn't matter does not help me get Richard to see the importance of what I am trying to make him see. Given that both options are open to the WBF (as are 37 others), why should one be more "wrong" or "right" than another. And simply stating that the WBF chose it will not do. -- Herman De Wael Wilrijk Antwerpen Belgium From PeterEidt at t-online.de Wed Jan 12 17:08:36 2011 From: PeterEidt at t-online.de (Peter Eidt) Date: Wed, 12 Jan 2011 17:08:36 +0100 Subject: [BLML] =?utf-8?q?Ecclesiastes_=5BSEC=3DUNOFFICIAL=5D?= In-Reply-To: <4D2DD099.3000907@skynet.be> References: <4D2DD099.3000907@skynet.be> Message-ID: <1Pd3FV-021OIi0@fwd08.aul.t-online.de> From: Herman De Wael > Given that both > options are open to the WBF (as are 37 others), why should one be more > "wrong" or "right" than another. And simply stating that the WBF chose > it will not do. [Peter] Veto; it surely does. The WBF is the only authority that defines the laws (unless it gives an option to an Regulating Authority - which it does not give in this context). From Hermandw at skynet.be Wed Jan 12 17:26:41 2011 From: Hermandw at skynet.be (Herman De Wael) Date: Wed, 12 Jan 2011 17:26:41 +0100 Subject: [BLML] Ecclesiastes [SEC=UNOFFICIAL] In-Reply-To: <1Pd3FV-021OIi0@fwd08.aul.t-online.de> References: <4D2DD099.3000907@skynet.be> <1Pd3FV-021OIi0@fwd08.aul.t-online.de> Message-ID: <4D2DD641.80800@skynet.be> Peter Eidt wrote: > From: Herman De Wael >> Given that both >> options are open to the WBF (as are 37 others), why should one be more >> "wrong" or "right" than another. And simply stating that the WBF chose >> it will not do. > > [Peter] > Veto; it surely does. > The WBF is the only authority that defines the laws (unless it > gives an option to an Regulating Authority - which it does not > give in this context). > Not when Richard and I re debating the relative merits of two potential sets of laws. And I hate it when people drop into discussions saying something which I exactly denied two posts previous. -- Herman De Wael Wilrijk Antwerpen Belgium From blml at arcor.de Wed Jan 12 17:38:01 2011 From: blml at arcor.de (Thomas Dehn) Date: Wed, 12 Jan 2011 17:38:01 +0100 (CET) Subject: [BLML] Ecclesiastes [SEC=UNOFFICIAL] In-Reply-To: <1Pd3FV-021OIi0@fwd08.aul.t-online.de> References: <1Pd3FV-021OIi0@fwd08.aul.t-online.de> <4D2DD099.3000907@skynet.be> Message-ID: <590903121.20221.1294850281161.JavaMail.ngmail@webmail12.arcor-online.net> Peter Eidt wrote: > From: Herman De Wael > > Given that both > > options are open to the WBF (as are 37 others), why should one be more > > "wrong" or "right" than another. And simply stating that the WBF chose > > it will not do. > > [Peter] > Veto; it surely does. > The WBF is the only authority that defines the laws (unless it > gives an option to an Regulating Authority - which it does not > give in this context). The WBF position is the official position of the WBF. That does not make it "right". (and neither "wrong") For example, for many decades the official position of the Roman Catholic Church was that condoms cause HIV. Thomas From blml at arcor.de Wed Jan 12 19:32:21 2011 From: blml at arcor.de (Thomas Dehn) Date: Wed, 12 Jan 2011 19:32:21 +0100 (CET) Subject: [BLML] EBU L&EC meeting 3rd November 2010 In-Reply-To: References: Message-ID: <2120355928.22652.1294857141249.JavaMail.ngmail@webmail18.arcor-online.net> Eric Landau > On Jan 11, 2011, at 11:36 PM, richard.hills at immi.gov.au wrote: > > > 5.11 New code of Laws > > Mr Endicott said that the first preliminary discussions about the > > 2017 code of laws was scheduled for about six months time. Any > > suggestions for law amendments should be sent directly to Grattan. > > > > +=+=+=+=+=+=+=+=+=+=+=+=+=+=+=+=+=+=+=+=+=+=+=+=+=+=+=+=+=+=+=+=+ > > > > 2018 LAW 12 - SCORE ADJUSTMENT, DIRECTOR'S DISCRETIONARY POWERS > > > > A. Power to Award an Adjusted Score > > > > On the application of a player within the period established > > under Law 92B, or on his own initiative, the Director may award > > an adjusted score when these Laws empower her to do so (in team > > play see Law 86). This includes: > > > > 1. (a) The Director may award an adjusted score when she judges > > that a rectification provided in these Laws does not redress > > damage (see B below) to the non-offending side for the particular > > type of infraction committed at the same table by the offending > > side. > > > > (b) The Director may not award an adjusted score on the ground > > that the rectification provided in these Laws is either unduly > > severe to the offending side or unduly advantageous to the non- > > offending side. > > > > 2. The Director awards an artificial adjusted score if no > > rectification can be made that will permit normal play of the > > board (see C2 below). > > > > 3. The Director may award an adjusted score if there has been an > > incorrect rectification of an irregularity. > > > > B. Objective of Score Adjustment > > > > The objective of score adjustment is to redress damage to a > > non-offending side and to remove all advantages gained by an > > offending side through its infraction. Damage exists when, > > because of an infraction, a non-offending side obtains a table > > result less favourable than would have been the expectation of > > the non-offending side at the instant before the infraction ? but > > see C1(b). > > > > C. Awarding an Adjusted Score > > > > 1. (a) When after an irregularity the Director is empowered by > > these laws to adjust a score and is able to award an assigned > > adjusted score, she does so. Such a score replaces the score > > obtained in play. > > > > (b) If subsequent to the irregularity the non-offending > > side has contributed to its own damage by a ridiculous* error > > (unrelated to the infraction), or by wild or gambling action, > > then the non-offending side receives partial rectification only, > > ignoring such part of the damage as is self-inflicted. The > > offending side, however, receives all of the score that was its > > expectation at the instant before its infraction (i.e. the non- > > offending side's subsequent error is irrelevant for the > > calculation of the score awarded to the offending side by the > > Director). > > > > (c) In order to do equity, and unless the Regulating > > Authority forbids it, an assigned adjusted score may be weighted > > to reflect the probabilities of a number of potential results. > > (But an illegally obtained result, potential or actual, must not > > be included in the weightings.) > > > > (d) If the possibilities are numerous or not obvious, the > > Director may award an artificial adjusted score. > > > > (e) In its discretion the Regulating Authority may apply all > > or part of the following procedure in place of (c): > > > > (i) The score assigned in place of the actual score for a > > non-offending side is the most favourable result that was likely > > at the instant before the infraction. > > > > (ii) For an offending side the score assigned is the most > > unfavourable result that was at all probable at the instant > > before the infraction. > > > > (f) The scores awarded to the two sides need not balance. > > > > 2. (a) When owing to an irregularity no result can be obtained > > [and see C1(d)] the Director awards an artificial adjusted score > > according to responsibility for the irregularity: average minus > > (at most 40% of the available matchpoints in pairs) to a > > contestant directly at fault, average (50% in pairs) to a > > contestant only partly at fault, and average plus (at least 60% > > in pairs) to a contestant in no way at fault. > > > > (b) When the Director awards an artificial adjusted score of > > average plus or minus at international match points that score > > is normally plus or minus 3 imps, but this may be varied as Law > > 86A allows. > > > > (c) The foregoing is modified for a non-offending contestant > > that obtains a session score exceeding 60% of the available > > matchpoints or for an offending contestant that obtains a > > session score that is less than 40% of the available matchpoints > > (or the equivalent in imps). Such contestants are awarded the > > percentage obtained (or the equivalent in imps) on the other > > boards of that session. This clause is merely a default, and > > may be freely amended by the Regulating Authority. (For example, > > a regulation may be created capping the number of average plus > > scores that a contestant may gain in a single session.) > > > > 3. In individual events the Director enforces the rectifications > > in these Laws, and the provisions requiring the award of > > adjusted scores, equally against both members of the offending > > side even though only one of them may be responsible for the > > irregularity. But the Director shall not award a procedural > > penalty against the offender's partner if of the opinion that > > she is in no way to blame. > > > > 4. When the Director awards non-balancing adjusted scores in > > knockout play, each contestant's score on the board is > > calculated separately and the average of them is assigned to > > each. > > > > * "Ridiculous" is assessed by the class of player involved. > > For a novice player a "ridiculous" error would be extremely > > rare (e.g. the rare major mechanical error of revoking). > > We've devoted a lot of virtual ink to debating the moral and > practical aspects of having laws by which the outcome of an > adjudication depends on the adjudicators assessment of the "class of > player involved". I am firmly in the camp which deprecates such > laws. I would argue for substituting something like "an irrational > action" for "a ridiculous error" in L12C1(b) above, and omitting the > footnote. We have debated the existing "irrational" in L70 many times. "irrational", without a detailed definition of "irrational", is not sufficiently clear to convey the intended meaning. Thomas From nigelguthrie at yahoo.co.uk Wed Jan 12 19:59:40 2011 From: nigelguthrie at yahoo.co.uk (Nigel Guthrie) Date: Wed, 12 Jan 2011 18:59:40 +0000 (GMT) Subject: [BLML] Ecclesiastes [SEC=UNOFFICIAL] In-Reply-To: <590903121.20221.1294850281161.JavaMail.ngmail@webmail12.arcor-online.net> References: <1Pd3FV-021OIi0@fwd08.aul.t-online.de> <4D2DD099.3000907@skynet.be> <590903121.20221.1294850281161.JavaMail.ngmail@webmail12.arcor-online.net> Message-ID: <413175.97644.qm@web28509.mail.ukl.yahoo.com> [Thomas Dehn] For example, for many decades the official position of the Roman Catholic Church was that condoms cause HIV. [Nigel] Some of official RC policies on sex have been and are questionable but that one is news to me. (I'm RC). Please supply a reference, Thomas. From jfusselman at gmail.com Wed Jan 12 20:14:59 2011 From: jfusselman at gmail.com (Jerry Fusselman) Date: Wed, 12 Jan 2011 13:14:59 -0600 Subject: [BLML] Ecclesiastes Message-ID: [Richard Hills] No. Indeed, definitely not. As Nigel Guthrie has pointed out, the changes proposed by Herman De Wael are so fundamental that if ever enacted (which fortunately will never happen) bridge could no longer be called bridge. [Nigel] I didn't point that out because I don't believe it's true. Bridge is what official law-makers say it is. [Jerry] Richard's style is often to intentionally misstate others' positions. Apparently, this is for humorous effect---what else could it be for? I don't like this aspect of his style, but that's just me. What I wish he would do, when he finds the impulse to do this irresistible, is show us exactly where he is being funny, so that we are not misled and waste time in the way that happened in this thread. For example, in the post Nigel quotes, Richard Hills made two intentional misattributions in two consecutive paragraphs: Richard Hills: No. Indeed, definitely not. As Nigel Guthrie has pointed out, the changes proposed by Herman De Wael are so fundamental that if ever enacted (which fortunately will never happen) bridge could no longer be called bridge. Harald Skjaran: >The way Richard H. responds, it's like fighting windmills. Richard Hills: Yes, I agree that Herman De Wael can aptly be described as Don Quixote tilting at windmills, thanks to his futile war for a (now closed) loophole in the Lawbook. [Jerry Fusselman] But I guess this is not a complete fix, because that would still leave Richard Hills's wild exaggerations in place. Maybe both of Richard Hill's paragraphs were meant to funny, not meant to be seriously considered. Who can tell? -------------- next part -------------- An HTML attachment was scrubbed... URL: http://lists.rtflb.org/pipermail/blml/attachments/20110112/a1be9c2d/attachment.html From swillner at nhcc.net Wed Jan 12 20:19:27 2011 From: swillner at nhcc.net (Steve Willner) Date: Wed, 12 Jan 2011 14:19:27 -0500 Subject: [BLML] EBU L&EC meeting 3rd November 2010 In-Reply-To: References: Message-ID: <4D2DFEBF.9010104@nhcc.net> On 1/12/2011 9:16 AM, Eric Landau wrote: > ... would also explicitly repudiate the "Kaplan paradigm" of full disclosure, That paradigm, alas, seems to have vanished in 2007. From swillner at nhcc.net Wed Jan 12 20:42:18 2011 From: swillner at nhcc.net (Steve Willner) Date: Wed, 12 Jan 2011 14:42:18 -0500 Subject: [BLML] Alain's case revisited In-Reply-To: <4D2B2CC8.2060308@ulb.ac.be> References: <4D2B0857.7050804@skynet.be> <4D2AC556.9070509@skynet.be> <4D2AC9E7.6060208@aol.com> <4D2AE13B.10605@skynet.be> <334994.97415.qm@web28503.mail.ukl.yahoo.com> <1391468851.32037.1294667187404.JavaMail.ngmail@webmail08.arcor-online.net> <4D2B1D0C.7090801@skynet.be> <4D2B2CC8.2060308@ulb.ac.be> Message-ID: <4D2E041A.8070508@nhcc.net> On 1/10/2011 10:59 AM, Alain Gottcheiner wrote: > This shows well the whole problem with applying L12 : it compels the TD > to twist normal bidding to a great extent. which lead to absurd > positions like imagining that we could reach 6NT. > > Now, using dWS would avoid such problems, of course. The Rubens School is even simpler. > You only need 4 little notebooks. The player making the > bid writes the meaning down on one's own notebook, then shows it to both > opponents in turn if they want to know. Kind of generalizing the > behind-screen procedure. What Alain describes is the online procedure: self-explanations seen only by opponents. The screen procedure would have pairs of opponents showing each other the notebooks (say North and East to each other and South and West to each other). A third procedure, perhaps even better, would have each player showing his notebook only to his own LHO. This could be used with screens now, but it would be awkward. On 1/11/2011 7:50 AM, Thomas Dehn wrote: > assuming the dWS becomes law, then the second alert > is disallowed. As Herman points out, the second alert is optional, not disallowed. Of course any damage from MI will be compensated at the end. In the Rubens school, all _correct_ alerts and explanations are AI, so there's much less need for either MI or UI adjustments. That would be an enormous simplification, though of course there will still be difficult cases. Another nice benefit is that the players will be able to understand the rules. On 1/10/2011 11:07 AM, Sven Pran wrote: > The communication between partners in bridge is limited to the "bridge > language" comprising a total of 38 different (possible) calls Actually, as of 2007 "expected" alerts are AI. They seldom contain useful information, but sometimes they do. From swillner at nhcc.net Wed Jan 12 20:43:18 2011 From: swillner at nhcc.net (Steve Willner) Date: Wed, 12 Jan 2011 14:43:18 -0500 Subject: [BLML] The Monty Hall trap In-Reply-To: <4D05E8D9.9080104@ulb.ac.be> References: <4D01F38B.4050701@ulb.ac.be> <4CFF8F9F.2080106@ulb.ac.be> <1D5D1F5A-8B37-4926-AF1D-5D5AE04FEE36@starpower.net> <4CFD2040.2060809@ulb.ac.be> <711338718.230129.1291656236226.JavaMail.ngmail@webmail15.arcor-online.net> <612752015.226595.1291658044395.JavaMail.ngmail@webmail16.arcor-online.net> <1868792466.285413.1291751153686.JavaMail.ngmail@webmail14.arcor-online.net> <1558291281.42999.1291927149888.JavaMail.ngmail@webmail12.arcor-online.net> <1870053642.78656.1292000629922.JavaMail.ngmail@webmail08.arcor-online.net> <4D05E8D9.9080104@ulb.ac.be> Message-ID: <4D2E0456.3060801@nhcc.net> On 12/13/2010 4:35 AM, Alain Gottcheiner wrote: > BTW, i know more than one player who uses mini-notrumps (9-11) and woud > be plleased to pass on 12. Isn't this brown sticker? From blml at arcor.de Wed Jan 12 20:55:17 2011 From: blml at arcor.de (Thomas Dehn) Date: Wed, 12 Jan 2011 20:55:17 +0100 (CET) Subject: [BLML] Ecclesiastes [SEC=UNOFFICIAL] Message-ID: <1955796701.25492.1294862117019.JavaMail.ngmail@webmail18.arcor-online.net> Nigel Guthrie wrote: > [Thomas Dehn] > > For example, for many decades the official position of the Roman Catholic > Church was that condoms cause HIV. > > [Nigel] > Some of official RC policies on sex have been and are questionable but that > one is news to me. (I'm RC). Please supply a reference, Thomas. Came in various ways, from the merely incorrect up to the completely absurd. Cardinal Alfonso Lopez Trujillo, as president of the Pontifical Council for the Family (2003): "The Aids virus is roughly 450 times smaller than the spermatozoon. The spermatozoon can easily pass through the 'net' that is formed by the condom." and "Relying on condoms is like betting on your own death" Monsignor Andre Fort, Bishop of Orleans (2009): "You know very well, and all the scientists know it: the AIDS virus is infinitely smaller than a sperm. This is proof that the condom is not a 100 per cent guarantee against AIDS" (Fort retracted) Pope Benedict XVI, 2009, on his first trip to Africa: "You can't resolve it[AIDS] with the distribution of condoms. On the contrary, it increases the problem." Francisco Chimoio, Archbishop of Maputo, Mozambique (2007) "Condoms are not sure because I know that there are two countries in Europe, .... they are making condoms with the virus on purpose. They want to finish with the African people. This is the programme. They want to colonise until up to now. If we are not careful we will finish in one century's time." (He did not name those two countries) Raphael Ndingi Mwana A'Nzeki, Archbishop of Nairobi: "HIV/AIDS is going so fast because of the availability of condoms. they should not be made at all." There also exist some similar John Paul II quotes, only by now those are hard to find. Thomas From richard.hills at immi.gov.au Wed Jan 12 21:19:23 2011 From: richard.hills at immi.gov.au (richard.hills at immi.gov.au) Date: Thu, 13 Jan 2011 07:19:23 +1100 Subject: [BLML] Queensland floods [SEC=UNOFFICIAL] Message-ID: The Australian Bridge Federation is supporting the Premier's Disaster Relief Appeal for the Queensland floods. See: http://www.abf.com.au/about/press/11QLDFloods.html Best wishes Richard Hills Recruitment Section Specialist Recruitment Team Level 5 Aqua, workstation W569, 6223 8453 DIAC Social Club movie tickets -------------------------------------------------------------------- Important Notice: If you have received this email by mistake, please advise the sender and delete the message and attachments immediately. This email, including attachments, may contain confidential, sensitive, legally privileged and/or copyright information. Any review, retransmission, dissemination or other use of this information by persons or entities other than the intended recipient is prohibited. DIAC respects your privacy and has obligations under the Privacy Act 1988. The official departmental privacy policy can be viewed on the department's website at www.immi.gov.au. See: http://www.immi.gov.au/functional/privacy.htm --------------------------------------------------------------------- From richard.hills at immi.gov.au Wed Jan 12 21:43:53 2011 From: richard.hills at immi.gov.au (richard.hills at immi.gov.au) Date: Thu, 13 Jan 2011 07:43:53 +1100 Subject: [BLML] Ecclesiastes [SEC=UNOFFICIAL] In-Reply-To: Message-ID: Ecclesiastes 7:6 As the crackling of thorns under a pot, so is the laughter of a fool. "Richard's style is often to intentionally misstate others' positions." Intentionally misstate, definitely not. Unintentionally misunderstand, occasionally. Stern correction (no longer Best wishes) Richard Hills Recruitment Section Specialist Recruitment Team Level 5 Aqua, workstation W569, 6223 8453 DIAC Social Club movie tickets -------------------------------------------------------------------- Important Notice: If you have received this email by mistake, please advise the sender and delete the message and attachments immediately. This email, including attachments, may contain confidential, sensitive, legally privileged and/or copyright information. Any review, retransmission, dissemination or other use of this information by persons or entities other than the intended recipient is prohibited. DIAC respects your privacy and has obligations under the Privacy Act 1988. The official departmental privacy policy can be viewed on the department's website at www.immi.gov.au. See: http://www.immi.gov.au/functional/privacy.htm --------------------------------------------------------------------- From ehaa at starpower.net Wed Jan 12 22:35:57 2011 From: ehaa at starpower.net (Eric Landau) Date: Wed, 12 Jan 2011 16:35:57 -0500 Subject: [BLML] EBU L&EC meeting 3rd November 2010 In-Reply-To: <4D2DCFFD.20808@skynet.be> References: <4D2C2DFC.8030502@skynet.be> <6EA476DD-1155-4C74-81F1-B046D3EC8E26@starpower.net> <4D2DCC28.3050700@ulb.ac.be> <4D2DCFFD.20808@skynet.be> Message-ID: <87D7A2D5-1268-46D6-BDEE-90E1456D7F78@starpower.net> On Jan 12, 2011, at 10:59 AM, Herman De Wael wrote: > Alain Gottcheiner wrote: > >> AG : basically agree, but please add to the sources 'the >> explanations he >> recieved, even if corrected', i.e. he's allowed to deduce that, since >> North explained South's bid as something else, North bid the hand >> under >> these conditions. > > No, I don't think that is true. > > South bids 2He, weak, as is their system. > North bids 4He, but explains 2He as strong. > Opponents are entitled to know the system, which includes that North's > 4He is based on a weakish hand, but they are not (IMO) entitled to > know > that North misbid. They are not entitled to know what North thought > the > system was. > > But, that is my opinion. > > Alains should not go around agreeing with Eric and me, and then go > spouting the exact opposite opinion. Agree, Eric? I don't think Alain is offering "the exact opposite opinion". He is simply reminding us that the actions of the opponents at the table are "authorized" information, from which deductions may be made at one's own risk, and that this particular context is not an exception. So Herman is correct to say "they are not entitled to know what North thought the system was" (there's that troublesome word "entitled" again!), but Alain is equally correct that if North happens to have told them, they may use the information to their advantage. Eric Landau 1107 Dale Drive Silver Spring MD 20910 ehaa at starpower.net From svenpran at online.no Wed Jan 12 23:28:33 2011 From: svenpran at online.no (Sven Pran) Date: Wed, 12 Jan 2011 23:28:33 +0100 Subject: [BLML] EBU L&EC meeting 3rd November 2010 In-Reply-To: <4D2DCEE4.2040606@skynet.be> References: <4D2C2DFC.8030502@skynet.be> <99220844-B551-4B55-B5F1-E27A4E76E32B@starpower.net> <000301cbb26a$3e7873f0$bb695bd0$@no> <4D2DCEE4.2040606@skynet.be> Message-ID: <000c01cbb2a8$0c9a4dc0$25cee940$@no> On Behalf Of Herman De Wael > > Such adjustment should be assessed on the presumption what NOS would > > have done had they had correct information in time. > > > > So NOS should not under any circumstances be worse off with an > > assigned adjusted score than had they had the correction in time to > > possibly change call during the auction. This has nothing to do with > > whether they are entitled to knowledge of a misunderstanding. > > > > The problem with your view here, Sven, is that you appear to be saying they ARE > entitled to the knowledge of the misunderstanding, while in fact you are saying just > the opposite. Since the current laws envisage that a correction need only be given > after the bidding is over, it turns out that, during the bidding, the opponents do not > get this information. And if a player for instance has cause for asserting that he would have made a different call during the auction had he at that time had the correct description of opponents' auction so far then the Director shall judge what might have been a likely result on the board given no misinformation. If the Director finds that such result would have been better for the non-offending side than the actual result obtained he shall award an assigned adjusted score. > The case is clearest when screens are in use. Sure, but the principles are the same. From svenpran at online.no Wed Jan 12 23:35:04 2011 From: svenpran at online.no (Sven Pran) Date: Wed, 12 Jan 2011 23:35:04 +0100 Subject: [BLML] EBU L&EC meeting 3rd November 2010 In-Reply-To: <4D2DCFFD.20808@skynet.be> References: <4D2C2DFC.8030502@skynet.be> <6EA476DD-1155-4C74-81F1-B046D3EC8E26@starpower.net> <4D2DCC28.3050700@ulb.ac.be> <4D2DCFFD.20808@skynet.be> Message-ID: <000d01cbb2a8$f57f1f70$e07d5e50$@no> On Behalf Of Herman De Wael > Alain Gottcheiner wrote: > > AG : basically agree, but please add to the sources 'the explanations > > he recieved, even if corrected', i.e. he's allowed to deduce that, > > since North explained South's bid as something else, North bid the > > hand under these conditions. > > No, I don't think that is true. > > South bids 2He, weak, as is their system. > North bids 4He, but explains 2He as strong. > Opponents are entitled to know the system, which includes that North's 4He is > based on a weakish hand, but they are not (IMO) entitled to know that North > misbid. They are not entitled to know what North thought the system was. > > But, that is my opinion. Opponents are entitled both to the information that North misexplained 2H as strong and then bid 4He, and also to the correct information that 2He is weak. What they can infer from this "double" information is their own business, but if they can show likely damage from the misexplanation they are eligible to some rectification. (Note: This does not include possible damage from the fact that North misbid because of his own mistaken explanation!) From svenpran at online.no Wed Jan 12 23:40:35 2011 From: svenpran at online.no (Sven Pran) Date: Wed, 12 Jan 2011 23:40:35 +0100 Subject: [BLML] Alain's case revisited In-Reply-To: <4D2E041A.8070508@nhcc.net> References: <4D2B0857.7050804@skynet.be> <4D2AC556.9070509@skynet.be> <4D2AC9E7.6060208@aol.com> <4D2AE13B.10605@skynet.be> <334994.97415.qm@web28503.mail.ukl.yahoo.com> <1391468851.32037.1294667187404.JavaMail.ngmail@webmail08.arcor-online.net> <4D2B1D0C.7090801@skynet.be> <4D2B2CC8.2060308@ulb.ac.be> <4D2E041A.8070508@nhcc.net> Message-ID: <000e01cbb2a9$ba3fc670$2ebf5350$@no> On Behalf Of Steve Willner ............. > On 1/10/2011 11:07 AM, Sven Pran wrote: > > The communication between partners in bridge is limited to the "bridge > > language" comprising a total of 38 different (possible) calls > > Actually, as of 2007 "expected" alerts are AI. They seldom contain useful > information, but sometimes they do. While alerts are AI they are NOT part of the "bridge language" see Law 73A1 From richard.hills at immi.gov.au Thu Jan 13 00:18:24 2011 From: richard.hills at immi.gov.au (richard.hills at immi.gov.au) Date: Thu, 13 Jan 2011 10:18:24 +1100 Subject: [BLML] Expected Alerts [SEC=UNOFFICIAL] Message-ID: Steve Willner: >>Actually, as of 2007 "expected" alerts are AI. They seldom >>contain useful information, but sometimes they do. Sven Pran: >While alerts are AI they are NOT part of the "bridge language" >see Law 73A1 Richard Hills: In my opinion, Steve is wrong and Sven is right. In my opinion by definition an Expected Alert can never contain any useful information. If you expect partner to forget "the system", but pard's Alert tells you that for once she has remembered "the system", then that is an Unexpected Alert. Furthermore, in that case "the system" is NOT your real system. Because you have more reason to be aware of the deviation than have the opponents, Law 40C1 means that you are actually playing a two-way system which incorporates pard's forgetfulness into your methods. (And also Law 40C1 suggests that it would be to your advantage to adopt a new and simpler system.) Best wishes Richard Hills Recruitment Section Specialist Recruitment Team Level 5 Aqua, workstation W569, 6223 8453 DIAC Social Club movie tickets -------------------------------------------------------------------- Important Notice: If you have received this email by mistake, please advise the sender and delete the message and attachments immediately. This email, including attachments, may contain confidential, sensitive, legally privileged and/or copyright information. Any review, retransmission, dissemination or other use of this information by persons or entities other than the intended recipient is prohibited. DIAC respects your privacy and has obligations under the Privacy Act 1988. The official departmental privacy policy can be viewed on the department's website at www.immi.gov.au. See: http://www.immi.gov.au/functional/privacy.htm --------------------------------------------------------------------- From david.j.barton at lineone.net Thu Jan 13 00:18:56 2011 From: david.j.barton at lineone.net (David) Date: Wed, 12 Jan 2011 23:18:56 -0000 Subject: [BLML] Entitlement to knowledge of a misunderstanding?? In-Reply-To: References: <4D2C2DFC.8030502@skynet.be><99220844-B551-4B55-B5F1-E27A4E76E32B@starpower.net> Message-ID: <64044EA383EB48A3AB3FD7B4E0FCD953@Lounge> Let us suppose that at the end of an auction the person on lead asks for an explanation of all calls. In contravention of L20F all the calls are explained by one member of the declaring side. All the explanations are subsequently determined to be correct. However if the correct procedure had been followed a misunderstanding would have been revealed which would have indicated the correct lead to defeat the contract. Now is the defending side entitled to redress? L12B1 Damage exists when, because of an infraction, an innocent side obtains a table result less favourable than would have been the expectation had the infraction not occurred. Or Have the defending side received their entitlement (a correct systemic explanation) and hence have not been damaged? ********************************** david.j.barton at lineone.net ********************************** From svenpran at online.no Thu Jan 13 01:09:37 2011 From: svenpran at online.no (Sven Pran) Date: Thu, 13 Jan 2011 01:09:37 +0100 Subject: [BLML] Entitlement to knowledge of a misunderstanding?? In-Reply-To: <64044EA383EB48A3AB3FD7B4E0FCD953@Lounge> References: <4D2C2DFC.8030502@skynet.be><99220844-B551-4B55-B5F1-E27A4E76E32B@starpower.net> <64044EA383EB48A3AB3FD7B4E0FCD953@Lounge> Message-ID: <000f01cbb2b6$2b3b2700$81b17500$@no> On Behalf Of David Sent: 13. januar 2011 00:19 > Let us suppose that at the end of an auction the person on lead asks for an > explanation of all calls. > In contravention of L20F all the calls are explained by one member of the > declaring side. > All the explanations are subsequently determined to be correct. > However if the correct procedure had been followed a misunderstanding would > have been revealed which would have indicated the correct lead to defeat the > contract. > > Now is the defending side entitled to redress? Short answer: NO Long answer: According to your description the only irregularity seems to be that the explanation of the entire auction is given by the same player rather than by alternating players for each specific call. I usually find it much better to have a summary of the auction presented and explained instead of having the information presented in bits and pieces as is often the result when each call is explained separately. I (for one) prefer for instance being told that a player has shown the distribution 5-4-3-1 rather than "his 1S bid shows at least 4, then his 2H bid shows at least 5 spades and at least 4 hearts and finally he shows exactly 3 diamonds and denies a void in clubs". The fact that all explanations are given by the same player is a violation of the prescription in Law 21 which however says "should", and according to the introduction: "should" do (failure to do it is an infraction jeopardizing the infractor's rights but not often penalized). > > L12B1 Damage exists when, because of an infraction, an innocent side obtains a > table result less favourable than would have been the expectation had the > infraction not occurred. I would like to be told exactly how an otherwise completely correct explanation given by one player instead of by both players in an alternating fashion can result in damage? > Or > > Have the defending side received their entitlement (a correct systemic > explanation) and hence > have not been damaged? Can you point to any incorrect information received by the defending side? From richard.hills at immi.gov.au Thu Jan 13 01:31:02 2011 From: richard.hills at immi.gov.au (richard.hills at immi.gov.au) Date: Thu, 13 Jan 2011 11:31:02 +1100 Subject: [BLML] EBU L&EC meeting 3rd November 2010 [SEC=UNOFFICIAL] In-Reply-To: <000c01cbb2a8$0c9a4dc0$25cee940$@no> Message-ID: 5.11 New code of Laws Mr Endicott said that the first preliminary discussions about the 2017 code of laws was scheduled for about six months time. Any suggestions for law amendments should be sent directly to Grattan. +=+=+=+=+=+=+=+=+=+=+=+=+=+=+=+=+=+=+=+=+=+=+=+=+=+=+=+=+=+=+=+=+ 2018 LAW 68 - CLAIM OR CONCESSION OF TRICKS For a statement or action to constitute a claim or concession of tricks under these Laws, it must refer to tricks other than one currently in progress*. If it does refer to subsequent tricks: A. Claim Defined Any statement by a player to the effect that that player will win a specific number of tricks is a claim of those tricks. A player also claims when she suggests that play be curtailed, or when she shows her cards (unless she demonstrably did not intend to claim - for example, if declarer faces her cards after an opening lead out of turn Law 54, not this Law, will apply). B. Concession Defined 1. Any statement by a player to the effect that that player will lose a specific number of tricks is a concession of those tricks; a claim of some number of tricks is a concession of the remainder, if any. A player concedes all the remaining tricks when she abandons her hand. 2. Regardless of 1 preceding, if a defender attempts to concede one or more tricks and her partner immediately objects, no concession (nor claim) has occurred. Unauthorized information may exist, so the Director should be summoned immediately. Play continues. Any card that has been exposed by a defender in these circumstances is not a penalty card [Editor's note: Penalty cards might have been abolished elsewhere in this Lawbook.] but Law 16D applies to information arising from its exposure and the information may not be used by the partner of the defender who has exposed it. C. Clarification Required for Claim 1. A claim should be accompanied at once by a clear statement as to the order in which cards will be played, of the line of play or defence through which the claimer proposes to win the tricks claimed. (For example, in a 7NT contract's two card ending of Kx in dummy and Ax in hand, declarer stating "the rest are mine" is sufficiently clear, even though it is entirely legal for declarer to crash ace and king at trick twelve to create a defensive winner at trick thirteen.) 2. A defender may clearly state in her claim only her own winning tricks. A defender who states that her partner has a winning card has illegally created unauthorised information, since her partner may be unaware that one (or more) of her cards is a winner. D. Play Ceases After any claim or concession, play ceases (but see Law 70D3). If the claim or concession is agreed, Law 69 applies; if it is doubted by any player (dummy included), the Director must be summoned immediately and Law 70 applies. No action may be taken pending the Director's arrival. * If the statement or action pertains only to the winning or losing of an uncompleted trick currently in progress, play proceeds regularly; cards exposed or revealed by a defender do not become penalty cards, but Law 16, Unauthorized Information, may apply, and see Law 57A, Premature Play. 2018 LAW 69 - AGREED CLAIM OR CONCESSION A. When Agreement is Established Agreement is established when a contestant assents to an opponent's claim or concession, and raises no objection to it before her side makes a call on a subsequent board or before the round ends, whichever occurs first. The board is scored as though the tricks claimed or conceded had been won or lost in play. B. Director's Decision Agreement with a claim or concession (see A) may be withdrawn within the Correction Period established under Law 79C: 1. if a player agreed to the loss of a trick her side had, in fact, won; or 2. if a player has agreed to the loss of a trick that her side would likely have won had the play continued; or 3. if the claiming side requests, for a demonstrable bridge reason, that the non-claiming side be awarded one or more additional tricks. The board is rescored with such trick awarded to her side. 2018 LAW 70 - CONTESTED CLAIM OR CONCESSION A. General Objective In ruling on a contested claim or concession, the Director adjudicates the result of the board as equitably as possible to both sides, but any doubtful point as to a claim shall be resolved against the claimer (unless the non-claiming side has committed an unestablished revoke, in which case any doubtful point as to the claim shall be resolved in favour of the claimer). The Director proceeds as follows. B. Clarification Statement Repeated 1. The Director requires claimer to repeat the clarification statement she made at the time of his claim. 2. Next, the Director hears the opponents' objections to the claim (but the Director's considerations are not limited only to the opponents' objections). 3. The Director may require players to put their remaining cards face up on the table. C. There Is an Outstanding Trump When a trump remains in one of the opponents' hands, the Director shall award a trick or tricks to the opponents if: 1. claimer made no statement about that trump, and 2. it is at all likely* that claimer at the time of her claim was unaware that a trump remained in an opponent's hand, and 3. a trick could be lost to that trump by any normal** play. D. Director's Considerations 1. The Director shall not accept from claimer any successful line of play not embraced in the original clarification statement if there is an alternative normal** line of play that would be less successful. 2. The Director does not accept any part of a defender's claim that depends on her partner's selecting a particular play from among alternative normal** plays. 3. In accordance with Law 68D play should have ceased, but if any play has occurred after the claim this may provide evidence to be deemed part of the clarification of the claim. The Director may accept it as evidence of the players' probable plays subsequent to the claim and/or of the accuracy of the claim. E. Unstated Line of Play 1. The Director shall not accept from claimer any unstated line of play the success of which depends upon finding one opponent rather than the other with a particular card, unless an opponent failed to follow to the suit of that card before the claim was made, or would subsequently fail to follow to that suit on any normal** line of play, or unless failure to adopt that line of play would be weird or very inferior. 2. The Regulating Authority may specify an order (e.g. "from the top down") in which the Director shall deem a suit played if this was not clarified in the statement of claim (but always subject to any other requirement of this Law). * i.e. to rule in favour of the claimer under this clause, the Director must be satisfied beyond reasonable doubt that the claimer was aware of the outstanding trump. ** For the purposes of Laws 70 and 71, "normal" includes play that would be careless or somewhat inferior. "Normal" does not include play that would be weird or very inferior. For example, in a 7NT contract's two card ending of Kx in dummy and Ax in hand, it would be weird and very inferior for declarer to crash ace and king at trick twelve to create a defensive winner at trick thirteen. 2018 LAW 71 - CONCESSION CANCELLED A concession must stand, once made, except that within the Correction Period established under Law 79C the Director shall cancel a concession: 1. if a player conceded a trick her side had, in fact, won; or 2. if a player has conceded a trick that could not be lost by any normal** play of the remaining cards; or 3. if the non-conceding side requests, for a demonstrable bridge reason, that the conceding side be awarded one or more additional tricks. The board is rescored with such trick awarded to her side. ** For the purposes of Laws 70 and 71, "normal" includes play that would be careless or somewhat inferior. "Normal" does not include play that would be weird or very inferior. For example, in a 7NT contract's two card ending of Kx in dummy and Ax in hand, it would be weird and very inferior for declarer to crash ace and king at trick twelve to create a defensive winner at trick thirteen. Best wishes Richard Hills Recruitment Section Specialist Recruitment Team Level 5 Aqua, workstation W569, 6223 8453 DIAC Social Club movie tickets -------------------------------------------------------------------- Important Notice: If you have received this email by mistake, please advise the sender and delete the message and attachments immediately. This email, including attachments, may contain confidential, sensitive, legally privileged and/or copyright information. Any review, retransmission, dissemination or other use of this information by persons or entities other than the intended recipient is prohibited. DIAC respects your privacy and has obligations under the Privacy Act 1988. The official departmental privacy policy can be viewed on the department's website at www.immi.gov.au. See: http://www.immi.gov.au/functional/privacy.htm --------------------------------------------------------------------- From JffEstrsn at aol.com Thu Jan 13 01:37:03 2011 From: JffEstrsn at aol.com (Jeff Easterson) Date: Thu, 13 Jan 2011 01:37:03 +0100 Subject: [BLML] Ecclesiastes [SEC=UNOFFICIAL] In-Reply-To: <4D2D64D2.9030705@skynet.be> References: <4D2D64D2.9030705@skynet.be> Message-ID: <4D2E492F.2050000@aol.com> Wow! And Herman wants an apology when he feels he has been insulted. JE Am 12.01.2011 09:22, schrieb Herman De Wael: > richard.hills at immi.gov.au wrote: >>> It's a real possibility that he is right about that aspect. >> Richard Hills: >> >> No. Indeed, definitely not. As Nigel Guthrie has pointed out, the >> changes proposed by Herman De Wael are so fundamental that if ever >> enacted (which fortunately will never happen) bridge could no >> longer be called bridge. >> > Richard Hills not only knows what is best for bridge - he also knows > when a change would make it no longer bridge. > Richard Hills has not understood a word of what has been said in the > past 15 years. > Richard Hills has not seen that in the past 15 years, only one true > example of a dWS ruling has been seen. That ruling has gone unnoticed by > the opponents, who certainly thought they were playing bridge. > From JffEstrsn at aol.com Thu Jan 13 01:37:10 2011 From: JffEstrsn at aol.com (Jeff Easterson) Date: Thu, 13 Jan 2011 01:37:10 +0100 Subject: [BLML] Ecclesiastes [SEC=UNOFFICIAL] In-Reply-To: <4D2D651F.4070000@skynet.be> References: <4D2D651F.4070000@skynet.be> Message-ID: <4D2E4936.4060605@aol.com> See my previous (Wow!) comment. JE Am 12.01.2011 09:23, schrieb Herman De Wael: > richard.hills at immi.gov.au wrote: >> "Carefully exploring a minority opinion is what BLML is all >> about" >> >> No. >> > Not only does Richard know what is best for bridge, he also knows what > is best for blml. Why do we bother listening to such a tyrant? > > From nigelguthrie at yahoo.co.uk Thu Jan 13 01:40:46 2011 From: nigelguthrie at yahoo.co.uk (Nigel Guthrie) Date: Thu, 13 Jan 2011 00:40:46 +0000 (GMT) Subject: [BLML] Ecclesiastes [SEC=UNOFFICIAL] In-Reply-To: <1955796701.25492.1294862117019.JavaMail.ngmail@webmail18.arcor-online.net> References: <1955796701.25492.1294862117019.JavaMail.ngmail@webmail18.arcor-online.net> Message-ID: <95930.87169.qm@web28501.mail.ukl.yahoo.com> [Thomas Dehn] For example, for many decades the official position of the Roman Catholic Church was that condoms cause HIV. [Nigel] Some of official RC policies on sex have been and are questionable but that one is news to me. (I'm RC). Please supply a reference, Thomas. [Thomas] Came in various ways, from the merely incorrect up to the completely absurd. Cardinal Alfonso Lopez Trujillo, as president of the Pontifical Council for the Family (2003): "The Aids virus is roughly 450 times smaller than the spermatozoon. The spermatozoon can easily pass through the 'net' that is formed by the condom." and "Relying on condoms is like betting on your own death" Monsignor Andre Fort, Bishop of Orleans (2009): "You know very well, and all the scientists know it: the AIDS virus is infinitely smaller than a sperm. This is proof that the condom is not a 100 per cent guarantee against AIDS" (Fort retracted) Pope Benedict XVI, 2009, on his first trip to Africa: "You can't resolve it[AIDS] with the distribution of condoms. On the contrary, it increases the problem." Francisco Chimoio, Archbishop of Maputo, Mozambique (2007) "Condoms are not sure because I know that there are two countries in Europe, .... they are making condoms with the virus on purpose. They want to finish with the African people. This is the programme. They want to colonise until up to now. If we are not careful we will finish in one century's time." (He did not name those two countries) Raphael Ndingi Mwana A'Nzeki, Archbishop of Nairobi: "HIV/AIDS is going so fast because of the availability of condoms. ? they should not be made at all." There also exist some similar John Paul II quotes, only by now those are hard to find. [Nige2] Thank you Thomas. Not exactly "condoms cause HIV" but frightening all the same. From nigelguthrie at yahoo.co.uk Thu Jan 13 02:01:59 2011 From: nigelguthrie at yahoo.co.uk (Nigel Guthrie) Date: Thu, 13 Jan 2011 01:01:59 +0000 (GMT) Subject: [BLML] Alain's case revisited In-Reply-To: <000e01cbb2a9$ba3fc670$2ebf5350$@no> References: <4D2B0857.7050804@skynet.be> <4D2AC556.9070509@skynet.be> <4D2AC9E7.6060208@aol.com> <4D2AE13B.10605@skynet.be> <334994.97415.qm@web28503.mail.ukl.yahoo.com> <1391468851.32037.1294667187404.JavaMail.ngmail@webmail08.arcor-online.net> <4D2B1D0C.7090801@skynet.be> <4D2B2CC8.2060308@ulb.ac.be> <4D2E041A.8070508@nhcc.net> <000e01cbb2a9$ba3fc670$2ebf5350$@no> Message-ID: <707261.93703.qm@web28501.mail.ukl.yahoo.com> [Sven Pran] The communication between partners in bridge is limited to the "bridge language" comprising a total of 38 different (possible) calls [Steve Wilner] Actually, as of 2007 "expected" alerts are AI. They seldom contain useful information, but sometimes they do. [Sven Pran] While alerts are AI they are NOT part of the "bridge language" see Law 73A1 [Nigel] In RAs that permit such variations, each side's choice of option after a call-out-of-turn or insufficient bid, can trigger different sets of understandings. This provides an additional channel of communication. Even in RAs that ostensibly forbid such ploys, some directors believe there are variations contingent on such choices that are a matter of Bridge-logic rather than agreement. ________ From grabiner at alumni.princeton.edu Thu Jan 13 02:53:16 2011 From: grabiner at alumni.princeton.edu (David Grabiner) Date: Wed, 12 Jan 2011 20:53:16 -0500 Subject: [BLML] EBU L&EC meeting 3rd November 2010 [SEC=UNOFFICIAL] In-Reply-To: References: Message-ID: <7CD959DDDFFB4B31972F6272669E7CB0@erdos> Richard Hills wrote: > 2018 LAW 70 - CONTESTED CLAIM OR CONCESSION > > A. General Objective > > In ruling on a contested claim or concession, the Director > adjudicates the result of the board as equitably as possible to > both sides, but any doubtful point as to a claim shall be > resolved against the claimer (unless the non-claiming side has > committed an unestablished revoke, in which case any doubtful > point as to the claim shall be resolved in favour of the > claimer). It should not matter whether the revoke is established or not; rather, it should matter whether the revoke affects the claim. For example, consider the following ending in a spade contract: xx - AJx xx xxxx - KTx - East has shown out of spades, leaving West with the good SK, and West has shown out of clubs; the DQ is still out South now says, "I have all but one of the tricks. I lead a spade, West wins the SK, and he must either lead a diamond for a free finesse, or lead a heart and let me ruff in dummy and pitch a diamond from hand." However, West has a club, and can lead it to avoid the endplay, forcing South to guess the DQ. Whether the revoke was unestablished (West just discarded on a club trick) or established (West discarded on a previous club trick and the last trick was a heart), South could have guessed the queen right had he not claimed. Since he had a perfect claim assuming that West followed the laws of bridge, he should not lose expectation by claiming. From richard.hills at immi.gov.au Thu Jan 13 04:20:12 2011 From: richard.hills at immi.gov.au (richard.hills at immi.gov.au) Date: Thu, 13 Jan 2011 14:20:12 +1100 Subject: [BLML] EBU L&EC meeting 3rd November 2010 [SEC=UNOFFICIAL] In-Reply-To: Message-ID: 5.11 New code of Laws Mr Endicott said that the first preliminary discussions about the 2017 code of laws was scheduled for about six months time. Any suggestions for law amendments should be sent directly to Grattan. +=+=+=+=+=+=+=+=+=+=+=+=+=+=+=+=+=+=+=+=+=+=+=+=+=+=+=+=+=+=+=+=+ 2018 LAW 90 - PROCEDURAL PENALTIES A. Director's Authority after any Infraction The Director, in addition to implementing a rectification prescribed by these Laws, may also judge it appropriate to assess a procedural penalty (usually subtracting matchpoints or imps or victory points, etc) upon the offending side for any infraction it might have made. The appropriate procedural penalty varies in magnitude according to the severity of the infraction and the expertise of the offending side. (For example, a severe infraction by a novice player deserves the mildest possible procedural penalty of an educative lecture from the Director.) Regulating Authorities may provide guidance to Directors on the application of procedural penalties. B. Infractions Particularly Subject to Procedural Penalty These infractions are particularly subject to procedural penalty: 1. arrival of a contestant after the specified starting time. 2. unduly slow play by a contestant. 3. discussion of the bidding, play or result of a board, which may be overheard at another table. 4. unauthorized comparison of scores with another contestant. 5. touching or handling of cards belonging to another player (see Law 7). 6. placing one or more cards in an incorrect pocket of the board. 7. errors in procedure (such as failure to count cards in one's hand, playing the wrong board, etc.) that require an adjusted score for any contestant. 8. failure to comply promptly with tournament regulations or with instructions of the Director. 2018 LAW 91 - DISCIPLINARY PENALTIES OR SUSPENSIONS A. Director's Powers In performing her duty to maintain order and discipline, the Director is empowered to assess disciplinary penalties in points or to suspend a contestant for the current session* or any part thereof. The Director's decision under this clause is final and may not be overruled by an appeals committee (see Law 93B3). B. Right to Disqualify The Director is empowered to disqualify a contestant for cause, subject to approval** by the Tournament Organizer. * See the Definition of "session". ** The Tournament Organizer may empower the Director in advance. Best wishes Richard Hills Recruitment Section Specialist Recruitment Team Level 5 Aqua, workstation W569, 6223 8453 DIAC Social Club movie tickets -------------------------------------------------------------------- Important Notice: If you have received this email by mistake, please advise the sender and delete the message and attachments immediately. This email, including attachments, may contain confidential, sensitive, legally privileged and/or copyright information. Any review, retransmission, dissemination or other use of this information by persons or entities other than the intended recipient is prohibited. DIAC respects your privacy and has obligations under the Privacy Act 1988. The official departmental privacy policy can be viewed on the department's website at www.immi.gov.au. See: http://www.immi.gov.au/functional/privacy.htm --------------------------------------------------------------------- From richard.hills at immi.gov.au Thu Jan 13 04:41:40 2011 From: richard.hills at immi.gov.au (richard.hills at immi.gov.au) Date: Thu, 13 Jan 2011 14:41:40 +1100 Subject: [BLML] EBU L&EC meeting 3rd November 2010 [SEC=UNOFFICIAL] In-Reply-To: Message-ID: Richard Willey: >From my own perspective, I think that the Laws for Online Bridge >are the ones the deserve/require the most serious attention. WBF Laws Committee, 12th October 2010, item 10: "It was reported that a draft revision of the laws of online bridge had been prepared. It was agreed to consult with interested parties on this and subsequently communicate again with the committee on the subject." Best wishes Richard Hills Recruitment Section Specialist Recruitment Team Level 5 Aqua, workstation W569, 6223 8453 DIAC Social Club movie tickets -------------------------------------------------------------------- Important Notice: If you have received this email by mistake, please advise the sender and delete the message and attachments immediately. This email, including attachments, may contain confidential, sensitive, legally privileged and/or copyright information. Any review, retransmission, dissemination or other use of this information by persons or entities other than the intended recipient is prohibited. DIAC respects your privacy and has obligations under the Privacy Act 1988. The official departmental privacy policy can be viewed on the department's website at www.immi.gov.au. See: http://www.immi.gov.au/functional/privacy.htm --------------------------------------------------------------------- From richard.hills at immi.gov.au Thu Jan 13 05:02:02 2011 From: richard.hills at immi.gov.au (richard.hills at immi.gov.au) Date: Thu, 13 Jan 2011 15:02:02 +1100 Subject: [BLML] EBU L&EC meeting 3rd November 2010 [SEC=UNOFFICIAL] In-Reply-To: <001201cbb25b$6e69d9f0$4b3d8dd0$@nl> Message-ID: Hans van Staveren: >12C1b is just restated using a word unlikely to make it into >the law. Richard Hills: The so-called Wicked Bible had just one word missing from the Seventh Commandment, the word "not". And anyway I do not insist upon the word "ridiculous". I merely suggested that word to be food for thought for the new 2018 Drafting Committee, given that the current word "serious" was so inadequate for its intended purpose that a special WBF Laws Committee interpretation was consequently required. Best wishes Richard Hills Recruitment Section Specialist Recruitment Team Level 5 Aqua, workstation W569, 6223 8453 DIAC Social Club movie tickets -------------------------------------------------------------------- Important Notice: If you have received this email by mistake, please advise the sender and delete the message and attachments immediately. This email, including attachments, may contain confidential, sensitive, legally privileged and/or copyright information. Any review, retransmission, dissemination or other use of this information by persons or entities other than the intended recipient is prohibited. DIAC respects your privacy and has obligations under the Privacy Act 1988. The official departmental privacy policy can be viewed on the department's website at www.immi.gov.au. See: http://www.immi.gov.au/functional/privacy.htm --------------------------------------------------------------------- From richard.hills at immi.gov.au Thu Jan 13 05:42:10 2011 From: richard.hills at immi.gov.au (richard.hills at immi.gov.au) Date: Thu, 13 Jan 2011 15:42:10 +1100 Subject: [BLML] EBU L&EC meeting 3rd November 2010 [SEC=UNOFFICIAL] In-Reply-To: Message-ID: Richard Hills suggestion for 2018 Law 20: >>..... >>(for example, if the pre-existing mutual partnership >>understanding of the opponents in a particular auction is >>that 4NT promises 5/5 in the minors, the player is not >>entitled at that time to know the opponents' responses to >>their 4NT Keycard Blackwood convention). >>..... Eric Landau: >Why not? This seems unnecessary and problematic. Richard Hills: Why not? I do not know the reasoning behind the WBF Laws Committee support of this idea. But my answer to "Why not?" is to limit the legal but harassing questions of a Secretary Bird to the minimum number necessary (even the highly ethical gafiated blmler Ed Reppert once unintentionally over-questioned a clueless novice opponent). Eric Landau: >It would mean either abandoning the established right of a >player to peruse his opponents' convention card freely at >his turn to bid, or introducing for the first time the >notion that one's right to disclosure depends on what the >opponents happen to have written on their CC. Richard Hills: Good point by Eric. So I will modify my suggestion to -> (for example, if the pre-existing mutual partnership understanding of the opponents in a particular auction is that 4NT promises 5/5 in the minors, the player is not entitled at that time to ask about the opponents' responses to their 4NT Keycard Blackwood convention, but the player retains her right to peruse the opponents' System Card). Eric Landau: >It would also explicitly repudiate the "Kaplan paradigm" >of full disclosure, leaving us without an overarching set >of principles to guide us in those inevitable situations >not explicitly and unambiguously covered by the rules. WBF Laws Committee minutes, 8th October 2010, item 7: The committee read a comment by a player that something he termed the "Kaplan doctrine" had been overturned by the minute regarding Law 20F1 recorded on 10th October 2008 and reaffirmed in item 13 of the minutes of 8th September 2009. Remarking that the limits of enquiry are defined in Law 20F1 the committee drew attention to the fact that they do not extend to exploration of hypothetical situations not related to the partnership understandings applicable in the current auction. Best wishes Richard Hills Recruitment Section Specialist Recruitment Team Level 5 Aqua, workstation W569, 6223 8453 DIAC Social Club movie tickets -------------------------------------------------------------------- Important Notice: If you have received this email by mistake, please advise the sender and delete the message and attachments immediately. This email, including attachments, may contain confidential, sensitive, legally privileged and/or copyright information. Any review, retransmission, dissemination or other use of this information by persons or entities other than the intended recipient is prohibited. DIAC respects your privacy and has obligations under the Privacy Act 1988. The official departmental privacy policy can be viewed on the department's website at www.immi.gov.au. See: http://www.immi.gov.au/functional/privacy.htm --------------------------------------------------------------------- From richard.hills at immi.gov.au Thu Jan 13 07:12:18 2011 From: richard.hills at immi.gov.au (richard.hills at immi.gov.au) Date: Thu, 13 Jan 2011 17:12:18 +1100 Subject: [BLML] Ecclesiastes [SEC=UNOFFICIAL] In-Reply-To: <139528.76055.qm@web28503.mail.ukl.yahoo.com> Message-ID: Ecclesiastes 1:9 and there is no new thing under the sun. Nigel Guthrie: >That question has been answered many times. Neither is >intrinsically right. A game is an artificial world where >ordinary rules need not apply. In the artificial context >of a game, the rules of the game tell you what is right >and wrong. The rule-makers of a game are free to repeal >the Geneva Convention or the law of Gravity, if they feel >it makes the game more enjoyable. > >Have you ever played GTA? Richard Hills: No, I have not played the graphically violent video game Grand Theft Auto. Nor (despite being a movie buff) have I ever deigned to watch the even more graphically violent movies in the Saw series. Thus "more enjoyable" can only be defined in the context of the current person or group. Many years ago, a survey of Aussie Summer Festival players discovered that the majority would find smoke-free bridge more enjoyable. So the ABF rule-makers repealed their previous Law of Smoker's Rights, and introduced a new rule barring smoking in the playing areas of ABF national championships. The new 2007 Law 65B3 was introduced to make life more enjoyable for the majority of bridge players (as opposed to the tiny minority of world-class experts who almost never make a mechanical error, and who therefore hated the fact that Law 65B3 reduced the number of tops they scored against bunnies). Ergo, when Herman De Wael describes his opponents as "the majority school", he undermines the moral basis for his ideas to ever be adopted by the majority of rule-makers (who, despite Nigel Guthrie's repeated assertions to the contrary, have the interests of the majority of their players ever-present in their hearts and minds as they create and repeal rules). And one of my suggestions, which perhaps might be enjoyable for the majority, is -> 2018 LAW 74 - CONDUCT AND ETIQUETTE A. Proper Attitude 1. A player must maintain a courteous attitude at all times. 2. A player must carefully avoid any remark or action that might cause annoyance or embarrassment to another player (including partner) or might interfere with the enjoyment of the game. This clause is the most important of all clauses in the entire 2018 Laws of Duplicate Bridge. 3. Every player should follow uniform and correct procedure in calling and playing. B. Etiquette As a matter of courtesy a player should refrain from: 1. paying insufficient attention to the game (but unintentional misbids are fully legal unless Law 40C1 applies). 2. making gratuitous comments during the auction and play. 3. detaching a card before it is her turn to play. 4. prolonging play unnecessarily (for example, in playing on although she knows that all the tricks are surely hers) for the purpose of disconcerting an opponent. 5. summoning and addressing the Director in a manner discourteous (discourtesy includes a frivolous summoning of the Director when the summoner could well have known that such a summoning was unnecessary) to her or to other contestants. C. Violations of Procedure The following are examples of violations of procedure: 1. using different designations for the same call. 2. indicating approval or disapproval of a call or play. 3. indicating the expectation or intention of winning or losing a trick that has not been completed. 4. commenting or acting during the auction or play so as to call attention to a significant occurrence, or to the number of tricks still required for success. 5. looking intently at any other player during the auction and play, or at another player's hand as for the purpose of seeing her cards or of observing the place from which she draws a card (but it is appropriate to avert one's gaze or tell an opponent to better conceal her cards). 6. showing an obvious lack of further interest in a deal (as by folding one's cards). 7. varying the normal tempo of bidding or play for the purpose of disconcerting an opponent. 8. leaving the table needlessly before the round is called. Best wishes Richard Hills Recruitment Section Specialist Recruitment Team Level 5 Aqua, workstation W569, 6223 8453 DIAC Social Club movie tickets -------------------------------------------------------------------- Important Notice: If you have received this email by mistake, please advise the sender and delete the message and attachments immediately. This email, including attachments, may contain confidential, sensitive, legally privileged and/or copyright information. Any review, retransmission, dissemination or other use of this information by persons or entities other than the intended recipient is prohibited. DIAC respects your privacy and has obligations under the Privacy Act 1988. The official departmental privacy policy can be viewed on the department's website at www.immi.gov.au. See: http://www.immi.gov.au/functional/privacy.htm --------------------------------------------------------------------- From blml at arcor.de Thu Jan 13 07:25:08 2011 From: blml at arcor.de (Thomas Dehn) Date: Thu, 13 Jan 2011 07:25:08 +0100 (CET) Subject: [BLML] EBU L&EC meeting 3rd November 2010 - L20 Message-ID: <1242552473.588.1294899908423.JavaMail.ngmail@webmail17.arcor-online.net> richard.hills at immi.gov.au wrote: > 5.11 New code of Laws > Mr Endicott said that the first preliminary discussions about the > 2017 code of laws was scheduled for about six months time. Any > suggestions for law amendments should be sent directly to Grattan. First, a point on use of language. Below, "he" has been replaced with "she", "his" with "her" and so on. I consider this to be a bad idea. If you can create gender neutral language, great. But that looks silly to me. > +=+=+=+=+=+=+=+=+=+=+=+=+=+=+=+=+=+=+=+=+=+=+=+=+=+=+=+=+=+=+=+=+ > > 2018 LAW 20 - REVIEW AND EXPLANATION OF CALLS > > A. Call Not Clearly Recognised > > A player may require clarification forthwith if she is in doubt > what call has been made. > > B. Review of Auction during Auction Period > > During the auction period, a player is entitled to have all > previous calls restated* when it is her turn to call, unless she > is required by law to pass. Alerts should be included when > responding to the request. A player may not ask for a partial > review of previous calls and may not halt the review before it is > completed. > > C. Review after Final Pass > > 1. After the final pass either defender has the right to ask if > it is her opening lead (see Laws 47E and 41). > > 2. Declarer** or either defender may, at her first turn to play, > require all previous calls to be restated*. (See Laws 41B and > 41C). As in B the player may not ask for only a partial > restatement or halt the review. > > D. Who May Review the Auction > > A request to have calls restated* shall be responded to only by > an opponent (unless the Director at the table rules otherwise). > > E. Correction of Error in Review > > All players, including dummy or a player required by law to > pass, are responsible for prompt correction of errors in > restatement* (see Law 12C1 when an uncorrected review causes > damage). > > F. Explanation of Calls > > 1. (a) During the auction before the final pass, any player may > request, but only at her own turn to call, honest*** explanations > of the opponents' prior auction. She is entitled to know about > calls actually made, about relevant alternative calls available > that were not made, and about relevant inferences from the choice > of action where these are matters of partnership understanding. > She is not entitled to know about calls that might be made in an > entirely different auction (for example, if the pre-existing > mutual partnership understanding of the opponents in a particular > auction is that 4NT promises 5/5 in the minors, the player is not > entitled at that time to know the opponents' responses to their > 4NT Keycard Blackwood convention). I disagree with this. Assume opponents suspect there was a misunderstanding, and the player who responded to 4NT actually answered RKCB. You are putting opponents at a disadvantage here - the player who bid 4NT can work out that his partner had N key cards, whereas opponents can not. > (b) Except on the instruction of the Director (who may, in > accordance with guidance from her Regulating Authority, direct a > player to temporarily leave the table) replies should be given by > the partner of the player who made the call in question. The > partner of a player who asks a question may not ask a > supplementary question until her turn to call or play. Law 16 may > apply and the Regulating Authority may establish regulations for > written explanations. > > 2. After the final pass and throughout the play period, either > defender at her own turn to play may request honest*** > explanations of the opposing auction. At her turn to play from > her hand or from dummy declarer may request honest*** > explanations of a defender's call or card play understandings. > Honest*** explanations should be given on a like basis to 1 and > by the partner of the player whose action is explained. > > 3. Under 1 and 2 above a player may ask concerning a single call > but Law 16B1 may apply. > > 4. If a player subsequently realises that her own explanation was > erroneous or incomplete she must call the Director immediately. > The Director applies Law 21B or Law 40B4. mmmh, did you change the spelling of "realizes" here? If yes, is TFLB supposed to have British spelling, or American spelling? The 2007 laws seem to have American spelling. I believe my downloaded copy is the official WBF version. Beware of those spell checker devils! > 5. (a) A player whose partner has given a mistaken explanation > may not correct the error during the auction, nor may she > indicate by any mannerism (as in a shrug, glare, frown etc.) > that a mistake has been made. "Mistaken explanation" here > includes failure to alert or announce as regulations require or > an alert (or an announcement) that regulations do not require. > > (b) Nevertheless, the player must still give honest*** > explanations of her partner's subsequent calls whether or not > those honest*** explanations indirectly indicate to partner her > partner's error. Such an indirect indication is unauthorised > information, so Law 73C and/or Law 75A may apply to partner. "unauthorized" became "unauthorised" here. See above. > (c) The player must call the Director and inform her > opponents that, in her opinion, her partner's explanation was > erroneous (see Law 75) but only at her first legal opportunity, > which is > (i) for a defender, at the end of the play. > (ii) for declarer or dummy, after the final pass of the > auction (but before the opening lead is faced, and > preferably before the opening lead is selected). > > 6. If the Director judges that a player has based an action on > misinformation given to him by an opponent see, as appropriate, > Law 21 or Law 47E. > > G. Incorrect Procedure > > 1. (a) Asking a question solely for partner's benefit is an > infraction. The Director applies Law 90 or Law 91. > > (b) Asking a question solely for the purpose of making > either side create unauthorised information is an infraction. > The Director applies Law 90 or Law 91. "unauthorised". See above. 1(b) here is new, and I have doubts whether this is an improvement. > 2. Except as the Regulating Authority allows a player may not > consult her own System Card and notes during the auction and play > periods, but see Law 40B2(b). > > * When the calls are not spoken responders must ensure that it is > clear to an enquiring opponent what calls have been made. "enquiring" vs. "inquiring". See above. > ** Declarer's first turn to play is from dummy unless accepting an > opening lead out of turn. > > *** Unintentional errors in explanation are honest. Dishonest > intentional misexplanations could well receive Law 72B1 and Law 91 > Disciplinary Penalties. The *** footnote looks confusing to me. Thomas From richard.hills at immi.gov.au Thu Jan 13 07:25:33 2011 From: richard.hills at immi.gov.au (richard.hills at immi.gov.au) Date: Thu, 13 Jan 2011 17:25:33 +1100 Subject: [BLML] Ecclesiastes [SEC=UNOFFICIAL] In-Reply-To: <981233.77121.qm@web28511.mail.ukl.yahoo.com> Message-ID: Richard Hills: >>As Nigel Guthrie has pointed out, the changes proposed >>by Herman De Wael are so fundamental that if ever >>enacted (which fortunately will never happen) bridge >>could no longer be called bridge. Nigel Guthrie: >My stated view is that if a group of ordinary players >unilaterally decide to play a new game with their own >rules, then they shouldn't call that game "Bridge" Richard Hills: My apologies to Nigel Guthrie for my unintentional misunderstanding (not intentional misstatement) of his view. My request for an apology from Jerry Fusselman to me, since otherwise Jerry's animadversion to "ad hominem" attacks would appear to be hypocritical. Ecclesiastes 3:1 A time to love, and a time to hate; a time of war, and a time of peace. Best wishes (no longer Stern correction) Richard Hills Recruitment Section Specialist Recruitment Team Level 5 Aqua, workstation W569, 6223 8453 DIAC Social Club movie tickets -------------------------------------------------------------------- Important Notice: If you have received this email by mistake, please advise the sender and delete the message and attachments immediately. This email, including attachments, may contain confidential, sensitive, legally privileged and/or copyright information. Any review, retransmission, dissemination or other use of this information by persons or entities other than the intended recipient is prohibited. DIAC respects your privacy and has obligations under the Privacy Act 1988. The official departmental privacy policy can be viewed on the department's website at www.immi.gov.au. See: http://www.immi.gov.au/functional/privacy.htm --------------------------------------------------------------------- From Hermandw at skynet.be Thu Jan 13 10:34:07 2011 From: Hermandw at skynet.be (Herman De Wael) Date: Thu, 13 Jan 2011 10:34:07 +0100 Subject: [BLML] EBU L&EC meeting 3rd November 2010 [SEC=UNOFFICIAL] In-Reply-To: References: Message-ID: <4D2EC70F.5000807@skynet.be> richard.hills at immi.gov.au wrote: > Richard Hills suggestion for 2018 Law 20: > >>> ..... >>> (for example, if the pre-existing mutual partnership >>> understanding of the opponents in a particular auction is >>> that 4NT promises 5/5 in the minors, the player is not >>> entitled at that time to know the opponents' responses to >>> their 4NT Keycard Blackwood convention). >>> ..... > > Eric Landau: > >> Why not? This seems unnecessary and problematic. > > Richard Hills: > > Why not? > > I do not know the reasoning behind the WBF Laws Committee > support of this idea. But my answer to "Why not?" is to > limit the legal but harassing questions of a Secretary > Bird to the minimum number necessary (even the highly > ethical gafiated blmler Ed Reppert once unintentionally > over-questioned a clueless novice opponent). > The problem with this reasoning is that you use the wrong stick to beat this problem. Excessive questioning can be dealt with in other articles (and is), and this rule will not stop excessive questioning. All the excessive questioner needs to do is limit his questions to the calls actually made "could he have, say, 5 spades to the 10? to the 9? ..." This regulation made it into the WBF books for the sole purpose of taking away from me one small argument in favour of the dWS. It is a silly step, and I am glad that many people find it abhorrant. If a question has a valid purpose (which the question "how many aces?" certainly has when asked of the partner of a player who has just told you he's going to reply to Blackwood), why should it be banned? > Eric Landau: > >> It would mean either abandoning the established right of a >> player to peruse his opponents' convention card freely at >> his turn to bid, or introducing for the first time the >> notion that one's right to disclosure depends on what the >> opponents happen to have written on their CC. > > Richard Hills: > > Good point by Eric. So I will modify my suggestion to -> > > (for example, if the pre-existing mutual partnership > understanding of the opponents in a particular auction is > that 4NT promises 5/5 in the minors, the player is not > entitled at that time to ask about the opponents' > responses to their 4NT Keycard Blackwood convention, but > the player retains her right to peruse the opponents' > System Card). > > Eric Landau: > >> It would also explicitly repudiate the "Kaplan paradigm" >> of full disclosure, leaving us without an overarching set >> of principles to guide us in those inevitable situations >> not explicitly and unambiguously covered by the rules. > > WBF Laws Committee minutes, 8th October 2010, item 7: > > The committee read a comment by a player that something he > termed the "Kaplan doctrine" had been overturned by the > minute regarding Law 20F1 recorded on 10th October 2008 and > reaffirmed in item 13 of the minutes of 8th September 2009. > Remarking that the limits of enquiry are defined in Law > 20F1 the committee drew attention to the fact that they do > not extend to exploration of hypothetical situations not > related to the partnership understandings applicable in the > current auction. > Hands up those who believes the Kaplan paradigm ought to be reinstated! -- Herman De Wael Wilrijk Antwerpen Belgium From svenpran at online.no Thu Jan 13 10:57:51 2011 From: svenpran at online.no (Sven Pran) Date: Thu, 13 Jan 2011 10:57:51 +0100 Subject: [BLML] Alain's case revisited In-Reply-To: <707261.93703.qm@web28501.mail.ukl.yahoo.com> References: <4D2B0857.7050804@skynet.be> <4D2AC556.9070509@skynet.be> <4D2AC9E7.6060208@aol.com> <4D2AE13B.10605@skynet.be> <334994.97415.qm@web28503.mail.ukl.yahoo.com> <1391468851.32037.1294667187404.JavaMail.ngmail@webmail08.arcor-online.net> <4D2B1D0C.7090801@skynet.be> <4D2B2CC8.2060308@ulb.ac.be> <4D2E041A.8070508@nhcc.net> <000e01cbb2a9$ba3fc670$2ebf5350$@no> <707261.93703.qm@web28501.mail.ukl.yahoo.com> Message-ID: <001001cbb308$57dd6af0$079840d0$@no> On Behalf Of Nigel Guthrie > [Sven Pran] > The communication between partners in bridge is limited to the "bridge language" > comprising a total of 38 different (possible) calls > > [Steve Wilner] > Actually, as of 2007 "expected" alerts are AI. They seldom contain useful > information, but sometimes they do. > > [Sven Pran] > While alerts are AI they are NOT part of the "bridge language" see Law 73A1 > > [Nigel] > In RAs that permit such variations, each side's choice of option after a call-out-of- > turn or insufficient bid, can trigger different sets of understandings. This provides > an additional channel of communication. Even in RAs that ostensibly forbid such > ploys, some directors believe there are variations contingent on such choices that > are a matter of Bridge-logic rather than agreement. This is not communication between partners any more than the fact that a different set of partnership understandings may apply depending on opponents' disclosed agreements. Varying partnership understandings following an irregularity by opponents can be necessary and/or a logical consequence of the situation based on general bridge knowledge. An example from my own directing experience: 4NT was followed by 4D (IB) from LHO. Partner accepted the IB and responded to 4NT with a logical alternative call among also the five more calls now becoming available. (The partnership had of course no prearranged agreement for such a situation, but they were strong players with sufficient understanding of auction theory to in flight deploy the five unexpected additionally available calls.) From JffEstrsn at aol.com Thu Jan 13 11:04:06 2011 From: JffEstrsn at aol.com (Jeff Easterson) Date: Thu, 13 Jan 2011 11:04:06 +0100 Subject: [BLML] EBU L&EC meeting 3rd November 2010 [SEC=UNOFFICIAL] In-Reply-To: <4D2EC70F.5000807@skynet.be> References: <4D2EC70F.5000807@skynet.be> Message-ID: <4D2ECE16.3070104@aol.com> Am 13.01.2011 10:34, schrieb Herman De Wael: > richard.hills at immi.gov.au wrote: >> Richard Hills suggestion for 2018 Law 20: >> >>>> ..... >>>> (for example, if the pre-existing mutual partnership >>>> understanding of the opponents in a particular auction is >>>> that 4NT promises 5/5 in the minors, the player is not >>>> entitled at that time to know the opponents' responses to >>>> their 4NT Keycard Blackwood convention). >>>> ..... >> Eric Landau: >> >>> Why not? This seems unnecessary and problematic. >> Richard Hills: >> >> Why not? >> >> I do not know the reasoning behind the WBF Laws Committee >> support of this idea. But my answer to "Why not?" is to >> limit the legal but harassing questions of a Secretary >> Bird to the minimum number necessary (even the highly >> ethical gafiated blmler Ed Reppert once unintentionally >> over-questioned a clueless novice opponent). >> > The problem with this reasoning is that you use the wrong stick to beat > this problem. Excessive questioning can be dealt with in other articles > (and is), and this rule will not stop excessive questioning. All the > excessive questioner needs to do is limit his questions to the calls > actually made "could he have, say, 5 spades to the 10? to the 9? ..." > This regulation made it into the WBF books for the sole purpose of > taking away from me one small argument in favour of the dWS. Do you really believe that you are so important that the WBF enacts regulations ("for the sole purpose of....")? > It is a > silly step, and I am glad that many people find it abhorrant. If a > question has a valid purpose (which the question "how many aces?" > certainly has when asked of the partner of a player who has just told > you he's going to reply to Blackwood), why should it be banned? > >> Eric Landau: >> >>> It would mean either abandoning the established right of a >>> player to peruse his opponents' convention card freely at >>> his turn to bid, or introducing for the first time the >>> notion that one's right to disclosure depends on what the >>> opponents happen to have written on their CC. >> Richard Hills: >> >> Good point by Eric. So I will modify my suggestion to -> >> >> (for example, if the pre-existing mutual partnership >> understanding of the opponents in a particular auction is >> that 4NT promises 5/5 in the minors, the player is not >> entitled at that time to ask about the opponents' >> responses to their 4NT Keycard Blackwood convention, but >> the player retains her right to peruse the opponents' >> System Card). >> >> Eric Landau: >> >>> It would also explicitly repudiate the "Kaplan paradigm" >>> of full disclosure, leaving us without an overarching set >>> of principles to guide us in those inevitable situations >>> not explicitly and unambiguously covered by the rules. >> WBF Laws Committee minutes, 8th October 2010, item 7: >> >> The committee read a comment by a player that something he >> termed the "Kaplan doctrine" had been overturned by the >> minute regarding Law 20F1 recorded on 10th October 2008 and >> reaffirmed in item 13 of the minutes of 8th September 2009. >> Remarking that the limits of enquiry are defined in Law >> 20F1 the committee drew attention to the fact that they do >> not extend to exploration of hypothetical situations not >> related to the partnership understandings applicable in the >> current auction. >> > Hands up those who believes the Kaplan paradigm ought to be reinstated! > From blml at arcor.de Thu Jan 13 11:07:39 2011 From: blml at arcor.de (Thomas Dehn) Date: Thu, 13 Jan 2011 11:07:39 +0100 (CET) Subject: [BLML] EBU L&EC meeting 3rd November 2010 [SEC=UNOFFICIAL] In-Reply-To: <4D2EC70F.5000807@skynet.be> References: <4D2EC70F.5000807@skynet.be> Message-ID: <537413437.6690.1294913259948.JavaMail.ngmail@webmail17.arcor-online.net> Herman De Wael wrote: > richard.hills at immi.gov.au wrote: > > Richard Hills suggestion for 2018 Law 20: > > Eric Landau: > > > >> It would also explicitly repudiate the "Kaplan paradigm" > >> of full disclosure, leaving us without an overarching set > >> of principles to guide us in those inevitable situations > >> not explicitly and unambiguously covered by the rules. > > > > WBF Laws Committee minutes, 8th October 2010, item 7: > > > > The committee read a comment by a player that something he > > termed the "Kaplan doctrine" had been overturned by the > > minute regarding Law 20F1 recorded on 10th October 2008 and > > reaffirmed in item 13 of the minutes of 8th September 2009. > > Remarking that the limits of enquiry are defined in Law > > 20F1 the committee drew attention to the fact that they do > > not extend to exploration of hypothetical situations not > > related to the partnership understandings applicable in the > > current auction. > > Hands up those who believes the Kaplan paradigm ought to be reinstated! I believe that "full disclosure" is necessary. Thomas From Hermandw at skynet.be Thu Jan 13 12:26:34 2011 From: Hermandw at skynet.be (Herman De Wael) Date: Thu, 13 Jan 2011 12:26:34 +0100 Subject: [BLML] EBU L&EC meeting 3rd November 2010 [SEC=UNOFFICIAL] In-Reply-To: <4D2ECE16.3070104@aol.com> References: <4D2EC70F.5000807@skynet.be> <4D2ECE16.3070104@aol.com> Message-ID: <4D2EE16A.8020304@skynet.be> Jeff Easterson wrote: >> This regulation made it into the WBF books for the sole purpose of >> taking away from me one small argument in favour of the dWS. > Do you really believe that you are so important that the WBF enacts > regulations ("for the sole purpose of....")? Well Jeff, I don't think I am, apparently I am! The example which even makes it into Richard's version of the 2017 lawws is precisely the one I used to explain the dWS argument that the MS tells opponents something which they have no use for, after which they will also have to tell them what the dWS says in the first place. To which Grattan replied with the convoluted reasoning which then made it onto the WBF inerpretations in Beijing. And if you think that is being important, then so be it. -- Herman De Wael Wilrijk Antwerpen Belgium From richard.willey at gmail.com Thu Jan 13 14:21:46 2011 From: richard.willey at gmail.com (richard willey) Date: Thu, 13 Jan 2011 08:21:46 -0500 Subject: [BLML] EBU L&EC meeting 3rd November 2010 [SEC=UNOFFICIAL] In-Reply-To: <4D2EE16A.8020304@skynet.be> References: <4D2EC70F.5000807@skynet.be> <4D2ECE16.3070104@aol.com> <4D2EE16A.8020304@skynet.be> Message-ID: On Thu, Jan 13, 2011 at 6:26 AM, Herman De Wael wrote: > Jeff Easterson wrote: > > >> This regulation made it into the WBF books for the sole purpose of > >> taking away from me one small argument in favour of the dWS. > > Do you really believe that you are so important that the WBF enacts > > regulations ("for the sole purpose of....")? > > Well Jeff, I don't think I am, apparently I am! > And if you think that is being important, then so be it. > > -- > Herman De Wael > Wilrijk Antwerpen Belgium > I sincerely apologize for ever characterizing you as an egotistical little twit. No idea where I ever got such a silly idea... > -- > I think back to the halcyon dates of my youth, when indeterminate Hessians > had something to do with the Revolutionary War, where conjugate priors were > monks who had broken their vows, and the expression (X'X)^-1(X'Y) was greek > > Those were simpler times > -------------- next part -------------- An HTML attachment was scrubbed... URL: http://lists.rtflb.org/pipermail/blml/attachments/20110113/78ac1a9a/attachment.html From ehaa at starpower.net Thu Jan 13 15:27:44 2011 From: ehaa at starpower.net (Eric Landau) Date: Thu, 13 Jan 2011 09:27:44 -0500 Subject: [BLML] Ecclesiastes In-Reply-To: <590903121.20221.1294850281161.JavaMail.ngmail@webmail12.arcor-online.net> References: <1Pd3FV-021OIi0@fwd08.aul.t-online.de> <4D2DD099.3000907@skynet.be> <590903121.20221.1294850281161.JavaMail.ngmail@webmail12.arcor-online.net> Message-ID: On Jan 12, 2011, at 11:38 AM, Thomas Dehn wrote: >> From: Herman De Wael >> >>> Given that both >>> options are open to the WBF (as are 37 others), why should one be >>> more >>> "wrong" or "right" than another. And simply stating that the WBF >>> chose >>> it will not do. > > The WBF position is the official position of the WBF. > That does not make it "right". (and neither "wrong") > > For example, for many decades the official position > of the Roman Catholic Church was that condoms cause HIV. There is an international convention regarding traffic signals to which all of the world's governments subscribe. It dictates that green shall signal "go", and red shall signal "stop". Both options were open to the authorities. Neither, logically, is more "wrong" nor "right" than the other (or any other pairing of colors that might have been chosen). Yet we all go on green and stop on red, simply because that is what the authorities chose. And it "does" quite nicely, thank you. Eric Landau 1107 Dale Drive Silver Spring MD 20910 ehaa at starpower.net From ehaa at starpower.net Thu Jan 13 15:38:38 2011 From: ehaa at starpower.net (Eric Landau) Date: Thu, 13 Jan 2011 09:38:38 -0500 Subject: [BLML] EBU L&EC meeting 3rd November 2010 In-Reply-To: <2120355928.22652.1294857141249.JavaMail.ngmail@webmail18.arcor-online.net> References: <2120355928.22652.1294857141249.JavaMail.ngmail@webmail18.arcor-online.net> Message-ID: <4D740FAD-0D83-40B1-879C-5252F6BA6E0F@starpower.net> On Jan 12, 2011, at 1:32 PM, Thomas Dehn wrote: > Eric Landau > >> We've devoted a lot of virtual ink to debating the moral and >> practical aspects of having laws by which the outcome of an >> adjudication depends on the adjudicators assessment of the "class of >> player involved". I am firmly in the camp which deprecates such >> laws. I would argue for substituting something like "an irrational >> action" for "a ridiculous error" in L12C1(b) above, and omitting the >> footnote. > > We have debated the existing "irrational" in L70 many times. > "irrational", without a detailed definition of "irrational", > is not sufficiently clear to convey the intended meaning. I don't see how defining or interpreting "irrational" is any more difficult than defining or interpreting "ridiculous". The point of debating the choice of word is that, using "dictionary" (as opposed to bridge-specific) definitions, what constitutes an "irrational" action (in a given situation) cannot depend on the "class" of the actor, whereas what consitutes a "ridiculous" action can. The issue for debate here is whether we want the application of L12C1 (b) to produce different outcomes in otherwise identical situations for different "classes of player". Once we decide what the law should say, we can quibble productively about how best to say it. Eric Landau 1107 Dale Drive Silver Spring MD 20910 ehaa at starpower.net From ehaa at starpower.net Thu Jan 13 16:10:55 2011 From: ehaa at starpower.net (Eric Landau) Date: Thu, 13 Jan 2011 10:10:55 -0500 Subject: [BLML] EBU L&EC meeting 3rd November 2010 In-Reply-To: References: Message-ID: <0CB8DAD0-BBFC-4220-BAB0-ED65F327E6CE@starpower.net> On Jan 12, 2011, at 7:31 PM, richard.hills at immi.gov.au wrote: > 5.11 New code of Laws > Mr Endicott said that the first preliminary discussions about the > 2017 code of laws was scheduled for about six months time. Any > suggestions for law amendments should be sent directly to Grattan. > > +=+=+=+=+=+=+=+=+=+=+=+=+=+=+=+=+=+=+=+=+=+=+=+=+=+=+=+=+=+=+=+=+ > > 2018 LAW 70 - CONTESTED CLAIM OR CONCESSION > > A. General Objective > > In ruling on a contested claim or concession, the Director > adjudicates the result of the board as equitably as possible to > both sides, but any doubtful point as to a claim shall be > resolved against the claimer (unless the non-claiming side has > committed an unestablished revoke, in which case any doubtful > point as to the claim shall be resolved in favour of the > claimer). The Director proceeds as follows. [...] I don't understand the rationale for the parenthetical clause. There's no logical connection between an unestablished revoke and the "benefit of the doubt" on specific points of the claim, and it seems contrary to the "equity" objective defined in the introduction. And I see no reason to muck up the claims/concession rules with a fine point of revoke-handling procedure. IMO, we should decide how we want to handle revokes (established or not) that occur prior to a claim, incorporate the language into the section on revokes, and cross-reference it from the part on claims. Eric Landau 1107 Dale Drive Silver Spring MD 20910 ehaa at starpower.net From agot at ulb.ac.be Thu Jan 13 16:26:08 2011 From: agot at ulb.ac.be (Alain Gottcheiner) Date: Thu, 13 Jan 2011 16:26:08 +0100 Subject: [BLML] Ecclesiastes In-Reply-To: References: <1Pd3FV-021OIi0@fwd08.aul.t-online.de> <4D2DD099.3000907@skynet.be> <590903121.20221.1294850281161.JavaMail.ngmail@webmail12.arcor-online.net> Message-ID: <4D2F1990.7020903@ulb.ac.be> Le 13/01/2011 15:27, Eric Landau a ?crit : > On Jan 12, 2011, at 11:38 AM, Thomas Dehn wrote: > >>> From: Herman De Wael >>> >>>> Given that both >>>> options are open to the WBF (as are 37 others), why should one be >>>> more >>>> "wrong" or "right" than another. And simply stating that the WBF >>>> chose >>>> it will not do. >> The WBF position is the official position of the WBF. >> That does not make it "right". (and neither "wrong") >> >> For example, for many decades the official position >> of the Roman Catholic Church was that condoms cause HIV. > There is an international convention regarding traffic signals to > which all of the world's governments subscribe. It dictates that > green shall signal "go", and red shall signal "stop". Both options > were open to the authorities. Neither, logically, is more "wrong" > nor "right" than the other (or any other pairing of colors that might > have been chosen). Yet we all go on green and stop on red, simply > because that is what the authorities chose. And it "does" quite > nicely, thank you. > AG : I see the point, but actually you're wrong in saying that options are equivalent in this field. Many studies have proven that a green light is much easier to spot when it lights up (our eyes are more sensible to green). So, the opposite option would have much to recommend it on security criteria. Of course, that doesn't change anything to your argument that we'd rather accept the common convention, but it shows that "wrong" has a sense when it comes to conventions. Best regards Alainb From ehaa at starpower.net Thu Jan 13 16:42:17 2011 From: ehaa at starpower.net (Eric Landau) Date: Thu, 13 Jan 2011 10:42:17 -0500 Subject: [BLML] EBU L&EC meeting 3rd November 2010 - L20 In-Reply-To: <1242552473.588.1294899908423.JavaMail.ngmail@webmail17.arcor-online.net> References: <1242552473.588.1294899908423.JavaMail.ngmail@webmail17.arcor-online.net> Message-ID: <0DB11DA8-A475-4963-9896-F51A427619C7@starpower.net> On Jan 13, 2011, at 1:25 AM, Thomas Dehn wrote: > richard.hills at immi.gov.au wrote: > >> 2018 LAW 20 - REVIEW AND EXPLANATION OF CALLS [...] >> F. Explanation of Calls >> >> 1. (a) During the auction before the final pass, any player may >> request, but only at her own turn to call, honest*** explanations >> of the opponents' prior auction. She is entitled to know about >> calls actually made, about relevant alternative calls available >> that were not made, and about relevant inferences from the choice >> of action where these are matters of partnership understanding. >> She is not entitled to know about calls that might be made in an >> entirely different auction (for example, if the pre-existing >> mutual partnership understanding of the opponents in a particular >> auction is that 4NT promises 5/5 in the minors, the player is not >> entitled at that time to know the opponents' responses to their >> 4NT Keycard Blackwood convention). > > I disagree with this. > Assume opponents suspect there was a misunderstanding, and the player > who responded to 4NT actually answered RKCB. > You are putting opponents at a disadvantage here - the player > who bid 4NT can work out that his partner had N key cards, > whereas opponents can not. Thomas is right. The "Kaplan paradigm" may be defunct, but, hopefully, the much simpler ("ACBL") paradigm still lives: Your opponents are entitled to know as much about the meaning of partner's call as you do. If you bid 4NT, systemically requiring partner to pick a minor, and he bids 5H, you know that if he mistakenly thought he was responding to a keycard-ask he has some agreed holding in key cards, while all your opponents would be allowed to know under Richard's proposal is that 5H is meaningless per your agreements. You get to decide for yourself whether some "call[] that might be made in an entirely different auction" is relevant to understanding this one; why should your opponents be denied the same opportunity? Eric Landau 1107 Dale Drive Silver Spring MD 20910 ehaa at starpower.net From ehaa at starpower.net Thu Jan 13 17:10:52 2011 From: ehaa at starpower.net (Eric Landau) Date: Thu, 13 Jan 2011 11:10:52 -0500 Subject: [BLML] EBU L&EC meeting 3rd November 2010 In-Reply-To: <4D2EC70F.5000807@skynet.be> References: <4D2EC70F.5000807@skynet.be> Message-ID: On Jan 13, 2011, at 4:34 AM, Herman De Wael wrote: > Eric Landau: > >> It would also explicitly repudiate the "Kaplan paradigm" >> of full disclosure, leaving us without an overarching set >> of principles to guide us in those inevitable situations >> not explicitly and unambiguously covered by the rules. > > Hands up those who believes the Kaplan paradigm ought to be > reinstated! If not reinstated, then replaced by something equally cogent. No body of law can cover every possible detail of every possible situation. To be applied fairly and consistently, a body of law must have overarching stated fundamental objectives and paradigms that can by used by those applying it to those details of those situations which it does not explicitly address. To be adequate to its purpose, the law must not only provide specific prescriptions for specific situations, but must also provide guidance to the resolution of situations that are not specifically addressed or for which the wording of the law is ambiguous. Eric Landau 1107 Dale Drive Silver Spring MD 20910 ehaa at starpower.net From ehaa at starpower.net Thu Jan 13 17:23:53 2011 From: ehaa at starpower.net (Eric Landau) Date: Thu, 13 Jan 2011 11:23:53 -0500 Subject: [BLML] Alain's case revisited In-Reply-To: <001001cbb308$57dd6af0$079840d0$@no> References: <4D2B0857.7050804@skynet.be> <4D2AC556.9070509@skynet.be> <4D2AC9E7.6060208@aol.com> <4D2AE13B.10605@skynet.be> <334994.97415.qm@web28503.mail.ukl.yahoo.com> <1391468851.32037.1294667187404.JavaMail.ngmail@webmail08.arcor-online.net> <4D2B1D0C.7090801@skynet.be> <4D2B2CC8.2060308@ulb.ac.be> <4D2E041A.8070508@nhcc.net> <000e01cbb2a9$ba3fc670$2ebf5350$@no> <707261.93703.qm@web28501.mail.ukl.yahoo.com> <001001cbb308$57dd6af0$079840d0$@no> Message-ID: <51527230-BFC9-4547-AA27-C974619AC3AB@starpower.net> On Jan 13, 2011, at 4:57 AM, Sven Pran wrote: >> On Behalf Of Nigel Guthrie >> >> In RAs that permit such variations, each side's choice of option >> after a call-out-of- >> turn or insufficient bid, can trigger different sets of >> understandings. This provides >> an additional channel of communication. Even in RAs that >> ostensibly forbid such >> ploys, some directors believe there are variations contingent on >> such choices that >> are a matter of Bridge-logic rather than agreement. > > This is not communication between partners any more than the fact > that a > different set of partnership understandings may apply depending on > opponents' disclosed agreements. > > Varying partnership understandings following an irregularity by > opponents > can be necessary and/or a logical consequence of the situation > based on > general bridge knowledge. > > An example from my own directing experience: 4NT was followed by 4D > (IB) > from LHO. Partner accepted the IB and responded to 4NT with a logical > alternative call among also the five more calls now becoming > available. (The > partnership had of course no prearranged agreement for such a > situation, but > they were strong players with sufficient understanding of auction > theory to > in flight deploy the five unexpected additionally available calls.) Isn't this obvious? What is the alternative? That you're allowed to accept 4D but not allowed to bid 4H, 4S, or 4NT over it? Ridiculous! That you're allowed to bid 4H, 4S or 4NT over the accepted 4D but partner isn't allowed to think about what you might have intended by doing so? Even ridiculouser! If we're going to abandon playing bridge because someone made an insufficient bid, why not just candel the board, award A-/A+, and be done with it? Eric Landau 1107 Dale Drive Silver Spring MD 20910 ehaa at starpower.net From Hermandw at skynet.be Thu Jan 13 17:28:32 2011 From: Hermandw at skynet.be (Herman De Wael) Date: Thu, 13 Jan 2011 17:28:32 +0100 Subject: [BLML] Ecclesiastes In-Reply-To: References: <1Pd3FV-021OIi0@fwd08.aul.t-online.de> <4D2DD099.3000907@skynet.be> <590903121.20221.1294850281161.JavaMail.ngmail@webmail12.arcor-online.net> Message-ID: <4D2F2830.5030606@skynet.be> Eric Landau wrote: > On Jan 12, 2011, at 11:38 AM, Thomas Dehn wrote: > > > There is an international convention regarding traffic signals to > which all of the world's governments subscribe. It dictates that > green shall signal "go", and red shall signal "stop". Both options > were open to the authorities. Neither, logically, is more "wrong" > nor "right" than the other (or any other pairing of colors that might > have been chosen). Yet we all go on green and stop on red, simply > because that is what the authorities chose. And it "does" quite > nicely, thank you. > It would be silly to argue for the change of these colors, but I don't consider it equally silly to change the Beijing interpretation. Anyway, lots of people are currently argueing about this, so whether or not it is silly, that is moot. But if you do want to discuss whether green ought to be changed into blue, then the argument "the whole world does it" is not a valid argument. And if you want to discuss whether the Beijing interpretation ought to be changed, saying "the Beijing interpretation exists" helps nothing. And if you don't want to discuss this, because you feel there is no use in trying to change anything, then please don't post in this thread. > > Eric Landau > 1107 Dale Drive > Silver Spring MD 20910 > ehaa at starpower.net > > _______________________________________________ > Blml mailing list > Blml at rtflb.org > http://lists.rtflb.org/mailman/listinfo/blml > > > > > No virus found in this incoming message. > Checked by AVG - www.avg.com > Version: 9.0.872 / Virus Database: 271.1.1/3376 - Release Date: 01/12/11 20:34:00 > -- Herman De Wael Wilrijk Antwerpen Belgium From svenpran at online.no Thu Jan 13 18:58:37 2011 From: svenpran at online.no (Sven Pran) Date: Thu, 13 Jan 2011 18:58:37 +0100 Subject: [BLML] Alain's case revisited In-Reply-To: <51527230-BFC9-4547-AA27-C974619AC3AB@starpower.net> References: <4D2B0857.7050804@skynet.be> <4D2AC556.9070509@skynet.be> <4D2AC9E7.6060208@aol.com> <4D2AE13B.10605@skynet.be> <334994.97415.qm@web28503.mail.ukl.yahoo.com> <1391468851.32037.1294667187404.JavaMail.ngmail@webmail08.arcor-online.net> <4D2B1D0C.7090801@skynet.be> <4D2B2CC8.2060308@ulb.ac.be> <4D2E041A.8070508@nhcc.net> <000e01cbb2a9$ba3fc670$2ebf5350$@no> <707261.93703.qm@web28501.mail.ukl.yahoo.com> <001001cbb308$57dd6af0$079840d0$@no> <51527230-BFC9-4547-AA27-C974619AC3AB@starpower.net> Message-ID: <000901cbb34b$81150a70$833f1f50$@no> On Behalf Of Eric Landau > On Jan 13, 2011, at 4:57 AM, Sven Pran wrote: > > >> On Behalf Of Nigel Guthrie > >> > >> In RAs that permit such variations, each side's choice of option > >> after a call-out-of- turn or insufficient bid, can trigger different > >> sets of understandings. This provides an additional channel of > >> communication. Even in RAs that ostensibly forbid such ploys, some > >> directors believe there are variations contingent on such choices > >> that are a matter of Bridge-logic rather than agreement. > > > > This is not communication between partners any more than the fact that > > a different set of partnership understandings may apply depending on > > opponents' disclosed agreements. > > > > Varying partnership understandings following an irregularity by > > opponents can be necessary and/or a logical consequence of the > > situation based on general bridge knowledge. > > > > An example from my own directing experience: 4NT was followed by 4D > > (IB) > > from LHO. Partner accepted the IB and responded to 4NT with a logical > > alternative call among also the five more calls now becoming > > available. (The partnership had of course no prearranged agreement for > > such a situation, but they were strong players with sufficient > > understanding of auction theory to in flight deploy the five > > unexpected additionally available calls.) > > Isn't this obvious? What is the alternative? That you're allowed to > accept 4D but not allowed to bid 4H, 4S, or 4NT over it? > Ridiculous! That you're allowed to bid 4H, 4S or 4NT over the accepted 4D but > partner isn't allowed to think about what you might have intended by doing so? > Even ridiculouser! If we're going to abandon playing bridge because someone > made an insufficient bid, why not just candel the board, award A-/A+, and be done > with it? This ought to be obvious, but it isn't, Quote from Law 40B3: The Regulating Authority may disallow prior agreement by a partnership to vary its understandings during the auction or play following a question asked, a response to a question, or any irregularity. When we worked on these laws back in 2007 I suggested (without luck) the following changes: - a response given by own side to a question asked by opponents - any irregularity committed by own side. Note that technically Law 40B3 can be used to prohibit a partnership from varying their agreement according to the nature of opponents' calls revealed through a question! Regards Sven From richard.hills at immi.gov.au Thu Jan 13 21:54:19 2011 From: richard.hills at immi.gov.au (richard.hills at immi.gov.au) Date: Fri, 14 Jan 2011 07:54:19 +1100 Subject: [BLML] EBU L&EC meeting 3rd November 2010 [SEC=UNOFFICIAL] In-Reply-To: <4D740FAD-0D83-40B1-879C-5252F6BA6E0F@starpower.net> Message-ID: Eric Landau: [snip] >The issue for debate here is whether we want the application of >L12C1(b) to produce different outcomes in otherwise identical >situations for different "classes of player". Once we decide >what the law should say, we can quibble productively about how >best to say it. Richard Hills: Agreed. In my opinion, the WBF LC was quite right in its interpretation of Law 12C1(b) that "class of player" would apply, since that makes Law 12C1(b) less punitive towards grass-roots players and more punitive towards expert players. In my opinion, the WBF Drafting Committee was quite right in including "class of player" in its definition of Logical Alternative, since that makes Law 16B less punitive towards grass-roots players and more punitive towards expert players. But..... In my opinion, "class of player" should be deleted from the claim Laws footnote, since currently assessment of claims is much more punitive towards grass roots players and way too insufficiently punitive towards expert players. Best wishes Richard Hills Recruitment Section Specialist Recruitment Team Level 5 Aqua, workstation W569, 6223 8453 DIAC Social Club movie tickets -------------------------------------------------------------------- Important Notice: If you have received this email by mistake, please advise the sender and delete the message and attachments immediately. This email, including attachments, may contain confidential, sensitive, legally privileged and/or copyright information. Any review, retransmission, dissemination or other use of this information by persons or entities other than the intended recipient is prohibited. DIAC respects your privacy and has obligations under the Privacy Act 1988. The official departmental privacy policy can be viewed on the department's website at www.immi.gov.au. See: http://www.immi.gov.au/functional/privacy.htm --------------------------------------------------------------------- From ardelm at optusnet.com.au Thu Jan 13 22:56:26 2011 From: ardelm at optusnet.com.au (Tony Musgrove) Date: Fri, 14 Jan 2011 08:56:26 +1100 Subject: [BLML] Ecclesiastes In-Reply-To: <4D2F1990.7020903@ulb.ac.be> References: <1Pd3FV-021OIi0@fwd08.aul.t-online.de> <4D2DD099.3000907@skynet.be> <590903121.20221.1294850281161.JavaMail.ngmail@webmail12.arcor-online.net> <4D2F1990.7020903@ulb.ac.be> Message-ID: <201101132156.p0DLukxJ005124@mail02.syd.optusnet.com.au> At 02:26 AM 14/01/2011, you wrote: >Le 13/01/2011 15:27, Eric Landau a ?crit : > > On Jan 12, 2011, at 11:38 AM, Thomas Dehn wrote: > > > >>> From: Herman De Wael > >>> > >>>> Given that both > >>>> options are open to the WBF (as are 37 others), why should one be > >>>> more > >>>> "wrong" or "right" than another. And simply stating that the WBF > >>>> chose > >>>> it will not do. > >> The WBF position is the official position of the WBF. > >> That does not make it "right". (and neither "wrong") > >> > >> For example, for many decades the official position > >> of the Roman Catholic Church was that condoms cause HIV. > > There is an international convention regarding traffic signals to > > which all of the world's governments subscribe. It dictates that > > green shall signal "go", and red shall signal "stop". Both options > > were open to the authorities. Neither, logically, is more "wrong" > > nor "right" than the other (or any other pairing of colors that might > > have been chosen). Yet we all go on green and stop on red, simply > > because that is what the authorities chose. And it "does" quite > > nicely, thank you. > > >AG : I see the point, but actually you're wrong in saying that options >are equivalent in this field. > >Many studies have proven that a green light is much easier to spot when >it lights up (our eyes are more sensible to green). >So, the opposite option would have much to recommend it on security >criteria. > >Of course, that doesn't change anything to your argument that we'd >rather accept the common convention, but it shows that "wrong" has a >sense when it comes to conventions. > >Best regards > > Alainb What about that non inconsiderable proportion of drivers who are red/green colourblind? Cheers, Tony (Sydney) >_______________________________________________ >Blml mailing list >Blml at rtflb.org >http://lists.rtflb.org/mailman/listinfo/blml From svenpran at online.no Thu Jan 13 23:22:30 2011 From: svenpran at online.no (Sven Pran) Date: Thu, 13 Jan 2011 23:22:30 +0100 Subject: [BLML] Ecclesiastes In-Reply-To: <201101132156.p0DLukxJ005124@mail02.syd.optusnet.com.au> References: <1Pd3FV-021OIi0@fwd08.aul.t-online.de> <4D2DD099.3000907@skynet.be> <590903121.20221.1294850281161.JavaMail.ngmail@webmail12.arcor-online.net> <4D2F1990.7020903@ulb.ac.be> <201101132156.p0DLukxJ005124@mail02.syd.optusnet.com.au> Message-ID: <000f01cbb370$5e421810$1ac64830$@no> Of Tony Musgrove > What about that non inconsiderable proportion of drivers who are red/green > colourblind? According to what I have read somewhere this is taken care of in two ways: 1: The red light is always uppermost and the green light is always at the bottom. 2: The particular shades of the red and green lights are selected so that red/green colourblind people shall still recognize the difference. From richard.hills at immi.gov.au Thu Jan 13 23:34:07 2011 From: richard.hills at immi.gov.au (richard.hills at immi.gov.au) Date: Fri, 14 Jan 2011 09:34:07 +1100 Subject: [BLML] He and she, whe and ble [SEC=UNOFFICIAL] Message-ID: Thomas Dehn: >First, a point on use of language. >Below, "he" has been replaced with "she", "his" with "her" >and so on. I consider this to be a bad idea. > >If you can create gender neutral language, great. >But that looks silly to me. Richard Hills: What's the problem? The 2007 Introduction states "the masculine includes the feminine, and vice versa" which is logically identical to "the feminine includes the masculine, and vice versa". Best wishes Richard Hills Recruitment Section Specialist Recruitment Team Level 5 Aqua, workstation W569, 6223 8453 DIAC Social Club movie tickets A Person Paper on Purity in Language William Satire (alias Douglas R. Hofstadter) From Metamagical Themas: Questing for the Essence of Mind and Pattern, by Douglas R. Hofstadter, Basic Books, 1985. (Original web version: http://www.bloomington.in.us/~abangert/person.html) It's high time someone blew the whistle on all the silly prattle about revamping our language to suit the purposes of certain political fanatics. You know what I'm talking about-those who accuse speakers of English of what they call "racism." This awkward neologism, constructed by analogy with the well-established term "sexism," does not sit well in the ears, if I may mix my metaphors. But let us grant that in our society there may be injustices here and there in the treatment of either race from time to time, and let us even grant these people their terms "racism" and "racist." How valid, however, are the claims of the self-proclaimed "black libbers," or "negrists"-those who would radically change our language in order to "liberate" us poor dupes from its supposed racist bias? Most of the clamor,as you certainly know by now, revolves around the age-old usage of the noun "white" and words built from it, such as chairwhite, mailwhite, repairwhite, clergywhite, middlewhite, Frenchwhite, forewhite, whitepower, whiteslaughter, oneupuwhiteship, straw white, whitehandle, and so on. The negrists claim that using the word "white," either on its own or as a component, to talk about all the members of the human species is somehow degrading to blacks and reinforces racism. Therefore the libbers propose that we substitute "person" everywhere where "white" now occurs. Sensitive speakers of our secretary tongue of course find this preposterous. There is great beauty to a phrase such as "All whites are created equal." Our forebosses who framed the Declaration of Independence well understood the poetry of our language. Think how ugly it would be to say "All persons are created equal," or "All whites and blacks are created equal." Besides, as any schoolwhitey can tell you, such phrases are redundant. In most contexts, it is self-evident when "white" is being used in an inclusive sense, in which case it subsumes members of the darker race just as much as fairskins. There is nothing denigrating to black people in being subsumed under the rubric "white"-no more than under the rubric "person." After all, white is a mixture of all the colors of the rainbow, including black. Used inclusively, the word "white" has no connotations whatsoever of race. Yet many people are hung up on this point. A prime example is Abraham Moses, one of the more vocal spokeswhites for making such a shift. For years, Niss Moses, authoroon of the well-known negrist tracts A Handbook of Nonracist Writing and Words and Blacks, has had nothing better to do than go around the country making speeches advocating the downfall of "racist language" that ble objects to. But when you analyze bler objections, you find they all fall apart at the seams. Niss Moses says that words like "chairwhite" suggest to people-most especially impressionable young whiteys and blackeys-that all chairwhites belong to the white race. How absurd! It's quite obvious, for instance, that the chairwhite of the League of Black Voters is going to be a black, not a white. Nobody need think twice about it. As a matter of fact, the suffix "white" is usually not pronounced with a long "i" as in the noun "white," but like "wit," as in the terms saleswhite, freshwhite, penwhiteship, first basewhite, and so on. It's just a simple and useful component in building race-neutral words. But Niss Moses would have you sit up and start hollering "Racism!" In fact, Niss Moses sees evidence of racism under every stone. Ble has written a famous article, in which ble vehemently objects to the immortal and poetic words of the first white on the moon, Captain Nellie Strongarm. If you will recall, whis words were: "One small step for a white, a giant step for whitekind." This noble sentiment is anything but racist; it is simply a celebration of a glorious moment in the history of White. Another of Niss Moses' shrill objections is to the age-old differentiation of whites from blacks by the third-person pronouns "whe" and "ble." Ble promotes an absurd notion: that what we really need in English is a single pronoun covering both races. Numerous suggestions have been made, such as "pe," "tey," and others, These are all repugnant to the nature of the English language, as the average white in the street will testify, even if whe has no linguistic training whatsoever. Then there are advocates of usages such as "whe or ble," "whis or bler," and so forth. This makes for monstrosities such as the sentence "When the next President takes office, whe or ble will have to choose whis or bler cabinet with great care, for whe or ble would not want to offend any minorities." Contrast this with the spare elegance of the normal way of putting it, and there is no question which way we ought to speak. There are, of course, some yapping black libbers who advocate writing "bl/whe" everywhere, which, aside from looking terrible, has no reasonable pronunciation. Shall we say "blooey" all the time when we simply mean "whe"? Who wants to sound like a white with a chronic sneeze? One of the more hilarious suggestions made by the squawkers for this point of view is to abandon the natural distinction along racial lines, and to replace it with a highly unnatural one along sexual lines. One such suggestion-emanating, no doubt, from the mind of a madwhite-would have us say "he" for male whites (and blacks) and "she" for female whites (and blacks). Can you imagine the outrage with which sensible folk of either sex would greet this "modest proposal"? Another suggestion is that the plural pronoun "they" be used in place of the inclusive "whe." This would turn the charming proverb "Whe who laughs last, laughs best" into the bizarre concoction "They who laughs last, laughs best." As if anyone in whis right mind could have thought that the original proverb applied only to the white race! No, we don't need a new pronoun to "liberate" our minds. That's the lazy white's way of solving the pseudoproblem of racism. In any case, it's ungrammatical. The pronoun "they" is a plural pronoun, and it grates on the civilized ear to hear it used to denote only one person. Such a usage, if adopted, would merely promote illiteracy and accelerate the already scandalously rapid nosedive of the average intelligence level in our society. Niss Moses would have us totally revamp the English language to suit bler purposes. If, for instance, we are to substitute "person" for "white," where are we to stop? If we were to follow Niss Moses' ideas to their logical conclusion, we would have to conclude that ble would like to see small blackeys and whiteys playing the game of "Hangperson" and reading the story of "Snow Person and the Seven Dwarfs." And would ble have us rewrite history to say, "Don't shoot until you see the persons of their eyes"? Will pundits and politicians henceforth issue person papers? Will we now have egg yolks and egg persons? And pledge allegiance to the good old Red, Person, and Blue? Will we sing, "I'm dreaming of a person Christmas"? Say of a frightened white, "Whe's person as a sheet!"? Lament the increase of person-collar crime? Thrill to the chirping of bobpersons in our gardens? Ask a friend to person the table while we go visit the persons'room? Come off it, Niss Moses-don't personwash our language! What conceivable harm is there in such beloved phrases as "No white is an island," "Dog is white's best friend," or "White's inhumanity to white"? Who would revise such classic book titles as Bronob Jacowski's The Ascent of White or Eric Steeple Bell's Whites of Mathematics? Did the poet who wrote "The best-laid plans of mice and whites gang aft agley" believe that blacks' plans gang ne'er agley? Surely not! Such phrases are simply metaphors: everyone can see beyond that. Whe who interprets them as reinforcing racism must have a perverse desire to feel oppressed. "Personhandling" the language is a habit that not only Niss Moses but quite a few others have taken up recently For instance, Nrs. Delilah Buford has urged that we drop the useful distinction between "Niss" and "Nrs." (which, as everybody knows, is pronounced "Nissiz," the reason for which nobody knows!). Bler argument is that there is no need for the public to know whether a black is employed or not. Need is, of course, not the point. Ble conveniently sidesteps the fact that there is a tradition in our society of calling unemployed blacks "Niss" and employed blacks "Nrs." Most blacks-in fact, the vast ma jority-prefer it that way. They want the world to know what their employment status is, and for good reason. Unemployed blacks want prospective employers to know they are available, without having to ask embarrassing questions. Likewise, employed blacks are proud of having found a job, and wish to let the world know they are employed. This distinction provides a sense of security to all involved, in that everyone knows where ble fits into the scheme of things. But Nrs. Buford refuses to recognize this simple truth. Instead, ble shiftily turns the argument into one about whites, asking why it is that whites are universally addressed as "Master," without any differentiation between employed and unemployed ones. The answer, of course, is that in America and other Northern societies, we set little store by the employment status of whites, Nrs. Buford can do little to change that reality, for it seems to be tied to innate biological differences between whites and blacks. Many white-years of research, in fact, have gone into trying to understand why it is that employment status matters so much to black, yet relatively little to whites. It is true that both races have a longer life expectancy if employed, but of course people often do not act so as to maximize their life expectancy. So far, it remains a mystery. In any case, whites and blacks clearly have different constitutional inclinations, and different goals in life. And so I say, Vive na diff?rence! As for Nrs. Buford's suggestion that both "Niss" and "Nrs." be unified into the single form of address "Ns." (supposed to rhyme with "fizz"), all I have to say is, it is arbitrary and clearly a thousand years ahead of its time. Mind you, this "Ns. " is an abbreviation concocted out of thin air: it stands for absolutely nothing. Who ever heard of such toying with language? And while we're on this subject, have you yet run across the recently founded Ns. magazine, dedicated to the concerns of the "liberated black"? It's sure to attract the attention of a trendy band of black airheads for a little while, but serious blacks surely will see through its thin veneer of slick, glossy Madison Avenue approaches to life. Nrs. Buford also finds it insultingly asymmetric that when a black is employed by a white, ble changes bler firmly name to whis firmly name. But what's so bad about that? Every firm's core consists of a boss (whis job is to make sure long-term policies are well charted out) and a secretary (bler job is to keep corporate affairs running smoothly on a day-to-day basis). They are both equally important and vital to the firm's success. No one disputes this. Beyond them there may of course be other firmly members. Now it's quite obvious that all members of a given firm should bear the same firmly name-otherwise, what are you going to call the firm's products? And since it would be nonsense for the boss to change whis name, it falls to the secretary to change bler name. Logic, not racism, dictates this simple convention. What puzzles me the most is when people cut off their nose to spite their faces. Such is the case with the time-honored colored suffixes "oon" and "roon," found in familiar words such as ambassadroon, stewardoon, and sculptroon. Most blacks find it natur al and sensible to add those suffixes onto -nouns such as "aviator" or "waiter." A black who flies an airplane may proudly proclaim, "I'm an aviatroon!" But it would sound silly, if not ridiculous, for a black to say of blerself, "I work as a waiter." On the other hand, who could object to my saying that the lively Ticely Cyson is a great actroon, or that the hilarious Quill Bosby is a great comedioon? You guessed it-authoroons such as Niss Mildred Hempsley and Nrs. Charles White, both of whom angrily reject the appellation "authoroon," deep though its roots are in our language. Nrs. White, perhaps one of the finest poetoons of our day, for some reason insists on being known as a "poet." It leads on to wonder, is Nrs. White ashamed of being black, perhaps? I should hope not. White needs Black, and Black needs White, and neither race should feel ashamed. Some extreme negrists object to being treated with politeness and courtesy by whites. For example, they reject the traditional notion of "Negroes first," preferring to open doors for themselves, claiming that having doors opened for them suggest implicitly that society considers them inferior. Well, would they have it the other way? Would these incorrigible grousers prefer to open doors for whites? What do blacks want? Another unlikely word has recently become a subject of controversy: "blackey." This is, of course, the ordinary term for black children (including teenagers), and by affectionate extension it is often applied to older blacks. Yet, incredible though it seems, many blacks-even teen-age blackeys-now claim to have had their "consciousness raised," and are voguishly skittish about being called "blackeys." Yet it's as old as the hills for blacks employed in the same office to refer to themselves as "the office blackeys," And for their superior to call them "my blackeys" helps make the ambiance more relaxed and comfy for all. It's hardly the mortal insult that libbers claim it to be. Fortunately, most blacks are sensible people and realize that mere words do not demean; they know it's how they are used that counts. Most of the time, calling a black-especially an older black-a "blackey" is a thoughtful way of complimenting bler, making bler feel young, fresh, and hirable again. Lord knows, I certainly wouldn't object if someone told me that I looked whiteyish these days! Many young blackeys go through a stage of wishing they had been born white. Perhaps this is due to popular television shows like Superwhite and Batwhite, but it doesn't really matter. It is perfectly normal and healthy. Many of our most successful blacks were once tomwhiteys and feel no shame about it. Why should they? Frankly, I think tomwhiteys are often the cutest little blackeys-but that's just my opinion. In any case, Niss Moses (once again) raises a ruckus on this score, asking why we don't have a corresponding word for young whiteys who play blackeys' games and generally manifest a desire to be black. Well, Niss Moses, if this were a common phenomenon, we most assuredly would have such a word, but it just happens not to be. Who can say why? But given that tomwhiteys are a dime a dozen, it's nice to have a word for them. The lesson is that White must learn to fit language to reality; White cannot manipulate the world by manipulating mere words. An elementary lesson, to be sure, but for some reason Niss Moses and others of bler ilk resist learning it. Shifting from the ridiculous to the sublime, let us consider the Holy Bible. The Good Book is of course the source of some of the most beautiful language and profound imagery to be found anywhere. And who is the central character of the Bible? I am sure I need hardly remind you; it is God. As everyone knows, Whe is male and white, and that is an indisputable fact. But have you heard the latest joke promulgated by tasteless negrists? It is said that one of them died and went to Heaven and then returned. What did ble report? "I have seen God, and guess what? Ble's female!" Can anyone say that this is not blasphemy of the highest order? It just goes to show that some people will stoop to any depths in order to shock. I have shared this "joke" with a number of friends of mine (including several blacks, by the way), and, to a white, they have agreed that it sickens them to the core to see Our Lord so shabbily mocked. Some things are just in bad taste, and there are no two ways about it. It is scum like this who are responsible for some of the great problems in our society today, I am sorry to say. Well, all of this is just another skirmish in the age-old Battle of the Races, I guess, and we shouldn't take it too seriously. I am reminded of words spoken by the great British philosopher Alfred West Malehead in whis commencement address to my alma secretaria the University of North Virginia: "To enrich the language of whites is, certainly, to enlarge the range of their ideas." I agree with this admirable sentiment wholeheartedly. I would merely point out to the overzealous that there are some extravagant notions about language that should be recognized for what they are: cheap attempts to let dogmatic, narrow minds enforce their views on the speakers lucky enough to have inherited the richest, most beautiful and flexible language on earth, a language whose traditions run back through the centuries to such deathless poets as Milton, Shakespeare, Wordsworth, Keats, Walt Whitwhite, and so many others... Our language owes an incalculable debt to these whites for their clarity of vision and expression, and if the shallow minds of bandwagon-jumping negrists succeed in destroying this precious heritage for all whites of good will, that will be, without any doubt, a truly female day in the history of Northern White. Post Scriptum Perhaps this piece shocks you. It is meant to. The entire point of it is to use something that we find shocking as leverage to illustrate the fact that something that we usually close our eyes to is also very shocking. The most effective way I know to do so is to develop an extended analogy with something known as shocking and reprehensible. Racism is that thing, in this case. I am happy with this piece, despite-but also because of-its shock value. I think it makes its point better than any factual article could. As a friend of mine said, "It makes you so uncomfortable that you can't ignore it." I admit that rereading it makes even me, the author, uncomfortable! Numerous friends have warned me that in publishing this piece I am taking a serious risk of earning myself a reputation as a terrible racist. I guess I cannot truly believe that anyone would see this piece that way. To misperceive it this way would be like calling someone a vicious racist for telling other people "The word 'nigger' is extremely offensive." If allusions to racism, especially for the purpose of satirizing racism and its cousins, are confused with racism itself, then I think it is time to stop writing. Some people have asked me if to write this piece, I simply took a genuine William Safire column (appearing weekly in the New York Times Magazine under the title "On Language") and "fiddled" with it. That is far from the truth. For years I have collected examples of sexist language, and in order to produce this piece, I dipped into this collection, selected some of the choicest, and ordered them very carefully. "Translating" them into this alternate world was sometimes extremely difficult, and some words took weeks. The hardest terms of all, surprisingly enough, were "Niss," "Nrs.," and "Ns.," even though "Master" came immediately. The piece itself is not based on any particular article by William Safire, but Safire has without doubt been one of the most vocal opponents of nonsexist language reforms, and therefore merits being safired upon. Interestingly, Master Safire has recently spoken out on sexism in whis column (August 5, 1984). Lamenting the inaccuracy of writing either "Mrs. Ferraro" or "Miss Ferraro" to designate the Democratic vice-presidential candidate whose husband's name is "Zaccaro," whe writes: It breaks my heart to suggest this, but the time has come for Ms. We are no longer faced with a theory, but a condition. It is unacceptable for journalists to dictate to a candidate that she call herself Miss or else use her married name; it is equally unacceptable for a candidate to demand that newspapers print a blatant inaccuracy by applying a married honorific to a maiden name. How disappointing it is when someone finally winds up doing the right thing but for the wrong reasons! In Safire's case, this shift was entirely for journalistic rather than humanistic reasons! It's as if Safire wished that women had never entered the political ring, so that the Grand Old Conventions of English-good enough for our grandfathers- would never have had to be challenged. How heartless of women! How heartbreaking the toll on our beautiful language! A couple of weeks after I finished this piece, I ran into the book The Nonsexist Communicator, by Bobbye Sorrels. In it, there is a satire called "A Tale of Two Sexes," which is very interesting to compare with my "Person Paper," Whereas in mine, I slice the world orthogonally to the way it is actually sliced and then perform a mapping of worlds to establish a disorienting yet powerful new vision of our world, in hers, Mrs. Sorrels simply reverses the two halves of our world as it is actually sliced. Her satire is therefore in some ways very much like mine, and in other ways extremely different. It should be read. I do not know too many publications that discuss sexist language in depth. The finest I have come across are the aforementioned Handbook of Nonsexist Writing, by Casey Miller and Kate Swift; Words and Women, by the same authors; Sexist Language: A Modern Philosophical Analysis, edited by Mary Vetterling-Braggin; The Nonsexist Communicator, by Bobbye Sorrels; and a very good journal titled Women and Language News. Subscriptions are available at Centenary College of Louisiana, 2911 Centenary Boulevard, Shreveport, Louisiana 71104. My feeling about nonsexist English is that it is like a foreign language that I am learning. I find that even after years of practice, I still have to translate sometimes from my native language, which is sexist English. I know of no human being who speaks Nonsexist as their native tongue. It will be very interesting to see if such people come to exist. If so, it will have taken a lot of work by a lot of people to reach that point. One final footnote: My book G?del, Escher, Bach, whose dialogues were the source of my very first trepidations about my own sexism, is now being translated into various languages, and to my delight, the Tortoise, a green-blooded male if ever there was one in English, is becoming Madame Tortue in French, Signorina Ttirtaruga in Italian, and so on. Full circle ahead! -------------------------------------------------------------------- Important Notice: If you have received this email by mistake, please advise the sender and delete the message and attachments immediately. This email, including attachments, may contain confidential, sensitive, legally privileged and/or copyright information. Any review, retransmission, dissemination or other use of this information by persons or entities other than the intended recipient is prohibited. DIAC respects your privacy and has obligations under the Privacy Act 1988. The official departmental privacy policy can be viewed on the department's website at www.immi.gov.au. See: http://www.immi.gov.au/functional/privacy.htm --------------------------------------------------------------------- From larry at charmschool.orangehome.co.uk Thu Jan 13 23:46:58 2011 From: larry at charmschool.orangehome.co.uk (Larry) Date: Thu, 13 Jan 2011 22:46:58 -0000 Subject: [BLML] Ecclesiastes References: <1Pd3FV-021OIi0@fwd08.aul.t-online.de><4D2DD099.3000907@skynet.be> <590903121.20221.1294850281161.JavaMail.ngmail@webmail12.arcor-online.net> <4D2F1990.7020903@ulb.ac.be><201101132156.p0DLukxJ005124@mail02.syd.optusnet.com.au> <000f01cbb370$5e421810$1ac64830$@no> Message-ID: Red / Green is more prevalent in males. The fact that the POSITION is top=red etc. helps. However, if you drive fast enough the lights have little significance. What about that non inconsiderable proportion of drivers who are red/green >> colourblind? > > According to what I have read somewhere this is taken care of in two ways: > 1: The red light is always uppermost and the green light is always at the > bottom. > 2: The particular shades of the red and green lights are selected so that > red/green colourblind people shall still recognize the difference. > > _______________________________________________ > Blml mailing list > Blml at rtflb.org > http://lists.rtflb.org/mailman/listinfo/blml > > > > ----- > No virus found in this message. > Checked by AVG - www.avg.com > Version: 10.0.1191 / Virus Database: 1435/3377 - Release Date: 01/13/11 > From richard.hills at immi.gov.au Fri Jan 14 00:38:14 2011 From: richard.hills at immi.gov.au (richard.hills at immi.gov.au) Date: Fri, 14 Jan 2011 10:38:14 +1100 Subject: [BLML] Ecclesiastes [SEC=UNOFFICIAL] In-Reply-To: <4D2F2830.5030606@skynet.be> Message-ID: Ecclesiates 7:20 There is no one on earth who does what is right all the time and never makes a mistake. Herman De Wael: >..... >But if you do want to discuss whether green ought to be changed >into blue, then the argument "the whole world does it" is not a >valid argument. >..... Richard Hills: No, "the whole world does it" is the very valid QWERTY argument. Ergonomists have universally concluded that (for the English language) the Dvorak Simplified Keyboard is more efficient than QWERTY. But because the whole world does QWERTY, changing to Dvorak would require a substantial investment in time for typists to learn the new keyboard, and the cost of this time investment is greater than the eventual benefits received by using Dvorak. Thus a world-wide change from green traffic lights to blue traffic lights would be hugely costly for mere marginal benefit. Likewise, Nigel Guthrie's idea of very frequent online updates of the Laws of Duplicate Bridge is more efficient than the current decennial process, but would require a substantial investment in time by Directors to keep up-to-date. And the cost of this time investment (not to mention the cost of grass-roots Directors becoming confused by the constant changes) is greater than the eventual benefits of promptly perfecting pristine Laws. Best wishes Richard Hills Recruitment Section Specialist Recruitment Team Level 5 Aqua, workstation W569, 6223 8453 DIAC Social Club movie tickets -------------------------------------------------------------------- Important Notice: If you have received this email by mistake, please advise the sender and delete the message and attachments immediately. This email, including attachments, may contain confidential, sensitive, legally privileged and/or copyright information. Any review, retransmission, dissemination or other use of this information by persons or entities other than the intended recipient is prohibited. DIAC respects your privacy and has obligations under the Privacy Act 1988. The official departmental privacy policy can be viewed on the department's website at www.immi.gov.au. See: http://www.immi.gov.au/functional/privacy.htm --------------------------------------------------------------------- From diggadog at iinet.net.au Fri Jan 14 03:02:54 2011 From: diggadog at iinet.net.au (Bill & Helen Kemp) Date: Fri, 14 Jan 2011 10:02:54 +0800 Subject: [BLML] Ecclesiastes References: <1Pd3FV-021OIi0@fwd08.aul.t-online.de><4D2DD099.3000907@skynet.be> <590903121.20221.1294850281161.JavaMail.ngmail@webmail12.arcor-online.net> <4D2F1990.7020903@ulb.ac.be><201101132156.p0DLukxJ005124@mail02.syd.optusnet.com.au><000f01cbb370$5e421810$1ac64830$@no> Message-ID: <8279137F6E4644299878804880B3C25A@acer> Thank God this topic has finally morphed into something sensible. I was about to give up on it cheers bill ----- Original Message ----- From: "Larry" To: "Bridge Laws Mailing List" Sent: Friday, January 14, 2011 6:46 AM Subject: Re: [BLML] Ecclesiastes > Red / Green is more prevalent in males. The fact that the POSITION is > top=red etc. helps. > However, if you drive fast enough the lights have little significance. > > > > > What about that non inconsiderable proportion of drivers who are red/green >>> colourblind? >> >> According to what I have read somewhere this is taken care of in two >> ways: >> 1: The red light is always uppermost and the green light is always at the >> bottom. >> 2: The particular shades of the red and green lights are selected so that >> red/green colourblind people shall still recognize the difference. >> >> _______________________________________________ >> Blml mailing list >> Blml at rtflb.org >> http://lists.rtflb.org/mailman/listinfo/blml >> >> >> >> ----- >> No virus found in this message. >> Checked by AVG - www.avg.com >> Version: 10.0.1191 / Virus Database: 1435/3377 - Release Date: 01/13/11 >> > > > > _______________________________________________ > Blml mailing list > Blml at rtflb.org > http://lists.rtflb.org/mailman/listinfo/blml From swillner at nhcc.net Fri Jan 14 03:37:20 2011 From: swillner at nhcc.net (Steve Willner) Date: Thu, 13 Jan 2011 21:37:20 -0500 Subject: [BLML] Ecclesiastes In-Reply-To: References: <1Pd3FV-021OIi0@fwd08.aul.t-online.de> <4D2DD099.3000907@skynet.be> <590903121.20221.1294850281161.JavaMail.ngmail@webmail12.arcor-online.net> Message-ID: <4D2FB6E0.8000705@nhcc.net> On 1/13/2011 9:27 AM, Eric Landau wrote: > Yet we all go on green and stop on red, Obviously Eric has never driven in Boston. The following is not original, but it is, alas, only slightly exaggerated: Yellow means "go." Red means "speed up." Green means you better stop to avoid getting hit by all the cars going through on red. From blml at arcor.de Fri Jan 14 03:42:25 2011 From: blml at arcor.de (Thomas Dehn) Date: Fri, 14 Jan 2011 03:42:25 +0100 (CET) Subject: [BLML] He and she, whe and ble [SEC=UNOFFICIAL] Message-ID: <1923802510.31506.1294972945375.JavaMail.ngmail@webmail13.arcor-online.net> richard.hills at immi.gov.au wrote: > Thomas Dehn: > > >First, a point on use of language. > >Below, "he" has been replaced with "she", "his" with "her" > >and so on. I consider this to be a bad idea. > > > >If you can create gender neutral language, great. > >But that looks silly to me. > > Richard Hills: > > What's the problem? You are forcing translators to deviate from the English language version of TFLB. That is undesirable. This type of language usage offends some people. It will offend many people in some cultures. Then, you are using a language construct that is specific to English and cannot be used in any language where nouns have a grammatical gender. I think that effect can be demonstrated by replacing "player" with "sorcerer". Consider this line: "During the auction before the final pass, any player may request, but only at her own turn to call, honest*** explanations of the opponents' prior auction." There exist at least four different ways to translate this: 1.) "During the auction before the final pass, any sorceress may request, but only at her own turn to call, honest*** explanations of the opponents' prior auction." This is grammatically correct, but that radical feminist language usage offends many men. In many countries not accepted by society. 2.) "During the auction before the final pass, any sorcerer may request, but only at her own turn to call, honest*** explanations of the opponents' prior auction." This is grammatically incorrect, and will be considered silly by many, both men and women. 3.) "During the auction before the final pass, any sorcer(er|ess) may request, but only at h(is|er) own turn to call, honest*** explanations of the opponents' prior auction." or "During the auction before the final pass, any sorcerer/ess may request, but only at his/er own turn to call, honest*** explanations of the opponents' prior auction." Unfortunately, also considered silly by many. Also makes it difficult to read such a text to somebody else. Various such approaches have been tried, and all of them have already been soundly rejected. 4.) "During the auction before the final pass, any sorcerer may request, but only at his own turn to call, honest*** explanations of the opponents' prior auction." Well, that is the previous text, but it is now much more offensive than before because the "translation" removed the "her". This topic has created extensive flamewars in Germany and a few other countries, and society has basically decided to not use such language at this point. In some other countries, there isn't really a feminist movement yet, and the language issue hasn't even been discussed there. While you might want to change that, I think that TFLB is not the right place to try to change society. Now, here the gender neutral text: "During the auction before the final pass, players may request, but only at their own turn to call, honest*** explanations of the opponents' prior auction." No such problem there, this language usage is not offensive to anybody. Thomas From richard.hills at immi.gov.au Fri Jan 14 04:25:19 2011 From: richard.hills at immi.gov.au (richard.hills at immi.gov.au) Date: Fri, 14 Jan 2011 14:25:19 +1100 Subject: [BLML] Ecclesiastes [SEC=UNOFFICIAL] In-Reply-To: <4D2C10A3.2070408@gmail.com> Message-ID: Brian Meadows: >..... >If the lines of communication are broken at some point, the >level at which the break occurs is largely irrelevant to those >at the lowest levels. The WBF will continue to be seen as some >aloof and uncontactable body handing down their ex-cathedra >judgments. >..... Richard Hills: Yes, there is a problem. But it is the penultimate problem the fact that WBF LC interpretations of the Laws fail to reach most grass-roots Directors and interested players. The ultimate problem is that the theoretically unnecessary WBF LC interpretations are instead practically necessary because the Lawbook, which is freely available to grass-roots Directors and interested players, is not written in plain English. Grattan Endicott, 6th March 2006: >>..... >>plain English, nor that I have ever claimed perfection for >>any words that I have suggested. What I regard as open to >>severe attack is any complacency that is not interested to >>substitute simpler words where possible for the obscurities >>of language in the current laws (something that should be >>done, in my view, even when the current text, correctly >>understood, is adequate). >> The default position on this lies, of course, in the >>expectation that NBOs will have opportunity to consider the >>next code before it is set in concrete, and propose further >>improvements of expression. >> ~ Grattan ~ +=+ Ecclesiastes 10:12 What the wise say brings them honour Best wishes Richard Hills Recruitment Section Specialist Recruitment Team Level 5 Aqua, workstation W569, 6223 8453 DIAC Social Club movie tickets -------------------------------------------------------------------- Important Notice: If you have received this email by mistake, please advise the sender and delete the message and attachments immediately. This email, including attachments, may contain confidential, sensitive, legally privileged and/or copyright information. Any review, retransmission, dissemination or other use of this information by persons or entities other than the intended recipient is prohibited. DIAC respects your privacy and has obligations under the Privacy Act 1988. The official departmental privacy policy can be viewed on the department's website at www.immi.gov.au. See: http://www.immi.gov.au/functional/privacy.htm --------------------------------------------------------------------- From richard.hills at immi.gov.au Fri Jan 14 05:38:25 2011 From: richard.hills at immi.gov.au (richard.hills at immi.gov.au) Date: Fri, 14 Jan 2011 15:38:25 +1100 Subject: [BLML] Ecclesiastes [SEC=UNOFFICIAL] In-Reply-To: Message-ID: Jack Olsen, The Mad World of Bridge, page 82: "Taking the Kaufman convention as a starter and adding to it, one can easily see how one's holdings of fours and fives could then be shown, then sixes and sevens, and so on up to kings and aces, simply by a series of asking bids. Five no- trump would be an invitation bid asking for fours, six no- trump would ask for fives, and so on up to fourteen no- trump, which would be the asking bid for aces. Who could deny the accuracy with which one could determine one's partner's holding after such a series of Kaufman bids?" Jerry Fusselman: >>>..... >>>Carefully exploring a minority opinion is what BLML is all >>>about >>>..... Richard Hills: >>No. >> >>[instead] >> >>"Sufficiently exploring a minority opinion is what blml is >>all about." More than sufficient time has been given to >>the De Wael loophole on blml over the past decade. Herman De Wael: >Not only does Richard know what is best for bridge, he also >knows what is best for blml. Why do we bother listening to >such a tyrant? Ecclesiastes 10:12-14 but fools are destroyed by their own words. They start out with silly talk and end up with pure madness. A fool talks on and on. Richard Hills: Since I am a tyrannical fool, I know that it is best for bridge that Law 27 (Insufficient Bid) and Law 38 (Bid of More Than Seven) be abolished. This would permit a bid of 14 NT to ask for aces, followed by a conversion to 6 NT when an ace was discovered to be missing. All good bridge players would gain enjoyment from these new rules, since all good bridge players would be able to enhance their Symmetric Relay methods (notes emailed on request). And as a tyrannical fool I would not only flood blmlers with postings over the next decade advocating the abolition of Laws 27 and 38, I would also intentionally infract those two Laws at the table, since they are very obviously too complicated for me to obey or for the Director to enforce. But as a tyrannical fool I now suddenly realise that I did "indicate in any manner" the foolishness of the so-called De Wael School. Since any and all creation of unauthorised information should be an infraction (despite the fuddy- duddies sitting on the WBF Laws Committee choosing to rule otherwise), my unauthorised information revealing the foolishness of the so-called De Wael School deserves an extremely severe Disciplinary Penalty. Thus as a tyrannical fool I order the Disciplinary Penalty of my suspension from blml for the next two weeks. A very severe Disciplinary Penalty, since I will be deprived of the dulcet tones of Jerry Fusselman and Herman De Wael constructively criticising my character flaws. (It is pure coincidence that I will be playing in the Aussie Summer Festival of Bridge for the next two weeks, hence I would in any case lack the time to post to blml.) Best wishes Richard Hills tyrannical fool -------------------------------------------------------------------- Important Notice: If you have received this email by mistake, please advise the sender and delete the message and attachments immediately. This email, including attachments, may contain confidential, sensitive, legally privileged and/or copyright information. Any review, retransmission, dissemination or other use of this information by persons or entities other than the intended recipient is prohibited. DIAC respects your privacy and has obligations under the Privacy Act 1988. The official departmental privacy policy can be viewed on the department's website at www.immi.gov.au. See: http://www.immi.gov.au/functional/privacy.htm --------------------------------------------------------------------- From jfusselman at gmail.com Fri Jan 14 08:45:36 2011 From: jfusselman at gmail.com (Jerry Fusselman) Date: Fri, 14 Jan 2011 01:45:36 -0600 Subject: [BLML] EBU L&EC meeting 3rd November 2010 - L20 In-Reply-To: <0DB11DA8-A475-4963-9896-F51A427619C7@starpower.net> References: <1242552473.588.1294899908423.JavaMail.ngmail@webmail17.arcor-online.net> <0DB11DA8-A475-4963-9896-F51A427619C7@starpower.net> Message-ID: On Thu, Jan 13, 2011 at 9:42 AM, Eric Landau wrote: > > Thomas is right. ?The "Kaplan paradigm" may be defunct, but, > hopefully, the much simpler ("ACBL") paradigm still lives: ?Your > opponents are entitled to know as much about the meaning of partner's > call as you do. ?If you bid 4NT, systemically requiring partner to > pick a minor, and he bids 5H, you know that if he mistakenly thought > he was responding to a keycard-ask he has some agreed holding in key > cards, while all your opponents would be allowed to know under > Richard's proposal is that 5H is meaningless per your agreements. > You get to decide for yourself whether some "call[] that might be > made in an entirely different auction" is relevant to understanding > this one; why should your opponents be denied the same opportunity? > Thomas and Eric are right. Why give the advantage to Secretary Birds who would learn this trick over ethical players who freely give the opponents their useful system information? And why set up rules that give players a disadvantage if their convention card is complete? Jerry Fusselman From jfusselman at gmail.com Fri Jan 14 08:54:00 2011 From: jfusselman at gmail.com (Jerry Fusselman) Date: Fri, 14 Jan 2011 01:54:00 -0600 Subject: [BLML] EBU L&EC meeting 3rd November 2010 In-Reply-To: References: Message-ID: On Wed, Jan 12, 2011 at 8:30 AM, Eric Landau wrote: > > We've devoted a lot of virtual ink to debating the moral and > practical aspects of having laws by which the outcome of an > adjudication depends on the adjudicators assessment of the "class of > player involved". ?I am firmly in the camp which deprecates such > laws. ?I would argue for substituting something like "an irrational > action" for "a ridiculous error" in L12C1(b) above, and omitting the > footnote. > I recall this discussion, and Eric won me over. For me, the practical aspects decide the issue all by themselves. I now consider assessing "class of player" another case of attempted mind reading beyond human abilities to get right. I wonder if others see the practical aspect in the same way. Jerry Fusselman From jfusselman at gmail.com Fri Jan 14 09:17:00 2011 From: jfusselman at gmail.com (Jerry Fusselman) Date: Fri, 14 Jan 2011 02:17:00 -0600 Subject: [BLML] Expected Alerts [SEC=UNOFFICIAL] In-Reply-To: References: Message-ID: [Richard Hills] In my opinion by definition an Expected Alert can never contain any useful information. [Jerry Fusselman] I don't agree. In brief, expected is not the same as certain. But the value of this rule (i.e., expected alerts are AI), I think, is interesting. Suppose I am 90% sure that partner will remember our fancy slam-investigating continuations. If partner shows he remembers by his alerts, then his actions suggest that I would do well to continue to use the fancy system. If not, his actions suggest that I bang out 6S (or whatever). We allow the former, but not the latter. When the expected alert occurred, I became 99% sure partner remembered, so if I use this information, I will continue with the fun fancy stuff. This is allowed, has always been allowed, and I agree with it. But if the expected alert did not occur, partner has most likely forgot, and the fancy stuff is suddenly much more likely to lead to disaster. Jumping to slam to avoid a misunderstanding in this case is not allowed, and I agree with that. If, instead, the rules stated that the expected alert was UI, then we would tend to require the expected alerts to be answered by jumps to slam (or sign offs) because we cannot use UI to our advantage; fortunately, directors know better than to do this. And making the law state that expected alerts are AI is just the formal acceptance of what directors wisely do anyway. Have I explained it well? Jerry Fusselman From jfusselman at gmail.com Fri Jan 14 09:41:25 2011 From: jfusselman at gmail.com (Jerry Fusselman) Date: Fri, 14 Jan 2011 02:41:25 -0600 Subject: [BLML] Nigel's case supported [SEC=UNOFFICIAL] In-Reply-To: <4D2C1E71.7060004@skynet.be> References: <4D2C1E71.7060004@skynet.be> Message-ID: On Tue, Jan 11, 2011 at 3:10 AM, Herman De Wael wrote: > richard.hills at immi.gov.au wrote: >> >> there is a 0.00% chance that the De >> Wael loophole will be restored by the "intransigent" WBF LC. >> > > And why do you so categorically state this? > 0.00% is very little. Make it 0.001% and I'll wager 1 whatever on it, > 100000 to 1. > Yeah, me too. Actually, I would guess that there is about a 50% chance that dWS will be legal in 30 years. I will try to stay alive that long to see. I suppose, to really be sure about my estimate, that I should find out how the WBF LC is formed. How are they chosen? Jerry Fusselman From jfusselman at gmail.com Fri Jan 14 09:51:19 2011 From: jfusselman at gmail.com (Jerry Fusselman) Date: Fri, 14 Jan 2011 02:51:19 -0600 Subject: [BLML] Alain's case revisited [SEC=UNOFFICIAL] In-Reply-To: References: <4D258277.5030000@skynet.be> Message-ID: On Thu, Jan 6, 2011 at 9:43 PM, Richard Hills wrote: > > If one ethical partner unintentionally infracts Law 21B1(a), > thus preventing the later application of Law 75A, the > Director takes any advantage by adjusting the offending > side's score from +110 to -800. > Has this kind of rectification, where you assume a player has UI that never happened, has that ever, even once, been documented on planet earth? If no, then the Beijing minute is a wild change to the rules of bridge, despite the claims of those who pushed for it. Jerry Fusselman From harald.skjaran at gmail.com Fri Jan 14 11:05:15 2011 From: harald.skjaran at gmail.com (=?UTF-8?Q?Harald_Skj=C3=A6ran?=) Date: Fri, 14 Jan 2011 11:05:15 +0100 Subject: [BLML] Expected Alerts [SEC=UNOFFICIAL] In-Reply-To: References: Message-ID: 2011/1/14 Jerry Fusselman : > [Richard Hills] > > In my opinion > by definition an Expected Alert can never contain any useful > information. > > [Jerry Fusselman] > > I don't agree. ?In brief, expected is not the same as certain. ?But > the value of this rule (i.e., expected alerts are AI), I think, is > interesting. > > Suppose I am 90% sure that partner will remember our fancy > slam-investigating continuations. ?If partner shows he remembers by > his alerts, then his actions suggest that I would do well to continue > to use the fancy system. ?If not, his actions suggest that I bang out > 6S (or whatever). ?We allow the former, but not the latter. > > When the expected alert occurred, I became 99% sure partner > remembered, so if I use this information, I will continue with the fun > fancy stuff. ?This is allowed, has always been allowed, and I agree > with it. > > But if the expected alert did not occur, partner has most likely > forgot, and the fancy stuff is suddenly much more likely to lead to > disaster. ?Jumping to slam to avoid a misunderstanding in this case is > not allowed, and I agree with that. > > If, instead, the rules stated that the expected alert was UI, then we > would tend to require the expected alerts to be answered by jumps to > slam (or sign offs) because we cannot use UI to our advantage; > fortunately, directors know better than to do this. ?And making the > law state that expected alerts are AI is just the formal acceptance of > what directors wisely do anyway. > > Have I explained it well? Yes. But, unless those fancy slam-investigating continuations is done before reaching 3NT, there would be no alterts. At least in Norway. And in many other RA's as well. Your points are still valid though, in similar positions at lower levels. > > Jerry Fusselman > _______________________________________________ > Blml mailing list > Blml at rtflb.org > http://lists.rtflb.org/mailman/listinfo/blml > -- Kind regards, Harald Skj?ran From nigelguthrie at yahoo.co.uk Fri Jan 14 11:05:41 2011 From: nigelguthrie at yahoo.co.uk (Nigel Guthrie) Date: Fri, 14 Jan 2011 10:05:41 +0000 (GMT) Subject: [BLML] EBU L&EC meeting 3rd November 2010 In-Reply-To: References: Message-ID: <557247.99714.qm@web28504.mail.ukl.yahoo.com> [Jerry Fusselman] I recall this discussion, and Eric won me over. For me, the practical aspects decide the issue all by themselves. I now consider assessing "class of player" another case of attempted mind reading beyond human abilities to get right. I wonder if others see the practical aspect in the same way. {Nigel] I think a director can make crude judgements about the skills of the field as a whole. As far as individual assessments go, Eric and Jerry are right. A typical player has different levels of skill in different areas. The director cannot properly assess strangers and foreigners. It is demeaning for a players to have his ability assessed by a director and players may take reasonable offence. Experience as a local selector shows that a person's impression of another player's ability is usually wildly mistaken and more influenced by personal relationships than objective criteria. From Hermandw at skynet.be Fri Jan 14 11:38:08 2011 From: Hermandw at skynet.be (Herman De Wael) Date: Fri, 14 Jan 2011 11:38:08 +0100 Subject: [BLML] Ecclesiastes [SEC=UNOFFICIAL] In-Reply-To: References: Message-ID: <4D302790.8050401@skynet.be> richard.hills at immi.gov.au wrote: > Ecclesiates 7:20 > > There is no one on earth who does what is right all the time and > never makes a mistake. > > Herman De Wael: > >> ..... >> But if you do want to discuss whether green ought to be changed >> into blue, then the argument "the whole world does it" is not a >> valid argument. >> ..... > > Richard Hills: > > No, "the whole world does it" is the very valid QWERTY argument. > > Ergonomists have universally concluded that (for the English > language) the Dvorak Simplified Keyboard is more efficient than > QWERTY. But because the whole world does QWERTY, changing to Not me, or Alain. We use AZERTY! :) At last we know why the dWS is prevalent in Belgium! -- Herman De Wael Wilrijk Antwerpen Belgium From Hermandw at skynet.be Fri Jan 14 11:46:10 2011 From: Hermandw at skynet.be (Herman De Wael) Date: Fri, 14 Jan 2011 11:46:10 +0100 Subject: [BLML] He and she, whe and ble [SEC=UNOFFICIAL] In-Reply-To: <1923802510.31506.1294972945375.JavaMail.ngmail@webmail13.arcor-online.net> References: <1923802510.31506.1294972945375.JavaMail.ngmail@webmail13.arcor-online.net> Message-ID: <4D302972.9060002@skynet.be> Thomas Dehn wrote: > > > Now, here the gender neutral text: > > "During the auction before the final pass, players may > request, but only at their own turn to call, honest*** explanations > of the opponents' prior auction." > > No such problem there, this language usage is not offensive to anybody. > And it can be translated into German, indeed. But not into Franch, sadly. Joueurs and Joueuses do not share the same word. And there may be problems in other languages that we don't realize. I believe it is best that a general sentence is stated at the beginning saying that the masculine includes the feminine, and that gender-neutral languages is not used. I believe the use of the passive voice in L40A has done far more bad than good. And I can think of other examples, if I need. > > Thomas > -- Herman De Wael Wilrijk Antwerpen Belgium From blml at arcor.de Fri Jan 14 11:58:18 2011 From: blml at arcor.de (Thomas Dehn) Date: Fri, 14 Jan 2011 11:58:18 +0100 (CET) Subject: [BLML] Ecclesiastes [SEC=UNOFFICIAL] In-Reply-To: <4D302790.8050401@skynet.be> References: <4D302790.8050401@skynet.be> Message-ID: <1748200137.98748.1295002698502.JavaMail.ngmail@webmail08.arcor-online.net> Herman De Wael wrote: > richard.hills at immi.gov.au wrote: > > Ecclesiates 7:20 > > > > There is no one on earth who does what is right all the time and > > never makes a mistake. > > > > Herman De Wael: > > > >> ..... > >> But if you do want to discuss whether green ought to be changed > >> into blue, then the argument "the whole world does it" is not a > >> valid argument. > >> ..... > > > > Richard Hills: > > > > No, "the whole world does it" is the very valid QWERTY argument. > > > > Ergonomists have universally concluded that (for the English > > language) the Dvorak Simplified Keyboard is more efficient than > > QWERTY. But because the whole world does QWERTY, changing to > > Not me, or Alain. We use AZERTY! > > :) QUERTZ actually. :-) Thomas From agot at ulb.ac.be Fri Jan 14 13:59:00 2011 From: agot at ulb.ac.be (Alain Gottcheiner) Date: Fri, 14 Jan 2011 13:59:00 +0100 Subject: [BLML] Ecclesiastes [SEC=UNOFFICIAL] In-Reply-To: <1748200137.98748.1295002698502.JavaMail.ngmail@webmail08.arcor-online.net> References: <4D302790.8050401@skynet.be> <1748200137.98748.1295002698502.JavaMail.ngmail@webmail08.arcor-online.net> Message-ID: <4D304894.6070603@ulb.ac.be> Le 14/01/2011 11:58, Thomas Dehn a ?crit : > Herman De Wael wrote: >> richard.hills at immi.gov.au wrote: >>> Ecclesiates 7:20 >>> >>> There is no one on earth who does what is right all the time and >>> never makes a mistake. >>> >>> Herman De Wael: >>> >>>> ..... >>>> But if you do want to discuss whether green ought to be changed >>>> into blue, then the argument "the whole world does it" is not a >>>> valid argument. >>>> ..... >>> Richard Hills: >>> >>> No, "the whole world does it" is the very valid QWERTY argument. >>> >>> Ergonomists have universally concluded that (for the English >>> language) the Dvorak Simplified Keyboard is more efficient than >>> QWERTY. But because the whole world does QWERTY, changing to >> Not me, or Alain. We use AZERTY! >> >> :) > QUERTZ actually. :-) > > AG : never heard about QUERTZ, but I confirm the use of AZERTY. Those who have ergonomics in mind should remember that AZERTY and QWERTY had as main objective *poor* ergonomics (in order to avoid typing arm messes), and fairly well succeeded. From blml at arcor.de Fri Jan 14 14:18:47 2011 From: blml at arcor.de (Thomas Dehn) Date: Fri, 14 Jan 2011 14:18:47 +0100 (CET) Subject: [BLML] Ecclesiastes [SEC=UNOFFICIAL] In-Reply-To: <4D304894.6070603@ulb.ac.be> References: <4D304894.6070603@ulb.ac.be> <4D302790.8050401@skynet.be> <1748200137.98748.1295002698502.JavaMail.ngmail@webmail08.arcor-online.net> Message-ID: <1830091869.109325.1295011127645.JavaMail.ngmail@webmail08.arcor-online.net> Alain Gottcheiner wrote: > Le 14/01/2011 11:58, Thomas Dehn a ?crit : > > Herman De Wael wrote: > >> richard.hills at immi.gov.au wrote: > >>> Ecclesiates 7:20 > >>> > >>> There is no one on earth who does what is right all the time and > >>> never makes a mistake. > >>> > >>> Herman De Wael: > >>> > >>>> ..... > >>>> But if you do want to discuss whether green ought to be changed > >>>> into blue, then the argument "the whole world does it" is not a > >>>> valid argument. > >>>> ..... > >>> Richard Hills: > >>> > >>> No, "the whole world does it" is the very valid QWERTY argument. > >>> > >>> Ergonomists have universally concluded that (for the English > >>> language) the Dvorak Simplified Keyboard is more efficient than > >>> QWERTY. But because the whole world does QWERTY, changing to > >> Not me, or Alain. We use AZERTY! > >> > >> :) > > QUERTZ actually. :-) > > > > > AG : never heard about QUERTZ, but I confirm the use of AZERTY. AZERTY is French. QUERTZ is German. It is pretty similar to QUERTY, but it has the Y and the Z exchanged (many more Z than Y in German), it has umlauts, and as result of that many special characters are not where they are on a QUERTY. Thomas From agot at ulb.ac.be Fri Jan 14 14:23:28 2011 From: agot at ulb.ac.be (Alain Gottcheiner) Date: Fri, 14 Jan 2011 14:23:28 +0100 Subject: [BLML] Ecclesiastes [SEC=UNOFFICIAL] In-Reply-To: <1748200137.98748.1295002698502.JavaMail.ngmail@webmail08.arcor-online.net> References: <4D302790.8050401@skynet.be> <1748200137.98748.1295002698502.JavaMail.ngmail@webmail08.arcor-online.net> Message-ID: <4D304E50.7080502@ulb.ac.be> Hi all, This is a relation of an incident of last night, about which I need your advice : E S W N Kxx A10xxx Jxxx Q xx Jx Axx KQ10xxx K98x A10x Qxx Jxx AQ9x xxx 10xx KJx 1NT (1) p p 2C (2) p 2D (3) p 2H p p p (1) 12-14 (2) 6 hearts ; or 4 hearts + 5 any (3) pass / correct Lead : D8. EW play their own version of Rusinow. North asks to West whether the lead could come from 98x(x) : yes. From H98x ? No. Small diamond, to the Queen ; club to the Jack and Queen. Small diamond. Declarer puts up the Ace. 1 down. You're summoned at the table, for possible MI. This is easy to settle : EW have a fully-filled convention card. But West tells you that it isn't the right one, but the old one : he just forgot the new CC at home. He then puts it back in his pocket. I don't care about the problem of possibly changing the score ; it would have been marginal (North's line of play only caters for a doubleton lead, and then avoiding only the second undertrick), and wouldn't have affected the VP score anyway (no difference between +1 and -3 on the 24-board scale). But I'd like to know how you would react to West's antics ? Does the fact that there still is a bone to pick between those two pairs affecrt you in any way ? (North lives under the impression that, on their last match, East used UI from an alert, but found it impossible to prove) Thank you for your help. Best regards Alain From harald.skjaran at gmail.com Fri Jan 14 14:47:21 2011 From: harald.skjaran at gmail.com (=?UTF-8?Q?Harald_Skj=C3=A6ran?=) Date: Fri, 14 Jan 2011 14:47:21 +0100 Subject: [BLML] Ecclesiastes [SEC=UNOFFICIAL] In-Reply-To: <4D304E50.7080502@ulb.ac.be> References: <4D302790.8050401@skynet.be> <1748200137.98748.1295002698502.JavaMail.ngmail@webmail08.arcor-online.net> <4D304E50.7080502@ulb.ac.be> Message-ID: 2011/1/14 Alain Gottcheiner : > Hi all, > > This is a relation of an incident of last night, about which I need your > advice : > > > E ? ? ? ? ? ? ? ? ? ? ? ? ? ?S > W ? ? ? ? ? ? ? ? ? ? ? ? ?N > > Kxx ? ? ? ? ? ? ? ? ? ? ? A10xxx ? ? ? ? ? ? ? ? ? ?Jxxx > ? ? ? ? ? ? ? ? ? ? ?Q > xx ? ? ? ? ? ? ? ? ? ? ? ? Jx ? ? ? ? ? ? ? ? ? ? ? ? ? ? Axx > ? ? ? ? ? ? KQ10xxx > K98x ? ? ? ? ? ? ? ? ? ?A10x ? ? ? ? ? ? ? ? ? ? ? Qxx > ? ? Jxx > AQ9x ? ? ? ? ? ? ? ? ? xxx ? ? ? ? ? ? ? ? ? ? ? ? ? 10xx > ? ? ? ?KJx > > 1NT (1) ? ? ? ? ? ? ? ?p ? ? ? ? ? ? ? ? ? ? ? ? ? ? ?p > ? ? ? ? 2C (2) > p ? ? ? ? ? ? ? ? ? ? ? ? ?2D (3) > p ? ? ? ? ? ? ? ? ? ? ? ?2H > p ? ? ? ? ? ? ? ? ? ? ? ? ?p ? ? ? ? ? ? ? ? ? ? ? ? ? ? ? p > > (1) 12-14 > (2) 6 hearts ; or 4 hearts + 5 any > (3) pass / correct > > > Lead : D8. EW play their own version of Rusinow. > North asks to West whether the lead could come from 98x(x) : yes. > ?From H98x ? No. > > Small diamond, to the Queen ; club to the Jack and Queen. Small diamond. > Declarer puts up the Ace. 1 down. > > You're summoned at the table, for possible MI. This is easy to settle : > EW have a fully-filled convention card. But West tells you that it isn't > the right one, but the old one : he just forgot the new CC at home. He > then puts it back in his pocket. > > I don't care about the problem of possibly changing the score ; it would > have been marginal (North's line of play only caters for a doubleton > lead, and then avoiding only the second undertrick), and wouldn't have > affected the VP score anyway (no difference between +1 and -3 on the > 24-board scale). > > But I'd like to know how you would react to West's antics ? I'd demand to see the CC he put away. If he refused to let me have it, I'd rule MI and adjust the score to 2H making. And apply a severe PP on EW. > > > Does the fact that there still is a bone to pick between those two pairs > affecrt you in any way ? ?(North lives under the impression that, on > their last match, East used UI from an alert, but found it impossible to > prove) No, this has no impact on my ruling. > > > Thank you for your help. > > > Best regards > > ? ?Alain > _______________________________________________ > Blml mailing list > Blml at rtflb.org > http://lists.rtflb.org/mailman/listinfo/blml > -- Kind regards, Harald Skj?ran From ehaa at starpower.net Fri Jan 14 15:16:48 2011 From: ehaa at starpower.net (Eric Landau) Date: Fri, 14 Jan 2011 09:16:48 -0500 Subject: [BLML] Alain's case revisited In-Reply-To: <000901cbb34b$81150a70$833f1f50$@no> References: <4D2B0857.7050804@skynet.be> <4D2AC556.9070509@skynet.be> <4D2AC9E7.6060208@aol.com> <4D2AE13B.10605@skynet.be> <334994.97415.qm@web28503.mail.ukl.yahoo.com> <1391468851.32037.1294667187404.JavaMail.ngmail@webmail08.arcor-online.net> <4D2B1D0C.7090801@skynet.be> <4D2B2CC8.2060308@ulb.ac.be> <4D2E041A.8070508@nhcc.net> <000e01cbb2a9$ba3fc670$2ebf5350$@no> <707261.93703.qm@web28501.mail.ukl.yahoo.com> <001001cbb308$57dd6af0$079840d0$@no> <51527230-BFC9-4547-AA27-C974619AC3AB@starpower.net> <000901cbb34b$81150a70$833f1f50$@no> Message-ID: <79301020-6658-4713-B8A8-098AF1B57621@starpower.net> On Jan 13, 2011, at 12:58 PM, Sven Pran wrote: > On Behalf Of Eric Landau > >> On Jan 13, 2011, at 4:57 AM, Sven Pran wrote: >> >>>> On Behalf Of Nigel Guthrie >>>> >>>> In RAs that permit such variations, each side's choice of option >>>> after a call-out-of- turn or insufficient bid, can trigger >>>> different >>>> sets of understandings. This provides an additional channel of >>>> communication. Even in RAs that ostensibly forbid such ploys, some >>>> directors believe there are variations contingent on such choices >>>> that are a matter of Bridge-logic rather than agreement. >>> >>> This is not communication between partners any more than the fact >>> that >>> a different set of partnership understandings may apply depending on >>> opponents' disclosed agreements. >>> >>> Varying partnership understandings following an irregularity by >>> opponents can be necessary and/or a logical consequence of the >>> situation based on general bridge knowledge. >>> >>> An example from my own directing experience: 4NT was followed by 4D >>> (IB) >>> from LHO. Partner accepted the IB and responded to 4NT with a >>> logical >>> alternative call among also the five more calls now becoming >>> available. (The partnership had of course no prearranged >>> agreement for >>> such a situation, but they were strong players with sufficient >>> understanding of auction theory to in flight deploy the five >>> unexpected additionally available calls.) >> >> Isn't this obvious? What is the alternative? That you're allowed to >> accept 4D but not allowed to bid 4H, 4S, or 4NT over it? >> Ridiculous! That you're allowed to bid 4H, 4S or 4NT over the >> accepted 4D but >> partner isn't allowed to think about what you might have intended >> by doing so? >> Even ridiculouser! If we're going to abandon playing bridge >> because someone >> made an insufficient bid, why not just candel the board, award A-/A >> +, and be done >> with it? > > This ought to be obvious, but it isn't, Quote from Law 40B3: > The Regulating Authority may disallow prior agreement by a > partnership to > vary its understandings during the auction or play following a > question > asked, a response to a question, or any irregularity. > > When we worked on these laws back in 2007 I suggested (without > luck) the > following changes: > - a response given by own side to a question asked by opponents > - any irregularity committed by own side. > > Note that technically Law 40B3 can be used to prohibit a > partnership from > varying their agreement according to the nature of opponents' calls > revealed > through a question! It can indeed. "Whether explicit or implicit, an agreement between partners is a partnership understanding." [L40B1(b)] "In its discretion the RA may designate certain partnership understandings as 'special partnership understandings'." [L40B1(a)] "The RA is empowered without restriction to allow, disallow, or allow conditionally any special partnership understanding." [L40B2(a)] L40 is so broadly written that an RA that wished to could write regulations that would make it illegal to bid diamonds under any circumstances. Which wouldn't be a problem if our RAs were run entirely by saints and Vulcans. Eric Landau 1107 Dale Drive Silver Spring MD 20910 ehaa at starpower.net From blml at arcor.de Fri Jan 14 15:27:58 2011 From: blml at arcor.de (Thomas Dehn) Date: Fri, 14 Jan 2011 15:27:58 +0100 (CET) Subject: [BLML] Ecclesiastes [SEC=UNOFFICIAL] In-Reply-To: <4D304E50.7080502@ulb.ac.be> References: <4D304E50.7080502@ulb.ac.be> <4D302790.8050401@skynet.be> <1748200137.98748.1295002698502.JavaMail.ngmail@webmail08.arcor-online.net> Message-ID: <1530733423.113812.1295015278961.JavaMail.ngmail@webmail08.arcor-online.net> Alain Gottcheiner wrote: > You're summoned at the table, for possible MI. This is easy to settle : > EW have a fully-filled convention card. But West tells you that it isn't > the right one, but the old one : he just forgot the new CC at home. He > then puts it back in his pocket. > > I don't care about the problem of possibly changing the score ; it would > have been marginal (North's line of play only caters for a doubleton > lead, and then avoiding only the second undertrick), and wouldn't have > affected the VP score anyway (no difference between +1 and -3 on the > 24-board scale). > > But I'd like to know how you would react to West's antics ? I will check whether a) the difference between the old CC and their actual agreements is significant and b) if yes, whether E/W pre-alerted that the CC they put on the table does not match their actual agreements. If yes, I assign a PP. The size of the PP will depend on the number of rounds in which they have misinformed opponents, and on how different their CC is to their actual agreements. If, say, their card showed ACOL with a 12-14 NT while they actually play precision with a 15-17 NT, and they silently put that old card on the table, the PP will be such that next time they'll know better. This will also depend on the local requirements for the CC. Thomas From ehaa at starpower.net Fri Jan 14 16:20:38 2011 From: ehaa at starpower.net (Eric Landau) Date: Fri, 14 Jan 2011 10:20:38 -0500 Subject: [BLML] Ecclesiastes In-Reply-To: <4D304E50.7080502@ulb.ac.be> References: <4D302790.8050401@skynet.be> <1748200137.98748.1295002698502.JavaMail.ngmail@webmail08.arcor-online.net> <4D304E50.7080502@ulb.ac.be> Message-ID: <38C47DA0-80AF-4FEA-80A7-DEC66047C854@starpower.net> On Jan 14, 2011, at 8:23 AM, Alain Gottcheiner wrote: > This is a relation of an incident of last night, about which I need > your > advice : > > > E S > W N > > Kxx A10xxx Jxxx > Q > xx Jx Axx > KQ10xxx > K98x A10x Qxx > Jxx > AQ9x xxx 10xx > KJx > > 1NT (1) p p > 2C (2) > p 2D (3) > p 2H > p p p > > (1) 12-14 > (2) 6 hearts ; or 4 hearts + 5 any > (3) pass / correct > > Lead : D8. EW play their own version of Rusinow. > North asks to West whether the lead could come from 98x(x) : yes. > From H98x ? No. > > Small diamond, to the Queen ; club to the Jack and Queen. Small > diamond. > Declarer puts up the Ace. 1 down. > > You're summoned at the table, for possible MI. This is easy to > settle : > EW have a fully-filled convention card. But West tells you that it > isn't > the right one, but the old one : he just forgot the new CC at home. He > then puts it back in his pocket. > > I don't care about the problem of possibly changing the score ; it > would > have been marginal (North's line of play only caters for a doubleton > lead, and then avoiding only the second undertrick), and wouldn't have > affected the VP score anyway (no difference between +1 and -3 on the > 24-board scale). > > But I'd like to know how you would react to West's antics ? > > Does the fact that there still is a bone to pick between those two > pairs > affecrt you in any way ? (North lives under the impression that, on > their last match, East used UI from an alert, but found it > impossible to > prove) West, it seems, has admitted to deliberately and knowingly playing one set of agreements while offering his opponents a convention card describing a different set of agreements. Put that way, one cannot avoid the C-word. Even if I'm convinced that he simply failed to understand the severity of the offense he admitted to, I don't see how I can do less than disqualify and eject him from the game. In exceptional circumstances, if he takes it in good grace and is appropriately contrite and educable, I might spare him from facing a C&E committee if he wants to return. Eric Landau 1107 Dale Drive Silver Spring MD 20910 ehaa at starpower.net From blml at arcor.de Fri Jan 14 16:46:30 2011 From: blml at arcor.de (Thomas Dehn) Date: Fri, 14 Jan 2011 16:46:30 +0100 (CET) Subject: [BLML] Ecclesiastes In-Reply-To: <38C47DA0-80AF-4FEA-80A7-DEC66047C854@starpower.net> References: <38C47DA0-80AF-4FEA-80A7-DEC66047C854@starpower.net> <4D302790.8050401@skynet.be> <1748200137.98748.1295002698502.JavaMail.ngmail@webmail08.arcor-online.net> <4D304E50.7080502@ulb.ac.be> Message-ID: <30086410.17079.1295019990120.JavaMail.ngmail@webmail16.arcor-online.net> Eric Landau wrote: > On Jan 14, 2011, at 8:23 AM, Alain Gottcheiner wrote: > > > This is a relation of an incident of last night, about which I need > > your > > advice : > > > > > > E S > > W N > > > > Kxx A10xxx Jxxx > > Q > > xx Jx Axx > > KQ10xxx > > K98x A10x Qxx > > Jxx > > AQ9x xxx 10xx > > KJx > > > > 1NT (1) p p > > 2C (2) > > p 2D (3) > > p 2H > > p p p > > > > (1) 12-14 > > (2) 6 hearts ; or 4 hearts + 5 any > > (3) pass / correct > > > > Lead : D8. EW play their own version of Rusinow. > > North asks to West whether the lead could come from 98x(x) : yes. > > From H98x ? No. > > > > Small diamond, to the Queen ; club to the Jack and Queen. Small > > diamond. > > Declarer puts up the Ace. 1 down. > > > > You're summoned at the table, for possible MI. This is easy to > > settle : > > EW have a fully-filled convention card. But West tells you that it > > isn't > > the right one, but the old one : he just forgot the new CC at home. He > > then puts it back in his pocket. > > > > I don't care about the problem of possibly changing the score ; it > > would > > have been marginal (North's line of play only caters for a doubleton > > lead, and then avoiding only the second undertrick), and wouldn't have > > affected the VP score anyway (no difference between +1 and -3 on the > > 24-board scale). > > > > But I'd like to know how you would react to West's antics ? > > > > Does the fact that there still is a bone to pick between those two > > pairs > > affecrt you in any way ? (North lives under the impression that, on > > their last match, East used UI from an alert, but found it > > impossible to > > prove) > > West, it seems, has admitted to deliberately and knowingly playing > one set of agreements while offering his opponents a convention card > describing a different set of agreements. Put that way, one cannot > avoid the C-word. Even if I'm convinced that he simply failed to > understand the severity of the offense he admitted to, I don't see > how I can do less than disqualify and eject him from the game. In > exceptional circumstances, if he takes it in good grace and is > appropriately contrite and educable, I might spare him from facing a > C&E committee if he wants to return. I think you are using the C-word too fast. The TD first needs to investigate a bit more. It does happen that players forget their CC at home, or that they lose their CC during the tournament. Then they have two options: o fill in a new one, which will result in an incomplete CC during the next few rounds (think about how long it would take you to correctly and completely fill in the WBF convention card). This also requires that empty CCs are available, which is not always the case. o use an older one they happen to have with them, and update that to the extent possible. Depending on how complex the system is and how far off from their current agreements the older card is, using the older card and trying to update that might be more feasible than trying to fill in a new card. So I don't see a problem with them using an older card, that might be the best approach. I do see a problem, though, if they simply put some outdated CC on the table, without even trying to update it, and without even information opponents that the CC is not accurate. Thomas From jean-pierre.rocafort at meteo.fr Fri Jan 14 16:55:30 2011 From: jean-pierre.rocafort at meteo.fr (Jean-Pierre Rocafort) Date: Fri, 14 Jan 2011 16:55:30 +0100 Subject: [BLML] Ecclesiastes In-Reply-To: <30086410.17079.1295019990120.JavaMail.ngmail@webmail16.arcor-online.net> References: <38C47DA0-80AF-4FEA-80A7-DEC66047C854@starpower.net><4D302790.80 50401@skynet.be> <1748200137.98748.1295002698502.JavaMail.ngmail@webmail08.arco r-online.net><4D304E50.7080502@ulb.ac.be> <30086410.17079.1295019990120.JavaMail.ngmail@webmail16.arcor-online.net> Message-ID: <4D3071F2.9030706@meteo.fr> Thomas Dehn a ?crit : > Eric Landau wrote: >> On Jan 14, 2011, at 8:23 AM, Alain Gottcheiner wrote: >> >>> This is a relation of an incident of last night, about which I need >>> your >>> advice : ok, i understand: all the cards fell on the floor and we are required to sort them. >>> >>> >>> E S >>> W N >>> >>> Kxx A10xxx Jxxx >>> Q >>> xx Jx Axx >>> KQ10xxx >>> K98x A10x Qxx >>> Jxx >>> AQ9x xxx 10xx >>> KJx >>> >>> 1NT (1) p p >>> 2C (2) >>> p 2D (3) >>> p 2H >>> p p p >>> >>> (1) 12-14 >>> (2) 6 hearts ; or 4 hearts + 5 any >>> (3) pass / correct >>> >>> Lead : D8. EW play their own version of Rusinow. >>> North asks to West whether the lead could come from 98x(x) : yes. >>> From H98x ? No. >>> >>> Small diamond, to the Queen ; club to the Jack and Queen. Small >>> diamond. >>> Declarer puts up the Ace. 1 down. >>> >>> You're summoned at the table, for possible MI. This is easy to >>> settle : >>> EW have a fully-filled convention card. But West tells you that it >>> isn't >>> the right one, but the old one : he just forgot the new CC at home. He >>> then puts it back in his pocket. >>> >>> I don't care about the problem of possibly changing the score ; it >>> would >>> have been marginal (North's line of play only caters for a doubleton >>> lead, and then avoiding only the second undertrick), and wouldn't have >>> affected the VP score anyway (no difference between +1 and -3 on the >>> 24-board scale). >>> >>> But I'd like to know how you would react to West's antics ? >>> >>> Does the fact that there still is a bone to pick between those two >>> pairs >>> affecrt you in any way ? (North lives under the impression that, on >>> their last match, East used UI from an alert, but found it >>> impossible to >>> prove) >> West, it seems, has admitted to deliberately and knowingly playing >> one set of agreements while offering his opponents a convention card >> describing a different set of agreements. Put that way, one cannot >> avoid the C-word. Even if I'm convinced that he simply failed to >> understand the severity of the offense he admitted to, I don't see >> how I can do less than disqualify and eject him from the game. In >> exceptional circumstances, if he takes it in good grace and is >> appropriately contrite and educable, I might spare him from facing a >> C&E committee if he wants to return. > > I think you are using the C-word too fast. The TD first > needs to investigate a bit more. > > It does happen that players forget their CC at home, or that they lose their > CC during the tournament. > Then they have two options: > o fill in a new one, which will result in an incomplete CC during the next few rounds > (think about how long it would take you to correctly and completely fill in the WBF convention card). > This also requires that empty CCs are available, which is not always the case. > o use an older one they happen to have with them, and update that to the extent possible. > > Depending on how complex the system is and how far off from their > current agreements the older card is, ... and how far they are from home, > using the older card and trying > to update that might be more feasible than trying to fill in a new card. > So I don't see a problem with them using an older card, that might > be the best approach. > > I do see a problem, though, if they simply put some outdated CC on the table, > without even trying to update it, and without even information opponents > that the CC is not accurate. > > > Thomas -- _______________________________________________ Jean-Pierre Rocafort METEO-FRANCE DSI/CM 42 Avenue Gaspard Coriolis 31057 Toulouse CEDEX Tph: 05 61 07 81 02 (33 5 61 07 81 02) Fax: 05 61 07 81 09 (33 5 61 07 81 09) e-mail: jean-pierre.rocafort at meteo.fr Serveur WWW METEO-France: http://www.meteo.fr _______________________________________________ From agot at ulb.ac.be Fri Jan 14 17:37:58 2011 From: agot at ulb.ac.be (Alain Gottcheiner) Date: Fri, 14 Jan 2011 17:37:58 +0100 Subject: [BLML] Alain's case revisited In-Reply-To: <79301020-6658-4713-B8A8-098AF1B57621@starpower.net> References: <4D2B0857.7050804@skynet.be> <4D2AC556.9070509@skynet.be> <4D2AC9E7.6060208@aol.com> <4D2AE13B.10605@skynet.be> <334994.97415.qm@web28503.mail.ukl.yahoo.com> <1391468851.32037.1294667187404.JavaMail.ngmail@webmail08.arcor-online.net> <4D2B1D0C.7090801@skynet.be> <4D2B2CC8.2060308@ulb.ac.be> <4D2E041A.8070508@nhcc.net> <000e01cbb2a9$ba3fc670$2ebf5350$@no> <707261.93703.qm@web28501.mail.ukl.yahoo.com> <001001cbb308$57dd6af0$079840d0$@no> <51527230-BFC9-4547-AA27-C974619AC3AB@starpower.net> <000901cbb34b$81150a70$833f1f50$@no> <79301020-6658-4713-B8A8-098AF1B57621@starpower.net> Message-ID: <4D307BE6.7060509@ulb.ac.be> Le 14/01/2011 15:16, Eric Landau a ?crit : > On Jan 13, 2011, at 12:58 PM, Sven Pran wrote: > >> On Behalf Of Eric Landau >> >>> On Jan 13, 2011, at 4:57 AM, Sven Pran wrote: >>> >>>>> On Behalf Of Nigel Guthrie >>>>> >>>>> In RAs that permit such variations, each side's choice of option >>>>> after a call-out-of- turn or insufficient bid, can trigger >>>>> different >>>>> sets of understandings. This provides an additional channel of >>>>> communication. Even in RAs that ostensibly forbid such ploys, some >>>>> directors believe there are variations contingent on such choices >>>>> that are a matter of Bridge-logic rather than agreement. >>>> This is not communication between partners any more than the fact >>>> that >>>> a different set of partnership understandings may apply depending on >>>> opponents' disclosed agreements. >>>> >>>> Varying partnership understandings following an irregularity by >>>> opponents can be necessary and/or a logical consequence of the >>>> situation based on general bridge knowledge. >>>> >>>> An example from my own directing experience: 4NT was followed by 4D >>>> (IB) >>>> from LHO. Partner accepted the IB and responded to 4NT with a >>>> logical >>>> alternative call among also the five more calls now becoming >>>> available. (The partnership had of course no prearranged >>>> agreement for >>>> such a situation, but they were strong players with sufficient >>>> understanding of auction theory to in flight deploy the five >>>> unexpected additionally available calls.) >>> Isn't this obvious? What is the alternative? That you're allowed to >>> accept 4D but not allowed to bid 4H, 4S, or 4NT over it? >>> Ridiculous! That you're allowed to bid 4H, 4S or 4NT over the >>> accepted 4D but >>> partner isn't allowed to think about what you might have intended >>> by doing so? >>> Even ridiculouser! If we're going to abandon playing bridge >>> because someone >>> made an insufficient bid, why not just candel the board, award A-/A >>> +, and be done >>> with it? >> This ought to be obvious, but it isn't, Quote from Law 40B3: >> The Regulating Authority may disallow prior agreement by a >> partnership to >> vary its understandings during the auction or play following a >> question >> asked, a response to a question, or any irregularity. >> >> When we worked on these laws back in 2007 I suggested (without >> luck) the >> following changes: >> - a response given by own side to a question asked by opponents >> - any irregularity committed by own side. >> >> Note that technically Law 40B3 can be used to prohibit a >> partnership from >> varying their agreement according to the nature of opponents' calls >> revealed >> through a question! > It can indeed. > > "Whether explicit or implicit, an agreement between partners is a > partnership understanding." [L40B1(b)] > > "In its discretion the RA may designate certain partnership > understandings as 'special partnership understandings'." [L40B1(a)] > > "The RA is empowered without restriction to allow, disallow, or allow > conditionally any special partnership understanding." [L40B2(a)] > > L40 is so broadly written that an RA that wished to could write > regulations that would make it illegal to bid diamonds under any > circumstances. AG : that's the problem indeed. The intent (wrong IMOBO) was to allow RAs to disallow conventions that might prove unpleasant to participants. The effect is to allow RAs to disallow conventions that are unpleasant to them. RAs have been very inconsistent in allowing or disallowing such or such call, but since they have every right to be, nothing can stop them. Some examples from my home country : Opening 3S as a transfer to 3NT is allowed in most competition, while opening 3D as such is not, notwithstanding that it is less destructive and used for a longer time. Opening 2C as a nondescript preempt on 0-5 HCP is allowed in long matches ("brown sticker"), while opening 1D isn't ("yellow"). Same comments as above. Opening 1C, either natural or a strong bid, is allowed, while responding 1S to 1H, either natural or a strong bid, is only allowed at high levels. etc. Don't ask what the logic is, because it isn't written that those rulings need to be logical. At least ACs decisions have to be (that's called jurisprudency). Best regards Alain From adam at irvine.com Fri Jan 14 17:52:10 2011 From: adam at irvine.com (Adam Beneschan) Date: Fri, 14 Jan 2011 08:52:10 -0800 Subject: [BLML] He and she, whe and ble In-Reply-To: Your message of "Fri, 14 Jan 2011 03:42:25 +0100." <1923802510.31506.1294972945375.JavaMail.ngmail@webmail13.arcor-online.net> Message-ID: <20110114165211.B77DEA8C815@mailhub.irvine.com> Thomas wrote: > Consider this line: > "During the auction before the final pass, any player may > request, but only at her own turn to call, honest*** explanations > of the opponents' prior auction." ..... > Now, here the gender neutral text: > > "During the auction before the final pass, players may > request, but only at their own turn to call, honest*** explanations > of the opponents' prior auction." > > No such problem there, this language usage is not offensive to anybody. I'm not sure what the context of this discussion is. But while this isn't offensive, I am not comfortable with changing singular nouns and pronouns to plural in a setting where precision is extremely important (such as legal text, including the laws of the game). I can't say for sure why, but doing so seems to have the potential to subtly change the meaning, or to make it fuzzier. Perhaps Eric or another lawyerly type could explain this. I wouldn't have any problem with this sort of language in a more informal context. -- Adam From blml at arcor.de Fri Jan 14 18:17:10 2011 From: blml at arcor.de (Thomas Dehn) Date: Fri, 14 Jan 2011 18:17:10 +0100 (CET) Subject: [BLML] He and she, whe and ble In-Reply-To: <20110114165211.B77DEA8C815@mailhub.irvine.com> References: <20110114165211.B77DEA8C815@mailhub.irvine.com> Message-ID: <1870849530.19167.1295025430440.JavaMail.ngmail@webmail12.arcor-online.net> Adam Beneschan wrote: > Thomas wrote: > > > Consider this line: > > "During the auction before the final pass, any player may > > request, but only at her own turn to call, honest*** explanations > > of the opponents' prior auction." > > ..... > > > Now, here the gender neutral text: > > > > "During the auction before the final pass, players may > > request, but only at their own turn to call, honest*** explanations > > of the opponents' prior auction." > > > > No such problem there, this language usage is not offensive to anybody. > > I'm not sure what the context of this discussion is. But while this > isn't offensive, I am not comfortable with changing singular nouns and > pronouns to plural in a setting where precision is extremely important > (such as legal text, including the laws of the game). I can't say for > sure why, but doing so seems to have the potential to subtly change > the meaning, or to make it fuzzier. Yes, that problem exists. Thomas From daisy_duck at btopenworld.com Fri Jan 14 18:25:55 2011 From: daisy_duck at btopenworld.com (Stefanie Rohan) Date: Fri, 14 Jan 2011 17:25:55 +0000 (GMT) Subject: [BLML] He and she, whe and ble [SEC=UNOFFICIAL] In-Reply-To: <1923802510.31506.1294972945375.JavaMail.ngmail@webmail13.arcor-online.net> References: <1923802510.31506.1294972945375.JavaMail.ngmail@webmail13.arcor-online.net> Message-ID: <284989.38459.qm@web87112.mail.ird.yahoo.com> > You are forcing translators to deviate from > the English language version of TFLB. That > is undesirable. > > This type of language usage offends some people. > It will offend many people in some cultures. > From daisy_duck at btopenworld.com Fri Jan 14 18:34:59 2011 From: daisy_duck at btopenworld.com (Stefanie Rohan) Date: Fri, 14 Jan 2011 17:34:59 +0000 (GMT) Subject: [BLML] He and she, whe and ble [SEC=UNOFFICIAL] In-Reply-To: <1923802510.31506.1294972945375.JavaMail.ngmail@webmail13.arcor-online.net> References: <1923802510.31506.1294972945375.JavaMail.ngmail@webmail13.arcor-online.net> Message-ID: <973223.53871.qm@web87103.mail.ird.yahoo.com> From: Thomas Dehn > You are forcing translators to deviate from > the English language version of TFLB. That > is undesirable. > > This type of language usage offends some people. > It will offend many people in some cultures. It is interesting that you consider "feminine" language potentially "offensive". Perhaps this means that you have some insight into how all women in "masculine-language" "cultures" feel all the time. When, as a small child, I was told that one must say, for example, "everyone must hand in his homework on time", I nearly cried. 30-odd years later, I still feel excluded when "man" or "him" is used "inclusively". It is high time English adopted gender-neutral language. Or perhaps it could be recognised that the men have had their turn, and it is now time to switch to the inclusive feminine. It is more appropriate, really, as "woman" contains "man" and "she" contains "he". The words are inclusive already, so it makes sense to follow their lead. From blml at arcor.de Fri Jan 14 18:42:30 2011 From: blml at arcor.de (Thomas Dehn) Date: Fri, 14 Jan 2011 18:42:30 +0100 (CET) Subject: [BLML] He and she, whe and ble [SEC=UNOFFICIAL] In-Reply-To: <973223.53871.qm@web87103.mail.ird.yahoo.com> References: <973223.53871.qm@web87103.mail.ird.yahoo.com> <1923802510.31506.1294972945375.JavaMail.ngmail@webmail13.arcor-online.net> Message-ID: <1003254343.19635.1295026950522.JavaMail.ngmail@webmail12.arcor-online.net> Stefanie Rohan > From: Thomas Dehn > > You are forcing translators to deviate from > > the English language version of TFLB. That > > is undesirable. > > > > This type of language usage offends some people. > > It will offend many people in some cultures. > > It is interesting that you consider "feminine" language potentially > "offensive". Both the masculine and the feminine version are offensive, just to different people. There exist the moderate feminist approach to replace all masculine language with gender neutral language, and the radical feminist approach to replace all masculine language with feminine language (and to state that we now need 5,000 years of oppression of men to counterbalance 5,000 years of oppression of women). The latter approach isn't very popular. Thomas From jfusselman at gmail.com Fri Jan 14 19:36:57 2011 From: jfusselman at gmail.com (Jerry Fusselman) Date: Fri, 14 Jan 2011 12:36:57 -0600 Subject: [BLML] Ecclesiastes [SEC=UNOFFICIAL] In-Reply-To: References: <505171.31412.qm@web28513.mail.ukl.yahoo.com> Message-ID: [Richard Hills] The traditional nature of Duplicate Bridge requires that all questions be honestly answered. [Jerry Fusselman] Does anyone really agree with this? It seems to me that there are several exceptions in our current laws. We all know these places, don't we? From jfusselman at gmail.com Fri Jan 14 20:33:54 2011 From: jfusselman at gmail.com (Jerry Fusselman) Date: Fri, 14 Jan 2011 13:33:54 -0600 Subject: [BLML] Fwd: Ecclesiastes In-Reply-To: References: Message-ID: [Michael Polanyi, Personal Knowledge, chapter 8.6] A truthful statement commits the speaker to a belief in what he has asserted: he embarks in it on an open sea of limitless implications. An untruthful statement withholds this belief, launching a leaking vessel for others to sink in. [Polanyi, op. cit., Chapter 10.1] I believe that in spite of the hazards involved, I am called upon to search for the truth and state my findings. [Jerry, a few days ago] Richard's style is often to intentionally misstate others' positions. [Richard, in response] Intentionally misstate, definitely not. [Jerry] Alright, I accept the implied challenge. Shall we have a look? [2011/1/11 richard willey's ad hominem deleted] [Harald Skjaran] Why on earth did you say such a thing? Herman is trying to argue for what he believes is best for bridge. It's a real possibility that he is right about that aspect. The way Richard H. responds, it's like fighting windmills. [Richard?Hills, in response]?Yes, I agree that?Herman De Wael can aptly be described as Don Quixote tilting at windmills [...] [Jerry Fusselman] Would anyone care to argue that Richard H's misstatement of Harald's point might possibly be unintentional? I don't particularly care whether it is conscious or unconscious, but I read this as intentional misstatement. [Richard Hills] My request for an apology from Jerry Fusselman to me, since otherwise Jerry's animadversion to "ad hominem" attacks would appear to be hypocritical. [Jerry Fusselman] I don't do ad hominem abuse. At least I don't think so. Maybe someone can show me a case. In just the last few days, Richard Hills has described certain BLMLers as prostitutes (hetaerae) and one BLMLer as an asexual (agamous) something-or-other. That's what we call ad hominem abuse. [Richard Hills] I will be deprived of the dulcet tones of Jerry Fusselman and Herman De Wael constructively criticising my character flaws. [Jerry Fusselman] I don't know and I don't care if Richard Hills has any character flaws. That was neither my interest nor my subject. My request of Richard is meant for the betterment of BLML, and I consider it easy for a man of his abilities to do it. When summarizing someones views, please really try for accuracy, for otherwise, as Polanyi wrote, you are "launching a leaking vessel for others to sink in." From JffEstrsn at aol.com Sat Jan 15 01:16:18 2011 From: JffEstrsn at aol.com (Jeff Easterson) Date: Sat, 15 Jan 2011 01:16:18 +0100 Subject: [BLML] Ecclesiastes [SEC=UNOFFICIAL] In-Reply-To: <1748200137.98748.1295002698502.JavaMail.ngmail@webmail08.arcor-online.net> References: <4D302790.8050401@skynet.be> <1748200137.98748.1295002698502.JavaMail.ngmail@webmail08.arcor-online.net> Message-ID: <4D30E752.4020809@aol.com> I have: QWERTZ. JE Am 14.01.2011 11:58, schrieb Thomas Dehn: > Herman De Wael wrote: >> richard.hills at immi.gov.au wrote: >>> Ecclesiates 7:20 >>> >>> There is no one on earth who does what is right all the time and >>> never makes a mistake. >>> >>> Herman De Wael: >>> >>>> ..... >>>> But if you do want to discuss whether green ought to be changed >>>> into blue, then the argument "the whole world does it" is not a >>>> valid argument. >>>> ..... >>> Richard Hills: >>> >>> No, "the whole world does it" is the very valid QWERTY argument. >>> >>> Ergonomists have universally concluded that (for the English >>> language) the Dvorak Simplified Keyboard is more efficient than >>> QWERTY. But because the whole world does QWERTY, changing to >> Not me, or Alain. We use AZERTY! >> >> :) > QUERTZ actually. :-) > > > Thomas > _______________________________________________ > Blml mailing list > Blml at rtflb.org > http://lists.rtflb.org/mailman/listinfo/blml > From JffEstrsn at aol.com Sat Jan 15 01:24:01 2011 From: JffEstrsn at aol.com (Jeff Easterson) Date: Sat, 15 Jan 2011 01:24:01 +0100 Subject: [BLML] Fwd: Ecclesiastes In-Reply-To: References: Message-ID: <4D30E921.9050600@aol.com> I have the impression (perhaps incorrect) that Jerry (and perhaps a few others) do not realise when Richard H. is being (or trying to be) humorous or facetious. I am unable to judge if the "problem" lies by Richard or Jerry. JE Am 14.01.2011 20:33, schrieb Jerry Fusselman: > [Michael Polanyi, Personal Knowledge, chapter 8.6] A truthful > statement commits the speaker to a belief in what he has asserted: he > embarks in it on an open sea of limitless implications. An untruthful > statement withholds this belief, launching a leaking vessel for others > to sink in. > > [Polanyi, op. cit., Chapter 10.1] I believe that in spite of the > hazards involved, I am called upon to search for the truth and state > my findings. > > [Jerry, a few days ago] Richard's style is often to intentionally > misstate others' positions. > > [Richard, in response] Intentionally misstate, definitely not. > > [Jerry] Alright, I accept the implied challenge. Shall we have a look? > > [2011/1/11 richard willey's ad hominem deleted] > > [Harald Skjaran] > Why on earth did you say such a thing? > Herman is trying to argue for what he believes is best for bridge. > It's a real possibility that he is right about that aspect. > The way Richard H. responds, it's like fighting windmills. > > [Richard Hills, in response] Yes, I agree that Herman De Wael can > aptly be described as Don > Quixote tilting at windmills [...] > > [Jerry Fusselman] Would anyone care to argue that Richard H's > misstatement of Harald's point might possibly be unintentional? I > don't particularly care whether it is conscious or unconscious, but I > read this as intentional misstatement. > > [Richard Hills] > My request for an apology from Jerry Fusselman to me, > since otherwise Jerry's animadversion to "ad hominem" > attacks would appear to be hypocritical. > > [Jerry Fusselman] I don't do ad hominem abuse. At least I don't think > so. Maybe someone can show me a case. > > In just the last few days, Richard Hills has described certain BLMLers > as prostitutes (hetaerae) and one BLMLer as an asexual (agamous) > something-or-other. That's what we call ad hominem abuse. > > [Richard Hills] I will be deprived of > the dulcet tones of Jerry Fusselman and Herman De Wael > constructively criticising my character flaws. > > [Jerry Fusselman] I don't know and I don't care if Richard Hills has > any character flaws. That was neither my interest nor my subject. My > request of Richard is meant for the betterment of BLML, and I consider > it easy for a man of his abilities to do it. When summarizing > someones views, please really try for accuracy, for otherwise, as > Polanyi wrote, you are "launching a leaking vessel for others to sink > in." > _______________________________________________ > Blml mailing list > Blml at rtflb.org > http://lists.rtflb.org/mailman/listinfo/blml > From jfusselman at gmail.com Sat Jan 15 03:17:07 2011 From: jfusselman at gmail.com (Jerry Fusselman) Date: Fri, 14 Jan 2011 20:17:07 -0600 Subject: [BLML] Fwd: Ecclesiastes In-Reply-To: <4D30E921.9050600@aol.com> References: <4D30E921.9050600@aol.com> Message-ID: On Fri, Jan 14, 2011 at 6:24 PM, Jeff Easterson wrote: > I have the impression (perhaps incorrect) that Jerry (and perhaps a few > others) do not realise when Richard H. is being (or trying to be) > humorous or facetious. Yes, of course, I thought so too, as I wrote originally. But Richard H. denied it. Jerry Fusselman From grandaeval at tiscali.co.uk Sat Jan 15 05:50:29 2011 From: grandaeval at tiscali.co.uk (Grattan) Date: Sat, 15 Jan 2011 04:50:29 -0000 Subject: [BLML] He and she, whe and ble References: <20110114165211.B77DEA8C815@mailhub.irvine.com> <1870849530.19167.1295025430440.JavaMail.ngmail@webmail12.arcor-online.net> Message-ID: Grattan Endicott To: Sent: Friday, January 14, 2011 5:17 PM Subject: Re: [BLML] He and she, whe and ble >> Now, here the gender neutral text: > "During the auction before the final pass, players may request, but only at their own turn to call, honest*** explanations of the opponents' prior auction." > ................................................................ +=+ " During the auction before the final pass, a player may request an explanation of the opponents' prior auction, but only at that player's turn to call. " +=+ Substituting 'her" for "his" merely inverts the nature of the sexism. And it is a prissy nicety when there is established custom and practice that is accepted in law. Drafting committees incline to wariness of cosmetic change. Personally I like well-chosen words carefully composed to convey the intention accurately. I look at sentences when people say "if it ain't broke don't mend it" and think "but the language is inferior, broke." ~ Grattan ~ +=+ From blml at arcor.de Sat Jan 15 06:11:25 2011 From: blml at arcor.de (Thomas Dehn) Date: Sat, 15 Jan 2011 06:11:25 +0100 (CET) Subject: [BLML] He and she, whe and ble In-Reply-To: References: <20110114165211.B77DEA8C815@mailhub.irvine.com> <1870849530.19167.1295025430440.JavaMail.ngmail@webmail12.arcor-online.net> Message-ID: <53760247.655.1295068285279.JavaMail.ngmail@webmail19.arcor-online.net> Grattan wrote: > ----- Original Message ----- > From: "Thomas Dehn" > To: > Sent: Friday, January 14, 2011 5:17 PM > Subject: Re: [BLML] He and she, whe and ble > > > >> > Now, here the gender neutral text: > > > "During the auction before the final pass, players may > request, but only at their own turn to call, honest*** > explanations of the opponents' prior auction." > > ................................................................ > +=+ " During the auction before the final pass, a player > may request an explanation of the opponents' prior > auction, but only at that player's turn to call. " +=+ Yes, that is gender neutral in English. It doesn't stay gender neutral when translated into German or French etc. Not easy, not easy at all. Thomas From bmeadows666 at gmail.com Sat Jan 15 10:07:18 2011 From: bmeadows666 at gmail.com (Brian) Date: Sat, 15 Jan 2011 04:07:18 -0500 Subject: [BLML] He and she, whe and ble In-Reply-To: <53760247.655.1295068285279.JavaMail.ngmail@webmail19.arcor-online.net> References: <20110114165211.B77DEA8C815@mailhub.irvine.com> <1870849530.19167.1295025430440.JavaMail.ngmail@webmail12.arcor-online.net> <53760247.655.1295068285279.JavaMail.ngmail@webmail19.arcor-online.net> Message-ID: <4D3163C6.80104@gmail.com> On 01/15/2011 12:11 AM, Thomas Dehn wrote: > Grattan wrote: >> ----- Original Message ----- >> From: "Thomas Dehn" >> To: >> Sent: Friday, January 14, 2011 5:17 PM >> Subject: Re: [BLML] He and she, whe and ble >> >> >>>> >> Now, here the gender neutral text: >>> >> "During the auction before the final pass, players may >> request, but only at their own turn to call, honest*** >> explanations of the opponents' prior auction." >>> ................................................................ >> +=+ " During the auction before the final pass, a player >> may request an explanation of the opponents' prior >> auction, but only at that player's turn to call. " +=+ > > Yes, that is gender neutral in English. > > It doesn't stay gender neutral when translated into German or French etc. > Not easy, not easy at all. > > Refer to players as "it". That way you can offend everybody, and then at least you know where you stand. Brian. From svenpran at online.no Sat Jan 15 11:18:04 2011 From: svenpran at online.no (Sven Pran) Date: Sat, 15 Jan 2011 11:18:04 +0100 Subject: [BLML] He and she, whe and ble In-Reply-To: <4D3163C6.80104@gmail.com> References: <20110114165211.B77DEA8C815@mailhub.irvine.com> <1870849530.19167.1295025430440.JavaMail.ngmail@webmail12.arcor-online.net> <53760247.655.1295068285279.JavaMail.ngmail@webmail19.arcor-online.net> <4D3163C6.80104@gmail.com> Message-ID: <000901cbb49d$7ffdb670$7ff92350$@no> On Behalf Of Brian > >> Now, here the gender neutral text: > >>> > >> "During the auction before the final pass, players may request, but > >> only at their own turn to call, honest*** explanations of the > >> opponents' prior auction." > >>> ................................................................ > >> +=+ " During the auction before the final pass, a player > >> may request an explanation of the opponents' prior auction, but only > >> at that player's turn to call. " +=+ > > > > Yes, that is gender neutral in English. > > > > It doesn't stay gender neutral when translated into German or French etc. > > Not easy, not easy at all. > > > > > > Refer to players as "it". That way you can offend everybody, and then at least you > know where you stand. As far as I know "it" is an improper reference to player or players in the English language; and "it" can simply not be translated to acceptable Norwegian in this context. From t.kooyman at worldonline.nl Sat Jan 15 15:14:47 2011 From: t.kooyman at worldonline.nl (ton) Date: Sat, 15 Jan 2011 15:14:47 +0100 Subject: [BLML] He and she, whe and ble In-Reply-To: <20110114165211.B77DEA8C815@mailhub.irvine.com> References: Your message of "Fri, 14 Jan 2011 03:42:25 +0100." <1923802510.31506.1294972945375.JavaMail.ngmail@webmail13.arcor-online.net> <20110114165211.B77DEA8C815@mailhub.irvine.com> Message-ID: <002b01cbb4be$90c6e190$b254a4b0$@kooyman@worldonline.nl> ton: My knowledge of the English language is such that I am happy to write messages which give at least an impression about my opinion. My language of the Dutch language is such that I dare to say that had our laws been written in that language the word 'player' can be considered to be gender neutral and 'his' is the right way to refer to player. If the English language has a similar clever approach to this issue the issue has disappeared. Let us hope so. Van: blml-bounces at rtflb.org [mailto:blml-bounces at rtflb.org] Namens Adam Beneschan Verzonden: vrijdag 14 januari 2011 17:52 Aan: Bridge Laws Mailing List CC: adam Onderwerp: Re: [BLML] He and she, whe and ble Thomas wrote: > Consider this line: > "During the auction before the final pass, any player may > request, but only at her own turn to call, honest*** explanations > of the opponents' prior auction." ..... > Now, here the gender neutral text: > > "During the auction before the final pass, players may > request, but only at their own turn to call, honest*** explanations > of the opponents' prior auction." > > No such problem there, this language usage is not offensive to anybody. I'm not sure what the context of this discussion is. But while this isn't offensive, I am not comfortable with changing singular nouns and pronouns to plural in a setting where precision is extremely important (such as legal text, including the laws of the game). I can't say for sure why, but doing so seems to have the potential to subtly change the meaning, or to make it fuzzier. Perhaps Eric or another lawyerly type could explain this. I wouldn't have any problem with this sort of language in a more informal context. -- Adam _______________________________________________ Blml mailing list Blml at rtflb.org http://lists.rtflb.org/mailman/listinfo/blml _____ Geen virus gevonden in dit bericht. Gecontroleerd door AVG - www.avg.com Versie: 10.0.1191 / Virusdatabase: 1435/3380 - datum van uitgifte: 01/14/11 -------------- next part -------------- An HTML attachment was scrubbed... URL: http://lists.rtflb.org/pipermail/blml/attachments/20110115/19b4caba/attachment.html From grandaeval at tiscali.co.uk Sun Jan 16 04:38:17 2011 From: grandaeval at tiscali.co.uk (Grattan) Date: Sun, 16 Jan 2011 03:38:17 -0000 Subject: [BLML] He and she, whe and ble References: <20110114165211.B77DEA8C815@mailhub.irvine.com><1870849530.19167.1295025430440.JavaMail.ngmail@webmail12.arcor-online.net> <53760247.655.1295068285279.JavaMail.ngmail@webmail19.arcor-online.net> Message-ID: <20C5F9BA48BB4BE183370E99812C5606@Mildred> Grattan Endicott To: Sent: Saturday, January 15, 2011 5:11 AM Subject: Re: [BLML] He and she, whe and ble > Grattan wrote: >> ----- Original Message ----- >> From: "Thomas Dehn" >> To: >> Sent: Friday, January 14, 2011 5:17 PM >> Subject: Re: [BLML] He and she, whe and ble >> >> >> >> >> Now, here the gender neutral text: >> > >> "During the auction before the final pass, players may >> request, but only at their own turn to call, honest*** >> explanations of the opponents' prior auction." >> > ................................................................ >> +=+ " During the auction before the final pass, a player >> may request an explanation of the opponents' prior >> auction, but only at that player's turn to call. " +=+ > > Yes, that is gender neutral in English. > > It doesn't stay gender neutral when translated into > German or French etc. Not easy, not easy at all. > > Thomas > +=+ I can understand this. In 1943-44 one of my duties was to express in English for the benefit of the British Admiral. his Chief of Staff and his US Commodore deputy, the devious communications from the Italian and French navies, and compose replies in equally ambiguous terms. ~ Grattan ~ +=+ From grandaeval at tiscali.co.uk Sun Jan 16 04:52:31 2011 From: grandaeval at tiscali.co.uk (Grattan) Date: Sun, 16 Jan 2011 03:52:31 -0000 Subject: [BLML] At best an echo. [SEC = UNOFFICIAL] Message-ID: <4F05628CFC744D98A501853467114821@Mildred> Grattan Endicott It doesn't stay gender neutral when translated > into German or French etc. Not easy, not easy > at all." < +=+ The translator's exercise is to take whatever is written in the original English version and convert it idiomatically to the language of choice. Translations that are too literal are inadequate; one must rewrite the effect of the English text - but, of course, this requires the translator to grasp the nuances of the original language. "Translation is at best an echo." ~ Grattan ~ +=+ From jfusselman at gmail.com Sun Jan 16 06:37:08 2011 From: jfusselman at gmail.com (Jerry Fusselman) Date: Sat, 15 Jan 2011 23:37:08 -0600 Subject: [BLML] Alain's case revisited In-Reply-To: <4D307BE6.7060509@ulb.ac.be> References: <4D2B0857.7050804@skynet.be> <4D2AC556.9070509@skynet.be> <4D2AC9E7.6060208@aol.com> <4D2AE13B.10605@skynet.be> <334994.97415.qm@web28503.mail.ukl.yahoo.com> <1391468851.32037.1294667187404.JavaMail.ngmail@webmail08.arcor-online.net> <4D2B1D0C.7090801@skynet.be> <4D2B2CC8.2060308@ulb.ac.be> <4D2E041A.8070508@nhcc.net> <000e01cbb2a9$ba3fc670$2ebf5350$@no> <707261.93703.qm@web28501.mail.ukl.yahoo.com> <001001cbb308$57dd6af0$079840d0$@no> <51527230-BFC9-4547-AA27-C974619AC3AB@starpower.net> <000901cbb34b$81150a70$833f1f50$@no> <79301020-6658-4713-B8A8-098AF1B57621@starpower.net> <4D307BE6.7060509@ulb.ac.be> Message-ID: On Mon, Jan 10, 2011 at 12:58 AM, Richard Hills wrote: > Jerry Fusselman: > >>>This happens at two tables---one with a confirmed >>>cheater bidding 4S, and another with a confirmed >>>innocent newbie bidding 4S. Do you want the >>>rectifications in these two cases to be the same or >>>different? ?(I am not asking about penalty points here, >>>only the rectification.) > > Grattan Endicott: > >>+=+ Could I enquire whether the two players have entered >>under the same Conditions of Contest? >> ? ?If so, should one be advantaged by comparison with >>the other against the field? > ? ? ? ? ? ? ? ? ? ? ? ? ? ? ? ? ? ? ? ? ? ? ?~ G ~ ? +=+ > > Richard Hills (male chauvinist pig): > > I can understand why an innocent newbie would be playing > in the Venice Cup. ?:-) ?:-) > > But could I enquire why the confirmed cheater has also > been allowed to enter under the same Conditions of > Contest in this Venice Cup? > I never mentioned the Venice Cup. The terms "innocent newbie" and "confirmed cheater" in this thread originated in Eric's very interesting post, which I still promise to return to soon. I meant them exactly how Eric meant them. Let's see if I understand Eric correctly. An innocent newbie ... well, I guess this is probably clear. Everybody knows. A confirmed cheater is not someone who has been banned from playing bridge, obviously, because he is here, playing bridge. I assume that Eric's notion of a confirmed cheater is a player who the (wise) director is personally is sure is (or was) a cheater, with one of these conditions: 1. The director cannot prove it. 2. The proof has not yet made its way through the proper channels for sanction. 3. The player has been sanctioned, but not banished. 4. The director does not want to lose a paying customer, and so he therefore does not go through channels of sanctioning. 5. The cheater has paid his debt to bridge society, and is now back to playing after his sanction or period of forced absence. I don't think it matters which of these is the case, but if it does, please say so. I am going into this detail only because Grattan and Richard are curious about it. I am also unclear as to what rights the confirmed cheater loses under Eric's ideal director for having sinned in the past. Indeed, I cannot yet tell what value the information of the player being a confirmed cheat has to what a director should do (in Eric's eyes), but it is obvious that Eric thinks it should have some effect on the wise director's actions, for otherwise Eric would not have brought the subject up. Jerry Fusselman From jfusselman at gmail.com Sun Jan 16 06:37:51 2011 From: jfusselman at gmail.com (Jerry Fusselman) Date: Sat, 15 Jan 2011 23:37:51 -0600 Subject: [BLML] Alain's case revisited In-Reply-To: References: <4D257698.6010201@skynet.be> <9647244.332577.1294302281035.JavaMail.ngmail@webmail17.arcor-online.net> <0F0A4E0E-3319-4F02-8EC6-F49CA1492683@starpower.net> Message-ID: Eric, I appreciate your thoughtful responses. Was it really six days ago? Shame on me. Anyway, finally, I am responding. On Mon, Jan 10, 2011 at 3:56 PM, Eric Landau wrote: > On Jan 8, 2011, at 5:00 PM, Jerry Fusselman wrote: > >> I want to give a clarification and request a clarification. ?In brief, >> I am ignoring clubs and penalty points right now, and I want to know >> how Eric (or someone who agrees with Eric's post) would rule if his >> innocent newbie unconsciously used the same UI to take the same >> improper action as a conscious cheat. >> >> On Fri, Jan 7, 2011 at 4:11 PM, Eric Landau wrote: >> >>> What some people call "reading minds", others call "good >>> communication skills", and still others, bridge club directors >>> especially, call "knowing their customers". >> >> Here, and later, Eric refers to bridge club directors. ?I would rather >> leave clubs entirely out of the discussion: ?For my post, I was >> thinking only of events run in (using ACBL names) sectionals, >> regionals, and nationals. ?Almost anything can happen in clubs, and >> frequently does. ?Lets face it: ?Clubs run by their own rules. ?I want >> to hear if what I said is right in the domain of sectionals, >> regionals, and nationals, for I consider clubs a separate issue. >> >>> What Jerry calls "our best bridge laws" are precisely those that >>> require us to give identical rulings to those whom we know to be >>> innocent newbies as to confirmed cheaters. >> >> Eric, are you thinking that the requirement you mention here is a bad >> thing? ?For example, please consider the noncompetitive auction >> >> 1S-2S >> 3C-...3S >> 4S. >> >> I.e., there was a slow 3S signoff after the 3C game try. ?It gets >> raised to 4S anyway by a hand that some players would have taken >> straight to game (instead of 3C), but no one would tried for a slam. >> Please also assume that the slow signoff demonstrably shows extras. >> In the play, it easily makes ten tricks, as it should in just about >> all cases, and both sides play it well. >> >> This happens at two tables---one with a confirmed cheater bidding 4S, >> and another with a confirmed innocent newbie bidding 4S. Do you want >> the rectifications in these two cases to be the same or different? > > I would hope to see vast differences in the processes by which the > rectification is determined, notwithstanding that I would expect the > resulting rectifications to be the same in almost every case. > Fascinating. What is the nature of these vast differences in processes that Eric hopes for? I guess Eric covers this below. >> (I >> am not asking about penalty points here, only the rectification.) ?If >> it helps to reach a decision, please assume that the confirmed cheater >> consciously used the delay to upgrade his hand, but the confirmed >> innocent newbie's upgrade caused by the pause was entirely >> unconscious. ?And please assume also that the director knows all of >> this perfectly. ?How should he rule in the two cases if you had the >> laws arranged the way you think best? ?Does my choice, +170/-170 in >> both cases, match yours? > > Yes. ?But note that you have forced me to this position by > stipuation: "the confirmed innocent newbie's upgrade caused by the > pause was entirely unconscious." ?It is, after all, the "upgrade > caused by the pause" that is to be rectified, "entirely unconscious" > or not. > > But when it comes to the sort of understanding of the game of bridge > that Jerry and I and I'd assume all of us take for granted, things > are often not so clear; I have discovered in my half-century of > competitive bridge playing that "confirmed innocent newbies" think > and act in all manner of odd and amazing ways. ?So upon being > confronted with Jerry's scenario, I would use my not insignificant > (albeit far from perfect) "mind reading" skills by interrogating the > newbie as to why he bid 3C, and then 4S, and listening to their > answers, before accepting the conlusion that there was an "upgrade > cause by the pause" that happened to be, irrelevantly, as Jerry is > "not asking about penalty points here, only the rectification", > "entirely unconscious". ?Then, having done so, I give him +170 and a > gentle education on his responsibilities after his partner pauses at > a critical point in the auction. ?But it is not beyond the realm of > possibility, or even of my own experience, that this innocent newbie, > unlike the confirmed cheater, could wind up convincing me that he was > never ever stopping short of game no matter what, had his own, not > necessarily sensible (to Jerry or me), reason for his 3C bid, and > that, *for him*, not bidding game was not an LA. Does Eric mean *for him*, or for *his peers using his methods who bid 3C*? I thought that the latter was the proper issue, not the former. Anyway, I cannot easily think of what would convince Eric to allow 4S after a 3C game try in my hypothetical auction. Yes, I am assuming that it is a game try. Which of these explanations by the innocent newbie would Eric (or anyone else) say should lead a wise director to allow 4S? 1. When I bid 3C, I forgot my distribution points, so I changed my mind as to what my hand was worth even before LHO passed. 2. I was always planning to go to game, and I was just using 3C to try to confuse the opponents. We have not done this trick ever before, but this is a big event, so I was trying new stuff. 3. I meant to bid 3D, but I goofed, and since I was borderline anyway for a game try, I just went to game ignoring my partner's response. 4. My table feel told me that LHO was broke, RHO had the trapped honors, and the cards were likely to lie well for our side, so I went to game based on table feel. 5. Some other explanation that would talk a wise director into allowing 4S. Please assume that each explanation is honest and that the director somehow knows this. I am really curious as to what it would take for a wise director to allow 4S, given that 3C was a game try and 3S was a sign off. Probably one example will satisfy me. > >>> And because our clubs >>> (and our SOs) are businesses, that means being far too tolerant to >>> the cheaters rather than driving the innocent infractors away (and >>> having to listen to those in this forum who constantly carp about how >>> our laws let people get away with things they shouldn't). ?Only the >>> real-life lawyers among our players seem to actually understand that >>> subtle bit about being punished for doing just what a cheater would >>> do but, of course, we're not calling you cheaters, we'd never ever do >>> that, no sirree, so do please come back next week. >>> >>> If we had genuinely psychic directors who actually could read >>> players' minds, wouldn't we want to write our laws to take advantage >>> of those skills, and expect our game to be better for it? ?If so, >>> shouldn't we work to hone our communications skills and our knowledge >>> of our customers, write laws that take advantage of those (admittedly >>> somewhat less impressive, but not entirely useless) skills, and do >>> our best to come as close as we can to our ideal of perfection? >> >> I plan to respond to this later. ?I suspect that we have a conflict of >> visions about directors and players. > > I suspect more a conflict of interpretation of all of those "could > have"s and "might have"s scattered throughout TFLB. ?Do we take those > as determinations of conceivability (possibly leading to, "He didn't, > but he could have, so we rule against him") or as determinations of > finite probability (possibly leading to, "He could have, but he > didn't, so we rule in his favor")? ?Jerry seems to favor the former > interpretation, while I would prefer the latter, notwithstanding the > difficulty of operationalizing "he didn't" for us non-psychics. > Yes, Eric certainly has me pegged here. Indeed, I thought that the former was the only valid way to interpret the law book. My guess is that Grattan would not consider it bridge to do it the latter way. I am quite surprised with Eric's choice, given what he has written in the past about the wisdom of good, objective, predictable procedures. I am very interested to hear what others think, too. Jerry Fusselman From jfusselman at gmail.com Sun Jan 16 07:02:31 2011 From: jfusselman at gmail.com (Jerry Fusselman) Date: Sun, 16 Jan 2011 00:02:31 -0600 Subject: [BLML] EBU L&EC meeting 3rd November 2010 - L20 In-Reply-To: References: <1242552473.588.1294899908423.JavaMail.ngmail@webmail17.arcor-online.net> <0DB11DA8-A475-4963-9896-F51A427619C7@starpower.net> Message-ID: On Fri, Jan 14, 2011 at 1:45 AM, Jerry Fusselman wrote: > On Thu, Jan 13, 2011 at 9:42 AM, Eric Landau wrote: >> >> Thomas is right. ?The "Kaplan paradigm" may be defunct, but, >> hopefully, the much simpler ("ACBL") paradigm still lives: ?Your >> opponents are entitled to know as much about the meaning of partner's >> call as you do. ?If you bid 4NT, systemically requiring partner to >> pick a minor, and he bids 5H, you know that if he mistakenly thought >> he was responding to a keycard-ask he has some agreed holding in key >> cards, while all your opponents would be allowed to know under >> Richard's proposal is that 5H is meaningless per your agreements. >> You get to decide for yourself whether some "call[] that might be >> made in an entirely different auction" is relevant to understanding >> this one; why should your opponents be denied the same opportunity? >> > > Thomas and Eric are right. ?Why give the advantage to Secretary Birds > who would learn this trick over ethical players who freely give the > opponents their useful system information? > > And why set up rules that give players a disadvantage if their > convention card is complete? > > Jerry Fusselman > On further review, Thomas and Eric are even more right than I had thought. Do we really want to listen to a player berate his partner, "Why did you tell them that? It was too useful to them! Next time, only tell them information about our system that is required. If in doubt, ask the director first if it is required. Be jealous about divulging our system details." Do we want to penalize an ethical, generous impulse to answer questions about system when asked at the proper time? Do we want players to withhold convention cards when they deem that the information the opponent is looking for is not proper because it is not, strictly speaking, part the of the system that is strictly relevant to the current auction? If I volunteer systemic information that I am not required to, am I sandbagging and in violation of law 72A: "The chief object is to obtain a higher score than other contestants whilst complying with the lawful procedures and ethical standards set out in these laws." What about a player that only gives the extra information when he could have known or did know that it was likely to be in his interest? The ACBL paradigm sure seems superior to me. Jerry Fusselman From grabiner at alumni.princeton.edu Sun Jan 16 07:10:29 2011 From: grabiner at alumni.princeton.edu (David Grabiner) Date: Sun, 16 Jan 2011 01:10:29 -0500 Subject: [BLML] Alain's case revisited In-Reply-To: References: <4D257698.6010201@skynet.be> <9647244.332577.1294302281035.JavaMail.ngmail@webmail17.arcor-online.net> <0F0A4E0E-3319-4F02-8EC6-F49CA1492683@starpower.net> Message-ID: Jerry Fusselman writes: >> 1S-2S >> 3C-...3S >> 4S. >Anyway, I cannot easily think of what would convince Eric to allow 4S >after a 3C game try in my hypothetical auction. Yes, I am assuming >that it is a game try. Which of these explanations by the innocent >newbie would Eric (or anyone else) say should lead a wise director to >allow 4S? >1. When I bid 3C, I forgot my distribution points, so I changed my >mind as to what my hand was worth even before LHO passed. >2. I was always planning to go to game, and I was just using 3C to >try to confuse the opponents. We have not done this trick ever >before, but this is a big event, so I was trying new stuff. >3. I meant to bid 3D, but I goofed, and since I was borderline anyway >for a game try, I just went to game ignoring my partner's response. >4. My table feel told me that LHO was broke, RHO had the trapped >honors, and the cards were likely to lie well for our side, so I went >to game based on table feel. >5. Some other explanation that would talk a wise director into allowing 4S. >Please assume that each explanation is honest and that the director >somehow knows this. I am really curious as to what it would take for >a wise director to allow 4S, given that 3C was a game try and 3S was a >sign off. Probably one example will satisfy me. I might accept any of these but example 4; the evidence can come from the player's hand. If it is clear that opener has a game force, not a game try, then 4S is not a LA. The slow pass does not wake opener up to his mistake, so I would allow example 1 if there is a clear miscount. Example 2, as a lead-inhibiting psyche, is allowed as long as the psyche was not fielded. And there is one other possibility: he might have bid 3C, always intending to bid game, in the hope that partner held both red suits and would bid 3NT for a matchpoint top. Example 3 is possible if the AI makes 4S not a logical alternative. For example, if opener mispulled 3C rather than 3D and has a singleton club, partner's lack of club help may now make 4S clear. I would not accept example 4; table feel may make one bid more attractive than another, but it cannot eliminate a logical alternative. From jfusselman at gmail.com Sun Jan 16 07:25:45 2011 From: jfusselman at gmail.com (Jerry Fusselman) Date: Sun, 16 Jan 2011 00:25:45 -0600 Subject: [BLML] Alain's case revisited In-Reply-To: References: <4D257698.6010201@skynet.be> <9647244.332577.1294302281035.JavaMail.ngmail@webmail17.arcor-online.net> <0F0A4E0E-3319-4F02-8EC6-F49CA1492683@starpower.net> Message-ID: On Sun, Jan 16, 2011 at 12:10 AM, David Grabiner wrote: > Jerry Fusselman writes: > >>> 1S-2S >>> 3C-...3S >>> 4S. > >>Anyway, I cannot easily think of what would convince Eric to allow 4S >>after a 3C game try in my hypothetical auction. ?Yes, I am assuming >>that it is a game try. ?Which of these explanations by the innocent >>newbie would Eric (or anyone else) say should lead a wise director to >>allow 4S? > >>1. ?When I bid 3C, I forgot my distribution points, so I changed my >>mind as to what my hand was worth even before LHO passed. >>2. ?I was always planning to go to game, and I was just using 3C to >>try to confuse the opponents. ?We have not done this trick ever >>before, but this is a big event, so I was trying new stuff. >>3. ?I meant to bid 3D, but I goofed, and since I was borderline anyway >>for a game try, I just went to game ignoring my partner's response. > >>Please assume that each explanation is honest and that the director >>somehow knows this. ?I am really curious as to what it would take for >>a wise director to allow 4S, given that 3C was a game try and 3S was a >>sign off. ?Probably one example will satisfy me. > > If it is clear that opener has a game force, not a game try, then 4S is not a > LA. ?The slow pass does not wake opener up to his mistake, so I would allow > example 1 if there is a clear miscount. ?Example 2, as a lead-inhibiting psyche, > is allowed as long as the psyche was not fielded. ?And there is one other > possibility: he might have bid 3C, always intending to bid game, in the hope > that partner held both red suits and would bid 3NT for a matchpoint top. > > Example 3 is possible if the AI makes 4S not a logical alternative. ?For > example, if opener mispulled 3C rather than 3D and has a singleton club, > partner's lack of club help may now make 4S clear. > David, thanks so much for your help. Would you be willing to re-answer 1--3 with a slight change? This time, please assume that the director does not know for sure that the answer is honest, having never seen the players before, but he has no particular reason to believe the answer is dishonest either. Does that affect your answers in any of these three cases? Jerry Fusselman From jkljkl at gmx.de Sun Jan 16 11:24:57 2011 From: jkljkl at gmx.de (Stefan Filonardi) Date: Sun, 16 Jan 2011 11:24:57 +0100 Subject: [BLML] He and she, whe and ble [SEC=UNOFFICIAL] In-Reply-To: <1003254343.19635.1295026950522.JavaMail.ngmail@webmail12.arcor-online.net> References: <973223.53871.qm@web87103.mail.ird.yahoo.com> <1923802510.31506.1294972945375.JavaMail.ngmail@webmail13.arcor-online.net> <1003254343.19635.1295026950522.JavaMail.ngmail@webmail12.arcor-online.net> Message-ID: <4D32C779.5040605@gmx.de> Hello, Am 14.01.2011 18:42, schrieb Thomas Dehn: > There exist the moderate feminist approach > to replace all masculine language with > gender neutral language, and the radical > feminist approach to replace all masculine > language with feminine language (and to > state that we now need 5,000 years of > oppression of men to counterbalance > 5,000 years of oppression of women). It seems that you have omitted the symmetrical option when you use both gender side by side. "Der Spieler und die Spielerin sollen ..." ciao stefan For the interested: You will find a ~200 pages document (last link on the page) about possible solution in the german language, published by the "swiss federal chancellery " (schweizerische Bundeskanzlei), here: http://www.bk.admin.ch/dokumentation/sprachen/04915/05313/index.html?lang=de The Title of the document: "Geschlechtergerechte Sprache - Leitfaden zum geschlechtergerechten Formulieren im Deutschen" From daisy_duck at btopenworld.com Sun Jan 16 12:51:18 2011 From: daisy_duck at btopenworld.com (Stefanie Rohan) Date: Sun, 16 Jan 2011 11:51:18 +0000 (GMT) Subject: [BLML] He and she, whe and ble [SEC=UNOFFICIAL] In-Reply-To: <4D302972.9060002@skynet.be> References: <1923802510.31506.1294972945375.JavaMail.ngmail@webmail13.arcor-online.net> <4D302972.9060002@skynet.be> Message-ID: <58652.11845.qm@web87111.mail.ird.yahoo.com> Herman DW: > And it can be translated into German, indeed. > But not into Franch, sadly. > Joueurs and Joueuses do not share the same word. > And there may be problems in other languages that we don't realize. > It seems to me that the Laws cannot possibly be translated into a language that does not have a gender-neutral word for "player". But in any case, translations are necessarily approximations, and the only authoritative version is the one in the original language. It will be good if English cleans up its own act with regard to sexist language. Marginal languages can shift for themselves... From blml at arcor.de Sun Jan 16 12:54:30 2011 From: blml at arcor.de (Thomas Dehn) Date: Sun, 16 Jan 2011 12:54:30 +0100 (CET) Subject: [BLML] He and she, whe and ble [SEC=UNOFFICIAL] In-Reply-To: <4D32C779.5040605@gmx.de> References: <4D32C779.5040605@gmx.de> <973223.53871.qm@web87103.mail.ird.yahoo.com> <1923802510.31506.1294972945375.JavaMail.ngmail@webmail13.arcor-online.net> <1003254343.19635.1295026950522.JavaMail.ngmail@webmail12.arcor-online.net> Message-ID: <1734302429.20662.1295178870629.JavaMail.ngmail@webmail17.arcor-online.net> Stefan Filonardi wrote: > Am 14.01.2011 18:42, schrieb Thomas Dehn: > > There exist the moderate feminist approach > > to replace all masculine language with > > gender neutral language, and the radical > > feminist approach to replace all masculine > > language with feminine language (and to > > state that we now need 5,000 years of > > oppression of men to counterbalance > > 5,000 years of oppression of women). > > It seems that you have omitted the symmetrical option when you use > both gender side by side. > > "Der Spieler und die Spielerin sollen ..." It is one version of gender neutral language in German. I did not list all possible approaches that lead to gender neutral language in various languages - that would be just confusing for those who do not speak the respective languages. Thomas From blml at arcor.de Sun Jan 16 13:05:36 2011 From: blml at arcor.de (Thomas Dehn) Date: Sun, 16 Jan 2011 13:05:36 +0100 (CET) Subject: [BLML] He and she, whe and ble [SEC=UNOFFICIAL] In-Reply-To: <58652.11845.qm@web87111.mail.ird.yahoo.com> References: <58652.11845.qm@web87111.mail.ird.yahoo.com> <1923802510.31506.1294972945375.JavaMail.ngmail@webmail13.arcor-online.net> <4D302972.9060002@skynet.be> Message-ID: <1798339896.20948.1295179536428.JavaMail.ngmail@webmail17.arcor-online.net> Stefanie Rohan wrote: > Herman DW: > > > And it can be translated into German, indeed. > > But not into Franch, sadly. > > Joueurs and Joueuses do not share the same word. > > And there may be problems in other languages that we don't realize. > > > It seems to me that the Laws cannot possibly be translated into a language that > does not have a gender-neutral word for "player". But in any case, translations > are necessarily approximations, and the only authoritative version is the one in > the original language. It will be good if English cleans up its own act with > regard to sexist language. Marginal languages can shift for themselves... I agree that the existing sexist language should be removed from TFLB. I just disagree with Richard's approach of simply replacing "his" with "her" and "he" with "she". That is sexist language, too. I like Grattan's suggested approach to replace "his" with "the player's". Thomas From Hermandw at skynet.be Sun Jan 16 13:28:06 2011 From: Hermandw at skynet.be (Herman De Wael) Date: Sun, 16 Jan 2011 13:28:06 +0100 Subject: [BLML] At best an echo. [SEC = UNOFFICIAL] In-Reply-To: <4F05628CFC744D98A501853467114821@Mildred> References: <4F05628CFC744D98A501853467114821@Mildred> Message-ID: <4D32E456.4030501@skynet.be> Grattan wrote: > > > > "> It doesn't stay gender neutral when translated >> into German or French etc. Not easy, not easy >> at all." > < > +=+ The translator's exercise is to take whatever is > written in the original English version and convert it > idiomatically to the language of choice. Translations > that are too literal are inadequate; one must rewrite > the effect of the English text - but, of course, this > requires the translator to grasp the nuances of the > original language. "Translation is at best an echo." > ~ Grattan ~ +=+ True Grattan, but it can do no harm to write the English in such a way that translation becomes easy. -- Herman De Wael Wilrijk Antwerpen Belgium From svenpran at online.no Sun Jan 16 13:44:02 2011 From: svenpran at online.no (Sven Pran) Date: Sun, 16 Jan 2011 13:44:02 +0100 Subject: [BLML] He and she, whe and ble [SEC=UNOFFICIAL] In-Reply-To: <58652.11845.qm@web87111.mail.ird.yahoo.com> References: <1923802510.31506.1294972945375.JavaMail.ngmail@webmail13.arcor-online.net> <4D302972.9060002@skynet.be> <58652.11845.qm@web87111.mail.ird.yahoo.com> Message-ID: <000e01cbb57b$0ed81c40$2c8854c0$@no> On Behalf Of Stefanie Rohan > Herman DW: > > > And it can be translated into German, indeed. > > But not into Franch, sadly. > > Joueurs and Joueuses do not share the same word. > > And there may be problems in other languages that we don't realize. > > > It seems to me that the Laws cannot possibly be translated into a language that > does not have a gender-neutral word for "player". But in any case, translations > are necessarily approximations, and the only authoritative version is the one in the > original language. It will be good if English cleans up its own act with regard to > sexist language. Marginal languages can shift for themselves... I do not understand why in any language they cannot adopt something like what is specified in the official English text version of the laws: Wherever a law refers to male players it applies equally to female players unless otherwise is explicitly specified. From blml at arcor.de Sun Jan 16 13:50:31 2011 From: blml at arcor.de (Thomas Dehn) Date: Sun, 16 Jan 2011 13:50:31 +0100 (CET) Subject: [BLML] He and she, whe and ble [SEC=UNOFFICIAL] In-Reply-To: <000e01cbb57b$0ed81c40$2c8854c0$@no> References: <000e01cbb57b$0ed81c40$2c8854c0$@no> <1923802510.31506.1294972945375.JavaMail.ngmail@webmail13.arcor-online.net> <4D302972.9060002@skynet.be> <58652.11845.qm@web87111.mail.ird.yahoo.com> Message-ID: <1019856396.21844.1295182231182.JavaMail.ngmail@webmail17.arcor-online.net> Sven Pran wrote: > On Behalf Of Stefanie Rohan > > Herman DW: > > > > > And it can be translated into German, indeed. > > > But not into Franch, sadly. > > > Joueurs and Joueuses do not share the same word. > > > And there may be problems in other languages that we don't realize. > > > > > It seems to me that the Laws cannot possibly be translated into a language that > > does not have a gender-neutral word for "player". But in any case, translations > > are necessarily approximations, and the only authoritative version is the one in the > > original language. It will be good if English cleans up its own act with regard to > > sexist language. Marginal languages can shift for themselves... > > I do not understand why in any language they cannot adopt something like > what is specified in the official English text version of the laws: > > Wherever a law refers to male players it applies equally to female players > unless otherwise is explicitly specified. Because it is sexist. Thomas From Hermandw at skynet.be Sun Jan 16 14:30:37 2011 From: Hermandw at skynet.be (Herman De Wael) Date: Sun, 16 Jan 2011 14:30:37 +0100 Subject: [BLML] He and she, whe and ble [SEC=UNOFFICIAL] In-Reply-To: <1019856396.21844.1295182231182.JavaMail.ngmail@webmail17.arcor-online.net> References: <000e01cbb57b$0ed81c40$2c8854c0$@no> <1923802510.31506.1294972945375.JavaMail.ngmail@webmail13.arcor-online.net> <4D302972.9060002@skynet.be> <58652.11845.qm@web87111.mail.ird.yahoo.com> <1019856396.21844.1295182231182.JavaMail.ngmail@webmail17.arcor-online.net> Message-ID: <4D32F2FD.3080109@skynet.be> Thomas Dehn wrote: >> >> I do not understand why in any language they cannot adopt something like >> what is specified in the official English text version of the laws: >> >> Wherever a law refers to male players it applies equally to female players >> unless otherwise is explicitly specified. > > Because it is sexist. > Yes, so what? Language is sexist - not bridge laws. This is a fact of life, try to live with it. The main objective of the laws is to be understandable. I much prefer "he" to the weird constructions "he or she" "(s)he" or "it". Maybe we ought to allow the final say on this to Joan Gerard. I know that when she chairs an AC, she prefers to be called chairman, above all constructions "chairperson", "chairwoman" or "chair". Maybe we ought to look at the "he" as a simple grammatical gender. In French, all words have a gender, even those that in English would be neutral. The grammatical gender of joueur is male. So it can be in English, if we state that "he" refers to a player, of either sex. And yes, I agree that it has been the male form for 5000 years, and maybe the female form should get the upper hand for the next 5000. But fight that battle somewhere else, please. -- Herman De Wael Wilrijk Antwerpen Belgium From jkljkl at gmx.de Sun Jan 16 15:06:53 2011 From: jkljkl at gmx.de (Stefan Filonardi) Date: Sun, 16 Jan 2011 15:06:53 +0100 Subject: [BLML] He and she, whe and ble [SEC=UNOFFICIAL] In-Reply-To: <4D32F2FD.3080109@skynet.be> References: <000e01cbb57b$0ed81c40$2c8854c0$@no> <1923802510.31506.1294972945375.JavaMail.ngmail@webmail13.arcor-online.net> <4D302972.9060002@skynet.be> <58652.11845.qm@web87111.mail.ird.yahoo.com> <1019856396.21844.1295182231182.JavaMail.ngmail@webmail17.arcor-online.net> <4D32F2FD.3080109@skynet.be> Message-ID: <4D32FB7D.9040506@gmx.de> Hello, Am 16.01.2011 14:30, schrieb Herman De Wael: > And yes, I agree that it has been the male form for 5000 years, and > maybe the female form should get the upper hand for the next 5000. But > fight that battle somewhere else, please. Well, considering that Bridge is a game played mostly by female players and has an image of 4 little old ladies sitting at the table it looks to me as the perfect ground for this battle ;-) ciao stefan From lumenco at ono.com Sun Jan 16 19:11:42 2011 From: lumenco at ono.com (Lucas Mendoza) Date: Sun, 16 Jan 2011 18:11:42 -0000 Subject: [BLML] He and she, whe and ble In-Reply-To: <58652.11845.qm@web87111.mail.ird.yahoo.com> References: <1923802510.31506.1294972945375.JavaMail.ngmail@webmail13.arcor-online.net> <4D302972.9060002@skynet.be> <58652.11845.qm@web87111.mail.ird.yahoo.com> Message-ID: <000f01cbb5a8$d42b1ec0$7c815c40$@com> Stephanie Rohan said.. Marginal languages can shift for themselves... So what?? Which are "marginal languages"?? When I saw "marginal" my heart bleed. I nearly cried. This type of language usage offends some people. It will offend many people in many cultures. -----Original Message----- From: blml-bounces at rtflb.org [mailto:blml-bounces at rtflb.org] On Behalf Of Stefanie Rohan Sent: domingo, 16 de enero de 2011 11:51 To: Bridge Laws Mailing List Subject: Re: [BLML] He and she, whe and ble [SEC=UNOFFICIAL] Herman DW: > And it can be translated into German, indeed. > But not into Franch, sadly. > Joueurs and Joueuses do not share the same word. > And there may be problems in other languages that we don't realize. > It seems to me that the Laws cannot possibly be translated into a language that does not have a gender-neutral word for "player". But in any case, translations are necessarily approximations, and the only authoritative version is the one in the original language. It will be good if English cleans up its own act with regard to sexist language. Marginal languages can shift for themselves... -------------- next part -------------- An HTML attachment was scrubbed... URL: http://lists.rtflb.org/pipermail/blml/attachments/20110116/e2e251d7/attachment-0001.html From nigelguthrie at yahoo.co.uk Mon Jan 17 01:44:00 2011 From: nigelguthrie at yahoo.co.uk (Nigel Guthrie) Date: Mon, 17 Jan 2011 00:44:00 +0000 (GMT) Subject: [BLML] He and she, whe and ble [SEC=UNOFFICIAL] In-Reply-To: <000e01cbb57b$0ed81c40$2c8854c0$@no> References: <1923802510.31506.1294972945375.JavaMail.ngmail@webmail13.arcor-online.net> <4D302972.9060002@skynet.be> <58652.11845.qm@web87111.mail.ird.yahoo.com> <000e01cbb57b$0ed81c40$2c8854c0$@no> Message-ID: <202219.85950.qm@web28515.mail.ukl.yahoo.com> Stephanie points out that She contains he and WOman contains man. As far a English is concerned, IMO, most players would be happy with Stephanie Rowan's resolution. If law-makers find it hard to avoid sexist language, they can harness it to make the laws clearer and more concise: Designate the putative OS as male and the NOS as female (or vice versa). :) From JffEstrsn at aol.com Mon Jan 17 10:30:19 2011 From: JffEstrsn at aol.com (Jeff Easterson) Date: Mon, 17 Jan 2011 10:30:19 +0100 Subject: [BLML] He and she, whe and ble In-Reply-To: <000f01cbb5a8$d42b1ec0$7c815c40$@com> References: <1923802510.31506.1294972945375.JavaMail.ngmail@webmail13.arcor-online.net> <4D302972.9060002@skynet.be> <58652.11845.qm@web87111.mail.ird.yahoo.com> <000f01cbb5a8$d42b1ec0$7c815c40$@com> Message-ID: <4D340C2B.7060402@aol.com> An HTML attachment was scrubbed... URL: http://lists.rtflb.org/pipermail/blml/attachments/20110117/65d42cd2/attachment.html From Hermandw at skynet.be Mon Jan 17 10:45:48 2011 From: Hermandw at skynet.be (Herman De Wael) Date: Mon, 17 Jan 2011 10:45:48 +0100 Subject: [BLML] He and she, whe and ble In-Reply-To: <4D340C2B.7060402@aol.com> References: <1923802510.31506.1294972945375.JavaMail.ngmail@webmail13.arcor-online.net> <4D302972.9060002@skynet.be> <58652.11845.qm@web87111.mail.ird.yahoo.com> <000f01cbb5a8$d42b1ec0$7c815c40$@com> <4D340C2B.7060402@aol.com> Message-ID: <4D340FCC.1070806@skynet.be> Jeff Easterson wrote: > I wondered about "marginal languages" as well. Anything but English? JE > I think Stephanie was being overly one-sided. There are indeed only two possible stances for the WBF to take: - write the laws in English only, and consider all other languages "marginal"; - write the laws in all languages at the same time; Of course the second one is infeasible. But I do urge the WBF to take into account the fact that the laws have to be translated, and that this means easy language is not only useful for the English text, but also for the ease of translation. -- Herman De Wael Wilrijk Antwerpen Belgium From swillner at nhcc.net Mon Jan 17 15:54:38 2011 From: swillner at nhcc.net (Steve Willner) Date: Mon, 17 Jan 2011 09:54:38 -0500 Subject: [BLML] Ecclesiastes In-Reply-To: <4D304E50.7080502@ulb.ac.be> References: <4D302790.8050401@skynet.be> <1748200137.98748.1295002698502.JavaMail.ngmail@webmail08.arcor-online.net> <4D304E50.7080502@ulb.ac.be> Message-ID: <4D34582E.8070509@nhcc.net> On 1/14/2011 8:23 AM, Alain Gottcheiner wrote: > Lead : D8. EW play their own version of Rusinow. > North asks to West whether the lead could come from 98x(x) : yes. > From H98x ? No. [and the lead is in fact from K98x] I couldn't read the hand diagram or bidding. In future, please format no wider than 72 characters per line and with spaces, no tabs. Despite that, the above seems an obvious case of MI unless EW can demonstrate by very clear evidence that their agreement was as explained. > EW have a fully-filled convention card. But West tells you that it isn't > the right one, but the old one : he just forgot the new CC at home. He > then puts it back in his pocket. That's a separate issue. What are the CC regulations, and what is customary for such events in your jurisdiction? _At best_, EW have played so far with no valid CC. At worst, they've provided a deliberately misleading CC. If the only differences between the new agreements and old ones are minor, I wouldn't necessarily be quite so severe as Eric (disqualification + C&E hearing). But if changes are minor, why couldn't EW have pencilled them in? If the whole system has changed, I think Eric has it right. How far through the round has this occurred? If you don't disqualify EW, there's a need to investigate whether NS were damaged on other boards. > I don't care about the problem of possibly changing the score ; it would > have been marginal (North's line of play only caters for ... Here you seem to be considering whether declarer should have taken the correct line even after the MI. As we've discussed here many times, the relevant question is what would have happened with correct information. Only after deciding that do you rule whether declarer's actual line was "serious error." > Does the fact that there still is a bone to pick between those two pairs > affecrt you in any way ? I'd like to say no, but probably it would cause me to think more carefully and make sure my ruling is fully defensible. From grabiner at alumni.princeton.edu Mon Jan 17 15:56:07 2011 From: grabiner at alumni.princeton.edu (David Grabiner) Date: Mon, 17 Jan 2011 09:56:07 -0500 Subject: [BLML] Alain's case revisited In-Reply-To: References: <4D257698.6010201@skynet.be> <9647244.332577.1294302281035.JavaMail.ngmail@webmail17.arcor-online.net> <0F0A4E0E-3319-4F02-8EC6-F49CA1492683@starpower.net> Message-ID: "Jerry Fusselman" wrote: >On Sun, Jan 16, 2011 at 12:10 AM, David Grabiner wrote: >> Jerry Fusselman writes: >> >>>> 1S-2S >>>> 3C-...3S >>>> 4S. >> >>>Anyway, I cannot easily think of what would convince Eric to allow 4S >>>after a 3C game try in my hypothetical auction. Yes, I am assuming >>>that it is a game try. Which of these explanations by the innocent >>>newbie would Eric (or anyone else) say should lead a wise director to >>>allow 4S? >> >>>1. When I bid 3C, I forgot my distribution points, so I changed my >>>mind as to what my hand was worth even before LHO passed. >>>2. I was always planning to go to game, and I was just using 3C to >>>try to confuse the opponents. We have not done this trick ever >>>before, but this is a big event, so I was trying new stuff. >>>3. I meant to bid 3D, but I goofed, and since I was borderline anyway >>>for a game try, I just went to game ignoring my partner's response. >> >>>Please assume that each explanation is honest and that the director >>>somehow knows this. I am really curious as to what it would take for >>>a wise director to allow 4S, given that 3C was a game try and 3S was a >>>sign off. Probably one example will satisfy me. >> >> If it is clear that opener has a game force, not a game try, then 4S is not a >> LA. The slow pass does not wake opener up to his mistake, so I would allow >> example 1 if there is a clear miscount. Example 2, as a lead-inhibiting >> psyche, >> is allowed as long as the psyche was not fielded. And there is one other >> possibility: he might have bid 3C, always intending to bid game, in the hope >> that partner held both red suits and would bid 3NT for a matchpoint top. >> >> Example 3 is possible if the AI makes 4S not a logical alternative. For >> example, if opener mispulled 3C rather than 3D and has a singleton club, >> partner's lack of club help may now make 4S clear. >Would you be willing to re-answer 1--3 with a slight change? This >time, please assume that the director does not know for sure that the >answer is honest, having never seen the players before, but he has no >particular reason to believe the answer is dishonest either. Does >that affect your answers in any of these three cases? In all of these examples, I would rule the same way. The issue is not the player's honesty, but whether he has a LA, and the player's hand determines the LA. For example, if opener has AKJxx Kx Ax Kxxx, I accept his claim that he miscounted his points, and he still expects to make 4S even when he knows that partner doesn't have club help. If he has AQJxxx KJx AQ xx, his hand implies that 3C was a lead-inhibiting psyche, not a game try, and he still expects to make 4S despite the declined game try. If opener has AKQxx Kxx Axxx x and claims to have mispulled 3C rather than 3D, I accept his hand as evidence that he mispulled, and the fact that responder declined the 3C game try implies that his values are in hearts and diamonds where they are needed for game. In none of these cases does it matter why opener bid 3C, only that, after partner bid 3S over 3C, 4S is still a good contract. And an honest player is still constrained by the UI rules. Weaken that first hand to KQJxx Kx Ax Kxxx and I would not allow an honest player to bid 4S based on miscounting his points unless the raise guarantees constructive values. The hand counts to 4.5 losers, so many players would go straight to 4S, but once you have asked with 3C and partner has declined, many of your peers would decide that 4S is unlikely to make. From agot at ulb.ac.be Mon Jan 17 16:00:27 2011 From: agot at ulb.ac.be (Alain Gottcheiner) Date: Mon, 17 Jan 2011 16:00:27 +0100 Subject: [BLML] Ecclesiastes In-Reply-To: <4D34582E.8070509@nhcc.net> References: <4D302790.8050401@skynet.be> <1748200137.98748.1295002698502.JavaMail.ngmail@webmail08.arcor-online.net> <4D304E50.7080502@ulb.ac.be> <4D34582E.8070509@nhcc.net> Message-ID: <4D34598B.4030602@ulb.ac.be> Le 17/01/2011 15:54, Steve Willner a ?crit : > On 1/14/2011 8:23 AM, Alain Gottcheiner wrote: >> Lead : D8. EW play their own version of Rusinow. >> North asks to West whether the lead could come from 98x(x) : yes. >> From H98x ? No. > [and the lead is in fact from K98x] > > I couldn't read the hand diagram or bidding. In future, please format > no wider than 72 characters per line and with spaces, no tabs. > > Despite that, the above seems an obvious case of MI unless EW can > demonstrate by very clear evidence that their agreement was as explained. > >> EW have a fully-filled convention card. But West tells you that it isn't >> the right one, but the old one : he just forgot the new CC at home. He >> then puts it back in his pocket. > That's a separate issue. What are the CC regulations, and what is > customary for such events in your jurisdiction? _At best_, EW have > played so far with no valid CC. At worst, they've provided a > deliberately misleading CC. If the only differences between the new > agreements and old ones are minor, I wouldn't necessarily be quite so > severe as Eric (disqualification + C&E hearing). But if changes are > minor, why couldn't EW have pencilled them in? Apparently because they didn't realize the error. Remember that you aren't allowed to look at your own CC. Now, when East had to handle the CC to the TD, he saw the problem. From swillner at nhcc.net Mon Jan 17 16:28:14 2011 From: swillner at nhcc.net (Steve Willner) Date: Mon, 17 Jan 2011 10:28:14 -0500 Subject: [BLML] Ecclesiastes In-Reply-To: <4D34598B.4030602@ulb.ac.be> References: <4D302790.8050401@skynet.be> <1748200137.98748.1295002698502.JavaMail.ngmail@webmail08.arcor-online.net> <4D304E50.7080502@ulb.ac.be> <4D34582E.8070509@nhcc.net> <4D34598B.4030602@ulb.ac.be> Message-ID: <4D34600E.5000603@nhcc.net> >> But if changes [to the CC] are >> minor, why couldn't EW have pencilled them in? On 1/17/2011 10:00 AM, Alain Gottcheiner wrote: > Apparently because they didn't realize the error. Remember that you > aren't allowed to look at your own CC. Now, when East had to handle the > CC to the TD, he saw the problem. They could have checked at the start of the session and certainly should have known in advance of the event that changing the system would require changing the CC. But OK, perhaps this was a careless error, not a malicious one. One thing that still gives me pause is West's hiding the CC rather than explaining its errors. Here in the ACBL, despite our regulations, players in practice are often careless about proper CCs. In that sort of environment, you probably shouldn't disqualify if indeed the changes were minor, but you do need to make sure the wrong CC hasn't caused damage on other deals. Whatever you do, you have to make sure the innocent side isn't put at a disadvantage on any deals, past, present, or future. Ruling that MI existed on the current deal seems automatic. (You still need to rule on damage and serious error to decide the actual score adjustment, if any.) For future deals, you make whatever ruling is standard for "no CC" in your jurisdiction, considering the stage of the round. From rfrick at rfrick.info Mon Jan 17 19:58:07 2011 From: rfrick at rfrick.info (Robert Frick) Date: Mon, 17 Jan 2011 13:58:07 -0500 Subject: [BLML] In the trenches. Message-ID: The yesterday auction was S W N E 1NT 2C X 2S X 3H ? At this point NS discover that 2C is "Cappelletti", unalerted even though ACBL regulations call for an alert if 2C is not natural. It is described (by the 2C bidder! but pd agrees) as showing both majors. 2S is simply a foolish bid, she has 10xxxx of spades. In practice, North shot out a 5C bid for a bottom, as they belong in 3NT. The critical point seems to be if opener (South) would have bid 2NT (or 3NT) if she had known the correct explanation at the time. She had AKQ of spades and KQJx of heart (and the queen of diamonds). So a NT bid is very plausible. I am willing to consider the possibility that she would have bid no trump. And I think I would have ruled in their favor, or at least checked things out more, except.... I offered her the chance to change her double and she said no. HOWEVER... If she had been in the actual situation, instead of being offered a chance to change her bid with the director watching and a lot of confusing things happening, I still think it is reasonable that she would have bid no trump. I decided I cannot give the NOS credit for possibly making a bid (with the correct information) that in fact they had a chance to make (with the correct information but giving in an untimely may that might have disrupted her thinking). From blml at arcor.de Mon Jan 17 20:23:15 2011 From: blml at arcor.de (Thomas Dehn) Date: Mon, 17 Jan 2011 20:23:15 +0100 (CET) Subject: [BLML] In the trenches. In-Reply-To: References: Message-ID: <1236200011.166424.1295292195352.JavaMail.ngmail@webmail08.arcor-online.net> Robert Frick wrote: > The yesterday auction was > S W N E > 1NT 2C X 2S > X 3H ? > > At this point NS discover that 2C is "Cappelletti", unalerted even though > ACBL regulations call for an alert if 2C is not natural. It is described > (by the 2C bidder! but pd agrees) as showing both majors. 2S is simply a > foolish bid, she has 10xxxx of spades. > > In practice, North shot out a 5C bid for a bottom, as they belong in 3NT. This confuses me. The director is called. The director opens TFLB and reads L21B1(a). He asks S whether he wants to change his X over 2S. Thomas From nigelguthrie at yahoo.co.uk Mon Jan 17 20:59:32 2011 From: nigelguthrie at yahoo.co.uk (Nigel Guthrie) Date: Mon, 17 Jan 2011 19:59:32 +0000 (GMT) Subject: [BLML] Deal Format In-Reply-To: <4D34582E.8070509@nhcc.net> References: <4D302790.8050401@skynet.be> <1748200137.98748.1295002698502.JavaMail.ngmail@webmail08.arcor-online.net> <4D304E50.7080502@ulb.ac.be> <4D34582E.8070509@nhcc.net> Message-ID: <511188.15574.qm@web28504.mail.ukl.yahoo.com> [Steve Willner] I couldn't read the hand diagram or bidding. In future, please format no wider than 72 characters per line and with spaces, no tabs. [Nige1] If you view posts in plain text, spaces are fine; but if you rich text, then multiple spaces may be condensed. A possible solution is to use dots (full-stops). Provided that you specify a monospace font, that should produce legible diagrams. You can still use *single* spaces but you should avoid multiple spaces. If you can't read this diagram, then please say so... PIVOT STRIP.. 32 5S by South.. Q32 Heart lead... 65432 ............. T32 J9........... ............. QT T864......... ............. KJ975 97........... ............. QT8 K9754........ ............. Q86 ............. AK98765 ............. A ............. AKJ ............. xx From rfrick at rfrick.info Mon Jan 17 21:01:12 2011 From: rfrick at rfrick.info (Robert Frick) Date: Mon, 17 Jan 2011 15:01:12 -0500 Subject: [BLML] In the trenches. In-Reply-To: <1236200011.166424.1295292195352.JavaMail.ngmail@webmail08.arcor-online.net> References: <1236200011.166424.1295292195352.JavaMail.ngmail@webmail08.arcor-online.net> Message-ID: On Mon, 17 Jan 2011 14:23:15 -0500, Thomas Dehn wrote: > Robert Frick wrote: >> The yesterday auction was >> S W N E >> 1NT 2C X 2S >> X 3H ? >> >> At this point NS discover that 2C is "Cappelletti", unalerted even >> though >> ACBL regulations call for an alert if 2C is not natural. It is described >> (by the 2C bidder! but pd agrees) as showing both majors. 2S is simply a >> foolish bid, she has 10xxxx of spades. >> >> In practice, North shot out a 5C bid for a bottom, as they belong in >> 3NT. > > This confuses me. > > The director is called. The director opens TFLB and reads > L21B1(a). He asks S whether he wants to change his X over 2S. She decides not to change. But that is a different context from taking the bid in a free auction. > > > > Thomas > > _______________________________________________ > Blml mailing list > Blml at rtflb.org > http://lists.rtflb.org/mailman/listinfo/blml -- somepsychology.com From blml at arcor.de Mon Jan 17 21:08:59 2011 From: blml at arcor.de (Thomas Dehn) Date: Mon, 17 Jan 2011 21:08:59 +0100 (CET) Subject: [BLML] Deal Format In-Reply-To: <511188.15574.qm@web28504.mail.ukl.yahoo.com> References: <511188.15574.qm@web28504.mail.ukl.yahoo.com> <4D302790.8050401@skynet.be> <1748200137.98748.1295002698502.JavaMail.ngmail@webmail08.arcor-online.net> <4D304E50.7080502@ulb.ac.be> <4D34582E.8070509@nhcc.net> Message-ID: <114049437.170264.1295294939565.JavaMail.ngmail@webmail08.arcor-online.net> Nigel Guthrie wrote: > [Steve Willner] > > I couldn't read the hand diagram or bidding. In future, please format no > wider > than 72 characters per line and with spaces, no tabs. > > [Nige1] > If you view posts in plain text, spaces are fine; but if you rich text, then > > multiple spaces may be condensed. A possible solution is to use dots > (full-stops). Provided that you specify a monospace font, that should > produce > legible diagrams. You can still use *single* spaces but you should avoid > multiple spaces. If you can't read this diagram, then please say so... > > PIVOT STRIP.. 32 > 5S by South.. Q32 > Heart lead... 65432 > ............. T32 > J9........... ............. QT > T864......... ............. KJ975 > 97........... ............. QT8 > K9754........ ............. Q86 > ............. AK98765 > ............. A > ............. AKJ > ............. xx This is readable - but it seems to lack a CA and a CJ. Thomas From nigelguthrie at yahoo.co.uk Mon Jan 17 21:42:07 2011 From: nigelguthrie at yahoo.co.uk (Nigel Guthrie) Date: Mon, 17 Jan 2011 20:42:07 +0000 (GMT) Subject: [BLML] Deal Format In-Reply-To: <114049437.170264.1295294939565.JavaMail.ngmail@webmail08.arcor-online.net> References: <511188.15574.qm@web28504.mail.ukl.yahoo.com> <4D302790.8050401@skynet.be> <1748200137.98748.1295002698502.JavaMail.ngmail@webmail08.arcor-online.net> <4D304E50.7080502@ulb.ac.be> <4D34582E.8070509@nhcc.net> <114049437.170264.1295294939565.JavaMail.ngmail@webmail08.arcor-online.net> Message-ID: <865914.85212.qm@web28515.mail.ukl.yahoo.com> PIVOT STRIP.. 32 5S by South.. Q32 Heart lead... 65432 ............. T32 J9........... ............. QT T864......... ............. KJ975 97........... ............. QT8 A976......... ............. KQJ8 ............. AK98765 ............. A ............. AKJ ............. 54 [Thomas Dehn] This is readable - but it seems to lack a CA and a CJ. [Nige2] Thank you. Corrected above. From nigelguthrie at yahoo.co.uk Mon Jan 17 21:44:59 2011 From: nigelguthrie at yahoo.co.uk (Nigel Guthrie) Date: Mon, 17 Jan 2011 20:44:59 +0000 (GMT) Subject: [BLML] Deal Format In-Reply-To: <865914.85212.qm@web28515.mail.ukl.yahoo.com> References: <511188.15574.qm@web28504.mail.ukl.yahoo.com> <4D302790.8050401@skynet.be> <1748200137.98748.1295002698502.JavaMail.ngmail@webmail08.arcor-online.net> <4D304E50.7080502@ulb.ac.be> <4D34582E.8070509@nhcc.net> <114049437.170264.1295294939565.JavaMail.ngmail@webmail08.arcor-online.net> <865914.85212.qm@web28515.mail.ukl.yahoo.com> Message-ID: <783603.93469.qm@web28515.mail.ukl.yahoo.com> Mon, 17 January, 2011 20:42:07 Re: [BLML] Deal Format... From: Nigel Guthrie ...Add to Contacts To: Bridge Laws Mailing List ________________________________ PIVOT STRIP.. 32 5S by South.. Q32 Heart lead... 65432 ............. T32 J9........... ............. QT T8654........ ............. KJ97 97........... ............. QT8 A976......... ............. KQJ8 ............. AK98765 ............. A ............. AKJ ............. 54 [Thomas Dehn] This is readable - but it seems to lack a CA and a CJ. [Nige2] Thank you. Corrected above. And right number of cards in each hand. From adam at irvine.com Mon Jan 17 21:45:24 2011 From: adam at irvine.com (Adam Beneschan) Date: Mon, 17 Jan 2011 12:45:24 -0800 Subject: [BLML] Deal Format In-Reply-To: Your message of "Mon, 17 Jan 2011 20:42:07 GMT." <865914.85212.qm@web28515.mail.ukl.yahoo.com> Message-ID: <20110117204526.BC659A8C883@mailhub.irvine.com> > PIVOT STRIP.. 32 > > 5S by South.. Q32 > Heart lead... 65432 > ............. T32 > J9........... ............. QT > T864......... ............. KJ975 > 97........... ............. QT8 > A976......... ............. KQJ8 > ............. AK98765 > ............. A > ............. AKJ > ............. 54 > > [Thomas Dehn] > This is readable - but it seems to lack a CA and a CJ. > > [Nige2] > Thank you. Corrected above. Except that now one of the hands has 12 cards and another has 14... But at least it's still readable. -- Adam From blml at arcor.de Mon Jan 17 21:55:50 2011 From: blml at arcor.de (Thomas Dehn) Date: Mon, 17 Jan 2011 21:55:50 +0100 (CET) Subject: [BLML] Deal Format In-Reply-To: <783603.93469.qm@web28515.mail.ukl.yahoo.com> References: <783603.93469.qm@web28515.mail.ukl.yahoo.com> <511188.15574.qm@web28504.mail.ukl.yahoo.com> <4D302790.8050401@skynet.be> <1748200137.98748.1295002698502.JavaMail.ngmail@webmail08.arcor-online.net> <4D304E50.7080502@ulb.ac.be> <4D34582E.8070509@nhcc.net> <114049437.170264.1295294939565.JavaMail.ngmail@webmail08.arcor-online.net> <865914.85212.qm@web28515.mail.ukl.yahoo.com> Message-ID: <174902428.130919.1295297750027.JavaMail.ngmail@webmail07.arcor-online.net> Nigel Guthrie wrote: > Mon, 17 January, 2011 20:42:07 > Re: [BLML] Deal Format... > From: Nigel Guthrie ...Add to Contacts > To: Bridge Laws Mailing List > > ________________________________ > > PIVOT STRIP.. 32 > 5S by South.. Q32 > Heart lead... 65432 > ............. T32 > J9........... ............. QT > T8654........ ............. KJ97 > 97........... ............. QT8 > A976......... ............. KQJ8 > ............. AK98765 > ............. A > ............. AKJ > ............. 54 > > [Thomas Dehn] > This is readable - but it seems to lack a CA and a CJ. > > [Nige2] > Thank you. Corrected above. And right number of cards in each hand. Nice squeeze. Thomas From hirsch9000 at gmail.com Mon Jan 17 22:49:12 2011 From: hirsch9000 at gmail.com (Hirsch Davis) Date: Mon, 17 Jan 2011 16:49:12 -0500 Subject: [BLML] In the trenches. In-Reply-To: <1236200011.166424.1295292195352.JavaMail.ngmail@webmail08.arcor-online.net> References: <1236200011.166424.1295292195352.JavaMail.ngmail@webmail08.arcor-online.net> Message-ID: <4D34B958.9070602@gmail.com> On 1/17/2011 2:23 PM, Thomas Dehn wrote: > Robert Frick wrote: >> The yesterday auction was >> S W N E >> 1NT 2C X 2S >> X 3H ? >> >> At this point NS discover that 2C is "Cappelletti", unalerted even though >> ACBL regulations call for an alert if 2C is not natural. It is described >> (by the 2C bidder! but pd agrees) as showing both majors. 2S is simply a >> foolish bid, she has 10xxxx of spades. >> >> In practice, North shot out a 5C bid for a bottom, as they belong in 3NT. > This confuses me. > > The director is called. The director opens TFLB and reads > L21B1(a). He asks S whether he wants to change his X over 2S. > > > > Thomas Yes, but incomplete. Was N aware that the call was Cappelleti? If not, would N still have doubled 2C? The N double is also compromised by the MI, and the auction cannot be reversed to that point, so an adjustment might still be necessary. From the information provided, it appears that N might have thought he was making a penalty double of a natural 2C. Would double have a different meaning over an artificial call? Since the TD does not appear to have investigated fully, it's not clear. Not clear why the TD thinks 2S is foolish opposite a call showing majors. With five-card support for one of partner's suits, the only issue might be that the call was too low. Final ruling is not clear due to absence of relevant information. TD deserves a procedural penalty for sloppy investigation. Hirsch From grandaeval at tiscali.co.uk Tue Jan 18 02:40:14 2011 From: grandaeval at tiscali.co.uk (Grattan) Date: Tue, 18 Jan 2011 01:40:14 -0000 Subject: [BLML] He and she, whe and ble :-) References: <1923802510.31506.1294972945375.JavaMail.ngmail@webmail13.arcor-online.net> <4D302972.9060002@skynet.be> <58652.11845.qm@web87111.mail.ird.yahoo.com> <000f01cbb5a8$d42b1ec0$7c815c40$@com><4D340C2B.7060402@aol.com> <4D340FCC.1070806@skynet.be> Message-ID: <4CCF9CBA19C0473B9668C4A8E3196EAA@Mildred> Grattan Endicott To: "Bridge Laws Mailing List" Sent: Monday, January 17, 2011 9:45 AM Subject: Re: [BLML] He and she, whe and ble Jeff Easterson wrote: I wondered about "marginal languages" as well. Anything but English? > ..... and Herman De Wael wrote: But I do urge the WBF to take into account the fact that the laws have to be translated, and that this means easy language is not only useful for the English text, but also for the ease of translation. > +=+ Personally I think the term 'marginal' is infelicitous. One might think of all languages but the one in which the original is written as 'second stage' languages. Or call them 'subsequent' if you will. As to Herman's thought, please excuse me if I laugh. The task of those drafting the original text of the law is to find words that express the intention of the law accurately and securely. The evidence is that divers persons, working in felicitous conjunction, have found such perfection difficult. When it comes to translation into other languages, as from the hieroglyphs of the Rosetta Stone, it is a task for those who come later to the feast. What is more it is hardly a likely thought that one should find words in English to cross the divide to those languages where even a humble pan has sex - and a pot for cooking is feminine while a pot for jam is male. ~ Grattan ~ From grandaeval at tiscali.co.uk Tue Jan 18 03:00:22 2011 From: grandaeval at tiscali.co.uk (Grattan) Date: Tue, 18 Jan 2011 02:00:22 -0000 Subject: [BLML] In the trenches. References: <1236200011.166424.1295292195352.JavaMail.ngmail@webmail08.arcor-online.net> <4D34B958.9070602@gmail.com> Message-ID: Grattan Endicott To: "Bridge Laws Mailing List" Sent: Monday, January 17, 2011 9:49 PM Subject: Re: [BLML] In the trenches. > On 1/17/2011 2:23 PM, Thomas Dehn wrote: >> Robert Frick wrote: >>> The yesterday auction was >>> S W N E >>> 1NT 2C X 2S >>> X 3H ? >>> >>> At this point NS discover that 2C is "Cappelletti", unalerted even >>> though >>> ACBL regulations call for an alert if 2C is not natural. It is described >>> (by the 2C bidder! but pd agrees) as showing both majors. 2S is simply a >>> foolish bid, she has 10xxxx of spades. >>> >>> In practice, North shot out a 5C bid for a bottom, as they belong in >>> 3NT. >> This confuses me. >> >> The director is called. The director opens TFLB and reads >> L21B1(a). He asks S whether he wants to change his X over 2S. >> >> >> >> Thomas > > Yes, but incomplete. Was N aware that the call was Cappelleti? If not, > would N still have doubled 2C? The N double is also compromised by the > MI, and the auction cannot be reversed to that point, so an adjustment > might still be necessary. From the information provided, it appears that > N might have thought he was making a penalty double of a natural 2C. > Would double have a different meaning over an artificial call? Since the > TD does not appear to have investigated fully, it's not clear. > > Not clear why the TD thinks 2S is foolish opposite a call showing > majors. With five-card support for one of partner's suits, the only > issue might be that the call was too low. > > Final ruling is not clear due to absence of relevant information. TD > deserves a procedural penalty for sloppy investigation. > > Hirsch > +=+ Or a gold star for invention? +=+ From jfusselman at gmail.com Tue Jan 18 03:16:05 2011 From: jfusselman at gmail.com (Jerry Fusselman) Date: Mon, 17 Jan 2011 20:16:05 -0600 Subject: [BLML] In the trenches. In-Reply-To: References: <1236200011.166424.1295292195352.JavaMail.ngmail@webmail08.arcor-online.net> <4D34B958.9070602@gmail.com> Message-ID: +=+ Or a gold star for invention? +=+ That's a good one, Grattan. I think Robert believes that what a person says he would do in a hypothetical situation is not always a reliable indicator of what he would actually do. This was emphasized in my first year of graduate study in economics, and I agree with it. Its implications for ideal directing seem unclear to me. Probably that is why Robert brings it to our attention: He wants to explore this issue. It could be interesting. Jerry Fusselman From harald.skjaran at gmail.com Tue Jan 18 10:24:10 2011 From: harald.skjaran at gmail.com (=?UTF-8?Q?Harald_Skj=C3=A6ran?=) Date: Tue, 18 Jan 2011 10:24:10 +0100 Subject: [BLML] Ecclesiastes In-Reply-To: <4D34598B.4030602@ulb.ac.be> References: <4D302790.8050401@skynet.be> <1748200137.98748.1295002698502.JavaMail.ngmail@webmail08.arcor-online.net> <4D304E50.7080502@ulb.ac.be> <4D34582E.8070509@nhcc.net> <4D34598B.4030602@ulb.ac.be> Message-ID: 2011/1/17 Alain Gottcheiner : > Le 17/01/2011 15:54, Steve Willner a ?crit : >> On 1/14/2011 8:23 AM, Alain Gottcheiner wrote: >>> Lead : D8. EW play their own version of Rusinow. >>> North asks to West whether the lead could come from 98x(x) : yes. >>> ? ?From H98x ? No. >> [and the lead is in fact from K98x] >> >> I couldn't read the hand diagram or bidding. ?In future, please format >> no wider than 72 characters per line and with spaces, no tabs. >> >> Despite that, the above seems an obvious case of MI unless EW can >> demonstrate by very clear evidence that their agreement was as explained. >> >>> EW have a fully-filled convention card. But West tells you that it isn't >>> the right one, but the old one : he just forgot the new CC at home. He >>> then puts it back in his pocket. >> That's a separate issue. ?What are the CC regulations, and what is >> customary for such events in your jurisdiction? ?_At best_, EW have >> played so far with no valid CC. ?At worst, they've provided a >> deliberately misleading CC. ?If the only differences between the new >> agreements and old ones are minor, I wouldn't necessarily be quite so >> severe as Eric (disqualification + C&E hearing). ?But if changes are >> minor, why couldn't EW have pencilled them in? > Apparently because they didn't realize the error. Remember that you > aren't allowed to look at your own CC. Now, when East had to handle the > CC to the TD, he saw the problem. That's not my impression of the facts from the OP. It seemed to me that he knew all along that the CC was incorrect (or maybe realized when asked), never looked at the CC but put it away, refusing to show it to the TD. > > > _______________________________________________ > Blml mailing list > Blml at rtflb.org > http://lists.rtflb.org/mailman/listinfo/blml > -- Kind regards, Harald Skj?ran From blml at arcor.de Tue Jan 18 12:50:36 2011 From: blml at arcor.de (Thomas Dehn) Date: Tue, 18 Jan 2011 12:50:36 +0100 (CET) Subject: [BLML] In the trenches. In-Reply-To: References: Message-ID: <417823899.156866.1295351436908.JavaMail.ngmail@webmail07.arcor-online.net> Back to Robert's original question: Robert Frick wrote: > The yesterday auction was > S W N E > 1NT 2C X 2S > X 3H ? > > At this point NS discover that 2C is "Cappelletti", unalerted even though > ACBL regulations call for an alert if 2C is not natural. It is described > (by the 2C bidder! but pd agrees) as showing both majors. 2S is simply a > foolish bid, she has 10xxxx of spades. > > In practice, North shot out a 5C bid for a bottom, as they belong in 3NT. > The critical point seems to be if opener (South) would have bid 2NT (or > 3NT) if she had known the correct explanation at the time. She had AKQ of > spades and KQJx of heart (and the queen of diamonds). So a NT bid is very > plausible. > > I am willing to consider the possibility that she would have bid no trump. > And I think I would have ruled in their favor, or at least checked things > out more, except.... I offered her the chance to change her double and she > said no. HOWEVER... If she had been in the actual situation, instead of > being offered a chance to change her bid with the director watching and a > lot of confusing things happening, I still think it is reasonable that she > would have bid no trump. > > I decided I cannot give the NOS credit for possibly making a bid (with the > correct information) that in fact they had a chance to make (with the > correct information but giving in an untimely may that might have > disrupted her thinking). As S got a chance to change the double over 2S, the director cannot rule MI here. However, there are more points the director has to evaluate: 1.) MI when N doubled 2C 2.) UI when W bid 3H rather than pass 2S X. Thomas From Hermandw at skynet.be Tue Jan 18 13:03:38 2011 From: Hermandw at skynet.be (Herman De Wael) Date: Tue, 18 Jan 2011 13:03:38 +0100 Subject: [BLML] He and she, whe and ble :-) In-Reply-To: <4CCF9CBA19C0473B9668C4A8E3196EAA@Mildred> References: <1923802510.31506.1294972945375.JavaMail.ngmail@webmail13.arcor-online.net> <4D302972.9060002@skynet.be> <58652.11845.qm@web87111.mail.ird.yahoo.com> <000f01cbb5a8$d42b1ec0$7c815c40$@com><4D340C2B.7060402@aol.com> <4D340FCC.1070806@skynet.be> <4CCF9CBA19C0473B9668C4A8E3196EAA@Mildred> Message-ID: <4D35819A.4050706@skynet.be> Grattan wrote: > As to Herman's thought, please excuse me if I laugh. > The task of those drafting the original text of the law > is to find words that express the intention of the law > accurately and securely. The evidence is that divers > persons, working in felicitous conjunction, have found > such perfection difficult. When it comes to translation > into other languages, as from the hieroglyphs of the > Rosetta Stone, it is a task for those who come later to > the feast. What is more it is hardly a likely thought that > one should find words in English to cross the divide to > those languages where even a humble pan has sex - and > a pot for cooking is feminine while a pot for jam is male. > ~ Grattan ~ Excuse me if I laugh too, Grattan. The Rosetta stone was not written by an Egyptian who wanted to leave a record of his script for future generations to decypher. The Laws of bridge should be written with the intended audience in mind. That audience consists of a minority of English speakers (A). The rest of the audience will use one of three methods: (B) understand the English text as well as a native; (C) try to understand the English text within his own limited knowledge of the language; (D) rely on a translation. I doubt if the second category will tip the balance from a minority to a majority, but using simpler language (like serious in stead of egregious) might help already. If the English is written in such a way that: - people of category (C) have great difficulty in understanding it; or - the translators have difficulty in expressing the nuances, then the text defeats its own purpose. I urge the Committee (consisting of a majority of English speakers and thus not really reflecting the bridge community as a whole) to take these thoughts into consideration. -- Herman De Wael (member of group B) Wilrijk Antwerpen Belgium From JffEstrsn at aol.com Tue Jan 18 15:03:38 2011 From: JffEstrsn at aol.com (Jeff Easterson) Date: Tue, 18 Jan 2011 15:03:38 +0100 Subject: [BLML] He and she, whe and ble :-) In-Reply-To: <4D35819A.4050706@skynet.be> References: <1923802510.31506.1294972945375.JavaMail.ngmail@webmail13.arcor-online.net> <4D302972.9060002@skynet.be> <58652.11845.qm@web87111.mail.ird.yahoo.com> <000f01cbb5a8$d42b1ec0$7c815c40$@com><4D340C2B.7060402@aol.com> <4D340FCC.1070806@skynet.be> <4CCF9CBA19C0473B9668C4A8E3196EAA@Mildred> <4D35819A.4050706@skynet.be> Message-ID: <4D359DBA.1030903@aol.com> I suspect that substituting "serious" for "egregious" might cause problems: lead to complications, not exactly express the same meaning. Or is my memory of the meanings (not meaning) of these words inaccurate? Ciao, JE Am 18.01.2011 13:03, schrieb Herman De Wael: > Grattan wrote: >> As to Herman's thought, please excuse me if I laugh. >> The task of those drafting the original text of the law >> is to find words that express the intention of the law >> accurately and securely. The evidence is that divers >> persons, working in felicitous conjunction, have found >> such perfection difficult. When it comes to translation >> into other languages, as from the hieroglyphs of the >> Rosetta Stone, it is a task for those who come later to >> the feast. What is more it is hardly a likely thought that >> one should find words in English to cross the divide to >> those languages where even a humble pan has sex - and >> a pot for cooking is feminine while a pot for jam is male. >> ~ Grattan ~ > Excuse me if I laugh too, Grattan. > The Rosetta stone was not written by an Egyptian who wanted to leave a > record of his script for future generations to decypher. > The Laws of bridge should be written with the intended audience in mind. > That audience consists of a minority of English speakers (A). The rest > of the audience will use one of three methods: > (B) understand the English text as well as a native; > (C) try to understand the English text within his own limited knowledge > of the language; > (D) rely on a translation. > I doubt if the second category will tip the balance from a minority to a > majority, but using simpler language (like serious in stead of > egregious) might help already. > If the English is written in such a way that: > - people of category (C) have great difficulty in understanding it; or > - the translators have difficulty in expressing the nuances, > then the text defeats its own purpose. > > I urge the Committee (consisting of a majority of English speakers and > thus not really reflecting the bridge community as a whole) to take > these thoughts into consideration. > From ehaa at starpower.net Tue Jan 18 15:39:18 2011 From: ehaa at starpower.net (Eric Landau) Date: Tue, 18 Jan 2011 09:39:18 -0500 Subject: [BLML] He and she, whe and ble In-Reply-To: <973223.53871.qm@web87103.mail.ird.yahoo.com> References: <1923802510.31506.1294972945375.JavaMail.ngmail@webmail13.arcor-online.net> <973223.53871.qm@web87103.mail.ird.yahoo.com> Message-ID: <4660EEAB-36CA-496B-B86F-3A33EE1DAE28@starpower.net> On Jan 14, 2011, at 12:34 PM, Stefanie Rohan wrote: >> From: Thomas Dehn You are forcing translators to >> deviate from >> the English language version of TFLB. That >> is undesirable. >> >> This type of language usage offends some people. >> It will offend many people in some cultures. > > It is interesting that you consider "feminine" language potentially > "offensive". > Perhaps this means that you have some insight into how all women in > "masculine-language" "cultures" feel all the time. When, as a small > child, I was > told that one must say, for example, "everyone must hand in his > homework on > time", I nearly cried. 30-odd years later, I still feel excluded > when "man" or > "him" is used "inclusively". It is high time English adopted gender- > neutral > language. Or perhaps it could be recognised that the men have had > their turn, > and it is now time to switch to the inclusive feminine. It is more > appropriate, > really, as "woman" contains "man" and "she" contains "he". The > words are > inclusive already, so it makes sense to follow their lead. Although denigrated by prescriptivists, the use of plural pronouns (and matching verbs) as genderless singular pronouns is now becoming generally accepted by descriptivists, who cite a literary history that goes back at least to Jane Austin. Today it is OK, at least by most authorities, to say, "Everyone must hand in their homework on time" (RHUD gives the example, "Everyone should do their own thing"). It makes for gender-neutral constructions without having to either use "he or she", "him or her", "his or hers", etc. repeatedly, and without having to introduce "artificial" new words, both of which make for rather clumsy reading. Of course, I don't think this does TFLB any good, since I doubt that it works in most other languages. Eric Landau 1107 Dale Drive Silver Spring MD 20910 ehaa at starpower.net From ehaa at starpower.net Tue Jan 18 16:14:32 2011 From: ehaa at starpower.net (Eric Landau) Date: Tue, 18 Jan 2011 10:14:32 -0500 Subject: [BLML] Alain's case revisited In-Reply-To: References: <4D2B0857.7050804@skynet.be> <4D2AC556.9070509@skynet.be> <4D2AC9E7.6060208@aol.com> <4D2AE13B.10605@skynet.be> <334994.97415.qm@web28503.mail.ukl.yahoo.com> <1391468851.32037.1294667187404.JavaMail.ngmail@webmail08.arcor-online.net> <4D2B1D0C.7090801@skynet.be> <4D2B2CC8.2060308@ulb.ac.be> <4D2E041A.8070508@nhcc.net> <000e01cbb2a9$ba3fc670$2ebf5350$@no> <707261.93703.qm@web28501.mail.ukl.yahoo.com> <001001cbb308$57dd6af0$079840d0$@no> <51527230-BFC9-4547-AA27-C974619AC3AB@starpower.net> <000901cbb34b$81150a70$833f1f50$@no> <79301020-6658-4713-B8A8-098AF1B57621@starpower.net> <4D307BE6.7060509@ulb.ac.be> Message-ID: <3A060781-A0FB-4A51-A935-C5AF90186680@starpower.net> On Jan 16, 2011, at 12:37 AM, Jerry Fusselman wrote: > On Mon, Jan 10, 2011 at 12:58 AM, Richard Hills wrote: > >> Jerry Fusselman: >> >>>> This happens at two tables---one with a confirmed >>>> cheater bidding 4S, and another with a confirmed >>>> innocent newbie bidding 4S. Do you want the >>>> rectifications in these two cases to be the same or >>>> different? (I am not asking about penalty points here, >>>> only the rectification.) >> >> Grattan Endicott: >> >>> +=+ Could I enquire whether the two players have entered >>> under the same Conditions of Contest? >>> If so, should one be advantaged by comparison with >>> the other against the field? >> >> Richard Hills (male chauvinist pig): >> >> I can understand why an innocent newbie would be playing >> in the Venice Cup. :-) :-) >> >> But could I enquire why the confirmed cheater has also >> been allowed to enter under the same Conditions of >> Contest in this Venice Cup? > > I never mentioned the Venice Cup. > > The terms "innocent newbie" and "confirmed cheater" in this thread > originated in Eric's very interesting post, which I still promise to > return to soon. I meant them exactly how Eric meant them. Let's see > if I understand Eric correctly. > > An innocent newbie ... well, I guess this is probably clear. > Everybody knows. > > A confirmed cheater is not someone who has been banned from playing > bridge, obviously, because he is here, playing bridge. I assume that > Eric's notion of a confirmed cheater is a player who the (wise) > director is personally is sure is (or was) a cheater, with one of > these conditions: > > 1. The director cannot prove it. > 2. The proof has not yet made its way through the proper channels > for sanction. > 3. The player has been sanctioned, but not banished. > 4. The director does not want to lose a paying customer, and so he > therefore does not go through channels of sanctioning. > 5. The cheater has paid his debt to bridge society, and is now back > to playing after his sanction or period of forced absence. > > I don't think it matters which of these is the case, but if it does, > please say so. I am going into this detail only because Grattan and > Richard are curious about it. > > I am also unclear as to what rights the confirmed cheater loses under > Eric's ideal director for having sinned in the past. Indeed, I cannot > yet tell what value the information of the player being a confirmed > cheat has to what a director should do (in Eric's eyes), but it is > obvious that Eric thinks it should have some effect on the wise > director's actions, for otherwise Eric would not have brought the > subject up. Only the obvious. L85 requires the TD to "determin[e] the facts... base[d on] his view on the balance of probabilities... in accordance with the weight of the evidence he is able to collect". Typically, "the evidence he is able to collect" consists of the statements made by the players at the table. Assigning "weights" to the collected pieces of verbal evidence perforce requires making judgments as to the credibility of the players supplying it. Absent genuine psychic powers, the best way to judge someone's credibility in a given situation is based on how credible he has proved himself in previous ones. Eric Landau 1107 Dale Drive Silver Spring MD 20910 ehaa at starpower.net From nigelguthrie at yahoo.co.uk Tue Jan 18 16:43:12 2011 From: nigelguthrie at yahoo.co.uk (Nigel Guthrie) Date: Tue, 18 Jan 2011 15:43:12 +0000 (GMT) Subject: [BLML] He and she, whe and ble :-) In-Reply-To: <4D35819A.4050706@skynet.be> References: <1923802510.31506.1294972945375.JavaMail.ngmail@webmail13.arcor-online.net> <4D302972.9060002@skynet.be> <58652.11845.qm@web87111.mail.ird.yahoo.com> <000f01cbb5a8$d42b1ec0$7c815c40$@com><4D340C2B.7060402@aol.com> <4D340FCC.1070806@skynet.be> <4CCF9CBA19C0473B9668C4A8E3196EAA@Mildred> <4D35819A.4050706@skynet.be> Message-ID: <644388.30087.qm@web28505.mail.ukl.yahoo.com> [Grattan] As to Herman's thought, please excuse me if I laugh. [Herman] Excuse me if I laugh too, Grattan. [Nigel] I agree with Herman rather than Grattan. Some suggestions to law-makers: 1. Simplify the laws and cut out unnecesary laws, until the *average player* can understand them and comply with them. 2. Make sure that the *average director* can *consistently* enforce each law. Drop equity and introduce an element of deterrence. Make sure that the laws rely as little as possible on subjective judgement. 3. Complete the laws, so that no local legislature needs to supplement them with local regulations. (They can still over-ride the defaults if they want to do so). 4. Try to organise the laws as a *Decision Tree* or *Flow diagram*, for easy access. So that a player (or a director) can easily understand it. 5. Keep a *Concordance* of words (computer program) as you go along. Only permit a new word if really necessary. Try to use the same word rather than go for elegant variation. 6. Check the *Dictionary*. Limit the vocabulary to simple and unambiguous words. For example, if possible use the word "must"; 7. Short sentences. Avoid the passive; Avoid negative and double-negative constructions. 8. If any word (eg serious) is used in a specific sense, define it in the *Glossary* 9. Before publishing the laws, submit a first draft to translators for ... a) Translate into local languages. b) Translate back into English c) Compare the two English versions for semantic discrepancies. d) Change the English if possible to minimise the problem. (This will also make the English version clearer). 10. Post the second draft on the web for feedback from players before publishing the final version. 11. The authoritative version should be a web-edition on the WBF site (which anybody can mirror). 12. Be prepared to produce interim web-editions if anomalies are discovered. Include a list of corrigenda in each new edition but also make the corrections in place, in the text. Like Herman's, and Grattan's ideas, my suggestions are so obvious and correct, as usual, that I expect more derision. -------------- next part -------------- An HTML attachment was scrubbed... URL: http://lists.rtflb.org/pipermail/blml/attachments/20110118/dab037f2/attachment-0001.html From ehaa at starpower.net Tue Jan 18 16:56:24 2011 From: ehaa at starpower.net (Eric Landau) Date: Tue, 18 Jan 2011 10:56:24 -0500 Subject: [BLML] Alain's case revisited In-Reply-To: References: <4D257698.6010201@skynet.be> <9647244.332577.1294302281035.JavaMail.ngmail@webmail17.arcor-online.net> <0F0A4E0E-3319-4F02-8EC6-F49CA1492683@starpower.net> Message-ID: <25C791BB-5242-4815-AC1A-6A5E08FAE079@starpower.net> On Jan 16, 2011, at 12:37 AM, Jerry Fusselman wrote: > Eric, I appreciate your thoughtful responses. Was it really six days > ago? Shame on me. Anyway, finally, I am responding. > > On Mon, Jan 10, 2011 at 3:56 PM, Eric Landau > wrote: > >> On Jan 8, 2011, at 5:00 PM, Jerry Fusselman wrote: >> >>> I want to give a clarification and request a clarification. In >>> brief, >>> I am ignoring clubs and penalty points right now, and I want to know >>> how Eric (or someone who agrees with Eric's post) would rule if his >>> innocent newbie unconsciously used the same UI to take the same >>> improper action as a conscious cheat. >>> >>> On Fri, Jan 7, 2011 at 4:11 PM, Eric Landau wrote: >>> >>>> What some people call "reading minds", others call "good >>>> communication skills", and still others, bridge club directors >>>> especially, call "knowing their customers". >>> >>> Here, and later, Eric refers to bridge club directors. I would >>> rather >>> leave clubs entirely out of the discussion: For my post, I was >>> thinking only of events run in (using ACBL names) sectionals, >>> regionals, and nationals. Almost anything can happen in clubs, and >>> frequently does. Lets face it: Clubs run by their own rules. I >>> want >>> to hear if what I said is right in the domain of sectionals, >>> regionals, and nationals, for I consider clubs a separate issue. >>> >>>> What Jerry calls "our best bridge laws" are precisely those that >>>> require us to give identical rulings to those whom we know to be >>>> innocent newbies as to confirmed cheaters. >>> >>> Eric, are you thinking that the requirement you mention here is a >>> bad >>> thing? For example, please consider the noncompetitive auction >>> >>> 1S-2S >>> 3C-...3S >>> 4S. >>> >>> I.e., there was a slow 3S signoff after the 3C game try. It gets >>> raised to 4S anyway by a hand that some players would have taken >>> straight to game (instead of 3C), but no one would tried for a slam. >>> Please also assume that the slow signoff demonstrably shows extras. >>> In the play, it easily makes ten tricks, as it should in just about >>> all cases, and both sides play it well. >>> >>> This happens at two tables---one with a confirmed cheater bidding >>> 4S, >>> and another with a confirmed innocent newbie bidding 4S. Do you want >>> the rectifications in these two cases to be the same or different? >> >> I would hope to see vast differences in the processes by which the >> rectification is determined, notwithstanding that I would expect the >> resulting rectifications to be the same in almost every case. > > Fascinating. What is the nature of these vast differences in > processes that Eric hopes for? I guess Eric covers this below. > >>> (I >>> am not asking about penalty points here, only the >>> rectification.) If >>> it helps to reach a decision, please assume that the confirmed >>> cheater >>> consciously used the delay to upgrade his hand, but the confirmed >>> innocent newbie's upgrade caused by the pause was entirely >>> unconscious. And please assume also that the director knows all of >>> this perfectly. How should he rule in the two cases if you had the >>> laws arranged the way you think best? Does my choice, +170/-170 in >>> both cases, match yours? >> >> Yes. But note that you have forced me to this position by >> stipuation: "the confirmed innocent newbie's upgrade caused by the >> pause was entirely unconscious." It is, after all, the "upgrade >> caused by the pause" that is to be rectified, "entirely unconscious" >> or not. >> >> But when it comes to the sort of understanding of the game of bridge >> that Jerry and I and I'd assume all of us take for granted, things >> are often not so clear; I have discovered in my half-century of >> competitive bridge playing that "confirmed innocent newbies" think >> and act in all manner of odd and amazing ways. So upon being >> confronted with Jerry's scenario, I would use my not insignificant >> (albeit far from perfect) "mind reading" skills by interrogating the >> newbie as to why he bid 3C, and then 4S, and listening to their >> answers, before accepting the conlusion that there was an "upgrade >> cause by the pause" that happened to be, irrelevantly, as Jerry is >> "not asking about penalty points here, only the rectification", >> "entirely unconscious". Then, having done so, I give him +170 and a >> gentle education on his responsibilities after his partner pauses at >> a critical point in the auction. But it is not beyond the realm of >> possibility, or even of my own experience, that this innocent newbie, >> unlike the confirmed cheater, could wind up convincing me that he was >> never ever stopping short of game no matter what, had his own, not >> necessarily sensible (to Jerry or me), reason for his 3C bid, and >> that, *for him*, not bidding game was not an LA. > > Does Eric mean *for him*, or for *his peers using his methods who bid > 3C*? I thought that the latter was the proper issue, not the former. > > Anyway, I cannot easily think of what would convince Eric to allow 4S > after a 3C game try in my hypothetical auction. Yes, I am assuming > that it is a game try. Which of these explanations by the innocent > newbie would Eric (or anyone else) say should lead a wise director to > allow 4S? > > 1. When I bid 3C, I forgot my distribution points, so I changed my > mind as to what my hand was worth even before LHO passed. > 2. I was always planning to go to game, and I was just using 3C to > try to confuse the opponents. We have not done this trick ever > before, but this is a big event, so I was trying new stuff. > 3. I meant to bid 3D, but I goofed, and since I was borderline anyway > for a game try, I just went to game ignoring my partner's response. > 4. My table feel told me that LHO was broke, RHO had the trapped > honors, and the cards were likely to lie well for our side, so I went > to game based on table feel. > 5. Some other explanation that would talk a wise director into > allowing 4S. > > Please assume that each explanation is honest and that the director > somehow knows this. I am really curious as to what it would take for > a wise director to allow 4S, given that 3C was a game try and 3S was a > sign off. Probably one example will satisfy me. Impossible by stipulation. "Given that 3C was a game try" (presumably so intended), we must conclude that the player was trying for game. That's no different from Jerry's previous stipulation of "an upgrade caused by the pause", and mandates a finding of infraction. But if we refrain from building the conlusion into the probem, we might hear something like... "Our bridge teacher taught us that when we have two long suits, we should always try to bid them both, unless bidding the second one will get us too high. Since partner promised 13 points and I had 14 points, I knew we had more than 26 points and had to bid game no matter what, so I knew 3C wouldn't get us too high." >>>> And because our clubs >>>> (and our SOs) are businesses, that means being far too tolerant to >>>> the cheaters rather than driving the innocent infractors away (and >>>> having to listen to those in this forum who constantly carp >>>> about how >>>> our laws let people get away with things they shouldn't). Only the >>>> real-life lawyers among our players seem to actually understand >>>> that >>>> subtle bit about being punished for doing just what a cheater would >>>> do but, of course, we're not calling you cheaters, we'd never >>>> ever do >>>> that, no sirree, so do please come back next week. >>>> >>>> If we had genuinely psychic directors who actually could read >>>> players' minds, wouldn't we want to write our laws to take >>>> advantage >>>> of those skills, and expect our game to be better for it? If so, >>>> shouldn't we work to hone our communications skills and our >>>> knowledge >>>> of our customers, write laws that take advantage of those >>>> (admittedly >>>> somewhat less impressive, but not entirely useless) skills, and do >>>> our best to come as close as we can to our ideal of perfection? >>> >>> I plan to respond to this later. I suspect that we have a >>> conflict of >>> visions about directors and players. >> >> I suspect more a conflict of interpretation of all of those "could >> have"s and "might have"s scattered throughout TFLB. Do we take those >> as determinations of conceivability (possibly leading to, "He didn't, >> but he could have, so we rule against him") or as determinations of >> finite probability (possibly leading to, "He could have, but he >> didn't, so we rule in his favor")? Jerry seems to favor the former >> interpretation, while I would prefer the latter, notwithstanding the >> difficulty of operationalizing "he didn't" for us non-psychics. > > Yes, Eric certainly has me pegged here. Indeed, I thought that the > former was the only valid way to interpret the law book. My guess is > that Grattan would not consider it bridge to do it the latter way. I > am quite surprised with Eric's choice, given what he has written in > the past about the wisdom of good, objective, predictable procedures. > I am very interested to hear what others think, too. The problem with the a priori interpretation is that it means that any time you perform an action that results in a particular outcome you "could have" known that that action would produce that outcome, by logical necessity, which renders the phrase meaningless as an operative criterion. If we interpret it as a determination of conceivability, we can, for example, strike "and who could have known, at the time of the action, that the action could work to his benefit" from L73F without any substantive effect on its application -- you *could* have known at the time that the action you took *could* work to your benefit any time it did, by definition. Constructions like that require an a posteriori interpretation of "could have" to avoid being meaningless. Eric Landau 1107 Dale Drive Silver Spring MD 20910 ehaa at starpower.net From ehaa at starpower.net Tue Jan 18 17:24:11 2011 From: ehaa at starpower.net (Eric Landau) Date: Tue, 18 Jan 2011 11:24:11 -0500 Subject: [BLML] He and she, whe and ble :-) In-Reply-To: <4D359DBA.1030903@aol.com> References: <1923802510.31506.1294972945375.JavaMail.ngmail@webmail13.arcor-online.net> <4D302972.9060002@skynet.be> <58652.11845.qm@web87111.mail.ird.yahoo.com> <000f01cbb5a8$d42b1ec0$7c815c40$@com><4D340C2B.7060402@aol.com> <4D340FCC.1070806@skynet.be> <4CCF9CBA19C0473B9668C4A8E3196EAA@Mildred> <4D35819A.4050706@skynet.be> <4D359DBA.1030903@aol.com> Message-ID: On Jan 18, 2011, at 9:03 AM, Jeff Easterson wrote: > I suspect that substituting "serious" for "egregious" might cause > problems: lead to complications, not exactly express the same meaning. > Or is my memory of the meanings (not meaning) of these words > inaccurate? It not only might cause problems, it already has. Widespread misinterpretation has forced the WBF to issue one of its infamous "leopard-loo" minutes emphasizing that "serious" is be read as meaning something like "outstandingly bad, blatant, outrageous". As it happens, that is precisely the definition of "egregious" in The American Heritage Dictionary. Eric Landau 1107 Dale Drive Silver Spring MD 20910 ehaa at starpower.net From rfrick at rfrick.info Tue Jan 18 17:26:59 2011 From: rfrick at rfrick.info (Robert Frick) Date: Tue, 18 Jan 2011 11:26:59 -0500 Subject: [BLML] In the trenches. In-Reply-To: References: <1236200011.166424.1295292195352.JavaMail.ngmail@webmail08.arcor-online.net> <4D34B958.9070602@gmail.com> Message-ID: On Mon, 17 Jan 2011 21:16:05 -0500, Jerry Fusselman wrote: > +=+ Or a gold star for invention? +=+ > > That's a good one, Grattan. > > I think Robert believes that what a person says he would do in a > hypothetical situation is not always a reliable indicator of what he > would actually do. This was emphasized in my first year of graduate > study in economics, and I agree with it. Its implications for ideal > directing seem unclear to me. Probably that is why Robert brings it > to our attention: He wants to explore this issue. It could be > interesting. You said it more clearly than I was thinking. But when I actually pulled out the book to look at the exact wording, I discovered that the laws about mistaken explanations clearly differentiate changing a call and calls that cannot be changed. Adjusted scores apply only to the second, not to calls that could have been changed. Um, the instruction to presume ME instead of mistaken call applies only the first case. I think that could be rewritten to better correspond to how everyone understands it and L75. From adam at irvine.com Tue Jan 18 17:29:19 2011 From: adam at irvine.com (Adam Beneschan) Date: Tue, 18 Jan 2011 08:29:19 -0800 Subject: [BLML] He and she, whe and ble :-) In-Reply-To: Your message of "Tue, 18 Jan 2011 11:24:11 EST." Message-ID: <20110118162920.84943A8C883@mailhub.irvine.com> Eric wrote: > On Jan 18, 2011, at 9:03 AM, Jeff Easterson wrote: > > > I suspect that substituting "serious" for "egregious" might cause > > problems: lead to complications, not exactly express the same meaning. > > Or is my memory of the meanings (not meaning) of these words > > inaccurate? > > It not only might cause problems, it already has. Widespread > misinterpretation has forced the WBF to issue one of its infamous > "leopard-loo" minutes emphasizing that "serious" is be read as > meaning something like "outstandingly bad, blatant, outrageous". I'll keep that in mind next time an opponent tells me they play "serious 3NT". :) :) :) They definitely picked the wrong word. -- Adam From rfrick at rfrick.info Tue Jan 18 17:39:43 2011 From: rfrick at rfrick.info (Robert Frick) Date: Tue, 18 Jan 2011 11:39:43 -0500 Subject: [BLML] In the trenches. In-Reply-To: <4D34B958.9070602@gmail.com> References: <1236200011.166424.1295292195352.JavaMail.ngmail@webmail08.arcor-online.net> <4D34B958.9070602@gmail.com> Message-ID: On Mon, 17 Jan 2011 16:49:12 -0500, Hirsch Davis wrote: > On 1/17/2011 2:23 PM, Thomas Dehn wrote: >> Robert Frick wrote: >>> The yesterday auction was >>> S W N E >>> 1NT 2C X 2S >>> X 3H ? >>> >>> At this point NS discover that 2C is "Cappelletti", unalerted even >>> though >>> ACBL regulations call for an alert if 2C is not natural. It is >>> described >>> (by the 2C bidder! but pd agrees) as showing both majors. 2S is simply >>> a >>> foolish bid, she has 10xxxx of spades. >>> >>> In practice, North shot out a 5C bid for a bottom, as they belong in >>> 3NT. >> This confuses me. >> >> The director is called. The director opens TFLB and reads >> L21B1(a). He asks S whether he wants to change his X over 2S. >> >> >> >> Thomas > > Yes, but incomplete. Was N aware that the call was Cappelleti? If not, > would N still have doubled 2C? The N double is also compromised by the > MI, and the auction cannot be reversed to that point, so an adjustment > might still be necessary. From the information provided, it appears that > N might have thought he was making a penalty double of a natural 2C. > Would double have a different meaning over an artificial call? Since the > TD does not appear to have investigated fully, it's not clear. > > Not clear why the TD thinks 2S is foolish opposite a call showing > majors. With five-card support for one of partner's suits, the only > issue might be that the call was too low. > > Final ruling is not clear due to absence of relevant information. TD > deserves a procedural penalty for sloppy investigation. Sorry, I didn't know anyone would find the rest of the ruling interesting. It seemed murky and subjective but not particularly interesting to me from a point of law. Without the mistaken explanation, North said she would have passed. I accepted that as true. So with the ME she got to show her hand -- clubs and points. Without it, she didn't. So, short story, the ME helped her. Because she could show her points, South was now in a position to take a call rather than pass. So she too could show her hand. So North was much better placed to find the best contract instead of shooting out a guess. She shot out a guess anyway and guessed wrong. I couldn't find any damage from the ME. She said she would have doubled 3H for takeout. I didn't accept that, and she didn't seem so confident that her bid was takeout when I challenged her. And if it is takeout, she could have done that anyway. They claimed that if East bid 2D liked she was supposed to (2C just showed one major), then West would be bidding 2H and North would have room to act effectively. So they were damaged by the bidding. I ruled that I couldn't rectify for bad bidding. I wrote her an email explaining that and she seems okay with the ruling now. West had a pretty obvious pull to 3H. He had a singleton spade and six hearts to the AQ, and he had just heard a penalty double of 2S. He also had a partner who did not do what she was supposed to do and hence whose bid is suspicious. I realize that in expert circles, 2S shows a hand willing to play spades even if partner has a heart suit. But we were not close to expert circles, we are talking about people who couldn't execute Cappelletti. In fact, she thought that she had to bid (she didn't) and didn't realize she was supposed to bid 2D. From rfrick at rfrick.info Tue Jan 18 17:52:16 2011 From: rfrick at rfrick.info (Robert Frick) Date: Tue, 18 Jan 2011 11:52:16 -0500 Subject: [BLML] Ecclesiastes (proposed change in law) In-Reply-To: References: <4D302790.8050401@skynet.be> <1748200137.98748.1295002698502.JavaMail.ngmail@webmail08.arcor-online.net> <4D304E50.7080502@ulb.ac.be> <4D34582E.8070509@nhcc.net> <4D34598B.4030602@ulb.ac.be> Message-ID: I think we should add to the laws that there is no rectification for a correct description of partners hand. It's simple. It makes sense. It's fair. What more can you ask for in a law? In the classic example where this was violated, I think the only justification for the ruling was that it followed the law. My vague memory is that Kaplan faulted the ruling on every other criteria. But maybe that was somewhere else. In any case, I can't think of any situation where we would actually want to rectify for a correct description of partner's hand. For example, I describe my partner's bid as showing a singleton when in fact it could be a void. My partner has a singleton. The opps claim they would have done something different if they had known my partner could have a void. In the long run, they gain from the mistaken explanation -- they get better information, and they get protection if there is a void. It is just "rub of the green" that they were damaged, and the laws can handle that conceptually. Then dWS becomes legal, but violations seem to be unpunishable (players are just guessing what to do) and I can't see how bridge would be a worse game if dWS wasn't punished. Jerry asked me offlist what I care about in the laws. I would like to have laws I was not embarassed to enforce. From grabiner at alumni.princeton.edu Wed Jan 19 01:35:30 2011 From: grabiner at alumni.princeton.edu (David Grabiner) Date: Tue, 18 Jan 2011 19:35:30 -0500 Subject: [BLML] Ecclesiastes (proposed change in law) In-Reply-To: References: <4D302790.8050401@skynet.be> <1748200137.98748.1295002698502.JavaMail.ngmail@webmail08.arcor-online.net> <4D304E50.7080502@ulb.ac.be> <4D34582E.8070509@nhcc.net> <4D34598B.4030602@ulb.ac.be> Message-ID: <77ABDD9E2DFD40D595E9B486A9A91E87@erdos> Consider the following: RHO opens 1NT with a 15 count, and it is explained as 15-17. The correct agreement is 13-15. I would have doubled a 13-15 1NT but do not double the 15-17 1NT, either because I need a slightly stronger hand or because my agreement is that a double of a 15-17 1NT is not penalty. Given the MI, I pass and we beat 1NT two tricks undoubled for +100 and a bad score; a penalty double would have given us +300. Am I entitled to adjustment? At the very least, you can only deny rectification if the correct explanation is less specific than the actual explanation, and no secondary inferences can be drawn. ----- Original Message ----- From: "Robert Frick" To: "Bridge Laws Mailing List" Sent: Tuesday, January 18, 2011 11:52 AM Subject: Re: [BLML] Ecclesiastes (proposed change in law) >I think we should add to the laws that there is no rectification for a > correct description of partners hand. > > It's simple. It makes sense. It's fair. What more can you ask for in a law? > > In the classic example where this was violated, I think the only > justification for the ruling was that it followed the law. My vague memory > is that Kaplan faulted the ruling on every other criteria. But maybe that > was somewhere else. In any case, I can't think of any situation where we > would actually want to rectify for a correct description of partner's hand. > > For example, I describe my partner's bid as showing a singleton when in > fact it could be a void. My partner has a singleton. The opps claim they > would have done something different if they had known my partner could > have a void. In the long run, they gain from the mistaken explanation -- > they get better information, and they get protection if there is a void. > It is just "rub of the green" that they were damaged, and the laws can > handle that conceptually. > > Then dWS becomes legal, but violations seem to be unpunishable (players > are just guessing what to do) and I can't see how bridge would be a worse > game if dWS wasn't punished. Jerry asked me offlist what I care about in > the laws. I would like to have laws I was not embarassed to enforce. > _______________________________________________ > Blml mailing list > Blml at rtflb.org > http://lists.rtflb.org/mailman/listinfo/blml > From ardelm at optusnet.com.au Wed Jan 19 04:10:05 2011 From: ardelm at optusnet.com.au (Tony Musgrove) Date: Wed, 19 Jan 2011 14:10:05 +1100 Subject: [BLML] Ecclesiastes (proposed change in law) In-Reply-To: References: <4D302790.8050401@skynet.be> <1748200137.98748.1295002698502.JavaMail.ngmail@webmail08.arcor-online.net> <4D304E50.7080502@ulb.ac.be> <4D34582E.8070509@nhcc.net> <4D34598B.4030602@ulb.ac.be> Message-ID: <201101190310.p0J3AJ8h029070@mail01.syd.optusnet.com.au> At 03:52 AM 19/01/2011, you wrote: >I think we should add to the laws that there is no rectification for a >correct description of partners hand. > >It's simple. It makes sense. It's fair. What more can you ask for in a law? > >In the classic example where this was violated, I think the only >justification for the ruling was that it followed the law. My vague memory >is that Kaplan faulted the ruling on every other criteria. But maybe that >was somewhere else. In any case, I can't think of any situation where we >would actually want to rectify for a correct description of partner's hand. > >For example, I describe my partner's bid as showing a singleton when in >fact it could be a void. My partner has a singleton. The opps claim they >would have done something different if they had known my partner could >have a void. In the long run, they gain from the mistaken explanation -- >they get better information, and they get protection if there is a void. >It is just "rub of the green" that they were damaged, and the laws can >handle that conceptually. > >Then dWS becomes legal, but violations seem to be unpunishable (players >are just guessing what to do) and I can't see how bridge would be a worse >game if dWS wasn't punished. Jerry asked me offlist what I care about in >the laws. I would like to have laws I was not embarassed to enforce. What about the one where you give both sides the "feel good" 60% because that is what the club manager wants? Cheers, Tony (Sydney) >_______________________________________________ >Blml mailing list >Blml at rtflb.org >http://lists.rtflb.org/mailman/listinfo/blml From jfusselman at gmail.com Wed Jan 19 06:41:56 2011 From: jfusselman at gmail.com (Jerry Fusselman) Date: Tue, 18 Jan 2011 23:41:56 -0600 Subject: [BLML] Question on the Beijing minute of 2008 on LAW 20 Message-ID: The 2008 Beijing minute relating to LAW 20 is this: "There is no infraction when a correct explanation discloses that partner?s prior explanation was mistaken. The words ?nor may he indicate in any manner that a mistake has been made? (in Law 20F5(a)) do not refer to compliance with the overriding requirement of the laws always to respond to enquiries under Law 20F with correct explanations of the partnership understandings." It appears on the last page of http://www.ecatsbridge.com/Documents/files/WBFInformation/Reports_Minutes/LawsCommitteeMinutes/2008-Beijing.pdf But how is this minute to be implemented in practice? I think the minute covers a lot less ground than is widely believed. For weeks, I have been thinking about how to ask this question most efficiently, and because it ties in somewhat with Robert Frick's post today, I thought I might as well submit my question now. Believe it or not, this is a shortened version: Suppose the auction is 1S - (X) - 1NT* - (P) 2C. In this hypothetical auction, suppose that our partnership understanding is one of these: Understanding A. 1NT 6-9 HCP, one or two spades, and no long suit. 2C next in this case would be nonforcing with long clubs. Understanding B. 1NT is the forcing notrump with systems on as if the doubler had passed. 2C next in this case could be a balanced minimum hand with only two clubs (5332 shape). I was the one bidding 1NT and I have to explain 2C, because the opponents ask about it. When I chose to bid 1NT, I was thinking that we were playing A, and indeed, I have a balanced hand with two spades and 8 HCP. But partner alerted 1NT, and when asked, explained that 1NT is "a forcing NT, just as if there had been no double." Beijing enters the picture because next, I explained to the opponents that 2C shows "at least two clubs, possibly with a balanced minimum." My punishment depends on why I said that. I will give several alternate scenarios, and since mind reading must be involved, my examples make it easier for you (the director) to rule, because I state here exactly what is on my mind in each alternate case. In this entire email, please assume that the director knows that I know the rules and the Beijing minute. Also, the director can determine precisely what I am thinking, and he knows that he can. And I know that he can, etc.. Under Beijing, if I am sure that we are operating under understanding A, then my explanation about 2C was MI, and I am due for a PP as well as a potential rectification *as* *if* I had provided UI to partner by saying "he has at least five clubs." Correct? What follows makes sense only if I am right so far. We could call that case 0, where I am 100% sure that A (rather than B) is our understanding. In case 0, I am due for a PP as well as an assume-UI-even-though-there-was-none rectification. If I am not due for at least one of these, then the Beijing minute has no teeth. Though the Beijing minute covers case 0, it appears to me to say nothing about alternate cases 1--11 below. (Remember, in the auction, I initially thought that we were playing A, but I explained partner's 2C as if we were playing B.) We are discussing here my state of mind the instant before my explaining what 2C showed. Here are the alternate cases: 1. I am 90% sure our understanding is A. Probably pard forgot. But since I am unsure as to what our understanding is, I can explain it either way. 2. I am 50% sure it is A. Since it is a tossup, I can explain it either way. 3. I am not sure whether it is A or B, and I cannot give a number, for I simply do not believe in subjective probability. Since I am unsure as to what our understanding is, I can explain it either way. 4. Our system notes say it is A, but we decided this morning to go with B tonight to make things easier. Until I heard 2C, I had forgotten about that. 5. Our system notes say it is B, but we decided this morning to go with A tonight. Pard just forgot, but at least our system notes say B, so that might be good enough. 6. Our system notes say it is A, but we have not played together in so long that naturally, partner forgot this stuff. And since his forgetting should have been anticipated by me, our true understanding must be B. 7. Our system notes say it is B, but we have not played together in so long that naturally, I forgot this stuff. The system notes are objective, so I can safely go with that. 8. In my gut I feel certain that our understanding is A, but my head reminds me that most of the times in the past in which I have been sure that partner erred, it turned out that partner was the one that was right. On that basis, I am assuming that B is right, even though I can offer no evidence other than the reasoning I just stated. 9. Our understanding is A (for sure), but I accidentally said it was B. Just a slip of the tongue that might happen to anyone while under the gun in a timed game with a sudden injection of confusion. 10. I feel that our system is A, but I also feel that the director will rule that it is B, and since the director is always the one who determines what our system is under the law, I used B to follow the Beijing minute. 11. I feel that our system is B. Despite my expectation that director will rule that it is A, the Beijing minute does not ask me to take into account my beliefs as to what the director will decide is our system, so I explain it as B. So this is my question: Which of these 11 cases are violations of the Beijing minute, and which are not? Of course there may well be UI and MI considerations as well later, but I want to focus now specifically on is what the Beijing minute tells us in these cases. Which of these 11 is punishable or rectifiable using a PP or an assume-UI-even-though-there-was-none rectification of the kind authorized or required by the Beijing minute? -------------- next part -------------- An HTML attachment was scrubbed... URL: http://lists.rtflb.org/pipermail/blml/attachments/20110119/bc70e19f/attachment.html From jfusselman at gmail.com Wed Jan 19 06:43:59 2011 From: jfusselman at gmail.com (Jerry Fusselman) Date: Tue, 18 Jan 2011 23:43:59 -0600 Subject: [BLML] Ecclesiastes (proposed change in law) In-Reply-To: <201101190310.p0J3AJ8h029070@mail01.syd.optusnet.com.au> References: <4D302790.8050401@skynet.be> <1748200137.98748.1295002698502.JavaMail.ngmail@webmail08.arcor-online.net> <4D304E50.7080502@ulb.ac.be> <4D34582E.8070509@nhcc.net> <4D34598B.4030602@ulb.ac.be> <201101190310.p0J3AJ8h029070@mail01.syd.optusnet.com.au> Message-ID: On Tue, Jan 18, 2011 at 9:10 PM, Tony Musgrove wrote: > > What about the one where you give both sides the "feel good" > 60% because that is what the club manager wants? > I am afraid that this reply may be over my head. Care to restate? From blml at arcor.de Wed Jan 19 07:17:37 2011 From: blml at arcor.de (Thomas Dehn) Date: Wed, 19 Jan 2011 07:17:37 +0100 (CET) Subject: [BLML] Ecclesiastes (proposed change in law) In-Reply-To: References: <4D302790.8050401@skynet.be> <1748200137.98748.1295002698502.JavaMail.ngmail@webmail08.arcor-online.net> <4D304E50.7080502@ulb.ac.be> <4D34582E.8070509@nhcc.net> <4D34598B.4030602@ulb.ac.be> <201101190310.p0J3AJ8h029070@mail01.syd.optusnet.com.au> Message-ID: <1615417801.255813.1295417857689.JavaMail.ngmail@webmail09.arcor-online.net> Jerry Fusselman wrote: > On Tue, Jan 18, 2011 at 9:10 PM, Tony Musgrove wrote: > > > > What about the one where you give both sides the "feel good" > > 60% because that is what the club manager wants? > > > > I am afraid that this reply may be over my head. Care to restate? Robert sometimes gives illegal A+/A+ scores after an infraction, because he thinks it is good for business. Thomas From jfusselman at gmail.com Wed Jan 19 07:21:14 2011 From: jfusselman at gmail.com (Jerry Fusselman) Date: Wed, 19 Jan 2011 00:21:14 -0600 Subject: [BLML] Ecclesiastes (proposed change in law) In-Reply-To: References: <4D302790.8050401@skynet.be> <1748200137.98748.1295002698502.JavaMail.ngmail@webmail08.arcor-online.net> <4D304E50.7080502@ulb.ac.be> <4D34582E.8070509@nhcc.net> <4D34598B.4030602@ulb.ac.be> Message-ID: On Tue, Jan 18, 2011 at 10:52 AM, Robert Frick wrote: > I think we should add to the laws that there is no rectification for a > correct description of partners hand. Though it is probably my fault, I don't fully understand this. For example, in a dWS scenario, would the MI be wiped out as if it were not present? Or would it simply mean that choosing an answer that causes MI as opposed to UI is an option for the explainer? > > It's simple. It makes sense. It's fair. What more can you ask for in a law? Crystal clarity would be nice too. I'm a simple man. > > In the classic example where this was violated, I think the only > justification for the ruling was that it followed the law. My vague memory > is that Kaplan faulted the ruling on every other criteria. But maybe that > was somewhere else. In any case, I can't think of any situation where we > would actually want to rectify for a correct description of partner's hand. > > For example, I describe my partner's bid as showing a singleton when in > fact it could be a void. My partner has a singleton. The opps claim they > would have done something different if they had known my partner could > have a void. In the long run, they gain from the mistaken explanation -- > they get better information, and they get protection if there is a void. > It is just "rub of the green" that they were damaged, and the laws can > handle that conceptually. > > Then dWS becomes legal, but violations seem to be unpunishable (players > are just guessing what to do) and I can't see how bridge would be a worse > game if dWS wasn't punished. Jerry asked me offlist what I care about in > the laws. I would like to have laws I was not embarassed to enforce. > Interesting how this idea would handle the question I just asked an hour or two ago about the Beijing minute of 2008 on LAW 20. Under Robert's proposed rule change, answering my question would trivial. All cases are allowed, even case 0. For anyone who does not like this rule change, am I safe in assuming that you approve of the Beijing minute and its requirement for mind reading (or whatever else you want to call it)? I suggest that the kind of mind reading called for in the Beijing minute is beyond the capabilities of humans to do well. And even for those rare human directors who can read minds with high precision, it is not clear to me what they would do with their knowledge. After all, a mind can be a messy thing. Can someone who understands the Beijing minute answer my scenarios 1--11 (yes/no/maybe) in a systematic way that is likely to match what other directors with similar mind-reading skills would do? Or does the Beijing minute reduce to guesswork and random rulings even when you are lucky enough to be able to read minds with high precision? I am honestly curious. Jerry Fusselman From jfusselman at gmail.com Wed Jan 19 07:21:14 2011 From: jfusselman at gmail.com (Jerry Fusselman) Date: Wed, 19 Jan 2011 00:21:14 -0600 Subject: [BLML] Ecclesiastes (proposed change in law) In-Reply-To: References: <4D302790.8050401@skynet.be> <1748200137.98748.1295002698502.JavaMail.ngmail@webmail08.arcor-online.net> <4D304E50.7080502@ulb.ac.be> <4D34582E.8070509@nhcc.net> <4D34598B.4030602@ulb.ac.be> Message-ID: On Tue, Jan 18, 2011 at 10:52 AM, Robert Frick wrote: > I think we should add to the laws that there is no rectification for a > correct description of partners hand. Though it is probably my fault, I don't fully understand this. For example, in a dWS scenario, would the MI be wiped out as if it were not present? Or would it simply mean that choosing an answer that causes MI as opposed to UI is an option for the explainer? > > It's simple. It makes sense. It's fair. What more can you ask for in a law? Crystal clarity would be nice too. I'm a simple man. > > In the classic example where this was violated, I think the only > justification for the ruling was that it followed the law. My vague memory > is that Kaplan faulted the ruling on every other criteria. But maybe that > was somewhere else. In any case, I can't think of any situation where we > would actually want to rectify for a correct description of partner's hand. > > For example, I describe my partner's bid as showing a singleton when in > fact it could be a void. My partner has a singleton. The opps claim they > would have done something different if they had known my partner could > have a void. In the long run, they gain from the mistaken explanation -- > they get better information, and they get protection if there is a void. > It is just "rub of the green" that they were damaged, and the laws can > handle that conceptually. > > Then dWS becomes legal, but violations seem to be unpunishable (players > are just guessing what to do) and I can't see how bridge would be a worse > game if dWS wasn't punished. Jerry asked me offlist what I care about in > the laws. I would like to have laws I was not embarassed to enforce. > Interesting how this idea would handle the question I just asked an hour or two ago about the Beijing minute of 2008 on LAW 20. Under Robert's proposed rule change, answering my question would trivial. All cases are allowed, even case 0. For anyone who does not like this rule change, am I safe in assuming that you approve of the Beijing minute and its requirement for mind reading (or whatever else you want to call it)? I suggest that the kind of mind reading called for in the Beijing minute is beyond the capabilities of humans to do well. And even for those rare human directors who can read minds with high precision, it is not clear to me what they would do with their knowledge. After all, a mind can be a messy thing. Can someone who understands the Beijing minute answer my scenarios 1--11 (yes/no/maybe) in a systematic way that is likely to match what other directors with similar mind-reading skills would do? Or does the Beijing minute reduce to guesswork and random rulings even when you are lucky enough to be able to read minds with high precision? I am honestly curious. Jerry Fusselman From grandaeval at tiscali.co.uk Wed Jan 19 11:22:48 2011 From: grandaeval at tiscali.co.uk (Grattan) Date: Wed, 19 Jan 2011 10:22:48 -0000 Subject: [BLML] Ecclesiastes (proposed change in law) References: <4D302790.8050401@skynet.be><1748200137.98748.1295002698502.JavaMail.ngmail@webmail08.arcor-online.net><4D304E50.7080502@ulb.ac.be> <4D34582E.8070509@nhcc.net><4D34598B.4030602@ulb.ac.be> Message-ID: <392B56614DA74455BF35622B7FB77BAD@Mildred> Grattan Endicott To: "Bridge Laws Mailing List" Sent: Tuesday, January 18, 2011 4:52 PM Subject: Re: [BLML] Ecclesiastes (proposed change in law) >I think we should add to the laws that there is no rectification for a > correct description of partners hand. > > It's simple. It makes sense. It's fair. What more can you ask for in a > law? > > In the classic example where this was violated, I think the only > justification for the ruling was that it followed the law. My vague memory > is that Kaplan faulted the ruling on every other criteria. But maybe that > was somewhere else. In any case, I can't think of any situation where we > would actually want to rectify for a correct description of partner's > hand. > > For example, I describe my partner's bid as showing a singleton when in > fact it could be a void. My partner has a singleton. The opps claim they > would have done something different if they had known my partner could > have a void. In the long run, they gain from the mistaken explanation -- > they get better information, and they get protection if there is a void. > It is just "rub of the green" that they were damaged, and the laws can > handle that conceptually. > > Then dWS becomes legal, but violations seem to be unpunishable (players > are just guessing what to do) and I can't see how bridge would be a worse > game if dWS wasn't punished. Jerry asked me offlist what I care about in > the laws. I would like to have laws I was not embarassed to enforce. > +=+ There was a Venice Cup appeal in which a correct description of the hand was given; the partnership understanding was different. The player on lead claimed it affected her opening lead. Kaplan argued forcefully that there was MI and that the score must be adjusted. And so it was. All to do with how many Aces were shown. I believe it was USA v. India. ~ Grattan ~ +=+ From Hermandw at skynet.be Wed Jan 19 11:57:32 2011 From: Hermandw at skynet.be (Herman De Wael) Date: Wed, 19 Jan 2011 11:57:32 +0100 Subject: [BLML] Ecclesiastes (proposed change in law) In-Reply-To: <392B56614DA74455BF35622B7FB77BAD@Mildred> References: <4D302790.8050401@skynet.be><1748200137.98748.1295002698502.JavaMail.ngmail@webmail08.arcor-online.net><4D304E50.7080502@ulb.ac.be> <4D34582E.8070509@nhcc.net><4D34598B.4030602@ulb.ac.be> <392B56614DA74455BF35622B7FB77BAD@Mildred> Message-ID: <4D36C39C.9080607@skynet.be> Grattan wrote: > > > Grattan Endicott> > +=+ There was a Venice Cup appeal in which a correct description of the > hand was given; the partnership understanding was different. The player > on lead claimed it affected her opening lead. Kaplan argued forcefully > that there was MI and that the score must be adjusted. And so it was. > All to do with how many Aces were shown. I believe it was USA v. India. > ~ Grattan ~ > +=+ > One small question, Grattan: You knew Edgar, so you might shed some light on this. Which way was the ruling, and would that have influenced Edgar as an American? -- Herman De Wael Wilrijk Antwerpen Belgium From ehaa at starpower.net Wed Jan 19 15:24:54 2011 From: ehaa at starpower.net (Eric Landau) Date: Wed, 19 Jan 2011 09:24:54 -0500 Subject: [BLML] Ecclesiastes (proposed change in law) In-Reply-To: References: <4D302790.8050401@skynet.be> <1748200137.98748.1295002698502.JavaMail.ngmail@webmail08.arcor-online.net> <4D304E50.7080502@ulb.ac.be> <4D34582E.8070509@nhcc.net> <4D34598B.4030602@ulb.ac.be> Message-ID: On Jan 19, 2011, at 1:21 AM, Jerry Fusselman wrote: > On Tue, Jan 18, 2011 at 10:52 AM, Robert Frick > wrote: > >> I think we should add to the laws that there is no rectification >> for a >> correct description of partners hand. > > Though it is probably my fault, I don't fully understand this. For > example, in a dWS scenario, would the MI be wiped out as if it were > not present? Or would it simply mean that choosing an answer that > causes MI as opposed to UI is an option for the explainer? > >> It's simple. It makes sense. It's fair. What more can you ask for >> in a law? > > Crystal clarity would be nice too. I'm a simple man. > >> In the classic example where this was violated, I think the only >> justification for the ruling was that it followed the law. My >> vague memory >> is that Kaplan faulted the ruling on every other criteria. But >> maybe that >> was somewhere else. In any case, I can't think of any situation >> where we >> would actually want to rectify for a correct description of >> partner's hand. >> >> For example, I describe my partner's bid as showing a singleton >> when in >> fact it could be a void. My partner has a singleton. The opps >> claim they >> would have done something different if they had known my partner >> could >> have a void. In the long run, they gain from the mistaken >> explanation -- >> they get better information, and they get protection if there is a >> void. >> It is just "rub of the green" that they were damaged, and the laws >> can >> handle that conceptually. >> >> Then dWS becomes legal, but violations seem to be unpunishable >> (players >> are just guessing what to do) and I can't see how bridge would be >> a worse >> game if dWS wasn't punished. Jerry asked me offlist what I care >> about in >> the laws. I would like to have laws I was not embarassed to enforce. > > Interesting how this idea would handle the question I just asked an > hour or two ago about the Beijing minute of 2008 on LAW 20. Under > Robert's proposed rule change, answering my question would trivial. > All cases are allowed, even case 0. > > For anyone who does not like this rule change, am I safe in assuming > that you approve of the Beijing minute and its requirement for mind > reading (or whatever else you want to call it)? > > I suggest that the kind of mind reading called for in the Beijing > minute is beyond the capabilities of humans to do well. And even for > those rare human directors who can read minds with high precision, it > is not clear to me what they would do with their knowledge. After > all, a mind can be a messy thing. Can someone who understands the > Beijing minute answer my scenarios 1--11 (yes/no/maybe) in a > systematic way that is likely to match what other directors with > similar mind-reading skills would do? Or does the Beijing minute > reduce to guesswork and random rulings even when you are lucky enough > to be able to read minds with high precision? I am honestly curious. Several people have cited as a flaw in the Beijing interpretation that it gives no guidance on how to act when one is uncertain as to what one's actual partnership agreements are. The proposed solution is to simply describe the meaning of partner's call as whatever he thinks it is, rather than what it would be by virtue of one's actual agreements. But that presupposes that one is in a position where you aren't sure about your agreements, but are 100% sure as to your partner's intention. One could just as easily postulate that one is certain of one's agreements, then ask Jerry's 12 questions about perceived differences in one's certainty regarding partner's intention. The Beijing interpretation is problematic when you aren't 100% sure about your actual agreements. The "dWS" alternative is problematic when you aren't 100% sure about partner's intention. Neither expectation can be met to perfection, but one would think it more difficult to come close to the latter than to the former, especially those that are quick to deprecate "mind reading" when done by the TD: Should we not have similar qualms about expecting "mind reading" of partners by players at the table? Eric Landau 1107 Dale Drive Silver Spring MD 20910 ehaa at starpower.net From nigelguthrie at yahoo.co.uk Wed Jan 19 15:43:13 2011 From: nigelguthrie at yahoo.co.uk (Nigel Guthrie) Date: Wed, 19 Jan 2011 14:43:13 +0000 (GMT) Subject: [BLML] Question on the Beijing minute of 2008 on LAW 20 In-Reply-To: References: Message-ID: <439152.51716.qm@web28503.mail.ukl.yahoo.com> Great stuff Jerry. Disclosure laws and UI laws are too sophisticated. I don't know the answers to your questions. A typical player would have no clue. That is the *central issue*, upon which law-makers should focus: If you make laws too complex for most people to understand, then how do you expect players to comply with them? or directors to enforce them? Almost always, in practice, without screens, the UI wakes a partnership up to a misexplanation or misbid. If they can get away with it, most pairs seem to use this unauthorised information. Players seem to find this easy to rationalise. (Ignorance and the confusion over these laws is almost universal). Some law-makers and directors imagine that they can read players' minds but that, too, may be wishful thinking. Anyway, in Jerry's kind of scenario, use of UI is rarely penalised. Nevertheless, it will be fascinating to read directors' replies to Jerry's questions. Few of those who closely study the law, seem to understand it. If any do understand the law, fewer still will be capable of the mental contortions necessary to comply with it or to enforce it. The Emperor wears too many badly fitting clothes but still walks about half-naked. Unfortunately, few directors or administrators are prepared to stick their heads above the parapet to point this out. -------------- next part -------------- An HTML attachment was scrubbed... URL: http://lists.rtflb.org/pipermail/blml/attachments/20110119/43941356/attachment.html From nigelguthrie at yahoo.co.uk Wed Jan 19 16:20:28 2011 From: nigelguthrie at yahoo.co.uk (Nigel Guthrie) Date: Wed, 19 Jan 2011 15:20:28 +0000 (GMT) Subject: [BLML] Unsure In-Reply-To: References: <4D302790.8050401@skynet.be> <1748200137.98748.1295002698502.JavaMail.ngmail@webmail08.arcor-online.net> <4D304E50.7080502@ulb.ac.be> <4D34582E.8070509@nhcc.net> <4D34598B.4030602@ulb.ac.be> Message-ID: <580906.17257.qm@web28513.mail.ukl.yahoo.com> [Nigel] Many BLMLers are certain about law-makers' intentions. Such players may also be certain of their partnership agreements. I've some beliefs but I'm uncertain about most things. I'm *never* certain of any Bridge agreement. Even when I'm fairly sure, I'm sometimes wrong. Hence, when asked about partner's call, I suggest that the opponent consult my system-card. If the opponent can't find an explanation on that and if I'm fairly confident, then I tend to give my best guess. Am I breaking the law? Should I just say "I'm unsure" or "no agreement", in the first place, as many other players do? From Hermandw at skynet.be Wed Jan 19 16:32:30 2011 From: Hermandw at skynet.be (Herman De Wael) Date: Wed, 19 Jan 2011 16:32:30 +0100 Subject: [BLML] Ecclesiastes (proposed change in law) In-Reply-To: References: <4D302790.8050401@skynet.be> <1748200137.98748.1295002698502.JavaMail.ngmail@webmail08.arcor-online.net> <4D304E50.7080502@ulb.ac.be> <4D34582E.8070509@nhcc.net> <4D34598B.4030602@ulb.ac.be> Message-ID: <4D37040E.90902@skynet.be> Eric Landau wrote: > > The Beijing interpretation is problematic when you aren't 100% sure > about your actual agreements. The "dWS" alternative is problematic > when you aren't 100% sure about partner's intention. Neither > expectation can be met to perfection, but one would think it more > difficult to come close to the latter than to the former, especially > those that are quick to deprecate "mind reading" when done by the > TD: Should we not have similar qualms about expecting "mind reading" > of partners by players at the table? > There is one problem here with Eric's reasoning: the similarity exists only in Eric's mind. Sure, there are problems with the dWS interpretationg WHEN one is not 100% certain of what is going on in partner's mind, but the difference is that this seldom happens (or at least far more selfom than the other one) since PARTNER HAS JUST TOLD YOU WHAT IS GOING ON IN HIS MIND. Further more, even when you are not certain of how he makes his responses, answering by one of the possibilitiess will merely make him think you are mistaken about the answer, not about the meaning of your question, so it will not carry any UI. Take this as an example: You bid 4NT, which you are 50% certain is for the minors. Partner explains it as Blackwood and bids 5D. You are now 100% certain that partner interprets 5Di as showing a number of aces. You are 75% certain he means one ace, but it could be 0 aces as well. They ask what 5Di is and you say "1 ace". If partner happens to hold 0 aces, he will merely think you are mistaken in your Blackwood responses, not that you were asking for minors = no UI there. Understand? > > Eric Landau -- Herman De Wael Wilrijk Antwerpen Belgium From Hermandw at skynet.be Wed Jan 19 16:35:04 2011 From: Hermandw at skynet.be (Herman De Wael) Date: Wed, 19 Jan 2011 16:35:04 +0100 Subject: [BLML] Unsure In-Reply-To: <580906.17257.qm@web28513.mail.ukl.yahoo.com> References: <4D302790.8050401@skynet.be> <1748200137.98748.1295002698502.JavaMail.ngmail@webmail08.arcor-online.net> <4D304E50.7080502@ulb.ac.be> <4D34582E.8070509@nhcc.net> <4D34598B.4030602@ulb.ac.be> <580906.17257.qm@web28513.mail.ukl.yahoo.com> Message-ID: <4D3704A8.8090006@skynet.be> Nigel Guthrie wrote: > [Nigel] > > Many BLMLers are certain about law-makers' intentions. Such players may also be > certain of their partnership agreements. > > > I've some beliefs but I'm uncertain about most things. I'm *never* certain of > any Bridge agreement. Even when I'm fairly sure, I'm sometimes wrong. > > Hence, when asked about partner's call, I suggest that the opponent consult my > system-card. If the opponent can't find an explanation on that and if I'm fairly > confident, then I tend to give my best guess. Am I breaking the law? Should I > just say "I'm unsure" or "no agreement", in the first place, as many other > players do? Well Nigel, Legally you are free to do as you wish. But politely, it is better to give an answer than to ask them to check the SC. -- Herman De Wael Wilrijk Antwerpen Belgium From jfusselman at gmail.com Wed Jan 19 16:41:28 2011 From: jfusselman at gmail.com (Jerry Fusselman) Date: Wed, 19 Jan 2011 09:41:28 -0600 Subject: [BLML] Ecclesiastes (proposed change in law) In-Reply-To: References: <4D302790.8050401@skynet.be> <1748200137.98748.1295002698502.JavaMail.ngmail@webmail08.arcor-online.net> <4D304E50.7080502@ulb.ac.be> <4D34582E.8070509@nhcc.net> <4D34598B.4030602@ulb.ac.be> Message-ID: On Wed, Jan 19, 2011 at 8:24 AM, Eric Landau wrote: > > Several people have cited as a flaw in the Beijing interpretation > that it gives no guidance on how to act when one is uncertain as to > what one's actual partnership agreements are. ?The proposed solution > is to simply describe the meaning of partner's call as whatever he > thinks it is, rather than what it would be by virtue of one's actual > agreements. That's not how I would describe the proposed solution. > ?But that presupposes that one is in a position where you > aren't sure about your agreements, but are 100% sure as to your > partner's intention. ?One could just as easily postulate that one is > certain of one's agreements, then ask Jerry's 12 questions about > perceived differences in one's certainty regarding partner's intention. > I promise to answer any questions Eric or others might have along these lines. I don't see it as any difficulty. Please tell me if I am wrong, but it sounds to me that Eric is assuming that the essence of dWS is to always explain your understandings the way partner would. But that's no an accurate description. The essence of dWS is that if you have a problem where your answer seems likely to provide UI, MI, or both, you can choose which answer to give, but you stand ready for a rectification for whatever UI or MI your answer caused. It does not require any sort of mind reading. You take your best shot, and accept the consequences under the laws that are already in place. Alright, suppose case 1 under Eric's program is that I know that our agreement is A, and I am 90% sure that partner thinks it is B. (The remaining 10% is that partner incorrectly alerted 1NT but still knows that our agreement is A.) Probably Eric wants me to remove the assumption that the opponents asked for an explanation of 1NT. If this is one of the cases that Eric has in mind, then my choice of how to describe 2C is this: If I explain 2C as if our understanding is A, then there is a 90% chance my answer provides UI, and a 10% chance if provides neither UI nor MI. If I explain 2C as if our understanding is B, then there is a 90% chance my answer provides MI, and a 10% chance that it provides both UI and MI. Like any other decision in bridge, I make my choice and accept the consequences. That's what the UI and MI rules are there for. At no point does dWS require certainty about what partner is thinking. The more uncertainty you have about partner's understanding, the more likely you will just try to minimize MI, because otherwise you are in danger of providing both MI and UI. > The Beijing interpretation is problematic when you aren't 100% sure > about your actual agreements. Yes, agreed. > ?The "dWS" alternative is problematic > when you aren't 100% sure about partner's intention. I don't see that. Maybe you can provide an example. > ?Neither > expectation can be met to perfection, but one would think it more > difficult to come close to the latter than to the former, especially > those that are quick to deprecate "mind reading" when done by the > TD: I am not deprecating mind reading by a TD when the laws call for it. I am deprecating whichever laws unnecessarily require it. > ?Should we not have similar qualms about expecting "mind reading" > of partners by players at the table? > But dWS does not expect mind reading. It recognizes that if you attempt to minimize UI by providing temporary MI, but you're wrong in your estimate of what's in partner's mind, then you will have created both UI and MI. It sounds to me that Eric has interpreted Robert's suggestion as this: "There is no rectification for a correct description of partner's hand, provided you are ex ante positive that's what you are doing." I feel sure that Robert's suggestion is more along the lines of this: "There is no rectification for a correct description of partner's hand, provided it happens to turn out to be correct." If it turns out incorrect, you may well have created both UI and MI. Jerry Fusselman From posundelin at yahoo.se Wed Jan 19 17:27:12 2011 From: posundelin at yahoo.se (PO Sundelin) Date: Wed, 19 Jan 2011 16:27:12 +0000 (GMT) Subject: [BLML] Unsure In-Reply-To: <4D3704A8.8090006@skynet.be> Message-ID: <128171.88697.qm@web25402.mail.ukl.yahoo.com> Interesting view by Herman... Legally free to do as you wish!!! ? You should definitely NOT say No agreement when you know there is one (even if you?re unsure what it is) Nor should you go the NO A way?if your experience from this partnership clearly indicates something - you can say I?m not sure but... or I have forgotten which one of the following two... or And Herman?s recommendation "better to give an answer than referring to your system card" is of course?wrong if you are guessing. Tell them you?re unsure or have forgotten, and by all means then ask them to look for themselves. Rgds PO Sundelin --- Den ons 2011-01-19 skrev Herman De Wael : Fr?n: Herman De Wael ?mne: Re: [BLML] Unsure Till: "Bridge Laws Mailing List" Datum: onsdag 19 januari 2011 16:35 Nigel Guthrie wrote: > [Nigel] > > Many BLMLers are certain about law-makers' intentions. Such players may also be > certain of their partnership agreements. > > > I've some beliefs but I'm uncertain about most things. I'm *never* certain of > any Bridge agreement. Even when I'm fairly sure, I'm sometimes wrong. > > Hence, when asked about partner's call, I suggest that the opponent consult my > system card. If the opponent can't find an explanation on that and if I'm fairly > confident, then I tend to give my best guess. Am I breaking the law? Should I > just say "I'm unsure" or "no agreement", in the first place, as many other > players do? Well Nigel, Legally you are free to do as you wish. But politely, it is better to give an answer than to ask them to check the SC. -- Herman De Wael Wilrijk Antwerpen Belgium _______________________________________________ Blml mailing list Blml at rtflb.org http://lists.rtflb.org/mailman/listinfo/blml -------------- next part -------------- An HTML attachment was scrubbed... URL: http://lists.rtflb.org/pipermail/blml/attachments/20110119/9fa7a93c/attachment.html From jfusselman at gmail.com Wed Jan 19 19:34:14 2011 From: jfusselman at gmail.com (Jerry Fusselman) Date: Wed, 19 Jan 2011 12:34:14 -0600 Subject: [BLML] Unsure In-Reply-To: <128171.88697.qm@web25402.mail.ukl.yahoo.com> References: <4D3704A8.8090006@skynet.be> <128171.88697.qm@web25402.mail.ukl.yahoo.com> Message-ID: On Wed, Jan 19, 2011 at 10:27 AM, PO Sundelin wrote: > > Interesting view by Herman... > Legally free to do as you wish!!! I am sure all that Herman was referring to here is that Nigel can tell them to look at the SC if he wishes to, but he thinks it is generally politer to just answer their question. > > You should definitely NOT say No agreement when you know there is one (even if you?re unsure what it is) Herman agrees with this. He is just about the least likely person on this board to suggest saying no agreement when there is one. > Nor should you go the NO A way?if your experience from this partnership clearly indicates something He certainly agrees here too. - you can say I?m not sure but... or I have forgotten which one of the following two... or This is where Herman will start to disagree with you. I think he will say that it is usually better to choose one answer. For example, if your agreement depends on whether the double is penalty or not, what good does it do anyone for the opponent to "it is either penalty or not, and your guess is as good as mine." I hear that from time to time when the SC can be of no help, and no, my guess is not as good as the partner's guess. > And Herman?s recommendation "better to give an answer than referring to your system card" is of course?wrong if you are guessing. I did not see him write what you quoted. > Tell them you?re unsure or have forgotten, and by all means then ask them to look for themselves. The best procedure probably depends on the likelihood that opponents can figure out the agreement from the SC. Jerry Fusselman From blml at arcor.de Wed Jan 19 19:47:28 2011 From: blml at arcor.de (Thomas Dehn) Date: Wed, 19 Jan 2011 19:47:28 +0100 (CET) Subject: [BLML] Unsure In-Reply-To: <580906.17257.qm@web28513.mail.ukl.yahoo.com> References: <580906.17257.qm@web28513.mail.ukl.yahoo.com> <4D302790.8050401@skynet.be> <1748200137.98748.1295002698502.JavaMail.ngmail@webmail08.arcor-online.net> <4D304E50.7080502@ulb.ac.be> <4D34582E.8070509@nhcc.net> <4D34598B.4030602@ulb.ac.be> Message-ID: <959789736.33643.1295462848459.JavaMail.ngmail@webmail20.arcor-online.net> Nigel Guthrie > [Nigel] > > Many BLMLers are certain about law-makers' intentions. Such players may also > be certain of their partnership agreements. > > > I've some beliefs but I'm uncertain about most things. I'm *never* certain > of any Bridge agreement. Even when I'm fairly sure, I'm sometimes wrong. I suggest that you spend a little bit more time in learning your own system ;-) > Hence, when asked about partner's call, I suggest that the opponent consult > my system-card. To some extent that depends on what the RA has ordered, but generally I deem this insufficient. Your obligations are detailed in L20F 1. During the auction and before the final pass, any player may request, but only at his own turn to call, an explanation of the opponents? prior auction. He is entitled to know about calls actually made, about relevant alternative calls available that were not made, and about relevant inferences from the choice of action where these are matters of partnership understanding. Except on the instruction of the Director replies should be given by the partner of the player who made the call in question. Simply pointing opponents to your CC will at best tell opponents the meaning of calls actually made. But that fulfills only a subset of your obligations. > If the opponent can't find an explanation on that and if I'm > fairly confident, then I tend to give my best guess. Am I breaking the law? Should > I just say "I'm unsure" or "no agreement", in the first place, as many other > players do? That would depend on what you remember, but generally "no agreement" is not the correct answer when have an agreement, but cannot remember it. I'd normally give my best guess, and would correct later on if I figure out that my initial explanation was wrong. Thomas From Hermandw at skynet.be Thu Jan 20 11:14:45 2011 From: Hermandw at skynet.be (Herman De Wael) Date: Thu, 20 Jan 2011 11:14:45 +0100 Subject: [BLML] Unsure In-Reply-To: <128171.88697.qm@web25402.mail.ukl.yahoo.com> References: <128171.88697.qm@web25402.mail.ukl.yahoo.com> Message-ID: <4D380B15.9010308@skynet.be> PO Sundelin wrote: > Interesting view by Herman... > Legally free to do as you wish!!! Of course you are! Basically, you are always free to do as you wish, if you are willing to accept the consequences and penalties. But here, I must stress that MI is an irregularity, not an infraction. You tell them what you wish, and accept the rectification afterwards. > *You should definitely NOT say No agreement* when you know there is one > (even if you?re unsure what it is) Of course not, and that is certainly not what I intended. Even if you are "free to do", it would be silly to simply say an untruth, because the consequennces would be too much worse. > Nor should you go the NO A way if your experience from this partnership > clearly indicates something - you can say I?m not sure but... or I have > forgotten which one of the following two... or You can say that, but I advise against it. When you say "I'm not sure but I believe it is A", then the opponents are entitled to use "it is A" as their basis for deciding what to do. And if it turns out to be B, the TD will rule against you, whether you have said "I'm not sure" or not. So for the purposes of the MI ruling, saying "it is A" is exactly the same as "I'm not sure but I believe it is A". And because the second gives the opponents information they are not entitled to, but will be able to use to their advantage, I advise against it. Please note that they cannot ask for a rectification because you did not say "I'm not sure but ...". > And Herman?s recommendation "better to give an answer than referring to > your system card" is of course wrong if you are guessing. Tell them > you?re unsure or have forgotten, and by all means then ask them to look > for themselves. Well, the same applies as above. If you are guessing and tell them, they have some information they are not entitled to. And your partner has UI. Whereas if you simply tell them what you've guessed, and you have guessed correctly, there is no UI, no MI, and no non-entitled information. If you've guessed incorrectly, of course there is massive UI, but I believe it is best to trust yourself and not take this into account. You're headed for a bottom anyway. But I stress this is my personal experience, my personal advice to players, there is no discussion here about what the laws say or mean. A player is allowed to guess at the meaning of his partner's call and explain it with a voice of certainty to his opponents. I believe that tactic is a winning one. > Rgds > PO Sundelin > Anyway, nice to hear from you, PO. -- Herman De Wael Wilrijk Antwerpen Belgium From PeterEidt at t-online.de Thu Jan 20 11:19:36 2011 From: PeterEidt at t-online.de (Peter Eidt) Date: Thu, 20 Jan 2011 11:19:36 +0100 Subject: [BLML] =?utf-8?q?Unsure?= In-Reply-To: <4D380B15.9010308@skynet.be> References: <4D380B15.9010308@skynet.be> Message-ID: <1Pfrc8-1tcX1U0@fwd04.aul.t-online.de> From: Herman De Wael > PO Sundelin wrote: > > Interesting view by Herman... > > Legally free to do as you wish!!! > > Of course you are! > Basically, you are always free to do as you wish, if you are willing > to accept the consequences and penalties. [snip] Law 72 B1: "A player must not infringe a law intentionally, even if there is a prescribed rectification he is willing to accept." From agot at ulb.ac.be Thu Jan 20 13:38:17 2011 From: agot at ulb.ac.be (Alain Gottcheiner) Date: Thu, 20 Jan 2011 13:38:17 +0100 Subject: [BLML] Unsure In-Reply-To: References: <4D3704A8.8090006@skynet.be> <128171.88697.qm@web25402.mail.ukl.yahoo.com> Message-ID: <4D382CB9.2060700@ulb.ac.be> Le 19/01/2011 19:34, Jerry Fusselman a ?crit : > > - you can say I?m not sure but... or I have forgotten which one of the > following two... or > > This is where Herman will start to disagree with you. I think he will > say that it is usually better to choose one answer. AG : agree, and it's very much along dWS lines. If you state that you're unsure, you've transmitted UI in any case. If you give some answer, you've transmitted it only in one case : when you're wrong. However, there are cases when at least something is common between the two possibilities, and then you can state it and give the additional answer with all due caution. [off topic : I had a problem last tuesday. French doesn't have an exact word for "either", so when asked "is this a 3-card or a 4-card raise ?", I answered "yes". Absolutely right on ligic, perhaps not on usefulness, but after all he should have asked "what does it mean ?".] > > The best procedure probably depends on the likelihood that opponents > can figure out the agreement from the SC. > AG : yes, and that's the point. If you're fairly sure that the information is on the CC, but can't remember it, you should be allowed to suggest that they look. And to avoid UI, you should also do it occasionally when you know the answer ! Best regards Alain From ehaa at starpower.net Thu Jan 20 15:47:32 2011 From: ehaa at starpower.net (Eric Landau) Date: Thu, 20 Jan 2011 09:47:32 -0500 Subject: [BLML] Unsure In-Reply-To: <4D382CB9.2060700@ulb.ac.be> References: <4D3704A8.8090006@skynet.be> <128171.88697.qm@web25402.mail.ukl.yahoo.com> <4D382CB9.2060700@ulb.ac.be> Message-ID: <6410CABD-9AF3-45A7-9B10-F5EDC78C6095@starpower.net> On Jan 20, 2011, at 7:38 AM, Alain Gottcheiner wrote: > Le 19/01/2011 19:34, Jerry Fusselman a ?crit : > >> - you can say I?m not sure but... or I have forgotten which one of >> the >> following two... or >> >> This is where Herman will start to disagree with you. I think he >> will >> say that it is usually better to choose one answer. > > AG : agree, and it's very much along dWS lines. > > If you state that you're unsure, you've transmitted UI in any case. > If you give some answer, you've transmitted it only in one case : when > you're wrong. "Your opponents are entitled to know as much about the meaning of partner's call as you do." That you are uncertain as to its meaning is something you know about it that they don't. And you are 100% entitled to take your own uncertainty into account in choosing your subsequent calls in the auction. The fundamental notion of full disclosure demands that your opponents be able to do the same. If anything, when you are uncertain as to the meaning of partner's call you should be required to tell them so. On Jan 20, 2011, at 5:14 AM, Herman De Wael wrote: > A player is allowed to guess at the meaning of his partner's call and > explain it with a voice of certainty to his opponents. ...and then go on to choose his remaining calls in the light of his own uncertainty, having already gone out of his way to hide the information from the opponents? I don't think so. > I believe that tactic is a winning one. It sure is, if you don't get caught. Eric Landau 1107 Dale Drive Silver Spring MD 20910 ehaa at starpower.net From agot at ulb.ac.be Thu Jan 20 16:20:46 2011 From: agot at ulb.ac.be (Alain Gottcheiner) Date: Thu, 20 Jan 2011 16:20:46 +0100 Subject: [BLML] Unsure In-Reply-To: <6410CABD-9AF3-45A7-9B10-F5EDC78C6095@starpower.net> References: <4D3704A8.8090006@skynet.be> <128171.88697.qm@web25402.mail.ukl.yahoo.com> <4D382CB9.2060700@ulb.ac.be> <6410CABD-9AF3-45A7-9B10-F5EDC78C6095@starpower.net> Message-ID: <4D3852CE.6080908@ulb.ac.be> Le 20/01/2011 15:47, Eric Landau a ?crit : > On Jan 20, 2011, at 7:38 AM, Alain Gottcheiner wrote: > >> Le 19/01/2011 19:34, Jerry Fusselman a ?crit : >> >>> - you can say I?m not sure but... or I have forgotten which one of >>> the >>> following two... or >>> >>> This is where Herman will start to disagree with you. I think he >>> will >>> say that it is usually better to choose one answer. >> AG : agree, and it's very much along dWS lines. >> >> If you state that you're unsure, you've transmitted UI in any case. >> If you give some answer, you've transmitted it only in one case : when >> you're wrong. > "Your opponents are entitled to know as much about the meaning of > partner's call as you do." > > That you are uncertain as to its meaning is something you know about > it that they don't. AG : I can't agree with that. The fact that I'm uncertain about the meaning isn't part of the meaning. In other terms, they're entitled to know the meaning, but aren't entitled to know my state of mind (at least according to TFLB). Sometimes I play some gadget that partner insists on playing, but I consider inferior. Am I compelled to say that to the opponents ? > And you are 100% entitled to take your own > uncertainty into account in choosing your subsequent calls in the > auction. The fundamental notion of full disclosure demands that your > opponents be able to do the same. If anything, when you are > uncertain as to the meaning of partner's call you should be required > to tell them so. AG :and do you really think that, by giving them two explanations, you'll help them getting the meaning ? Or, putting it another way, is giving two alternative meanings "full disclosure" ? I'd rather say that it's useless. (notice that I did it in the past, but opponents didn't seem happy with it) From Hermandw at skynet.be Thu Jan 20 17:06:58 2011 From: Hermandw at skynet.be (Herman De Wael) Date: Thu, 20 Jan 2011 17:06:58 +0100 Subject: [BLML] Unsure In-Reply-To: <6410CABD-9AF3-45A7-9B10-F5EDC78C6095@starpower.net> References: <4D3704A8.8090006@skynet.be> <128171.88697.qm@web25402.mail.ukl.yahoo.com> <4D382CB9.2060700@ulb.ac.be> <6410CABD-9AF3-45A7-9B10-F5EDC78C6095@starpower.net> Message-ID: <4D385DA2.20805@skynet.be> Eric Landau wrote: > On Jan 20, 2011, at 7:38 AM, Alain Gottcheiner wrote: > >> Le 19/01/2011 19:34, Jerry Fusselman a ?crit : >> >>> - you can say I?m not sure but... or I have forgotten which one of >>> the >>> following two... or >>> >>> This is where Herman will start to disagree with you. I think he >>> will >>> say that it is usually better to choose one answer. >> >> AG : agree, and it's very much along dWS lines. >> >> If you state that you're unsure, you've transmitted UI in any case. >> If you give some answer, you've transmitted it only in one case : when >> you're wrong. > > "Your opponents are entitled to know as much about the meaning of > partner's call as you do." > This may be a useful shortcut to the laws, but it is not what the laws say. > That you are uncertain as to its meaning is something you know about > it that they don't. Indeed you know it, but that does not make it entitled information! > And you are 100% entitled to take your own > uncertainty into account in choosing your subsequent calls in the > auction. The fundamental notion of full disclosure demands that your > opponents be able to do the same. If anything, when you are > uncertain as to the meaning of partner's call you should be required > to tell them so. > Maybe you should be, and that is then your personal opinion, but I don't see it anywhere in the laws. The opponents are entitled to know what the meaning is of a call, but not how confident you are of it. > On Jan 20, 2011, at 5:14 AM, Herman De Wael wrote: > >> A player is allowed to guess at the meaning of his partner's call and >> explain it with a voice of certainty to his opponents. > > ...and then go on to choose his remaining calls in the light of his > own uncertainty, having already gone out of his way to hide the > information from the opponents? I don't think so. > Well, please state the law you are basing this thought on. I don't believe you are right here! >> I believe that tactic is a winning one. > > It sure is, if you don't get caught. > Well, that would be if your thoughts above were true - they are not. And besides, how is any director going to be supposed to rule on such a vague fact: "how certain were you of the meaning of that call? 99% or 99.9?". > > Eric Landau -- Herman De Wael Wilrijk Antwerpen Belgium From Hermandw at skynet.be Thu Jan 20 17:09:13 2011 From: Hermandw at skynet.be (Herman De Wael) Date: Thu, 20 Jan 2011 17:09:13 +0100 Subject: [BLML] Unsure In-Reply-To: <1Pfrc8-1tcX1U0@fwd04.aul.t-online.de> References: <4D380B15.9010308@skynet.be> <1Pfrc8-1tcX1U0@fwd04.aul.t-online.de> Message-ID: <4D385E29.7030905@skynet.be> Peter Eidt wrote: > From: Herman De Wael >> PO Sundelin wrote: >>> Interesting view by Herman... >>> Legally free to do as you wish!!! >> >> Of course you are! >> Basically, you are always free to do as you wish, if you are willing >> to accept the consequences and penalties. > > [snip] > > Law 72 B1: > "A player must not infringe a law intentionally, even if there is > a prescribed rectification he is willing to accept." > And what law am I infringing by telling them what I believe the meaning of my partner's call is? Or what law is PO following when he tells them to look at the SC? -- Herman De Wael Wilrijk Antwerpen Belgium From jean-pierre.rocafort at meteo.fr Thu Jan 20 17:43:17 2011 From: jean-pierre.rocafort at meteo.fr (Jean-Pierre Rocafort) Date: Thu, 20 Jan 2011 17:43:17 +0100 Subject: [BLML] Unsure In-Reply-To: <4D3852CE.6080908@ulb.ac.be> References: <4D3704A8.8090006@skynet.be> <128171.88697.qm@web25402.mail.ukl. yahoo.com> < 4D382CB9.2060700@ulb.ac.be><6410CABD-9AF3-45A7-9B10-F5EDC78C6095@starpower. net> <4D3852CE.6080908@ulb.ac.be> Message-ID: <4D386625.90105@meteo.fr> Alain Gottcheiner a ?crit : > Le 20/01/2011 15:47, Eric Landau a ?crit : >> On Jan 20, 2011, at 7:38 AM, Alain Gottcheiner wrote: >> >>> Le 19/01/2011 19:34, Jerry Fusselman a ?crit : >>> >>>> - you can say I?m not sure but... or I have forgotten which one of >>>> the >>>> following two... or >>>> >>>> This is where Herman will start to disagree with you. I think he >>>> will >>>> say that it is usually better to choose one answer. >>> AG : agree, and it's very much along dWS lines. >>> >>> If you state that you're unsure, you've transmitted UI in any case. >>> If you give some answer, you've transmitted it only in one case : when >>> you're wrong. >> "Your opponents are entitled to know as much about the meaning of >> partner's call as you do." it rather is: "Your opponents are entitled to know as much about the elements from which to infer the meaning of partner's call as you do." >> >> That you are uncertain as to its meaning is something you know about >> it that they don't. > AG : I can't agree with that. The fact that I'm uncertain about the > meaning isn't part of the meaning. yes and no, it depends why you are uncertain. if you are uncertain because you have an agreement and have forgotten it, they are untitled to the exact agreement but not to your confusion. as you must give them, most often you will not be able to hide your confusion as you will have to say that you know you should tell something you don't remember anymore. but sometimes you can hide it by telling the opponent it will gain time to read the information from the CC. i think this is lawful and fair. if you are uncertain because of conflicting agreements or imprecise agreements, they are untitled to the uncertainty as you have do describe not the meaning but the bases helping to infer the meaning of the call. and if you are uncertain because you don't have any more base from which to infer the meaning of the call than your opponent, the "no agreement" answer is adequate. > In other terms, they're entitled to know the meaning, but aren't > entitled to know my state of mind (at least according to TFLB). > Sometimes I play some gadget that partner insists on playing, but I > consider inferior. Am I compelled to say that to the opponents ? no, but what is the effect on your bidding? do you abandon agreements you dislike, in the middle of the bidding? > >> And you are 100% entitled to take your own >> uncertainty into account in choosing your subsequent calls in the >> auction. The fundamental notion of full disclosure demands that your >> opponents be able to do the same. If anything, when you are >> uncertain as to the meaning of partner's call you should be required >> to tell them so. > AG :and do you really think that, by giving them two explanations, > you'll help them getting the meaning ? Or, putting it another way, is > giving two alternative meanings "full disclosure" ? I'd rather say that > it's useless. > (notice that I did it in the past, but opponents didn't seem happy with it) it's easier to play against players with firm bidding than against improvising bidders; on the other side the second category will give you more presents. full disclosure is a simple thing once you have understood what it is all about. jpr -- _______________________________________________ Jean-Pierre Rocafort METEO-FRANCE DSI/CM 42 Avenue Gaspard Coriolis 31057 Toulouse CEDEX Tph: 05 61 07 81 02 (33 5 61 07 81 02) Fax: 05 61 07 81 09 (33 5 61 07 81 09) e-mail: jean-pierre.rocafort at meteo.fr Serveur WWW METEO-France: http://www.meteo.fr _______________________________________________ From agot at ulb.ac.be Thu Jan 20 17:59:09 2011 From: agot at ulb.ac.be (Alain Gottcheiner) Date: Thu, 20 Jan 2011 17:59:09 +0100 Subject: [BLML] Unsure In-Reply-To: <4D386625.90105@meteo.fr> References: <4D3704A8.8090006@skynet.be> <128171.88697.qm@web25402.mail.ukl. yahoo.com> < 4D382CB9.2060700@ulb.ac.be><6410CABD-9AF3-45A7-9B10-F5EDC78C6095@starpower. net> <4D3852CE.6080908@ulb.ac.be> <4D386625.90105@meteo.fr> Message-ID: <4D3869DD.9030708@ulb.ac.be> Le 20/01/2011 17:43, Jean-Pierre Rocafort a ?crit : > Alain Gottcheiner a ?crit : >> Le 20/01/2011 15:47, Eric Landau a ?crit : >>> On Jan 20, 2011, at 7:38 AM, Alain Gottcheiner wrote: >>> >>>> Le 19/01/2011 19:34, Jerry Fusselman a ?crit : >>>> >>>>> - you can say I?m not sure but... or I have forgotten which one of >>>>> the >>>>> following two... or >>>>> >>>>> This is where Herman will start to disagree with you. I think he >>>>> will >>>>> say that it is usually better to choose one answer. >>>> AG : agree, and it's very much along dWS lines. >>>> >>>> If you state that you're unsure, you've transmitted UI in any case. >>>> If you give some answer, you've transmitted it only in one case : when >>>> you're wrong. >>> "Your opponents are entitled to know as much about the meaning of >>> partner's call as you do." > it rather is: "Your opponents are entitled to know as much about the > elements from which to infer the meaning of partner's call as you do." >>> That you are uncertain as to its meaning is something you know about >>> it that they don't. >> AG : I can't agree with that. The fact that I'm uncertain about the >> meaning isn't part of the meaning. > yes and no, it depends why you are uncertain. > if you are uncertain because you have an agreement and have forgotten > it, they are untitled to the exact agreement but not to your confusion. > as you must give them, most often you will not be able to hide your > confusion as you will have to say that you know you should tell > something you don't remember anymore. but sometimes you can hide it by > telling the opponent it will gain time to read the information from the > CC. i think this is lawful and fair. > if you are uncertain because of conflicting agreements or imprecise > agreements, they are untitled to the uncertainty as you have do describe > not the meaning but the bases helping to infer the meaning of the call. AG : agreed ; however, in the case of conflicting agreements whose conflict hasn't been solved, the true answer would be "we didn't agree about that one" ; not "I'm unsure, but it should be ...", because it should be nothing. >> In other terms, they're entitled to know the meaning, but aren't >> entitled to know my state of mind (at least according to TFLB). >> Sometimes I play some gadget that partner insists on playing, but I >> consider inferior. Am I compelled to say that to the opponents ? > no, but what is the effect on your bidding? do you abandon agreements > you dislike, in the middle of the bidding? AG : I wouldn't, although I might refrain from using the bid at all, but what's the point of this question ? Notice that in the original problem, I might well suppose that the meaning is A, but choose a bid which won't turn out badly if not (for example by not passing a bid that might be artificial), which of course will be legal if and only if I didn't tell partner that I'm unsure. If I did, he could have UI that I acted in that way. Best regards Alain From jean-pierre.rocafort at meteo.fr Thu Jan 20 18:40:54 2011 From: jean-pierre.rocafort at meteo.fr (Jean-Pierre Rocafort) Date: Thu, 20 Jan 2011 18:40:54 +0100 Subject: [BLML] Unsure In-Reply-To: <4D3869DD.9030708@ulb.ac.be> References: <4D3704A8.8090006@skynet.be> <128171.88697.qm@web25402.mail.ukl. yahoo.com> < 4D382CB9.2060700@ulb.ac.be><6410CABD-9AF3-45A7-9B10-F5EDC78C6095@starpowe r. net> <4D3852CE.6080908@ulb.ac.be> <4D386625.90105@meteo.fr> <4D3869DD.9030708@ulb.ac.be> Message-ID: <4D3873A6.5070104@meteo.fr> Alain Gottcheiner a ?crit : > Le 20/01/2011 17:43, Jean-Pierre Rocafort a ?crit : >> Alain Gottcheiner a ?crit : >>> Le 20/01/2011 15:47, Eric Landau a ?crit : >>>> On Jan 20, 2011, at 7:38 AM, Alain Gottcheiner wrote: >>>> >>>>> Le 19/01/2011 19:34, Jerry Fusselman a ?crit : >>>>> >>>>>> - you can say I?m not sure but... or I have forgotten which one of >>>>>> the >>>>>> following two... or >>>>>> >>>>>> This is where Herman will start to disagree with you. I think he >>>>>> will >>>>>> say that it is usually better to choose one answer. >>>>> AG : agree, and it's very much along dWS lines. >>>>> >>>>> If you state that you're unsure, you've transmitted UI in any case. >>>>> If you give some answer, you've transmitted it only in one case : when >>>>> you're wrong. >>>> "Your opponents are entitled to know as much about the meaning of >>>> partner's call as you do." >> it rather is: "Your opponents are entitled to know as much about the >> elements from which to infer the meaning of partner's call as you do." >>>> That you are uncertain as to its meaning is something you know about >>>> it that they don't. >>> AG : I can't agree with that. The fact that I'm uncertain about the >>> meaning isn't part of the meaning. >> yes and no, it depends why you are uncertain. >> if you are uncertain because you have an agreement and have forgotten >> it, they are untitled to the exact agreement but not to your confusion. >> as you must give them, most often you will not be able to hide your >> confusion as you will have to say that you know you should tell >> something you don't remember anymore. but sometimes you can hide it by >> telling the opponent it will gain time to read the information from the >> CC. i think this is lawful and fair. >> if you are uncertain because of conflicting agreements or imprecise >> agreements, they are untitled to the uncertainty as you have do describe >> not the meaning but the bases helping to infer the meaning of the >> call. > AG : agreed ; however, in the case of conflicting agreements whose > conflict hasn't been solved, the true answer would be "we didn't agree > about that one" ; not "I'm unsure, but it should be ...", because it > should be nothing. the true answer is certainly not lack of agreement but superabundance of agreements: you disclose all relevant agreements and partnership experience and it's everybody's guess for the meaning. >>> In other terms, they're entitled to know the meaning, but aren't >>> entitled to know my state of mind (at least according to TFLB). >>> Sometimes I play some gadget that partner insists on playing, but I >>> consider inferior. Am I compelled to say that to the opponents ? >> no, but what is the effect on your bidding? do you abandon agreements >> you dislike, in the middle of the bidding? > > AG : I wouldn't, although I might refrain from using the bid at all, but > what's the point of this question? i didn't understand your point about telling or not telling something which had nothing to do with the understanding of the call. > > Notice that in the original problem, I might well suppose that the > meaning is A, but choose a bid which won't turn out badly if not (for > example by not passing a bid that might be artificial), which of course > will be legal if and only if I didn't tell partner that I'm unsure. If I > did, he could have UI that I acted in that way. it will always be legal: you may choose any bid you wish so long as it's not you who have UI. when you are unable to disclose your agreements and there is not a screen to prevent partner from knowing it, it may happen that you will suffer from your forgetfulness and i don't see it so undeserved. jpr > > > Best regards > > > Alain > -- _______________________________________________ Jean-Pierre Rocafort METEO-FRANCE DSI/CM 42 Avenue Gaspard Coriolis 31057 Toulouse CEDEX Tph: 05 61 07 81 02 (33 5 61 07 81 02) Fax: 05 61 07 81 09 (33 5 61 07 81 09) e-mail: jean-pierre.rocafort at meteo.fr Serveur WWW METEO-France: http://www.meteo.fr _______________________________________________ From nigelguthrie at yahoo.co.uk Thu Jan 20 21:40:10 2011 From: nigelguthrie at yahoo.co.uk (Nigel Guthrie) Date: Thu, 20 Jan 2011 20:40:10 +0000 (GMT) Subject: [BLML] Unsure In-Reply-To: <4D3873A6.5070104@meteo.fr> References: <4D3704A8.8090006@skynet.be> <128171.88697.qm@web25402.mail.ukl. yahoo.com> < 4D382CB9.2060700@ulb.ac.be><6410CABD-9AF3-45A7-9B10-F5EDC78C6095@starpowe r. net> <4D3852CE.6080908@ulb.ac.be> <4D386625.90105@meteo.fr> <4D3869DD.9030708@ulb.ac.be> <4D3873A6.5070104@meteo.fr> Message-ID: <107026.33279.qm@web28515.mail.ukl.yahoo.com> [Nigel] When asked the meaning of a call, the law says you must not speculate gratuitously. If you aren't certain (and that must be the case for most calls and almost all players), you must answer "unsure". IMO, this is silly. The law actively inhibits disclosure. Even I will guess right 80% of the time and younger players in long-term regular partnerships will guess right more than 90% of the time. From swillner at nhcc.net Fri Jan 21 18:04:30 2011 From: swillner at nhcc.net (Steve Willner) Date: Fri, 21 Jan 2011 12:04:30 -0500 Subject: [BLML] Unsure In-Reply-To: <107026.33279.qm@web28515.mail.ukl.yahoo.com> References: <4D3704A8.8090006@skynet.be> <128171.88697.qm@web25402.mail.ukl. yahoo.com> < 4D382CB9.2060700@ulb.ac.be><6410CABD-9AF3-45A7-9B10-F5EDC78C6095@starpowe r. net> <4D3852CE.6080908@ulb.ac.be> <4D386625.90105@meteo.fr> <4D3869DD.9030708@ulb.ac.be> <4D3873A6.5070104@meteo.fr> <107026.33279.qm@web28515.mail.ukl.yahoo.com> Message-ID: <4D39BC9E.9040108@nhcc.net> On 1/20/2011 3:40 PM, Nigel Guthrie wrote: > When asked the meaning of a call, the law says you must not speculate > gratuitously. Or rather it says any speculation is UI to your partner and doesn't mitigate any MI you give. So there's no point in it, but it isn't directly illegal as far as I can see. > If you aren't certain (and that must be the case for most calls > and almost all players), you must answer "unsure". But this is nonsense. Your duty is to tell opponents your agreements. If you can't, then they should be compensated if MI causes damage. Your personal state of mind about those agreements is irrelevant for MI purposes. If the agreement itself is unclear or if there's no agreement at all, that's a different matter altogether. "Unsure" still isn't an explanation, but "no agreement" may be. From nigelguthrie at yahoo.co.uk Sat Jan 22 03:24:17 2011 From: nigelguthrie at yahoo.co.uk (Nigel Guthrie) Date: Sat, 22 Jan 2011 02:24:17 +0000 (GMT) Subject: [BLML] Unsure In-Reply-To: <4D39BC9E.9040108@nhcc.net> References: <4D3704A8.8090006@skynet.be> <128171.88697.qm@web25402.mail.ukl. yahoo.com> < 4D382CB9.2060700@ulb.ac.be><6410CABD-9AF3-45A7-9B10-F5EDC78C6095@starpowe r. net> <4D3852CE.6080908@ulb.ac.be> <4D386625.90105@meteo.fr> <4D3869DD.9030708@ulb.ac.be> <4D3873A6.5070104@meteo.fr> <107026.33279.qm@web28515.mail.ukl.yahoo.com> <4D39BC9E.9040108@nhcc.net> Message-ID: <586818.75337.qm@web28510.mail.ukl.yahoo.com> {Nige1] If you aren't certain (and that must be the case for most calls and almost all players), you must answer "unsure". {Steve Wilner[ But this is nonsense. Your duty is to tell opponents your agreements. If you can't, then they should be compensated if MI causes damage. Your personal state of mind about those agreements is irrelevant for MI purposes. If the agreement itself is unclear or if there's no agreement at all, that's a different matter altogether. "Unsure" still isn't an explanation, but "no agreement" may be. {Nigel] We don't all live in your world of certainties, Steve. Saying "unsure" (if that is the truth) is not MI. In this context, IMO, it is the only truthful and legal option. Legal or not, it is the regular practice of many tournament players. From blml at arcor.de Sat Jan 22 06:54:08 2011 From: blml at arcor.de (Thomas Dehn) Date: Sat, 22 Jan 2011 06:54:08 +0100 (CET) Subject: [BLML] Unsure In-Reply-To: <586818.75337.qm@web28510.mail.ukl.yahoo.com> References: <586818.75337.qm@web28510.mail.ukl.yahoo.com> <4D3704A8.8090006@skynet.be> <128171.88697.qm@web25402.mail.ukl. yahoo.com> < 4D382CB9.2060700@ulb.ac.be><6410CABD-9AF3-45A7-9B10-F5EDC78C6095@starpowe r. net> <4D3852CE.6080908@ulb.ac.be> <4D386625.90105@meteo.fr> <4D3869DD.9030708@ulb.ac.be> <4D3873A6.5070104@meteo.fr> <107026.33279.qm@web28515.mail.ukl.yahoo.com> <4D39BC9E.9040108@nhcc.net> Message-ID: <1820574630.137446.1295675648497.JavaMail.ngmail@webmail11.arcor-online.net> Nigel Guthrie wrote: > {Nige1] > If you aren't certain (and that must be the case for most calls > > and almost all players), you must answer "unsure". > > {Steve Wilner[ > But this is nonsense. Your duty is to tell opponents your agreements. > If you can't, then they should be compensated if MI causes damage. Your > personal state of mind about those agreements is irrelevant for MI > purposes. > > If the agreement itself is unclear or if there's no agreement at all, > that's a different matter altogether. "Unsure" still isn't an > explanation, but "no agreement" may be. > > {Nigel] > We don't all live in your world of certainties, Steve. Saying "unsure" (if > that is the truth) is not MI. In this context, IMO, it is the only truthful and > legal option. Legal or not, it is the regular practice of many tournament > players. It is MI. Say, your agreement is that bid X shows five controls. You don't remember anymore, and say that you are "unsure". That is MI, you were required to tell opponents that bid X shows five controls. Thomas From JffEstrsn at aol.com Sat Jan 22 08:58:08 2011 From: JffEstrsn at aol.com (Jeff Easterson) Date: Sat, 22 Jan 2011 08:58:08 +0100 Subject: [BLML] Unsure In-Reply-To: <586818.75337.qm@web28510.mail.ukl.yahoo.com> References: <4D3704A8.8090006@skynet.be> <128171.88697.qm@web25402.mail.ukl. yahoo.com> < 4D382CB9.2060700@ulb.ac.be><6410CABD-9AF3-45A7-9B10-F5EDC78C6095@starpowe r. net> <4D3852CE.6080908@ulb.ac.be> <4D386625.90105@meteo.fr> <4D3869DD.9030708@ulb.ac.be> <4D3873A6.5070104@meteo.fr> <107026.33279.qm@web28515.mail.ukl.yahoo.com> <4D39BC9E.9040108@nhcc.net> <586818.75337.qm@web28510.mail.ukl.yahoo.com> Message-ID: <4D3A8E10.2070602@aol.com> I agree, basically, with Steve. We are not talking about party bridge on blml. In serious bridge I think we can expect that players know what they are playing and do not continually forget their system. If you aren't certain you should take more time to talk to your partner and decide what you are playing. Ciao, JE Am 22.01.2011 03:24, schrieb Nigel Guthrie: > {Nige1] > If you aren't certain (and that must be the case for most calls > > and almost all players), you must answer "unsure". > > {Steve Wilner[ > But this is nonsense. Your duty is to tell opponents your agreements. > If you can't, then they should be compensated if MI causes damage. Your > personal state of mind about those agreements is irrelevant for MI purposes. > > If the agreement itself is unclear or if there's no agreement at all, > that's a different matter altogether. "Unsure" still isn't an > explanation, but "no agreement" may be. > > {Nigel] > We don't all live in your world of certainties, Steve. Saying "unsure" (if that > is the truth) is not MI. In this context, IMO, it is the only truthful and legal > option. Legal or not, it is the regular practice of many tournament players. > > _______________________________________________ > Blml mailing list > Blml at rtflb.org > http://lists.rtflb.org/mailman/listinfo/blml > From Hermandw at skynet.be Sat Jan 22 09:21:55 2011 From: Hermandw at skynet.be (Herman De Wael) Date: Sat, 22 Jan 2011 09:21:55 +0100 Subject: [BLML] Unsure In-Reply-To: <1820574630.137446.1295675648497.JavaMail.ngmail@webmail11.arcor-online.net> References: <586818.75337.qm@web28510.mail.ukl.yahoo.com> <4D3704A8.8090006@skynet.be> <128171.88697.qm@web25402.mail.ukl. yahoo.com> < 4D382CB9.2060700@ulb.ac.be><6410CABD-9AF3-45A7-9B10-F5EDC78C6095@starpowe r. net> <4D3852CE.6080908@ulb.ac.be> <4D386625.90105@meteo.fr> <4D3869DD.9030708@ulb.ac.be> <4D3873A6.5070104@meteo.fr> <107026.33279.qm@web28515.mail.ukl.yahoo.com> <4D39BC9E.9040108@nhcc.net> <1820574630.137446.1295675648497.JavaMail.ngmail@webmail11.arcor-online.net> Message-ID: <4D3A93A3.8020906@skynet.be> Thomas Dehn wrote: > > It is MI. Say, your agreement is that bid X shows five controls. > You don't remember anymore, and say that you are "unsure". > That is MI, you were required to tell opponents > that bid X shows five controls. > Yes, but that was not what we were talking about. If you say "5 controls" (omitting that you are unsure), have you then given MI? I say you haven't. > > Thomas -- Herman De Wael Wilrijk Antwerpen Belgium From blml at arcor.de Sat Jan 22 09:30:56 2011 From: blml at arcor.de (Thomas Dehn) Date: Sat, 22 Jan 2011 09:30:56 +0100 (CET) Subject: [BLML] Unsure In-Reply-To: <4D3A93A3.8020906@skynet.be> References: <4D3A93A3.8020906@skynet.be> <586818.75337.qm@web28510.mail.ukl.yahoo.com> <4D3704A8.8090006@skynet.be> <128171.88697.qm@web25402.mail.ukl. yahoo.com> < 4D382CB9.2060700@ulb.ac.be><6410CABD-9AF3-45A7-9B10-F5EDC78C6095@starpowe r. net> <4D3852CE.6080908@ulb.ac.be> <4D386625.90105@meteo.fr> <4D3869DD.9030708@ulb.ac.be> <4D3873A6.5070104@meteo.fr> <107026.33279.qm@web28515.mail.ukl.yahoo.com> <4D39BC9E.9040108@nhcc.net> <1820574630.137446.1295675648497.JavaMail.ngmail@webmail11.arcor-online.net> Message-ID: <1084173630.57148.1295685056117.JavaMail.ngmail@webmail13.arcor-online.net> Herman De Wael > Thomas Dehn wrote: > > > > It is MI. Say, your agreement is that bid X shows five controls. > > You don't remember anymore, and say that you are "unsure". > > That is MI, you were required to tell opponents > > that bid X shows five controls. > > > > Yes, but that was not what we were talking about. > If you say "5 controls" (omitting that you are unsure), have you then > given MI? > I say you haven't. If you say "5 controls", you have not given MI. If you say "4 controls", you have given MI. If you say "I am unsure, might be five controls, might be four controls", you have given MI. Thomas From svenpran at online.no Sat Jan 22 09:38:54 2011 From: svenpran at online.no (Sven Pran) Date: Sat, 22 Jan 2011 09:38:54 +0100 Subject: [BLML] Unsure In-Reply-To: <4D3A93A3.8020906@skynet.be> References: <586818.75337.qm@web28510.mail.ukl.yahoo.com> <4D3704A8.8090006@skynet.be> <128171.88697.qm@web25402.mail.ukl. yahoo.com> < 4D382CB9.2060700@ulb.ac.be><6410CABD-9AF3-45A7-9B10-F5EDC78C6095@starpowe r. net> <4D3852CE.6080908@ulb.ac.be> <4D386625.90105@meteo.fr> <4D3869DD.9030708@ulb.ac.be> <4D3873A6.5070104@meteo.fr> <107026.33279.qm@web28515.mail.ukl.yahoo.com> <4D39BC9E.9040108@nhcc.net> <1820574630.137446.1295675648497.JavaMail.ngmail@webmail11.arcor-online.net> <4D3A93A3.8020906@skynet.be> Message-ID: <000901cbba0f$ce55b3f0$6b011bd0$@no> On Behalf Of Herman De Wael > Thomas Dehn wrote: > > > > It is MI. Say, your agreement is that bid X shows five controls. > > You don't remember anymore, and say that you are "unsure". > > That is MI, you were required to tell opponents that bid X shows five > > controls. > > > > Yes, but that was not what we were talking about. > If you say "5 controls" (omitting that you are unsure), have you then given MI? > I say you haven't. I had a similar talk with the Norwegian LC some time ago; our line is that if the correct agreement cannot be established then we rule that the call has been made according to agreements. (Ref Law 75) This means that a player who is unsure but guesses the correct agreement has not given MI while a player who does not provide the correct agreement (if necessary confirmed by other evidence) is always deemed to having given MI. This includes the "not discussed" and "I have no idea" statements! Harsh? No I don't think so. The prime intention with Law 20 is to protect opponents. From svenpran at online.no Sat Jan 22 09:40:28 2011 From: svenpran at online.no (Sven Pran) Date: Sat, 22 Jan 2011 09:40:28 +0100 Subject: [BLML] Unsure In-Reply-To: <1084173630.57148.1295685056117.JavaMail.ngmail@webmail13.arcor-online.net> References: <4D3A93A3.8020906@skynet.be> <586818.75337.qm@web28510.mail.ukl.yahoo.com> <4D3704A8.8090006@skynet.be> <128171.88697.qm@web25402.mail.ukl. yahoo.com> < 4D382CB9.2060700@ulb.ac.be><6410CABD-9AF3-45A7-9B10-F5EDC78C6095@starpowe r. net> <4D3852CE.6080908@ulb.ac.be> <4D386625.90105@meteo.fr> <4D3869DD.9030708@ulb.ac.be> <4D3873A6.5070104@meteo.fr> <107026.33279.qm@web28515.mail.ukl.yahoo.com> <4D39BC9E.9040108@nhcc.net> <1820574630.137446.1295675648497.JavaMail.ngmail@webmail11.arcor-online.net> <1084173630.57148.1295685056117.JavaMail.ngmail@webmail13.arcor-online.net> Message-ID: <000a01cbba10$05b14da0$1113e8e0$@no> On Behalf Of Thomas Dehn > Herman De Wael > > Thomas Dehn wrote: > > > > > > It is MI. Say, your agreement is that bid X shows five controls. > > > You don't remember anymore, and say that you are "unsure". > > > That is MI, you were required to tell opponents that bid X shows > > > five controls. > > > > > > > Yes, but that was not what we were talking about. > > If you say "5 controls" (omitting that you are unsure), have you then > > given MI? > > I say you haven't. > > If you say "5 controls", you have not given MI. > If you say "4 controls", you have given MI. > If you say "I am unsure, might be five controls, might be four controls", you have > given MI. This is true, at least in Norway. From blml at arcor.de Sat Jan 22 10:01:36 2011 From: blml at arcor.de (Thomas Dehn) Date: Sat, 22 Jan 2011 10:01:36 +0100 (CET) Subject: [BLML] Unsure In-Reply-To: <000901cbba0f$ce55b3f0$6b011bd0$@no> References: <000901cbba0f$ce55b3f0$6b011bd0$@no> <586818.75337.qm@web28510.mail.ukl.yahoo.com> <4D3704A8.8090006@skynet.be> <128171.88697.qm@web25402.mail.ukl. yahoo.com> < 4D382CB9.2060700@ulb.ac.be><6410CABD-9AF3-45A7-9B10-F5EDC78C6095@starpowe r. net> <4D3852CE.6080908@ulb.ac.be> <4D386625.90105@meteo.fr> <4D3869DD.9030708@ulb.ac.be> <4D3873A6.5070104@meteo.fr> <107026.33279.qm@web28515.mail.ukl.yahoo.com> <4D39BC9E.9040108@nhcc.net> <1820574630.137446.1295675648497.JavaMail.ngmail@webmail11.arcor-online.net> <4D3A93A3.8020906@skynet.be> Message-ID: <1896834273.57934.1295686896979.JavaMail.ngmail@webmail13.arcor-online.net> Sven Pran wrote: > On Behalf Of Herman De Wael > > Thomas Dehn wrote: > > > > > > It is MI. Say, your agreement is that bid X shows five controls. > > > You don't remember anymore, and say that you are "unsure". > > > That is MI, you were required to tell opponents that bid X shows five > > > controls. > > > > > > > Yes, but that was not what we were talking about. > > If you say "5 controls" (omitting that you are unsure), have you then given MI? > > I say you haven't. > > I had a similar talk with the Norwegian LC some time ago; our line is that > if the correct agreement cannot be established then we rule that the call > has been made according to agreements. (Ref Law 75) > > This means that a player who is unsure but guesses the correct agreement > has not given MI while a player who does not provide the correct agreement (if > necessary confirmed by other evidence) is always deemed to having given MI. > This includes the "not discussed" and "I have no idea" statements! Looks sensible to me. Thomas From david.j.barton at lineone.net Sat Jan 22 14:37:07 2011 From: david.j.barton at lineone.net (David) Date: Sat, 22 Jan 2011 13:37:07 -0000 Subject: [BLML] Unsure In-Reply-To: <1896834273.57934.1295686896979.JavaMail.ngmail@webmail13.arcor-online.net> References: <000901cbba0f$ce55b3f0$6b011bd0$@no><586818.75337.qm@web28510.mail.ukl.yahoo.com> <4D3704A8.8090006@skynet.be> <128171.88697.qm@web25402.mail.ukl. yahoo.com> < 4D382CB9.2060700@ulb.ac.be><6410CABD-9AF3-45A7-9B10-F5EDC78C6095@starpowe r. net><4D3852CE.6080908@ulb.ac.be> <4D386625.90105@meteo.fr> <4D3869DD.9030708@ulb.ac.be> <4D3873A6.5070104@meteo.fr> <107026.33279.qm@web28515.mail.ukl.yahoo.com> <4D39BC9E.9040108@nhcc.net> <1820574630.137446.1295675648497.JavaMail.ngmail@webmail11.arcor-online.net><4D3A93A3.8020906@skynet.be> <1896834273.57934.1295686896979.JavaMail.ngmail@webmail13.arcor-online.net> Message-ID: -----Original Message----- From: Thomas Dehn Sent: Saturday, January 22, 2011 9:01 AM To: blml at rtflb.org Subject: Re: [BLML] Unsure Sven Pran wrote: > On Behalf Of Herman De Wael > > Thomas Dehn wrote: > > > > > > It is MI. Say, your agreement is that bid X shows five controls. > > > You don't remember anymore, and say that you are "unsure". > > > That is MI, you were required to tell opponents that bid X shows five > > > controls. > > > > > > > Yes, but that was not what we were talking about. > > If you say "5 controls" (omitting that you are unsure), have you then > > given MI? > > I say you haven't. > > I had a similar talk with the Norwegian LC some time ago; our line is that > if the correct agreement cannot be established then we rule that the call > has been made according to agreements. (Ref Law 75) > > This means that a player who is unsure but guesses the correct agreement > has not given MI while a player who does not provide the correct agreement > (if > necessary confirmed by other evidence) is always deemed to having given > MI. > This includes the "not discussed" and "I have no idea" statements! Looks sensible to me. Thomas Well it looks crazy to me. This is effectively saying that every partnership has agreements about every sequence and hence "no agreement" can never be the appropriate answer even when it is absolutely correct. I am glad I do not have to try and enforce anything like that in my local club. I would be down to 2 tables in no time flat. Now I would accept that "no agreement" is rarely a full disclosure as there are likely to be other sequences for which there are agreements which would influence the likely meanings of this particular sequence and these inferences should be disclosed. But at the end of the day, if the partnership does not have an agreement it is not up to a National Authority to deem they have an agreement and penalise them for failing to disclose it. ********************************** david.j.barton at lineone.net ********************************** From ehaa at starpower.net Sat Jan 22 17:07:46 2011 From: ehaa at starpower.net (Eric Landau) Date: Sat, 22 Jan 2011 11:07:46 -0500 Subject: [BLML] Unsure In-Reply-To: <586818.75337.qm@web28510.mail.ukl.yahoo.com> References: <4D3704A8.8090006@skynet.be> <128171.88697.qm@web25402.mail.ukl. yahoo.com> < 4D382CB9.2060700@ulb.ac.be><6410CABD-9AF3-45A7-9B10-F5EDC78C6095@starpowe r. net> <4D3852CE.6080908@ulb.ac.be> <4D386625.90105@meteo.fr> <4D3869DD.9030708@ulb.ac.be> <4D3873A6.5070104@meteo.fr> <107026.33279.qm@web28515.mail.ukl.yahoo.com> <4D39BC9E.9040108@nhcc.net> <586818.75337.qm@web28510.mail.ukl.yahoo.com> Message-ID: <3C6DB678-EF61-46D5-AF42-0E625ECBC3C3@starpower.net> On Jan 21, 2011, at 9:24 PM, Nigel Guthrie wrote: > {Nige1] > If you aren't certain (and that must be the case for most calls > and almost all players), you must answer "unsure". > > {Steve Wilner[ > But this is nonsense. Your duty is to tell opponents your agreements. > If you can't, then they should be compensated if MI causes damage. > Your > personal state of mind about those agreements is irrelevant for MI > purposes. > > If the agreement itself is unclear or if there's no agreement at all, > that's a different matter altogether. "Unsure" still isn't an > explanation, but "no agreement" may be. > > {Nigel] > We don't all live in your world of certainties, Steve. Saying > "unsure" (if that > is the truth) is not MI. In this context, IMO, it is the only > truthful and legal > option. Legal or not, it is the regular practice of many tournament > players. If we apply the Kaplan paradigm, it depends. If you are unsure because the asked-about call is poorly or ambiguously defined in your methods, you must reveal this, and you must disclose those agreements that might be relevant to trying to figure it out. If, however, your lack of certainty is a function of your own mental state rather than your system -- you do have a clear and unambiguous agreement, but you're not sure you remember it -- claiming uncertainty, even though it's the truth, is MI. This doesn't mean much, now that the WBF has effectively repealed the Kaplan paradigm. What it does do is illustrate the importance of having *some* overriding paradigm, so that questions like these can be resolved clearly and consistently. Eric Landau 1107 Dale Drive Silver Spring MD 20910 ehaa at starpower.net From bmeadows666 at gmail.com Sat Jan 22 17:48:36 2011 From: bmeadows666 at gmail.com (Brian) Date: Sat, 22 Jan 2011 11:48:36 -0500 Subject: [BLML] Unsure In-Reply-To: References: <000901cbba0f$ce55b3f0$6b011bd0$@no><586818.75337.qm@web28510.mail.ukl.yahoo.com> <4D3704A8.8090006@skynet.be> <128171.88697.qm@web25402.mail.ukl. yahoo.com> < 4D382CB9.2060700@ulb.ac.be><6410CABD-9AF3-45A7-9B10-F5EDC78C6095@starpowe r. net><4D3852CE.6080908@ulb.ac.be> <4D386625.90105@meteo.fr> <4D3869DD.9030708@ulb.ac.be> <4D3873A6.5070104@meteo.fr> <107026.33279.qm@web28515.mail.ukl.yahoo.com> <4D39BC9E.9040108@nhcc.net> <1820574630.137446.1295675648497.JavaMail.ngmail@webmail11.arcor-online.net><4D3A93A3.8020906@skynet.be> <1896834273.57934.1295686896979.JavaMail.ngmail@webmail13.arcor-online.net> Message-ID: <4D3B0A64.5060306@gmail.com> On 01/22/2011 08:37 AM, David wrote: > > Well it looks crazy to me. > > This is effectively saying that every partnership has agreements about every > sequence > and hence "no agreement" can never be the appropriate answer even when it is > absolutely > correct. > > I am glad I do not have to try and enforce anything like that in my local > club. I would be > down to 2 tables in no time flat. > > Now I would accept that "no agreement" is rarely a full disclosure as there > are likely to > be other sequences for which there are agreements which would influence the > likely > meanings of this particular sequence and these inferences should be > disclosed. But at > the end of the day, if the partnership does not have an agreement it is not > up to a > National Authority to deem they have an agreement and penalise them for > failing to disclose it. > I try to resist posting "Me too" messages but I can't resist it this time. I begin to wonder how many people on this list actually remember what it's like to direct at a grass-roots club. The last club at which I played, and was occasional TD, had the host rota system so as to accommodate single players. If your partner was the host for that night, it meant you had a high probability of playing with someone you'd not partnered before with about two minutes to rough out the details of a system. I think there would have been many occasions where, had the bidding gone (e.g.) 1NT-(dbl)-2D, an inquiry about 2D would have got an answer something like "Well, we agreed transfers, but we didn't have time to agree whether they're on or off over a double, so partner might have either diamonds or hearts, or just possibly he's three suited short in diamonds and is going to redouble your presumed double as an SOS takeout for me to pick one of the other three suits". Like David, I think that penalising such explanations would have been very damaging to the club. This scenario is *the norm* in online bridge, BTW, at least in my experience. SAYC specifies transfers off over a double, AFAIR, but many people seem to believe otherwise. You get that sequence from a pickup partner on BBO, and I reckon it's about even money on which red suit partner has. Brian. From blml at arcor.de Sat Jan 22 20:47:27 2011 From: blml at arcor.de (Thomas Dehn) Date: Sat, 22 Jan 2011 20:47:27 +0100 (CET) Subject: [BLML] Unsure In-Reply-To: References: <000901cbba0f$ce55b3f0$6b011bd0$@no><586818.75337.qm@web28510.mail.ukl.yahoo.com> <4D3704A8.8090006@skynet.be> <128171.88697.qm@web25402.mail.ukl. yahoo.com> < 4D382CB9.2060700@ulb.ac.be><6410CABD-9AF3-45A7-9B10-F5EDC78C6095@starpowe r. net><4D3852CE.6080908@ulb.ac.be> <4D386625.90105@meteo.fr> <4D3869DD.9030708@ulb.ac.be> <4D3873A6.5070104@meteo.fr> <107026.33279.qm@web28515.mail.ukl.yahoo.com> <4D39BC9E.9040108@nhcc.net> <1820574630.137446.1295675648497.JavaMail.ngmail@webmail11.arcor-online.net><4D3A93A3.8020906@skynet.be> <1896834273.57934.1295686896979.JavaMail.ngmail@webmail13.arcor-online.net> Message-ID: <514721598.171618.1295725647449.JavaMail.ngmail@webmail10.arcor-online.net> David wrote: > -----Original Message----- > From: Thomas Dehn > Sent: Saturday, January 22, 2011 9:01 AM > To: blml at rtflb.org > Subject: Re: [BLML] Unsure > > Sven Pran wrote: > > On Behalf Of Herman De Wael > > > Thomas Dehn wrote: > > > > > > > > It is MI. Say, your agreement is that bid X shows five controls. > > > > You don't remember anymore, and say that you are "unsure". > > > > That is MI, you were required to tell opponents that bid X shows five > > > > controls. > > > > > > > > > > Yes, but that was not what we were talking about. > > > If you say "5 controls" (omitting that you are unsure), have you then > > > given MI? > > > I say you haven't. > > > > I had a similar talk with the Norwegian LC some time ago; our line is that > > if the correct agreement cannot be established then we rule that the call > > has been made according to agreements. (Ref Law 75) > > > > This means that a player who is unsure but guesses the correct agreement > > has not given MI while a player who does not provide the correct agreement > > > (if > > necessary confirmed by other evidence) is always deemed to having given > > MI. > > This includes the "not discussed" and "I have no idea" statements! > > Looks sensible to me. > > > > Thomas > > Well it looks crazy to me. > > This is effectively saying that every partnership has agreements about every > sequence and hence "no agreement" can never be the appropriate answer even when it is > absolutely correct. > > I am glad I do not have to try and enforce anything like that in my local > club. I would be down to 2 tables in no time flat. > > Now I would accept that "no agreement" is rarely a full disclosure as there > are likely to be other sequences for which there are agreements which would influence the > likely meanings of this particular sequence and these inferences should be > disclosed. But at the end of the day, if the partnership does not have an agreement it is not > up to a National Authority to deem they have an agreement and penalise them for > failing to disclose it. It is up to the RA to decide to which extent a pair must have agreements. Of course a pickup partnership will have only a skeleton of agreements, and the same holds for beginners. It then is within the RA's rights to force those partnerships to play some standard system. Thomas From david.j.barton at lineone.net Sat Jan 22 22:58:18 2011 From: david.j.barton at lineone.net (David) Date: Sat, 22 Jan 2011 21:58:18 -0000 Subject: [BLML] Unsure In-Reply-To: <514721598.171618.1295725647449.JavaMail.ngmail@webmail10.arcor-online.net> References: <000901cbba0f$ce55b3f0$6b011bd0$@no><586818.75337.qm@web28510.mail.ukl.yahoo.com> <4D3704A8.8090006@skynet.be> <128171.88697.qm@web25402.mail.ukl. yahoo.com> < 4D382CB9.2060700@ulb.ac.be><6410CABD-9AF3-45A7-9B10-F5EDC78C6095@starpowe r. net><4D3852CE.6080908@ulb.ac.be> <4D386625.90105@meteo.fr> <4D3869DD.9030708@ulb.ac.be> <4D3873A6.5070104@meteo.fr> <107026.33279.qm@web28515.mail.ukl.yahoo.com> <4D39BC9E.9040108@nhcc.net> <1820574630.137446.1295675648497.JavaMail.ngmail@webmail11.arcor-online.net><4D3A93A3.8020906@skynet.be><1896834273.57934.1295686896979.JavaMail.ngmail@webmail13.arcor-online.net> <514721598.171618.1295725647449.JavaMail.ngmail@webmail10.arcor-online.net> Message-ID: -----Original Message----- From: Thomas Dehn Sent: Saturday, January 22, 2011 7:47 PM To: blml at rtflb.org Subject: Re: [BLML] Unsure David wrote: > -----Original Message----- > From: Thomas Dehn > Sent: Saturday, January 22, 2011 9:01 AM > To: blml at rtflb.org > Subject: Re: [BLML] Unsure > > Sven Pran wrote: > > On Behalf Of Herman De Wael > > > Thomas Dehn wrote: > > > > > > > > It is MI. Say, your agreement is that bid X shows five controls. > > > > You don't remember anymore, and say that you are "unsure". > > > > That is MI, you were required to tell opponents that bid X shows > > > > five > > > > controls. > > > > > > > > > > Yes, but that was not what we were talking about. > > > If you say "5 controls" (omitting that you are unsure), have you then > > > given MI? > > > I say you haven't. > > > > I had a similar talk with the Norwegian LC some time ago; our line is > > that > > if the correct agreement cannot be established then we rule that the > > call > > has been made according to agreements. (Ref Law 75) > > > > This means that a player who is unsure but guesses the correct agreement > > has not given MI while a player who does not provide the correct > > agreement > > > (if > > necessary confirmed by other evidence) is always deemed to having given > > MI. > > This includes the "not discussed" and "I have no idea" statements! > > Looks sensible to me. > > > > Thomas > > Well it looks crazy to me. > > This is effectively saying that every partnership has agreements about > every > sequence and hence "no agreement" can never be the appropriate answer even > when it is > absolutely correct. > > I am glad I do not have to try and enforce anything like that in my local > club. I would be down to 2 tables in no time flat. > > Now I would accept that "no agreement" is rarely a full disclosure as > there > are likely to be other sequences for which there are agreements which > would influence the > likely meanings of this particular sequence and these inferences should be > disclosed. But at the end of the day, if the partnership does not have an > agreement it is not > up to a National Authority to deem they have an agreement and penalise > them for > failing to disclose it. *It is up to the RA to decide to which extent a pair must have agreements. Law reference for this assertion please. An RA may specify a form of (pre) disclosure (a convention card) and hence specifies situations where partnerships must have an agreement. But outside of this I am not aware of any justification for an obligation for partnerships to have agreements. I suppose an RA could make it a condition of entry to its competitions but we are discussing the laws as applying generally. *Of course a pickup partnership will have only a skeleton of agreements, *and the same holds for beginners. 99.99% of partnerships will come across an undiscussed situation at some point. *It then is within the RA's rights to force those partnerships to play some standard system. And what will that achieve? The probability of coming across such a situation will be thousands of times higher when both members of a partnership are playing a system with which they are not familiar. ********************************** david.j.barton at lineone.net ********************************** From svenpran at online.no Sat Jan 22 23:23:18 2011 From: svenpran at online.no (Sven Pran) Date: Sat, 22 Jan 2011 23:23:18 +0100 Subject: [BLML] Unsure In-Reply-To: References: <000901cbba0f$ce55b3f0$6b011bd0$@no><586818.75337.qm@web28510.mail.ukl.yahoo.com> <4D3704A8.8090006@skynet.be> <128171.88697.qm@web25402.mail.ukl. yahoo.com> < 4D382CB9.2060700@ulb.ac.be><6410CABD-9AF3-45A7-9B10-F5EDC78C6095@starpowe r. net><4D3852CE.6080908@ulb.ac.be> <4D386625.90105@meteo.fr> <4D3869DD.9030708@ulb.ac.be> <4D3873A6.5070104@meteo.fr> <107026.33279.qm@web28515.mail.ukl.yahoo.com> <4D39BC9E.9040108@nhcc.net> <1820574630.137446.1295675648497.JavaMail.ngmail@webmail11.arcor-online.net><4D3A93A3.8020906@skynet.be><1896834273.57934.1295686896979.JavaMail.ngmail@webmail13.arcor-online.net> <514721598.171618.1295725647449.JavaMail.ngmail@webmail10.arcor-online.net> Message-ID: <000901cbba82$f84fdf50$e8ef9df0$@no> On Behalf Of David Sent: 22. januar 2011 22:58 > > Sven Pran wrote: ............. > > > I had a similar talk with the Norwegian LC some time ago; our line > > > is that if the correct agreement cannot be established then we rule > > > that the call has been made according to agreements. (Ref Law 75) > > > > > > This means that a player who is unsure but guesses the correct > > > agreement has not given MI while a player who does not provide the > > > correct agreement > > > > > (if > > > necessary confirmed by other evidence) is always deemed to having > > > given MI. > > > This includes the "not discussed" and "I have no idea" statements! > > > > Looks sensible to me. > > > > > > > > Thomas > > > > Well it looks crazy to me. > > > > This is effectively saying that every partnership has agreements about > > every sequence and hence "no agreement" can never be the appropriate > > answer even when it is absolutely correct. > > > > I am glad I do not have to try and enforce anything like that in my > > local club. I would be down to 2 tables in no time flat. > > > > Now I would accept that "no agreement" is rarely a full disclosure as > > there are likely to be other sequences for which there are agreements > > which would influence the likely meanings of this particular sequence > > and these inferences should be disclosed. But at the end of the day, > > if the partnership does not have an agreement it is not up to a > > National Authority to deem they have an agreement and penalise them > > for failing to disclose it. > > *It is up to the RA to decide to which extent a pair must have agreements. > > Law reference for this assertion please. > An RA may specify a form of (pre) disclosure (a convention card) and hence > specifies situations where partnerships must have an agreement. But outside of > this I am not aware of any justification for an obligation for partnerships to have > agreements. > I suppose an RA could make it a condition of entry to its competitions but we are > discussing the laws as applying generally. > > *Of course a pickup partnership will have only a skeleton of agreements, *and the > same holds for beginners. > > 99.99% of partnerships will come across an undiscussed situation at some point. > > *It then is within the RA's rights to force those partnerships to play some standard > system. > > And what will that achieve? The probability of coming across such a situation will > be thousands of times higher when both members of a partnership are playing a > system with which they are not familiar. I have a very strong feeling that what many of you forget is this: Misinformation is no automatic cause for rectification; rectification shall only be applied if the non-offending side can show probable damage caused by the misinformation. "No agreement" or "undiscussed" is more likely to cause damage to opponents if they in spite of their alleged lack of agreements still most of the time work out a reasonable auction. To me that looks like a strong indication of a probable concealed partnership understanding. Sven From david.j.barton at lineone.net Sun Jan 23 01:31:31 2011 From: david.j.barton at lineone.net (David) Date: Sun, 23 Jan 2011 00:31:31 -0000 Subject: [BLML] Unsure In-Reply-To: <000901cbba82$f84fdf50$e8ef9df0$@no> References: <000901cbba0f$ce55b3f0$6b011bd0$@no><586818.75337.qm@web28510.mail.ukl.yahoo.com> <4D3704A8.8090006@skynet.be> <128171.88697.qm@web25402.mail.ukl. yahoo.com> < 4D382CB9.2060700@ulb.ac.be><6410CABD-9AF3-45A7-9B10-F5EDC78C6095@starpowe r. net><4D3852CE.6080908@ulb.ac.be> <4D386625.90105@meteo.fr> <4D3869DD.9030708@ulb.ac.be> <4D3873A6.5070104@meteo.fr> <107026.33279.qm@web28515.mail.ukl.yahoo.com> <4D39BC9E.9040108@nhcc.net> <1820574630.137446.1295675648497.JavaMail.ngmail@webmail11.arcor-online.net><4D3A93A3.8020906@skynet.be><1896834273.57934.1295686896979.JavaMail.ngmail@webmail13.arcor-online.net> <514721598.171618.1295725647449.JavaMail.ngmail@webmail10.arcor-online.net> <000901cbba82$f84fdf50$e8ef9df0$@no> Message-ID: <41319555EF7045089AF47AA66D4BC62C@Lounge> -----Original Message----- From: Sven Pran Sent: Saturday, January 22, 2011 10:23 PM To: 'Bridge Laws Mailing List' Subject: Re: [BLML] Unsure On Behalf Of David Sent: 22. januar 2011 22:58 > > Sven Pran wrote: ............. > > > I had a similar talk with the Norwegian LC some time ago; our line > > > is that if the correct agreement cannot be established then we rule > > > that the call has been made according to agreements. (Ref Law 75) > > > > > > This means that a player who is unsure but guesses the correct > > > agreement has not given MI while a player who does not provide the > > > correct agreement > > > > > (if > > > necessary confirmed by other evidence) is always deemed to having > > > given MI. > > > This includes the "not discussed" and "I have no idea" statements! > > > > Looks sensible to me. > > > > > > > > Thomas > > > > Well it looks crazy to me. > > > > This is effectively saying that every partnership has agreements about > > every sequence and hence "no agreement" can never be the appropriate > > answer even when it is absolutely correct. > > > > I am glad I do not have to try and enforce anything like that in my > > local club. I would be down to 2 tables in no time flat. > > > > Now I would accept that "no agreement" is rarely a full disclosure as > > there are likely to be other sequences for which there are agreements > > which would influence the likely meanings of this particular sequence > > and these inferences should be disclosed. But at the end of the day, > > if the partnership does not have an agreement it is not up to a > > National Authority to deem they have an agreement and penalise them > > for failing to disclose it. > > *It is up to the RA to decide to which extent a pair must have agreements. > > Law reference for this assertion please. > An RA may specify a form of (pre) disclosure (a convention card) and hence > specifies situations where partnerships must have an agreement. But outside of > this I am not aware of any justification for an obligation for partnerships to have > agreements. > I suppose an RA could make it a condition of entry to its competitions but we are > discussing the laws as applying generally. > > *Of course a pickup partnership will have only a skeleton of agreements, *and the > same holds for beginners. > > 99.99% of partnerships will come across an undiscussed situation at some point. > > *It then is within the RA's rights to force those partnerships to play some standard > system. > > And what will that achieve? The probability of coming across such a situation will > be thousands of times higher when both members of a partnership are playing a > system with which they are not familiar. I have a very strong feeling that what many of you forget is this: Misinformation is no automatic cause for rectification; rectification shall only be applied if the non-offending side can show probable damage caused by the misinformation. "No agreement" or "undiscussed" is more likely to cause damage to opponents if they in spite of their alleged lack of agreements still most of the time work out a reasonable auction. To me that looks like a strong indication of a probable concealed partnership understanding. Sven But here lies the nub of the issue. If you were able to give a REGULAR (and absolutely honest) partnership 10 undiscussed bidding situations you will probably find them in agreement about 8 times. There will be no way to tell in advance which they will agree on because each will involve a judgement as to what your partner thinks that you will think that the bid means. In my opinion such statistics would only provide evidence that the partnership had become good at predicting how partner would act - not that they had pre-existing **agreements** on some or all of the 8 hands they got right. How one instance of judging correctly can provide **strong** indication of a CPU (cheating) is beyond me. ********************************** david.j.barton at lineone.net ********************************** From jfusselman at gmail.com Sun Jan 23 05:20:54 2011 From: jfusselman at gmail.com (Jerry Fusselman) Date: Sat, 22 Jan 2011 22:20:54 -0600 Subject: [BLML] Naive question about double shots Message-ID: Would someone please give an example of an action that he would term a double shot that should not be allowed? Ideally, the example would teach naive bridge players why the laws do not allow double shots. From grabiner at alumni.princeton.edu Sun Jan 23 06:02:13 2011 From: grabiner at alumni.princeton.edu (David Grabiner) Date: Sun, 23 Jan 2011 00:02:13 -0500 Subject: [BLML] Naive question about double shots In-Reply-To: References: Message-ID: The example I usually give is the following (I was East and agreed with the split ruling): North had a 3-3-3-4 6-count, and the following auction occurred, both vulnerable at IMPs: S W N E 1S P 2S ..P P X 3S X AP 3SX went down three for -800. The TD was called because West's double was suggested by East's slow pass, and he ruled that West's double was an infraction. However, he adjusted only the E-W score to +200 for 2S undoubled down two. For N-S, he ruled that the damage was caused by North's 3S bid, which was not a normal bridge action, rather than by the infraction; had North passed, East would have bid 3D making four and N-S would have been -130. Therefore, N-S kept their -800. This is an example of a double shot; North bid 3S on a hand which had no good bridge reason to do so, possibly with the expectation that he would be able to keep his +730 if it was doubled and made, but would get the result rolled back to 2S if it went down. ----- Original Message ----- From: "Jerry Fusselman" To: "Bridge Laws Mailing List" Sent: Saturday, January 22, 2011 11:20 PM Subject: [BLML] Naive question about double shots > Would someone please give an example of an action that he would term a > double shot that should not be allowed? Ideally, the example would > teach naive bridge players why the laws do not allow double shots. > _______________________________________________ > Blml mailing list > Blml at rtflb.org > http://lists.rtflb.org/mailman/listinfo/blml > From grandaeval at tiscali.co.uk Sun Jan 23 05:06:25 2011 From: grandaeval at tiscali.co.uk (Grattan) Date: Sun, 23 Jan 2011 04:06:25 -0000 Subject: [BLML] Unsure References: <128171.88697.qm@web25402.mail.ukl.yahoo.com> <4D380B15.9010308@skynet.be> Message-ID: <8B7ADEBCA04D458694CDCDF576145C42@Mildred> Grattan Endicott To: "Bridge Laws Mailing List" Sent: Thursday, January 20, 2011 10:14 AM Subject: Re: [BLML] Unsure Basically, you are always free to do as you wish, if you are willing to accept the consequences and penalties. But here, I must stress that MI is an irregularity, not an infraction. You tell them what you wish, and accept the rectification afterwards. +=+ If something is not done that the law requires it is a violation of law, an infraction. If something that a player "should" do is not done it is an infraction. ~ Grattan ~ +=+ From Hermandw at skynet.be Sun Jan 23 12:31:37 2011 From: Hermandw at skynet.be (Herman De Wael) Date: Sun, 23 Jan 2011 12:31:37 +0100 Subject: [BLML] Unsure In-Reply-To: <4D3B0A64.5060306@gmail.com> References: <000901cbba0f$ce55b3f0$6b011bd0$@no><586818.75337.qm@web28510.mail.ukl.yahoo.com> <4D3704A8.8090006@skynet.be> <128171.88697.qm@web25402.mail.ukl. yahoo.com> < 4D382CB9.2060700@ulb.ac.be><6410CABD-9AF3-45A7-9B10-F5EDC78C6095@starpowe r. net><4D3852CE.6080908@ulb.ac.be> <4D386625.90105@meteo.fr> <4D3869DD.9030708@ulb.ac.be> <4D3873A6.5070104@meteo.fr> <107026.33279.qm@web28515.mail.ukl.yahoo.com> <4D39BC9E.9040108@nhcc.net> <1820574630.137446.1295675648497.JavaMail.ngmail@webmail11.arcor-online.net><4D3A93A3.8020906@skynet.be> <1896834273.57934.1295686896979.JavaMail.ngmail@webmail13.arcor-online.net> <4D3B0A64.5060306@gmail.com> Message-ID: <4D3C1199.8090100@skynet.be> Brian wrote: > On 01/22/2011 08:37 AM, David wrote: >> >> Well it looks crazy to me. >> >> This is effectively saying that every partnership has agreements about every >> sequence >> and hence "no agreement" can never be the appropriate answer even when it is >> absolutely >> correct. >> >> I am glad I do not have to try and enforce anything like that in my local >> club. I would be >> down to 2 tables in no time flat. >> >> Now I would accept that "no agreement" is rarely a full disclosure as there >> are likely to >> be other sequences for which there are agreements which would influence the >> likely >> meanings of this particular sequence and these inferences should be >> disclosed. But at >> the end of the day, if the partnership does not have an agreement it is not >> up to a >> National Authority to deem they have an agreement and penalise them for >> failing to disclose it. >> > > I try to resist posting "Me too" messages but I can't resist it this > time. I begin to wonder how many people on this list actually remember > what it's like to direct at a grass-roots club. > > The last club at which I played, and was occasional TD, had the host > rota system so as to accommodate single players. If your partner was > the host for that night, it meant you had a high probability of > playing with someone you'd not partnered before with about two minutes > to rough out the details of a system. I think there would have been > many occasions where, had the bidding gone (e.g.) 1NT-(dbl)-2D, an > inquiry about 2D would have got an answer something like > > "Well, we agreed transfers, but we didn't have time to agree whether > they're on or off over a double, so partner might have either diamonds > or hearts, or just possibly he's three suited short in diamonds and is > going to redouble your presumed double as an SOS takeout for me to > pick one of the other three suits". > Well, you've just proven the exact opposite of what you ae argueing against. The above sentence is the correct explanation, and "no agreement" is not the correct explanation. What you seem to forget is that the above explanation is probably what the opponents will understand when you say "no agreement", and therefore that shortcut is acceptable. But that is only so because you and your partner and your opponents belong to the same club. Go with the same opponent to another club, and you will have to start with "well, we never played together and had only two minutes of discussion, but in our club, the majority play ...". If you simply say "no agreement" in that club, you will have misinformed your opponents there. OK? -- Herman De Wael Wilrijk Antwerpen Belgium From david.j.barton at lineone.net Sun Jan 23 13:11:57 2011 From: david.j.barton at lineone.net (David) Date: Sun, 23 Jan 2011 12:11:57 -0000 Subject: [BLML] Unsure In-Reply-To: <4D3C1199.8090100@skynet.be> References: <000901cbba0f$ce55b3f0$6b011bd0$@no><586818.75337.qm@web28510.mail.ukl.yahoo.com> <4D3704A8.8090006@skynet.be> <128171.88697.qm@web25402.mail.ukl. yahoo.com> < 4D382CB9.2060700@ulb.ac.be><6410CABD-9AF3-45A7-9B10-F5EDC78C6095@starpowe r. net><4D3852CE.6080908@ulb.ac.be> <4D386625.90105@meteo.fr> <4D3869DD.9030708@ulb.ac.be> <4D3873A6.5070104@meteo.fr> <107026.33279.qm@web28515.mail.ukl.yahoo.com> <4D39BC9E.9040108@nhcc.net> <1820574630.137446.1295675648497.JavaMail.ngmail@webmail11.arcor-online.net><4D3A93A3.8020906@skynet.be> <1896834273.57934.1295686896979.JavaMail.ngmail@webmail13.arcor-online.net> <4D3B0A64.5060306@gmail.com> <4D3C1199.8090100@skynet.be> Message-ID: -----Original Message----- From: Herman De Wael Sent: Sunday, January 23, 2011 11:31 AM To: Bridge Laws Mailing List Subject: Re: [BLML] Unsure Brian wrote: > On 01/22/2011 08:37 AM, David wrote: >> >> Well it looks crazy to me. >> >> This is effectively saying that every partnership has agreements about >> every >> sequence >> and hence "no agreement" can never be the appropriate answer even when it >> is >> absolutely >> correct. >> >> I am glad I do not have to try and enforce anything like that in my local >> club. I would be >> down to 2 tables in no time flat. >> >> Now I would accept that "no agreement" is rarely a full disclosure as >> there >> are likely to >> be other sequences for which there are agreements which would influence >> the >> likely >> meanings of this particular sequence and these inferences should be >> disclosed. But at >> the end of the day, if the partnership does not have an agreement it is >> not >> up to a >> National Authority to deem they have an agreement and penalise them for >> failing to disclose it. >> > > I try to resist posting "Me too" messages but I can't resist it this > time. I begin to wonder how many people on this list actually remember > what it's like to direct at a grass-roots club. > > The last club at which I played, and was occasional TD, had the host > rota system so as to accommodate single players. If your partner was > the host for that night, it meant you had a high probability of > playing with someone you'd not partnered before with about two minutes > to rough out the details of a system. I think there would have been > many occasions where, had the bidding gone (e.g.) 1NT-(dbl)-2D, an > inquiry about 2D would have got an answer something like > > "Well, we agreed transfers, but we didn't have time to agree whether > they're on or off over a double, so partner might have either diamonds > or hearts, or just possibly he's three suited short in diamonds and is > going to redouble your presumed double as an SOS takeout for me to > pick one of the other three suits". > Well, you've just proven the exact opposite of what you ae argueing against. The above sentence is the correct explanation, and "no agreement" is not the correct explanation. What you seem to forget is that the above explanation is probably what the opponents will understand when you say "no agreement", and therefore that shortcut is acceptable. But that is only so because you and your partner and your opponents belong to the same club. Go with the same opponent to another club, and you will have to start with "well, we never played together and had only two minutes of discussion, but in our club, the majority play ...". If you simply say "no agreement" in that club, you will have misinformed your opponents there. OK? -- Herman De Wael Wilrijk Antwerpen Belgium No not OK. I agree with you that simply saying no agreement is not sufficient. I do NOT agree that the statement that the "in our club the majority .." forms ANY part of your explicit or implicit agreements. I agree it will form part of your rationale for deciding what to call, but so will the respective number of Hearts and Diamonds you hold, your assessment of the difficulties your RHO opponent is having, etc etc. Can you imagine what will happen if you make the true statement that the majority in your club play it as showing hearts and then pass when your RHO passes. ********************************** david.j.barton at lineone.net ********************************** From bmeadows666 at gmail.com Sun Jan 23 13:51:25 2011 From: bmeadows666 at gmail.com (Brian) Date: Sun, 23 Jan 2011 07:51:25 -0500 Subject: [BLML] Unsure In-Reply-To: <4D3C1199.8090100@skynet.be> References: <000901cbba0f$ce55b3f0$6b011bd0$@no><586818.75337.qm@web28510.mail.ukl.yahoo.com> <4D3704A8.8090006@skynet.be> <128171.88697.qm@web25402.mail.ukl. yahoo.com> < 4D382CB9.2060700@ulb.ac.be><6410CABD-9AF3-45A7-9B10-F5EDC78C6095@starpowe r. net><4D3852CE.6080908@ulb.ac.be> <4D386625.90105@meteo.fr> <4D3869DD.9030708@ulb.ac.be> <4D3873A6.5070104@meteo.fr> <107026.33279.qm@web28515.mail.ukl.yahoo.com> <4D39BC9E.9040108@nhcc.net> <1820574630.137446.1295675648497.JavaMail.ngmail@webmail11.arcor-online.net><4D3A93A3.8020906@skynet.be> <1896834273.57934.1295686896979.JavaMail.ngmail@webmail13.arcor-online.net> <4D3B0A64.5060306@gmail.com> <4D3C1199.8090100@skynet.be> Message-ID: <4D3C244D.4090507@gmail.com> On 01/23/2011 06:31 AM, Herman De Wael wrote: > Brian wrote: >> On 01/22/2011 08:37 AM, David wrote: >>> >>> Well it looks crazy to me. >>> >>> This is effectively saying that every partnership has agreements about every >>> sequence >>> and hence "no agreement" can never be the appropriate answer even when it is >>> absolutely >>> correct. >>> >>> I am glad I do not have to try and enforce anything like that in my local >>> club. I would be >>> down to 2 tables in no time flat. >>> >>> Now I would accept that "no agreement" is rarely a full disclosure as there >>> are likely to >>> be other sequences for which there are agreements which would influence the >>> likely >>> meanings of this particular sequence and these inferences should be >>> disclosed. But at >>> the end of the day, if the partnership does not have an agreement it is not >>> up to a >>> National Authority to deem they have an agreement and penalise them for >>> failing to disclose it. >>> >> >> I try to resist posting "Me too" messages but I can't resist it this >> time. I begin to wonder how many people on this list actually remember >> what it's like to direct at a grass-roots club. >> >> The last club at which I played, and was occasional TD, had the host >> rota system so as to accommodate single players. If your partner was >> the host for that night, it meant you had a high probability of >> playing with someone you'd not partnered before with about two minutes >> to rough out the details of a system. I think there would have been >> many occasions where, had the bidding gone (e.g.) 1NT-(dbl)-2D, an >> inquiry about 2D would have got an answer something like >> >> "Well, we agreed transfers, but we didn't have time to agree whether >> they're on or off over a double, so partner might have either diamonds >> or hearts, or just possibly he's three suited short in diamonds and is >> going to redouble your presumed double as an SOS takeout for me to >> pick one of the other three suits". >> > > Well, you've just proven the exact opposite of what you ae argueing > against. The above sentence is the correct explanation, and "no > agreement" is not the correct explanation. What you seem to forget is > that the above explanation is probably what the opponents will > understand when you say "no agreement", and therefore that shortcut is > acceptable. But that is only so because you and your partner and your > opponents belong to the same club. Go with the same opponent to another > club, and you will have to start with "well, we never played together > and had only two minutes of discussion, but in our club, the majority > play ...". If you simply say "no agreement" in that club, you will have > misinformed your opponents there. > > OK? > No, Herman, not OK. I agreed with David's statement (cut and pasted here for convenience) "Now I would accept that "no agreement" is rarely a full disclosure as there are likely to be other sequences for which there are agreements which would influence the likely meanings of this particular sequence and these inferences should be disclosed." As far as I remember the postings in this thread (maybe I snipped too much?) there was a suggestion that giving alternative meanings to an opponent was MI. In the example I gave, clearly there is a possibility the pair are playing transfers, as they do so without the double. Equally, there's a possibility that it's "system off" over the double. If I'm correct about what I believe I read, and the suggestion was that giving an opponent more than one possible meaning and saying "I don't think we have an agreement, you decide" was automatically MI, then I stick by what I said. If I misread/misunderstood something earlier in the thread, then fair enough, my mistake. Brian. From Hermandw at skynet.be Sun Jan 23 15:07:42 2011 From: Hermandw at skynet.be (Herman De Wael) Date: Sun, 23 Jan 2011 15:07:42 +0100 Subject: [BLML] Unsure In-Reply-To: References: <000901cbba0f$ce55b3f0$6b011bd0$@no><586818.75337.qm@web28510.mail.ukl.yahoo.com> <4D3704A8.8090006@skynet.be> <128171.88697.qm@web25402.mail.ukl. yahoo.com> < 4D382CB9.2060700@ulb.ac.be><6410CABD-9AF3-45A7-9B10-F5EDC78C6095@starpowe r. net><4D3852CE.6080908@ulb.ac.be> <4D386625.90105@meteo.fr> <4D3869DD.9030708@ulb.ac.be> <4D3873A6.5070104@meteo.fr> <107026.33279.qm@web28515.mail.ukl.yahoo.com> <4D39BC9E.9040108@nhcc.net> <1820574630.137446.1295675648497.JavaMail.ngmail@webmail11.arcor-online.net><4D3A93A3.8020906@skynet.be> <1896834273.57934.1295686896979.JavaMail.ngmail@webmail13.arcor-online.net> <4D3B0A64.5060306@gmail.com> <4D3C1199.8090100@skynet.be> Message-ID: <4D3C362E.9090107@skynet.be> David wrote: > > > -----Original Message----- > From: Herman De Wael > Sent: Sunday, January 23, 2011 11:31 AM > To: Bridge Laws Mailing List > Subject: Re: [BLML] Unsure > > Brian wrote: >> On 01/22/2011 08:37 AM, David wrote: >>> >>> Well it looks crazy to me. >>> >>> This is effectively saying that every partnership has agreements about >>> every >>> sequence >>> and hence "no agreement" can never be the appropriate answer even when it >>> is >>> absolutely >>> correct. >>> >>> I am glad I do not have to try and enforce anything like that in my local >>> club. I would be >>> down to 2 tables in no time flat. >>> >>> Now I would accept that "no agreement" is rarely a full disclosure as >>> there >>> are likely to >>> be other sequences for which there are agreements which would influence >>> the >>> likely >>> meanings of this particular sequence and these inferences should be >>> disclosed. But at >>> the end of the day, if the partnership does not have an agreement it is >>> not >>> up to a >>> National Authority to deem they have an agreement and penalise them for >>> failing to disclose it. >>> >> >> I try to resist posting "Me too" messages but I can't resist it this >> time. I begin to wonder how many people on this list actually remember >> what it's like to direct at a grass-roots club. >> >> The last club at which I played, and was occasional TD, had the host >> rota system so as to accommodate single players. If your partner was >> the host for that night, it meant you had a high probability of >> playing with someone you'd not partnered before with about two minutes >> to rough out the details of a system. I think there would have been >> many occasions where, had the bidding gone (e.g.) 1NT-(dbl)-2D, an >> inquiry about 2D would have got an answer something like >> >> "Well, we agreed transfers, but we didn't have time to agree whether >> they're on or off over a double, so partner might have either diamonds >> or hearts, or just possibly he's three suited short in diamonds and is >> going to redouble your presumed double as an SOS takeout for me to >> pick one of the other three suits". >> > > Well, you've just proven the exact opposite of what you ae argueing > against. The above sentence is the correct explanation, and "no > agreement" is not the correct explanation. What you seem to forget is > that the above explanation is probably what the opponents will > understand when you say "no agreement", and therefore that shortcut is > acceptable. But that is only so because you and your partner and your > opponents belong to the same club. Go with the same opponent to another > club, and you will have to start with "well, we never played together > and had only two minutes of discussion, but in our club, the majority > play ...". If you simply say "no agreement" in that club, you will have > misinformed your opponents there. > > OK? > No not OK. I agree with you that simply saying no agreement is not sufficient. I do NOT agree that the statement that the "in our club the majority .." forms ANY part of your explicit or implicit agreements. I agree it will form part of your rationale for deciding what to call, but so will the respective number of Hearts and Diamonds you hold, your assessment of the difficulties your RHO opponent is having, etc etc. Can you imagine what will happen if you make the true statement that the majority in your club play it as showing hearts and then pass when your RHO passes. Nothing different from what would happen at your own club when you say "no agreement". Can you imagine what would happen if you say "no agreement" in that other club and then correctly assume what your partner's intention was? My point is that "no agreement" is not true. It always means "no other agreements than those you may readily assume from knowing us both". And that last statement is useless against strangers. -- Herman De Wael Wilrijk Antwerpen Belgium From Hermandw at skynet.be Sun Jan 23 15:12:35 2011 From: Hermandw at skynet.be (Herman De Wael) Date: Sun, 23 Jan 2011 15:12:35 +0100 Subject: [BLML] Unsure In-Reply-To: <4D3C244D.4090507@gmail.com> References: <000901cbba0f$ce55b3f0$6b011bd0$@no><586818.75337.qm@web28510.mail.ukl.yahoo.com> <4D3704A8.8090006@skynet.be> <128171.88697.qm@web25402.mail.ukl. yahoo.com> < 4D382CB9.2060700@ulb.ac.be><6410CABD-9AF3-45A7-9B10-F5EDC78C6095@starpowe r. net><4D3852CE.6080908@ulb.ac.be> <4D386625.90105@meteo.fr> <4D3869DD.9030708@ulb.ac.be> <4D3873A6.5070104@meteo.fr> <107026.33279.qm@web28515.mail.ukl.yahoo.com> <4D39BC9E.9040108@nhcc.net> <1820574630.137446.1295675648497.JavaMail.ngmail@webmail11.arcor-online.net><4D3A93A3.8020906@skynet.be> <1896834273.57934.1295686896979.JavaMail.ngmail@webmail13.arcor-online.net> <4D3B0A64.5060306@gmail.com> <4D3C1199.8090100@skynet.be> <4D3C244D.4090507@gmail.com> Message-ID: <4D3C3753.8060008@skynet.be> Brian wrote: > > No, Herman, not OK. > > I agreed with David's statement (cut and pasted here for convenience) > "Now I would accept that "no agreement" is rarely a full disclosure as > there are likely to be other sequences for which there are agreements > which would influence the likely meanings of this particular sequence > and these inferences should be disclosed." > > As far as I remember the postings in this thread (maybe I snipped too > much?) there was a suggestion that giving alternative meanings to an > opponent was MI. > > In the example I gave, clearly there is a possibility the pair are > playing transfers, as they do so without the double. Equally, there's > a possibility that it's "system off" over the double. > > If I'm correct about what I believe I read, and the suggestion was > that giving an opponent more than one possible meaning and saying "I > don't think we have an agreement, you decide" was automatically MI, > then I stick by what I said. > > If I misread/misunderstood something earlier in the thread, then fair > enough, my mistake. > The problem with your point of view is that, true though it is, it helps us no further along. When you have to guess at partner's intentions, and you guess correctly, then there is always the possibility that your guess was based on something you did not mention. That something may be enough for the TD to rule that there has been MI. In that light, to be calling it MI when you guess and tell them, is going far too way left field. And that is what some people in this thread suggested. > > Brian. > > > > _______________________________________________ > Blml mailing list > Blml at rtflb.org > http://lists.rtflb.org/mailman/listinfo/blml > > > > > No virus found in this incoming message. > Checked by AVG - www.avg.com > Version: 9.0.872 / Virus Database: 271.1.1/3398 - Release Date: 01/23/11 08:34:00 > -- Herman De Wael Wilrijk Antwerpen Belgium From PeterEidt at t-online.de Sun Jan 23 20:50:59 2011 From: PeterEidt at t-online.de (Peter Eidt) Date: Sun, 23 Jan 2011 20:50:59 +0100 Subject: [BLML] =?utf-8?q?Naive_question_about_double_shots?= In-Reply-To: References: Message-ID: <1Ph5xk-1j3FTc0@fwd04.aul.t-online.de> From: "olivier.beauvillain" > isn't the exact ruling to give : > EW 130 in 3D+1, the worse of scores (130/200/800) > and NS -800 [Peter] No. EW's score has to be adjusted on the basis of the infraction (X) not being committed and therefore the -130 score not being possible. > ----- Original Message ----- > From: "David Grabiner" > To: "Bridge Laws Mailing List" > Sent: Sunday, January 23, 2011 6:02 AM > Subject: Re: [BLML] Naive question about double shots > > > > > The example I usually give is the following (I was East and agreed > > with the > > split ruling): > > > > > > North had a 3-3-3-4 6-count, and the following auction occurred, > > both vulnerable > > at IMPs: > > > > > > S W N E > > 1S P 2S ..P > > P X 3S X > > AP > > > > 3SX went down three for -800. The TD was called because West's > > double was suggested by East's slow pass, and he ruled that West's > > double was an infraction. However, he adjusted only the E-W score > > to +200 for 2S undoubled > > down two. For N-S, he ruled that the damage was caused by North's > > 3S bid, which > > was not a normal bridge action, rather than by the infraction; had > > North passed, > > East would have bid 3D making four and N-S would have been -130. > > Therefore, N-S > > kept their -800. > > > > > > This is an example of a double shot; North bid 3S on a hand which > > had no good > > bridge reason to do so, possibly with the expectation that he would > > be able to > > keep his +730 if it was doubled and made, but would get the result > > rolled back > > to 2S if it went down. > > > > > > ----- Original Message ----- > > From: "Jerry Fusselman" > > To: "Bridge Laws Mailing List" > > Sent: Saturday, January 22, 2011 11:20 PM > > Subject: [BLML] Naive question about double shots > > > > > >> Would someone please give an example of an action that he would > term a >> double shot that should not be allowed? Ideally, the > example would >> teach naive bridge players why the laws do not allow > double shots. > >> _______________________________________________ > >> Blml mailing list > >> Blml at rtflb.org > >> http://lists.rtflb.org/mailman/listinfo/blml > >> > > > > > > > _______________________________________________ > > Blml mailing list > > Blml at rtflb.org > > http://lists.rtflb.org/mailman/listinfo/blml > > > > __________ Information provenant d'ESET Smart Security, version de > > la base des signatures de virus 5809 (20110122) __________ > > > > > > Le message a ?t? v?rifi? par ESET Smart Security. > > > > http://www.eset.com > > > > > > > > > __________ Information provenant d'ESET Smart Security, version de la > base des signatures de virus 5809 (20110122) __________ > Le message a From swillner at nhcc.net Mon Jan 24 00:02:08 2011 From: swillner at nhcc.net (Steve Willner) Date: Sun, 23 Jan 2011 18:02:08 -0500 Subject: [BLML] Naive question about double shots In-Reply-To: References: Message-ID: <4D3CB370.3050005@nhcc.net> On 1/23/2011 12:02 AM, David Grabiner wrote: > This is an example of a double shot; North bid 3S on a hand which had no good > bridge reason to do so, possibly with the expectation that he would be able to > keep his +730 if it was doubled and made, but would get the result rolled back > to 2S if it went down. There's a long history of thinking this is a bad thing, but I think Jerry's question was _why_ it's bad. Except in the ACBL after 2008, I thought the OS side score could include the infraction, so I too don't see why they don't get +130. (This seems to be the same situation Marvin has been writing about.) From david.j.barton at lineone.net Mon Jan 24 00:35:56 2011 From: david.j.barton at lineone.net (David) Date: Sun, 23 Jan 2011 23:35:56 -0000 Subject: [BLML] Unsure In-Reply-To: <4D3C3753.8060008@skynet.be> References: <000901cbba0f$ce55b3f0$6b011bd0$@no><586818.75337.qm@web28510.mail.ukl.yahoo.com> <4D3704A8.8090006@skynet.be> <128171.88697.qm@web25402.mail.ukl. yahoo.com> < 4D382CB9.2060700@ulb.ac.be><6410CABD-9AF3-45A7-9B10-F5EDC78C6095@starpowe r. net><4D3852CE.6080908@ulb.ac.be> <4D386625.90105@meteo.fr> <4D3869DD.9030708@ulb.ac.be> <4D3873A6.5070104@meteo.fr> <107026.33279.qm@web28515.mail.ukl.yahoo.com> <4D39BC9E.9040108@nhcc.net> <1820574630.137446.1295675648497.JavaMail.ngmail@webmail11.arcor-online.net><4D3A93A3.8020906@skynet.be> <1896834273.57934.1295686896979.JavaMail.ngmail@webmail13.arcor-online.net> <4D3B0A64.5060306@gmail.com> <4D3C1199.8090100@skynet.be><4D3C244D.4090507@gmail.com> <4D3C3753.8060008@skynet.be> Message-ID: -----Original Message----- From: Herman De Wael Sent: Sunday, January 23, 2011 2:12 PM To: Bridge Laws Mailing List Subject: Re: [BLML] Unsure Brian wrote: > > No, Herman, not OK. > > I agreed with David's statement (cut and pasted here for convenience) > "Now I would accept that "no agreement" is rarely a full disclosure as > there are likely to be other sequences for which there are agreements > which would influence the likely meanings of this particular sequence > and these inferences should be disclosed." > > As far as I remember the postings in this thread (maybe I snipped too > much?) there was a suggestion that giving alternative meanings to an > opponent was MI. > > In the example I gave, clearly there is a possibility the pair are > playing transfers, as they do so without the double. Equally, there's > a possibility that it's "system off" over the double. > > If I'm correct about what I believe I read, and the suggestion was > that giving an opponent more than one possible meaning and saying "I > don't think we have an agreement, you decide" was automatically MI, > then I stick by what I said. > > If I misread/misunderstood something earlier in the thread, then fair > enough, my mistake. > *The problem with your point of view is that, true though it is, it helps *us no further along. When you have to guess at partner's intentions, and *you guess correctly, then there is always the possibility that your *guess was based on something you did not mention. That something may be *enough for the TD to rule that there has been MI. * *In that light, to be calling it MI when you guess and tell them, is *going far too way left field. * *And that is what some people in this thread suggested. Let me give you a real life example. (P) P (1S) 1N (2S) 2N I alert the 2N and explain that it MAY be conventional. Upon further questioning I explain that we have no agreement or previous experience that covers this situation but we do have the following agreements (P) P (1S) 1N//(P) 2N would be natural invitational 1N (2S) 2N would be lebensohl 1N (P) 2N would be transfer to D (P) P (1S) 1N//(2S) X would be take out as it is covered by our general agreement on doubles So I have a straight coin flip between natural and lebensohl right? WRONG. Spades have been bid and supported together with the three I am looking at implies partner is likely to be short. Both the oppos bidding reduces the likelihood that partner holds the values for an invite. The guy on my right is clearly thinking of bidding again reinforcing the above 2 points. If partner has an invite he MAY have chosen a different route (3N or X) because he will know that 2N is undiscussed. When I place my bet on lebensohl I would guess I have an 80% chance of being right. These sort of inferences are ALWAYS available and it is no surprise to me that (honest) partnerships get these decisions right far more often than they get them wrong. Evidence of a CPU? You must be joking. ********************************** david.j.barton at lineone.net ********************************** From grabiner at alumni.princeton.edu Mon Jan 24 01:54:15 2011 From: grabiner at alumni.princeton.edu (David Grabiner) Date: Sun, 23 Jan 2011 19:54:15 -0500 Subject: [BLML] Naive question about double shots In-Reply-To: References: Message-ID: The score for the OS is the worst score that was at all probable, and may (under many interpretations of the Laws) include scores which could have occurred only with the infraction. However, we do not need to deal with probabilities except in counterfactual situations; we know what results would have occurred if West had doubled 2S, because that is what actually happened at the table. ----- Original Message ----- From: "olivier.beauvillain" To: "Bridge Laws Mailing List" Sent: Sunday, January 23, 2011 6:31 AM Subject: Re: [BLML] Naive question about double shots > isn't the exact ruling to give : > EW 130 in 3D+1, the worse of scores (130/200/800) > and > NS -800 > Olivier Beauvillain > http://www.lebridgeur.com/shopping-bridge/livres/commentaires-code-2007.html > Pour commander : me contacter par courriel > 29,5?+port > ----- Original Message ----- > From: "David Grabiner" > To: "Bridge Laws Mailing List" > Sent: Sunday, January 23, 2011 6:02 AM > Subject: Re: [BLML] Naive question about double shots > > >> The example I usually give is the following (I was East and agreed with >> the >> split ruling): >> >> North had a 3-3-3-4 6-count, and the following auction occurred, both >> vulnerable >> at IMPs: >> >> S W N E >> 1S P 2S ..P >> P X 3S X >> AP >> >> 3SX went down three for -800. The TD was called because West's double was >> suggested by East's slow pass, and he ruled that West's double was an >> infraction. However, he adjusted only the E-W score to +200 for 2S >> undoubled >> down two. For N-S, he ruled that the damage was caused by North's 3S bid, >> which >> was not a normal bridge action, rather than by the infraction; had North >> passed, >> East would have bid 3D making four and N-S would have been -130. >> Therefore, N-S >> kept their -800. >> >> This is an example of a double shot; North bid 3S on a hand which had no >> good >> bridge reason to do so, possibly with the expectation that he would be >> able to >> keep his +730 if it was doubled and made, but would get the result rolled >> back >> to 2S if it went down. >> >> ----- Original Message ----- >> From: "Jerry Fusselman" >> To: "Bridge Laws Mailing List" >> Sent: Saturday, January 22, 2011 11:20 PM >> Subject: [BLML] Naive question about double shots >> >> >>> Would someone please give an example of an action that he would term a >>> double shot that should not be allowed? Ideally, the example would >>> teach naive bridge players why the laws do not allow double shots. >>> _______________________________________________ >>> Blml mailing list >>> Blml at rtflb.org >>> http://lists.rtflb.org/mailman/listinfo/blml >>> >> >> >> _______________________________________________ >> Blml mailing list >> Blml at rtflb.org >> http://lists.rtflb.org/mailman/listinfo/blml >> >> __________ Information provenant d'ESET Smart Security, version de la base >> des signatures de virus 5809 (20110122) __________ >> >> Le message a ?t? v?rifi? par ESET Smart Security. >> >> http://www.eset.com >> >> >> > > > __________ Information provenant d'ESET Smart Security, version de la base des > signatures de virus 5809 (20110122) __________ > > Le message a ?t? v?rifi? par ESET Smart Security. > > http://www.eset.com > > > > -------------------------------------------------------------------------------- > _______________________________________________ > Blml mailing list > Blml at rtflb.org > http://lists.rtflb.org/mailman/listinfo/blml > From grandaeval at tiscali.co.uk Mon Jan 24 02:46:34 2011 From: grandaeval at tiscali.co.uk (Grattan) Date: Mon, 24 Jan 2011 01:46:34 -0000 Subject: [BLML] Unsure References: <000901cbba0f$ce55b3f0$6b011bd0$@no><586818.75337.qm@web28510.mail.ukl.yahoo.com> <4D3704A8.8090006@skynet.be> <128171.88697.qm@web25402.mail.ukl. yahoo.com> < 4D382CB9.2060700@ulb.ac.be><6410CABD-9AF3-45A7-9B10-F5EDC78C6095@starpowe r. net><4D3852CE.6080908@ulb.ac.be> <4D386625.90105@meteo.fr> <4D3869DD.9030708@ulb.ac.be> <4D3873A6.5070104@meteo.fr> <107026.33279.qm@web28515.mail.ukl.yahoo.com> <4D39BC9E.9040108@nhcc.net> <1820574630.137446.1295675648497.JavaMail.ngmail@webmail11.arcor-online.net><4D3A93A3.8020906@skynet.be><1896834273.57934.1295686896979.JavaMail.ngmail@webmail13.arcor-online.net><514721598.171618.1295725647449.JavaMail.ngmail@webmail10.arcor-online.net> Message-ID: <57164744586E451BB649853C5401A7E5@Mildred> Grattan Endicott To: "Bridge Laws Mailing List" Sent: Saturday, January 22, 2011 9:58 PM Subject: Re: [BLML] Unsure > > *It is up to the RA to decide to which extent > a pair must have agreements. > > Law reference for this assertion please. > An RA may specify a form of (pre) disclosure > (a convention card) and hence specifies situations > where partnerships must have an agreement. But > outside of this I am not aware of any justification for an obligation for partnerships to have agreements. > > ********************************** > david.j.barton at lineone.net > ********************************** ::::::::::::::::::::::::::::::::::::::::::::::::::::::::::::::::: +=+ Some clarification is needed of the assertions appearing in this topic. The RA regulates the use of system cards. It tells players what information must be included on the card. This does not override the requirements in Law 20F1 as to what an opponent is entitled to know. So in reply to a lawful enquiry a player must ensure that all such knowledge is given to his opponent; merely referring opponent to the system card is insufficient unless all the information is present on it. Failure to provide the information is an infraction and will result in sanctions if opponent is damaged because he did not receive a due explanation. ~ Grattan ~ +=+ From Hermandw at skynet.be Mon Jan 24 09:41:19 2011 From: Hermandw at skynet.be (Herman De Wael) Date: Mon, 24 Jan 2011 09:41:19 +0100 Subject: [BLML] Exam question Message-ID: <4D3D3B2F.5090107@skynet.be> This one may well make it into an exam question for a TD course, if you ever need one: On Saturday, I was playing 2He. I had just won the first six tricks, and I showed AK of hearts, saying "I think I am going to make this contract". Did I claim or am I allowed to play on? -- Herman De Wael Wilrijk Antwerpen Belgium From Hermandw at skynet.be Mon Jan 24 09:45:44 2011 From: Hermandw at skynet.be (Herman De Wael) Date: Mon, 24 Jan 2011 09:45:44 +0100 Subject: [BLML] Unsure In-Reply-To: References: <000901cbba0f$ce55b3f0$6b011bd0$@no><586818.75337.qm@web28510.mail.ukl.yahoo.com> <4D3704A8.8090006@skynet.be> <128171.88697.qm@web25402.mail.ukl. yahoo.com> < 4D382CB9.2060700@ulb.ac.be><6410CABD-9AF3-45A7-9B10-F5EDC78C6095@starpowe r. net><4D3852CE.6080908@ulb.ac.be> <4D386625.90105@meteo.fr> <4D3869DD.9030708@ulb.ac.be> <4D3873A6.5070104@meteo.fr> <107026.33279.qm@web28515.mail.ukl.yahoo.com> <4D39BC9E.9040108@nhcc.net> <1820574630.137446.1295675648497.JavaMail.ngmail@webmail11.arcor-online.net><4D3A93A3.8020906@skynet.be> <1896834273.57934.1295686896979.JavaMail.ngmail@webmail13.arcor-online.net> <4D3B0A64.5060306@gmail.com> <4D3C1199.8090100@skynet.be><4D3C244D.4090507@gmail.com> <4D3C3753.8060008@skynet.be> Message-ID: <4D3D3C38.2080701@skynet.be> David wrote: > > Let me give you a real life example. > Thank you, but I don't quite grasp which way you are argueing here, and with/against whom. > (P) P (1S) 1N > (2S) 2N > > I alert the 2N and explain that it MAY be conventional. > Upon further questioning I explain that we have no agreement > or previous experience that covers this situation but we do have > the following agreements > > (P) P (1S) 1N//(P) 2N would be natural invitational > 1N (2S) 2N would be lebensohl > 1N (P) 2N would be transfer to D > (P) P (1S) 1N//(2S) X would be take out as it is covered > by our general agreement on doubles > > So I have a straight coin flip between natural and lebensohl right? > > WRONG. > > Spades have been bid and supported together with the three I am looking > at implies partner is likely to be short. > > Both the oppos bidding reduces the likelihood that partner holds the values > for an invite. > > The guy on my right is clearly thinking of bidding again reinforcing the > above 2 points. > > If partner has an invite he MAY have chosen a different route (3N or X) > because he will know that 2N is undiscussed. > > When I place my bet on lebensohl I would guess I have an 80% chance > of being right. > > These sort of inferences are ALWAYS available and it is no surprise to > me that (honest) partnerships get these decisions right far more often > than they get them wrong. > And your point is? Honest partnerships understand one another better than dishonest ones? I really don't understand what you are getting at. But you are right of course. > Evidence of a CPU? Well, evidence of a PU, certainly. It's a CPU only if it remains concealed. > You must be joking. > I'm not laughing. > -- Herman De Wael Wilrijk Antwerpen Belgium From PeterEidt at t-online.de Mon Jan 24 09:56:20 2011 From: PeterEidt at t-online.de (Peter Eidt) Date: Mon, 24 Jan 2011 09:56:20 +0100 Subject: [BLML] =?utf-8?q?Exam_question?= In-Reply-To: <4D3D3B2F.5090107@skynet.be> References: <4D3D3B2F.5090107@skynet.be> Message-ID: <1PhIDl-0dAMts0@fwd00.aul.t-online.de> From: Herman De Wael > This one may well make it into an exam question for a TD course, if > you ever need one: > > On Saturday, I was playing 2He. I had just won the first six tricks, > and I showed AK of hearts, saying "I think I am going to make this > contract". Did I claim or am I allowed to play on? Law 68: "For a statement or action to constitute a claim or concession of tricks under these Laws, it must refer to tricks other than one currently in progress. If it does refer to subsequent tricks: A. Claim Defined Any statement to the effect that a contestant will win a specific number of tricks is a claim of those tricks. A contestant also claims when he suggests that play be curtailed, or when he shows his cards (unless he demonstrably did not intend to claim - for example, if declarer faces his cards after an opening lead out of turn Law 54, not this Law, will apply)." [Peter] Yes, you did claim and no, you are not allowed to play on. From david.j.barton at lineone.net Mon Jan 24 13:13:25 2011 From: david.j.barton at lineone.net (David) Date: Mon, 24 Jan 2011 12:13:25 -0000 Subject: [BLML] Unsure In-Reply-To: <4D3D3C38.2080701@skynet.be> References: <000901cbba0f$ce55b3f0$6b011bd0$@no><586818.75337.qm@web28510.mail.ukl.yahoo.com> <4D3704A8.8090006@skynet.be> <128171.88697.qm@web25402.mail.ukl. yahoo.com> < 4D382CB9.2060700@ulb.ac.be><6410CABD-9AF3-45A7-9B10-F5EDC78C6095@starpowe r. net><4D3852CE.6080908@ulb.ac.be> <4D386625.90105@meteo.fr> <4D3869DD.9030708@ulb.ac.be> <4D3873A6.5070104@meteo.fr> <107026.33279.qm@web28515.mail.ukl.yahoo.com> <4D39BC9E.9040108@nhcc.net> <1820574630.137446.1295675648497.JavaMail.ngmail@webmail11.arcor-online.net><4D3A93A3.8020906@skynet.be> <1896834273.57934.1295686896979.JavaMail.ngmail@webmail13.arcor-online.net> <4D3B0A64.5060306@gmail.com> <4D3C1199.8090100@skynet.be><4D3C244D.4090507@gmail.com> <4D3C3753.8060008@skynet.be> <4D3D3C38.2080701@skynet.be> Message-ID: <46D99948D8E446A9995179C055A0B47B@Lounge> -----Original Message----- From: Herman De Wael Sent: Monday, January 24, 2011 8:45 AM To: Bridge Laws Mailing List Subject: Re: [BLML] Unsure David wrote: > > Let me give you a real life example. > Thank you, but I don't quite grasp which way you are argueing here, and with/against whom. > (P) P (1S) 1N > (2S) 2N > > I alert the 2N and explain that it MAY be conventional. > Upon further questioning I explain that we have no agreement > or previous experience that covers this situation but we do have > the following agreements > > (P) P (1S) 1N//(P) 2N would be natural invitational > 1N (2S) 2N would be lebensohl > 1N (P) 2N would be transfer to D > (P) P (1S) 1N//(2S) X would be take out as it is covered > by our general agreement on doubles > > So I have a straight coin flip between natural and lebensohl right? > > WRONG. > > Spades have been bid and supported together with the three I am looking > at implies partner is likely to be short. > > Both the oppos bidding reduces the likelihood that partner holds the > values > for an invite. > > The guy on my right is clearly thinking of bidding again reinforcing the > above 2 points. > > If partner has an invite he MAY have chosen a different route (3N or X) > because he will know that 2N is undiscussed. > > When I place my bet on lebensohl I would guess I have an 80% chance > of being right. > > These sort of inferences are ALWAYS available and it is no surprise to > me that (honest) partnerships get these decisions right far more often > than they get them wrong. > *And your point is? *Honest partnerships understand one another better than dishonest ones? *I really don't understand what you are getting at. *But you are right of course. The points I was attempting to make was that the Norwegian LC position as given by Sven:- *I had a similar talk with the Norwegian LC some time ago; our line is that *if the correct agreement cannot be established then we rule that the call *has been made according to agreements. (Ref Law 75) * *This means that a player who is unsure but guesses the correct agreement has *not given MI while a player who does not provide the correct agreement (if *necessary confirmed by other evidence) is always deemed to having given MI. *This includes the "not discussed" and "I have no idea" statements! * *Harsh? No I don't think so. The prime intention with Law 20 is to protect *opponents. is neither desirable (in that it will kill Club duplicate stone dead), nor legal because the example in L25 does not cover the case of no agreement and it puts the onus on the pair to prove the impossible (that an agreement does not exist). I was also taking issue with Sven's assertion *"No agreement" or "undiscussed" is more likely to cause damage to opponents *if they in spite of their alleged lack of agreements still most of the time *work out a reasonable auction. To me that looks like a strong indication of *a probable concealed partnership understanding. by arguing that "No agreement" is perfectly legal and hence cannot "cause damage" (since damage can ONLY arise as a result of an infraction) and that to "work out a reasonable auction" is most definitely NOT "a strong indication of a probable concealed partnership understanding" in that partnerships will arrive at this "reasonable auction" far more often than not following a genuine "no agreement" auction. ********************************** david.j.barton at lineone.net ********************************** From jfusselman at gmail.com Mon Jan 24 13:24:37 2011 From: jfusselman at gmail.com (Jerry Fusselman) Date: Mon, 24 Jan 2011 06:24:37 -0600 Subject: [BLML] Exam question In-Reply-To: <1PhIDl-0dAMts0@fwd00.aul.t-online.de> References: <4D3D3B2F.5090107@skynet.be> <1PhIDl-0dAMts0@fwd00.aul.t-online.de> Message-ID: [Herman] This one may well make it into an exam question for a TD course, if you ever need one: On Saturday, I was playing 2He. I had just won the first six tricks, and I showed AK of hearts, saying "I think I am going to make this contract". Did I claim or am I allowed to play on? [Law 68] For a statement or action to constitute a claim or concession of tricks under these Laws, it must refer to tricks other than one currently in progress. If it does refer to subsequent tricks: [Jerry] This is a necessary condition, not a sufficient condition. Referring to tricks other than the one currently in progress is not sufficient to make it a claim. [Law 68A. Claim Defined] Any statement to the effect that a contestant will win a specific number of tricks is a claim of those tricks. A contestant also claims when he suggests that play be curtailed, or when he shows his cards (unless he demonstrably did not intend to claim - for example, if declarer faces his cards after an opening lead out of turn Law 54, not this Law, will apply)." [Jerry] There are three ways sufficient to make it a claim: A statement of winning a specific number, a statement suggesting play be curtailed, showing his cards---but there are exceptions to the latter. We have none of these here. He did not suggest play be curtailed, he did not imply a specific number, he did not show all of his cards. He stated he would take 8 or more; he did not state he would take specifically 8. So a mathematician naively reading the law would conclude that it not a claim. However, what happens next if the defenders think it is a claim and face their hands but declarer wants to play on? [Peter] Yes, you did claim and no, you are not allowed to play on. [Jerry] Peter, would you face your hand? From agot at ulb.ac.be Mon Jan 24 13:31:48 2011 From: agot at ulb.ac.be (Alain Gottcheiner) Date: Mon, 24 Jan 2011 13:31:48 +0100 Subject: [BLML] Unsure In-Reply-To: <1820574630.137446.1295675648497.JavaMail.ngmail@webmail11.arcor-online.net> References: <586818.75337.qm@web28510.mail.ukl.yahoo.com> <4D3704A8.8090006@skynet.be> <128171.88697.qm@web25402.mail.ukl. yahoo.com> < 4D382CB9.2060700@ulb.ac.be><6410CABD-9AF3-45A7-9B10-F5EDC78C6095@starpowe r. net> <4D3852CE.6080908@ulb.ac.be> <4D386625.90105@meteo.fr> <4D3869DD.9030708@ulb.ac.be> <4D3873A6.5070104@meteo.fr> <107026.33279.qm@web28515.mail.ukl.yahoo.com> <4D39BC9E.9040108@nhcc.net> <1820574630.137446.1295675648497.JavaMail.ngmail@webmail11.arcor-online.net> Message-ID: <4D3D7134.4000803@ulb.ac.be> Le 22/01/2011 6:54, Thomas Dehn a ?crit : > Nigel Guthrie wrote: >> {Nige1] >> If you aren't certain (and that must be the case for most calls >> >> and almost all players), you must answer "unsure". >> >> {Steve Wilner[ >> But this is nonsense. Your duty is to tell opponents your agreements. >> If you can't, then they should be compensated if MI causes damage. Your >> personal state of mind about those agreements is irrelevant for MI >> purposes. >> >> If the agreement itself is unclear or if there's no agreement at all, >> that's a different matter altogether. "Unsure" still isn't an >> explanation, but "no agreement" may be. >> >> {Nigel] >> We don't all live in your world of certainties, Steve. Saying "unsure" (if >> that is the truth) is not MI. In this context, IMO, it is the only truthful and >> legal option. Legal or not, it is the regular practice of many tournament >> players. > It is MI. Say, your agreement is that bid X shows five controls. > You don't remember anymore, and say that you are "unsure". > That is MI, you were required to tell opponents > that bid X shows five controls. > AG : indeed. "conflicting agreements" might be the right explanation ; "unsure" will never be. You are unsure, not the system, and TFLB says you must speak about the system, not about your state of mind. From agot at ulb.ac.be Mon Jan 24 13:38:07 2011 From: agot at ulb.ac.be (Alain Gottcheiner) Date: Mon, 24 Jan 2011 13:38:07 +0100 Subject: [BLML] Unsure In-Reply-To: <000901cbba0f$ce55b3f0$6b011bd0$@no> References: <586818.75337.qm@web28510.mail.ukl.yahoo.com> <4D3704A8.8090006@skynet.be> <128171.88697.qm@web25402.mail.ukl. yahoo.com> < 4D382CB9.2060700@ulb.ac.be><6410CABD-9AF3-45A7-9B10-F5EDC78C6095@starpowe r. net> <4D3852CE.6080908@ulb.ac.be> <4D386625.90105@meteo.fr> <4D3869DD.9030708@ulb.ac.be> <4D3873A6.5070104@meteo.fr> <107026.33279.qm@web28515.mail.ukl.yahoo.com> <4D39BC9E.9040108@nhcc.net> <1820574630.137446.1295675648497.JavaMail.ngmail@webmail11.arcor-online.net> <4D3A93A3.8020906@skynet.be> <000901cbba0f$ce55b3f0$6b011bd0$@no> Message-ID: <4D3D72AF.3060606@ulb.ac.be> Le 22/01/2011 9:38, Sven Pran a ?crit : > On Behalf Of Herman De Wael >> Thomas Dehn wrote: >>> It is MI. Say, your agreement is that bid X shows five controls. >>> You don't remember anymore, and say that you are "unsure". >>> That is MI, you were required to tell opponents that bid X shows five >>> controls. >>> >> Yes, but that was not what we were talking about. >> If you say "5 controls" (omitting that you are unsure), have you then > given MI? >> I say you haven't. > I had a similar talk with the Norwegian LC some time ago; our line is that > if the correct agreement cannot be established then we rule that the call > has been made according to agreements. (Ref Law 75) > > This means that a player who is unsure but guesses the correct agreement has > not given MI while a player who does not provide the correct agreement (if > necessary confirmed by other evidence) is always deemed to having given MI. > This includes the "not discussed" and "I have no idea" statements! AG : there must be limits to this. How long your list of agremeents, there must be some non-covered area. The limits of "undiscussion" must be determined according to the competition level. At intermediate level, I don't think many would have discussed their interventions after e.g. 1C - pass - 2D (both minors, 8+), although some would be able to apply meta-agreements. And there are always the really strange cases. The Belgian international who said to me that the unopposed sequence 1NT-2C-3NT was undiscussed (playing 3-step Stayman) would have had a strange feeling if you had told him that he gave MI. (the explanation : partner had found an Ace back) Best regards Alain From svenpran at online.no Mon Jan 24 13:41:32 2011 From: svenpran at online.no (Sven Pran) Date: Mon, 24 Jan 2011 13:41:32 +0100 Subject: [BLML] Exam question In-Reply-To: References: <4D3D3B2F.5090107@skynet.be> <1PhIDl-0dAMts0@fwd00.aul.t-online.de> Message-ID: <001101cbbbc4$08087ff0$18197fd0$@no> On Behalf Of Jerry Fusselman > [Herman] > This one may well make it into an exam question for a TD course, if you ever need > one: > > On Saturday, I was playing 2He. I had just won the first six tricks, and I showed AK > of hearts, saying "I think I am going to make this contract". Did I claim or am I > allowed to play on? > > [Law 68] > For a statement or action to constitute a claim or concession of tricks under these > Laws, it must refer to tricks other than one currently in progress. If it does refer to > subsequent tricks: > > [Jerry] > This is a necessary condition, not a sufficient condition. Referring to tricks other > than the one currently in progress is not sufficient to make it a claim. > > [Law 68A. Claim Defined] > Any statement to the effect that a contestant will win a specific number of tricks is > a claim of those tricks. A contestant also claims when he suggests that play be > curtailed, or when he shows his cards (unless he demonstrably did not intend to > claim - for example, if declarer faces his cards after an opening lead out of turn > Law 54, not this Law, will apply)." > > [Jerry] > There are three ways sufficient to make it a claim: A statement of winning a > specific number, a statement suggesting play be curtailed, showing his cards---but > there are exceptions to the latter. We have none of these here. He did not > suggest play be curtailed, he did not imply a specific number, he did not show all > of his cards. He stated he would take 8 or more; he did not state he would take > specifically 8. > > So a mathematician naively reading the law would conclude that it not a claim. > However, what happens next if the defenders think it is a claim and face their > hands but declarer wants to play on? On the contrary I shall without any doubt rule that Herman claimed two tricks and said nothing about the remaining tricks. Consequently I would give him those two tricks for a total of eight, and give to the opponents all remaining tricks except such tricks that Herman could not lose with any normal line of play. From agot at ulb.ac.be Mon Jan 24 14:11:18 2011 From: agot at ulb.ac.be (Alain Gottcheiner) Date: Mon, 24 Jan 2011 14:11:18 +0100 Subject: [BLML] Unsure In-Reply-To: <000901cbba82$f84fdf50$e8ef9df0$@no> References: <000901cbba0f$ce55b3f0$6b011bd0$@no><586818.75337.qm@web28510.mail.ukl.yahoo.com> <4D3704A8.8090006@skynet.be> <128171.88697.qm@web25402.mail.ukl. yahoo.com> < 4D382CB9.2060700@ulb.ac.be><6410CABD-9AF3-45A7-9B10-F5EDC78C6095@starpowe r. net><4D3852CE.6080908@ulb.ac.be> <4D386625.90105@meteo.fr> <4D3869DD.9030708@ulb.ac.be> <4D3873A6.5070104@meteo.fr> <107026.33279.qm@web28515.mail.ukl.yahoo.com> <4D39BC9E.9040108@nhcc.net> <1820574630.137446.1295675648497.JavaMail.ngmail@webmail11.arcor-online.net><4D3A93A3.8020906@skynet.be><1896834273.57934.1295686896979.JavaMail.ngmail@webmail13.arcor-online.net> <514721598.171618.1295725647449.JavaMail.ngmail@webmail10.arcor-online.net> <000901cbba82$f84fdf50$e8ef9df0$@no> Message-ID: <4D3D7A76.5050303@ulb.ac.be> Le 22/01/2011 23:23, Sven Pran a ?crit : > On Behalf Of David Sent: 22. januar 2011 22:58 >>> Sven Pran wrote: > ............. >>>> I had a similar talk with the Norwegian LC some time ago; our line >>>> is that if the correct agreement cannot be established then we rule >>>> that the call has been made according to agreements. (Ref Law 75) >>>> >>>> This means that a player who is unsure but guesses the correct >>>> agreement has not given MI while a player who does not provide the >>>> correct agreement >>>> (if >>>> necessary confirmed by other evidence) is always deemed to having >>>> given MI. >>>> This includes the "not discussed" and "I have no idea" statements! >>> Looks sensible to me. >>> >>> >>> >>> Thomas >>> >>> Well it looks crazy to me. >>> >>> This is effectively saying that every partnership has agreements about >>> every sequence and hence "no agreement" can never be the appropriate >>> answer even when it is absolutely correct. >>> >>> I am glad I do not have to try and enforce anything like that in my >>> local club. I would be down to 2 tables in no time flat. >>> >>> Now I would accept that "no agreement" is rarely a full disclosure as >>> there are likely to be other sequences for which there are agreements >>> which would influence the likely meanings of this particular sequence >>> and these inferences should be disclosed. But at the end of the day, >>> if the partnership does not have an agreement it is not up to a >>> National Authority to deem they have an agreement and penalise them >>> for failing to disclose it. >> *It is up to the RA to decide to which extent a pair must have agreements. >> >> Law reference for this assertion please. >> An RA may specify a form of (pre) disclosure (a convention card) and hence >> specifies situations where partnerships must have an agreement. But > outside of >> this I am not aware of any justification for an obligation for > partnerships to have >> agreements. >> I suppose an RA could make it a condition of entry to its competitions but > we are >> discussing the laws as applying generally. >> >> *Of course a pickup partnership will have only a skeleton of agreements, > *and the >> same holds for beginners. >> >> 99.99% of partnerships will come across an undiscussed situation at some > point. >> *It then is within the RA's rights to force those partnerships to play > some standard >> system. >> >> And what will that achieve? The probability of coming across such a > situation will >> be thousands of times higher when both members of a partnership are > playing a >> system with which they are not familiar. > I have a very strong feeling that what many of you forget is this: > > Misinformation is no automatic cause for rectification; rectification shall > only be applied if the non-offending side can show probable damage caused by > the misinformation. AG : I'd like to bring a small change to this : "... applied if one can show ...". There could be damage that a non-expert NOS is unable to assess quickly. The TD / AC should check by themselves, if needed. From nigelguthrie at yahoo.co.uk Mon Jan 24 14:15:45 2011 From: nigelguthrie at yahoo.co.uk (Nigel Guthrie) Date: Mon, 24 Jan 2011 13:15:45 +0000 (GMT) Subject: [BLML] Unsure In-Reply-To: <4D3D7134.4000803@ulb.ac.be> References: <586818.75337.qm@web28510.mail.ukl.yahoo.com> <4D3704A8.8090006@skynet.be> <128171.88697.qm@web25402.mail.ukl. yahoo.com> < 4D382CB9.2060700@ulb.ac.be><6410CABD-9AF3-45A7-9B10-F5EDC78C6095@starpowe r. net> <4D3852CE.6080908@ulb.ac.be> <4D386625.90105@meteo.fr> <4D3869DD.9030708@ulb.ac.be> <4D3873A6.5070104@meteo.fr> <107026.33279.qm@web28515.mail.ukl.yahoo.com> <4D39BC9E.9040108@nhcc.net> <1820574630.137446.1295675648497.JavaMail.ngmail@webmail11.arcor-online.net> <4D3D7134.4000803@ulb.ac.be> Message-ID: <431431.44115.qm@web28515.mail.ukl.yahoo.com> {Alain] AG : indeed. "conflicting agreements" might be the right explanation ; "unsure" will never be. You are unsure, not the system, and TFLB says you must speak about the system, not about your state of mind. [Nigel] Unfortunately, Allain's interpretation contrasts with Richard's. I sympathise more with Alain's interpretation. I once claimed that if you agree to play, say "Jannerston Precision", have a summary of it on your system card, and declare that to your opponents then that is your systemic agreement. Even if you've forgotten it or never learnt it. In common sense or ordinary law, such an interpretation seems obvious. Richard Hills contradicted me. For Bridge law purposes, Richard explained that your agreement is what is in your head -- not what is written on your system-card or notes. That is: what you've forgotten or never learnt properly is not your legal systemic agreement. Current Bridge law does resolve which of these incompatible interpretations is correct. Perhaps this issue will be addressed in 2018 0r 2028? From agot at ulb.ac.be Mon Jan 24 14:26:21 2011 From: agot at ulb.ac.be (Alain Gottcheiner) Date: Mon, 24 Jan 2011 14:26:21 +0100 Subject: [BLML] Exam question In-Reply-To: <4D3D3B2F.5090107@skynet.be> References: <4D3D3B2F.5090107@skynet.be> Message-ID: <4D3D7DFD.2020801@ulb.ac.be> Le 24/01/2011 9:41, Herman De Wael a ?crit : > This one may well make it into an exam question for a TD course, if you > ever need one: > > On Saturday, I was playing 2He. I had just won the first six tricks, and > I showed AK of hearts, saying "I think I am going to make this > contract". Did I claim or am I allowed to play on? I would vote for claim : "suggest in any way that the play be shortened". From gordonrainsford at btinternet.com Mon Jan 24 14:28:49 2011 From: gordonrainsford at btinternet.com (Gordon Rainsford) Date: Mon, 24 Jan 2011 13:28:49 +0000 Subject: [BLML] Exam question In-Reply-To: <4D3D7DFD.2020801@ulb.ac.be> References: <4D3D3B2F.5090107@skynet.be> <4D3D7DFD.2020801@ulb.ac.be> Message-ID: <928BA273-A923-4DDA-A9A5-F7F824D1B12C@btinternet.com> On 24 Jan 2011, at 13:26, Alain Gottcheiner wrote: > Le 24/01/2011 9:41, Herman De Wael a ?crit : >> This one may well make it into an exam question for a TD course, >> if you >> ever need one: >> >> On Saturday, I was playing 2He. I had just won the first six >> tricks, and >> I showed AK of hearts, saying "I think I am going to make this >> contract". Did I claim or am I allowed to play on? > I would vote for claim : "suggest in any way that the play be > shortened". Did he? From agot at ulb.ac.be Mon Jan 24 14:32:01 2011 From: agot at ulb.ac.be (Alain Gottcheiner) Date: Mon, 24 Jan 2011 14:32:01 +0100 Subject: [BLML] Unsure In-Reply-To: <431431.44115.qm@web28515.mail.ukl.yahoo.com> References: <586818.75337.qm@web28510.mail.ukl.yahoo.com> <4D3704A8.8090006@skynet.be> <128171.88697.qm@web25402.mail.ukl. yahoo.com> < 4D382CB9.2060700@ulb.ac.be><6410CABD-9AF3-45A7-9B10-F5EDC78C6095@starpowe r. net> <4D3852CE.6080908@ulb.ac.be> <4D386625.90105@meteo.fr> <4D3869DD.9030708@ulb.ac.be> <4D3873A6.5070104@meteo.fr> <107026.33279.qm@web28515.mail.ukl.yahoo.com> <4D39BC9E.9040108@nhcc.net> <1820574630.137446.1295675648497.JavaMail.ngmail@webmail11.arcor-online.net> <4D3D7134.4000803@ulb.ac.be> <431431.44115.qm@web28515.mail.ukl.yahoo.com> Message-ID: <4D3D7F51.70805@ulb.ac.be> Le 24/01/2011 14:15, Nigel Guthrie a ?crit : > {Alain] > > AG : indeed. "conflicting agreements" might be the right explanation ; > "unsure" will never be. You are unsure, not the system, and TFLB says > you must speak about the system, not about your state of mind. > > [Nigel] > Unfortunately, Allain's interpretation contrasts with Richard's. > > I sympathise more with Alain's interpretation. I once claimed that if you agree > to play, say "Jannerston Precision", have a summary of it on your system card, > and declare that to your opponents then that is your systemic agreement. Even if > you've forgotten it or never learnt it. In common sense or ordinary law, such an > interpretation seems obvious. > > Richard Hills contradicted me. For Bridge law purposes, Richard explained that > your agreement is what is in your head -- not what is written on your > system-card or notes. That is: what you've forgotten or never learnt properly is > not your legal systemic agreement. AG : doesn't hold. How could your agreement be precise on mondays and inexistent on tuesdays, because you happen to work hard on tuesdays ? From nigelguthrie at yahoo.co.uk Mon Jan 24 14:31:42 2011 From: nigelguthrie at yahoo.co.uk (Nigel Guthrie) Date: Mon, 24 Jan 2011 13:31:42 +0000 (GMT) Subject: [BLML] Exam question In-Reply-To: <4D3D7DFD.2020801@ulb.ac.be> References: <4D3D3B2F.5090107@skynet.be> <4D3D7DFD.2020801@ulb.ac.be> Message-ID: <125118.22847.qm@web28516.mail.ukl.yahoo.com> [Alain Gottcheiner] I would vote for claim : "suggest in any way that the play be shortened". {Nigel] I agree. For the same reason. However, if Herman protests he was interrupted in mid-claim, perhaps the director should let him expand further (metaphorically, anyway). From nigelguthrie at yahoo.co.uk Mon Jan 24 14:45:09 2011 From: nigelguthrie at yahoo.co.uk (Nigel Guthrie) Date: Mon, 24 Jan 2011 13:45:09 +0000 (GMT) Subject: [BLML] Unsure In-Reply-To: <4D3D7F51.70805@ulb.ac.be> References: <586818.75337.qm@web28510.mail.ukl.yahoo.com> <4D3704A8.8090006@skynet.be> <128171.88697.qm@web25402.mail.ukl. yahoo.com> < 4D382CB9.2060700@ulb.ac.be><6410CABD-9AF3-45A7-9B10-F5EDC78C6095@starpowe r. net> <4D3852CE.6080908@ulb.ac.be> <4D386625.90105@meteo.fr> <4D3869DD.9030708@ulb.ac.be> <4D3873A6.5070104@meteo.fr> <107026.33279.qm@web28515.mail.ukl.yahoo.com> <4D39BC9E.9040108@nhcc.net> <1820574630.137446.1295675648497.JavaMail.ngmail@webmail11.arcor-online.net> <4D3D7134.4000803@ulb.ac.be> <431431.44115.qm@web28515.mail.ukl.yahoo.com> <4D3D7F51.70805@ulb.ac.be> Message-ID: <395644.68863.qm@web28514.mail.ukl.yahoo.com> [Alain] doesn't hold. How could your agreement be precise on mondays and nonexistent on tuesdays, because you happen to work hard on tuesdays ? {Nigel] Pass. Ask me another. I was merely trying to represent Richard's interpretation. Alain and others maintain that saying "Unsure" is an MI infraction. But can anybody tell us what the law prescribes you *should* say if "Unsure" is the truth? From agot at ulb.ac.be Mon Jan 24 14:56:13 2011 From: agot at ulb.ac.be (Alain Gottcheiner) Date: Mon, 24 Jan 2011 14:56:13 +0100 Subject: [BLML] Exam question In-Reply-To: <928BA273-A923-4DDA-A9A5-F7F824D1B12C@btinternet.com> References: <4D3D3B2F.5090107@skynet.be> <4D3D7DFD.2020801@ulb.ac.be> <928BA273-A923-4DDA-A9A5-F7F824D1B12C@btinternet.com> Message-ID: <4D3D84FD.1020308@ulb.ac.be> Le 24/01/2011 14:28, Gordon Rainsford a ?crit : > On 24 Jan 2011, at 13:26, Alain Gottcheiner wrote: > >> Le 24/01/2011 9:41, Herman De Wael a ?crit : >>> This one may well make it into an exam question for a TD course, >>> if you >>> ever need one: >>> >>> On Saturday, I was playing 2He. I had just won the first six >>> tricks, and >>> I showed AK of hearts, saying "I think I am going to make this >>> contract". Did I claim or am I allowed to play on? >> I would vote for claim : "suggest in any way that the play be >> shortened". > Did he? > He said that those two cards would take tricks, before they could effectively take them (and before any risk of a revoke, which is also a good reason for claiming). Notice that it is usual -if not kosher- to show the cards which will be taking tricks, saying e.g. "making four more tricks" and fold the others, and this is usually considered a valid claim. And above that, saying "I won't go down" is a declaration that one will make n more tricks; and such a declaration is considered as a claim with concession of other remaining tricks. Best regards Alain From svenpran at online.no Mon Jan 24 14:57:22 2011 From: svenpran at online.no (Sven Pran) Date: Mon, 24 Jan 2011 14:57:22 +0100 Subject: [BLML] Unsure In-Reply-To: <4D3D72AF.3060606@ulb.ac.be> References: <586818.75337.qm@web28510.mail.ukl.yahoo.com> <4D3704A8.8090006@skynet.be> <128171.88697.qm@web25402.mail.ukl. yahoo.com> < 4D382CB9.2060700@ulb.ac.be><6410CABD-9AF3-45A7-9B10-F5EDC78C6095@starpowe r. net> <4D3852CE.6080908@ulb.ac.be> <4D386625.90105@meteo.fr> <4D3869DD.9030708@ulb.ac.be> <4D3873A6.5070104@meteo.fr> <107026.33279.qm@web28515.mail.ukl.yahoo.com> <4D39BC9E.9040108@nhcc.net> <1820574630.137446.1295675648497.JavaMail.ngmail@webmail11.arcor-online.net> <4D3A93A3.8020906@skynet.be> <000901cbba0f$ce55b3f0$6b011bd0$@no> <4D3D72AF.3060606@ulb.ac.be> Message-ID: <000001cbbbce$a0c010f0$e24032d0$@no> > -----Original Message----- > From: Alain Gottcheiner [mailto:agot at ulb.ac.be] > Sven Pran a ?crit : .............. > > This means that a player who does not provide the > > correct agreement (if necessary confirmed by other > > evidence) is always deemed to having given MI. > > This includes the "not discussed" and "I have no idea" statements! > AG : there must be limits to this. How long your list of agremeents, there must be > some non-covered area. Any player is entitled to a full explanation of opponents' auction whether the relevant partnership understanding is explicit or implicit. "Not discussed", "I have no idea" and similar statements are not such explanations, they are simply statements to the effect that the player does not know what the correct explanation is. According to law this is synonymous with misexplanation, i.e. failure to give correct information. From agot at ulb.ac.be Mon Jan 24 15:00:39 2011 From: agot at ulb.ac.be (Alain Gottcheiner) Date: Mon, 24 Jan 2011 15:00:39 +0100 Subject: [BLML] Unsure In-Reply-To: <395644.68863.qm@web28514.mail.ukl.yahoo.com> References: <586818.75337.qm@web28510.mail.ukl.yahoo.com> <4D3704A8.8090006@skynet.be> <128171.88697.qm@web25402.mail.ukl. yahoo.com> < 4D382CB9.2060700@ulb.ac.be><6410CABD-9AF3-45A7-9B10-F5EDC78C6095@starpowe r. net> <4D3852CE.6080908@ulb.ac.be> <4D386625.90105@meteo.fr> <4D3869DD.9030708@ulb.ac.be> <4D3873A6.5070104@meteo.fr> <107026.33279.qm@web28515.mail.ukl.yahoo.com> <4D39BC9E.9040108@nhcc.net> <1820574630.137446.1295675648497.JavaMail.ngmail@webmail11.arcor-online.net> <4D3D7134.4000803@ulb.ac.be> <431431.44115.qm@web28515.mail.ukl.yahoo.com> <4D3D7F51.70805@ulb.ac.be> <395644.68863.qm@web28514.mail.ukl.yahoo.com> Message-ID: <4D3D8607.2000800@ulb.ac.be> Le 24/01/2011 14:45, Nigel Guthrie a ?crit : > [Alain] > doesn't hold. How could your agreement be precise on mondays and > > nonexistent on tuesdays, because you happen to work hard on tuesdays ? > > {Nigel] > Pass. Ask me another. I was merely trying to represent Richard's interpretation. > > Alain and others maintain that saying "Unsure" is an MI infraction. AG : not really. It is, in fact, no statement about the system, and you have to give one. > But can > anybody tell us what the law prescribes you *should* say if "Unsure" is the > truth? AG : you should give the correct meaning in the system. If, for whatever reason, you don't mention a relevant agreement (e.g. because you are unable to remember), you infracted L20. Not really by giving MI, but not giving I is also an infraction. From david.j.barton at lineone.net Mon Jan 24 15:00:23 2011 From: david.j.barton at lineone.net (David) Date: Mon, 24 Jan 2011 14:00:23 -0000 Subject: [BLML] Unsure In-Reply-To: <395644.68863.qm@web28514.mail.ukl.yahoo.com> References: <586818.75337.qm@web28510.mail.ukl.yahoo.com><4D3704A8.8090006@skynet.be><128171.88697.qm@web25402.mail.ukl. yahoo.com> <4D382CB9.2060700@ulb.ac.be><6410CABD-9AF3-45A7-9B10-F5EDC78C6095@starpower. net> <4D3852CE.6080908@ulb.ac.be> <4D386625.90105@meteo.fr><4D3869DD.9030708@ulb.ac.be> <4D3873A6.5070104@meteo.fr><107026.33279.qm@web28515.mail.ukl.yahoo.com><4D39BC9E.9040108@nhcc.net><1820574630.137446.1295675648497.JavaMail.ngmail@webmail11.arcor-online.net><4D3D7134.4000803@ulb.ac.be><431431.44115.qm@web28515.mail.ukl.yahoo.com><4D3D7F51.70805@ulb.ac.be> <395644.68863.qm@web28514.mail.ukl.yahoo.com> Message-ID: <43FB511CB5684DA490AAB2D8E8666177@Lounge> -----Original Message----- From: Nigel Guthrie Sent: Monday, January 24, 2011 1:45 PM To: Bridge Laws Mailing List Subject: Re: [BLML] Unsure [Alain] doesn't hold. How could your agreement be precise on mondays and nonexistent on tuesdays, because you happen to work hard on tuesdays ? {Nigel] Pass. Ask me another. I was merely trying to represent Richard's interpretation. Alain and others maintain that saying "Unsure" is an MI infraction. But can anybody tell us what the law prescribes you *should* say if "Unsure" is the truth? {David} In my opinion this depends on what "unsure" means. If you have an agreement but you are unable to tell the opposition what it is then you will be guilty of MI. If you have NO agreement but are sure it means A or B but cannot tell them which then there is NO MI. ********************************** david.j.barton at lineone.net ********************************** From svenpran at online.no Mon Jan 24 15:02:17 2011 From: svenpran at online.no (Sven Pran) Date: Mon, 24 Jan 2011 15:02:17 +0100 Subject: [BLML] Unsure In-Reply-To: <395644.68863.qm@web28514.mail.ukl.yahoo.com> References: <586818.75337.qm@web28510.mail.ukl.yahoo.com> <4D3704A8.8090006@skynet.be> <128171.88697.qm@web25402.mail.ukl. yahoo.com> < 4D382CB9.2060700@ulb.ac.be><6410CABD-9AF3-45A7-9B10-F5EDC78C6095@starpowe r. net> <4D3852CE.6080908@ulb.ac.be> <4D386625.90105@meteo.fr> <4D3869DD.9030708@ulb.ac.be> <4D3873A6.5070104@meteo.fr> <107026.33279.qm@web28515.mail.ukl.yahoo.com> <4D39BC9E.9040108@nhcc.net> <1820574630.137446.1295675648497.JavaMail.ngmail@webmail11.arcor-online.net> <4D3D7134.4000803@ulb.ac.be> <431431.44115.qm@web28515.mail.ukl.yahoo.com> <4D3D7F51.70805@ulb.ac.be> <395644.68863.qm@web28514.mail.ukl.yahoo.com> Message-ID: <000101cbbbcf$4f8dd270$eea97750$@no> On Behalf Of Nigel > Guthrie > Sent: 24. januar 2011 14:45 > To: Bridge Laws Mailing List > Subject: Re: [BLML] Unsure > > [Alain] > doesn't hold. How could your agreement be precise on mondays and > > nonexistent on tuesdays, because you happen to work hard on tuesdays ? > > {Nigel] > Pass. Ask me another. I was merely trying to represent Richard's interpretation. > > Alain and others maintain that saying "Unsure" is an MI infraction. But can > anybody tell us what the law prescribes you *should* say if "Unsure" is the truth? If "unsure" is the truth (and your answer) then you fail to give opponents the explanation they are entitled to. According to laws this is misexplanation. "Unsure" is only an explanation of why you cannot comply with the laws; it is not an explanation of the auction as required by the laws. From sater at xs4all.nl Mon Jan 24 15:07:07 2011 From: sater at xs4all.nl (Hans van Staveren) Date: Mon, 24 Jan 2011 15:07:07 +0100 Subject: [BLML] Unsure In-Reply-To: <000101cbbbcf$4f8dd270$eea97750$@no> References: <586818.75337.qm@web28510.mail.ukl.yahoo.com> <4D3704A8.8090006@skynet.be> <128171.88697.qm@web25402.mail.ukl. yahoo.com> < 4D382CB9.2060700@ulb.ac.be><6410CABD-9AF3-45A7-9B10-F5EDC78C6095@starpowe r. net> <4D3852CE.6080908@ulb.ac.be> <4D386625.90105@meteo.fr> <4D3869DD.9030708@ulb.ac.be> <4D3873A6.5070104@meteo.fr> <107026.33279.qm@web28515.mail.ukl.yahoo.com> <4D39BC9E.9040108@nhcc.net> <1820574630.137446.1295675648497.JavaMail.ngmail@webmail11.arcor-online.net> <4D3D7134.4000803@ulb.ac.be> <431431.44115.qm@web28515.mail.ukl.yahoo.com> <4D3D7F51.70805@ulb.ac.be> <395644.68863.qm@web28514.mail.ukl.yahoo.com> <000101cbbbcf$4f8dd270$eea97750$@no> Message-ID: <011301cbbbcf$fc8610a0$f59231e0$@nl> Any halfway aspiring TD to-be in the Netherlands knows that after an unsure answer you call the TD, the TD sends the unsure-sayer out of earshot, let's his partner explain and calls the player back. This is no problem whatsoever. So why is this discussion so long-winded? Hans -----Original Message----- From: blml-bounces at rtflb.org [mailto:blml-bounces at rtflb.org] On Behalf Of Sven Pran Sent: maandag 24 januari 2011 15:02 To: 'Bridge Laws Mailing List' Subject: Re: [BLML] Unsure On Behalf Of Nigel > Guthrie > Sent: 24. januar 2011 14:45 > To: Bridge Laws Mailing List > Subject: Re: [BLML] Unsure > > [Alain] > doesn't hold. How could your agreement be precise on mondays and > > nonexistent on tuesdays, because you happen to work hard on tuesdays ? > > {Nigel] > Pass. Ask me another. I was merely trying to represent Richard's interpretation. > > Alain and others maintain that saying "Unsure" is an MI infraction. But can > anybody tell us what the law prescribes you *should* say if "Unsure" is the truth? If "unsure" is the truth (and your answer) then you fail to give opponents the explanation they are entitled to. According to laws this is misexplanation. "Unsure" is only an explanation of why you cannot comply with the laws; it is not an explanation of the auction as required by the laws. _______________________________________________ Blml mailing list Blml at rtflb.org http://lists.rtflb.org/mailman/listinfo/blml From svenpran at online.no Mon Jan 24 15:11:12 2011 From: svenpran at online.no (Sven Pran) Date: Mon, 24 Jan 2011 15:11:12 +0100 Subject: [BLML] Unsure In-Reply-To: <43FB511CB5684DA490AAB2D8E8666177@Lounge> References: <586818.75337.qm@web28510.mail.ukl.yahoo.com><4D3704A8.8090006@skynet.be><128171.88697.qm@web25402.mail.ukl. yahoo.com> <4D382CB9.2060700@ulb.ac.be><6410CABD-9AF3-45A7-9B10-F5EDC78C6095@starpower. net> <4D3852CE.6080908@ulb.ac.be> <4D386625.90105@meteo.fr><4D3869DD.9030708@ulb.ac.be> <4D3873A6.5070104@meteo.fr><107026.33279.qm@web28515.mail.ukl.yahoo.com><4D39BC9E.9040108@nhcc.net><1820574630.137446.1295675648497.JavaMail.ngmail@webmail11.arcor-online.net><4D3D7134.4000803@ulb.ac.be><431431.44115.qm@web28515.mail.ukl.yahoo.com><4D3D7F51.70805@ulb.ac.be> <395644.68863.qm@web28514.mail.ukl.yahoo.com> <43FB511CB5684DA490AAB2D8E8666177@Lounge> Message-ID: <000201cbbbd0$8f4693b0$add3bb10$@no> On Behalf Of David ............. > If you have NO agreement but are sure it means A or B but cannot tell them which > then there is NO MI. Making a call completely at random (with the agreement that it has no meaning) is probably illegal anywhere, the laws assume that a call always have a meaning according to explicit or implicit partnership understanding. If no evidence to the contrary can be provided then the meaning of the call shall be deemed to correspond with the card holding of the player making that call. Any explanation not conforming to this meaning is to be ruled incorrect. Your example: "It is either A or B, I don't know which" is therefore still a failure to give correct explanation. From blml at arcor.de Mon Jan 24 15:12:47 2011 From: blml at arcor.de (Thomas Dehn) Date: Mon, 24 Jan 2011 15:12:47 +0100 (CET) Subject: [BLML] Unsure In-Reply-To: <011301cbbbcf$fc8610a0$f59231e0$@nl> References: <011301cbbbcf$fc8610a0$f59231e0$@nl> <586818.75337.qm@web28510.mail.ukl.yahoo.com> <4D3704A8.8090006@skynet.be> <128171.88697.qm@web25402.mail.ukl. yahoo.com> < 4D382CB9.2060700@ulb.ac.be><6410CABD-9AF3-45A7-9B10-F5EDC78C6095@starpowe r. net> <4D3852CE.6080908@ulb.ac.be> <4D386625.90105@meteo.fr> <4D3869DD.9030708@ulb.ac.be> <4D3873A6.5070104@meteo.fr> <107026.33279.qm@web28515.mail.ukl.yahoo.com> <4D39BC9E.9040108@nhcc.net> <1820574630.137446.1295675648497.JavaMail.ngmail@webmail11.arcor-online.net> <4D3D7134.4000803@ulb.ac.be> <431431.44115.qm@web28515.mail.ukl.yahoo.com> <4D3D7F51.70805@ulb.ac.be> <395644.68863.qm@web28514.mail.ukl.yahoo.com> <000101cbbbcf$4f8dd270$eea97750$@no> Message-ID: <1033690204.15554.1295878367268.JavaMail.ngmail@webmail16.arcor-online.net> Hans van Staveren > Any halfway aspiring TD to-be in the Netherlands knows that after an unsure > answer you call the TD, the TD sends the unsure-sayer out of earshot, let's > his partner explain and calls the player back. > > This is no problem whatsoever. So why is this discussion so long-winded? The procedure you outline, and which Richard mentioned earlier in this thread, is not part of TFLB, but merely a procedure established by various RAs. The discussion in this thread is about how to handle the situation with just TFLB, and no extra procedures established by an RA. Thomas From svenpran at online.no Mon Jan 24 15:15:20 2011 From: svenpran at online.no (Sven Pran) Date: Mon, 24 Jan 2011 15:15:20 +0100 Subject: [BLML] Unsure In-Reply-To: <011301cbbbcf$fc8610a0$f59231e0$@nl> References: <586818.75337.qm@web28510.mail.ukl.yahoo.com> <4D3704A8.8090006@skynet.be> <128171.88697.qm@web25402.mail.ukl. yahoo.com> < 4D382CB9.2060700@ulb.ac.be><6410CABD-9AF3-45A7-9B10-F5EDC78C6095@starpowe r. net> <4D3852CE.6080908@ulb.ac.be> <4D386625.90105@meteo.fr> <4D3869DD.9030708@ulb.ac.be> <4D3873A6.5070104@meteo.fr> <107026.33279.qm@web28515.mail.ukl.yahoo.com> <4D39BC9E.9040108@nhcc.net> <1820574630.137446.1295675648497.JavaMail.ngmail@webmail11.arcor-online.net> <4D3D7134.4000803@ulb.ac.be> <431431.44115.qm@web28515.mail.ukl.yahoo.com> <4D3D7F51.70805@ulb.ac.be> <395644.68863.qm@web28514.mail.ukl.yahoo.com> <000101cbbbcf$4f8dd270$eea97750$@no> <011301cbbbcf$fc8610a0$f59231e0$@nl> Message-ID: <000301cbbbd1$22a7eaa0$67f7bfe0$@no> On Behalf Of Hans van Staveren > Sent: 24. januar 2011 15:07 > To: 'Bridge Laws Mailing List' > Subject: Re: [BLML] Unsure > > Any halfway aspiring TD to-be in the Netherlands knows that after an unsure > answer you call the TD, the TD sends the unsure-sayer out of earshot, let's his > partner explain and calls the player back. > > This is no problem whatsoever. So why is this discussion so long-winded? > > Hans Because there seems to be a "school" developing that a partnership can always get around the duty to explain their understandings by simply claiming "undiscussed" or "no agreement" when they are asked to explain some part of their auction. From blml at arcor.de Mon Jan 24 15:19:59 2011 From: blml at arcor.de (Thomas Dehn) Date: Mon, 24 Jan 2011 15:19:59 +0100 (CET) Subject: [BLML] Unsure In-Reply-To: <000301cbbbd1$22a7eaa0$67f7bfe0$@no> References: <000301cbbbd1$22a7eaa0$67f7bfe0$@no> <586818.75337.qm@web28510.mail.ukl.yahoo.com> <4D3704A8.8090006@skynet.be> <128171.88697.qm@web25402.mail.ukl. yahoo.com> < 4D382CB9.2060700@ulb.ac.be><6410CABD-9AF3-45A7-9B10-F5EDC78C6095@starpowe r. net> <4D3852CE.6080908@ulb.ac.be> <4D386625.90105@meteo.fr> <4D3869DD.9030708@ulb.ac.be> <4D3873A6.5070104@meteo.fr> <107026.33279.qm@web28515.mail.ukl.yahoo.com> <4D39BC9E.9040108@nhcc.net> <1820574630.137446.1295675648497.JavaMail.ngmail@webmail11.arcor-online.net> <4D3D7134.4000803@ulb.ac.be> <431431.44115.qm@web28515.mail.ukl.yahoo.com> <4D3D7F51.70805@ulb.ac.be> <395644.68863.qm@web28514.mail.ukl.yahoo.com> <000101cbbbcf$4f8dd270$eea97750$@no> <011301cbbbcf$fc8610a0$f59231e0$@nl> Message-ID: <2025614698.15834.1295878799813.JavaMail.ngmail@webmail16.arcor-online.net> Sven Pran > On Behalf Of Hans van Staveren > > Sent: 24. januar 2011 15:07 > > To: 'Bridge Laws Mailing List' > > Subject: Re: [BLML] Unsure > > > > Any halfway aspiring TD to-be in the Netherlands knows that after an unsure > > answer you call the TD, the TD sends the unsure-sayer out of earshot, let's his > > partner explain and calls the player back. > > > > This is no problem whatsoever. So why is this discussion so long-winded? > > > > Hans > > Because there seems to be a "school" developing that a partnership can > always get around the duty to explain their understandings by simply > claiming "undiscussed" or "no agreement" when they are asked to explain > some > part of their auction. I *love* it when the little old ladies who have played together for 20 years claim that they have no agreement about their leads. Of course LHLL was leading 4th best. Thomas From david.j.barton at lineone.net Mon Jan 24 15:21:38 2011 From: david.j.barton at lineone.net (David) Date: Mon, 24 Jan 2011 14:21:38 -0000 Subject: [BLML] Unsure In-Reply-To: <011301cbbbcf$fc8610a0$f59231e0$@nl> References: <586818.75337.qm@web28510.mail.ukl.yahoo.com> <4D3704A8.8090006@skynet.be> <128171.88697.qm@web25402.mail.ukl. yahoo.com> < 4D382CB9.2060700@ulb.ac.be><6410CABD-9AF3-45A7-9B10-F5EDC78C6095@starpowe r. net><4D3852CE.6080908@ulb.ac.be> <4D386625.90105@meteo.fr> <4D3869DD.9030708@ulb.ac.be> <4D3873A6.5070104@meteo.fr> <107026.33279.qm@web28515.mail.ukl.yahoo.com> <4D39BC9E.9040108@nhcc.net> <1820574630.137446.1295675648497.JavaMail.ngmail@webmail11.arcor-online.net> <4D3D7134.4000803@ulb.ac.be> <431431.44115.qm@web28515.mail.ukl.yahoo.com> <4D3D7F51.70805@ulb.ac.be> <395644.68863.qm@web28514.mail.ukl.yahoo.com><000101cbbbcf$4f8dd270$eea97750$@no> <011301cbbbcf$fc8610a0$f59231e0$@nl> Message-ID: <4A3418FD24684B579A369D685BF6D6FB@Lounge> -----Original Message----- From: Hans van Staveren Sent: Monday, January 24, 2011 2:07 PM To: 'Bridge Laws Mailing List' Subject: Re: [BLML] Unsure Any halfway aspiring TD to-be in the Netherlands knows that after an unsure answer you call the TD, the TD sends the unsure-sayer out of earshot, let's his partner explain and calls the player back. This is no problem whatsoever. So why is this discussion so long-winded? Hans Because you would get exactly the same answer from the person who made the call. We have no systemic agreement about this call. It could be A or B. David ********************************** david.j.barton at lineone.net ********************************** From jfusselman at gmail.com Mon Jan 24 15:23:28 2011 From: jfusselman at gmail.com (Jerry Fusselman) Date: Mon, 24 Jan 2011 08:23:28 -0600 Subject: [BLML] Naive question about double shots In-Reply-To: References: Message-ID: [David Grabiner] This is an example of a double shot; North bid 3S on a hand which had no good bridge reason to do so, possibly with the expectation that he would be able to keep his +730 if it was doubled and made, but would get the result rolled back to 2S if it went down. [Steve Willner] There's a long history of thinking this is a bad thing, but I think Jerry's question was _why_ it's bad. [Jerry] Right. That's my main question. I also want to know what constitutes a double shot. By the way, I think Marv would say OS should get +130, and Adam Wildavsky would say OS should get +200. And Eric would say NOS should get the negative of OS's score on the grounds that 3S is nowhere near egregious. [naive player] In practice, sometimes 3S works out better than passing. To decide what is a double shot, do directors check to see what works on the specific deal and what doesn't and call everything that does not work a double shot? Would pass (instead of 3S) also be a double shot? After all, it seems one could paraphrase David's answer as follows: Pass is an example of a double shot; North passed on a hand that should have competed better. Possibly North passed with the expectation that he could maybe set their contract two tricks for +200 if everyone was balanced, but would get the result rolled back to 2S if that was better. If everything is a double shot, then nothing is. So what makes 3S a double shot and keeps pass from being a double shot? "Possibly with the expectation" does not sound that convincing as a basis of ruling NOS get nothing. Besides, 3S was not a free double shot. There was a chance that the director would rule no UI or no LA to the first double. A very good chance. You want 3S to have risk. OK, it did. I can see why OS should get +200, but what is wrong with giving NOS -200? Jerry Fusselman From agot at ulb.ac.be Mon Jan 24 15:28:33 2011 From: agot at ulb.ac.be (Alain Gottcheiner) Date: Mon, 24 Jan 2011 15:28:33 +0100 Subject: [BLML] Unsure In-Reply-To: <000001cbbbce$a0c010f0$e24032d0$@no> References: <586818.75337.qm@web28510.mail.ukl.yahoo.com> <4D3704A8.8090006@skynet.be> <128171.88697.qm@web25402.mail.ukl. yahoo.com> < 4D382CB9.2060700@ulb.ac.be><6410CABD-9AF3-45A7-9B10-F5EDC78C6095@starpowe r. net> <4D3852CE.6080908@ulb.ac.be> <4D386625.90105@meteo.fr> <4D3869DD.9030708@ulb.ac.be> <4D3873A6.5070104@meteo.fr> <107026.33279.qm@web28515.mail.ukl.yahoo.com> <4D39BC9E.9040108@nhcc.net> <1820574630.137446.1295675648497.JavaMail.ngmail@webmail11.arcor-online.net> <4D3A93A3.8020906@skynet.be> <000901cbba0f$ce55b3f0$6b011bd0$@no> <4D3D72AF.3060606@ulb.ac.be> <000001cbbbce$a0c010f0$e24032d0$@no> Message-ID: <4D3D8C91.5020007@ulb.ac.be> Le 24/01/2011 14:57, Sven Pran a ?crit : >> -----Original Message----- >> From: Alain Gottcheiner [mailto:agot at ulb.ac.be] >> Sven Pran a ?crit : > .............. >>> This means that a player who does not provide the >>> correct agreement (if necessary confirmed by other >>> evidence) is always deemed to having given MI. >>> This includes the "not discussed" and "I have no idea" statements! >> AG : there must be limits to this. How long your list of agremeents, there > must be >> some non-covered area. > Any player is entitled to a full explanation of opponents' auction whether > the relevant partnership understanding is explicit or implicit. > > "Not discussed", "I have no idea" and similar statements are not such > explanations, they are simply statements to the effect that the player does > not know what the correct explanation is. So, you're basically saying that every partnership has at least an implicit understanding of any bid they could legally make. What does 1NT-2D-5S mean in your system ? I would be happy with responding "undiscussed" (or "impossible", which also is non-descriptive) From ehaa at starpower.net Mon Jan 24 15:28:20 2011 From: ehaa at starpower.net (Eric Landau) Date: Mon, 24 Jan 2011 09:28:20 -0500 Subject: [BLML] Unsure In-Reply-To: References: <000901cbba0f$ce55b3f0$6b011bd0$@no><586818.75337.qm@web28510.mail.ukl.yahoo.com> <4D3704A8.8090006@skynet.be> <128171.88697.qm@web25402.mail.ukl. yahoo.com> < 4D382CB9.2060700@ulb.ac.be><6410CABD-9AF3-45A7-9B10-F5EDC78C6095@starpowe r. net><4D3852CE.6080908@ulb.ac.be> <4D386625.90105@meteo.fr> <4D3869DD.9030708@ulb.ac.be> <4D3873A6.5070104@meteo.fr> <107026.33279.qm@web28515.mail.ukl.yahoo.com> <4D39BC9E.9040108@nhcc.net> <1820574630.137446.1295675648497.JavaMail.ngmail@webmail11.arcor-online.net><4D3A93A3.8020906@skynet.be><1896834273.57934.1295686896979.JavaMail.ngmail@webmail13.arcor-online.net> <514721598.171618.1295725647449.JavaMail.ngmail@webmail10.arcor-online.net> Message-ID: <12BD2F99-1916-4AB5-B922-DE33438819A9@starpower.net> On Jan 22, 2011, at 4:58 PM, David wrote: > From: Thomas Dehn > >> Well it looks crazy to me. >> >> This is effectively saying that every partnership has agreements >> about >> every >> sequence and hence "no agreement" can never be the appropriate >> answer even >> when it is >> absolutely correct. >> >> I am glad I do not have to try and enforce anything like that in >> my local >> club. I would be down to 2 tables in no time flat. >> >> Now I would accept that "no agreement" is rarely a full disclosure as >> there >> are likely to be other sequences for which there are agreements which >> would influence the >> likely meanings of this particular sequence and these inferences >> should be >> disclosed. But at the end of the day, if the partnership does not >> have an >> agreement it is not >> up to a National Authority to deem they have an agreement and >> penalise >> them for >> failing to disclose it. > > *It is up to the RA to decide to which extent a pair must have > agreements. > > Law reference for this assertion please. > > An RA may specify a form of (pre) disclosure (a convention card) > and hence > specifies situations > where partnerships must have an agreement. But outside of this I am > not > aware of any > justification for an obligation for partnerships to have agreements. > I suppose an RA could make it a condition of entry to its > competitions but > we are discussing > the laws as applying generally. L40A-B permits an RA to publish its preferred system, then declare that anything not included is a banned "special partnership understanding". Eric Landau 1107 Dale Drive Silver Spring MD 20910 ehaa at starpower.net From agot at ulb.ac.be Mon Jan 24 15:37:45 2011 From: agot at ulb.ac.be (Alain Gottcheiner) Date: Mon, 24 Jan 2011 15:37:45 +0100 Subject: [BLML] Unsure In-Reply-To: <2025614698.15834.1295878799813.JavaMail.ngmail@webmail16.arcor-online.net> References: <000301cbbbd1$22a7eaa0$67f7bfe0$@no> <586818.75337.qm@web28510.mail.ukl.yahoo.com> <4D3704A8.8090006@skynet.be> <128171.88697.qm@web25402.mail.ukl. yahoo.com> < 4D382CB9.2060700@ulb.ac.be><6410CABD-9AF3-45A7-9B10-F5EDC78C6095@starpowe r. net> <4D3852CE.6080908@ulb.ac.be> <4D386625.90105@meteo.fr> <4D3869DD.9030708@ulb.ac.be> <4D3873A6.5070104@meteo.fr> <107026.33279.qm@web28515.mail.ukl.yahoo.com> <4D39BC9E.9040108@nhcc.net> <1820574630.137446.1295675648497.JavaMail.ngmail@webmail11.arcor-online.net> <4D3D7134.4000803@ulb.ac.be> <431431.44115.qm@web28515.mail.ukl.yahoo.com> <4D3D7F51.70805@ulb.ac.be> <395644.68863.qm@web28514.mail.ukl.yahoo.com> <000101cbbbcf$4f8dd270$eea97750$@no> <011301cbbbcf$fc8610a0$f59231e0$@nl> <2025614698.15834.1295878799813.JavaMail.ngmail@webmail16.arcor-online.net> Message-ID: <4D3D8EB9.2090304@ulb.ac.be> Le 24/01/2011 15:19, Thomas Dehn a ?crit : > Sven Pran >> On Behalf Of Hans van Staveren >>> Sent: 24. januar 2011 15:07 >>> To: 'Bridge Laws Mailing List' >>> Subject: Re: [BLML] Unsure >>> >>> Any halfway aspiring TD to-be in the Netherlands knows that after an unsure >>> answer you call the TD, the TD sends the unsure-sayer out of earshot, let's his >>> partner explain and calls the player back. >>> >>> This is no problem whatsoever. So why is this discussion so long-winded? >>> >>> Hans >> Because there seems to be a "school" developing that a partnership can >> always get around the duty to explain their understandings by simply >> claiming "undiscussed" or "no agreement" when they are asked to explain >> some >> part of their auction. > I *love* it when the little old ladies > who have played together for 20 years claim > that they have no agreement about their leads. AG : please notice that our agreement for leading small cards against slams is *all agreements are off*, and indeed we do lead small cards in a pseudo-random fashion. From ehaa at starpower.net Mon Jan 24 15:44:31 2011 From: ehaa at starpower.net (Eric Landau) Date: Mon, 24 Jan 2011 09:44:31 -0500 Subject: [BLML] Unsure In-Reply-To: <000901cbba82$f84fdf50$e8ef9df0$@no> References: <000901cbba0f$ce55b3f0$6b011bd0$@no><586818.75337.qm@web28510.mail.ukl.yahoo.com> <4D3704A8.8090006@skynet.be> <128171.88697.qm@web25402.mail.ukl. yahoo.com> < 4D382CB9.2060700@ulb.ac.be><6410CABD-9AF3-45A7-9B10-F5EDC78C6095@starpowe r. net><4D3852CE.6080908@ulb.ac.be> <4D386625.90105@meteo.fr> <4D3869DD.9030708@ulb.ac.be> <4D3873A6.5070104@meteo.fr> <107026.33279.qm@web28515.mail.ukl.yahoo.com> <4D39BC9E.9040108@nhcc.net> <1820574630.137446.1295675648497.JavaMail.ngmail@webmail11.arcor-online.net><4D3A93A3.8020906@skynet.be><1896834273.57934.1295686896979.JavaMail.ngmail@webmail13.arcor-online.net> <514721598.171618.1295725647449.JavaMail.ngmail@webmail10.arcor-online.net> <000901cbba82$f84fdf50$e8ef9df0$@no> Message-ID: On Jan 22, 2011, at 5:23 PM, Sven Pran wrote: > I have a very strong feeling that what many of you forget is this: > > Misinformation is no automatic cause for rectification; > rectification shall > only be applied if the non-offending side can show probable damage > caused by > the misinformation. > > "No agreement" or "undiscussed" is more likely to cause damage to > opponents > if they in spite of their alleged lack of agreements still most of > the time > work out a reasonable auction. To me that looks like a strong > indication of > a probable concealed partnership understanding. For the last forty-plus years, bridge has been getting consistently more "scientific". Systems and methods have gotten consistently more complex and detailed. Today's top-level partnerships have extensive agreements covering the vast majority of situations, often requiring hundreds of pages of notes to specify. But fifty years ago, even the highest-level partnerships got by with a relative handful of basic agreements covering opening bids, basic responses and rebids, and a few simple "artifical" conventions (e.g. Blackwood, Stayman), along with a general understanding of how they bid (what we would call "meta-agreements"). Beyond that, pretty much everything else was "no agreement" or "undiscussed". Despite their lack of agreements, they still managed most of the time to work out a reasonable auction; this was generally considered to be a strong indication that they knew a little something about how to play bridge. Eric Landau 1107 Dale Drive Silver Spring MD 20910 ehaa at starpower.net From david.j.barton at lineone.net Mon Jan 24 15:45:03 2011 From: david.j.barton at lineone.net (David) Date: Mon, 24 Jan 2011 14:45:03 -0000 Subject: [BLML] Unsure In-Reply-To: <000301cbbbd1$22a7eaa0$67f7bfe0$@no> References: <586818.75337.qm@web28510.mail.ukl.yahoo.com> <4D3704A8.8090006@skynet.be> <128171.88697.qm@web25402.mail.ukl. yahoo.com> < 4D382CB9.2060700@ulb.ac.be><6410CABD-9AF3-45A7-9B10-F5EDC78C6095@starpowe r. net> <4D3852CE.6080908@ulb.ac.be> <4D386625.90105@meteo.fr> <4D3869DD.9030708@ulb.ac.be> <4D3873A6.5070104@meteo.fr> <107026.33279.qm@web28515.mail.ukl.yahoo.com> <4D39BC9E.9040108@nhcc.net> <1820574630.137446.1295675648497.JavaMail.ngmail@webmail11.arcor-online.net> <4D3D7134.4000803@ulb.ac.be> <431431.44115.qm@web28515.mail.ukl.yahoo.com> <4D3D7F51.70805@ulb.ac.be> <395644.68863.qm@web28514.mail.ukl.yahoo.com> <000101cbbbcf$4f8dd270$eea97750$@no><011301cbbbcf$fc8610a0$f59231e0$@nl> <000301cbbbd1$22a7eaa0$67f7bfe0$@no> Message-ID: <52B588C39DB7405893F7B11467F2BC4B@Lounge> -----Original Message----- From: Sven Pran Sent: Monday, January 24, 2011 2:15 PM To: 'Bridge Laws Mailing List' Subject: Re: [BLML] Unsure On Behalf Of Hans van Staveren > Sent: 24. januar 2011 15:07 > To: 'Bridge Laws Mailing List' > Subject: Re: [BLML] Unsure > > Any halfway aspiring TD to-be in the Netherlands knows that after an unsure > answer you call the TD, the TD sends the unsure-sayer out of earshot, let's his > partner explain and calls the player back. > > This is no problem whatsoever. So why is this discussion so long-winded? > > Hans *Because there seems to be a "school" developing that a partnership can *always get around the duty to explain their understandings by simply *claiming "undiscussed" or "no agreement" when they are asked to explain some *part of their auction. Not from me there isn't. If anyone is doing this to avoid giving information the opposition are entitled to they are cheating and should suffer severe penalties. However I do believe that when a partnership has no agreements that cover a situation then that is the information that the opposition are entitled to. Together with all other relevant systemic agreements of course. ********************************** david.j.barton at lineone.net ********************************** From sater at xs4all.nl Mon Jan 24 15:52:22 2011 From: sater at xs4all.nl (Hans van Staveren) Date: Mon, 24 Jan 2011 15:52:22 +0100 Subject: [BLML] Unsure In-Reply-To: <4A3418FD24684B579A369D685BF6D6FB@Lounge> References: <586818.75337.qm@web28510.mail.ukl.yahoo.com> <4D3704A8.8090006@skynet.be> <128171.88697.qm@web25402.mail.ukl. yahoo.com> < 4D382CB9.2060700@ulb.ac.be><6410CABD-9AF3-45A7-9B10-F5EDC78C6095@starpowe r. net><4D3852CE.6080908@ulb.ac.be> <4D386625.90105@meteo.fr> <4D3869DD.9030708@ulb.ac.be> <4D3873A6.5070104@meteo.fr> <107026.33279.qm@web28515.mail.ukl.yahoo.com> <4D39BC9E.9040108@nhcc.net> <1820574630.137446.1295675648497.JavaMail.ngmail@webmail11.arcor-online.net> <4D3D7134.4000803@ulb.ac.be> <431431.44115.qm@web28515.mail.ukl.yahoo.com> <4D3D7F51.70805@ulb.ac.be> <395644.68863.qm@web28514.mail.ukl.yahoo.com><000101cbbbcf$4f8dd270$eea97750$@no> <011301cbbbcf$fc8610a0$f59231e0$@nl> <4A3418FD24684B579A369D685BF6D6FB@Lounge> Message-ID: <011401cbbbd6$4e621f80$eb265e80$@nl> And in that case which TD could ever claim otherwise? Hans -----Original Message----- From: blml-bounces at rtflb.org [mailto:blml-bounces at rtflb.org] On Behalf Of David Sent: maandag 24 januari 2011 15:22 To: Bridge Laws Mailing List Subject: Re: [BLML] Unsure -----Original Message----- From: Hans van Staveren Sent: Monday, January 24, 2011 2:07 PM To: 'Bridge Laws Mailing List' Subject: Re: [BLML] Unsure Any halfway aspiring TD to-be in the Netherlands knows that after an unsure answer you call the TD, the TD sends the unsure-sayer out of earshot, let's his partner explain and calls the player back. This is no problem whatsoever. So why is this discussion so long-winded? Hans Because you would get exactly the same answer from the person who made the call. We have no systemic agreement about this call. It could be A or B. David ********************************** david.j.barton at lineone.net ********************************** _______________________________________________ Blml mailing list Blml at rtflb.org http://lists.rtflb.org/mailman/listinfo/blml From ehaa at starpower.net Mon Jan 24 16:05:14 2011 From: ehaa at starpower.net (Eric Landau) Date: Mon, 24 Jan 2011 10:05:14 -0500 Subject: [BLML] Unsure In-Reply-To: <41319555EF7045089AF47AA66D4BC62C@Lounge> References: <000901cbba0f$ce55b3f0$6b011bd0$@no><586818.75337.qm@web28510.mail.ukl.yahoo.com> <4D3704A8.8090006@skynet.be> <128171.88697.qm@web25402.mail.ukl. yahoo.com> < 4D382CB9.2060700@ulb.ac.be><6410CABD-9AF3-45A7-9B10-F5EDC78C6095@starpowe r. net><4D3852CE.6080908@ulb.ac.be> <4D386625.90105@meteo.fr> <4D3869DD.9030708@ulb.ac.be> <4D3873A6.5070104@meteo.fr> <107026.33279.qm@web28515.mail.ukl.yahoo.com> <4D39BC9E.9040108@nhcc.net> <1820574630.137446.1295675648497.JavaMail.ngmail@webmail11.arcor-online.net><4D3A93A3.8020906@skynet.be><1896834273.57934.1295686896979.JavaMail.ngmail@webmail13.arcor-online.net> <514721598.171618.1295725647449.JavaMail.ngmail@webmail10.arcor-online.net> <000901cbba82$f84fdf50$e8ef9df0$@no> <41319555EF7045089AF47AA66D4BC62C@Lounge> Message-ID: On Jan 22, 2011, at 7:31 PM, David wrote: > But here lies the nub of the issue. > If you were able to give a REGULAR (and absolutely honest) partnership > 10 undiscussed bidding situations you will probably find them in > agreement > about 8 times. There will be no way to tell in advance which they > will agree > on because each will involve a judgement as to what your partner > thinks > that you will think that the bid means. > > In my opinion such statistics would only provide evidence that the > partnership > had become good at predicting how partner would act - not that they > had > pre-existing **agreements** on some or all of the 8 hands they got > right. > > How one instance of judging correctly can provide **strong** > indication > of a CPU (cheating) is beyond me. The problem is that the laws regarding disclosure of one's partnership understandings (at least as currently interpreted) insist that "explicit partnership understandings" agreed to in discussion and "implicit partnership understandings" gained through mutual experience without discussion be treated absolutely identically. This has so obscured the already fine line between "becom[ing] good at predicting how partner would act" and "forming implicit partnership understandings through mutual experience" that it has disappeared entirely. Eric Landau 1107 Dale Drive Silver Spring MD 20910 ehaa at starpower.net From david.j.barton at lineone.net Mon Jan 24 16:07:00 2011 From: david.j.barton at lineone.net (David) Date: Mon, 24 Jan 2011 15:07:00 -0000 Subject: [BLML] Unsure In-Reply-To: <011401cbbbd6$4e621f80$eb265e80$@nl> References: <586818.75337.qm@web28510.mail.ukl.yahoo.com> <4D3704A8.8090006@skynet.be> <128171.88697.qm@web25402.mail.ukl. yahoo.com> < 4D382CB9.2060700@ulb.ac.be><6410CABD-9AF3-45A7-9B10-F5EDC78C6095@starpowe r. net><4D3852CE.6080908@ulb.ac.be> <4D386625.90105@meteo.fr> <4D3869DD.9030708@ulb.ac.be> <4D3873A6.5070104@meteo.fr> <107026.33279.qm@web28515.mail.ukl.yahoo.com> <4D39BC9E.9040108@nhcc.net> <1820574630.137446.1295675648497.JavaMail.ngmail@webmail11.arcor-online.net> <4D3D7134.4000803@ulb.ac.be> <431431.44115.qm@web28515.mail.ukl.yahoo.com> <4D3D7F51.70805@ulb.ac.be> <395644.68863.qm@web28514.mail.ukl.yahoo.com><000101cbbbcf$4f8dd270$eea97750$@no> <011301cbbbcf$fc8610a0$f59231e0$@nl><4A3418FD24684B579A369D685BF6D6FB@Lounge> <011401cbbbd6$4e621f80$eb265e80$@nl> Message-ID: Any halfway aspiring TD to-be in the Netherlands knows that after an unsure answer you call the TD, the TD sends the unsure-sayer out of earshot, let's his partner explain and calls the player back. This is no problem whatsoever. So why is this discussion so long-winded? Hans Because you would get exactly the same answer from the person who made the call. We have no systemic agreement about this call. It could be A or B. David And in that case which TD could ever claim otherwise? Hans Well Sven apparently * the laws assume that a call always *have a meaning according to explicit or implicit partnership understanding An assertion with which I strongly disagree David ********************************** david.j.barton at lineone.net ********************************** From svenpran at online.no Mon Jan 24 16:20:27 2011 From: svenpran at online.no (Sven Pran) Date: Mon, 24 Jan 2011 16:20:27 +0100 Subject: [BLML] Unsure In-Reply-To: <4D3D8C91.5020007@ulb.ac.be> References: <586818.75337.qm@web28510.mail.ukl.yahoo.com> <4D3704A8.8090006@skynet.be> <128171.88697.qm@web25402.mail.ukl. yahoo.com> < 4D382CB9.2060700@ulb.ac.be><6410CABD-9AF3-45A7-9B10-F5EDC78C6095@starpowe r. net> <4D3852CE.6080908@ulb.ac.be> <4D386625.90105@meteo.fr> <4D3869DD.9030708@ulb.ac.be> <4D3873A6.5070104@meteo.fr> <107026.33279.qm@web28515.mail.ukl.yahoo.com> <4D39BC9E.9040108@nhcc.net> <1820574630.137446.1295675648497.JavaMail.ngmail@webmail11.arcor-online.net> <4D3A93A3.8020906@skynet.be> <000901cbba0f$ce55b3f0$6b011bd0$@no> <4D3D72AF.3060606@ulb.ac.be> <000001cbbbce$a0c010f0$e24032d0$@no> <4D3D8C91.5020007@ulb.ac.be> Message-ID: <000001cbbbda$3b61c940$b2255bc0$@no> > -----Original Message----- > From: Alain Gottcheiner [mailto:agot at ulb.ac.be] > Sent: 24. januar 2011 15:29 > To: Bridge Laws Mailing List > Cc: Sven Pran > Subject: Re: [BLML] Unsure > > Le 24/01/2011 14:57, Sven Pran a ?crit : > >> -----Original Message----- > >> From: Alain Gottcheiner [mailto:agot at ulb.ac.be] Sven Pran a ?crit : > > .............. > >>> This means that a player who does not provide the correct agreement > >>> (if necessary confirmed by other > >>> evidence) is always deemed to having given MI. > >>> This includes the "not discussed" and "I have no idea" statements! > >> AG : there must be limits to this. How long your list of agremeents, > >> there > > must be > >> some non-covered area. > > Any player is entitled to a full explanation of opponents' auction > > whether the relevant partnership understanding is explicit or implicit. > > > > "Not discussed", "I have no idea" and similar statements are not such > > explanations, they are simply statements to the effect that the player > > does not know what the correct explanation is. > > So, you're basically saying that every partnership has at least an implicit > understanding of any bid they could legally make. > What does 1NT-2D-5S mean in your system ? > I would be happy with responding "undiscussed" (or "impossible", which also is > non-descriptive) If I ever should use such a bid it would be for some purpose trusting that my partner would understand, never else. I have an example: Partnering in a rubber with the late Arild Torp (in his time a very good Norwegian player) he "out of a blue sky" bid 5NT trusting that I would understand it as the "great, free 5NT convention" which indeed I did. But we had never talked about this particular convention. Would "Not discussed" have been a correct explanation? Certainly not, although it would have been 100% true. Yes, I am basically saying that every call during an auction has been made for a purpose and opponents are entitled to a full explanation of this purpose (i.e. the meaning of the call). Failing to provide opponents with such explanation is a violation of Law 20F regardless of whether the reason for such failure is ignorance or deliberate concealment. From ehaa at starpower.net Mon Jan 24 16:20:46 2011 From: ehaa at starpower.net (Eric Landau) Date: Mon, 24 Jan 2011 10:20:46 -0500 Subject: [BLML] Naive question about double shots In-Reply-To: References: Message-ID: On Jan 23, 2011, at 12:02 AM, David Grabiner wrote: > The example I usually give is the following (I was East and agreed > with the > split ruling): > > North had a 3-3-3-4 6-count, and the following auction occurred, > both vulnerable > at IMPs: > > S W N E > 1S P 2S ..P > P X 3S X > AP > > 3SX went down three for -800. The TD was called because West's > double was > suggested by East's slow pass, and he ruled that West's double was an > infraction. However, he adjusted only the E-W score to +200 for 2S > undoubled > down two. For N-S, he ruled that the damage was caused by North's > 3S bid, which > was not a normal bridge action, rather than by the infraction; had > North passed, > East would have bid 3D making four and N-S would have been -130. > Therefore, N-S > kept their -800. > > This is an example of a double shot; North bid 3S on a hand which > had no good > bridge reason to do so, possibly with the expectation that he would > be able to > keep his +730 if it was doubled and made, but would get the result > rolled back > to 2S if it went down. > > From: "Jerry Fusselman" > >> Would someone please give an example of an action that he would >> term a >> double shot that should not be allowed? Ideally, the example would >> teach naive bridge players why the laws do not allow double shots. David gives an excellent example of an alleged double shot that *would* not be allowed, but fails in the far more difficult (some would say impossible) task posed by Jerry's question of justifying why it *should* not be allowed. Eric Landau 1107 Dale Drive Silver Spring MD 20910 ehaa at starpower.net From svenpran at online.no Mon Jan 24 16:26:14 2011 From: svenpran at online.no (Sven Pran) Date: Mon, 24 Jan 2011 16:26:14 +0100 Subject: [BLML] Unsure In-Reply-To: References: <586818.75337.qm@web28510.mail.ukl.yahoo.com> <4D3704A8.8090006@skynet.be> <128171.88697.qm@web25402.mail.ukl. yahoo.com> < 4D382CB9.2060700@ulb.ac.be><6410CABD-9AF3-45A7-9B10-F5EDC78C6095@starpowe r. net><4D3852CE.6080908@ulb.ac.be> <4D386625.90105@meteo.fr> <4D3869DD.9030708@ulb.ac.be> <4D3873A6.5070104@meteo.fr> <107026.33279.qm@web28515.mail.ukl.yahoo.com> <4D39BC9E.9040108@nhcc.net> <1820574630.137446.1295675648497.JavaMail.ngmail@webmail11.arcor-online.net> <4D3D7134.4000803@ulb.ac.be> <431431.44115.qm@web28515.mail.ukl.yahoo.com> <4D3D7F51.70805@ulb.ac.be> <395644.68863.qm@web28514.mail.ukl.yahoo.com><000101cbbbcf$4f8dd270$eea97750$@no> <011301cbbbcf$fc8610a0$f59231e0$@nl><4A3418FD24684B579A369D685BF6D6FB@Lounge> <011401cbbbd6$4e621f80$eb265e80$@nl> Message-ID: <000101cbbbdb$0a0717a0$1e1546e0$@no> On Behalf Of David > Sent: 24. januar 2011 16:07 > To: Bridge Laws Mailing List > Subject: Re: [BLML] Unsure > > > > > Any halfway aspiring TD to-be in the Netherlands knows that after an unsure > answer you call the TD, the TD sends the unsure-sayer out of earshot, let's his > partner explain and calls the player back. > > This is no problem whatsoever. So why is this discussion so long-winded? > > Hans > > Because you would get exactly the same answer from the person who made the > call. > > We have no systemic agreement about this call. It could be A or B. > > David > > And in that case which TD could ever claim otherwise? > > Hans > > Well Sven apparently > > * the laws assume that a call always > *have a meaning according to explicit or implicit partnership understanding > > An assertion with which I strongly disagree If the partnership can show me some positive evidence (System documentation, CC or whatever) that the meaning of the call can be either A or B I shall accept that as the explanation. Otherwise, if the actual hand matches either A or B but not both I shall rule misinformation and eventually judge whether opponents have been damaged. Take a look at Law 75! From ehaa at starpower.net Mon Jan 24 16:28:46 2011 From: ehaa at starpower.net (Eric Landau) Date: Mon, 24 Jan 2011 10:28:46 -0500 Subject: [BLML] Unsure In-Reply-To: <4D3C1199.8090100@skynet.be> References: <000901cbba0f$ce55b3f0$6b011bd0$@no><586818.75337.qm@web28510.mail.ukl.yahoo.com> <4D3704A8.8090006@skynet.be> <128171.88697.qm@web25402.mail.ukl. yahoo.com> < 4D382CB9.2060700@ulb.ac.be><6410CABD-9AF3-45A7-9B10-F5EDC78C6095@starpowe r. net><4D3852CE.6080908@ulb.ac.be> <4D386625.90105@meteo.fr> <4D3869DD.9030708@ulb.ac.be> <4D3873A6.5070104@meteo.fr> <107026.33279.qm@web28515.mail.ukl.yahoo.com> <4D39BC9E.9040108@nhcc.net> <1820574630.137446.1295675648497.JavaMail.ngmail@webmail11.arcor-online.net><4D3A93A3.8020906@skynet.be> <1896834273.57934.1295686896979.JavaMail.ngmail@webmail13.arcor-online.net> <4D3B0A64.5060306@gmail.com> <4D3C1199.8090100@skynet.be> Message-ID: <7E77D564-FDC4-4455-9576-4D027554EA8E@starpower.net> On Jan 23, 2011, at 6:31 AM, Herman De Wael wrote: > Brian wrote: > >> On 01/22/2011 08:37 AM, David wrote: >> >>> Well it looks crazy to me. >>> >>> This is effectively saying that every partnership has agreements >>> about every >>> sequence >>> and hence "no agreement" can never be the appropriate answer even >>> when it is >>> absolutely >>> correct. >>> >>> I am glad I do not have to try and enforce anything like that in >>> my local >>> club. I would be >>> down to 2 tables in no time flat. >>> >>> Now I would accept that "no agreement" is rarely a full >>> disclosure as there >>> are likely to >>> be other sequences for which there are agreements which would >>> influence the >>> likely >>> meanings of this particular sequence and these inferences should be >>> disclosed. But at >>> the end of the day, if the partnership does not have an agreement >>> it is not >>> up to a >>> National Authority to deem they have an agreement and penalise >>> them for >>> failing to disclose it. >> >> I try to resist posting "Me too" messages but I can't resist it this >> time. I begin to wonder how many people on this list actually >> remember >> what it's like to direct at a grass-roots club. >> >> The last club at which I played, and was occasional TD, had the host >> rota system so as to accommodate single players. If your partner was >> the host for that night, it meant you had a high probability of >> playing with someone you'd not partnered before with about two >> minutes >> to rough out the details of a system. I think there would have been >> many occasions where, had the bidding gone (e.g.) 1NT-(dbl)-2D, an >> inquiry about 2D would have got an answer something like >> >> "Well, we agreed transfers, but we didn't have time to agree whether >> they're on or off over a double, so partner might have either >> diamonds >> or hearts, or just possibly he's three suited short in diamonds >> and is >> going to redouble your presumed double as an SOS takeout for me to >> pick one of the other three suits". > > Well, you've just proven the exact opposite of what you ae argueing > against. The above sentence is the correct explanation, Thank you, Herman! Anyone who doesn't understand and agree with those last seven words should be made to write them 100 times on the blackboard. > and "no > agreement" is not the correct explanation. What you seem to forget is > that the above explanation is probably what the opponents will > understand when you say "no agreement", and therefore that shortcut is > acceptable. But that is only so because you and your partner and your > opponents belong to the same club. Go with the same opponent to > another > club, and you will have to start with "well, we never played > together > and had only two minutes of discussion, but in our club, the majority > play ...". If you simply say "no agreement" in that club, you will > have > misinformed your opponents there. Eric Landau 1107 Dale Drive Silver Spring MD 20910 ehaa at starpower.net From ehaa at starpower.net Mon Jan 24 16:51:46 2011 From: ehaa at starpower.net (Eric Landau) Date: Mon, 24 Jan 2011 10:51:46 -0500 Subject: [BLML] Unsure In-Reply-To: References: <000901cbba0f$ce55b3f0$6b011bd0$@no><586818.75337.qm@web28510.mail.ukl.yahoo.com> <4D3704A8.8090006@skynet.be> <128171.88697.qm@web25402.mail.ukl. yahoo.com> < 4D382CB9.2060700@ulb.ac.be><6410CABD-9AF3-45A7-9B10-F5EDC78C6095@starpowe r. net><4D3852CE.6080908@ulb.ac.be> <4D386625.90105@meteo.fr> <4D3869DD.9030708@ulb.ac.be> <4D3873A6.5070104@meteo.fr> <107026.33279.qm@web28515.mail.ukl.yahoo.com> <4D39BC9E.9040108@nhcc.net> <1820574630.137446.1295675648497.JavaMail.ngmail@webmail11.arcor-online.net><4D3A93A3.8020906@skynet.be> <1896834273.57934.1295686896979.JavaMail.ngmail@webmail13.arcor-online.net> <4D3B0A64.5060306@gmail.com> <4D3C1199.8090100@skynet.be><4D3C244D.4090507@gmail.com> <4D3C3753.8060008@skynet.be> Message-ID: <769F18AF-BE33-4CDF-ABD7-65699FBEFD21@starpower.net> On Jan 23, 2011, at 6:35 PM, David wrote: > From: Herman De Wael > > Brian wrote: > >> No, Herman, not OK. >> >> I agreed with David's statement (cut and pasted here for convenience) >> "Now I would accept that "no agreement" is rarely a full >> disclosure as >> there are likely to be other sequences for which there are agreements >> which would influence the likely meanings of this particular sequence >> and these inferences should be disclosed." >> >> As far as I remember the postings in this thread (maybe I snipped too >> much?) there was a suggestion that giving alternative meanings to an >> opponent was MI. >> >> In the example I gave, clearly there is a possibility the pair are >> playing transfers, as they do so without the double. Equally, there's >> a possibility that it's "system off" over the double. >> >> If I'm correct about what I believe I read, and the suggestion was >> that giving an opponent more than one possible meaning and saying "I >> don't think we have an agreement, you decide" was automatically MI, >> then I stick by what I said. >> >> If I misread/misunderstood something earlier in the thread, then fair >> enough, my mistake. > > *The problem with your point of view is that, true though it is, it > helps > *us no further along. When you have to guess at partner's > intentions, and > *you guess correctly, then there is always the possibility that your > *guess was based on something you did not mention. That something > may be > *enough for the TD to rule that there has been MI. > * > *In that light, to be calling it MI when you guess and tell them, is > *going far too way left field. > * > *And that is what some people in this thread suggested. > > Let me give you a real life example. > > (P) P (1S) 1N > (2S) 2N > > I alert the 2N and explain that it MAY be conventional. > Upon further questioning I explain that we have no agreement > or previous experience that covers this situation but we do have > the following agreements > > (P) P (1S) 1N//(P) 2N would be natural invitational > 1N (2S) 2N would be lebensohl > 1N (P) 2N would be transfer to D > (P) P (1S) 1N//(2S) X would be take out as it is covered > by our general agreement on > doubles > > So I have a straight coin flip between natural and lebensohl right? > > WRONG. > > Spades have been bid and supported together with the three I am > looking > at implies partner is likely to be short. > > Both the oppos bidding reduces the likelihood that partner holds > the values > for an invite. > > The guy on my right is clearly thinking of bidding again > reinforcing the > above 2 points. > > If partner has an invite he MAY have chosen a different route (3N > or X) > because he will know that 2N is undiscussed. > > When I place my bet on lebensohl I would guess I have an 80% chance > of being right. > > These sort of inferences are ALWAYS available and it is no surprise to > me that (honest) partnerships get these decisions right far more often > than they get them wrong. > > Evidence of a CPU? > You must be joking. I agree. Short of revealing his actual hand, David gives his opponents a full dump of the knowledge base on which he bases his subsequent chain of reasoning. That "evens the playing field" by giving his opponents the same opportunity to construct their own chain of reasoning from the same knowledge that David has. If David is correct to conclude that his partner's call is 80% to be Lebensohl, and his opponents fail to correctly deduce the same conclusion from the same information, that's evidence of nothing more than that David knows a bit more about the game of bridge than his opponents do. Eric Landau 1107 Dale Drive Silver Spring MD 20910 ehaa at starpower.net From david.j.barton at lineone.net Mon Jan 24 16:55:01 2011 From: david.j.barton at lineone.net (David) Date: Mon, 24 Jan 2011 15:55:01 -0000 Subject: [BLML] Unsure In-Reply-To: <000101cbbbdb$0a0717a0$1e1546e0$@no> References: <586818.75337.qm@web28510.mail.ukl.yahoo.com> <4D3704A8.8090006@skynet.be> <128171.88697.qm@web25402.mail.ukl. yahoo.com> < 4D382CB9.2060700@ulb.ac.be><6410CABD-9AF3-45A7-9B10-F5EDC78C6095@starpowe r. net><4D3852CE.6080908@ulb.ac.be> <4D386625.90105@meteo.fr> <4D3869DD.9030708@ulb.ac.be> <4D3873A6.5070104@meteo.fr> <107026.33279.qm@web28515.mail.ukl.yahoo.com> <4D39BC9E.9040108@nhcc.net> <1820574630.137446.1295675648497.JavaMail.ngmail@webmail11.arcor-online.net> <4D3D7134.4000803@ulb.ac.be> <431431.44115.qm@web28515.mail.ukl.yahoo.com> <4D3D7F51.70805@ulb.ac.be> <395644.68863.qm@web28514.mail.ukl.yahoo.com><000101cbbbcf$4f8dd270$eea97750$@no> <011301cbbbcf$fc8610a0$f59231e0$@nl><4A3418FD24684B579A369D685BF6D6FB@Lounge> <011401cbbbd6$4e621f80$eb265e80$@nl> <000101cbbbdb$0a0717a0$1e1546e0$@no> Message-ID: <670306DF0C5446E8B8770FE884FA6E1C@Lounge> > Well Sven apparently > > * the laws assume that a call always > *have a meaning according to explicit or implicit partnership understanding > > An assertion with which I strongly disagree *If the partnership can show me some positive evidence (System documentation, *CC or whatever) that the meaning of the call can be either A or B I shall *accept that as the explanation. Otherwise, if the actual hand matches either *A or B but not both I shall rule misinformation and eventually judge whether *opponents have been damaged. Take a look at Law 75! Yes indeed let us look at L75. L75 starts After a misleading explanation........ The opposition are told that as a consequence of other agreements 2N is either natural or lebensohl but as the sequence has neither occurred before nor is the subject of an agreement it is impossible to say which. Now in my opinion (assuming that is a true statement) there is nothing misleading. L75B starts The actual partnership agreement is....... Well there is no actual partnership agreement and that is what the opposition have been told. So what is the problem here? L75 is the subject of Mistaken Explanations and Mistaken calls. Calls which are not covered by systemic agreements and are explained as such are neither mistakenly explained nor mistakenly bid. ********************************** david.j.barton at lineone.net ********************************** From agot at ulb.ac.be Mon Jan 24 17:00:54 2011 From: agot at ulb.ac.be (Alain Gottcheiner) Date: Mon, 24 Jan 2011 17:00:54 +0100 Subject: [BLML] Unsure In-Reply-To: <000001cbbbda$3b61c940$b2255bc0$@no> References: <586818.75337.qm@web28510.mail.ukl.yahoo.com> <4D3704A8.8090006@skynet.be> <128171.88697.qm@web25402.mail.ukl. yahoo.com> < 4D382CB9.2060700@ulb.ac.be><6410CABD-9AF3-45A7-9B10-F5EDC78C6095@starpowe r. net> <4D3852CE.6080908@ulb.ac.be> <4D386625.90105@meteo.fr> <4D3869DD.9030708@ulb.ac.be> <4D3873A6.5070104@meteo.fr> <107026.33279.qm@web28515.mail.ukl.yahoo.com> <4D39BC9E.9040108@nhcc.net> <1820574630.137446.1295675648497.JavaMail.ngmail@webmail11.arcor-online.net> <4D3A93A3.8020906@skynet.be> <000901cbba0f$ce55b3f0$6b011bd0$@no> <4D3D72AF.3060606@ulb.ac.be> <000001cbbbce$a0c010f0$e24032d0$@no> <4D3D8C91.5020007@ulb.ac.be> <000001cbbbda$3b61c940$b2255bc0$@no> Message-ID: <4D3DA236.8090506@ulb.ac.be> Le 24/01/2011 16:20, Sven Pran a ?crit : >> -----Original Message----- >> From: Alain Gottcheiner [mailto:agot at ulb.ac.be] >> Sent: 24. januar 2011 15:29 >> To: Bridge Laws Mailing List >> Cc: Sven Pran >> Subject: Re: [BLML] Unsure >> >> Le 24/01/2011 14:57, Sven Pran a ?crit : >>>> -----Original Message----- >>>> From: Alain Gottcheiner [mailto:agot at ulb.ac.be] Sven Pran a ?crit : >>> .............. >>>>> This means that a player who does not provide the correct agreement >>>>> (if necessary confirmed by other >>>>> evidence) is always deemed to having given MI. >>>>> This includes the "not discussed" and "I have no idea" statements! >>>> AG : there must be limits to this. How long your list of agremeents, >>>> there >>> must be >>>> some non-covered area. >>> Any player is entitled to a full explanation of opponents' auction >>> whether the relevant partnership understanding is explicit or implicit. >>> >>> "Not discussed", "I have no idea" and similar statements are not such >>> explanations, they are simply statements to the effect that the player >>> does not know what the correct explanation is. >> So, you're basically saying that every partnership has at least an > implicit >> understanding of any bid they could legally make. >> What does 1NT-2D-5S mean in your system ? >> I would be happy with responding "undiscussed" (or "impossible", which > also is >> non-descriptive) > If I ever should use such a bid it would be for some purpose trusting that > my partner would understand, never else. > > I have an example: Partnering in a rubber with the late Arild Torp (in his > time a very good Norwegian player) he "out of a blue sky" bid 5NT trusting > that I would understand it as the "great, free 5NT convention" which indeed > I did. But we had never talked about this particular convention. > > Would "Not discussed" have been a correct explanation? Certainly not, > although it would have been 100% true. > > Yes, I am basically saying that every call during an auction has been made > for a purpose and opponents are entitled to a full explanation of this > purpose (i.e. the meaning of the call). AG : wonderful ! So, I open 1NT, you answer 2D, and I mistankelny pull out 3S in lieu of 3H. One can argue for ever whether this call has been done for purpose, but you will be hard pressed to explain it. > Failing to provide opponents with > such explanation is a violation of Law 20F regardless of whether the reason > for such failure is ignorance or deliberate concealment. > AG : disagree. The TD is entitled to use an educated guess to determine the facts ; if he determines that the most probable explanation is a mechanical error, there is neither deliberate concealment nor ignorance. And in said case I would accept the claim : "I pulled the wrong card". From gordonrainsford at btinternet.com Mon Jan 24 17:01:46 2011 From: gordonrainsford at btinternet.com (Gordon Rainsford) Date: Mon, 24 Jan 2011 16:01:46 +0000 Subject: [BLML] Unsure In-Reply-To: <000301cbbbd1$22a7eaa0$67f7bfe0$@no> References: <586818.75337.qm@web28510.mail.ukl.yahoo.com> <4D3704A8.8090006@skynet.be> <128171.88697.qm@web25402.mail.ukl. yahoo.com> < 4D382CB9.2060700@ulb.ac.be><6410CABD-9AF3-45A7-9B10-F5EDC78C6095@starpowe r. net> <4D3852CE.6080908@ulb.ac.be> <4D386625.90105@meteo.fr> <4D3869DD.9030708@ulb.ac.be> <4D3873A6.5070104@meteo.fr> <107026.33279.qm@web28515.mail.ukl.yahoo.com> <4D39BC9E.9040108@nhcc.net> <1820574630.137446.1295675648497.JavaMail.ngmail@webmail11.arcor-online.net> <4D3D7134.4000803@ulb.ac.be> <431431.44115.qm@web28515.mail.ukl.yahoo.com> <4D3D7F51.70805@ulb.ac.be> <395644.68863.qm@web28514.mail.ukl.yahoo.com> <000101cbbbcf$4f8dd270$eea97750$@no> <011301cbbbcf$fc8610a0$f59231e0$@nl> <000301cbbbd1$22a7eaa0$67f7bfe0$@no> Message-ID: <3AD55E4C-78A8-46AC-B8E6-8E973DDF6DD9@btinternet.com> On 24 Jan 2011, at 14:15, Sven Pran wrote: >> > > Because there seems to be a "school" developing that a partnership can > always get around the duty to explain their understandings by simply > claiming "undiscussed" or "no agreement" when they are asked to > explain some > part of their auction. If you are referring to the current discussion on Bridgebase Forums, not only is no such "school" developing, but *no-one* is arguing as above. The question is whether, as you maintain, when you truly have had no discussion at all and no prior experience to develop an implicit agreement, you should make up an agreement that corresponds with your hopes when you made the call. Please note I haven't gone through all of this rapidly expanding thread before making this reply. Gordon Rainsford -------------- next part -------------- An HTML attachment was scrubbed... URL: http://lists.rtflb.org/pipermail/blml/attachments/20110124/e9b91bc9/attachment.html From svenpran at online.no Mon Jan 24 17:48:31 2011 From: svenpran at online.no (Sven Pran) Date: Mon, 24 Jan 2011 17:48:31 +0100 Subject: [BLML] Unsure In-Reply-To: <670306DF0C5446E8B8770FE884FA6E1C@Lounge> References: <586818.75337.qm@web28510.mail.ukl.yahoo.com> <4D3704A8.8090006@skynet.be> <128171.88697.qm@web25402.mail.ukl. yahoo.com> < 4D382CB9.2060700@ulb.ac.be><6410CABD-9AF3-45A7-9B10-F5EDC78C6095@starpowe r. net><4D3852CE.6080908@ulb.ac.be> <4D386625.90105@meteo.fr> <4D3869DD.9030708@ulb.ac.be> <4D3873A6.5070104@meteo.fr> <107026.33279.qm@web28515.mail.ukl.yahoo.com> <4D39BC9E.9040108@nhcc.net> <1820574630.137446.1295675648497.JavaMail.ngmail@webmail11.arcor-online.net> <4D3D7134.4000803@ulb.ac.be> <431431.44115.qm@web28515.mail.ukl.yahoo.com> <4D3D7F51.70805@ulb.ac.be> <395644.68863.qm@web28514.mail.ukl.yahoo.com><000101cbbbcf$4f8dd270$eea97750$@no> <011301cbbbcf$fc8610a0$f59231e0$@nl><4A3418FD24684B579A369D685BF6D6FB@Lounge> <011401cbbbd6$4e621f80$eb265e80$@nl> <000101cbbbdb$0a0717a0$1e1546e0$@no> <670306DF0C5446E8B8770FE884FA6E1C@Lounge> Message-ID: <000801cbbbe6$886147a0$9923d6e0$@no> On Behalf Of David ............... > *If the partnership can show me some positive evidence (System documentation, > *CC or whatever) that the meaning of the call can be either A or B I shall *accept > that as the explanation. Otherwise, if the actual hand matches either *A or B but > not both I shall rule misinformation and eventually judge whether *opponents have > been damaged. Take a look at Law 75! > > Yes indeed let us look at L75. > > L75 starts > After a misleading explanation........ > > The opposition are told that as a consequence of other agreements 2N is either > natural or lebensohl but as the sequence has neither occurred before nor is the > subject of an agreement it is impossible to say which. > > Now in my opinion (assuming that is a true statement) there is nothing misleading. > > L75B starts > The actual partnership agreement is....... > > Well there is no actual partnership agreement and that is what the opposition > have been told. > > So what is the problem here? > > L75 is the subject of Mistaken Explanations and Mistaken calls. > Calls which are not covered by systemic agreements and are explained as such > are neither mistakenly explained nor mistakenly bid. So you missed the possibly most important sentence in Law 75? the Director is to presume Mistaken Explanation, rather than Mistaken Call, in the absence of evidence to the contrary From david.j.barton at lineone.net Mon Jan 24 18:20:42 2011 From: david.j.barton at lineone.net (David) Date: Mon, 24 Jan 2011 17:20:42 -0000 Subject: [BLML] Unsure In-Reply-To: <000801cbbbe6$886147a0$9923d6e0$@no> References: <586818.75337.qm@web28510.mail.ukl.yahoo.com> <4D3704A8.8090006@skynet.be> <128171.88697.qm@web25402.mail.ukl. yahoo.com> < 4D382CB9.2060700@ulb.ac.be><6410CABD-9AF3-45A7-9B10-F5EDC78C6095@starpowe r. net><4D3852CE.6080908@ulb.ac.be> <4D386625.90105@meteo.fr> <4D3869DD.9030708@ulb.ac.be> <4D3873A6.5070104@meteo.fr> <107026.33279.qm@web28515.mail.ukl.yahoo.com> <4D39BC9E.9040108@nhcc.net> <1820574630.137446.1295675648497.JavaMail.ngmail@webmail11.arcor-online.net> <4D3D7134.4000803@ulb.ac.be> <431431.44115.qm@web28515.mail.ukl.yahoo.com> <4D3D7F51.70805@ulb.ac.be> <395644.68863.qm@web28514.mail.ukl.yahoo.com><000101cbbbcf$4f8dd270$eea97750$@no> <011301cbbbcf$fc8610a0$f59231e0$@nl><4A3418FD24684B579A369D685BF6D6FB@Lounge> <011401cbbbd6$4e621f80$eb265e80$@nl> <000101cbbbdb$0a0717a0$1e1546e0$@no><670306DF0C5446E8B8770FE884FA6E1C@Lounge> <000801cbbbe6$886147a0$9923d6 e0$@no> Message-ID: <61F91A343F83423DBF87BC39B487C84B@Lounge> -----Original Message----- From: Sven Pran Sent: Monday, January 24, 2011 4:48 PM To: 'Bridge Laws Mailing List' Subject: Re: [BLML] Unsure On Behalf Of David ............... > *If the partnership can show me some positive evidence (System documentation, > *CC or whatever) that the meaning of the call can be either A or B I shall *accept > that as the explanation. Otherwise, if the actual hand matches either *A or B but > not both I shall rule misinformation and eventually judge whether *opponents have > been damaged. Take a look at Law 75! > > Yes indeed let us look at L75. > > L75 starts > After a misleading explanation........ > > The opposition are told that as a consequence of other agreements 2N is either > natural or lebensohl but as the sequence has neither occurred before nor is the > subject of an agreement it is impossible to say which. > > Now in my opinion (assuming that is a true statement) there is nothing misleading. > > L75B starts > The actual partnership agreement is....... > > Well there is no actual partnership agreement and that is what the opposition > have been told. > > So what is the problem here? > > L75 is the subject of Mistaken Explanations and Mistaken calls. > Calls which are not covered by systemic agreements and are explained as such > are neither mistakenly explained nor mistakenly bid. *So you missed the possibly most important sentence in Law 75? * *the Director is to presume Mistaken Explanation, rather than Mistaken Call, *in the absence of evidence to the contrary No I did not miss it. You have the testimony of *BOTH* players that this is not covered and as you would expect there is no reference to it on the system card. It might not be evidence that you would like to put a lot of weight on, but you know that is all you can expect in this situation. You may not even have the evidence of how the player would have taken the bid if the player next in turn calls (say 3S in the example case). So no there is not an *absence* of evidence to the contrary. So, please tell me, with law references, what you believe is the obligation of a player who KNOWS that the bid is NOT covered by his systemic agreements? ********************************** david.j.barton at lineone.net ********************************** From adam at tameware.com Mon Jan 24 18:29:14 2011 From: adam at tameware.com (Adam Wildavsky) Date: Mon, 24 Jan 2011 18:29:14 +0100 Subject: [BLML] ACBL New Orleans NABC Cases Posted Message-ID: On Tue, Nov 16, 2010 at 4:09 PM, I wrote: http://www.acbl.org/play/casebooks/NewOrleans2010.html > If you want to discuss a particular case please include the case number in the Subject: line and indicate whether it's an NABC+ or Non-NABC+ case. > I'll post my comments here shortly. Other panelist comments are not yet available. Panelist comments are now posted as well. -- Adam Wildavsky www.tameware.com -------------- next part -------------- An HTML attachment was scrubbed... URL: http://lists.rtflb.org/pipermail/blml/attachments/20110124/14d471ce/attachment-0001.html From ehaa at starpower.net Mon Jan 24 18:47:52 2011 From: ehaa at starpower.net (Eric Landau) Date: Mon, 24 Jan 2011 12:47:52 -0500 Subject: [BLML] Unsure In-Reply-To: <4D3D7134.4000803@ulb.ac.be> References: <586818.75337.qm@web28510.mail.ukl.yahoo.com> <4D3704A8.8090006@skynet.be> <128171.88697.qm@web25402.mail.ukl. yahoo.com> < 4D382CB9.2060700@ulb.ac.be><6410CABD-9AF3-45A7-9B10-F5EDC78C6095@starpowe r. net> <4D3852CE.6080908@ulb.ac.be> <4D386625.90105@meteo.fr> <4D3869DD.9030708@ulb.ac.be> <4D3873A6.5070104@meteo.fr> <107026.33279.qm@web28515.mail.ukl.yahoo.com> <4D39BC9E.9040108@nhcc.net> <1820574630.137446.1295675648497.JavaMail.ngmail@webmail11.arcor-online.net> <4D3D7134.4000803@ulb.ac.be> Message-ID: On Jan 24, 2011, at 7:31 AM, Alain Gottcheiner wrote: > AG : indeed. "conflicting agreements" might be the right explanation ; > "unsure" will never be. You are unsure, not the system, and TFLB says > you must speak about the system, not about your state of mind. Of course, as I'm sure Alain well knows, "conflicting agreements" by itself cannot constitute a "right explanation"; you must also disclose what those conflicting agreements are. Eric Landau 1107 Dale Drive Silver Spring MD 20910 ehaa at starpower.net From ehaa at starpower.net Mon Jan 24 19:03:41 2011 From: ehaa at starpower.net (Eric Landau) Date: Mon, 24 Jan 2011 13:03:41 -0500 Subject: [BLML] Unsure In-Reply-To: <431431.44115.qm@web28515.mail.ukl.yahoo.com> References: <586818.75337.qm@web28510.mail.ukl.yahoo.com> <4D3704A8.8090006@skynet.be> <128171.88697.qm@web25402.mail.ukl. yahoo.com> < 4D382CB9.2060700@ulb.ac.be><6410CABD-9AF3-45A7-9B10-F5EDC78C6095@starpowe r. net> <4D3852CE.6080908@ulb.ac.be> <4D386625.90105@meteo.fr> <4D3869DD.9030708@ulb.ac.be> <4D3873A6.5070104@meteo.fr> <107026.33279.qm@web28515.mail.ukl.yahoo.com> <4D39BC9E.9040108@nhcc.net> <1820574630.137446.1295675648497.JavaMail.ngmail@webmail11.arcor-online.net> <4D3D7134.4000803@ulb.ac.be> <431431.44115.qm@web28515.mail.ukl.yahoo.com> Message-ID: On Jan 24, 2011, at 8:15 AM, Nigel Guthrie wrote: > {Alain] > > AG : indeed. "conflicting agreements" might be the right explanation ; > "unsure" will never be. You are unsure, not the system, and TFLB says > you must speak about the system, not about your state of mind. > > [Nigel] > Unfortunately, Allain's interpretation contrasts with Richard's. > > I sympathise more with Alain's interpretation. I once claimed that > if you agree > to play, say "Jannerston Precision", have a summary of it on your > system card, > and declare that to your opponents then that is your systemic > agreement. Even if > you've forgotten it or never learnt it. In common sense or ordinary > law, such an > interpretation seems obvious. > > Richard Hills contradicted me. For Bridge law purposes, Richard > explained that > your agreement is what is in your head -- not what is written on your > system-card or notes. That is: what you've forgotten or never > learnt properly is > not your legal systemic agreement. > > Current Bridge law does resolve which of these incompatible > interpretations is > correct. Perhaps this issue will be addressed in 2018 0r 2028? These interpretations are not necessarily incompatible. You have an explicit agreement that you have agreed to play Jannerston Precision, in which your call means whatever. Your may lack a correct understanding of that explicit agreement, but if and when partner becomes aware of your lack of understanding, you perforce form a separate, additional implicit agreement based on partnership experience, which may affect the implications of your explicit agreement. The totality of your partnership understanding includes both agreements, the explicit one and the potentially modifying implicit one; the rules require you to disclose them both. Eric Landau 1107 Dale Drive Silver Spring MD 20910 ehaa at starpower.net From david.j.barton at lineone.net Mon Jan 24 19:04:02 2011 From: david.j.barton at lineone.net (David) Date: Mon, 24 Jan 2011 18:04:02 -0000 Subject: [BLML] Unsure In-Reply-To: <61F91A343F83423DBF87BC39B487C84B@Lounge> References: e0$@no> <61F91A343F83423DBF87BC39B487C84B@Lounge> Message-ID: -----Original Message----- From: David Sent: Monday, January 24, 2011 5:20 PM To: Bridge Laws Mailing List Subject: Re: [BLML] Unsure -----Original Message----- From: Sven Pran Sent: Monday, January 24, 2011 4:48 PM To: 'Bridge Laws Mailing List' Subject: Re: [BLML] Unsure On Behalf Of David ............... > *If the partnership can show me some positive evidence (System documentation, > *CC or whatever) that the meaning of the call can be either A or B I shall *accept > that as the explanation. Otherwise, if the actual hand matches either *A or B but > not both I shall rule misinformation and eventually judge whether *opponents have > been damaged. Take a look at Law 75! > > Yes indeed let us look at L75. > > L75 starts > After a misleading explanation........ > > The opposition are told that as a consequence of other agreements 2N is either > natural or lebensohl but as the sequence has neither occurred before nor is the > subject of an agreement it is impossible to say which. > > Now in my opinion (assuming that is a true statement) there is nothing misleading. > > L75B starts > The actual partnership agreement is....... > > Well there is no actual partnership agreement and that is what the opposition > have been told. > > So what is the problem here? > > L75 is the subject of Mistaken Explanations and Mistaken calls. > Calls which are not covered by systemic agreements and are explained as such > are neither mistakenly explained nor mistakenly bid. *So you missed the possibly most important sentence in Law 75? * *the Director is to presume Mistaken Explanation, rather than Mistaken Call, *in the absence of evidence to the contrary No I did not miss it. You have the testimony of *BOTH* players that this is not covered and as you would expect there is no reference to it on the system card. It might not be evidence that you would like to put a lot of weight on, but you know that is all you can expect in this situation. You may not even have the evidence of how the player would have taken the bid if the player next in turn calls (say 3S in the example case). So no there is not an *absence* of evidence to the contrary. So, please tell me, with law references, what you believe is the obligation of a player who KNOWS that the bid is NOT covered by his systemic agreements? I have just come across this advice in an official EBU publication (Tangerine Book). I should have looked there earlier. Answering Questions Players often err by saying something like, ?I?m going to take my partner?s bid to mean...? They are trying to be helpful, but it is the wrong thing to do. Only explain specific agreements. If the truthful answer is, ?We have never discussed that and nothing like it has come up before?, then say so. Another possible answer might be, ?We have never discussed this particular bid but in similar situations we play this...? In a regular partnership you will acquire knowledge of partner?s habits. Answers such as, ?No agreement? or ?Random? are then unlikely to be the full story. ********************************** david.j.barton at lineone.net ********************************** _______________________________________________ Blml mailing list Blml at rtflb.org http://lists.rtflb.org/mailman/listinfo/blml From ehaa at starpower.net Mon Jan 24 19:20:51 2011 From: ehaa at starpower.net (Eric Landau) Date: Mon, 24 Jan 2011 13:20:51 -0500 Subject: [BLML] Exam question In-Reply-To: <4D3D7DFD.2020801@ulb.ac.be> References: <4D3D3B2F.5090107@skynet.be> <4D3D7DFD.2020801@ulb.ac.be> Message-ID: <6C8DE7A4-A82A-494F-AD8A-95D7CF3849FB@starpower.net> On Jan 24, 2011, at 8:26 AM, Alain Gottcheiner wrote: > Le 24/01/2011 9:41, Herman De Wael a ?crit : > >> This one may well make it into an exam question for a TD course, >> if you >> ever need one: >> >> On Saturday, I was playing 2He. I had just won the first six >> tricks, and >> I showed AK of hearts, saying "I think I am going to make this >> contract". Did I claim or am I allowed to play on? > > I would vote for claim : "suggest in any way that the play be > shortened". It's clear that Herman's showing the H AK meet the criteria of L68A unless "he demonstrably did not intend to claim". The latter, IMO, is not a particularly loose criterion; it applies only if the action was either accidental and unintended, or was taken in a mistaken attempt to conform to proper procedure unrelated to claims (as in the example in TFLB). "I wasn't claiming; I was just screwing around," cannot, in general, reach the level of "demonstrable", so I also vote for claim. Eric Landau 1107 Dale Drive Silver Spring MD 20910 ehaa at starpower.net From ehaa at starpower.net Mon Jan 24 19:37:15 2011 From: ehaa at starpower.net (Eric Landau) Date: Mon, 24 Jan 2011 13:37:15 -0500 Subject: [BLML] Unsure In-Reply-To: <4D3D7F51.70805@ulb.ac.be> References: <586818.75337.qm@web28510.mail.ukl.yahoo.com> <4D3704A8.8090006@skynet.be> <128171.88697.qm@web25402.mail.ukl. yahoo.com> < 4D382CB9.2060700@ulb.ac.be><6410CABD-9AF3-45A7-9B10-F5EDC78C6095@starpowe r. net> <4D3852CE.6080908@ulb.ac.be> <4D386625.90105@meteo.fr> <4D3869DD.9030708@ulb.ac.be> <4D3873A6.5070104@meteo.fr> <107026.33279.qm@web28515.mail.ukl.yahoo.com> <4D39BC9E.9040108@nhcc.net> <1820574630.137446.1295675648497.JavaMail.ngmail@webmail11.arcor-online.net> <4D3D7134.4000803@ulb.ac.be> <431431.44115.qm@web28515.mail.ukl.yahoo.com> <4D3D7F51.70805@ulb.ac.be> Message-ID: On Jan 24, 2011, at 8:32 AM, Alain Gottcheiner wrote: > Le 24/01/2011 14:15, Nigel Guthrie a ?crit : > >> {Alain] >> >> AG : indeed. "conflicting agreements" might be the right >> explanation ; >> "unsure" will never be. You are unsure, not the system, and TFLB says >> you must speak about the system, not about your state of mind. >> >> [Nigel] >> Unfortunately, Allain's interpretation contrasts with Richard's. >> >> I sympathise more with Alain's interpretation. I once claimed that >> if you agree >> to play, say "Jannerston Precision", have a summary of it on your >> system card, >> and declare that to your opponents then that is your systemic >> agreement. Even if >> you've forgotten it or never learnt it. In common sense or >> ordinary law, such an >> interpretation seems obvious. >> >> Richard Hills contradicted me. For Bridge law purposes, Richard >> explained that >> your agreement is what is in your head -- not what is written on your >> system-card or notes. That is: what you've forgotten or never >> learnt properly is >> not your legal systemic agreement. > > AG : doesn't hold. How could your agreement be precise on mondays and > inexistent on tuesdays, because you happen to work hard on tuesdays ? Perhaps your Monday and Tuesday night partner knows you work hard on Tuesdays, and has seen the results. This has created an implicit partnership understanding based on experience that you will apply your explicit agreement quite precisely on Mondays but rather sloppily on Tuesdays. Since the law demands that explicit and implicit understandings be treated identically, they apply with equal weight: the difference between Mondays and Tuesdays have become as much a part of your systemic understandings as are the actual details of the explicitly agreed method. (I don't necessarily find the law to be sensible in this, but it is what it is.) Eric Landau 1107 Dale Drive Silver Spring MD 20910 ehaa at starpower.net From ehaa at starpower.net Mon Jan 24 19:52:14 2011 From: ehaa at starpower.net (Eric Landau) Date: Mon, 24 Jan 2011 13:52:14 -0500 Subject: [BLML] Unsure In-Reply-To: <000001cbbbce$a0c010f0$e24032d0$@no> References: <586818.75337.qm@web28510.mail.ukl.yahoo.com> <4D3704A8.8090006@skynet.be> <128171.88697.qm@web25402.mail.ukl. yahoo.com> < 4D382CB9.2060700@ulb.ac.be><6410CABD-9AF3-45A7-9B10-F5EDC78C6095@starpowe r. net> <4D3852CE.6080908@ulb.ac.be> <4D386625.90105@meteo.fr> <4D3869DD.9030708@ulb.ac.be> <4D3873A6.5070104@meteo.fr> <107026.33279.qm@web28515.mail.ukl.yahoo.com> <4D39BC9E.9040108@nhcc.net> <1820574630.137446.1295675648497.JavaMail.ngmail@webmail11.arcor-online.net> <4D3A93A3.8020906@skynet.be> <000901cbba0f$ce55b3f0$6b011bd0$@no> <4D3D72AF.3060606@ulb.ac.be> <000001cbbbce$a0c010f0$e24032d0$@no> Message-ID: <78BC7D50-DDDC-4459-BA0B-87467613E5B0@starpower.net> On Jan 24, 2011, at 8:57 AM, Sven Pran wrote: >> From: Alain Gottcheiner [mailto:agot at ulb.ac.be] >> >> Sven Pran a ?crit : >> >>>> This means that a player who does not provide the >>>> correct agreement (if necessary confirmed by other >>>> evidence) is always deemed to having given MI. >>>> This includes the "not discussed" and "I have no idea" statements! >> >> AG : there must be limits to this. How long your list of >> agremeents, there >> must be some non-covered area. > > Any player is entitled to a full explanation of opponents' auction > whether > the relevant partnership understanding is explicit or implicit. True. > "Not discussed", "I have no idea" and similar statements are not such > explanations, they are simply statements to the effect that the > player does > not know what the correct explanation is. According to law this is > synonymous with misexplanation, i.e. failure to give correct > information. But when you presume lack of explanation to be synonymous with misexplanation, you are mandating that any player is entitled to a full explanation of opponents' auction whether the relevant partnership understanding is explicit, implicit, or non-existent. Eric Landau 1107 Dale Drive Silver Spring MD 20910 ehaa at starpower.net From ehaa at starpower.net Mon Jan 24 20:09:45 2011 From: ehaa at starpower.net (Eric Landau) Date: Mon, 24 Jan 2011 14:09:45 -0500 Subject: [BLML] Unsure In-Reply-To: <000101cbbbcf$4f8dd270$eea97750$@no> References: <586818.75337.qm@web28510.mail.ukl.yahoo.com> <4D3704A8.8090006@skynet.be> <128171.88697.qm@web25402.mail.ukl. yahoo.com> < 4D382CB9.2060700@ulb.ac.be><6410CABD-9AF3-45A7-9B10-F5EDC78C6095@starpowe r. net> <4D3852CE.6080908@ulb.ac.be> <4D386625.90105@meteo.fr> <4D3869DD.9030708@ulb.ac.be> <4D3873A6.5070104@meteo.fr> <107026.33279.qm@web28515.mail.ukl.yahoo.com> <4D39BC9E.9040108@nhcc.net> <1820574630.137446.1295675648497.JavaMail.ngmail@webmail11.arcor-online.net> <4D3D7134.4000803@ulb.ac.be> <431431.44115.qm@web28515.mail.ukl.yahoo.com> <4D3D7F51.70805@ulb.ac.be> <395644.68863.qm@web28514.mail.ukl.yahoo.com> <000101cbbbcf$4f8dd270$eea97750$@no> Message-ID: On Jan 24, 2011, at 9:02 AM, Sven Pran wrote: > On Behalf Of Nigel Guthrie >> >> [Alain] >> doesn't hold. How could your agreement be precise on mondays and >> nonexistent on tuesdays, because you happen to work hard on >> tuesdays ? >> >> {Nigel] >> Pass. Ask me another. I was merely trying to represent Richard's >> interpretation. >> >> Alain and others maintain that saying "Unsure" is an MI >> infraction. But can >> anybody tell us what the law prescribes you *should* say if >> "Unsure" is the truth? > > If "unsure" is the truth (and your answer) then you fail to give > opponents > the explanation they are entitled to. According to laws this is > misexplanation. "Unsure" is only an explanation of why you cannot > comply > with the laws; it is not an explanation of the auction as required > by the > laws. There is no law that requires one's body of partnership understandings to provide explanations for every possible auction, and you can't give what you ain't got. I do agree that "unsure" is a poor choice of word, implying that the uncertainty is one's own rather than the partnership's, but that's a semanic side-issue in this discussion, where we've been using it as the equivalent of "I don't know" or "we have no agreement". The laws cannot and do not require you to provide an explanation of an auction based on "understandings" that don't exist and never have. Eric Landau 1107 Dale Drive Silver Spring MD 20910 ehaa at starpower.net From ehaa at starpower.net Mon Jan 24 20:37:50 2011 From: ehaa at starpower.net (Eric Landau) Date: Mon, 24 Jan 2011 14:37:50 -0500 Subject: [BLML] Unsure In-Reply-To: <000201cbbbd0$8f4693b0$add3bb10$@no> References: <586818.75337.qm@web28510.mail.ukl.yahoo.com><4D3704A8.8090006@skynet.be><128171.88697.qm@web25402.mail.ukl. yahoo.com> <4D382CB9.2060700@ulb.ac.be><6410CABD-9AF3-45A7-9B10-F5EDC78C6095@starpower. net> <4D3852CE.6080908@ulb.ac.be> <4D386625.90105@meteo.fr><4D3869DD.9030708@ulb.ac.be> <4D3873A6.5070104@meteo.fr><107026.33279.qm@web28515.mail.ukl.yahoo.com><4D39BC9E.9040108@nhcc.net><1820574630.137446.1295675648497.JavaMail.ngmail@webmail11.arcor-online.net><4D3D7134.4000803@ulb.ac.be><431431.44115.qm@web28515.mail.ukl.yahoo.com><4D3D7F51.70805@ulb.ac.be> <395644.68863.qm@web28514.mail.ukl.yahoo.com> <43FB511CB5684DA490AAB2D8E8666177@Lounge> <000201cbbbd0$8f4693b0$add3bb10$@no> Message-ID: <2567B08F-AB43-4785-B914-3E50539A44D5@starpower.net> On Jan 24, 2011, at 9:11 AM, Sven Pran wrote: > On Behalf Of David > >> If you have NO agreement but are sure it means A or B but cannot >> tell them which >> then there is NO MI. > > Making a call completely at random (with the agreement that it has no > meaning) is probably illegal anywhere, the laws assume that a call > always > have a meaning according to explicit or implicit partnership > understanding. To get from A to B, Sven seems to assume that "no agreement on a meaning" is (legally) equivalent to "an agreement on no meaning". But they are not the same thing, nor even close. > If no evidence to the contrary can be provided then the meaning of > the call > shall be deemed to correspond with the card holding of the player > making > that call. > > Any explanation not conforming to this meaning is to be ruled > incorrect. > > Your example: "It is either A or B, I don't know which" is > therefore still a > failure to give correct explanation. Eric Landau 1107 Dale Drive Silver Spring MD 20910 ehaa at starpower.net From ehaa at starpower.net Mon Jan 24 21:05:38 2011 From: ehaa at starpower.net (Eric Landau) Date: Mon, 24 Jan 2011 15:05:38 -0500 Subject: [BLML] Naive question about double shots In-Reply-To: References: Message-ID: <327C53E4-386A-40BF-9871-4071863E8144@starpower.net> On Jan 24, 2011, at 9:23 AM, Jerry Fusselman wrote: > [David Grabiner] > > This is an example of a double shot; North bid 3S on a hand which > had no good > bridge reason to do so, possibly with the expectation that he would > be able to > keep his +730 if it was doubled and made, but would get the result > rolled back > to 2S if it went down. > > [Steve Willner] > There's a long history of thinking this is a bad thing, but I think > Jerry's question was _why_ it's bad. > > [Jerry] > Right. That's my main question. I also want to know what constitutes > a double shot. > > By the way, I think Marv would say OS should get +130, and Adam > Wildavsky would say OS should get +200. And Eric would say NOS should > get the negative of OS's score on the grounds that 3S is nowhere near > egregious. Well, no, I wouldn't say that. Not here. So-called "self-inflicted damage" can arise from *either* an egregious error or an apparent double shot. These are independent determinations requiring totally different sorts of findings. Jerry is quite right insofar as I would vociferously object if my fellow committee members were considering denying redress on the grounds that bidding 3S was an egregious error. But denying redress on the grounds that 3S was a deliberate double shot doesn't require any finding of error at all, egregious or otherwise. > [naive player] > In practice, sometimes 3S works out better than passing. To decide > what is a double shot, do directors check to see what works on the > specific deal and what doesn't and call everything that does not work > a double shot? Would pass (instead of 3S) also be a double shot? > After all, it seems one could paraphrase David's answer as follows: > Pass is an example of a double shot; North passed on a hand that > should have competed better. Possibly North passed with the > expectation that he could maybe set their contract two tricks for +200 > if everyone was balanced, but would get the result rolled back to 2S > if that was better. > > If everything is a double shot, then nothing is. So what makes 3S a > double shot and keeps pass from being a double shot? "Possibly with > the expectation" does not sound that convincing as a basis of ruling > NOS get nothing. > > Besides, 3S was not a free double shot. There was a chance that the > director would rule no UI or no LA to the first double. A very good > chance. You want 3S to have risk. OK, it did. > > I can see why OS should get +200, but what is wrong with giving NOS > -200? Eric Landau 1107 Dale Drive Silver Spring MD 20910 ehaa at starpower.net From blml at arcor.de Mon Jan 24 21:26:09 2011 From: blml at arcor.de (Thomas Dehn) Date: Mon, 24 Jan 2011 21:26:09 +0100 (CET) Subject: [BLML] Unsure In-Reply-To: References: <586818.75337.qm@web28510.mail.ukl.yahoo.com> <4D3704A8.8090006@skynet.be> <128171.88697.qm@web25402.mail.ukl. yahoo.com> < 4D382CB9.2060700@ulb.ac.be><6410CABD-9AF3-45A7-9B10-F5EDC78C6095@starpowe r. net> <4D3852CE.6080908@ulb.ac.be> <4D386625.90105@meteo.fr> <4D3869DD.9030708@ulb.ac.be> <4D3873A6.5070104@meteo.fr> <107026.33279.qm@web28515.mail.ukl.yahoo.com> <4D39BC9E.9040108@nhcc.net> <1820574630.137446.1295675648497.JavaMail.ngmail@webmail11.arcor-online.net> <4D3D7134.4000803@ulb.ac.be> <431431.44115.qm@web28515.mail.ukl.yahoo.com> <4D3D7F51.70805@ulb.ac.be> <395644.68863.qm@web28514.mail.ukl.yahoo.com> <000101cbbbcf$4f8dd270$eea97750$@no> Message-ID: <1868019136.66657.1295900769160.JavaMail.ngmail@webmail08.arcor-online.net> Eric Landau wrote: > On Jan 24, 2011, at 9:02 AM, Sven Pran wrote: > > > On Behalf Of Nigel Guthrie > >> > >> [Alain] > >> doesn't hold. How could your agreement be precise on mondays and > >> nonexistent on tuesdays, because you happen to work hard on > >> tuesdays ? > >> > >> {Nigel] > >> Pass. Ask me another. I was merely trying to represent Richard's > >> interpretation. > >> > >> Alain and others maintain that saying "Unsure" is an MI > >> infraction. But can > >> anybody tell us what the law prescribes you *should* say if > >> "Unsure" is the truth? > > > > If "unsure" is the truth (and your answer) then you fail to give > > opponents > > the explanation they are entitled to. According to laws this is > > misexplanation. "Unsure" is only an explanation of why you cannot > > comply > > with the laws; it is not an explanation of the auction as required > > by the > > laws. > > There is no law that requires one's body of partnership > understandings to provide explanations for every possible auction, > and you can't give what you ain't got. I do agree that "unsure" is a > poor choice of word, implying that the uncertainty is one's own > rather than the partnership's, but that's a semanic side-issue in > this discussion, where we've been using it as the equivalent of "I > don't know" or "we have no agreement". The laws cannot and do not > require you to provide an explanation of an auction based on > "understandings" that don't exist and never have. Try to see it from the other side: Your regular partner made that bid. This implies that partner has some sort of expectation that you can work out what the bid means. Thomas From ehaa at starpower.net Mon Jan 24 22:38:22 2011 From: ehaa at starpower.net (Eric Landau) Date: Mon, 24 Jan 2011 16:38:22 -0500 Subject: [BLML] Unsure In-Reply-To: <000001cbbbda$3b61c940$b2255bc0$@no> References: <586818.75337.qm@web28510.mail.ukl.yahoo.com> <4D3704A8.8090006@skynet.be> <128171.88697.qm@web25402.mail.ukl. yahoo.com> < 4D382CB9.2060700@ulb.ac.be><6410CABD-9AF3-45A7-9B10-F5EDC78C6095@starpowe r. net> <4D3852CE.6080908@ulb.ac.be> <4D386625.90105@meteo.fr> <4D3869DD.9030708@ulb.ac.be> <4D3873A6.5070104@meteo.fr> <107026.33279.qm@web28515.mail.ukl.yahoo.com> <4D39BC9E.9040108@nhcc.net> <1820574630.137446.1295675648497.JavaMail.ngmail@webmail11.arcor-online.net> <4D3A93A3.8020906@skynet.be> <000901cbba0f$ce55b3f0$6b011bd0$@no> <4D3D72AF.3060606@ulb.ac.be> <000001cbbbce$a0c010f0$e24032d0$@no> <4D3D8C91.5020007@ulb.ac.be> <000001cbbbda$3b61c940$b2255bc0$@no> Message-ID: <6881475C-27B3-4235-9F92-189A361E840A@starpower.net> On Jan 24, 2011, at 10:20 AM, Sven Pran wrote: >> From: Alain Gottcheiner [mailto:agot at ulb.ac.be] >> >> Le 24/01/2011 14:57, Sven Pran a ?crit : >> >>> "Not discussed", "I have no idea" and similar statements are not >>> such >>> explanations, they are simply statements to the effect that the >>> player >>> does not know what the correct explanation is. >> >> So, you're basically saying that every partnership has at least an >> implicit >> understanding of any bid they could legally make. >> What does 1NT-2D-5S mean in your system ? >> I would be happy with responding "undiscussed" (or "impossible", >> which also is >> non-descriptive) > > If I ever should use such a bid it would be for some purpose > trusting that > my partner would understand, never else. > > I have an example: Partnering in a rubber with the late Arild Torp > (in his > time a very good Norwegian player) he "out of a blue sky" bid 5NT > trusting > that I would understand it as the "great, free 5NT convention" > which indeed > I did. But we had never talked about this particular convention. > > Would "Not discussed" have been a correct explanation? Certainly not, > although it would have been 100% true. > > Yes, I am basically saying that every call during an auction has > been made > for a purpose and opponents are entitled to a full explanation of this > purpose (i.e. the meaning of the call). Failing to provide > opponents with > such explanation is a violation of Law 20F regardless of whether > the reason > for such failure is ignorance or deliberate concealment. "A full explanation of this purpose"? Sven seems to say that you owe your opponents not only a correct explanation as to what your partner's call means, but also a revelation of the thinking behind his decision to make it. In my book, the former is about the nature of your partnership understandings, while the latter is about the difference between good and bad bridge, and I see nothing in TFLC that obliges you to address that difference for your opponents' benefit. Eric Landau 1107 Dale Drive Silver Spring MD 20910 ehaa at starpower.net From svenpran at online.no Mon Jan 24 23:14:08 2011 From: svenpran at online.no (Sven Pran) Date: Mon, 24 Jan 2011 23:14:08 +0100 Subject: [BLML] Unsure In-Reply-To: <61F91A343F83423DBF87BC39B487C84B@Lounge> References: <586818.75337.qm@web28510.mail.ukl.yahoo.com> <4D3704A8.8090006@skynet.be> <128171.88697.qm@web25402.mail.ukl. yahoo.com> < 4D382CB9.2060700@ulb.ac.be><6410CABD-9AF3-45A7-9B10-F5EDC78C6095@starpowe r. net><4D3852CE.6080908@ulb.ac.be> <4D386625.90105@meteo.fr> <4D3869DD.9030708@ulb.ac.be> <4D3873A6.5070104@meteo.fr> <107026.33279.qm@web28515.mail.ukl.yahoo.com> <4D39BC9E.9040108@nhcc.net> <1820574630.137446.1295675648497.JavaMail.ngmail@webmail11.arcor-online.net> <4D3D7134.4000803@ulb.ac.be> <431431.44115.qm@web28515.mail.ukl.yahoo.com> <4D3D7F51.70805@ulb.ac.be> <395644.68863.qm@web28514.mail.ukl.yahoo.com><000101cbbbcf$4f8dd270$eea97750$@no> <011301cbbbcf$fc8610a0$f59231e0$@nl><4A3418FD24684B579A369D685BF6D6FB@Lounge> <011401cbbbd6$4e621f80$eb265e80$@nl> <000101cbbbdb$0a0717a0$1e1546e0$@no><670306DF0C5446E8B8770FE884FA6E1C@Lounge> <000801cbbbe6$886147a0$9923d! 6 e0$@no> <61F91A343F83 423DBF87BC39B487C84B@Lounge> Message-ID: <000601cbbc14$05898ee0$109caca0$@no> On Behalf Of David ............... > *So you missed the possibly most important sentence in Law 75? > * > *the Director is to presume Mistaken Explanation, rather than Mistaken Call, *in > the absence of evidence to the contrary > > No I did not miss it. > You have the testimony of *BOTH* players that this is not covered and as you > would expect there is no reference to it on the system card. > It might not be evidence that you would like to put a lot of weight on, but you know > that is all you can expect in this situation. > You may not even have the evidence of how the player would have taken the bid if > the player next in turn calls (say 3S in the example case). > > > So no there is not an *absence* of evidence to the contrary. > > So, please tell me, with law references, what you believe is the obligation of a > player who KNOWS that the bid is NOT covered by his systemic agreements? Why did the player in question make the call if it has no assigned meaning? How did he expect (or hope) that his partner would understand the call? From svenpran at online.no Mon Jan 24 23:28:26 2011 From: svenpran at online.no (Sven Pran) Date: Mon, 24 Jan 2011 23:28:26 +0100 Subject: [BLML] Unsure In-Reply-To: References: e0$@no> <61F91A343F83423DBF87BC39B487C84B@Lounge> Message-ID: <000701cbbc16$0500c4f0$0f024ed0$@no> On Behalf Of David ................... > I have just come across this advice in an official EBU publication > (Tangerine Book). I should have looked there earlier. > > Answering Questions > Players often err by saying something like, ?I?m going to take my partner?s > bid to > mean...? They are trying to be helpful, but it is the wrong thing to do. > Only explain > specific agreements. If the truthful answer is, ?We have never discussed > that and > nothing like it has come up before?, then say so. Another possible answer > might be, > ?We have never discussed this particular bid but in similar situations we > play this...? > In a regular partnership you will acquire knowledge of partner?s habits. > Answers such > as, ?No agreement? or ?Random? are then unlikely to be the full story. I am surprised if EBU really advices "no agreement" or similar to be a "full disclosure of partnership understandings" under any circumstances. Such statements may at the best disclose a reason why the player is unable to give a full explanation as required by the laws, but it is certainly no explanation of agreements. If a player uses a call which in his opinion conveys no information at all he should have some explanation to do why he does so, otherwise the information conveyed by the call is what shall be disclosed. From svenpran at online.no Mon Jan 24 23:34:38 2011 From: svenpran at online.no (Sven Pran) Date: Mon, 24 Jan 2011 23:34:38 +0100 Subject: [BLML] Unsure In-Reply-To: <78BC7D50-DDDC-4459-BA0B-87467613E5B0@starpower.net> References: <586818.75337.qm@web28510.mail.ukl.yahoo.com> <4D3704A8.8090006@skynet.be> <128171.88697.qm@web25402.mail.ukl. yahoo.com> < 4D382CB9.2060700@ulb.ac.be><6410CABD-9AF3-45A7-9B10-F5EDC78C6095@starpowe r. net> <4D3852CE.6080908@ulb.ac.be> <4D386625.90105@meteo.fr> <4D3869DD.9030708@ulb.ac.be> <4D3873A6.5070104@meteo.fr> <107026.33279.qm@web28515.mail.ukl.yahoo.com> <4D39BC9E.9040108@nhcc.net> <1820574630.137446.1295675648497.JavaMail.ngmail@webmail11.arcor-online.net> <4D3A93A3.8020906@skynet.be> <000901cbba0f$ce55b3f0$6b011bd0$@no> <4D3D72AF.3060606@ulb.ac.be> <000001cbbbce$a0c010f0$e24032d0$@no> <78BC7D50-DDDC-4459-BA0B-87467613E5B0@starpower.net> Message-ID: <000801cbbc16$e2466e00$a6d34a00$@no> On Behalf Of Eric Landau .................. > > Any player is entitled to a full explanation of opponents' auction > > whether the relevant partnership understanding is explicit or > > implicit. > > True. > > > "Not discussed", "I have no idea" and similar statements are not such > > explanations, they are simply statements to the effect that the player > > does not know what the correct explanation is. According to law this > > is synonymous with misexplanation, i.e. failure to give correct > > information. > > But when you presume lack of explanation to be synonymous with misexplanation, > you are mandating that any player is entitled to a full explanation of opponents' > auction whether the relevant partnership understanding is explicit, implicit, or non- > existent. I shall expect the player to provide a very good reason for using a call for which (in his own opinion) no understanding exists. I don't believe he can. From svenpran at online.no Mon Jan 24 23:42:52 2011 From: svenpran at online.no (Sven Pran) Date: Mon, 24 Jan 2011 23:42:52 +0100 Subject: [BLML] Unsure In-Reply-To: References: <586818.75337.qm@web28510.mail.ukl.yahoo.com> <4D3704A8.8090006@skynet.be> <128171.88697.qm@web25402.mail.ukl. yahoo.com> < 4D382CB9.2060700@ulb.ac.be><6410CABD-9AF3-45A7-9B10-F5EDC78C6095@starpowe r. net> <4D3852CE.6080908@ulb.ac.be> <4D386625.90105@meteo.fr> <4D3869DD.9030708@ulb.ac.be> <4D3873A6.5070104@meteo.fr> <107026.33279.qm@web28515.mail.ukl.yahoo.com> <4D39BC9E.9040108@nhcc.net> <1820574630.137446.1295675648497.JavaMail.ngmail@webmail11.arcor-online.net> <4D3D7134.4000803@ulb.ac.be> <431431.44115.qm@web28515.mail.ukl.yahoo.com> <4D3D7F51.70805@ulb.ac.be> <395644.68863.qm@web28514.mail.ukl.yahoo.com> <000101cbbbcf$4f8dd270$eea97750$@no> Message-ID: <000901cbbc18$08c7a610$1a56f230$@no> On Behalf Of Eric Landau ................... > > If "unsure" is the truth (and your answer) then you fail to give > > opponents the explanation they are entitled to. According to laws this > > is misexplanation. "Unsure" is only an explanation of why you cannot > > comply with the laws; it is not an explanation of the auction as > > required by the laws. > > There is no law that requires one's body of partnership understandings to provide > explanations for every possible auction, and you can't give what you ain't got. >From Law 20F1: "He is entitled to know about calls actually made, about relevant alternative calls available that were not made, and about relevant inferences from the choice of action where these are matters of partnership understanding." It is not a question of "every possible auction", but of "calls actually made" etc. > I do > agree that "unsure" is a poor choice of word, implying that the uncertainty is one's > own rather than the partnership's, but that's a semanic side-issue in this > discussion, where we've been using it as the equivalent of "I don't know" or "we > have no agreement". The laws cannot and do not require you to provide an > explanation of an auction based on "understandings" that don't exist and never > have. A player using a call which in his own opinion has no meaning has some explanation to do to defend why he uses such a call. From bmeadows666 at gmail.com Mon Jan 24 23:44:18 2011 From: bmeadows666 at gmail.com (Brian) Date: Mon, 24 Jan 2011 17:44:18 -0500 Subject: [BLML] Unsure In-Reply-To: <000701cbbc16$0500c4f0$0f024ed0$@no> References: e0$@no> <61F91A343F83423DBF87BC39B487C84B@Lounge> <000701cbbc16$0500c4f0$0f024ed0$@no> Message-ID: <4D3E00C2.8030709@gmail.com> On 01/24/2011 05:28 PM, Sven Pran wrote: > On Behalf Of David ................... >> I have just come across this advice in an official EBU >> publication (Tangerine Book). I should have looked there >> earlier. >> >> Answering Questions Players often err by saying something like, >> ?I?m going to take my partner?s bid to mean...? They are trying >> to be helpful, but it is the wrong thing to do. Only explain >> specific agreements. If the truthful answer is, ?We have never >> discussed that and nothing like it has come up before?, then say >> so. Another possible answer might be, ?We have never discussed >> this particular bid but in similar situations we play this...? In >> a regular partnership you will acquire knowledge of partner?s >> habits. Answers such as, ?No agreement? or ?Random? are then >> unlikely to be the full story. > > I am surprised if EBU really advices "no agreement" or similar to > be a "full disclosure of partnership understandings" under any > circumstances. Such statements may at the best disclose a reason > why the player is unable to give a full explanation as required by > the laws, but it is certainly no explanation of agreements. > I certainly remember that the EBU had a specific prohibition in the Yellow Book against inventing system where you believed you had no agreement, and stated that you should instead tell opponents that you had no agreement. That may be wildly out of date, I've not played bridge in England since 1997. Brian. > If a player uses a call which in his opinion conveys no information > at all he should have some explanation to do why he does so, > otherwise the information conveyed by the call is what shall be > disclosed. > From svenpran at online.no Mon Jan 24 23:48:42 2011 From: svenpran at online.no (Sven Pran) Date: Mon, 24 Jan 2011 23:48:42 +0100 Subject: [BLML] Unsure In-Reply-To: <6881475C-27B3-4235-9F92-189A361E840A@starpower.net> References: <586818.75337.qm@web28510.mail.ukl.yahoo.com> <4D3704A8.8090006@skynet.be> <128171.88697.qm@web25402.mail.ukl. yahoo.com> < 4D382CB9.2060700@ulb.ac.be><6410CABD-9AF3-45A7-9B10-F5EDC78C6095@starpowe r. net> <4D3852CE.6080908@ulb.ac.be> <4D386625.90105@meteo.fr> <4D3869DD.9030708@ulb.ac.be> <4D3873A6.5070104@meteo.fr> <107026.33279.qm@web28515.mail.ukl.yahoo.com> <4D39BC9E.9040108@nhcc.net> <1820574630.137446.1295675648497.JavaMail.ngmail@webmail11.arcor-online.net> <4D3A93A3.8020906@skynet.be> <000901cbba0f$ce55b3f0$6b011bd0$@no> <4D3D72AF.3060606@ulb.ac.be> <000001cbbbce$a0c010f0$e24032d0$@no> <4D3D8C91.5020007@ulb.ac.be> <000001cbbbda$3b61c940$b2255bc0$@no> <6881475C-27B3-4235-9F92-189A361E840A@starpower.net> Message-ID: <000a01cbbc18$d9fe9bd0$8dfbd370$@no> On Behalf Of Eric Landau ..................... > > Yes, I am basically saying that every call during an auction has been > > made for a purpose and opponents are entitled to a full explanation of > > this purpose (i.e. the meaning of the call). Failing to provide > > opponents with such explanation is a violation of Law 20F regardless > > of whether the reason for such failure is ignorance or deliberate > > concealment. > > "A full explanation of this purpose"? Sven seems to say that you owe your > opponents not only a correct explanation as to what your partner's call means, but > also a revelation of the thinking behind his decision to make it. In my book, the > former is about the nature of your partnership understandings, while the latter is > about the difference between good and bad bridge, and I see nothing in TFLC > that obliges you to address that difference for your opponents' benefit. Read "purpose" as "purpose to convey some meaning to partner". If you really need it spelled out in more detail than my "(i.e. the meaning of the call)" I suspect that your intention with this discussion is not to get anywhere but simply to create annoyance. From Hermandw at skynet.be Tue Jan 25 00:19:36 2011 From: Hermandw at skynet.be (Herman De Wael) Date: Tue, 25 Jan 2011 00:19:36 +0100 Subject: [BLML] Exam question In-Reply-To: <1PhIDl-0dAMts0@fwd00.aul.t-online.de> References: <4D3D3B2F.5090107@skynet.be> <1PhIDl-0dAMts0@fwd00.aul.t-online.de> Message-ID: <4D3E0908.5090009@skynet.be> Peter Eidt wrote: > From: Herman De Wael >> This one may well make it into an exam question for a TD course, if >> you ever need one: >> >> On Saturday, I was playing 2He. I had just won the first six tricks, >> and I showed AK of hearts, saying "I think I am going to make this >> contract". Did I claim or am I allowed to play on? > > Law 68: > "For a statement or action to constitute a claim or concession of tricks > under these Laws, it must refer to tricks other than one currently in > progress. If it does refer to subsequent tricks: > > A. Claim Defined > Any statement to the effect that a contestant will win a specific number of > tricks is a claim of those tricks. A contestant also claims when he > suggests that play be curtailed, or when he shows his cards (unless he > demonstrably did not intend to claim - for example, if declarer faces his > cards after an opening lead out of turn Law 54, not this Law, will apply)." > Is no-one noticing the word "specific" number here? I did not say I was going to make 8 tricks - I was clearly saying 8 or more. > [Peter] > > Yes, you did claim and no, you are not allowed to play on. > Then what is the word specific doing in that law-text? -- Herman De Wael Wilrijk Antwerpen Belgium From david.j.barton at lineone.net Tue Jan 25 00:21:40 2011 From: david.j.barton at lineone.net (David) Date: Mon, 24 Jan 2011 23:21:40 -0000 Subject: [BLML] Unsure In-Reply-To: <000601cbbc14$05898ee0$109caca0$@no> References: e0$@no> <61F91A343F83 423DBF87BC39B487C84B@Lounge> <000601cbbc14$05898ee0$109caca0$@no> Message-ID: <7558B97F13394D06B08E66CFA36EDC8F@Lounge> -----Original Message----- From: Sven Pran Sent: Monday, January 24, 2011 10:14 PM To: 'Bridge Laws Mailing List' Subject: Re: [BLML] Unsure On Behalf Of David ............... > *So you missed the possibly most important sentence in Law 75? > * > *the Director is to presume Mistaken Explanation, rather than Mistaken Call, *in > the absence of evidence to the contrary > > No I did not miss it. > You have the testimony of *BOTH* players that this is not covered and as you > would expect there is no reference to it on the system card. > It might not be evidence that you would like to put a lot of weight on, but you know > that is all you can expect in this situation. > You may not even have the evidence of how the player would have taken the bid if > the player next in turn calls (say 3S in the example case). > > > So no there is not an *absence* of evidence to the contrary. > > So, please tell me, with law references, what you believe is the obligation of a > player who KNOWS that the bid is NOT covered by his systemic agreements? *Why did the player in question make the call if it has no assigned meaning? *How did he expect (or hope) that his partner would understand the call? The auction under consideration was (1S) 1N (2S) 2N The person bidding 2N wanted to bid a non forcing 3D bid as he judged passing 2S would give a poor mp score. However our system had not defined whether 3D would be forcing nor had it defined whether 2N would be natural invitational or lebensohl. But it would be taken as one or the other. Not what I would understand as having NO assigned meaning. So with a damned if you do and damned if you don't choice he judged to bid 2N and hope it worked out. So both members of the partnership were aware that the meaning of the bid was ambiguous and this was the information that was given to the opposition. (along with the meanings of other relevant sequences of course). So the question remains. What information do you think should be given and why? ********************************** david.j.barton at lineone.net ********************************** From jfusselman at gmail.com Tue Jan 25 01:21:12 2011 From: jfusselman at gmail.com (Jerry Fusselman) Date: Mon, 24 Jan 2011 18:21:12 -0600 Subject: [BLML] Exam question In-Reply-To: <4D3E0908.5090009@skynet.be> References: <4D3D3B2F.5090107@skynet.be> <1PhIDl-0dAMts0@fwd00.aul.t-online.de> <4D3E0908.5090009@skynet.be> Message-ID: On Mon, Jan 24, 2011 at 5:19 PM, Herman De Wael wrote: > > Is no-one noticing the word "specific" number here? I did not say I was > going to make 8 tricks - I was clearly saying 8 or more. > I agree, and that's what I said, but I was the only one. Did I pass the exam question? Jerry Fusselman From grabiner at alumni.princeton.edu Tue Jan 25 01:47:17 2011 From: grabiner at alumni.princeton.edu (David Grabiner) Date: Mon, 24 Jan 2011 19:47:17 -0500 Subject: [BLML] Naive question about double shots In-Reply-To: References: Message-ID: <07E347798694429AB3338FF7BBA87E37@erdos> "Eric Landau" writes: > On Jan 23, 2011, at 12:02 AM, David Grabiner wrote: > >> The example I usually give is the following (I was East and agreed >> with the >> split ruling): >> >> North had a 3-3-3-4 6-count, and the following auction occurred, >> both vulnerable >> at IMPs: >> >> S W N E >> 1S P 2S ..P >> P X 3S X >> AP >> Result: down 3 for -800; had North passed, East would have bid 3D making four >> for +130. > David gives an excellent example of an alleged double shot that > *would* not be allowed, but fails in the far more difficult (some > would say impossible) task posed by Jerry's question of justifying > why it *should* not be allowed. The reason I believe it should not be allowed is that it is an attempt to abuse the penalty structure. North made an abnormal bid when the Laws could have protected him from the risk of the bid. However, we don't need to allege the double shot to deny adjustment. It suffices that the TD could rule that N-S were damaged by North's mistake rather than by West's infraction. Suppose that North held Jxx Kxx xxx Qxxx but had the CQ in with his spades. With the hand he thought he had, 3S would be a reasonable bid. But the TD would still rule that N-S were damaged by North's bad bidding; West's infraction created a situation in which normal play would give N-S -130 rather than -200, and then North made a serious error and should bear its consequences. From nigelguthrie at yahoo.co.uk Tue Jan 25 01:50:03 2011 From: nigelguthrie at yahoo.co.uk (Nigel Guthrie) Date: Tue, 25 Jan 2011 00:50:03 +0000 (GMT) Subject: [BLML] Exam question In-Reply-To: References: <4D3D3B2F.5090107@skynet.be> <1PhIDl-0dAMts0@fwd00.aul.t-online.de> <4D3E0908.5090009@skynet.be> Message-ID: <282108.88292.qm@web28507.mail.ukl.yahoo.com> [Herman de Wael] Is no-one noticing the word "specific" number here? I did not say I was going to make 8 tricks - I was clearly saying 8 or more. [Jerry Fusselman] I agree, and that's what I said, but I was the only one. Did I pass the exam question? {Nigel] "Pass the sick-bag Alice" would a more likely reaction from a player who waded through the endless threads on the vagaries of claim-law, From nigelguthrie at yahoo.co.uk Tue Jan 25 01:52:57 2011 From: nigelguthrie at yahoo.co.uk (Nigel Guthrie) Date: Tue, 25 Jan 2011 00:52:57 +0000 (GMT) Subject: [BLML] Unsure In-Reply-To: <43FB511CB5684DA490AAB2D8E8666177@Lounge> References: <586818.75337.qm@web28510.mail.ukl.yahoo.com><4D3704A8.8090006@skynet.be><128171.88697.qm@web25402.mail.ukl. yahoo.com> <4D382CB9.2060700@ulb.ac.be><6410CABD-9AF3-45A7-9B10-F5EDC78C6095@starpower. net> <4D3852CE.6080908@ulb.ac.be> <4D386625.90105@meteo.fr><4D3869DD.9030708@ulb.ac.be> <4D3873A6.5070104@meteo.fr><107026.33279.qm@web28515.mail.ukl.yahoo.com><4D39BC9E.9040108@nhcc.net><1820574630.137446.1295675648497.JavaMail.ngmail@webmail11.arcor-online.net><4D3D7134.4000803@ulb.ac.be><431431.44115.qm@web28515.mail.ukl.yahoo.com><4D3D7F51.70805@ulb.ac.be> <395644.68863.qm@web28514.mail.ukl.yahoo.com> <43FB511CB5684DA490AAB2D8E8666177@Lounge> Message-ID: <27919.7685.qm@web28506.mail.ukl.yahoo.com> {David} In my opinion this depends on what "unsure" means. If you have an agreement but you are unable to tell the opposition what it is then you will be guilty of MI. If you have NO agreement but are sure it means A or B but cannot tell them which then there is NO MI. {Nigel] So far BLMLers have evaded the central issue: For the typical player, the most frequent scenario is that he probably did agree a meaning but he is unsure what it is. Depending on the call asked about, his confidence varies on an open interval from completely unsure up to totally certain. Usually, but not always, his confidence exceeds 50% but he is *never* sure (and is sometimes wrong even when virtually certain). According to some law-makers and BLML directors, when a player is genuinely unsure, he must not guess. What then, should a typical player, who is 75% sure say to opponents? And what about other confidence levels? IMO the law should mandate that he guess, when unsure. From jfusselman at gmail.com Tue Jan 25 02:05:28 2011 From: jfusselman at gmail.com (Jerry Fusselman) Date: Mon, 24 Jan 2011 19:05:28 -0600 Subject: [BLML] Naive question about double shots In-Reply-To: <327C53E4-386A-40BF-9871-4071863E8144@starpower.net> References: <327C53E4-386A-40BF-9871-4071863E8144@starpower.net> Message-ID: On Mon, Jan 24, 2011 at 2:05 PM, Eric Landau wrote: > On Jan 24, 2011, at 9:23 AM, Jerry Fusselman wrote: > >> [Steve Willner] >> There's a long history of thinking this is a bad thing, but I think >> Jerry's question was _why_ it's bad. >> >> [Jerry] >> Right. ?That's my main question. ?I also want to know what constitutes >> a double shot. >> >> By the way, I think Marv would say OS should get +130, and Adam >> Wildavsky would say OS should get +200. ?And Eric would say NOS should >> get the negative of OS's score on the grounds that 3S is nowhere near >> egregious. > > Well, no, I wouldn't say that. ?Not here. Thanks so much for the clarification, and I apologize for getting your view mostly wrong. > > So-called "self-inflicted damage" can arise from *either* an > egregious error or an apparent double shot. ?These are independent > determinations requiring totally different sorts of findings. ?Jerry > is quite right insofar as I would vociferously object if my fellow > committee members were considering denying redress on the grounds > that bidding 3S was an egregious error. ?But denying redress on the > grounds that 3S was a deliberate double shot doesn't require any > finding of error at all, egregious or otherwise. > Alright, but I still want to know what defines a double shot. I guess we are talking about Law 12C1b: "If, subsequent to the irregularity, the non-offending side has contributed to its own damage by a serious error (unrelated to the infraction) or by wild or gambling action it does not receive relief in the adjustment for such part of the damage as is self-inflicted." The term double shot is undefined in the laws. Alright, I imagine that no one on BLML can define it. Fine. Perhaps someone can answer a few questions to help me decide what it means. Can the best bid ever be called a double shot? Can a good bid that is nearly the best be called a double shot? What frame of mind do you need to protect yourself from being told your action was a double shot? The EBU white book has the phrase "double shot" nine times. Here are the cases: 1. "Gamboling double" as opposed to the sure-thing doubles, I guess. And this would only explain what constitutes a double-shot double. Perhaps one that is less than x% likely to work out well in some sense? 2. "suggestion of a double shot" which leaves "double shot" undefined. 3. "no question of a double shot since they did not know there was an infraction" Great, but that is hardly a definition. This merely gives us one case of what is not a double shot. I would like to note, however, that the NOS rarely know for sure if there is an infraction: Perhaps the director will rule that there was no LA to the action taken. Seems to me that gamboling actions are always dangerous in this light. So why do we even bother with a concept of double shots? Anyway, on with the list. 4 and 5. "if there is an element of a double shot...." which leaves it undefined. 6. "apparent double shot attempt" which leaves it undefined. Apparent to whom? 7. describes an example of a double shot as "a stupid sacrifice ... with no possible justification." That sounds to me like a serious error, so one hardly needs the doctrine of "double shots" to handle it. I think we can therefore safely ignore this reference. 8. "What is commonly termed a ?double shot? is a gambling action within the meaning of Law 12C1B." Totally circular, and therefore of no help. 9. An index reference to 4 and 5. If you can tell what a double shot is from that, you must be smarter than me. Please let me know what it is, and I promise to try to learn it. Jerry Fusselman From jfusselman at gmail.com Tue Jan 25 02:13:55 2011 From: jfusselman at gmail.com (Jerry Fusselman) Date: Mon, 24 Jan 2011 19:13:55 -0600 Subject: [BLML] Naive question about double shots In-Reply-To: <07E347798694429AB3338FF7BBA87E37@erdos> References: <07E347798694429AB3338FF7BBA87E37@erdos> Message-ID: On Mon, Jan 24, 2011 at 6:47 PM, David Grabiner wrote: > "Eric Landau" writes: > >> David gives an excellent example of an alleged double shot that >> *would* not be allowed, but fails in the far more difficult (some >> would say impossible) task posed by Jerry's question of justifying >> why it *should* not be allowed. > > The reason I believe it should not be allowed is that it is an attempt to abuse > the penalty structure. Interesting. Do you mean we have to read minds to find that a call is a double shot? I had not noticed that possibility before. > ?North made an abnormal bid when the Laws could have > protected him from the risk of the bid. How does North know this? Has he been assured this somehow, or is it only ex post knowledge that we imagine that he had ex ante? > > However, we don't need to allege the double shot to deny adjustment. ?It > suffices that the TD could rule that N-S were damaged by North's mistake rather > than by West's infraction. ?Suppose that North held Jxx Kxx xxx Qxxx but had the > CQ in with his spades. ?With the hand he thought he had, 3S would be a > reasonable bid. ?But the TD would still rule that N-S were damaged by North's > bad bidding; West's infraction created a situation in which normal play would > give N-S -130 rather than -200, and then North made a serious error and should > bear its consequences. > Ah, it sounds like you disagree with Eric and would be willing (in some cases) to call 3S a serious error. Interesting. How would you find out if this was the case? Jerry Fusselman From swillner at nhcc.net Tue Jan 25 04:34:51 2011 From: swillner at nhcc.net (Steve Willner) Date: Mon, 24 Jan 2011 22:34:51 -0500 Subject: [BLML] Unsure In-Reply-To: <27919.7685.qm@web28506.mail.ukl.yahoo.com> References: <586818.75337.qm@web28510.mail.ukl.yahoo.com><4D3704A8.8090006@skynet.be><128171.88697.qm@web25402.mail.ukl. yahoo.com> <4D382CB9.2060700@ulb.ac.be><6410CABD-9AF3-45A7-9B10-F5EDC78C6095@starpower. net> <4D3852CE.6080908@ulb.ac.be> <4D386625.90105@meteo.fr><4D3869DD.9030708@ulb.ac.be> <4D3873A6.5070104@meteo.fr><107026.33279.qm@web28515.mail.ukl.yahoo.com><4D39BC9E.9040108@nhcc.net><1820574630.137446.1295675648497.JavaMail.ngmail@webmail11.arcor-online.net><4D3D7134.4000803@ulb.ac.be><431431.44115.qm@web28515.mail.ukl.yahoo.com><4D3D7F51.70805@ulb.ac.be> <395644.68863.qm@web28514.mail.ukl.yahoo.com> <43FB511CB5684DA490AAB2D8E8666177@Lounge> <27919.7685.qm@web28506.mail.ukl.yahoo.com> Message-ID: <4D3E44DB.8080802@nhcc.net> On 1/24/2011 7:52 PM, Nigel Guthrie wrote: > For the typical player, the most frequent scenario is that he probably did agree > a meaning Then that meaning is what the opponents are entitled to. > but he is unsure what it is. If a player does not fulfill his legal duties, his opponents are protected from any resulting damage. The reason for not fulfilling those duties is irrelevant. > According to some law-makers and BLML directors, when a player is genuinely > unsure, he must not guess. I think Nigel is confusing two different situations. What most people have been writing about is when you have no agreement or have contradictory or ambiguous agreements. In such cases, you tell the opponents the agreements you do have, and everyone guesses what the player actually has. Nigel's "most frequent scenario" (not all that frequent in my experience), where a player knows he has forgotten, is a different matter entirely. > What then, should a typical player, who is 75% sure > say to opponents? Probably best just to state the agreement you think is correct. If you get it right, no problem. If not, partner has UI, and opponents have MI, and you're probably making the wrong call anyway. Good luck! But expressing uncertainty changes nothing except to give partner more UI and opponents unnecessary AI. Alternatives are to refer opponents to your system card or call the TD and let partner explain with you out of earshot, but those have disadvantages of their own. (Referring them to the SC is probably OK if it's a very simple question such as which version of Blackwood you play.) Why is this so complicated? From grabiner at alumni.princeton.edu Tue Jan 25 05:12:34 2011 From: grabiner at alumni.princeton.edu (David Grabiner) Date: Mon, 24 Jan 2011 23:12:34 -0500 Subject: [BLML] Naive question about double shots In-Reply-To: References: <07E347798694429AB3338FF7BBA87E37@erdos> Message-ID: "Jerry Fusselman" writes: >On Mon, Jan 24, 2011 at 6:47 PM, David Grabiner wrote: >> However, we don't need to allege the double shot to deny adjustment. It >> suffices that the TD could rule that N-S were damaged by North's mistake >> rather >> than by West's infraction. Suppose that North held Jxx Kxx xxx Qxxx but had >> the >> CQ in with his spades. With the hand he thought he had, 3S would be a >> reasonable bid. But the TD would still rule that N-S were damaged by North's >> bad bidding; West's infraction created a situation in which normal play would >> give N-S -130 rather than -200, and then North made a serious error and >> should >> bear its consequences. >Ah, it sounds like you disagree with Eric and would be willing (in >some cases) to call 3S a serious error. Interesting. How would you >find out if this was the case? By using bridge judgement, just as we have to do in determining which results are likely. Competing to 3S, both vulnerable at IMPs, after raising to 2S on a minimum flat hand with only three trumps, is a serious bridge error (for a good player; I wouldn't rule against a novice) whether it is a deliberate double shot or not. The mis-sorted hand is an example of one reason that a good player might make this error. More often, these serious errors occur during the play. If the NOS is playing 4S rather than 3NT because of MI, and 3NT would have made three, then the NOS will not get an adjustment if 4S could have made four but declarer revoked, or forgot to draw a trump, or blocked a suit. From PeterEidt at t-online.de Tue Jan 25 06:44:34 2011 From: PeterEidt at t-online.de (Peter Eidt) Date: Tue, 25 Jan 2011 06:44:34 +0100 Subject: [BLML] =?utf-8?q?Exam_question?= In-Reply-To: <4D3E0908.5090009@skynet.be> References: <4D3E0908.5090009@skynet.be> Message-ID: <1Phbhi-0YKMXg0@fwd05.aul.t-online.de> From: Herman De Wael > Peter Eidt wrote: > > From: Herman De Wael > >> This one may well make it into an exam question for a TD course, if > >> you ever need one: > >> > >> On Saturday, I was playing 2He. I had just won the first six > tricks, >> and I showed AK of hearts, saying "I think I am going to > make this >> contract". Did I claim or am I allowed to play on? > > > > > Law 68: > > "For a statement or action to constitute a claim or concession of > > tricks under these Laws, it must refer to tricks other than one > > currently in progress. If it does refer to subsequent tricks: > > > > > > A. Claim Defined > > Any statement to the effect that a contestant will win a specific > > number of tricks is a claim of those tricks. A contestant also > > claims when he suggests that play be curtailed, or when he shows his > > cards (unless he demonstrably did not intend to claim - for example, > > if declarer faces his cards after an opening lead out of turn Law > > 54, not this Law, will apply)." > > > > Is no-one noticing the word "specific" number here? I did not say I > was going to make 8 tricks - I was clearly saying 8 or more. > > > > [Peter] > > > > Yes, you did claim and no, you are not allowed to play on. > > > > Then what is the word specific doing in that law-text? [Peter] It is part of the law ;-) The law does not say "to announce a specific number of tricks", it says "any statement to the effect that a contestant will win a specific number of tricks .." In my world a situation where declarer has won 6 tricks _and_ shows 2 more winners _and_ tells the table that he will _make_ his contract on the 2-level this does constitute a statement to the effect that declarer will win 8 tricks in hearts. Also, in my world a statement of the sort "I will not lose my contract" might show 8+ tricks whereas "I will make my contract" shows exactly 8 tricks. So, I doubt the word "clearly" in Hermans penultimate sentence. From jfusselman at gmail.com Tue Jan 25 08:20:17 2011 From: jfusselman at gmail.com (Jerry Fusselman) Date: Tue, 25 Jan 2011 01:20:17 -0600 Subject: [BLML] Naive question about double shots In-Reply-To: References: <07E347798694429AB3338FF7BBA87E37@erdos> Message-ID: On Mon, Jan 24, 2011 at 10:12 PM, David Grabiner wrote: > > By using bridge judgement, just as we have to do in determining which results > are likely. ?Competing to 3S, both vulnerable at IMPs, after raising to 2S on a > minimum flat hand with only three trumps, is a serious bridge error (for a good > player; I wouldn't rule against a novice) whether it is a deliberate double shot > or not. ?The mis-sorted hand is an example of one reason that a good player > might make this error. > > More often, these serious errors occur during the play. ?If the NOS is playing > 4S rather than 3NT because of MI, and 3NT would have made three, then the NOS > will not get an adjustment if 4S could have made four but declarer revoked, or > forgot to draw a trump, or blocked a suit. > Thanks to David for clarifying, but if 3S is a serious error, then whether or not 3S is a double shot seems moot. (I.e., the part about "wild or gambling action" is not needed because it has no effect on the final ruling.) I guess I should have asked for an example where the action taken is not a serious error but is a double shot. Ideally, clearly not a serious error, but a clear example of a double shot. Now I see that that's what I should have asked for. Sorry I did not ask the best question the first time. Also, it just does not feel like a double shot to me. Yes, rather bad in most cases, but double shot, I just don't see it yet. If I really wanted to play in 3SX like David initially suggested, a *much* better way seems to me to hope for a penalty pass of 2SX, which always scores better than 3SX, and if not lucky there, I would next try to appear to have committed a foolish late raise, for the slower reraise to 3S is considerably more likely to be doubled. Small world story: About 15 years ago in Chicago, my partner, one Mark K, very talented, who had recently played a lot with David when they both lived in Michigan (I think---perhaps Ann Arbor), managed to push me and lead our partnership to the NAOP championships (0-300) in Houston, where a similar sequence occurred. Our 1H-2H had been penalty doubled (which I should have realized, for we were overcalling). Well I took out 2HX with a bold 3H, and though I had a better shape than 4333, it would have gone down three. But, somehow, the opponents believed I had my bid. The guy that doubled the first time had such a perplexed look on his face when considering whether to double the second time that I had to laugh, but he tried 4D or something, for a bad result. Sometimes that stupid extra bid works. :) Jerry Fusselman From jfusselman at gmail.com Tue Jan 25 08:41:42 2011 From: jfusselman at gmail.com (Jerry Fusselman) Date: Tue, 25 Jan 2011 01:41:42 -0600 Subject: [BLML] Naive question about double shots In-Reply-To: References: <07E347798694429AB3338FF7BBA87E37@erdos> Message-ID: Oops, it was nine years ago that Mark and I played in Houston, not 15. From Hermandw at skynet.be Tue Jan 25 08:45:26 2011 From: Hermandw at skynet.be (Herman De Wael) Date: Tue, 25 Jan 2011 08:45:26 +0100 Subject: [BLML] Exam question In-Reply-To: <1Phbhi-0YKMXg0@fwd05.aul.t-online.de> References: <4D3E0908.5090009@skynet.be> <1Phbhi-0YKMXg0@fwd05.aul.t-online.de> Message-ID: <4D3E7F96.3020101@skynet.be> Peter Eidt wrote: > From: Herman De Wael >> Peter Eidt wrote: >>> From: Herman De Wael >>>> This one may well make it into an exam question for a TD course, if >>>> you ever need one: >>>> >>>> On Saturday, I was playing 2He. I had just won the first six >> tricks,>> and I showed AK of hearts, saying "I think I am going to >> make this>> contract". Did I claim or am I allowed to play on? >>> >> >>> Law 68: >>> "For a statement or action to constitute a claim or concession of >>> tricks under these Laws, it must refer to tricks other than one >>> currently in progress. If it does refer to subsequent tricks: >>> >>> >>> A. Claim Defined >>> Any statement to the effect that a contestant will win a specific >>> number of tricks is a claim of those tricks. A contestant also >>> claims when he suggests that play be curtailed, or when he shows his >>> cards (unless he demonstrably did not intend to claim - for example, >>> if declarer faces his cards after an opening lead out of turn Law >>> 54, not this Law, will apply)." >>> >> >> Is no-one noticing the word "specific" number here? I did not say I >> was going to make 8 tricks - I was clearly saying 8 or more. >> >> >>> [Peter] >>> >>> Yes, you did claim and no, you are not allowed to play on. >>> >> >> Then what is the word specific doing in that law-text? > > [Peter] > > It is part of the law ;-) > > The law does not say "to announce a specific number of tricks", > it says "any statement to the effect that a contestant will win a > specific number of tricks .." > > In my world a situation where declarer has won 6 tricks _and_ > shows 2 more winners _and_ tells the table that he will _make_ > his contract on the 2-level this does constitute a statement to > the effect that declarer will win 8 tricks in hearts. > > Also, in my world a statement of the sort "I will not lose my > contract" might show 8+ tricks whereas "I will make my contract" > shows exactly 8 tricks. So, I doubt the word "clearly" in Hermans > penultimate sentence. > I did not realize you thought there was a difference between "I will make" and "I won't go down". Frankly, I don't remember what I said precisely (in Antwerp dialect to boot), but my opponents clearly understood I was talking of 8+ tricks. And I thought that was clear from my question too. -- Herman De Wael Wilrijk Antwerpen Belgium From Hermandw at skynet.be Tue Jan 25 08:50:51 2011 From: Hermandw at skynet.be (Herman De Wael) Date: Tue, 25 Jan 2011 08:50:51 +0100 Subject: [BLML] Unsure In-Reply-To: <4D3D8C91.5020007@ulb.ac.be> References: <586818.75337.qm@web28510.mail.ukl.yahoo.com> <4D3704A8.8090006@skynet.be> <128171.88697.qm@web25402.mail.ukl. yahoo.com> < 4D382CB9.2060700@ulb.ac.be><6410CABD-9AF3-45A7-9B10-F5EDC78C6095@starpowe r. net> <4D3852CE.6080908@ulb.ac.be> <4D386625.90105@meteo.fr> <4D3869DD.9030708@ulb.ac.be> <4D3873A6.5070104@meteo.fr> <107026.33279.qm@web28515.mail.ukl.yahoo.com> <4D39BC9E.9040108@nhcc.net> <1820574630.137446.1295675648497.JavaMail.ngmail@webmail11.arcor-online.net> <4D3A93A3.8020906@skynet.be> <000901cbba0f$ce55b3f0$6b011bd0$@no> <4D3D72AF.3060606@ulb.ac.be> <000001cbbbce$a0c010f0$e24032d0$@no> <4D3D8C91.5020007@ulb.ac.be> Message-ID: <4D3E80DB.8030902@skynet.be> Alain Gottcheiner wrote: > > So, you're basically saying that every partnership has at least an > implicit understanding of any bid they could legally make. > What does 1NT-2D-5S mean in your system ? > I would be happy with responding "undiscussed" (or "impossible", which > also is non-descriptive) > Well, if that call shows diamonds, it's rather strange, no? And if you say undiscussed, and then bid 7D, and partner happens to have 7 of those, then there is something fishy going on, no? It's easy to come up with calls that are undiscussed, but do they happen at a table? And when they happen, do they result in disasters? On Saturday, the bidding went: me LHO part RHO 1Cl pass 1Di (transfer to hearts) pass 1He 2NT We had never seen that auction before, and we had never discussed it. Yet I explained it as diamonds and clubs, and that was exactly what partner had. If I had merely said "undiscussed", I would have spoken the truth, but how would a TD rule? How can I prove that it's really "undiscussed" when I apparently have a perfect idea of what partner had? -- Herman De Wael Wilrijk Antwerpen Belgium From blml at arcor.de Tue Jan 25 08:53:11 2011 From: blml at arcor.de (Thomas Dehn) Date: Tue, 25 Jan 2011 08:53:11 +0100 (CET) Subject: [BLML] Naive question about double shots In-Reply-To: References: <07E347798694429AB3338FF7BBA87E37@erdos> Message-ID: <1456683085.35571.1295941991732.JavaMail.ngmail@webmail18.arcor-online.net> Jerry Fusselman wrote: > Thanks to David for clarifying, but if 3S is a serious error, then > whether or not 3S is a double shot seems moot. (I.e., the part about > "wild or gambling action" is not needed because it has no effect on > the final ruling.) I guess I should have asked for an example where > the action taken is not a serious error but is a double shot. > Ideally, clearly not a serious error, but a clear example of a double > shot. Now I see that that's what I should have asked for. Sorry I > did not ask the best question the first time. After a hesitation sequence, opponents double Joe's aggressively bid 3NT contract. Joe judges that the director will remove the double, if needed. During the play, the key suit is diamonds, where Joe holds Ax opposite KQTxxx Joe decides to finesse the DT. If that works, he'll get an excellent score, 3NT doubled made with overtricks. If it fails, he'll claim that without the illegal double, he would have played diamonds normally. Thomas From Hermandw at skynet.be Tue Jan 25 08:55:08 2011 From: Hermandw at skynet.be (Herman De Wael) Date: Tue, 25 Jan 2011 08:55:08 +0100 Subject: [BLML] Unsure In-Reply-To: <4D3DA236.8090506@ulb.ac.be> References: <586818.75337.qm@web28510.mail.ukl.yahoo.com> <4D3704A8.8090006@skynet.be> <128171.88697.qm@web25402.mail.ukl. yahoo.com> < 4D382CB9.2060700@ulb.ac.be><6410CABD-9AF3-45A7-9B10-F5EDC78C6095@starpowe r. net> <4D3852CE.6080908@ulb.ac.be> <4D386625.90105@meteo.fr> <4D3869DD.9030708@ulb.ac.be> <4D3873A6.5070104@meteo.fr> <107026.33279.qm@web28515.mail.ukl.yahoo.com> <4D39BC9E.9040108@nhcc.net> <1820574630.137446.1295675648497.JavaMail.ngmail@webmail11.arcor-online.net> <4D3A93A3.8020906@skynet.be> <000901cbba0f$ce55b3f0$6b011bd0$@no> <4D3D72AF.3060606@ulb.ac.be> <000001cbbbce$a0c010f0$e24032d0$@no> <4D3D8C91.5020007@ulb.ac.be> <000001cbbbda$3b61c940$b2255bc0$@no> <4D3DA236.8090506@ulb.ac.be> Message-ID: <4D3E81DC.4090900@skynet.be> Alain Gottcheiner wrote: > > AG : wonderful ! So, I open 1NT, you answer 2D, and I mistankelny pull > out 3S in lieu of 3H. > > One can argue for ever whether this call has been done for purpose, but > you will be hard pressed to explain it. > Yes Alain, so? You do this behind screens, and the trey is passed before you notice your mistake. Then your screenmate asks "what is 3S?". You are surprised but notice quickly that you bid that by mistake. So you now make up some meaning for that call. And so does your partner, and you both figure out the same meaning. Opponents are correctly informed. Even if you are not able to disguise the fact that you mispulled, your screenmate is entitled to know what your partner will tell his partner. And you may well be able to imagine what your partner will say. Even calls that you made involontary have a meaning, and in well-oiled partnerships that meaning is known to both partners. -- Herman De Wael Wilrijk Antwerpen Belgium From Hermandw at skynet.be Tue Jan 25 09:01:44 2011 From: Hermandw at skynet.be (Herman De Wael) Date: Tue, 25 Jan 2011 09:01:44 +0100 Subject: [BLML] Unsure In-Reply-To: References: <000901cbba0f$ce55b3f0$6b011bd0$@no><586818.75337.qm@web28510.mail.ukl.yahoo.com> <4D3704A8.8090006@skynet.be> <128171.88697.qm@web25402.mail.ukl. yahoo.com> < 4D382CB9.2060700@ulb.ac.be><6410CABD-9AF3-45A7-9B10-F5EDC78C6095@starpowe r. net><4D3852CE.6080908@ulb.ac.be> <4D386625.90105@meteo.fr> <4D3869DD.9030708@ulb.ac.be> <4D3873A6.5070104@meteo.fr> <107026.33279.qm@web28515.mail.ukl.yahoo.com> <4D39BC9E.9040108@nhcc.net> <1820574630.137446.1295675648497.JavaMail.ngmail@webmail11.arcor-online.net><4D3A93A3.8020906@skynet.be><1896834273.57934.1295686896979.JavaMail.ngmail@webmail13.arcor-online.net> <514721598.171618.1295725647449.JavaMail.ngmail@webmail10.arcor-online.net> <000901cbba82$f84fdf50$e8ef9df0$@no> Message-ID: <4D3E8368.40706@skynet.be> Eric Landau wrote: > > For the last forty-plus years, bridge has been getting consistently > more "scientific". Systems and methods have gotten consistently more > complex and detailed. Today's top-level partnerships have extensive > agreements covering the vast majority of situations, often requiring > hundreds of pages of notes to specify. > > But fifty years ago, even the highest-level partnerships got by with > a relative handful of basic agreements covering opening bids, basic > responses and rebids, and a few simple "artifical" conventions (e.g. > Blackwood, Stayman), along with a general understanding of how they > bid (what we would call "meta-agreements"). Beyond that, pretty much > everything else was "no agreement" or "undiscussed". Despite their > lack of agreements, they still managed most of the time to work out a > reasonable auction; this was generally considered to be a strong > indication that they knew a little something about how to play bridge. > You are right, but I think you draw the wrong conclusion. Fifty years ago, when someone said "undiscussed", he assumed that his opponents knew as much as he did, and he was usually right. Today, the word "undiscussed" really means "not discussed specifically, we shall apply our default understandings". In some partnerships, those defaults will NOT be known by the opponents. Then, the partnership will know more than the opponents, and that is MI. > > Eric Landau -- Herman De Wael Wilrijk Antwerpen Belgium From agot at ulb.ac.be Tue Jan 25 09:43:06 2011 From: agot at ulb.ac.be (Alain Gottcheiner) Date: Tue, 25 Jan 2011 09:43:06 +0100 Subject: [BLML] Unsure In-Reply-To: <000801cbbbe6$886147a0$9923d6e0$@no> References: <586818.75337.qm@web28510.mail.ukl.yahoo.com> <128171.88697.qm@web25402.mail.ukl. yahoo.com> < 4D382CB9.2060700@ulb.ac.be><6410CABD-9AF3-45A7-9B10-F5EDC78C6095@starpowe r. net><4D3852CE.6080908@ulb.ac.be> <4D386625.90105@meteo.fr> <4D3869DD.9030708@ulb.ac.be> <4D3873A6.5070104@meteo.fr> <107026.33279.qm@web28515.mail.ukl.yahoo.com> <4D39BC9E.9040108@nhcc.net> <1820574630.137446.1295675648497.JavaMail.ngmail@webmail11.arcor-online.net> <4D3D7134.4000803@ulb.ac.be> <431431.44115.qm@web28515.mail.ukl.yahoo.com> <4D3D7F51.70805@ulb.ac.be> <395644.68863.qm@web28514.mail.ukl.yahoo.com><000101cbbbcf$4f8dd270$eea97750$@no> <011301cbbbcf$fc8610a0$f59231e0$@nl><4A3418FD24684B579A369D685BF6D6FB@Lounge> <011401cbbbd6$4e621f80$eb265e80$@nl> <000101cbbbdb$0a0717a0$1e1546e0$@no> <670306DF0C5446E8B8770FE884FA6E1C@Lounge> <000801cbbbe6$886147a0$9923d6e0$@no> Message-ID: <4D3E8D1A.5000201@ulb.ac.be> Le 24/01/2011 17:48, Sven Pran a ?crit : > On Behalf Of David > ............... >> *If the partnership can show me some positive evidence (System > documentation, >> *CC or whatever) that the meaning of the call can be either A or B I shall > *accept >> that as the explanation. Otherwise, if the actual hand matches either *A > or B but >> not both I shall rule misinformation and eventually judge whether > *opponents have >> been damaged. Take a look at Law 75! >> >> Yes indeed let us look at L75. >> >> L75 starts >> After a misleading explanation........ >> >> The opposition are told that as a consequence of other agreements 2N is > either >> natural or lebensohl but as the sequence has neither occurred before nor > is the >> subject of an agreement it is impossible to say which. >> >> Now in my opinion (assuming that is a true statement) there is nothing > misleading. >> L75B starts >> The actual partnership agreement is....... >> >> Well there is no actual partnership agreement and that is what the > opposition >> have been told. >> >> So what is the problem here? >> >> L75 is the subject of Mistaken Explanations and Mistaken calls. >> Calls which are not covered by systemic agreements and are explained as > such >> are neither mistakenly explained nor mistakenly bid. > So you missed the possibly most important sentence in Law 75? > > the Director is to presume Mistaken Explanation, rather than Mistaken Call, > in the absence of evidence to the contrary AG :and that's precisely where my example comes in. There might well be evidence to the contrary, other than the convention card and the player's hand, e.g. the bid is unknown to most players. From jean-pierre.rocafort at meteo.fr Tue Jan 25 10:07:11 2011 From: jean-pierre.rocafort at meteo.fr (Jean-Pierre Rocafort) Date: Tue, 25 Jan 2011 10:07:11 +0100 Subject: [BLML] Exam question In-Reply-To: <4D3E7F96.3020101@skynet.be> References: <4D3E0908.5090009@skynet.be> <1Phbhi-0YKMXg0@fwd05.aul.t-online.de> <4D3E7F96.3020101@skynet.be> Message-ID: <4D3E92BF.1050708@meteo.fr> Herman De Wael a ?crit : > Peter Eidt wrote: >> From: Herman De Wael >>> Peter Eidt wrote: >>>> From: Herman De Wael >>>>> This one may well make it into an exam question for a TD course, if >>>>> you ever need one: >>>>> >>>>> On Saturday, I was playing 2He. I had just won the first six >>> tricks,>> and I showed AK of hearts, saying "I think I am going to >>> make this>> contract". Did I claim or am I allowed to play on? >>>> Law 68: >>>> "For a statement or action to constitute a claim or concession of >>>> tricks under these Laws, it must refer to tricks other than one >>>> currently in progress. If it does refer to subsequent tricks: >>>> >>>> >>>> A. Claim Defined >>>> Any statement to the effect that a contestant will win a specific >>>> number of tricks is a claim of those tricks. A contestant also >>>> claims when he suggests that play be curtailed, or when he shows his >>>> cards (unless he demonstrably did not intend to claim - for example, >>>> if declarer faces his cards after an opening lead out of turn Law >>>> 54, not this Law, will apply)." >>>> >>> Is no-one noticing the word "specific" number here? I did not say I >>> was going to make 8 tricks - I was clearly saying 8 or more. >>> >>> >>>> [Peter] >>>> >>>> Yes, you did claim and no, you are not allowed to play on. >>>> >>> Then what is the word specific doing in that law-text? >> [Peter] >> >> It is part of the law ;-) >> >> The law does not say "to announce a specific number of tricks", >> it says "any statement to the effect that a contestant will win a >> specific number of tricks .." >> >> In my world a situation where declarer has won 6 tricks _and_ >> shows 2 more winners _and_ tells the table that he will _make_ >> his contract on the 2-level this does constitute a statement to >> the effect that declarer will win 8 tricks in hearts. >> >> Also, in my world a statement of the sort "I will not lose my >> contract" might show 8+ tricks whereas "I will make my contract" >> shows exactly 8 tricks. So, I doubt the word "clearly" in Hermans >> penultimate sentence. >> > > I did not realize you thought there was a difference between "I will > make" and "I won't go down". Frankly, I don't remember what I said > precisely (in Antwerp dialect to boot), but my opponents clearly > understood I was talking of 8+ tricks. And I thought that was clear from > my question too. > something i fail to understand but you are surely going to make clear: why did you say what you said if it was not in order to claim? jpr -- _______________________________________________ Jean-Pierre Rocafort METEO-FRANCE DSI/CM 42 Avenue Gaspard Coriolis 31057 Toulouse CEDEX Tph: 05 61 07 81 02 (33 5 61 07 81 02) Fax: 05 61 07 81 09 (33 5 61 07 81 09) e-mail: jean-pierre.rocafort at meteo.fr Serveur WWW METEO-France: http://www.meteo.fr _______________________________________________ From svenpran at online.no Tue Jan 25 10:08:09 2011 From: svenpran at online.no (Sven Pran) Date: Tue, 25 Jan 2011 10:08:09 +0100 Subject: [BLML] Exam question In-Reply-To: <4D3E0908.5090009@skynet.be> References: <4D3D3B2F.5090107@skynet.be> <1PhIDl-0dAMts0@fwd00.aul.t-online.de> <4D3E0908.5090009@skynet.be> Message-ID: <000201cbbc6f$6360ef80$2a22ce80$@no> On Behalf Of Herman De Wael ............. > >> On Saturday, I was playing 2He. I had just won the first six tricks, > >> and I showed AK of hearts, saying "I think I am going to make this > >> contract". Did I claim or am I allowed to play on? > > > > Law 68: > > "For a statement or action to constitute a claim or concession of > > tricks under these Laws, it must refer to tricks other than one > > currently in progress. If it does refer to subsequent tricks: > > > > A. Claim Defined > > Any statement to the effect that a contestant will win a specific > > number of tricks is a claim of those tricks. A contestant also claims > > when he suggests that play be curtailed, or when he shows his cards > > (unless he demonstrably did not intend to claim - for example, if > > declarer faces his cards after an opening lead out of turn Law 54, not this Law, > will apply)." > > > > Is no-one noticing the word "specific" number here? I did not say I was going to > make 8 tricks - I was clearly saying 8 or more. In my language you claimed that you will make your contract, a statement that is synonymous with claiming a total of (exactly) 8 tricks. You mentioned nothing about the further tricks so effectively you conceded those with your claim. From svenpran at online.no Tue Jan 25 10:21:52 2011 From: svenpran at online.no (Sven Pran) Date: Tue, 25 Jan 2011 10:21:52 +0100 Subject: [BLML] Unsure In-Reply-To: <7558B97F13394D06B08E66CFA36EDC8F@Lounge> References: e0$@no> <61F91A343F83 423DBF87BC39B487C84B@Lounge> <000601cbbc14$05898ee0$109caca0$@no> <7558B97F13394D06B08E66CFA36EDC8F@Lounge> Message-ID: <000301cbbc71$4dcb3340$e96199c0$@no> On Behalf Of David > From: Sven Pran > Sent: Monday, January 24, 2011 10:14 PM > To: 'Bridge Laws Mailing List' > Subject: Re: [BLML] Unsure > > On Behalf Of David > ............... > > *So you missed the possibly most important sentence in Law 75? > > * > > *the Director is to presume Mistaken Explanation, rather than Mistaken > Call, *in > > the absence of evidence to the contrary > > > > No I did not miss it. > > You have the testimony of *BOTH* players that this is not covered and > > as > you > > would expect there is no reference to it on the system card. > > It might not be evidence that you would like to put a lot of weight > > on, > but you know > > that is all you can expect in this situation. > > You may not even have the evidence of how the player would have taken > > the > bid if > > the player next in turn calls (say 3S in the example case). > > > > > > So no there is not an *absence* of evidence to the contrary. > > > > So, please tell me, with law references, what you believe is the > obligation of a > > player who KNOWS that the bid is NOT covered by his systemic agreements? > > *Why did the player in question make the call if it has no assigned meaning? > *How did he expect (or hope) that his partner would understand the call? > > The auction under consideration was > > (1S) 1N (2S) 2N > > The person bidding 2N wanted to bid a non forcing 3D bid as he judged passing > 2S would give a poor mp score. > > However our system had not defined whether 3D would be forcing nor had it > defined whether 2N would be natural invitational or lebensohl. But it would be > taken as one or the other. Not what I would understand as having NO assigned > meaning. So with a damned if you do and damned if you don't choice he judged to > bid 2N and hope it worked out. > > So both members of the partnership were aware that the meaning of the bid was > ambiguous and this was the information that was given to the opposition. (along > with the meanings of other relevant sequences of course). > > So the question remains. What information do you think should be given and why? Exactly the information you have provided above in the paragraph beginning with "However our system..." and ending with "hope it worked out". Why? Because that is the information they are entitled to receive as the meaning of the call. From svenpran at online.no Tue Jan 25 10:27:28 2011 From: svenpran at online.no (Sven Pran) Date: Tue, 25 Jan 2011 10:27:28 +0100 Subject: [BLML] Unsure In-Reply-To: <27919.7685.qm@web28506.mail.ukl.yahoo.com> References: <586818.75337.qm@web28510.mail.ukl.yahoo.com><4D3704A8.8090006@skynet.be><128171.88697.qm@web25402.mail.ukl. yahoo.com> <4D382CB9.2060700@ulb.ac.be><6410CABD-9AF3-45A7-9B10-F5EDC78C6095@starpower. net> <4D3852CE.6080908@ulb.ac.be> <4D386625.90105@meteo.fr><4D3869DD.9030708@ulb.ac.be> <4D3873A6.5070104@meteo.fr><107026.33279.qm@web28515.mail.ukl.yahoo.com><4D39BC9E.9040108@nhcc.net><1820574630.137446.1295675648497.JavaMail.ngmail@webmail11.arcor-online.net><4D3D7134.4000803@ulb.ac.be><431431.44115.qm@web28515.mail.ukl.yahoo.com><4D3D7F51.70805@ulb.ac.be> <395644.68863.qm@web28514.mail.ukl.yahoo.com> <43FB511CB5684DA490AAB2D8E8666177@Lounge> <27919.7685.qm@web28506.mail.ukl.yahoo.com> Message-ID: <000401cbbc72$15eedb60$41cc9220$@no> On Behalf Of Nigel Guthrie ............. > IMO the law should mandate that he guess, when unsure. The law mandates that opponents are told the truth. Any fact that the player is unsure or guesses is according to the law irrelevant; if correct explanation is given then fine, if not then there is a case of mistaken explanation. Period. From jean-pierre.rocafort at meteo.fr Tue Jan 25 10:30:03 2011 From: jean-pierre.rocafort at meteo.fr (Jean-Pierre Rocafort) Date: Tue, 25 Jan 2011 10:30:03 +0100 Subject: [BLML] Unsure In-Reply-To: <000801cbbc16$e2466e00$a6d34a00$@no> References: <586818.75337.qm@web28510.mail.ukl.yahoo.com> <4D3704A8.8090006@ skynet.be> <128171.88697.qm@web25402.mail.ukl. yahoo.com> < 4D382CB9.2060700@ulb.ac.be> <6410CABD-9AF3-45A7-9B10-F5EDC78C6095@starpowe r. net> <4D3852CE.6080908@ul b.ac.be> <4D386625.90105@meteo.fr> <4D3869DD.9030708@ulb.ac.be> <4D3873A6.5 070104@meteo.fr> <107026.33279.qm@web28515.mail.ukl.yahoo.com> <4D39BC9E.90 40108@nhcc.net> <1820574630.137446.1295675648497.JavaMail.ngmail@webmail11. arcor-online.net> <4D3A93A3.8020906@skynet.be> <000901cbba0f$ce55b3f0$6b011 bd0$@no><4D3D72AF.3060606@ulb.ac.be> <000001cbbbce$a0c010f0$e24032d0$@no><7 8BC7D50-DDDC-4459-BA0B-87467613E5B0@starpower.net> <000801cbbc16$e2466e00$a6d34a00$@no> Message-ID: <4D3E981B.1050009@meteo.fr> Sven Pran a ?crit : > On Behalf Of Eric Landau > .................. >>> Any player is entitled to a full explanation of opponents' auction >>> whether the relevant partnership understanding is explicit or >>> implicit. >> True. >> >>> "Not discussed", "I have no idea" and similar statements are not such >>> explanations, they are simply statements to the effect that the player >>> does not know what the correct explanation is. According to law this >>> is synonymous with misexplanation, i.e. failure to give correct >>> information. >> But when you presume lack of explanation to be synonymous with > misexplanation, >> you are mandating that any player is entitled to a full explanation of > opponents' >> auction whether the relevant partnership understanding is explicit, > implicit, or non- >> existent. > > I shall expect the player to provide a very good reason for using a call for > which (in his own opinion) no understanding exists. I don't believe he can. you don't believe but these things happen. you can call it improvisation and some players are very fond of it. and why should they have to justify themselves unless there is a specific regulation which forbids unexpected bids which are not based on any agreement? some players like to test their partner's reactions to innovation. on some circonstances they hope that partner will be able to understand from his own cards. on other times they think it will be harmless even if partner doesn't understand, or the preemptive effect will be enough to take the risk of a misunderstanding, or they will be able to clarify the situation later on... there are many situations where a player may think it would be profitable to make a call for which no understanding exists. once i had to adjudicate this case on a AC: - 1D 4D when asked (there were screens) both players said: no agreement, no prior occurence. they explained what agreements they had for possible alternative calls: double (take-out), 2D (5H-5S), 3D (asking for D stop). the AC was convinced by their arguments. would you not have been? jpr > > -- _______________________________________________ Jean-Pierre Rocafort METEO-FRANCE DSI/CM 42 Avenue Gaspard Coriolis 31057 Toulouse CEDEX Tph: 05 61 07 81 02 (33 5 61 07 81 02) Fax: 05 61 07 81 09 (33 5 61 07 81 09) e-mail: jean-pierre.rocafort at meteo.fr Serveur WWW METEO-France: http://www.meteo.fr _______________________________________________ From svenpran at online.no Tue Jan 25 10:31:20 2011 From: svenpran at online.no (Sven Pran) Date: Tue, 25 Jan 2011 10:31:20 +0100 Subject: [BLML] Unsure In-Reply-To: <4D3E80DB.8030902@skynet.be> References: <586818.75337.qm@web28510.mail.ukl.yahoo.com> <4D3704A8.8090006@skynet.be> <128171.88697.qm@web25402.mail.ukl. yahoo.com> < 4D382CB9.2060700@ulb.ac.be><6410CABD-9AF3-45A7-9B10-F5EDC78C6095@starpowe r. net> <4D3852CE.6080908@ulb.ac.be> <4D386625.90105@meteo.fr> <4D3869DD.9030708@ulb.ac.be> <4D3873A6.5070104@meteo.fr> <107026.33279.qm@web28515.mail.ukl.yahoo.com> <4D39BC9E.9040108@nhcc.net> <1820574630.137446.1295675648497.JavaMail.ngmail@webmail11.arcor-online.net> <4D3A93A3.8020906@skynet.be> <000901cbba0f$ce55b3f0$6b011bd0$@no> <4D3D72AF.3060606@ulb.ac.be> <000001cbbbce$a0c010f0$e24032d0$@no> <4D3D8C91.5020007@ulb.ac.be> <4D3E80DB.8030902@skynet.be> Message-ID: <000501cbbc72$a00c37c0$e024a740$@no> On Behalf Of Herman De Wael ............. > On Saturday, the bidding went: > > me LHO part RHO > 1Cl pass 1Di (transfer to hearts) > pass 1He 2NT > > We had never seen that auction before, and we had never discussed it. > Yet I explained it as diamonds and clubs, and that was exactly what partner had. > If I had merely said "undiscussed", I would have spoken the truth, but how would a > TD rule? How can I prove that it's really "undiscussed" when I apparently have a > perfect idea of what partner had? Very good example! And the answer to your (last) question is simply: You can't! From Hermandw at skynet.be Tue Jan 25 10:41:33 2011 From: Hermandw at skynet.be (Herman De Wael) Date: Tue, 25 Jan 2011 10:41:33 +0100 Subject: [BLML] Exam question In-Reply-To: <4D3E92BF.1050708@meteo.fr> References: <4D3E0908.5090009@skynet.be> <1Phbhi-0YKMXg0@fwd05.aul.t-online.de> <4D3E7F96.3020101@skynet.be> <4D3E92BF.1050708@meteo.fr> Message-ID: <4D3E9ACD.5020602@skynet.be> Jean-Pierre Rocafort wrote: > > something i fail to understand but you are surely going to make clear: > why did you say what you said if it was not in order to claim? > jpr > Do I need a reason? Am I not allowed to tell my opponents something they are allowed to use? I tell them they don't need to worry about playing me down. We shall now fight for overtricks. If they don't want to fatigue themselves, they don't have to. The claim laws are there to forbid players lulling their opponents into sleep, by telling them not to think when thinking might still do them good. I do no such thing. I tell them they don't need to worry about defeating the contract, which is true - I even showed the cards. I said nothing about the number of overtricks I intend to make, and allow them full thinking on that. If you think I do something wrong here, then what is the difference between this one and a player saying "ah, nice", when defenders follow to two tricks in trumps. Does he not show his nine-cards and so-many tricks in just the same way as I did? Did that person claim? -- Herman De Wael Wilrijk Antwerpen Belgium From Hermandw at skynet.be Tue Jan 25 10:42:47 2011 From: Hermandw at skynet.be (Herman De Wael) Date: Tue, 25 Jan 2011 10:42:47 +0100 Subject: [BLML] Exam question In-Reply-To: <000201cbbc6f$6360ef80$2a22ce80$@no> References: <4D3D3B2F.5090107@skynet.be> <1PhIDl-0dAMts0@fwd00.aul.t-online.de> <4D3E0908.5090009@skynet.be> <000201cbbc6f$6360ef80$2a22ce80$@no> Message-ID: <4D3E9B17.7050704@skynet.be> Sven Pran wrote: > > In my language you claimed that you will make your contract, a statement > that is synonymous with claiming a total of (exactly) 8 tricks. > You mentioned nothing about the further tricks so effectively you conceded > those with your claim. > Well, I was not speaking Norwegian. In my language, I told them I would make 8 or more tricks. Please rule on that one and not on the case you imagine. -- Herman De Wael Wilrijk Antwerpen Belgium From agot at ulb.ac.be Tue Jan 25 10:44:40 2011 From: agot at ulb.ac.be (Alain Gottcheiner) Date: Tue, 25 Jan 2011 10:44:40 +0100 Subject: [BLML] Unsure In-Reply-To: <000401cbbc72$15eedb60$41cc9220$@no> References: <586818.75337.qm@web28510.mail.ukl.yahoo.com><4D3704A8.8090006@skynet.be><128171.88697.qm@web25402.mail.ukl. yahoo.com> <4D382CB9.2060700@ulb.ac.be><6410CABD-9AF3-45A7-9B10-F5EDC78C6095@starpower. net> <4D3852CE.6080908@ulb.ac.be> <4D386625.90105@meteo.fr><4D3869DD.9030708@ulb.ac.be> <4D3873A6.5070104@meteo.fr><107026.33279.qm@web28515.mail.ukl.yahoo.com><4D39BC9E.9040108@nhcc.net><1820574630.137446.1295675648497.JavaMail.ngmail@webmail11.arcor-online.net><4D3D7134.4000803@ulb.ac.be><431431.44115.qm@web28515.mail.ukl.yahoo.com><4D3D7F51.70805@ulb.ac.be> <395644.68863.qm@web28514.mail.ukl.yahoo.com> <43FB511CB5684DA490AAB2D8E8666177@Lounge> <27919.7685.qm@web28506.mail.ukl.yahoo.com> <000401cbbc72$15eedb60$41cc9220$@no> Message-ID: <4D3E9B88.7020704@ulb.ac.be> Le 25/01/2011 10:27, Sven Pran a ?crit : > On Behalf Of Nigel Guthrie > ............. >> IMO the law should mandate that he guess, when unsure. > The law mandates that opponents are told the truth. > > Any fact that the player is unsure or guesses is according to the law > irrelevant; if correct explanation is given then fine, if not then there is > a case of mistaken explanation. Period. > If this is the case (and I think it is), then any statement that "I'm unsure" or the like is striclty prohibited, as extraneous communication. From Hermandw at skynet.be Tue Jan 25 10:47:45 2011 From: Hermandw at skynet.be (Herman De Wael) Date: Tue, 25 Jan 2011 10:47:45 +0100 Subject: [BLML] Unsure In-Reply-To: <000401cbbc72$15eedb60$41cc9220$@no> References: <586818.75337.qm@web28510.mail.ukl.yahoo.com><4D3704A8.8090006@skynet.be><128171.88697.qm@web25402.mail.ukl. yahoo.com> <4D382CB9.2060700@ulb.ac.be><6410CABD-9AF3-45A7-9B10-F5EDC78C6095@starpower. net> <4D3852CE.6080908@ulb.ac.be> <4D386625.90105@meteo.fr><4D3869DD.9030708@ulb.ac.be> <4D3873A6.5070104@meteo.fr><107026.33279.qm@web28515.mail.ukl.yahoo.com><4D39BC9E.9040108@nhcc.net><1820574630.137446.1295675648497.JavaMail.ngmail@webmail11.arcor-online.net><4D3D7134.4000803@ulb.ac.be><431431.44115.qm@web28515.mail.ukl.yahoo.com><4D3D7F51.70805@ulb.ac.be> <395644.68863.qm@web28514.mail.ukl.yahoo.com> <43FB511CB5684DA490AAB2D8E8666177@Lounge> <27919.7685.qm@web28506.mail.ukl.yahoo.com> <000401cbbc72$15eedb60$41cc9220$@no> Message-ID: <4D3E9C41.8060701@skynet.be> Sven Pran wrote: > On Behalf Of Nigel Guthrie > ............. >> IMO the law should mandate that he guess, when unsure. > > The law mandates that opponents are told the truth. > See: "are told". The laws do not mandate that the player tells the truth, but that the opponents get to know the truth. Subtle difference, but important nevertheless. > Any fact that the player is unsure or guesses is according to the law > irrelevant; if correct explanation is given then fine, if not then there is > a case of mistaken explanation. Period. > Very true. Now tell me Sven, in what way did you contradict the sentence you were responding to? How can a player, who is unsure, best try and make certain that his opponents are "told the truth". By guessing. So maybe the laws should not mandate that the guess be told, but the players should do so nevertheless, in order to maximise their winnings. That is like saying that the laws do not mandate following suit, but if you want to win a tournament, revoking is not the best way to go about it. -- Herman De Wael Wilrijk Antwerpen Belgium From agot at ulb.ac.be Tue Jan 25 10:48:10 2011 From: agot at ulb.ac.be (Alain Gottcheiner) Date: Tue, 25 Jan 2011 10:48:10 +0100 Subject: [BLML] Unsure In-Reply-To: <4D3E981B.1050009@meteo.fr> References: <586818.75337.qm@web28510.mail.ukl.yahoo.com> <4D3704A8.8090006@ skynet.be> <128171.88697.qm@web25402.mail.ukl. yahoo.com> < 4D382CB9.2060700@ulb.ac.be> <6410CABD-9AF3-45A7-9B10-F5EDC78C6095@starpowe r. net> <4D3852CE.6080908@ul b.ac.be> <4D386625.90105@meteo.fr> <4D3869DD.9030708@ulb.ac.be> <4D3873A6.5 070104@meteo.fr> <107026.33279.qm@web28515.mail.ukl.yahoo.com> <4D39BC9E.90 40108@nhcc.net> <1820574630.137446.1295675648497.JavaMail.ngmail@webmail11. arcor-online.net> <4D3A93A3.8020906@skynet.be> <000901cbba0f$ce55b3f0$6b011 bd0$@no><4D3D72AF.3060606@ulb.ac.be> <000001cbbbce$a0c010f0$e24032d0$@no><7 8BC7D50-DDDC-4459-BA0B-87467613E5B0@starpower.net> <000801cbbc16$e2466e00$a6d34a00$@no> <4D3E981B.1050009@meteo.fr> Message-ID: <4D3E9C5A.3090806@ulb.ac.be> Le 25/01/2011 10:30, Jean-Pierre Rocafort a ?crit : > > there are many situations where a player may think it would be > profitable to make a call for which no understanding exists. > once i had to adjudicate this case on a AC: > - 1D 4D > when asked (there were screens) both players said: no agreement, no > prior occurence. they explained what agreements they had for possible > alternative calls: double (take-out), 2D (5H-5S), 3D (asking for D > stop). the AC was convinced by their arguments. would you not have been? AG : not necessarily. This isn't such a complex situation, and there mught well exist relevant meta-agreements, like : - a direct bid of opp's suit is never natural - if nothing else applies, the bid is natural From agot at ulb.ac.be Tue Jan 25 10:49:28 2011 From: agot at ulb.ac.be (Alain Gottcheiner) Date: Tue, 25 Jan 2011 10:49:28 +0100 Subject: [BLML] Unsure In-Reply-To: <000501cbbc72$a00c37c0$e024a740$@no> References: <586818.75337.qm@web28510.mail.ukl.yahoo.com> <4D3704A8.8090006@skynet.be> <128171.88697.qm@web25402.mail.ukl. yahoo.com> < 4D382CB9.2060700@ulb.ac.be><6410CABD-9AF3-45A7-9B10-F5EDC78C6095@starpowe r. net> <4D3852CE.6080908@ulb.ac.be> <4D386625.90105@meteo.fr> <4D3869DD.9030708@ulb.ac.be> <4D3873A6.5070104@meteo.fr> <107026.33279.qm@web28515.mail.ukl.yahoo.com> <4D39BC9E.9040108@nhcc.net> <1820574630.137446.1295675648497.JavaMail.ngmail@webmail11.arcor-online.net> <4D3A93A3.8020906@skynet.be> <000901cbba0f$ce55b3f0$6b011bd0$@no> <4D3D72AF.3060606@ulb.ac.be> <000001cbbbce$a0c010f0$e24032d0$@no> <4D3D8C91.5020007@ulb.ac.be> <4D3E80DB.8030902@skynet.be> <000501cbbc72$a00c37c0$e024a740$@no> Message-ID: <4D3E9CA8.7040001@ulb.ac.be> Le 25/01/2011 10:31, Sven Pran a ?crit : > On Behalf Of Herman De Wael > ............. >> On Saturday, the bidding went: >> >> me LHO part RHO >> 1Cl pass 1Di (transfer to hearts) >> pass 1He 2NT >> >> We had never seen that auction before, and we had never discussed it. >> Yet I explained it as diamonds and clubs, and that was exactly what > partner had. >> If I had merely said "undiscussed", I would have spoken the truth, but how > would a >> TD rule? How can I prove that it's really "undiscussed" when I apparently > have a >> perfect idea of what partner had? > Very good example! And the answer to your (last) question is simply: You > can't! AG : that's why the response can't be "undiscussed", but rather "no agreement" - only if there is none, of course, which wasn't the case here. This is a very classical case of implicit agreement. From svenpran at online.no Tue Jan 25 10:49:55 2011 From: svenpran at online.no (Sven Pran) Date: Tue, 25 Jan 2011 10:49:55 +0100 Subject: [BLML] Unsure In-Reply-To: <4D3E981B.1050009@meteo.fr> References: <586818.75337.qm@web28510.mail.ukl.yahoo.com> <4D3704A8.8090006@ skynet.be> <128171.88697.qm@web25402.mail.ukl. yahoo.com> < 4D382CB9.2060700@ulb.ac.be> <6410CABD-9AF3-45A7-9B10-F5EDC78C6095@starpowe r. net> <4D3852CE.6080908@ul b.ac.be> <4D386625.90105@meteo.fr> <4D3869DD.9030708@ulb.ac.be> <4D3873A6.5 070104@meteo.fr> <107026.33279.qm@web28515.mail.ukl.yahoo.com> <4D39BC9E.90 40108@nhcc.net> <1820574630.137446.1295675648497.JavaMail.ngmail@webmail11. arcor-online.net> <4D3A93A3.8020906@skynet.be> <000901cbba0f$ce55b3f0$6b011 bd0$@no><4D3D72AF.3060606@ulb.ac.be> <000001cbbbce$a0c010f0$e24032d0$@no><7 8BC7D50-DDDC-4459-BA0B-87467613E5B0@starpower.net> <000801cbbc16$e2466e00$a6d34a00$@no> <4D3E981B.1050009@meteo.fr> Message-ID: <000301cbbc75$385ebf00$a91c3d00$@no> On Behalf Of Jean-Pierre Rocafort .... > > I shall expect the player to provide a very good reason for using a > > call for which (in his own opinion) no understanding exists. I don't believe he > can. > you don't believe but these things happen. you can call it improvisation and some > players are very fond of it. and why should they have to justify themselves unless > there is a specific regulation which forbids unexpected bids which are not based > on any agreement? > some players like to test their partner's reactions to innovation. on some > circonstances they hope that partner will be able to understand from his own > cards. on other times they think it will be harmless even if partner doesn't > understand, or the preemptive effect will be enough to take the risk of a > misunderstanding, or they will be able to clarify the situation later on... > there are many situations where a player may think it would be profitable to make > a call for which no understanding exists. > once i had to adjudicate this case on a AC: > - 1D 4D > when asked (there were screens) both players said: no agreement, no prior > occurence. they explained what agreements they had for possible alternative > calls: double (take-out), 2D (5H-5S), 3D (asking for D stop). the AC was convinced > by their arguments. would you not have been? > jpr NO. First of all I would want to know how the auction continued. I would have asked the bidder (without his partner present) why he made his 4D bid, how he expected (or hoped) his partner to understand it and why he selected his subsequent calls in the auction. Then I would similarly ask his partner (without the bidder present) how he understood the 4D bid and why he selected his following call and subsequent calls (if any) in the auction. From agot at ulb.ac.be Tue Jan 25 10:50:16 2011 From: agot at ulb.ac.be (Alain Gottcheiner) Date: Tue, 25 Jan 2011 10:50:16 +0100 Subject: [BLML] Exam question In-Reply-To: <4D3E9ACD.5020602@skynet.be> References: <4D3E0908.5090009@skynet.be> <1Phbhi-0YKMXg0@fwd05.aul.t-online.de> <4D3E7F96.3020101@skynet.be> <4D3E92BF.1050708@meteo.fr> <4D3E9ACD.5020602@skynet.be> Message-ID: <4D3E9CD8.7050400@ulb.ac.be> Le 25/01/2011 10:41, Herman De Wael a ?crit : > Jean-Pierre Rocafort wrote: > > >> something i fail to understand but you are surely going to make clear: >> why did you say what you said if it was not in order to claim? >> jpr >> > Do I need a reason? > Am I not allowed to tell my opponents something they are allowed to use? > I tell them they don't need to worry about playing me down. We shall now > fight for overtricks. If they don't want to fatigue themselves, they > don't have to. > The claim laws are there to forbid players lulling their opponents into > sleep, by telling them not to think when thinking might still do them > good. I do no such thing. I tell them they don't need to worry about > defeating the contract, which is true - I even showed the cards. I said > nothing about the number of overtricks I intend to make, and allow them > full thinking on that. > > If you think I do something wrong here, then what is the difference > between this one and a player saying "ah, nice", when defenders follow > to two tricks in trumps. Does he not show his nine-cards and so-many > tricks in just the same way as I did? Did that person claim? > > Did that person show one's cards ? From svenpran at online.no Tue Jan 25 10:58:34 2011 From: svenpran at online.no (Sven Pran) Date: Tue, 25 Jan 2011 10:58:34 +0100 Subject: [BLML] Exam question In-Reply-To: <4D3E9B17.7050704@skynet.be> References: <4D3D3B2F.5090107@skynet.be> <1PhIDl-0dAMts0@fwd00.aul.t-online.de> <4D3E0908.5090009@skynet.be> <000201cbbc6f$6360ef80$2a22ce80$@no> <4D3E9B17.7050704@skynet.be> Message-ID: <000401cbbc76$6e66ba20$4b342e60$@no> On Behalf Of Herman De Wael > Sven Pran wrote: > > > > In my language you claimed that you will make your contract, a > > statement that is synonymous with claiming a total of (exactly) 8 tricks. > > You mentioned nothing about the further tricks so effectively you > > conceded those with your claim. > > > > Well, I was not speaking Norwegian. > In my language, I told them I would make 8 or more tricks. > Please rule on that one and not on the case you imagine. Well, I have only a marginal knowledge of Flemish or Dutch, I can hardly read it. If anybody in your apparent position exposes his two remaining high trumps and just says "I shall make my contract" or "I have my contract" (in English, or the equivalent in Norwegian) and nothing more then I shall definitely rule that to be a claim of exactly two more tricks and a concession of any remaining tricks. The question of any overtricks shall then be decided under law 71. From jean-pierre.rocafort at meteo.fr Tue Jan 25 10:58:37 2011 From: jean-pierre.rocafort at meteo.fr (Jean-Pierre Rocafort) Date: Tue, 25 Jan 2011 10:58:37 +0100 Subject: [BLML] Exam question In-Reply-To: <4D3E9ACD.5020602@skynet.be> References: <4D3E0908.5090009@skynet.be><1Phbhi-0YKMXg0@fwd05.aul.t-online.d e> <4D3E7F96.3020101@skynet.be><4D3E92BF.1050708@meteo.fr> <4D3E9ACD.5020602@skynet.be> Message-ID: <4D3E9ECD.9010208@meteo.fr> Herman De Wael a ?crit : > Jean-Pierre Rocafort wrote: > > >> something i fail to understand but you are surely going to make clear: >> why did you say what you said if it was not in order to claim? >> jpr >> > > Do I need a reason? > Am I not allowed to tell my opponents something they are allowed to use? > I tell them they don't need to worry about playing me down. We shall now > fight for overtricks. If they don't want to fatigue themselves, they > don't have to. > The claim laws are there to forbid players lulling their opponents into > sleep, by telling them not to think when thinking might still do them > good. I do no such thing. I tell them they don't need to worry about > defeating the contract, which is true - I even showed the cards. I said > nothing about the number of overtricks I intend to make, and allow them > full thinking on that. > > If you think I do something wrong here, then what is the difference > between this one and a player saying "ah, nice", when defenders follow > to two tricks in trumps. Does he not show his nine-cards and so-many > tricks in just the same way as I did? Did that person claim? ok, don't worry, it's only me who am surprised by people who can simultaneously play and comment. jpr > > -- _______________________________________________ Jean-Pierre Rocafort METEO-FRANCE DSI/CM 42 Avenue Gaspard Coriolis 31057 Toulouse CEDEX Tph: 05 61 07 81 02 (33 5 61 07 81 02) Fax: 05 61 07 81 09 (33 5 61 07 81 09) e-mail: jean-pierre.rocafort at meteo.fr Serveur WWW METEO-France: http://www.meteo.fr _______________________________________________ From PeterEidt at t-online.de Tue Jan 25 10:58:42 2011 From: PeterEidt at t-online.de (Peter Eidt) Date: Tue, 25 Jan 2011 10:58:42 +0100 Subject: [BLML] =?utf-8?q?Exam_question?= In-Reply-To: <4D3E9ACD.5020602@skynet.be> References: <4D3E9ACD.5020602@skynet.be> Message-ID: <1Phfff-2FiDdA0@fwd06.aul.t-online.de> What do you want us to do, Herman? You asked an exam question, giving us (hopefully) the facts. The unanimous (execpt Mr. Fusselman) answer was: it was a claim - play ceased. So you should expect when giving your answer "Declarer did not claim and may play on" to get zero points for this. Now you are mixing up all counter-arguments bringing in your own language and tell us, we all are wrong because the facts were not as stated. You told us declarer showed Ace and King of trumps just to tell opponents he has them and in order to be nice (and smart?) to them - as if there wouldn't have been aware of that fact ... ts ts ts From: Herman De Wael > Jean-Pierre Rocafort wrote: > > > > something i fail to understand but you are surely going to make > > clear: why did you say what you said if it was not in order to > > claim? jpr > > > Do I need a reason? > Am I not allowed to tell my opponents something they are allowed to > use? I tell them they don't need to worry about playing me down. We > shall now fight for overtricks. If they don't want to fatigue > themselves, they don't have to. > The claim laws are there to forbid players lulling their opponents > into sleep, by telling them not to think when thinking might still do > them good. I do no such thing. I tell them they don't need to worry > about defeating the contract, which is true - I even showed the cards. > I said nothing about the number of overtricks I intend to make, and > allow them full thinking on that. > > If you think I do something wrong here, then what is the difference > between this one and a player saying "ah, nice", when defenders follow > to two tricks in trumps. Does he not show his nine-cards and so-many > tricks in just the same way as I did? Did that person claim? From agot at ulb.ac.be Tue Jan 25 11:07:23 2011 From: agot at ulb.ac.be (Alain Gottcheiner) Date: Tue, 25 Jan 2011 11:07:23 +0100 Subject: [BLML] Unsure In-Reply-To: <000301cbbc75$385ebf00$a91c3d00$@no> References: <586818.75337.qm@web28510.mail.ukl.yahoo.com> <4D3704A8.8090006@ skynet.be> <128171.88697.qm@web25402.mail.ukl. yahoo.com> < 4D382CB9.2060700@ulb.ac.be> <6410CABD-9AF3-45A7-9B10-F5EDC78C6095@starpowe r. net> <4D3852CE.6080908@ul b.ac.be> <4D386625.90105@meteo.fr> <4D3869DD.9030708@ulb.ac.be> <4D3873A6.5 070104@meteo.fr> <107026.33279.qm@web28515.mail.ukl.yahoo.com> <4D39BC9E.90 40108@nhcc.net> <1820574630.137446.1295675648497.JavaMail.ngmail@webmail11. arcor-online.net> <4D3A93A3.8020906@skynet.be> <000901cbba0f$ce55b3f0$6b011 bd0$@no><4D3D72AF.3060606@ulb.ac.be> <000001cbbbce$a0c010f0$e24032d0$@no><7 8BC7D50-DDDC-4459-BA0B-87467613E5B0@starpower.net> <000801cbbc16$e2466e00$a6d34a00$@no> <4D3E981B.1050009@meteo.fr> <000301cbbc75$385ebf00$a91c3d00$@no> Message-ID: <4D3EA0DB.7070103@ulb.ac.be> Le 25/01/2011 10:49, Sven Pran a ?crit : > On Behalf Of Jean-Pierre Rocafort > .... >>> I shall expect the player to provide a very good reason for using a >>> call for which (in his own opinion) no understanding exists. I don't > believe he >> can. >> you don't believe but these things happen. you can call it improvisation > and some >> players are very fond of it. and why should they have to justify > themselves unless >> there is a specific regulation which forbids unexpected bids which are not > based >> on any agreement? >> some players like to test their partner's reactions to innovation. on some >> circonstances they hope that partner will be able to understand from his > own >> cards. on other times they think it will be harmless even if partner > doesn't >> understand, or the preemptive effect will be enough to take the risk of a >> misunderstanding, or they will be able to clarify the situation later > on... >> there are many situations where a player may think it would be profitable > to make >> a call for which no understanding exists. >> once i had to adjudicate this case on a AC: >> - 1D 4D >> when asked (there were screens) both players said: no agreement, no prior >> occurence. they explained what agreements they had for possible > alternative >> calls: double (take-out), 2D (5H-5S), 3D (asking for D stop). the AC was > convinced >> by their arguments. would you not have been? >> jpr > NO. > First of all I would want to know how the auction continued. > I would have asked the bidder (without his partner present) why he made his > 4D bid "because 2D and 3D were defined as artificial" > , how he expected (or hoped) his partner to understand it "as natural" Those two responses will probably give an insight that there is a meta-agreement at work : "natural if nothing else fits". > and why he > selected his subsequent calls in the auction. > Then I would similarly ask his partner (without the bidder present) how he > understood the 4D bid and why he selected his following call and subsequent > calls (if any) in the auction. From svenpran at online.no Tue Jan 25 11:28:07 2011 From: svenpran at online.no (Sven Pran) Date: Tue, 25 Jan 2011 11:28:07 +0100 Subject: [BLML] Unsure In-Reply-To: <4D3EA0DB.7070103@ulb.ac.be> References: <586818.75337.qm@web28510.mail.ukl.yahoo.com> <4D3704A8.8090006@ skynet.be> <128171.88697.qm@web25402.mail.ukl. yahoo.com> < 4D382CB9.2060700@ulb.ac.be> <6410CABD-9AF3-45A7-9B10-F5EDC78C6095@starpowe r. net> <4D3852CE.6080908@ul b.ac.be> <4D386625.90105@meteo.fr> <4D3869DD.9030708@ulb.ac.be> <4D3873A6.5 070104@meteo.fr> <107026.33279.qm@web28515.mail.ukl.yahoo.com> <4D39BC9E.90 40108@nhcc.net> <1820574630.137446.1295675648497.JavaMail.ngmail@webmail11. arcor-online.net> <4D3A93A3.8020906@skynet.be> <000901cbba0f$ce55b3f0$6b011 bd0$@no><4D3D72AF.3060606@ulb.ac.be> <000001cbbbce$a0c010f0$e24032d0$@no><7 8BC7D50-DDDC-4459-BA0B-87467613E5B0@starpower.net> <000801cbbc16$e2466e00$a6d34a00$@no> <4D3E981B.1050009@meteo.fr> <000301cbbc75$385ebf00$a91c3d00$@no> <4D3EA0DB.7070103@ulb.ac.be> Message-ID: <000a01cbbc7a$9083f470$b18bdd50$@no> > -----Original Message----- > From: Alain Gottcheiner [mailto:agot at ulb.ac.be] ......... > >> when asked (there were screens) both players said: no agreement, no > >> prior occurence. they explained what agreements they had for possible > > alternative > >> calls: double (take-out), 2D (5H-5S), 3D (asking for D stop). the AC > >> was > > convinced > >> by their arguments. would you not have been? > >> jpr > > NO. > > First of all I would want to know how the auction continued. > > I would have asked the bidder (without his partner present) why he > > made his 4D bid > "because 2D and 3D were defined as artificial" > > > , how he expected (or hoped) his partner to understand it > > "as natural" > > Those two responses will probably give an insight that there is a meta-agreement > at work : "natural if nothing else fits". And now I would request a very good reason why they apparently deliberately suppressed the information that the 4D bid was natural by introducing lots of unasked for extraneous information instead of giving the correct explanation of the 4D bid. From agot at ulb.ac.be Tue Jan 25 11:30:17 2011 From: agot at ulb.ac.be (Alain Gottcheiner) Date: Tue, 25 Jan 2011 11:30:17 +0100 Subject: [BLML] Unsure In-Reply-To: <000a01cbbc7a$9083f470$b18bdd50$@no> References: <586818.75337.qm@web28510.mail.ukl.yahoo.com> <4D3704A8.8090006@ skynet.be> <128171.88697.qm@web25402.mail.ukl. yahoo.com> < 4D382CB9.2060700@ulb.ac.be> <6410CABD-9AF3-45A7-9B10-F5EDC78C6095@starpowe r. net> <4D3852CE.6080908@ul b.ac.be> <4D386625.90105@meteo.fr> <4D3869DD.9030708@ulb.ac.be> <4D3873A6.5 070104@meteo.fr> <107026.33279.qm@web28515.mail.ukl.yahoo.com> <4D39BC9E.90 40108@nhcc.net> <1820574630.137446.1295675648497.JavaMail.ngmail@webmail11. arcor-online.net> <4D3A93A3.8020906@skynet.be> <000901cbba0f$ce55b3f0$6b011 bd0$@no><4D3D72AF.3060606@ulb.ac.be> <000001cbbbce$a0c010f0$e24032d0$@no><7 8BC7D50-DDDC-4459-BA0B-87467613E5B0@starpower.net> <000801cbbc16$e2466e00$a6d34a00$@no> <4D3E981B.1050009@meteo.fr> <000301cbbc75$385ebf00$a91c3d00$@no> <4D3EA0DB.7070103@ulb.ac.be> <000a01cbbc7a$9083f470$b18bdd50$@no> Message-ID: <4D3EA639.6010203@ulb.ac.be> Le 25/01/2011 11:28, Sven Pran a ?crit : >> -----Original Message----- >> From: Alain Gottcheiner [mailto:agot at ulb.ac.be] > ......... >>>> when asked (there were screens) both players said: no agreement, no >>>> prior occurence. they explained what agreements they had for possible >>> alternative >>>> calls: double (take-out), 2D (5H-5S), 3D (asking for D stop). the AC >>>> was >>> convinced >>>> by their arguments. would you not have been? >>>> jpr >>> NO. >>> First of all I would want to know how the auction continued. >>> I would have asked the bidder (without his partner present) why he >>> made his 4D bid >> "because 2D and 3D were defined as artificial" >> >>> , how he expected (or hoped) his partner to understand it >> "as natural" >> >> Those two responses will probably give an insight that there is a > meta-agreement >> at work : "natural if nothing else fits". > And now I would request a very good reason why they apparently deliberately > suppressed the information that the 4D bid was natural by introducing lots > of unasked for extraneous information instead of giving the correct > explanation of the 4D bid. Which is why that was a very bad example. From jean-pierre.rocafort at meteo.fr Tue Jan 25 11:38:50 2011 From: jean-pierre.rocafort at meteo.fr (Jean-Pierre Rocafort) Date: Tue, 25 Jan 2011 11:38:50 +0100 Subject: [BLML] Unsure In-Reply-To: <4D3EA0DB.7070103@ulb.ac.be> References: <586818.75337.qm@web28510.mail.ukl.yahoo.com> <4D3704A8.8090006@ skynet.be> <128171.88697.qm@web25402.mail.ukl. yahoo.com> < 4D382CB9.2060700@ulb.ac.b e> <6410CABD-9AF3-45A7-9B10-F5EDC78C6095@starpowe r. net> <4D3852CE.6080908 @ul b.ac.be> <4D386625.90105@meteo.fr> <4D3869DD.9030708@ulb.ac.be> <4D3873 A6.5 070104@meteo.fr> <107026.33279.qm@web28515.mail.ukl.yahoo.com> <4D39BC 9E.90 40108@nhcc.net> <1820574630.137446.1295675648497.JavaMail.ngmail@webm ail11. arcor-online.net> <4D3A93A3.8020906@skynet.be> <000901cbba0f$ce55b3f 0$6b011 bd0$@no><4D3D72AF.3060606@ulb.ac.be> <000001cbbbce$a0c010f0$e24032d 0$@no><7 8BC7D50-DDDC-4459-BA0B-87467613E5B0@starpower.net> <000801cbbc16$e 2466e00$a6d34a00$@no> <4D3E981B.1050009@meteo.fr><000301cbbc75$385ebf00$a91 c3d00$@no> <4D3EA0DB.7070103@ulb.ac.be> Message-ID: <4D3EA83A.1060909@meteo.fr> Alain Gottcheiner a ?crit : > Le 25/01/2011 10:49, Sven Pran a ?crit : >> On Behalf Of Jean-Pierre Rocafort >> .... >>>> I shall expect the player to provide a very good reason for using a >>>> call for which (in his own opinion) no understanding exists. I don't >> believe he >>> can. >>> you don't believe but these things happen. you can call it improvisation >> and some >>> players are very fond of it. and why should they have to justify >> themselves unless >>> there is a specific regulation which forbids unexpected bids which are not >> based >>> on any agreement? >>> some players like to test their partner's reactions to innovation. on some >>> circonstances they hope that partner will be able to understand from his >> own >>> cards. on other times they think it will be harmless even if partner >> doesn't >>> understand, or the preemptive effect will be enough to take the risk of a >>> misunderstanding, or they will be able to clarify the situation later >> on... >>> there are many situations where a player may think it would be profitable >> to make >>> a call for which no understanding exists. >>> once i had to adjudicate this case on a AC: >>> - 1D 4D >>> when asked (there were screens) both players said: no agreement, no prior >>> occurence. they explained what agreements they had for possible >> alternative >>> calls: double (take-out), 2D (5H-5S), 3D (asking for D stop). the AC was >> convinced >>> by their arguments. would you not have been? >>> jpr >> NO. >> First of all I would want to know how the auction continued. opponents landed in an inadequate contract. >> I would have asked the bidder (without his partner present) why he made his >> 4D bid > "because 2D and 3D were defined as artificial" maybe it happened twice! i don't remember very well the one i refered (maybe 10 years ago) but the overcaller had 11 or 12 major cards and was happy to muddy waters. his partner had passed, he had not many defensive values. > >> , how he expected (or hoped) his partner to understand it > > "as natural" he thought the number of diamonds in partner's hand would help him more than anything else. on the other side, opener, if balanced, could be in doubt. if partner was short in diamonds, he didn't care very much if partner didn't understand, quite the contrary. > > Those two responses will probably give an insight that there is a > meta-agreement at work : "natural if nothing else fits". > > >> and why he >> selected his subsequent calls in the auction. he carefully selected green cards, seeing no need to clarify. >> Then I would similarly ask his partner (without the bidder present) how he >> understood the 4D bid he looked at his hand and understood. >> and why he selected his following call and subsequent >> calls (if any) in the auction. he didn't see any need to add to the confusion and was satisfied by the opponents'actions. jpr > -- _______________________________________________ Jean-Pierre Rocafort METEO-FRANCE DSI/CM 42 Avenue Gaspard Coriolis 31057 Toulouse CEDEX Tph: 05 61 07 81 02 (33 5 61 07 81 02) Fax: 05 61 07 81 09 (33 5 61 07 81 09) e-mail: jean-pierre.rocafort at meteo.fr Serveur WWW METEO-France: http://www.meteo.fr _______________________________________________ From ehaa at starpower.net Tue Jan 25 15:27:25 2011 From: ehaa at starpower.net (Eric Landau) Date: Tue, 25 Jan 2011 09:27:25 -0500 Subject: [BLML] Unsure In-Reply-To: <000801cbbbe6$886147a0$9923d6e0$@no> References: <586818.75337.qm@web28510.mail.ukl.yahoo.com> <4D3704A8.8090006@skynet.be> <128171.88697.qm@web25402.mail.ukl. yahoo.com> < 4D382CB9.2060700@ulb.ac.be><6410CABD-9AF3-45A7-9B10-F5EDC78C6095@starpowe r. net><4D3852CE.6080908@ulb.ac.be> <4D386625.90105@meteo.fr> <4D3869DD.9030708@ulb.ac.be> <4D3873A6.5070104@meteo.fr> <107026.33279.qm@web28515.mail.ukl.yahoo.com> <4D39BC9E.9040108@nhcc.net> <1820574630.137446.1295675648497.JavaMail.ngmail@webmail11.arcor-online.net> <4D3D7134.4000803@ulb.ac.be> <431431.44115.qm@web28515.mail.ukl.yahoo.com> <4D3D7F51.70805@ulb.ac.be> <395644.68863.qm@web28514.mail.ukl.yahoo.com><000101cbbbcf$4f8dd270$eea97750$@no> <011301cbbbcf$fc8610a0$f59231e0$@nl><4A3418FD24684B579A369D685BF6D6FB@Lounge> <011401cbbbd6$4e621f80$eb265e80$@nl> <000101cbbbdb$0a0717a0$1e1546e0$@no> <670306DF0C5446E8B8770FE884FA6E1C@Lounge> <000801cbbbe6$886147a0$9923d 6e0$@no> Message-ID: <297889D5-0605-4FF8-AB20-AE02FE64BE6D@starpower.net> On Jan 24, 2011, at 11:48 AM, Sven Pran wrote: > On Behalf Of David > >> *If the partnership can show me some positive evidence (System >> documentation, >> *CC or whatever) that the meaning of the call can be either A or B >> I shall *accept >> that as the explanation. Otherwise, if the actual hand matches >> either *A or B but >> not both I shall rule misinformation and eventually judge whether >> *opponents have >> been damaged. Take a look at Law 75! >> >> Yes indeed let us look at L75. >> >> L75 starts >> After a misleading explanation........ >> >> The opposition are told that as a consequence of other agreements >> 2N is either >> natural or lebensohl but as the sequence has neither occurred >> before nor is the >> subject of an agreement it is impossible to say which. >> >> Now in my opinion (assuming that is a true statement) there is >> nothing misleading. >> >> L75B starts >> The actual partnership agreement is....... >> >> Well there is no actual partnership agreement and that is what the >> opposition >> have been told. >> >> So what is the problem here? >> >> L75 is the subject of Mistaken Explanations and Mistaken calls. >> Calls which are not covered by systemic agreements and are >> explained as such >> are neither mistakenly explained nor mistakenly bid. > > So you missed the possibly most important sentence in Law 75? > > the Director is to presume Mistaken Explanation, rather than > Mistaken Call, > in the absence of evidence to the contrary That says that if the director cannot decide between mistaken explanation (ME) and mistaken call (MC), he is to presume the former. In David's scenario, there is no possibility of an MC; the director's determination is either ME (if he deems there was MI) or no irregularity (if he deems the explanation to be correct). The "possibly most important sentence in L75" (not that I would agree) is entirely irrelevant here. Eric Landau 1107 Dale Drive Silver Spring MD 20910 ehaa at starpower.net From ehaa at starpower.net Tue Jan 25 15:50:20 2011 From: ehaa at starpower.net (Eric Landau) Date: Tue, 25 Jan 2011 09:50:20 -0500 Subject: [BLML] Unsure In-Reply-To: <1868019136.66657.1295900769160.JavaMail.ngmail@webmail08.arcor-online.net> References: <586818.75337.qm@web28510.mail.ukl.yahoo.com> <4D3704A8.8090006@skynet.be> <128171.88697.qm@web25402.mail.ukl. yahoo.com> < 4D382CB9.2060700@ulb.ac.be><6410CABD-9AF3-45A7-9B10-F5EDC78C6095@starpowe r. net> <4D3852CE.6080908@ulb.ac.be> <4D386625.90105@meteo.fr> <4D3869DD.9030708@ulb.ac.be> <4D3873A6.5070104@meteo.fr> <107026.33279.qm@web28515.mail.ukl.yahoo.com> <4D39BC9E.9040108@nhcc.net> <1820574630.137446.1295675648497.JavaMail.ngmail@webmail11.arcor-online.net> <4D3D7134.4000803@ulb.ac.be> <431431.44115.qm@web28515.mail.ukl.yahoo.com> <4D3D7F51.70805@ulb.ac.be> <395644.68863.qm@web28514.mail.ukl.yahoo.com> <000101cbbbcf$4f8dd270$eea97750$@no> <1868019136.66657.1295900769160.JavaMail.ngmail@webmail08.arcor-online.net> Message-ID: <532598E0-1D81-4AA2-84BA-3EC0BE2D2843@starpower.net> On Jan 24, 2011, at 3:26 PM, Thomas Dehn wrote: > Eric Landau wrote: > >> On Jan 24, 2011, at 9:02 AM, Sven Pran wrote: >> >>> On Behalf Of Nigel Guthrie >>> >>>> [Alain] >>>> doesn't hold. How could your agreement be precise on mondays and >>>> nonexistent on tuesdays, because you happen to work hard on >>>> tuesdays ? >>>> >>>> {Nigel] >>>> Pass. Ask me another. I was merely trying to represent Richard's >>>> interpretation. >>>> >>>> Alain and others maintain that saying "Unsure" is an MI >>>> infraction. But can >>>> anybody tell us what the law prescribes you *should* say if >>>> "Unsure" is the truth? >>> >>> If "unsure" is the truth (and your answer) then you fail to give >>> opponents >>> the explanation they are entitled to. According to laws this is >>> misexplanation. "Unsure" is only an explanation of why you cannot >>> comply >>> with the laws; it is not an explanation of the auction as required >>> by the >>> laws. >> >> There is no law that requires one's body of partnership >> understandings to provide explanations for every possible auction, >> and you can't give what you ain't got. I do agree that "unsure" is a >> poor choice of word, implying that the uncertainty is one's own >> rather than the partnership's, but that's a semanic side-issue in >> this discussion, where we've been using it as the equivalent of "I >> don't know" or "we have no agreement". The laws cannot and do not >> require you to provide an explanation of an auction based on >> "understandings" that don't exist and never have. > > Try to see it from the other side: > Your regular partner made that bid. > This implies that partner has some sort of > expectation that you can work out what > the bid means. He expects me to work it out based on our meta-agreements and my understanding of our general style. My opponents are entitled to the same knowledge I have as to the systemic meaning of partner's bid. If they ask, I explain that we have no agreement about this particular auction, but..., and then proceed to disclose any meta- agreements or elements of our general style that might be in any way relevant to figuring out what partner's bid means. I have "leveled the playing field" to the best of my ability; from here there's only "bridge logic". I have no obligation to explain any conclusion (or lack thereof) that my bridge logic may have led me to, nor even, for that matter, any reason to think that my deductions are any more valid than what my opponents may have been able to deduce from the same facts. Eric Landau 1107 Dale Drive Silver Spring MD 20910 ehaa at starpower.net From Hermandw at skynet.be Tue Jan 25 15:53:28 2011 From: Hermandw at skynet.be (Herman De Wael) Date: Tue, 25 Jan 2011 15:53:28 +0100 Subject: [BLML] Exam question In-Reply-To: <1Phfff-2FiDdA0@fwd06.aul.t-online.de> References: <4D3E9ACD.5020602@skynet.be> <1Phfff-2FiDdA0@fwd06.aul.t-online.de> Message-ID: <4D3EE3E8.70308@skynet.be> Peter Eidt wrote: > What do you want us to do, Herman? > > You asked an exam question, giving us (hopefully) > the facts. > The unanimous (execpt Mr. Fusselman) answer > was: it was a claim - play ceased. > So you should expect when giving your answer > "Declarer did not claim and may play on" to get > zero points for this. > > Now you are mixing up all counter-arguments > bringing in your own language and tell us, we all > are wrong because the facts were not as stated. > You told us declarer showed Ace and King of > trumps just to tell opponents he has them and > in order to be nice (and smart?) to them - as if > there wouldn't have been aware of that fact ... > > ts ts ts > Well, Peter, what do you want me to do? I ask a question, believing to know the answer. You tell me another answer. Now of course the answer of the great Peter Eidt is the correct one, and that of lowly Herman De Wael must be wrong. ts ts ts. So I ask you again, Peter: what is the meaning of the word "specific" in L68A? -- Herman De Wael Wilrijk Antwerpen Belgium From svenpran at online.no Tue Jan 25 15:53:40 2011 From: svenpran at online.no (Sven Pran) Date: Tue, 25 Jan 2011 15:53:40 +0100 Subject: [BLML] Unsure In-Reply-To: <297889D5-0605-4FF8-AB20-AE02FE64BE6D@starpower.net> References: <586818.75337.qm@web28510.mail.ukl.yahoo.com> <4D3704A8.8090006@skynet.be> <128171.88697.qm@web25402.mail.ukl. yahoo.com> < 4D382CB9.2060700@ulb.ac.be><6410CABD-9AF3-45A7-9B10-F5EDC78C6095@starpowe r. net><4D3852CE.6080908@ulb.ac.be> <4D386625.90105@meteo.fr> <4D3869DD.9030708@ulb.ac.be> <4D3873A6.5070104@meteo.fr> <107026.33279.qm@web28515.mail.ukl.yahoo.com> <4D39BC9E.9040108@nhcc.net> <1820574630.137446.1295675648497.JavaMail.ngmail@webmail11.arcor-online.net> <4D3D7134.4000803@ulb.ac.be> <431431.44115.qm@web28515.mail.ukl.yahoo.com> <4D3D7F51.70805@ulb.ac.be> <395644.68863.qm@web28514.mail.ukl.yahoo.com><000101cbbbcf$4f8dd270$eea97750$@no> <011301cbbbcf$fc8610a0$f59231e0$@nl><4A3418FD24684B579A369D685BF6D6FB@Lounge> <011401cbbbd6$4e621f80$eb265e80$@nl> <000101cbbbdb$0a0717a0$1e1546e0$@no> <670306DF0C5446E8B8770FE884FA6E1C@Lounge> <000801cbbbe6$886147a0$9923! d 6e0$@no> <297889D5-06 05-4FF8-AB20-AE02FE64BE6D@starpower.net> Message-ID: <000b01cbbc9f$a7b1a410$f714ec30$@no> On Behalf Of Eric Landau ........... > > So you missed the possibly most important sentence in Law 75? > > > > the Director is to presume Mistaken Explanation, rather than Mistaken > > Call, in the absence of evidence to the contrary > > That says that if the director cannot decide between mistaken explanation (ME) > and mistaken call (MC), he is to presume the former. In David's scenario, there is > no possibility of an MC; the director's determination is either ME (if he deems > there was MI) or no irregularity (if he deems the explanation to be correct). The > "possibly most important sentence in L75" (not that I would agree) is entirely > irrelevant here. The explanation cannot be correct unless it explains the meaning of the call. If no evidence of an agreement (whether explicit or implied) can be presented the Director shall rule that the call corresponds with some features of the hand held by the player making that call and that such correspondence represents the meaning of the call. This is the essence of the clause: "the Director is to presume Mistaken Explanation, rather than Mistaken Call, in the absence of evidence to the contrary" An "explanation" that the player does not know the meaning of the call is not an explanation of the meaning of the call but rather an explanation of the reason why he cannot comply with the requirements of the law. The reason he might give for this is irrelevant, the fact is that he has not given the required explanation of the call. From Hermandw at skynet.be Tue Jan 25 15:57:21 2011 From: Hermandw at skynet.be (Herman De Wael) Date: Tue, 25 Jan 2011 15:57:21 +0100 Subject: [BLML] Exam question In-Reply-To: <000401cbbc76$6e66ba20$4b342e60$@no> References: <4D3D3B2F.5090107@skynet.be> <1PhIDl-0dAMts0@fwd00.aul.t-online.de> <4D3E0908.5090009@skynet.be> <000201cbbc6f$6360ef80$2a22ce80$@no> <4D3E9B17.7050704@skynet.be> <000401cbbc76$6e66ba20$4b342e60$@no> Message-ID: <4D3EE4D1.7010203@skynet.be> Sven Pran wrote: > On Behalf Of Herman De Wael >> Sven Pran wrote: >>> >>> In my language you claimed that you will make your contract, a >>> statement that is synonymous with claiming a total of (exactly) 8 > tricks. >>> You mentioned nothing about the further tricks so effectively you >>> conceded those with your claim. >>> >> >> Well, I was not speaking Norwegian. >> In my language, I told them I would make 8 or more tricks. >> Please rule on that one and not on the case you imagine. > > Well, I have only a marginal knowledge of Flemish or Dutch, I can hardly > read it. > > If anybody in your apparent position exposes his two remaining high trumps > and just says "I shall make my contract" or "I have my contract" (in > English, or the equivalent in Norwegian) and nothing more then I shall > definitely rule that to be a claim of exactly two more tricks and a > concession of any remaining tricks. > > The question of any overtricks shall then be decided under law 71. > You don't hear the intonation? "I'm definitely going to make this contract", while showing two cards and clearly thinking about how to play on. You don't understand this the way my opponents did "8 or mmore tricks"? Do you accept that I can say "I'm making a minimum of 8 tricks and I am not stopping play"? Did you see the word "specific" in L68A? Did you understand that it is needed (and probably intended) for just this purpose? To exclude opponents from insisting that a claim has been made whenever a player announces he has the remaining trumps? -- Herman De Wael Wilrijk Antwerpen Belgium From Hermandw at skynet.be Tue Jan 25 16:01:07 2011 From: Hermandw at skynet.be (Herman De Wael) Date: Tue, 25 Jan 2011 16:01:07 +0100 Subject: [BLML] Unsure In-Reply-To: <532598E0-1D81-4AA2-84BA-3EC0BE2D2843@starpower.net> References: <586818.75337.qm@web28510.mail.ukl.yahoo.com> <4D3704A8.8090006@skynet.be> <128171.88697.qm@web25402.mail.ukl. yahoo.com> < 4D382CB9.2060700@ulb.ac.be><6410CABD-9AF3-45A7-9B10-F5EDC78C6095@starpowe r. net> <4D3852CE.6080908@ulb.ac.be> <4D386625.90105@meteo.fr> <4D3869DD.9030708@ulb.ac.be> <4D3873A6.5070104@meteo.fr> <107026.33279.qm@web28515.mail.ukl.yahoo.com> <4D39BC9E.9040108@nhcc.net> <1820574630.137446.1295675648497.JavaMail.ngmail@webmail11.arcor-online.net> <4D3D7134.4000803@ulb.ac.be> <431431.44115.qm@web28515.mail.ukl.yahoo.com> <4D3D7F51.70805@ulb.ac.be> <395644.68863.qm@web28514.mail.ukl.yahoo.com> <000101cbbbcf$4f8dd270$eea97750$@no> <1868019136.66657.1295900769160.JavaMail.ngmail@webmail08.arcor-online.net> <532598E0-1D81-4AA2-84BA-3EC0BE2D2843@starpower.net> Message-ID: <4D3EE5B3.1030006@skynet.be> Eric Landau wrote: > > He expects me to work it out based on our meta-agreements and my > understanding of our general style. My opponents are entitled to the > same knowledge I have as to the systemic meaning of partner's bid. > If they ask, I explain that we have no agreement about this > particular auction, but..., and then proceed to disclose any meta- > agreements or elements of our general style that might be in any way > relevant to figuring out what partner's bid means. I have "leveled > the playing field" to the best of my ability; from here there's only > "bridge logic". I have no obligation to explain any conclusion (or > lack thereof) that my bridge logic may have led me to, nor even, for > that matter, any reason to think that my deductions are any more > valid than what my opponents may have been able to deduce from the > same facts. > In short, you rely on General Bridge Knowledge. And then they draw some conclusions, and those are not the same conclusions you took, and not the same ones your partner took, whereas you two took the same ones. Apparently then, your general bridge knowledge wasn't so General. I rule MI, and obfuscation to boot. > > Eric Landau > 1107 Dale Drive > Silver Spring MD 20910 > ehaa at starpower.net > > _______________________________________________ > Blml mailing list > Blml at rtflb.org > http://lists.rtflb.org/mailman/listinfo/blml > > > > > No virus found in this incoming message. > Checked by AVG - www.avg.com > Version: 9.0.872 / Virus Database: 271.1.1/3401 - Release Date: 01/24/11 20:34:00 > -- Herman De Wael Wilrijk Antwerpen Belgium From ehaa at starpower.net Tue Jan 25 16:06:44 2011 From: ehaa at starpower.net (Eric Landau) Date: Tue, 25 Jan 2011 10:06:44 -0500 Subject: [BLML] Exam question In-Reply-To: References: <4D3D3B2F.5090107@skynet.be> <1PhIDl-0dAMts0@fwd00.aul.t-online.de> <4D3E0908.5090009@skynet.be> Message-ID: <82D7A2DF-92A6-495E-ABC1-6C5C27F82B32@starpower.net> On Jan 24, 2011, at 7:21 PM, Jerry Fusselman wrote: > On Mon, Jan 24, 2011 at 5:19 PM, Herman De Wael > wrote: > >> Is no-one noticing the word "specific" number here? I did not say >> I was >> going to make 8 tricks - I was clearly saying 8 or more. > > I agree, and that's what I said, but I was the only one. Did I pass > the exam question? No, because that's not the issue. There are two sentences in L68A. When Herman deliberately exposed the trump AK, he already claimed (even had he said nothing) "unless he demonstrably did not intend to". That he subsequently stated the obvious out loud -- that the trump AK would both take tricks -- doesn't affect the determination of his demonstrable intention, hence doesn't matter. Eric Landau 1107 Dale Drive Silver Spring MD 20910 ehaa at starpower.net From ziffbridge at t-online.de Tue Jan 25 16:15:24 2011 From: ziffbridge at t-online.de (Matthias Berghaus) Date: Tue, 25 Jan 2011 16:15:24 +0100 Subject: [BLML] Exam question In-Reply-To: <4D3EE4D1.7010203@skynet.be> References: <4D3D3B2F.5090107@skynet.be> <1PhIDl-0dAMts0@fwd00.aul.t-online.de> <4D3E0908.5090009@skynet.be> <000201cbbc6f$6360ef80$2a22ce80$@no> <4D3E9B17.7050704@skynet.be> <000401cbbc76$6e66ba20$4b342e60$@no> <4D3EE4D1.7010203@skynet.be> Message-ID: <4D3EE90C.5020002@t-online.de> Herman, the problem with this kind of question (like many other questions related to claims and non-claims) is that they are completely impossible to answer on paper, while no experienced director would get them wrong at the table.I don`t think it possible - this being a multi-cultural and multi-lingual list - to pose a question of this kind in a way to make it 100% clear to everyone. Remember, there are lots of people here who are not native speakers, and quite a number of them have not mastered the english language as well as you have. If one has to translate a text into one`s native language in order to understand it, something is bound to get lost in translation. And even the "natives"... I have found a website dedicated to the differences between British and American English. Great fun to read. Best reagrds Matthias From ehaa at starpower.net Tue Jan 25 16:23:09 2011 From: ehaa at starpower.net (Eric Landau) Date: Tue, 25 Jan 2011 10:23:09 -0500 Subject: [BLML] Naive question about double shots In-Reply-To: References: <327C53E4-386A-40BF-9871-4071863E8144@starpower.net> Message-ID: On Jan 24, 2011, at 8:05 PM, Jerry Fusselman wrote: > Alright, but I still want to know what defines a double shot. I guess > we are talking about > Law 12C1b: "If, subsequent to the irregularity, the non-offending > side has > contributed to its own damage by a serious error (unrelated to the > infraction) or by wild or gambling action it does not receive > relief in the > adjustment for such part of the damage as is self-inflicted." The > term double shot is undefined in the laws. > > Alright, I imagine that no one on BLML can define it. WTP? A double shot is an action taken subsequent to an opponent's infraction that the player would not have taken in otherwise identical circumstances had there been no infraction. Eric Landau 1107 Dale Drive Silver Spring MD 20910 ehaa at starpower.net From JffEstrsn at aol.com Tue Jan 25 16:27:25 2011 From: JffEstrsn at aol.com (Jeff Easterson) Date: Tue, 25 Jan 2011 16:27:25 +0100 Subject: [BLML] Exam question In-Reply-To: <82D7A2DF-92A6-495E-ABC1-6C5C27F82B32@starpower.net> References: <4D3D3B2F.5090107@skynet.be> <1PhIDl-0dAMts0@fwd00.aul.t-online.de> <4D3E0908.5090009@skynet.be> <82D7A2DF-92A6-495E-ABC1-6C5C27F82B32@starpower.net> Message-ID: <4D3EEBDD.1070908@aol.com> Here we go again. Herman poses a question and receives numerous replies. He is not satisfied with the opinions expressed (because they are different from his) so we shall start a long boring discussion with constant repetition of the same arguments. The contrasting opinions have been clearly stated. Is it not possible to accept that? I wonder if Herman might have talents as a missionary. And, incidentally, it seems unusual, superfluous and bordering on showmanship to show two cards and tell the opponents they are high although not intending to claim. Why not play bridge as everyone else does and dispense with theatrical gestures? Or was it simply an attempt to impress the opponents? Ciao, JE Am 25.01.2011 16:06, schrieb Eric Landau: > On Jan 24, 2011, at 7:21 PM, Jerry Fusselman wrote: > >> On Mon, Jan 24, 2011 at 5:19 PM, Herman De Wael >> wrote: >> >>> Is no-one noticing the word "specific" number here? I did not say >>> I was >>> going to make 8 tricks - I was clearly saying 8 or more. >> I agree, and that's what I said, but I was the only one. Did I pass >> the exam question? > No, because that's not the issue. There are two sentences in L68A. > When Herman deliberately exposed the trump AK, he already claimed > (even had he said nothing) "unless he demonstrably did not intend > to". That he subsequently stated the obvious out loud -- that the > trump AK would both take tricks -- doesn't affect the determination > of his demonstrable intention, hence doesn't matter. > > > Eric Landau > 1107 Dale Drive > Silver Spring MD 20910 > ehaa at starpower.net > > _______________________________________________ > Blml mailing list > Blml at rtflb.org > http://lists.rtflb.org/mailman/listinfo/blml > From ehaa at starpower.net Tue Jan 25 16:31:28 2011 From: ehaa at starpower.net (Eric Landau) Date: Tue, 25 Jan 2011 10:31:28 -0500 Subject: [BLML] Naive question about double shots In-Reply-To: References: <07E347798694429AB3338FF7BBA87E37@erdos> Message-ID: <07D0CADA-6A43-415E-AB2C-0BC64452B0D5@starpower.net> On Jan 24, 2011, at 8:13 PM, Jerry Fusselman wrote: > On Mon, Jan 24, 2011 at 6:47 PM, David Grabiner wrote: > >> "Eric Landau" writes: >> >>> David gives an excellent example of an alleged double shot that >>> *would* not be allowed, but fails in the far more difficult (some >>> would say impossible) task posed by Jerry's question of justifying >>> why it *should* not be allowed. >> >> The reason I believe it should not be allowed is that it is an >> attempt to abuse >> the penalty structure. > > Interesting. Do you mean we have to read minds to find that a call is > a double shot? Yes, indeed we do. > I had not noticed that possibility before. > >> North made an abnormal bid when the Laws could have >> protected him from the risk of the bid. > > How does North know this? Has he been assured this somehow, or is it > only ex post knowledge that we imagine that he had ex ante? He needn't "know" it; he need only calculate that the odds are in his favor. >> However, we don't need to allege the double shot to deny >> adjustment. It >> suffices that the TD could rule that N-S were damaged by North's >> mistake rather >> than by West's infraction. Suppose that North held Jxx Kxx xxx >> Qxxx but had the >> CQ in with his spades. With the hand he thought he had, 3S would >> be a >> reasonable bid. But the TD would still rule that N-S were damaged >> by North's >> bad bidding; West's infraction created a situation in which normal >> play would >> give N-S -130 rather than -200, and then North made a serious >> error and should >> bear its consequences. > > Ah, it sounds like you disagree with Eric and would be willing (in > some cases) to call 3S a serious error. Interesting. How would you > find out if this was the case? Determinations of egregious error ("serious error", per the WBF minute) or of double shot ("wild or gambling action") are two entirely different things, based on different criteria, and unrelated. Eric Landau 1107 Dale Drive Silver Spring MD 20910 ehaa at starpower.net From JffEstrsn at aol.com Tue Jan 25 16:33:21 2011 From: JffEstrsn at aol.com (Jeff Easterson) Date: Tue, 25 Jan 2011 16:33:21 +0100 Subject: [BLML] Naive question about double shots In-Reply-To: References: <327C53E4-386A-40BF-9871-4071863E8144@starpower.net> Message-ID: <4D3EED41.90903@aol.com> I do like "gamboling" double. And happy I noticed it before Grattan did. Ciao, JE Am 25.01.2011 02:05, schrieb Jerry Fusselman: > On Mon, Jan 24, 2011 at 2:05 PM, Eric Landau wrote: >> On Jan 24, 2011, at 9:23 AM, Jerry Fusselman wrote: >> >>> [Steve Willner] >>> There's a long history of thinking this is a bad thing, but I think >>> Jerry's question was _why_ it's bad. >>> >>> [Jerry] >>> Right. That's my main question. I also want to know what constitutes >>> a double shot. >>> >>> By the way, I think Marv would say OS should get +130, and Adam >>> Wildavsky would say OS should get +200. And Eric would say NOS should >>> get the negative of OS's score on the grounds that 3S is nowhere near >>> egregious. >> Well, no, I wouldn't say that. Not here. > Thanks so much for the clarification, and I apologize for getting your > view mostly wrong. > >> So-called "self-inflicted damage" can arise from *either* an >> egregious error or an apparent double shot. These are independent >> determinations requiring totally different sorts of findings. Jerry >> is quite right insofar as I would vociferously object if my fellow >> committee members were considering denying redress on the grounds >> that bidding 3S was an egregious error. But denying redress on the >> grounds that 3S was a deliberate double shot doesn't require any >> finding of error at all, egregious or otherwise. >> > Alright, but I still want to know what defines a double shot. I guess > we are talking about > Law 12C1b: "If, subsequent to the irregularity, the non-offending side has > contributed to its own damage by a serious error (unrelated to the > infraction) or by wild or gambling action it does not receive relief in the > adjustment for such part of the damage as is self-inflicted." The > term double shot is undefined in the laws. > > Alright, I imagine that no one on BLML can define it. Fine. Perhaps > someone can answer a few questions to help me decide what it means. > Can the best bid ever be called a double shot? Can a good bid that is > nearly the best be called a double shot? What frame of mind do you > need to protect yourself from being told your action was a double > shot? > > The EBU white book has the phrase "double shot" nine times. Here are the cases: > > 1. "Gamboling double" as opposed to the sure-thing doubles, I guess. > And this would only explain what constitutes a double-shot double. > Perhaps one that is less than x% likely to work out well in some > sense? > > 2. "suggestion of a double shot" which leaves "double shot" undefined. > > 3. "no question of a double shot since they did not know there was an > infraction" Great, but that is hardly a definition. This merely > gives us one case of what is not a double shot. I would like to note, > however, that the NOS rarely know for sure if there is an infraction: > Perhaps the director will rule that there was no LA to the action > taken. Seems to me that gamboling actions are always dangerous in > this light. So why do we even bother with a concept of double shots? > Anyway, on with the list. > > 4 and 5. "if there is an element of a double shot...." which leaves > it undefined. > > 6. "apparent double shot attempt" which leaves it undefined. > Apparent to whom? > > 7. describes an example of a double shot as "a stupid sacrifice ... > with no possible justification." That sounds to me like a serious > error, so one hardly needs the doctrine of "double shots" to handle > it. I think we can therefore safely ignore this reference. > > 8. "What is commonly termed a ?double shot? is a gambling action > within the meaning of Law 12C1B." Totally circular, and therefore of > no help. > > 9. An index reference to 4 and 5. > > If you can tell what a double shot is from that, you must be smarter > than me. Please let me know what it is, and I promise to try to learn > it. > > Jerry Fusselman > _______________________________________________ > Blml mailing list > Blml at rtflb.org > http://lists.rtflb.org/mailman/listinfo/blml > From ehaa at starpower.net Tue Jan 25 16:47:02 2011 From: ehaa at starpower.net (Eric Landau) Date: Tue, 25 Jan 2011 10:47:02 -0500 Subject: [BLML] Unsure In-Reply-To: <4D3E44DB.8080802@nhcc.net> References: <586818.75337.qm@web28510.mail.ukl.yahoo.com><4D3704A8.8090006@skynet.be><128171.88697.qm@web25402.mail.ukl. yahoo.com> <4D382CB9.2060700@ulb.ac.be><6410CABD-9AF3-45A7-9B10-F5EDC78C6095@starpower. net> <4D3852CE.6080908@ulb.ac.be> <4D386625.90105@meteo.fr><4D3869DD.9030708@ulb.ac.be> <4D3873A6.5070104@meteo.fr><107026.33279.qm@web28515.mail.ukl.yahoo.com><4D39BC9E.9040108@nhcc.net><1820574630.137446.1295675648497.JavaMail.ngmail@webmail11.arcor-online.net><4D3D7134.4000803@ulb.ac.be><431431.44115.qm@web28515.mail.ukl.yahoo.com><4D3D7F51.70805@ulb.ac.be> <395644.68863.qm@web28514.mail.ukl.yahoo.com> <43FB511CB5684DA490AAB2D8E8666177@Lounge> <27919.7685.qm@web28506.mail.ukl.yahoo.com> <4D3E44DB.8080802@nhcc.net> Message-ID: <475904BC-C49C-4029-B585-EAF13C15D851@starpower.net> On Jan 24, 2011, at 10:34 PM, Steve Willner wrote: > On 1/24/2011 7:52 PM, Nigel Guthrie wrote: > >> For the typical player, the most frequent scenario is that he >> probably did agree >> a meaning > > Then that meaning is what the opponents are entitled to. > >> but he is unsure what it is. > > If a player does not fulfill his legal duties, his opponents are > protected from any resulting damage. The reason for not fulfilling > those duties is irrelevant. > >> According to some law-makers and BLML directors, when a player is >> genuinely >> unsure, he must not guess. > > I think Nigel is confusing two different situations. What most people > have been writing about is when you have no agreement or have > contradictory or ambiguous agreements. In such cases, you tell the > opponents the agreements you do have, and everyone guesses what the > player actually has. > > Nigel's "most frequent scenario" (not all that frequent in my > experience), where a player knows he has forgotten, is a different > matter entirely. > >> What then, should a typical player, who is 75% sure >> say to opponents? > > Probably best just to state the agreement you think is correct. If > you > get it right, no problem. If not, partner has UI, and opponents have > MI, and you're probably making the wrong call anyway. Good luck! But > expressing uncertainty changes nothing except to give partner more UI > and opponents unnecessary AI. > > Alternatives are to refer opponents to your system card or call the TD > and let partner explain with you out of earshot, but those have > disadvantages of their own. (Referring them to the SC is probably > OK if > it's a very simple question such as which version of Blackwood you > play.) > > Why is this so complicated? (1) Because Nigel is far from the only one who is confusing the two different situations. (2) Because in real life, making an objective determination as to which one it is may require an intensive examination of the partnership's methods, which may be impractical or even impossible (so that the TD/AC are instead forced to resort to "mind reading"). (3) Because it would be nice (although perhaps not possible) to give some kind of unambiguous guidance as to how a player should act when he is sufficiently confused not to be certain which one he is in. Eric Landau 1107 Dale Drive Silver Spring MD 20910 ehaa at starpower.net From blml at arcor.de Tue Jan 25 16:50:14 2011 From: blml at arcor.de (Thomas Dehn) Date: Tue, 25 Jan 2011 16:50:14 +0100 (CET) Subject: [BLML] Exam question In-Reply-To: <4D3EEBDD.1070908@aol.com> References: <4D3EEBDD.1070908@aol.com> <4D3D3B2F.5090107@skynet.be> <1PhIDl-0dAMts0@fwd00.aul.t-online.de> <4D3E0908.5090009@skynet.be> <82D7A2DF-92A6-495E-ABC1-6C5C27F82B32@starpower.net> Message-ID: <1022305150.53310.1295970614985.JavaMail.ngmail@webmail12.arcor-online.net> Jeff Easterson > Here we go again. Herman poses a question and receives numerous > replies. He is not satisfied with the opinions expressed (because they > are different from his) so we shall start a long boring discussion with > constant repetition of the same arguments. The contrasting opinions > have been clearly stated. Is it not possible to accept that? I wonder > if Herman might have talents as a missionary. > And, incidentally, it seems unusual, superfluous and bordering on > showmanship to show two cards and tell the opponents they are high > although not intending to claim. Why not play bridge as everyone else > does and dispense with theatrical gestures? Or was it simply an attempt > to impress the opponents? We need to investigate how exactly Herman showed the two cards so that we can decide whether he played two cards to the same trick :-) Thomas From JffEstrsn at aol.com Tue Jan 25 16:51:50 2011 From: JffEstrsn at aol.com (Jeff Easterson) Date: Tue, 25 Jan 2011 16:51:50 +0100 Subject: [BLML] Exam question In-Reply-To: <4D3EE3E8.70308@skynet.be> References: <4D3E9ACD.5020602@skynet.be> <1Phfff-2FiDdA0@fwd06.aul.t-online.de> <4D3EE3E8.70308@skynet.be> Message-ID: <4D3EF196.50205@aol.com> This is the sort of thing I dislike and I don't think itis appropriate in blml discussions. Herman poses a question and receives numerous answers including one from Peter Eidt. With the exception of Jerry Fusselman all disagree with Herman (as far as I can recall) and Jerry seems to always support Herman. (Sight unseen? - please excuse the irony, I don't think it is helpful.) Peter's response is neutrally stated. Herman disagrees so he writes "the great Peter Eidt" and "the lowly Herman". Is this irony helpful? Does it further the discussion? Incidentally, I know both of them fairly well and, in my opinion, as far as self-characterisation is concerned, the opposite (great Herman and lowly - at least modest- Peter) would seem true. Ciao, JE Am 25.01.2011 15:53, schrieb Herman De Wael: > Peter Eidt wrote: >> What do you want us to do, Herman? >> >> You asked an exam question, giving us (hopefully) >> the facts. >> The unanimous (execpt Mr. Fusselman) answer >> was: it was a claim - play ceased. >> So you should expect when giving your answer >> "Declarer did not claim and may play on" to get >> zero points for this. >> >> Now you are mixing up all counter-arguments >> bringing in your own language and tell us, we all >> are wrong because the facts were not as stated. >> You told us declarer showed Ace and King of >> trumps just to tell opponents he has them and >> in order to be nice (and smart?) to them - as if >> there wouldn't have been aware of that fact ... >> >> ts ts ts >> > Well, Peter, what do you want me to do? > I ask a question, believing to know the answer. > You tell me another answer. > Now of course the answer of the great Peter Eidt is the correct one, and > that of lowly Herman De Wael must be wrong. > > ts ts ts. > > So I ask you again, Peter: what is the meaning of the word "specific" in > L68A? > > From ehaa at starpower.net Tue Jan 25 17:07:55 2011 From: ehaa at starpower.net (Eric Landau) Date: Tue, 25 Jan 2011 11:07:55 -0500 Subject: [BLML] Unsure In-Reply-To: <4D3E8368.40706@skynet.be> References: <000901cbba0f$ce55b3f0$6b011bd0$@no><586818.75337.qm@web28510.mail.ukl.yahoo.com> <4D3704A8.8090006@skynet.be> <128171.88697.qm@web25402.mail.ukl. yahoo.com> < 4D382CB9.2060700@ulb.ac.be><6410CABD-9AF3-45A7-9B10-F5EDC78C6095@starpowe r. net><4D3852CE.6080908@ulb.ac.be> <4D386625.90105@meteo.fr> <4D3869DD.9030708@ulb.ac.be> <4D3873A6.5070104@meteo.fr> <107026.33279.qm@web28515.mail.ukl.yahoo.com> <4D39BC9E.9040108@nhcc.net> <1820574630.137446.1295675648497.JavaMail.ngmail@webmail11.arcor-online.net><4D3A93A3.8020906@skynet.be><1896834273.57934.1295686896979.JavaMail.ngmail@webmail13.arcor-online.net> <514721598.171618.1295725647449.JavaMail.ngmail@webmail10.arcor-online.net> <000901cbba82$f84fdf50$e8ef9df0$@no> <4D3E8368.40706@skynet.be> Message-ID: <527437D4-1F0D-456D-A00D-A633863A8627@starpower.net> On Jan 25, 2011, at 3:01 AM, Herman De Wael wrote: > Eric Landau wrote: > >> For the last forty-plus years, bridge has been getting consistently >> more "scientific". Systems and methods have gotten consistently more >> complex and detailed. Today's top-level partnerships have extensive >> agreements covering the vast majority of situations, often requiring >> hundreds of pages of notes to specify. >> >> But fifty years ago, even the highest-level partnerships got by with >> a relative handful of basic agreements covering opening bids, basic >> responses and rebids, and a few simple "artifical" conventions (e.g. >> Blackwood, Stayman), along with a general understanding of how they >> bid (what we would call "meta-agreements"). Beyond that, pretty much >> everything else was "no agreement" or "undiscussed". Despite their >> lack of agreements, they still managed most of the time to work out a >> reasonable auction; this was generally considered to be a strong >> indication that they knew a little something about how to play >> bridge. > > You are right, but I think you draw the wrong conclusion. The only conclusion I intended to offer was that managing most of the time to work out a reasonable auction was not, as was alleged, a strong indication of a CPU. > Fifty years > ago, when someone said "undiscussed", he assumed that his opponents > knew > as much as he did, and he was usually right. Today, the word > "undiscussed" really means "not discussed specifically, we shall apply > our default understandings". In some partnerships, those defaults will > NOT be known by the opponents. Then, the partnership will know more > than > the opponents, and that is MI. This may be true, but I'm an optimist. I prefer to think that when an ethical player claims that a call is totally undefined it is totally undefined, although I agree that this is extremely rare. I also prefer to think that if the reality is "not discussed specifically, we shall apply our default understandings", that is what gets said, and, moreover, gets followed by "which are..." Herman is exactly right to say that if those defaults are not known by the opponents that is MI. When the "undiscussed" call is inquired about, full disclosure requires disclosing any default understandings that may be relevant to deciding on the meaning of the call. That levels the playing field and lets the opponents draw their own conclusions from the same set of facts you will use to draw yours. Full disclosure does not, however, require you to additionally share those conclusions. Eric Landau 1107 Dale Drive Silver Spring MD 20910 ehaa at starpower.net From Hermandw at skynet.be Tue Jan 25 18:03:01 2011 From: Hermandw at skynet.be (Herman De Wael) Date: Tue, 25 Jan 2011 18:03:01 +0100 Subject: [BLML] Exam question In-Reply-To: <82D7A2DF-92A6-495E-ABC1-6C5C27F82B32@starpower.net> References: <4D3D3B2F.5090107@skynet.be> <1PhIDl-0dAMts0@fwd00.aul.t-online.de> <4D3E0908.5090009@skynet.be> <82D7A2DF-92A6-495E-ABC1-6C5C27F82B32@starpower.net> Message-ID: <4D3F0245.9010808@skynet.be> Eric Landau wrote: > On Jan 24, 2011, at 7:21 PM, Jerry Fusselman wrote: > >> On Mon, Jan 24, 2011 at 5:19 PM, Herman De Wael >> wrote: >> >>> Is no-one noticing the word "specific" number here? I did not say >>> I was >>> going to make 8 tricks - I was clearly saying 8 or more. >> >> I agree, and that's what I said, but I was the only one. Did I pass >> the exam question? > > No, because that's not the issue. There are two sentences in L68A. > When Herman deliberately exposed the trump AK, he already claimed > (even had he said nothing) "unless he demonstrably did not intend > to". That he subsequently stated the obvious out loud -- that the > trump AK would both take tricks -- doesn't affect the determination > of his demonstrable intention, hence doesn't matter. > There are three parts of L68, any of which makes it a claim a) I did not mention a specific number of tricks (only 8 or more) b) I did not suggest play be curtailed c) I did not show my cards (only two of them) How you can draw from my story that I did either of these is beyond me. -- Herman De Wael Wilrijk Antwerpen Belgium From Hermandw at skynet.be Tue Jan 25 18:04:54 2011 From: Hermandw at skynet.be (Herman De Wael) Date: Tue, 25 Jan 2011 18:04:54 +0100 Subject: [BLML] Exam question In-Reply-To: <4D3EF196.50205@aol.com> References: <4D3E9ACD.5020602@skynet.be> <1Phfff-2FiDdA0@fwd06.aul.t-online.de> <4D3EE3E8.70308@skynet.be> <4D3EF196.50205@aol.com> Message-ID: <4D3F02B6.30808@skynet.be> Jeff Easterson wrote: > This is the sort of thing I dislike and I don't think itis appropriate > in blml discussions. Herman poses a question and receives numerous > answers including one from Peter Eidt. With the exception of Jerry > Fusselman all disagree with Herman (as far as I can recall) and Jerry > seems to always support Herman. (Sight unseen? - please excuse the > irony, I don't think it is helpful.) Peter's response is neutrally > stated. Herman disagrees so he writes "the great Peter Eidt" and "the > lowly Herman". Is this irony helpful? Does it further the discussion? > Incidentally, I know both of them fairly well and, in my opinion, as far > as self-characterisation is concerned, the opposite (great Herman and > lowly - at least modest- Peter) would seem true. Ciao, JE > and how do you categorize a sentence like: ts ts ts in a post in which Peter tells me that since he said so, I am wrong and should accept that? > Am 25.01.2011 15:53, schrieb Herman De Wael: >> Peter Eidt wrote: >>> What do you want us to do, Herman? >>> >>> You asked an exam question, giving us (hopefully) >>> the facts. >>> The unanimous (execpt Mr. Fusselman) answer >>> was: it was a claim - play ceased. >>> So you should expect when giving your answer >>> "Declarer did not claim and may play on" to get >>> zero points for this. >>> >>> Now you are mixing up all counter-arguments >>> bringing in your own language and tell us, we all >>> are wrong because the facts were not as stated. >>> You told us declarer showed Ace and King of >>> trumps just to tell opponents he has them and >>> in order to be nice (and smart?) to them - as if >>> there wouldn't have been aware of that fact ... >>> >>> ts ts ts >>> >> Well, Peter, what do you want me to do? >> I ask a question, believing to know the answer. >> You tell me another answer. >> Now of course the answer of the great Peter Eidt is the correct one, and >> that of lowly Herman De Wael must be wrong. >> >> ts ts ts. >> >> So I ask you again, Peter: what is the meaning of the word "specific" in >> L68A? >> >> > > _______________________________________________ > Blml mailing list > Blml at rtflb.org > http://lists.rtflb.org/mailman/listinfo/blml > > > > > No virus found in this incoming message. > Checked by AVG - www.avg.com > Version: 9.0.872 / Virus Database: 271.1.1/3401 - Release Date: 01/24/11 20:34:00 > -- Herman De Wael Wilrijk Antwerpen Belgium From jfusselman at gmail.com Tue Jan 25 18:08:43 2011 From: jfusselman at gmail.com (Jerry Fusselman) Date: Tue, 25 Jan 2011 11:08:43 -0600 Subject: [BLML] Exam question In-Reply-To: <4D3EF196.50205@aol.com> References: <4D3E9ACD.5020602@skynet.be> <1Phfff-2FiDdA0@fwd06.aul.t-online.de> <4D3EE3E8.70308@skynet.be> <4D3EF196.50205@aol.com> Message-ID: On Tue, Jan 25, 2011 at 9:51 AM, Jeff Easterson wrote: > Jerry seems to always support Herman. Nonsense, even in this thread. I gave my answer before Herman gave his. As declarer, Herman exposes two cards, and makes a joke, not a claim. I got the joke, and I am surprised that so many others would not. Had the first name of the declarer in his example been different, it seems likely to me more would had answered like I did. Anyone interested in evidence? It should be pretty easy to poll several players whether they understand this case to be one of 8 or 8+ and whether they are calling the cops or playing on. I predict most players would play on, understanding the statement was for 8+. Especially the players with the best sense of humor will play on. It might be dangerous with grim players at the table, but most players know that declarer can expose cards without penalty, and that such exposed cards can help the hand go faster. This is the case for the poll: Your opponent was playing 2He. He had just won the first six tricks, and he showed AK of hearts, saying "I think I am going to make this contract". Is this a claim for specifically 8? Do you call the director? Or do you play on? Anyone care to try this poll on players who do not follow BLML and tell us the results? Jerry Fusselman From agot at ulb.ac.be Tue Jan 25 18:34:56 2011 From: agot at ulb.ac.be (Alain Gottcheiner) Date: Tue, 25 Jan 2011 18:34:56 +0100 Subject: [BLML] Exam question In-Reply-To: References: <4D3E9ACD.5020602@skynet.be> <1Phfff-2FiDdA0@fwd06.aul.t-online.de> <4D3EE3E8.70308@skynet.be> <4D3EF196.50205@aol.com> Message-ID: <4D3F09C0.3010803@ulb.ac.be> Le 25/01/2011 18:08, Jerry Fusselman a ?crit : > On Tue, Jan 25, 2011 at 9:51 AM, Jeff Easterson wrote: >> Jerry seems to always support Herman. > Nonsense, even in this thread. I gave my answer before Herman gave his. > > As declarer, Herman exposes two cards, and makes a joke, not a claim. > I got the joke, and I am surprised that so many others would not. Had > the first name of the declarer in his example been different, it seems > likely to me more would had answered like I did. Anyone interested in > evidence? It should be pretty easy to poll several players whether > they understand this case to be one of 8 or 8+ and whether they are > calling the cops or playing on. I predict most players would play on, > understanding the statement was for 8+. Especially the players with > the best sense of humor will play on. AG : notice that the question wasn't, "what does your sense of humor say ?" but rather "what do the laws say ?" Yes, I do believe that these are two different answers. > It might be dangerous with grim > players at the table, but most players know that declarer can expose > cards without penalty, and that such exposed cards can help the hand > go faster. Indeed, and that's calles claiming. > This is the case for the poll: > > Your opponent was playing 2He. He had just won the first six tricks, and > he showed AK of hearts, saying "I think I am going to make this > contract". Is this a claim for specifically 8? Do you call the > director? Or do you play on? AG : once again, that's a mix. The question isn't whether most would call the Td, but how the TD should act if called. From ehaa at starpower.net Tue Jan 25 18:52:03 2011 From: ehaa at starpower.net (Eric Landau) Date: Tue, 25 Jan 2011 12:52:03 -0500 Subject: [BLML] Exam question In-Reply-To: <4D3EE3E8.70308@skynet.be> References: <4D3E9ACD.5020602@skynet.be> <1Phfff-2FiDdA0@fwd06.aul.t-online.de> <4D3EE3E8.70308@skynet.be> Message-ID: <74D61212-85D5-42B3-9583-FFE26C078352@starpower.net> On Jan 25, 2011, at 9:53 AM, Herman De Wael wrote: > Peter Eidt wrote: > >> What do you want us to do, Herman? >> >> You asked an exam question, giving us (hopefully) >> the facts. >> The unanimous (execpt Mr. Fusselman) answer >> was: it was a claim - play ceased. >> So you should expect when giving your answer >> "Declarer did not claim and may play on" to get >> zero points for this. >> >> Now you are mixing up all counter-arguments >> bringing in your own language and tell us, we all >> are wrong because the facts were not as stated. >> You told us declarer showed Ace and King of >> trumps just to tell opponents he has them and >> in order to be nice (and smart?) to them - as if >> there wouldn't have been aware of that fact ... >> >> ts ts ts > > Well, Peter, what do you want me to do? > I ask a question, believing to know the answer. > You tell me another answer. > Now of course the answer of the great Peter Eidt is the correct > one, and > that of lowly Herman De Wael must be wrong. > > ts ts ts. > > So I ask you again, Peter: what is the meaning of the word > "specific" in > L68A? There is a TV show in the US called "The McLaughlin Group", a weekly panel discussion of American politics. It is hosted by John McLaughlin, who appears with four other panelists. It is famous for the following interaction, which occurs with considerable frequency: Mr. McLaughlin poses a question to the panel; each of the four panelists, in turn, gives essentially the same answer. Mr. McLaughlin then announces (his signiture phrase), "The answer is...," and proceeds to expound an opinion diametrically opposite to the unanimous opinion of his panelists. Of course, when Mr. McLaughlin does this, he understands that he is making a joke. Eric Landau 1107 Dale Drive Silver Spring MD 20910 ehaa at starpower.net From ehaa at starpower.net Tue Jan 25 19:13:14 2011 From: ehaa at starpower.net (Eric Landau) Date: Tue, 25 Jan 2011 13:13:14 -0500 Subject: [BLML] Unsure In-Reply-To: <000b01cbbc9f$a7b1a410$f714ec30$@no> References: <586818.75337.qm@web28510.mail.ukl.yahoo.com> <4D3704A8.8090006@skynet.be> <128171.88697.qm@web25402.mail.ukl. yahoo.com> < 4D382CB9.2060700@ulb.ac.be><6410CABD-9AF3-45A7-9B10-F5EDC78C6095@starpowe r. net><4D3852CE.6080908@ulb.ac.be> <4D386625.90105@meteo.fr> <4D3869DD.9030708@ulb.ac.be> <4D3873A6.5070104@meteo.fr> <107026.33279.qm@web28515.mail.ukl.yahoo.com> <4D39BC9E.9040108@nhcc.net> <1820574630.137446.1295675648497.JavaMail.ngmail@webmail11.arcor-online.net> <4D3D7134.4000803@ulb.ac.be> <431431.44115.qm@web28515.mail.ukl.yahoo.com> <4D3D7F51.70805@ulb.ac.be> <395644.68863.qm@web28514.mail.ukl.yahoo.com><000101cbbbcf$4f8dd270$eea97750$@no> <011301cbbbcf$fc8610a0$f59231e0$@nl><4A3418FD24684B579A369D685BF6D6FB@Lounge> <011401cbbbd6$4e621f80$eb265e80$@nl> <000101cbbbdb$0a0717a0$1e1546e0$@no> <670306DF0C5446E8B8770FE884FA6E1C@Lounge> <000801cbbbe6$886147a0$9923! d 6e0$@no> <297889D5-06 05-4FF8-AB20-AE02FE64BE6D@starpower.net> <000b01cbbc9f$a7b1a410$f714ec30$@no> Message-ID: <24CD765F-7CD6-44AF-998D-569D2C5463D4@starpower.net> On Jan 25, 2011, at 9:53 AM, Sven Pran wrote: > On Behalf Of Eric Landau > >>> So you missed the possibly most important sentence in Law 75? >>> >>> the Director is to presume Mistaken Explanation, rather than >>> Mistaken >>> Call, in the absence of evidence to the contrary >> >> That says that if the director cannot decide between mistaken >> explanation (ME) >> and mistaken call (MC), he is to presume the former. In David's >> scenario, there is >> no possibility of an MC; the director's determination is either ME >> (if he deems >> there was MI) or no irregularity (if he deems the explanation to >> be correct). The >> "possibly most important sentence in L75" (not that I would agree) >> is entirely >> irrelevant here. > > The explanation cannot be correct unless it explains the meaning of > the > call. That's just wrong. There is no obligation to explain "meaning" per se. The required explanation is of "all special information conveyed to him through partnership agreement or partnership experience" [L40B6 (a)] that may be "crucial for an opponent's choice of action" [L40B6 (b)]. Of course, that's the same thing as "meaning" *if* -- big if! -- partnership agreement or partnership experience have resulted in an agreed-upon meaning, but the law does not explicitly presume that it must do so in every case. > If no evidence of an agreement (whether explicit or implied) can be > presented the Director shall rule that the call corresponds with some > features of the hand held by the player making that call and that such > correspondence represents the meaning of the call. This is the > essence of > the clause: "the Director is to presume Mistaken Explanation, > rather than > Mistaken Call, in the absence of evidence to the contrary" > > An "explanation" that the player does not know the meaning of the > call is > not an explanation of the meaning of the call but rather an > explanation of > the reason why he cannot comply with the requirements of the law. > The reason > he might give for this is irrelevant, the fact is that he has not > given the > required explanation of the call. Sven interprets the quoted clause as though it actually read, "the Director is to presume mistaken explanation, rather than anything else, in the absence of evidence to the contrary." But that's not what it says. Eric Landau 1107 Dale Drive Silver Spring MD 20910 ehaa at starpower.net From JffEstrsn at aol.com Tue Jan 25 19:17:46 2011 From: JffEstrsn at aol.com (Jeff Easterson) Date: Tue, 25 Jan 2011 19:17:46 +0100 Subject: [BLML] Exam question In-Reply-To: <4D3F02B6.30808@skynet.be> References: <4D3E9ACD.5020602@skynet.be> <1Phfff-2FiDdA0@fwd06.aul.t-online.de> <4D3EE3E8.70308@skynet.be> <4D3EF196.50205@aol.com> <4D3F02B6.30808@skynet.be> Message-ID: <4D3F13CA.1070503@aol.com> ts ts ts: fairly neutral, not an insult. It means he disagrees and doesn't think your argument holds water. It appears that he is not alone in that opinion. I doubt that constant repetition will be very helpful. You have stated your opinion, and others have disagreed. Is it necessary to pursue the argument further unless you can introduce new facts? You have stated your opinion. We have carefully read it. That ought to be enough. Next case. Peter did not say "since he said so, I am wrong...." Why do you claim he did? This does not help. It is a red herring. Although your English is pretty good (and seems to have improved greatly in the past few years) you are still not a native speaker. That might help to explain your problem with "specific". Ciao, JE Am 25.01.2011 18:04, schrieb Herman De Wael: > Jeff Easterson wrote: >> This is the sort of thing I dislike and I don't think itis appropriate >> in blml discussions. Herman poses a question and receives numerous >> answers including one from Peter Eidt. With the exception of Jerry >> Fusselman all disagree with Herman (as far as I can recall) and Jerry >> seems to always support Herman. (Sight unseen? - please excuse the >> irony, I don't think it is helpful.) Peter's response is neutrally >> stated. Herman disagrees so he writes "the great Peter Eidt" and "the >> lowly Herman". Is this irony helpful? Does it further the discussion? >> Incidentally, I know both of them fairly well and, in my opinion, as far >> as self-characterisation is concerned, the opposite (great Herman and >> lowly - at least modest- Peter) would seem true. Ciao, JE >> > and how do you categorize a sentence like: > > ts ts ts > > in a post in which Peter tells me that since he said so, I am wrong and > should accept that? > >> Am 25.01.2011 15:53, schrieb Herman De Wael: >>> Peter Eidt wrote: >>>> What do you want us to do, Herman? >>>> >>>> You asked an exam question, giving us (hopefully) >>>> the facts. >>>> The unanimous (execpt Mr. Fusselman) answer >>>> was: it was a claim - play ceased. >>>> So you should expect when giving your answer >>>> "Declarer did not claim and may play on" to get >>>> zero points for this. >>>> >>>> Now you are mixing up all counter-arguments >>>> bringing in your own language and tell us, we all >>>> are wrong because the facts were not as stated. >>>> You told us declarer showed Ace and King of >>>> trumps just to tell opponents he has them and >>>> in order to be nice (and smart?) to them - as if >>>> there wouldn't have been aware of that fact ... >>>> >>>> ts ts ts >>>> >>> Well, Peter, what do you want me to do? >>> I ask a question, believing to know the answer. >>> You tell me another answer. >>> Now of course the answer of the great Peter Eidt is the correct one, and >>> that of lowly Herman De Wael must be wrong. >>> >>> ts ts ts. >>> >>> So I ask you again, Peter: what is the meaning of the word "specific" in >>> L68A? >>> >>> >> _______________________________________________ >> Blml mailing list >> Blml at rtflb.org >> http://lists.rtflb.org/mailman/listinfo/blml >> >> >> >> >> No virus found in this incoming message. >> Checked by AVG - www.avg.com >> Version: 9.0.872 / Virus Database: 271.1.1/3401 - Release Date: 01/24/11 20:34:00 >> From ehaa at starpower.net Tue Jan 25 19:19:42 2011 From: ehaa at starpower.net (Eric Landau) Date: Tue, 25 Jan 2011 13:19:42 -0500 Subject: [BLML] Unsure In-Reply-To: <4D3EE5B3.1030006@skynet.be> References: <586818.75337.qm@web28510.mail.ukl.yahoo.com> <4D3704A8.8090006@skynet.be> <128171.88697.qm@web25402.mail.ukl. yahoo.com> < 4D382CB9.2060700@ulb.ac.be><6410CABD-9AF3-45A7-9B10-F5EDC78C6095@starpowe r. net> <4D3852CE.6080908@ulb.ac.be> <4D386625.90105@meteo.fr> <4D3869DD.9030708@ulb.ac.be> <4D3873A6.5070104@meteo.fr> <107026.33279.qm@web28515.mail.ukl.yahoo.com> <4D39BC9E.9040108@nhcc.net> <1820574630.137446.1295675648497.JavaMail.ngmail@webmail11.arcor-online.net> <4D3D7134.4000803@ulb.ac.be> <431431.44115.qm@web28515.mail.ukl.yahoo.com> <4D3D7F51.70805@ulb.ac.be> <395644.68863.qm@web28514.mail.ukl.yahoo.com> <000101cbbbcf$4f8dd270$eea97750$@no> <1868019136.66657.1295900769160.JavaMail.ngmail@webmail08.arcor-online.net> <532598E0-1D81-4AA2-84BA-3EC0BE2D2843@starpower.net> <4D3EE5B3.1030006@skynet.be> Message-ID: <370FDAF7-DB79-4D9B-99A9-ADCF9F66E990@starpower.net> On Jan 25, 2011, at 10:01 AM, Herman De Wael wrote: > Eric Landau wrote: > >> He expects me to work it out based on our meta-agreements and my >> understanding of our general style. My opponents are entitled to the >> same knowledge I have as to the systemic meaning of partner's bid. >> If they ask, I explain that we have no agreement about this >> particular auction, but..., and then proceed to disclose any meta- >> agreements or elements of our general style that might be in any way >> relevant to figuring out what partner's bid means. I have "leveled >> the playing field" to the best of my ability; from here there's only >> "bridge logic". I have no obligation to explain any conclusion (or >> lack thereof) that my bridge logic may have led me to, nor even, for >> that matter, any reason to think that my deductions are any more >> valid than what my opponents may have been able to deduce from the >> same facts. > > In short, you rely on General Bridge Knowledge. > And then they draw some conclusions, and those are not the same > conclusions you took, and not the same ones your partner took, whereas > you two took the same ones. > Apparently then, your general bridge knowledge wasn't so General. > I rule MI, and obfuscation to boot. Huh? I think Herman may have confused my posts with someone else's. In short, I rely on my meta-agreements and my understanding of our general style, which I disclose. Just as I wrote. Who (besides Herman) has suggested bringing "general bridge knowledge" into this discussion? Eric Landau 1107 Dale Drive Silver Spring MD 20910 ehaa at starpower.net From ehaa at starpower.net Tue Jan 25 19:26:55 2011 From: ehaa at starpower.net (Eric Landau) Date: Tue, 25 Jan 2011 13:26:55 -0500 Subject: [BLML] Naive question about double shots In-Reply-To: <4D3EED41.90903@aol.com> References: <327C53E4-386A-40BF-9871-4071863E8144@starpower.net> <4D3EED41.90903@aol.com> Message-ID: On Jan 25, 2011, at 10:33 AM, Jeff Easterson wrote: > I do like "gamboling" double. And happy I noticed it before Grattan > did. Ciao, JE Well, I once had an opponent climb up onto his chair, thrust both his arms in the air (as if signaling "touchdown") and then double me, so I guess that would qualify. Not surprisingly, though, that particular double was about as far from "wild or gambling" as you can get. Eric Landau 1107 Dale Drive Silver Spring MD 20910 ehaa at starpower.net From jfusselman at gmail.com Tue Jan 25 19:32:07 2011 From: jfusselman at gmail.com (Jerry Fusselman) Date: Tue, 25 Jan 2011 12:32:07 -0600 Subject: [BLML] Exam question In-Reply-To: <4D3F13CA.1070503@aol.com> References: <4D3E9ACD.5020602@skynet.be> <1Phfff-2FiDdA0@fwd06.aul.t-online.de> <4D3EE3E8.70308@skynet.be> <4D3EF196.50205@aol.com> <4D3F02B6.30808@skynet.be> <4D3F13CA.1070503@aol.com> Message-ID: On Tue, Jan 25, 2011 at 12:17 PM, Jeff Easterson wrote: > > Although your English is pretty good (and seems to have improved greatly > in the past few years) you are still not a native speaker. ?That might > help to explain your problem with "specific". > specific: "stated explicitly or in detail." I don't think Herman has any trouble with the word. Specific would be a phrase like "I will take two more and give you the rest." Jerry Fusselman From ehaa at starpower.net Tue Jan 25 19:47:58 2011 From: ehaa at starpower.net (Eric Landau) Date: Tue, 25 Jan 2011 13:47:58 -0500 Subject: [BLML] Exam question In-Reply-To: <4D3F0245.9010808@skynet.be> References: <4D3D3B2F.5090107@skynet.be> <1PhIDl-0dAMts0@fwd00.aul.t-online.de> <4D3E0908.5090009@skynet.be> <82D7A2DF-92A6-495E-ABC1-6C5C27F82B32@starpower.net> <4D3F0245.9010808@skynet.be> Message-ID: <18090296-A6E9-4502-B8D1-2863FD6C7EC5@starpower.net> On Jan 25, 2011, at 12:03 PM, Herman De Wael wrote: > Eric Landau wrote: > >> On Jan 24, 2011, at 7:21 PM, Jerry Fusselman wrote: >> >>> On Mon, Jan 24, 2011 at 5:19 PM, Herman De Wael >>> wrote: >>> >>>> Is no-one noticing the word "specific" number here? I did not say >>>> I was >>>> going to make 8 tricks - I was clearly saying 8 or more. >>> >>> I agree, and that's what I said, but I was the only one. Did I pass >>> the exam question? >> >> No, because that's not the issue. There are two sentences in L68A. >> When Herman deliberately exposed the trump AK, he already claimed >> (even had he said nothing) "unless he demonstrably did not intend >> to". That he subsequently stated the obvious out loud -- that the >> trump AK would both take tricks -- doesn't affect the determination >> of his demonstrable intention, hence doesn't matter. > > There are three parts of L68, any of which makes it a claim > a) I did not mention a specific number of tricks (only 8 or more) How did you manage to mention "eight or more" without ever mentioning "eight"? > b) I did not suggest play be curtailed Perhaps you didn't; I don't need to read your mind in this case. L68D "suggested" that play be curtailed. > c) I did not show my cards (only two of them) Those two cards weren't yours? > How you can draw from my story that I did either of these is beyond > me. Because you presumably intended to do something (other than gratuitously disrupt the table), and the only evidence that you did not intend to claim is that you self-servingly state that you did not intend to claim, which doesn't rise to the level of "demonstrable". Eric Landau 1107 Dale Drive Silver Spring MD 20910 ehaa at starpower.net From jfusselman at gmail.com Tue Jan 25 19:54:17 2011 From: jfusselman at gmail.com (Jerry Fusselman) Date: Tue, 25 Jan 2011 12:54:17 -0600 Subject: [BLML] Exam question In-Reply-To: <4D3F09C0.3010803@ulb.ac.be> References: <4D3E9ACD.5020602@skynet.be> <1Phfff-2FiDdA0@fwd06.aul.t-online.de> <4D3EE3E8.70308@skynet.be> <4D3EF196.50205@aol.com> <4D3F09C0.3010803@ulb.ac.be> Message-ID: Hi Alain, On Tue, Jan 25, 2011 at 11:34 AM, Alain Gottcheiner wrote: > Le 25/01/2011 18:08, Jerry Fusselman a ?crit : >> >> Anyone interested in >> evidence? ?It should be pretty easy to poll several players whether >> they understand this case to be one of 8 or 8+ and whether they are >> calling the cops or playing on. ?I predict most players would play on, >> understanding the statement was for 8+. ?Especially the players with >> the best sense of humor will play on. > > AG : notice that the question wasn't, "what does your sense of humor say ?" > but rather "what do the laws say ?" Yes, I do believe that these are two > different answers. Yes, agreed they can be different. In the Bridge In The Menagerie books, as I recall, several times the Secretary Bird calls the director with "That's a claim! That's a claim!" And sometimes the director rules that way, though it might be just for an interesting story. I am guessing that the evidence of the poll would be that almost all players realize the scenario means 8+. Perhaps we shall see. If almost all players thinks it means 8+, then that is the meaning it should be deemed to have. That's how language works. > >> ? It might be dangerous with grim >> players at the table, but most players know that declarer can expose >> cards without penalty, and that such exposed cards can help the hand >> go faster. > > Indeed, and that's calles claiming. I have several times shown a tanking defender some of my cards, and no one has yet thought it constitutes a claim. They usually thank me for it, for it helps speed up the game. I could find nowhere in the laws explicitly stating that declarer cannot have penalty cards, but it seems easy to infer it by reading all 57 references to "penalty card" in the laws. I was hoping there would be something about this kind of case in the EBU White Book, but it seems not. I would like to hear the opinions of Ton, Steve Willner, and Robert Frick on this. > >> This is the case for the poll: >> >> Your opponent was playing 2He. He had just won the first six tricks, and >> he showed AK of hearts, saying "I think I am going to make this >> contract". ?Is this a claim for specifically 8? ?Do you call the >> director? ?Or do you play on? > > AG : once again, that's a mix. The question isn't whether most would call > the Td, but how the TD should act if called. > > I imagine that a TD should rule that if declarer said A meaning X, and defenders understood X as well, then the TD should rule that X is what was communicated. A is the explicit statement, X is the meaning 8+. Jerry Fusselman From jfusselman at gmail.com Tue Jan 25 20:09:02 2011 From: jfusselman at gmail.com (Jerry Fusselman) Date: Tue, 25 Jan 2011 13:09:02 -0600 Subject: [BLML] Exam question In-Reply-To: <18090296-A6E9-4502-B8D1-2863FD6C7EC5@starpower.net> References: <4D3D3B2F.5090107@skynet.be> <1PhIDl-0dAMts0@fwd00.aul.t-online.de> <4D3E0908.5090009@skynet.be> <82D7A2DF-92A6-495E-ABC1-6C5C27F82B32@starpower.net> <4D3F0245.9010808@skynet.be> <18090296-A6E9-4502-B8D1-2863FD6C7EC5@starpower.net> Message-ID: On Tue, Jan 25, 2011 at 12:47 PM, Eric Landau wrote: > On Jan 25, 2011, at 12:03 PM, Herman De Wael wrote: >> There are three parts of L68, any of which makes it a claim >> a) I did not mention a specific number of tricks (only 8 or more) > > How did you manage to mention "eight or more" without ever mentioning > "eight"? The same way "the rest are mine" implies a specific number without ever mentioning that number. > >> b) I did not suggest play be curtailed > > Perhaps you didn't; I don't need to read your mind in this case. > L68D "suggested" that play be curtailed. No, L68D starts with the phrase "After any claim or concession, play ceases...", so it presupposes there is a claim. > >> c) I did not show my cards (only two of them) > > Those two cards weren't yours? > >> How you can draw from my story that I did either of these is beyond >> me. > > Because you presumably intended to do something (other than > gratuitously disrupt the table), and the only evidence that you did > not intend to claim is that you self-servingly state that you did not > intend to claim, which doesn't rise to the level of "demonstrable". > [Law 68A. Claim Defined] Any statement to the effect that a contestant will win a specific number of tricks is a claim of those tricks. A contestant also claims when he suggests that play be curtailed, or when he shows his cards (unless he demonstrably did not intend to claim - for example, if declarer faces his cards after an opening lead out of turn Law 54, not this Law, will apply). Herman's position is not exactly what you inferred. I think Herman means that "shows his cards" refers to showing all of his remaining cards, which he did not do, so the second sentence is not relevant here. I thought declarer showing some of his cards happens every day---does it not? As for gratuitously disrupting the table, I guess it depends on the table. I often enjoy jokes at the table; even bad jokes sometimes. A table with a friendly atmosphere can handle and even enjoy declarer showing the AK of trump and making a funny comment, but a table with a poisoned, Secretary-Birdish atmosphere cannot, I guess. Jerry Fusselman From ehaa at starpower.net Tue Jan 25 20:21:22 2011 From: ehaa at starpower.net (Eric Landau) Date: Tue, 25 Jan 2011 14:21:22 -0500 Subject: [BLML] Exam question In-Reply-To: References: <4D3E9ACD.5020602@skynet.be> <1Phfff-2FiDdA0@fwd06.aul.t-online.de> <4D3EE3E8.70308@skynet.be> <4D3EF196.50205@aol.com> Message-ID: <9032E29D-79E3-4B55-9A49-8341D8678966@starpower.net> On Jan 25, 2011, at 12:08 PM, Jerry Fusselman wrote: > As declarer, Herman exposes two cards, and makes a joke, not a claim. > I got the joke, and I am surprised that so many others would not. Had > the first name of the declarer in his example been different, it seems > likely to me more would had answered like I did. Anyone interested in > evidence? It should be pretty easy to poll several players whether > they understand this case to be one of 8 or 8+ and whether they are > calling the cops or playing on. I predict most players would play on, > understanding the statement was for 8+. Especially the players with > the best sense of humor will play on. It might be dangerous with grim > players at the table, but most players know that declarer can expose > cards without penalty, and that such exposed cards can help the hand > go faster. > > This is the case for the poll: > > Your opponent was playing 2He. He had just won the first six > tricks, and > he showed AK of hearts, saying "I think I am going to make this > contract". Is this a claim for specifically 8? Do you call the > director? Or do you play on? > > Anyone care to try this poll on players who do not follow BLML and > tell us the results? As in any legal context, you make jokes at your own risk. Actions have consequences. What is the difference between "just" making a joke and gratuitously disrupting the table action? If your opponents take your intended joke seriously, you are more than likely to run afoul of L73F. "Sure I did it, but I didn't really mean anything by it; it was just a joke; can't we just pretend it didn't happen?" might indeed talk me, as a player, into deciding to violate L9B1(a) to your benefit one time. But if I'm the director called to the table and asked to rule, the laws do not give me the option to just pretend it didn't happen. And I feel pretty safe in presuming that if it were "demonstrably" just a joke rather than something else (such as a possible attempt to claim), the opponents would have "gotten" it. Eric Landau 1107 Dale Drive Silver Spring MD 20910 ehaa at starpower.net From jfusselman at gmail.com Tue Jan 25 20:32:29 2011 From: jfusselman at gmail.com (Jerry Fusselman) Date: Tue, 25 Jan 2011 13:32:29 -0600 Subject: [BLML] Unsure In-Reply-To: <370FDAF7-DB79-4D9B-99A9-ADCF9F66E990@starpower.net> References: <586818.75337.qm@web28510.mail.ukl.yahoo.com> <4D3704A8.8090006@skynet.be> <4D3852CE.6080908@ulb.ac.be> <4D386625.90105@meteo.fr> <4D3869DD.9030708@ulb.ac.be> <4D3873A6.5070104@meteo.fr> <107026.33279.qm@web28515.mail.ukl.yahoo.com> <4D39BC9E.9040108@nhcc.net> <1820574630.137446.1295675648497.JavaMail.ngmail@webmail11.arcor-online.net> <4D3D7134.4000803@ulb.ac.be> <431431.44115.qm@web28515.mail.ukl.yahoo.com> <4D3D7F51.70805@ulb.ac.be> <395644.68863.qm@web28514.mail.ukl.yahoo.com> <000101cbbbcf$4f8dd270$eea97750$@no> <1868019136.66657.1295900769160.JavaMail.ngmail@webmail08.arcor-online.net> <532598E0-1D81-4AA2-84BA-3EC0BE2D2843@starpower.net> <4D3EE5B3.1030006@skynet.be> <370FDAF7-DB79-4D9B-99A9-ADCF9F66E990@starpower.net> Message-ID: [Eric] I have "leveled the playing field" to the best of my ability; from here there's only "bridge logic". ? I have no obligation to explain any conclusion (or lack thereof) that my bridge logic may have led me to, nor even, for that matter, any reason to think that my deductions are any more valid than what my opponents may have been able to deduce from the same facts. [Herman] In short, you rely on General Bridge Knowledge. [Eric] Huh? ?I think Herman may have confused my posts with someone else's. In short, I rely on my meta-agreements and my understanding of our general style, which I disclose. ?Just as I wrote. Who (besides Herman) has suggested bringing "general bridge knowledge" into this discussion? [Jerry] It seems to me Eric did, unless there is some difference between "general bridge knowledge" and the "bridge logic" Eric states need not be disclosed. They sound pretty similar to me, but maybe I'm wrong. Personally, I would usually rather have my opponents distill the meaning down to something like what Sven, Herman, and Steve Willner recommend than have long explanations of the sources of confusion in the explainer's mind, like Eric and Nigel recommend. I don't think I have explained a call as "undiscussed" even once in the last fifteen years, and I play tons of gadgets. We are talking about understandings here, not agreements, and I really doubt that my opponents' guess is better than mine, even if I spoke to them for several minutes to "level the playing field." Jerry Fusselman From ehaa at starpower.net Tue Jan 25 21:08:02 2011 From: ehaa at starpower.net (Eric Landau) Date: Tue, 25 Jan 2011 15:08:02 -0500 Subject: [BLML] Exam question In-Reply-To: References: <4D3D3B2F.5090107@skynet.be> <1PhIDl-0dAMts0@fwd00.aul.t-online.de> <4D3E0908.5090009@skynet.be> <82D7A2DF-92A6-495E-ABC1-6C5C27F82B32@starpower.net> <4D3F0245.9010808@skynet.be> <18090296-A6E9-4502-B8D1-2863FD6C7EC5@starpower.net> Message-ID: On Jan 25, 2011, at 2:09 PM, Jerry Fusselman wrote: > On Tue, Jan 25, 2011 at 12:47 PM, Eric Landau > wrote: > >> On Jan 25, 2011, at 12:03 PM, Herman De Wael wrote: >> >>> There are three parts of L68, any of which makes it a claim >>> a) I did not mention a specific number of tricks (only 8 or more) >> >> How did you manage to mention "eight or more" without ever mentioning >> "eight"? > > The same way "the rest are mine" implies a specific number without > ever mentioning that number. "The rest are mine" indeed specifies a number without explicitly mentioning it: 13 minus the number of tricks already played (AKA "the rest"), which is the number of tricks you have specifically said "are mine". And in exactly the same way, "the ace and king of trumps are mine" specifies a number without explicitly mentioning it: 2, which is the number of tricks you have specifically said "are mine". Are you suggesting that "the rest are mine" isn't a claim? >>> b) I did not suggest play be curtailed >> >> Perhaps you didn't; I don't need to read your mind in this case. >> L68D "suggested" that play be curtailed. > > No, L68D starts with the phrase "After any claim or concession, play > ceases...", so it presupposes there is a claim. Sarcasm alert: "suggested" in quotes to suggest "suggested" to be an ironic understatement; L68D actually *requires* that play be curtailed. >>> c) I did not show my cards (only two of them) >> >> Those two cards weren't yours? >> >>> How you can draw from my story that I did either of these is beyond >>> me. >> >> Because you presumably intended to do something (other than >> gratuitously disrupt the table), and the only evidence that you did >> not intend to claim is that you self-servingly state that you did not >> intend to claim, which doesn't rise to the level of "demonstrable". > > [Law 68A. Claim Defined] > Any statement to the effect that a contestant will win a specific > number of > tricks is a claim of those tricks. A contestant also claims when he > suggests that play be curtailed, or when he shows his cards (unless he > demonstrably did not intend to claim - for example, if declarer > faces his > cards after an opening lead out of turn Law 54, not this Law, will > apply). > > Herman's position is not exactly what you inferred. I think Herman > means that "shows his cards" refers to showing all of his remaining > cards, which he did not do, so the second sentence is not relevant > here. If that's what he means, he is wrong. If I whip out my willie on the street and get arrested for indecent exposure, the judge will not care that I didn't show all of it. > I thought declarer showing some of his cards happens every > day---does it not? When you play to a trick, you demonstrably do not intend to claim. > As for gratuitously disrupting the table, I guess it depends on the > table. I often enjoy jokes at the table; even bad jokes sometimes. A > table with a friendly atmosphere can handle and even enjoy declarer > showing the AK of trump and making a funny comment, but a table with a > poisoned, Secretary-Birdish atmosphere cannot, I guess. The point Mollo was making when he invented the Secretary Bird is that the letter of the law may require the director to take actions sufficiently extreme to violate the common-sense spirit of the game. Normally, when Secretary-Bird situations arise, the spirit of the law is sufficiently obvious to everyone at the table that the director does not get called and the law goes unenforced. But it only takes one -- the Secretary Bird -- to prevent the table from being unanimous on the point of spirit and demand satisfaction on the point of law, and the director cannot refuse. Nothing in TFLB allows the director to decline to make a ruling when called to a table on the grounds that no other table would have called him. Perhaps there should be, but that's for another thread. Eric Landau 1107 Dale Drive Silver Spring MD 20910 ehaa at starpower.net From darkbystry at wp.pl Tue Jan 25 22:25:20 2011 From: darkbystry at wp.pl (Maciej Bystrzejewski) Date: Tue, 25 Jan 2011 22:25:20 +0100 Subject: [BLML] Difficult case from Polish Top Individual Tournament Message-ID: <276CBE3DABAE4CA1950AF97DB7452E1C@coa12> Hi all, I'm coming back from a long read-only hiatus. As could I see, dWS is like Lenin, evergreen :) To the point, I've got an interesting case for you. Individual with screens, all players are Premier League level, bidding goes: N E S W 2D* x 3H pass pass 3NT pass pass ?** *Minimulti, only weak two with one major ** N starts to think Now S makes a face-up lead (heart jack) and opens the screen without any warning. First part is obvious - L24B, N has to pass, HJ is a major penalty card (will be led per force) and UI to N. In the actual deal heart lead beats 3NT by two tricks (it was bid without heart stopper). So, there was an infraction, there was a rectification, but NOS is left with a poor score, and there was a non-zero chance, that N would bid 4H. In other words, 4H was a LA. What should the TD do, is he allowed to use specifically: L23 L12A1 L84D Take it as certain, that you, as a TD, strongly believe, that S simply thought that the auction had ended (he is a honest person, it was a friendly (albeit strong) tournament, it was swiss and that table was far from top). I would heartily welcome all the opinions, especially from Grattan and Ton, because there is no consensus among our top TD staff and each side stands strongly on their position. regards, Maciej From svenpran at online.no Tue Jan 25 22:45:45 2011 From: svenpran at online.no (Sven Pran) Date: Tue, 25 Jan 2011 22:45:45 +0100 Subject: [BLML] Difficult case from Polish Top Individual Tournament In-Reply-To: <276CBE3DABAE4CA1950AF97DB7452E1C@coa12> References: <276CBE3DABAE4CA1950AF97DB7452E1C@coa12> Message-ID: <000601cbbcd9$394fd750$abef85f0$@no> > -----Original Message----- > From: blml-bounces at rtflb.org [mailto:blml-bounces at rtflb.org] On Behalf Of Maciej > Bystrzejewski > Sent: 25. januar 2011 22:25 > To: blml at rtflb.org > Subject: [BLML] Difficult case from Polish Top Individual Tournament > > Hi all, I'm coming back from a long read-only hiatus. > > As could I see, dWS is like Lenin, evergreen :) > > To the point, I've got an interesting case for you. > > Individual with screens, all players are Premier League level, bidding goes: > > N E S W > 2D* x 3H pass > pass 3NT pass pass > ?** > > *Minimulti, only weak two with one major > ** N starts to think > > Now S makes a face-up lead (heart jack) and opens the screen without any > warning. > > First part is obvious - L24B, N has to pass, HJ is a major penalty card (will be led > per force) and UI to N. > > In the actual deal heart lead beats 3NT by two tricks (it was bid without heart > stopper). > > So, there was an infraction, there was a rectification, but NOS is left with a poor > score, and there was a non-zero chance, that N would bid 4H. In other words, 4H > was a LA. > > What should the TD do, is he allowed to use specifically: > > L23 > L12A1 > L84D > > Take it as certain, that you, as a TD, strongly believe, that S simply thought that > the auction had ended (he is a honest person, it was a friendly (albeit strong) > tournament, it was swiss and that table was far from top). > > I would heartily welcome all the opinions, especially from Grattan and Ton, > because there is no consensus among our top TD staff and each side stands > strongly on their position. I think that first of all you should consult the applicable regulations on screen use and what they have to say about ending the auction and premature opening of the screen. Absent any specific regulation rule I think that after applying Law 24B the director should try Law 23, and in my opinion he will normally find that South "could have known" that silencing his partner at this time could work to his advantage. What would have been the "normal" result in 4H by South? This should be used to determine possible damage to NOS. From darkbystry at wp.pl Tue Jan 25 22:58:34 2011 From: darkbystry at wp.pl (Maciej Bystrzejewski) Date: Tue, 25 Jan 2011 22:58:34 +0100 Subject: [BLML] Difficult case from Polish Top Individual Tournament References: <276CBE3DABAE4CA1950AF97DB7452E1C@coa12> <000601cbbcd9$394fd750$abef85f0$@no> Message-ID: <4B4F9B03CE154405B3CC0415A4D55A7C@coa12> Hi Sven. ----- Original Message ----- From: "Sven Pran" To: "'Bridge Laws Mailing List'" Sent: Tuesday, January 25, 2011 10:45 PM Subject: Re: [BLML] Difficult case from Polish Top Individual Tournament > > >> -----Original Message----- >> From: blml-bounces at rtflb.org [mailto:blml-bounces at rtflb.org] On Behalf Of > Maciej >> Bystrzejewski >> Sent: 25. januar 2011 22:25 >> To: blml at rtflb.org >> Subject: [BLML] Difficult case from Polish Top Individual Tournament >> >> Hi all, I'm coming back from a long read-only hiatus. >> >> As could I see, dWS is like Lenin, evergreen :) >> >> To the point, I've got an interesting case for you. >> >> Individual with screens, all players are Premier League level, bidding > goes: >> >> N E S W >> 2D* x 3H pass >> pass 3NT pass pass >> ?** >> >> *Minimulti, only weak two with one major >> ** N starts to think >> >> Now S makes a face-up lead (heart jack) and opens the screen without any >> warning. >> >> First part is obvious - L24B, N has to pass, HJ is a major penalty card > (will be led >> per force) and UI to N. >> >> In the actual deal heart lead beats 3NT by two tricks (it was bid without > heart >> stopper). >> >> So, there was an infraction, there was a rectification, but NOS is left > with a poor >> score, and there was a non-zero chance, that N would bid 4H. In other > words, 4H >> was a LA. >> >> What should the TD do, is he allowed to use specifically: >> >> L23 >> L12A1 >> L84D >> >> Take it as certain, that you, as a TD, strongly believe, that S simply > thought that >> the auction had ended (he is a honest person, it was a friendly (albeit > strong) >> tournament, it was swiss and that table was far from top). >> >> I would heartily welcome all the opinions, especially from Grattan and > Ton, >> because there is no consensus among our top TD staff and each side stands >> strongly on their position. > > I think that first of all you should consult the applicable regulations on > screen use and what they have to say about ending the auction and > premature > opening of the screen. Our regulations on screen use do not modify L24. > Absent any specific regulation rule I think that after applying Law 24B > the > director should try Law 23, and in my opinion he will normally find that > South "could have known" that silencing his partner at this time could > work > to his advantage. All right, but I'd like you (and everyone else) to specifically comment on the possible use of L12A1 and L84D in this case and maybe in the similar ones. > What would have been the "normal" result in 4H by South? > This should be used to determine possible damage to NOS. Down two, doubled, for -300. But I don't need help in calculating the adjusted score, I just want to know, whether a TD is in fact allowed to change the table score in this situation, especially if he doesn't want/can't use L23. regards, Maciej From ehaa at starpower.net Tue Jan 25 22:58:43 2011 From: ehaa at starpower.net (Eric Landau) Date: Tue, 25 Jan 2011 16:58:43 -0500 Subject: [BLML] Unsure In-Reply-To: References: <586818.75337.qm@web28510.mail.ukl.yahoo.com> <4D3704A8.8090006@skynet.be> <4D3852CE.6080908@ulb.ac.be> <4D386625.90105@meteo.fr> <4D3869DD.9030708@ulb.ac.be> <4D3873A6.5070104@meteo.fr> <107026.33279.qm@web28515.mail.ukl.yahoo.com> <4D39BC9E.9040108@nhcc.net> <1820574630.137446.1295675648497.JavaMail.ngmail@webmail11.arcor-online.net> <4D3D7134.4000803@ulb.ac.be> <431431.44115.qm@web28515.mail.ukl.yahoo.com> <4D3D7F51.70805@ulb.ac.be> <395644.68863.qm@web28514.mail.ukl.yahoo.com> <000101cbbbcf$4f8dd270$eea97750$@no> <1868019136.66657.1295900769160.JavaMail.ngmail@webmail08.arcor-online.net> <532598E0-1D81-4AA2-84BA-3EC0BE2D2843@starpower.net> <4D3EE5B3.1030006@skynet.be> <370FDAF7-DB79-4D9B-99A9-ADCF9F66E990@starpower.net> Message-ID: <599DCA0D-F013-4BA0-953E-8BA497B30BB1@starpower.net> On Jan 25, 2011, at 2:32 PM, Jerry Fusselman wrote: > [Eric] > > I have "leveled > the playing field" to the best of my ability; from here there's only > "bridge logic". I have no obligation to explain any conclusion (or > lack thereof) that my bridge logic may have led me to, nor even, for > that matter, any reason to think that my deductions are any more > valid than what my opponents may have been able to deduce from the > same facts. > > [Herman] > > In short, you rely on General Bridge Knowledge. > > [Eric] > Huh? I think Herman may have confused my posts with someone else's. > > In short, I rely on my meta-agreements and my understanding of our > general style, which I disclose. Just as I wrote. > > Who (besides Herman) has suggested bringing "general bridge > knowledge" into this discussion? > > [Jerry] > It seems to me Eric did, unless there is some difference between > "general bridge knowledge" and the "bridge logic" Eric states need not > be disclosed. They sound pretty similar to me, but maybe I'm wrong. If you look up "knowledge" and "logic" in a dictionary, you will discover that their definitions are quite dissimilar. "Bridge knowledge", general or otherwise, is knowledge specific to bridge. "Bridge logic" is the application of the principles of logic to bridge; the knowledge from which conclusions are derived may or may not be "bridge knowledge". I take a very narrow view of GBK, and will readily provide my opponents with any "bridge knowledge" I have that I think they might find useful. I have never refused to answer an opponents question on the grounds of the answer being GBK, and probably never will. If we need an example, "bridge knowledge" is what tells me that when partner opens a weak two-bid he almost certainly has a six-card suit, "bridge logic" is what tells me in addition that he almost certainly has seven cards in the remaining three suits. Would anyone confuse these to the point of thinking that I need not disclose the former, or that I am obliged to disclose the latter? > Personally, I would usually rather have my opponents distill the > meaning down to something like what Sven, Herman, and Steve Willner > recommend than have long explanations of the sources of confusion in > the explainer's mind, like Eric and Nigel recommend. I suspect we all would, if there were some way of assuring that their distillations would be guaranteed to produce correct meanings. Nobody likes it that the disclosure laws are so complicated and difficult that nobody knows what they're supposed to do at the table 100% of the time. That's why I like the "Kaplan paradigm" so much -- it tells me what I'm supposed to do at the table. But simplified procedure requires simplified laws, and that's not that easy to come by. Sven, Herman and Steve all have good ideas, but they're somewhat different ones, each with its advantages and disadvantages. By all means lets talk about how and why we should improve our laws, but let's remember that the laws don't change when BLML decides it would be a good idea; they remain what they are. > I don't think I have explained a call as "undiscussed" even once in > the last fifteen years, and I play tons of gadgets. We are talking > about understandings here, not agreements, and I really doubt that my > opponents' guess is better than mine, even if I spoke to them for > several minutes to "level the playing field." Nor have I. But we play in the ACBL, whose published guidance is, don't worry about the rules; if you just go out of your way to be as friendly and helpful to your opponents as you can be, you'll come out fine ("Ruling the Game", at least, has given up entirely on trying to explain legal disclosure obligations to mere players). I don't think deciding for them what I think they ought to decide for themselves and telling them just that and only that is being as friendly and helpful as possible, but I suppose some folks may feel otherwise. And it is true that doing it my way may well take longer. As an aside to Jerry, I would point out that a partnership that plays "tons of gadgets" is far less likely than one that doesn't to be tempted to make an undiscussed call. Eric Landau 1107 Dale Drive Silver Spring MD 20910 ehaa at starpower.net From svenpran at online.no Tue Jan 25 23:26:10 2011 From: svenpran at online.no (Sven Pran) Date: Tue, 25 Jan 2011 23:26:10 +0100 Subject: [BLML] Difficult case from Polish Top Individual Tournament In-Reply-To: <4B4F9B03CE154405B3CC0415A4D55A7C@coa12> References: <276CBE3DABAE4CA1950AF97DB7452E1C@coa12> <000601cbbcd9$394fd750$abef85f0$@no> <4B4F9B03CE154405B3CC0415A4D55A7C@coa12> Message-ID: <000701cbbcde$de194230$9a4bc690$@no> On Behalf Of Maciej .............. > >> To the point, I've got an interesting case for you. > >> > >> Individual with screens, all players are Premier League level, bidding > > goes: > >> > >> N E S W > >> 2D* x 3H pass > >> pass 3NT pass pass > >> ?** > >> > >> *Minimulti, only weak two with one major > >> ** N starts to think > >> > >> Now S makes a face-up lead (heart jack) and opens the screen without any > >> warning. > >> > >> First part is obvious - L24B, N has to pass, HJ is a major penalty card > > (will be led > >> per force) and UI to N. > >> > >> In the actual deal heart lead beats 3NT by two tricks (it was bid without > > heart > >> stopper). > >> > >> So, there was an infraction, there was a rectification, but NOS is left > > with a poor > >> score, and there was a non-zero chance, that N would bid 4H. In other > > words, 4H > >> was a LA. > >> > >> What should the TD do, is he allowed to use specifically: > >> > >> L23 > >> L12A1 > >> L84D > >> > >> Take it as certain, that you, as a TD, strongly believe, that S simply > > thought that > >> the auction had ended (he is a honest person, it was a friendly (albeit > > strong) > >> tournament, it was swiss and that table was far from top). > >> > >> I would heartily welcome all the opinions, especially from Grattan and > > Ton, > >> because there is no consensus among our top TD staff and each side stands > >> strongly on their position. > > > > I think that first of all you should consult the applicable regulations on > > screen use and what they have to say about ending the auction and > > premature > > opening of the screen. > > Our regulations on screen use do not modify L24. > > > Absent any specific regulation rule I think that after applying Law 24B > > the > > director should try Law 23, and in my opinion he will normally find that > > South "could have known" that silencing his partner at this time could > > work > > to his advantage. > > All right, but I'd like you (and everyone else) to specifically comment on > the possible use of L12A1 and L84D in this case and maybe in the similar > ones. > > > What would have been the "normal" result in 4H by South? > > This should be used to determine possible damage to NOS. > > Down two, doubled, for -300. But I don't need help in calculating the > adjusted score, I just want to know, whether a TD is in fact allowed to > change the table score in this situation, especially if he doesn't > want/can't use L23. Law 84D does not apply because Law 24 provides a specific rectification Law 12 does not apply directly because of Law 12B2: The Director may not award an adjusted score on the ground that the rectification provided in these Laws (i.e. L24) is either unduly severe or advantageous to either side. So the only way the Director may use Law 12 and award an adjusted score in your case is in my opinion through Law 23 as I have described previously. From l.kalbarczyk at gmail.com Wed Jan 26 00:41:24 2011 From: l.kalbarczyk at gmail.com (=?UTF-8?B?xYF1a2FzeiBLYWxiYXJjenlr?=) Date: Wed, 26 Jan 2011 00:41:24 +0100 Subject: [BLML] Difficult case from Polish Top Individual Tournament Message-ID: <4D3F5FA4.10000@gmail.com> Welcome all! Sven said: Law 84D does not apply because Law 24 provides a specific rectification. But Law 84D says: *D. Director's Option* The Director rules any doubtful point in favour of the non-offending side. He seeks to restore equity. If in his judgement it is probable that a non-offending side has been damaged by an irregularity for which these laws provide no rectification he adjusts the score (see Law 12). If _____in his judgement____ ______it is probable_____ that a non-offending side has been damaged by an irregularity. Why 84B doesn't allow to use 84D? ?K, Polish Bridge Union, TD. PS Sorry 4 my english ;) -------------- next part -------------- An HTML attachment was scrubbed... URL: http://lists.rtflb.org/pipermail/blml/attachments/20110125/09f3ab8a/attachment.html From jfusselman at gmail.com Wed Jan 26 01:15:27 2011 From: jfusselman at gmail.com (Jerry Fusselman) Date: Tue, 25 Jan 2011 18:15:27 -0600 Subject: [BLML] Exam question Message-ID: On Tue, Jan 25, 2011 at 1:21 PM, Eric Landau wrote: > On Jan 25, 2011, at 12:08 PM, Jerry Fusselman wrote: >> >> This is the case for the poll: >> >> Your opponent was playing 2He. He had just won the first six >> tricks, and >> he showed AK of hearts, saying "I think I am going to make this >> contract". Is this a claim for specifically 8? Do you call the >> director? Or do you play on? >> > > As in any legal context, you make jokes at your own risk. Actions > have consequences. Very true. > What is the difference between "just" making a > joke and gratuitously disrupting the table action? Speaking generally, I would say it is the joker's job to limit himself to jokes his audience appreciates. When he fails, he should try to be much better in the future. > If your opponents > take your intended joke seriously, you are more than likely to run > afoul of L73F. Depends on the joke. > > "Sure I did it, but I didn't really mean anything by it; it was just > a joke; can't we just pretend it didn't happen?" might indeed talk > me, as a player, into deciding to violate L9B1(a) to your benefit one > time. So Eric has answered the poll I suggested, speaking as a player. He might indeed call the director. Who else, as a player, would call the director? But I don't think Eric has to violate L9B1(a) at all. [L9B. After Attention Is Drawn to an Irregularity 1. (a) The Director should be summoned at once when attention is drawn to an irregularity.] All he has to do is to refrain from calling attention to the irregularity or decide that it is no irregularity. > But if I'm the director called to the table and asked to rule, > the laws do not give me the option to just pretend it didn't happen. > And I feel pretty safe in presuming that if it were "demonstrably" > just a joke rather than something else (such as a possible attempt to > claim), the opponents would have "gotten" it. > As I said before, I consider the phrase containing demonstrably to be irrelevant here, on the grounds that he only showed two of his seven remaining cards. I suppose this issue hinges on whether "his cards" refers to one or more, two or more, or all. I was assuming it means all. Has the issue been discussed before? Jerry Fusselman From ardelm at optusnet.com.au Wed Jan 26 03:17:14 2011 From: ardelm at optusnet.com.au (Tony Musgrove) Date: Wed, 26 Jan 2011 13:17:14 +1100 Subject: [BLML] Exam question In-Reply-To: <74D61212-85D5-42B3-9583-FFE26C078352@starpower.net> References: <4D3E9ACD.5020602@skynet.be> <1Phfff-2FiDdA0@fwd06.aul.t-online.de> <4D3EE3E8.70308@skynet.be> <74D61212-85D5-42B3-9583-FFE26C078352@starpower.net> Message-ID: <201101260217.p0Q2H3WK019057@mail07.syd.optusnet.com.au> At 04:52 AM 26/01/2011, you wrote: >On Jan 25, 2011, at 9:53 AM, Herman De Wael wrote: > > > Peter Eidt wrote: > > > >> What do you want us to do, Herman? > >> > >> You asked an exam question, giving us (hopefully) > >> the facts. > >> The unanimous (execpt Mr. Fusselman) answer > >> was: it was a claim - play ceased. > >> So you should expect when giving your answer > >> "Declarer did not claim and may play on" to get > >> zero points for this. > >> > >> Now you are mixing up all counter-arguments > >> bringing in your own language and tell us, we all > >> are wrong because the facts were not as stated. > >> You told us declarer showed Ace and King of > >> trumps just to tell opponents he has them and > >> in order to be nice (and smart?) to them - as if > >> there wouldn't have been aware of that fact ... > >> > >> ts ts ts > > > > Well, Peter, what do you want me to do? > > I ask a question, believing to know the answer. > > You tell me another answer. > > Now of course the answer of the great Peter Eidt is the correct > > one, and > > that of lowly Herman De Wael must be wrong. > > > > ts ts ts. > > > > So I ask you again, Peter: what is the meaning of the word > > "specific" in > > L68A? > >There is a TV show in the US called "The McLaughlin Group", a weekly >panel discussion of American politics. It is hosted by John >McLaughlin, who appears with four other panelists. It is famous for >the following interaction, which occurs with considerable frequency: >Mr. McLaughlin poses a question to the panel; each of the four >panelists, in turn, gives essentially the same answer. Mr. >McLaughlin then announces (his signiture phrase), "The answer is...," >and proceeds to expound an opinion diametrically opposite to the >unanimous opinion of his panelists. > >Of course, when Mr. McLaughlin does this, he understands that he is >making a joke. Correct. I am sure Herman was making the Belgian equivalent of a joke..very funny. However he had better not do it with me directing. I was a playing director in 3rd position. Partner led, and declarer immediiately started abusing partner with the usual .."you should have bid x or y. We are making 6". I naturally refused to play on and looked at declarer's hand, much to his annoyance. "You claimed", said I, "and now I get to adjudicate", Cheers, Tony (Sydney) From nigelguthrie at yahoo.co.uk Wed Jan 26 03:30:36 2011 From: nigelguthrie at yahoo.co.uk (Nigel Guthrie) Date: Wed, 26 Jan 2011 02:30:36 +0000 (GMT) Subject: [BLML] Unsure In-Reply-To: References: <586818.75337.qm@web28510.mail.ukl.yahoo.com> <4D3704A8.8090006@skynet.be> <4D3852CE.6080908@ulb.ac.be> <4D386625.90105@meteo.fr> <4D3869DD.9030708@ulb.ac.be> <4D3873A6.5070104@meteo.fr> <107026.33279.qm@web28515.mail.ukl.yahoo.com> <4D39BC9E.9040108@nhcc.net> <1820574630.137446.1295675648497.JavaMail.ngmail@webmail11.arcor-online.net> <4D3D7134.4000803@ulb.ac.be> <431431.44115.qm@web28515.mail.ukl.yahoo.com> <4D3D7F51.70805@ulb.ac.be> <395644.68863.qm@web28514.mail.ukl.yahoo.com> <000101cbbbcf$4f8dd270$eea97750$@no> <1868019136.66657.1295900769160.JavaMail.ngmail@webmail08.arcor-online.net> <532598E0-1D81-4AA2-84BA-3EC0BE2D2843@starpower.net> <4D3EE5B3.1030006@skynet.be> <370FDAF7-DB79-4D9B-99A9-ADCF9F66E990@starpower.net> Message-ID: <102015.57400.qm@web28504.mail.ukl.yahoo.com> [Jerry Fusselman] Personally, I would usually rather have my opponents distill the meaning down to something like what Sven, Herman, and Steve Willner recommend than have long explanations of the sources of confusion in the explainer's mind, like Eric and Nigel recommend. [Nigel] You are confusing me with somebody else Jerry :( When opponents ask you about the meaning of your partner's call, and you are unsure, I think the law should mandate that you make your best guess. I don't think you should attempt long-winded explanations of subconscious thought precesses, intuitions, confusing discussions, uncertain memories of system notes, or whatever. My concern is that law-interpreters, such as the EBU, seem to prohibit this straight-forward approach. From jfusselman at gmail.com Wed Jan 26 04:48:29 2011 From: jfusselman at gmail.com (Jerry Fusselman) Date: Tue, 25 Jan 2011 21:48:29 -0600 Subject: [BLML] Unsure In-Reply-To: <102015.57400.qm@web28504.mail.ukl.yahoo.com> References: <586818.75337.qm@web28510.mail.ukl.yahoo.com> <4D3704A8.8090006@skynet.be> <4D3852CE.6080908@ulb.ac.be> <4D386625.90105@meteo.fr> <4D3869DD.9030708@ulb.ac.be> <4D3873A6.5070104@meteo.fr> <107026.33279.qm@web28515.mail.ukl.yahoo.com> <4D39BC9E.9040108@nhcc.net> <1820574630.137446.1295675648497.JavaMail.ngmail@webmail11.arcor-online.net> <4D3D7134.4000803@ulb.ac.be> <431431.44115.qm@web28515.mail.ukl.yahoo.com> <4D3D7F51.70805@ulb.ac.be> <395644.68863.qm@web28514.mail.ukl.yahoo.com> <000101cbbbcf$4f8dd270$eea97750$@no> <1868019136.66657.1295900769160.JavaMail.ngmail@webmail08.arcor-online.net> <532598E0-1D81-4AA2-84BA-3EC0BE2D2843@starpower.net> <4D3EE5B3.1030006@skynet.be> <370FDAF7-DB79-4D9B-99A9-ADCF9F66E990@starpower.net> <102015.57400.qm@web28504.mail.ukl.yahoo.com> Message-ID: On Tue, Jan 25, 2011 at 8:30 PM, Nigel Guthrie wrote: > [Jerry Fusselman] > Personally, I would usually rather have my opponents distill the > meaning down to something like what Sven, Herman, and Steve Willner > recommend than have long explanations of the sources of confusion in > the explainer's mind, like Eric and Nigel recommend. > > [Nigel] > You are confusing me with somebody else Jerry :( > > When opponents ask you about the meaning of your partner's call, and you are > unsure, I think the law should mandate that you make your best guess. I don't > think you should attempt long-winded explanations of subconscious thought > precesses, intuitions, confusing discussions, uncertain memories of system > notes, or whatever. My concern is that law-interpreters, such as the EBU, seem > to prohibit this straight-forward approach. This is what I was going by: [Nigel] When asked the meaning of a call, the law says you must not speculate gratuitously. If you aren't certain (and that must be the case for most calls and almost all players), you must answer "unsure". [Jerry] Obviously, on this issue, Nigel is not in the camp of Sven, Herman, and Steve Willner, who argue against saying "unsure" and instead suggest giving a specific answer in most cases, while hiding your own doubt. In contrast, Nigel not only suggests saying "unsure," he says it is required. That sounds rather opposite to me. Now what is supposed to accompany Nigel's required word, "unsure"? Not silence. I thought it was obvious that he would suggest explaining the various options in much the same way that Eric recommends, ideally to give the opponents all of the same relevant information that explainer has, with no personal doubts left unrevealed. Nigel, do you still say that the word "unsure" is required? And in what way does your advice of ideal disclosure when uncertain differ from Eric's? Jerry Fusselman From hirsch9000 at gmail.com Wed Jan 26 05:07:45 2011 From: hirsch9000 at gmail.com (Hirsch Davis) Date: Tue, 25 Jan 2011 23:07:45 -0500 Subject: [BLML] Naive question about double shots In-Reply-To: References: <327C53E4-386A-40BF-9871-4071863E8144@starpower.net> Message-ID: <4D3F9E11.60602@gmail.com> On 1/25/2011 10:23 AM, Eric Landau wrote: > On Jan 24, 2011, at 8:05 PM, Jerry Fusselman wrote: > >> Alright, but I still want to know what defines a double shot. I guess >> we are talking about >> Law 12C1b: "If, subsequent to the irregularity, the non-offending >> side has >> contributed to its own damage by a serious error (unrelated to the >> infraction) or by wild or gambling action it does not receive >> relief in the >> adjustment for such part of the damage as is self-inflicted." The >> term double shot is undefined in the laws. >> >> Alright, I imagine that no one on BLML can define it. > WTP? A double shot is an action taken subsequent to an opponent's > infraction that the player would not have taken in otherwise > identical circumstances had there been no infraction. > > > Eric Landau > 1107 Dale Drive > Silver Spring MD 20910 > ehaa at starpower.net > > _ No, the action that comprises the double shot may well have occurred had there been no infraction, and in fact may be perfectly legal. A double shot can be described an action taken after an opponent's infraction that has an element of risk, where the risk is minimized by the probable rectification of the preceding infraction. If the action works out well, the NOS keeps their score. If it works out poorly, the NOS gets the benefit of the score adjustment for the earlier infraction. It's a "no-lose" scenario due to the infraction. There is no problem with the actions that would have occurred in the absence of an infraction, even if they should turn out to be "double shots". Both sides still get to play bridge even after there has been an infraction, and some perfectly normal bridge actions may become less risky after an opponent has committed an infraction. These may occur fairly frequently, and we would barely notice them. The ones that are problematical are the cases where the NOS does in fact take an action that would not have occurred in the absence of the infraction (the subset of double shots defined by Eric). These are the ones that make us queasy because they reek of taking advantage of the Laws rather than bridge ability to increase the score. These double shots remain legal regardless of whether we like them or not. The action may not succeed, should it be judged to be a serious error, or wild and gambling, but often will work out. There is no Law against taking advantage of an opponent's infraction. In fact, it could be argued that it would be a serious error not to do so. Hirsch From jfusselman at gmail.com Wed Jan 26 05:11:05 2011 From: jfusselman at gmail.com (Jerry Fusselman) Date: Tue, 25 Jan 2011 22:11:05 -0600 Subject: [BLML] Exam question In-Reply-To: <57364C95-04EF-47DA-AAE5-F16277E1496A@gmail.com> References: <4D3D3B2F.5090107@skynet.be> <1PhIDl-0dAMts0@fwd00.aul.t-online.de> <4D3E0908.5090009@skynet.be> <82D7A2DF-92A6-495E-ABC1-6C5C27F82B32@starpower.net> <4D3F0245.9010808@skynet.be> <18090296-A6E9-4502-B8D1-2863FD6C7EC5@starpower.net> <57364C95-04EF-47DA-AAE5-F16277E1496A@gmail.com> Message-ID: On Jan 25, 2011, at 2:08 PM, Eric Landau wrote: > > "The rest are mine" indeed specifies a number without explicitly > mentioning it: 13 minus the number of tricks already played (AKA "the > rest"), which is the number of tricks you have specifically said "are > mine". ?And in exactly the same way, "the ace and king of trumps are > mine" specifies a number without explicitly mentioning it: 2, which > is the number of tricks you have specifically said "are mine". ?Are > you suggesting that "the rest are mine" isn't a claim? No, I thought you were suggesting that by insisting that a number be stated. I was merely stating that some sentences imply a number, while some do not. > >>> >> >>> >>> >> >> [Law 68A. Claim Defined] >> Any statement to the effect that a contestant will win a specific >> number of >> tricks is a claim of those tricks. A contestant also claims when he >> suggests that play be curtailed, or when he shows his cards ... >> > ? If I whip out my willie on the > street and get arrested for indecent exposure, the judge will not > care that I didn't show all of it. What you do on the street is your own business. Shall we talk about cards? Does the expression "put your cards on the table" make you think of putting all of your cards on the table, or 29% of them? Note on Google that "put your cards on the table" and "put all your cards on the table" have the same meaning. No difference at all. In Herman's question, how many cards are his? ?Seven. ?If he shows two cards, then he has not yet shown his cards. Tonight after dinner, I asked my bridge playing mother-in-law (a lawyer who currently specializes in arbitration) what the phrase "shows his cards" means in this context, and she said that to her it means all of the cards. I also asked my wife, a mathematician who knows nothing about bridge, and her answer was interesting. Her answer was, "it depends." If you were playing a game, she said, where the common step of placing cards on the table is to place exactly four cards on the table, then "show his cards" would mean placing exactly four cards on the table. But if you were playing a game where the only common step when placing cards on the table is to place all of them there, then "show his cards" would mean placing all of your cards on the table. She does not know anything about claiming, but here answer supports that "show his cards" refers only to the case of all cards. Anyone care to try asking other people? Or would you rather just assume you are right without evidence and be done with it? > >> I thought declarer showing some of his cards happens every >> day---does it not? > > When you play to a trick, you demonstrably do not intend to claim. Plural alert. ?I wrote "some of his cards", meaning more than one. Playing to a trick does not apply here, because it is card, not cards. Jerry Fusselman From jfusselman at gmail.com Wed Jan 26 06:29:43 2011 From: jfusselman at gmail.com (Jerry Fusselman) Date: Tue, 25 Jan 2011 23:29:43 -0600 Subject: [BLML] Exam question In-Reply-To: <201101260217.p0Q2H3WK019057@mail07.syd.optusnet.com.au> References: <4D3E9ACD.5020602@skynet.be> <1Phfff-2FiDdA0@fwd06.aul.t-online.de> <4D3EE3E8.70308@skynet.be> <74D61212-85D5-42B3-9583-FFE26C078352@starpower.net> <201101260217.p0Q2H3WK019057@mail07.syd.optusnet.com.au> Message-ID: On Tue, Jan 25, 2011 at 8:17 PM, Tony Musgrove wrote: >?I am sure Herman was making the Belgian equivalent > of a joke..very funny. ?However he had better not do it with me > directing. ?I was a playing director in 3rd position. ?Partner led, > and declarer immediiately started abusing partner with the usual > .."you should have bid x or y. ?We are making 6". ?I naturally > refused to play on and looked at declarer's hand, much to his > annoyance. ?"You claimed", said I, "and now I get to adjudicate", > Herman's was a hypothetical question. I am familiar with this style of directing, but how worthwhile is it to ruin the board to make a point? Jerry Fusselman From diggadog at iinet.net.au Wed Jan 26 07:23:42 2011 From: diggadog at iinet.net.au (Bill & Helen Kemp) Date: Wed, 26 Jan 2011 14:23:42 +0800 Subject: [BLML] Exam question References: <4D3E9ACD.5020602@skynet.be> <1Phfff-2FiDdA0@fwd06.aul.t-online.de><4D3EE3E8.70308@skynet.be><74D61212-85D5-42B3-9583-FFE26C078352@starpower.net><201101260217.p0Q2H3WK019057@mail07.syd.optusnet.com.au> Message-ID: <45C793F2D7B543F5B67CAE0B25A72CA8@acer> ----- Original Message ----- From: "Jerry Fusselman" To: "Bridge Laws Mailing List" Sent: Wednesday, January 26, 2011 1:29 PM Subject: Re: [BLML] Exam question On Tue, Jan 25, 2011 at 8:17 PM, Tony Musgrove wrote: > I am sure Herman was making the Belgian equivalent > of a joke..very funny. However he had better not do it with me > directing. I was a playing director in 3rd position. Partner led, > and declarer immediiately started abusing partner with the usual > .."you should have bid x or y. We are making 6". I naturally > refused to play on and looked at declarer's hand, much to his > annoyance. "You claimed", said I, "and now I get to adjudicate", > Herman's was a hypothetical question. I am familiar with this style of directing, but how worthwhile is it to ruin the board to make a point? That's strange, I could have sworn Herman claimed 8 tricks and Tony's declarer claimed 12 tricks. Were they both hypothetical or only Herman's claim - or neither? This post is rhetorical bill Jerry Fusselman _______________________________________________ Blml mailing list Blml at rtflb.org http://lists.rtflb.org/mailman/listinfo/blml From ardelm at optusnet.com.au Wed Jan 26 07:27:39 2011 From: ardelm at optusnet.com.au (Tony Musgrove) Date: Wed, 26 Jan 2011 17:27:39 +1100 Subject: [BLML] Exam question In-Reply-To: References: <4D3E9ACD.5020602@skynet.be> <1Phfff-2FiDdA0@fwd06.aul.t-online.de> <4D3EE3E8.70308@skynet.be> <74D61212-85D5-42B3-9583-FFE26C078352@starpower.net> <201101260217.p0Q2H3WK019057@mail07.syd.optusnet.com.au> Message-ID: <201101260627.p0Q6RThY032476@mail06.syd.optusnet.com.au> At 04:29 PM 26/01/2011, you wrote: >On Tue, Jan 25, 2011 at 8:17 PM, Tony Musgrove wrote: > > I am sure Herman was making the Belgian equivalent > > of a joke..very funny. However he had better not do it with me > > directing. I was a playing director in 3rd position. Partner led, > > and declarer immediiately started abusing partner with the usual > > .."you should have bid x or y. We are making 6". I naturally > > refused to play on and looked at declarer's hand, much to his > > annoyance. "You claimed", said I, "and now I get to adjudicate", > > > >Herman's was a hypothetical question. > >I am familiar with this style of directing, but how worthwhile is it >to ruin the board to make a point? > >Jerry Fusselman >_______________________________________________ Not at all, according to Herman it occurred during play. I adjudicated making 6 quite correctly, Cheers and Happy Australia Day to all Tony(Sydney) From jfusselman at gmail.com Wed Jan 26 07:55:23 2011 From: jfusselman at gmail.com (Jerry Fusselman) Date: Wed, 26 Jan 2011 00:55:23 -0600 Subject: [BLML] Exam question In-Reply-To: <201101260627.p0Q6RThY032476@mail06.syd.optusnet.com.au> References: <4D3E9ACD.5020602@skynet.be> <1Phfff-2FiDdA0@fwd06.aul.t-online.de> <4D3EE3E8.70308@skynet.be> <74D61212-85D5-42B3-9583-FFE26C078352@starpower.net> <201101260217.p0Q2H3WK019057@mail07.syd.optusnet.com.au> <201101260627.p0Q6RThY032476@mail06.syd.optusnet.com.au> Message-ID: On Wed, Jan 26, 2011 at 12:27 AM, Tony Musgrove wrote: > At 04:29 PM 26/01/2011, you wrote: >>On Tue, Jan 25, 2011 at 8:17 PM, Tony Musgrove wrote: >> > I am sure Herman was making the Belgian equivalent >> > of a joke..very funny. ?However he had better not do it with me >> > directing. ?I was a playing director in 3rd position. ?Partner led, >> > and declarer immediiately started abusing partner with the usual >> > .."you should have bid x or y. ?We are making 6". ?I naturally >> > refused to play on and looked at declarer's hand, much to his >> > annoyance. ?"You claimed", said I, "and now I get to adjudicate", >> > >> >>Herman's was a hypothetical question. >> >>I am familiar with this style of directing, but how worthwhile is it >>to ruin the board to make a point? >> >>Jerry Fusselman >>_______________________________________________ > > Not at all, according to Herman it occurred during play. Maybe so. I had thought it was purely hypothetical, but maybe not. > ?I adjudicated making 6 quite correctly, Alright, did you ruin the board, or not? If not, did you know ahead of time whether you might ruin the board? > > Cheers and Happy Australia Day to all Happy Australia Day to you too. A great country, Australia, with recently now more freedom than the USA. I hope the USA may gain some of our freedoms back, soon. If not, do you think Australia would welcome me and my wife? Regards, Jerry Fusselman From jfusselman at gmail.com Wed Jan 26 07:56:53 2011 From: jfusselman at gmail.com (Jerry Fusselman) Date: Wed, 26 Jan 2011 00:56:53 -0600 Subject: [BLML] Exam question In-Reply-To: <45C793F2D7B543F5B67CAE0B25A72CA8@acer> References: <4D3E9ACD.5020602@skynet.be> <1Phfff-2FiDdA0@fwd06.aul.t-online.de> <4D3EE3E8.70308@skynet.be> <74D61212-85D5-42B3-9583-FFE26C078352@starpower.net> <201101260217.p0Q2H3WK019057@mail07.syd.optusnet.com.au> <45C793F2D7B543F5B67CAE0B25A72CA8@acer> Message-ID: On Wed, Jan 26, 2011 at 12:23 AM, Bill & Helen Kemp wrote: > > ----- Original Message ----- > From: "Jerry Fusselman" > To: "Bridge Laws Mailing List" > Sent: Wednesday, January 26, 2011 1:29 PM > Subject: Re: [BLML] Exam question > > > On Tue, Jan 25, 2011 at 8:17 PM, Tony Musgrove wrote: >> I am sure Herman was making the Belgian equivalent >> of a joke..very funny. However he had better not do it with me >> directing. I was a playing director in 3rd position. Partner led, >> and declarer immediiately started abusing partner with the usual >> .."you should have bid x or y. We are making 6". I naturally >> refused to play on and looked at declarer's hand, much to his >> annoyance. "You claimed", said I, "and now I get to adjudicate", >> > > Herman's was a hypothetical question. > > I am familiar with this style of directing, but how worthwhile is it > to ruin the board to make a point? > > That's strange, I could have sworn Herman claimed 8 tricks and Tony's > declarer claimed 12 tricks. Were they both hypothetical or only Herman's > claim - or neither? > This post is rhetorical > Alas, I don't get your point. From ardelm at optusnet.com.au Wed Jan 26 08:02:47 2011 From: ardelm at optusnet.com.au (Tony Musgrove) Date: Wed, 26 Jan 2011 18:02:47 +1100 Subject: [BLML] Exam question In-Reply-To: References: <4D3E9ACD.5020602@skynet.be> <1Phfff-2FiDdA0@fwd06.aul.t-online.de> <4D3EE3E8.70308@skynet.be> <74D61212-85D5-42B3-9583-FFE26C078352@starpower.net> <201101260217.p0Q2H3WK019057@mail07.syd.optusnet.com.au> <201101260627.p0Q6RThY032476@mail06.syd.optusnet.com.au> Message-ID: <201101260702.p0Q72aR7004045@mail02.syd.optusnet.com.au> At 05:55 PM 26/01/2011, you wrote: >On Wed, Jan 26, 2011 at 12:27 AM, Tony Musgrove wrote: > > At 04:29 PM 26/01/2011, you wrote: > >>On Tue, Jan 25, 2011 at 8:17 PM, Tony Musgrove wrote: > >> > I am sure Herman was making the Belgian equivalent > >> > of a joke..very funny. However he had better not do it with me > >> > directing. I was a playing director in 3rd position. Partner led, > >> > and declarer immediiately started abusing partner with the usual > >> > .."you should have bid x or y. We are making 6". I naturally > >> > refused to play on and looked at declarer's hand, much to his > >> > annoyance. "You claimed", said I, "and now I get to adjudicate", > >> > > >> > >>Herman's was a hypothetical question. > >> > >>I am familiar with this style of directing, but how worthwhile is it > >>to ruin the board to make a point? > >> > >>Jerry Fusselman > >>_______________________________________________ > > > > Not at all, according to Herman it occurred during play. > >Maybe so. I had thought it was purely hypothetical, but maybe not. > > > I adjudicated making 6 quite correctly, > >Alright, did you ruin the board, or not? If not, did you know ahead >of time whether you might ruin the board? > > > > > Cheers and Happy Australia Day to all > >Happy Australia Day to you too. A great country, Australia, with >recently now more freedom than the USA. I hope the USA may gain some >of our freedoms back, soon. If not, do you think Australia would >welcome me and my wife? > >Regards, >Jerry Fusselman The declarer's gratuitous abusing of his partner had already ruined the board for me. Do not attempt to come to Oz via boat, otherwise probably OK, Cheers, Tony (Sydney) >_______________________________________________ >Blml mailing list >Blml at rtflb.org >http://lists.rtflb.org/mailman/listinfo/blml From PeterEidt at t-online.de Wed Jan 26 09:17:22 2011 From: PeterEidt at t-online.de (Peter Eidt) Date: Wed, 26 Jan 2011 09:17:22 +0100 Subject: [BLML] =?utf-8?q?Difficult_case_from_Polish_Top_Individual_Tourna?= =?utf-8?q?ment?= In-Reply-To: <000601cbbcd9$394fd750$abef85f0$@no> References: <000601cbbcd9$394fd750$abef85f0$@no> Message-ID: <1Pi0Z8-1IM86S0@fwd00.aul.t-online.de> From: "Sven Pran" > On Behalf Of Maciej Bystrzejewski > > Individual with screens, all players are Premier League level, > > bidding goes: > > > > N E S W > > 2D* x 3H pass > > pass 3NT pass pass > > ?** > > > > *Minimulti, only weak two with one major > > ** N starts to think > > > > Now S makes a face-up lead (heart jack) and opens the screen without > > any warning. > > > > First part is obvious - L24B, N has to pass, HJ is a major penalty > > card (will be led per force) and UI to N. > > > > In the actual deal heart lead beats 3NT by two tricks (it was bid > > without heart stopper). > > > > So, there was an infraction, there was a rectification, but NOS is > > left with a poor score, and there was a non-zero chance, that N > > would bid 4H. In other words, 4H was a LA. > > > > What should the TD do, is he allowed to use specifically: > > > > L23 > > L12A1 > > L84D > > > > Take it as certain, that you, as a TD, strongly believe, that S > > simply thought that the auction had ended (he is a honest > > person, it was a friendly (albeit strong) tournament, it was > > swiss and that table was far from top). > > > > I would heartily welcome all the opinions, especially from Grattan > > and Ton, because there is no consensus among our top TD staff > > and each side stands strongly on their position. > > > > I think that first of all you should consult the applicable > regulations on screen use and what they have to say about ending the > auction and premature opening of the screen. > > Absent any specific regulation rule I think that after applying Law > 24B the director should try Law 23, and in my opinion he will normally > find that South "could have known" that silencing his partner at this > time could work to his advantage. What would have been the "normal" > result in 4H by South? This should be used to determine possible > damage to NOS. I agree with Sven. Imagine the situation. Your RHO bids 3NT, you pass and the tray is pushed through the screen with 2 players still to call. Now making the opening lead _and_ opening the screen aperture without waiting for the tray to come back is IMHO so [be polite, Peter !] bad behaviour, that I feel not sorry for South, if I had to rule against him. Law 24 B applies and points explicitely to Law 23 ("when the (enforced) pass damages to non-offending side"). Normally I would not believe the pass damaging the NOS here, but in our case North indeed started to think. If he thought about sacrificing in 4 hearts, then he might have bid 4 H, in which case the enforced pass indeed did damage the NOS and I would think of a weighted score representing the probabilities of passing and sacrificing. Only if North could persuade me that he was thinking about a double, NS might keep their score. Regarding the other 2 mentioned laws I believe that neither Law 12 A1 nor Law 84 D (which is almost the same) apply here, as there _is_ a prescribed rectification (Laws 23/24) that deals with this situation. From Hermandw at skynet.be Wed Jan 26 09:21:48 2011 From: Hermandw at skynet.be (Herman De Wael) Date: Wed, 26 Jan 2011 09:21:48 +0100 Subject: [BLML] Exam question In-Reply-To: <18090296-A6E9-4502-B8D1-2863FD6C7EC5@starpower.net> References: <4D3D3B2F.5090107@skynet.be> <1PhIDl-0dAMts0@fwd00.aul.t-online.de> <4D3E0908.5090009@skynet.be> <82D7A2DF-92A6-495E-ABC1-6C5C27F82B32@starpower.net> <4D3F0245.9010808@skynet.be> <18090296-A6E9-4502-B8D1-2863FD6C7EC5@starpower.net> Message-ID: <4D3FD99C.8070008@skynet.be> Eric Landau wrote: >> >> There are three parts of L68, any of which makes it a claim >> a) I did not mention a specific number of tricks (only 8 or more) > > How did you manage to mention "eight or more" without ever mentioning > "eight"? > Sorry, I said I'll make the contract, without mentioning that that was all. If you are going to rule that all mentions of tricks are specific, then that word need not be in the laws. The law should then read "any mention of any future trick is a claim. That is not what the law says. >> b) I did not suggest play be curtailed > > Perhaps you didn't; I don't need to read your mind in this case. > L68D "suggested" that play be curtailed. > Sorry, but that is just plain silly. A claim is defined as suggesting that play be curtailed. If you are going to rule that since play is curtailed because it is a claim, therefore it is a claim, then you have just ruled that steam locomotive to be a claim. Please be a little more careful with your posts, Eric. >> c) I did not show my cards (only two of them) > > Those two cards weren't yours? > Again sorry, Eric. I showed two of my cards, not "my cards". If the WBF had intended that the showing of any card would make it a claim, then they would have written the law differently. "he shows any card". >> How you can draw from my story that I did either of these is beyond >> me. > > Because you presumably intended to do something (other than > gratuitously disrupt the table), and the only evidence that you did > not intend to claim is that you self-servingly state that you did not > intend to claim, which doesn't rise to the level of "demonstrable". > So I do none of the 3 things that make an action a claim, but you rule that it is a claim anyway because I did not say it was not a claim? Really Eric, I had expected better of you! > > Eric Landau -- Herman De Wael Wilrijk Antwerpen Belgium From Hermandw at skynet.be Wed Jan 26 09:25:55 2011 From: Hermandw at skynet.be (Herman De Wael) Date: Wed, 26 Jan 2011 09:25:55 +0100 Subject: [BLML] Exam question In-Reply-To: <4D3F13CA.1070503@aol.com> References: <4D3E9ACD.5020602@skynet.be> <1Phfff-2FiDdA0@fwd06.aul.t-online.de> <4D3EE3E8.70308@skynet.be> <4D3EF196.50205@aol.com> <4D3F02B6.30808@skynet.be> <4D3F13CA.1070503@aol.com> Message-ID: <4D3FDA93.80707@skynet.be> Jeff Easterson wrote: > > Although your English is pretty good (and seems to have improved greatly > in the past few years) you are still not a native speaker. That might > help to explain your problem with "specific". > OK Jeff, please tell me, what does specific mean in this sentence? -- Herman De Wael Wilrijk Antwerpen Belgium From harald.skjaran at gmail.com Wed Jan 26 09:29:45 2011 From: harald.skjaran at gmail.com (=?UTF-8?Q?Harald_Skj=C3=A6ran?=) Date: Wed, 26 Jan 2011 09:29:45 +0100 Subject: [BLML] Exam question In-Reply-To: <4D3FD99C.8070008@skynet.be> References: <4D3D3B2F.5090107@skynet.be> <1PhIDl-0dAMts0@fwd00.aul.t-online.de> <4D3E0908.5090009@skynet.be> <82D7A2DF-92A6-495E-ABC1-6C5C27F82B32@starpower.net> <4D3F0245.9010808@skynet.be> <18090296-A6E9-4502-B8D1-2863FD6C7EC5@starpower.net> <4D3FD99C.8070008@skynet.be> Message-ID: It's time for this endless discussion to stop. Surely, Herman didn't make a claim, that's pretty obvious from the OP. What he did was to tell the table that the hand only was about overtricks. No claim intended by Herman, no claim anticipated by the opponents and no claim according to L68A. 2011/1/26 Herman De Wael : > Eric Landau wrote: >>> >>> There are three parts of L68, any of which makes it a claim >>> a) I did not mention a specific number of tricks (only 8 or more) >> >> How did you manage to mention "eight or more" without ever mentioning >> "eight"? >> > > Sorry, I said I'll make the contract, without mentioning that that was > all. If you are going to rule that all mentions of tricks are specific, > then that word need not be in the laws. The law should then read "any > mention of any future trick is a claim. That is not what the law says. > >>> b) I did not suggest play be curtailed >> >> Perhaps you didn't; I don't need to read your mind in this case. >> L68D "suggested" that play be curtailed. >> > > Sorry, but that is just plain silly. A claim is defined as suggesting > that play be curtailed. If you are going to rule that since play is > curtailed because it is a claim, therefore it is a claim, then you have > just ruled that steam locomotive to be a claim. Please be a little more > careful with your posts, Eric. > >>> c) I did not show my cards (only two of them) >> >> Those two cards weren't yours? >> > > Again sorry, Eric. I showed two of my cards, not "my cards". If the WBF > had intended that the showing of any card would make it a claim, then > they would have written the law differently. "he shows any card". > >>> How you can draw from my story that I did either of these is beyond >>> me. >> >> Because you presumably intended to do something (other than >> gratuitously disrupt the table), and the only evidence that you did >> not intend to claim is that you self-servingly state that you did not >> intend to claim, which doesn't rise to the level of "demonstrable". >> > > So I do none of the 3 things that make an action a claim, but you rule > that it is a claim anyway because I did not say it was not a claim? > > Really Eric, I had expected better of you! > >> >> Eric Landau > > -- > Herman De Wael > Wilrijk Antwerpen Belgium > _______________________________________________ > Blml mailing list > Blml at rtflb.org > http://lists.rtflb.org/mailman/listinfo/blml > -- Kind regards, Harald Skj?ran From Hermandw at skynet.be Wed Jan 26 09:31:39 2011 From: Hermandw at skynet.be (Herman De Wael) Date: Wed, 26 Jan 2011 09:31:39 +0100 Subject: [BLML] Exam question In-Reply-To: <4D3F09C0.3010803@ulb.ac.be> References: <4D3E9ACD.5020602@skynet.be> <1Phfff-2FiDdA0@fwd06.aul.t-online.de> <4D3EE3E8.70308@skynet.be> <4D3EF196.50205@aol.com> <4D3F09C0.3010803@ulb.ac.be> Message-ID: <4D3FDBEB.4000905@skynet.be> Alain Gottcheiner wrote: > Le 25/01/2011 18:08, Jerry Fusselman a ?crit : >> On Tue, Jan 25, 2011 at 9:51 AM, Jeff Easterson wrote: >>> Jerry seems to always support Herman. >> Nonsense, even in this thread. I gave my answer before Herman gave his. >> >> As declarer, Herman exposes two cards, and makes a joke, not a claim. >> I got the joke, and I am surprised that so many others would not. Had >> the first name of the declarer in his example been different, it seems >> likely to me more would had answered like I did. Anyone interested in >> evidence? It should be pretty easy to poll several players whether >> they understand this case to be one of 8 or 8+ and whether they are >> calling the cops or playing on. I predict most players would play on, >> understanding the statement was for 8+. Especially the players with >> the best sense of humor will play on. > AG : notice that the question wasn't, "what does your sense of humor say > ?" but rather "what do the laws say ?" Yes, I do believe that these are > two different answers. > Alain is right. I did intend it as a joke, and I am as surprised as Jerry that blml did not pick up on that. The question here should not be however, whether I intended it as 8 or 8+, the fact that I intended 8+ should be a given (let's say the TD at the table gets the opponents to agree to that one). The interesting question then is whether the sentence "I will make at least eight tricks" constitutes a claim or not. I have read some quite silly arguments here already, none more so than the one that said since it contained the word "eight" it was a claim. I don't know how to say "more than eight" without saying "eight". oh yes, I do: "I think I'll make this contract". >> >> Your opponent was playing 2He. He had just won the first six tricks, and >> he showed AK of hearts, saying "I think I am going to make this >> contract". Is this a claim for specifically 8? Do you call the >> director? Or do you play on? > AG : once again, that's a mix. The question isn't whether most would > call the Td, but how the TD should act if called. > Indeed. -- Herman De Wael Wilrijk Antwerpen Belgium From Hermandw at skynet.be Wed Jan 26 09:43:46 2011 From: Hermandw at skynet.be (Herman De Wael) Date: Wed, 26 Jan 2011 09:43:46 +0100 Subject: [BLML] Unsure In-Reply-To: <599DCA0D-F013-4BA0-953E-8BA497B30BB1@starpower.net> References: <586818.75337.qm@web28510.mail.ukl.yahoo.com> <4D3704A8.8090006@skynet.be> <4D3852CE.6080908@ulb.ac.be> <4D386625.90105@meteo.fr> <4D3869DD.9030708@ulb.ac.be> <4D3873A6.5070104@meteo.fr> <107026.33279.qm@web28515.mail.ukl.yahoo.com> <4D39BC9E.9040108@nhcc.net> <1820574630.137446.1295675648497.JavaMail.ngmail@webmail11.arcor-online.net> <4D3D7134.4000803@ulb.ac.be> <431431.44115.qm@web28515.mail.ukl.yahoo.com> <4D3D7F51.70805@ulb.ac.be> <395644.68863.qm@web28514.mail.ukl.yahoo.com> <000101cbbbcf$4f8dd270$eea97750$@no> <1868019136.66657.1295900769160.JavaMail.ngmail@webmail08.arcor-online.net> <532598E0-1D81-4AA2-84BA-3EC0BE2D2843@starpower.net> <4D3EE5B3.1030006@skynet.be> <370FDAF7-DB79-4D9B-99A9-ADCF9F66E990@starpower.net> <599DCA0D-F013-4BA0-953E-8BA497B30BB1@starpower.net> Message-ID: <4D3FDEC2.7050107@skynet.be> Eric Landau wrote: >> It seems to me Eric did, unless there is some difference between >> "general bridge knowledge" and the "bridge logic" Eric states need not >> be disclosed. They sound pretty similar to me, but maybe I'm wrong. > > If you look up "knowledge" and "logic" in a dictionary, you will > discover that their definitions are quite dissimilar. > > "Bridge knowledge", general or otherwise, is knowledge specific to > bridge. "Bridge logic" is the application of the principles of logic > to bridge; the knowledge from which conclusions are derived may or > may not be "bridge knowledge". I take a very narrow view of GBK, and > will readily provide my opponents with any "bridge knowledge" I have > that I think they might find useful. I have never refused to answer > an opponents question on the grounds of the answer being GBK, and > probably never will. > > If we need an example, "bridge knowledge" is what tells me that when > partner opens a weak two-bid he almost certainly has a six-card suit, > "bridge logic" is what tells me in addition that he almost certainly > has seven cards in the remaining three suits. Would anyone confuse > these to the point of thinking that I need not disclose the former, > or that I am obliged to disclose the latter? > Well Eric, maybe there is a difference between the two - but I find only GBK in the laws. Where do you get the idea that you do not need to disclose that he has 7 non-hearts when you disclose that he has 6 hearts? I don't see anywhere in the laws that you do not need to disclose bridge logic. So unless you call that GBK, you'll have to disclose it. But anyway, I was just using shorthand. Your point was that you did not need to disclose something because the opponents should be able to work it out for themselves. I called that using the GBK argument, and I argued against it. Calling it bridge logic does not change the argument, or my argument against it. What do you do when you claim that you did not tell something because opponents ought to be able to work it out, when in fact they did not work it out. I call it obfuscation and misinformation. Which is why I advise against it. -- Herman De Wael Wilrijk Antwerpen Belgium From jean-pierre.rocafort at meteo.fr Wed Jan 26 09:52:31 2011 From: jean-pierre.rocafort at meteo.fr (Jean-Pierre Rocafort) Date: Wed, 26 Jan 2011 09:52:31 +0100 Subject: [BLML] Exam question In-Reply-To: References: <4D3E9ACD.5020602@skynet.be> <1Phfff-2FiDdA0@fwd06.aul.t-online.de><4D3EE3E8.70308@skynet.be><74D61212-8 5D5-42B3-9583-FFE26C078352@starpower.net><201101260217.p0Q2H3WK019057@mail0 7.syd.optusnet.com.au><201101260627.p0Q6RThY032476@mail06.syd.optusnet.com.au> Message-ID: <4D3FE0CF.8010903@meteo.fr> Jerry Fusselman a ?crit : > On Wed, Jan 26, 2011 at 12:27 AM, Tony Musgrove wrote: >> At 04:29 PM 26/01/2011, you wrote: >>> On Tue, Jan 25, 2011 at 8:17 PM, Tony Musgrove wrote: >>>> I am sure Herman was making the Belgian equivalent >>>> of a joke..very funny. However he had better not do it with me >>>> directing. I was a playing director in 3rd position. Partner led, >>>> and declarer immediiately started abusing partner with the usual >>>> .."you should have bid x or y. We are making 6". I naturally >>>> refused to play on and looked at declarer's hand, much to his >>>> annoyance. "You claimed", said I, "and now I get to adjudicate", >>>> >>> Herman's was a hypothetical question. >>> >>> I am familiar with this style of directing, but how worthwhile is it >>> to ruin the board to make a point? >>> >>> Jerry Fusselman >>> _______________________________________________ >> Not at all, according to Herman it occurred during play. > > Maybe so. I had thought it was purely hypothetical, but maybe not. > >> I adjudicated making 6 quite correctly, > > Alright, did you ruin the board, or not? If not, did you know ahead > of time whether you might ruin the board? > >> Cheers and Happy Australia Day to all > > Happy Australia Day to you too. A great country, Australia, with > recently now more freedom than the USA. I hope the USA may gain some > of our freedoms back, soon. If not, do you think Australia would > welcome me and my wife? what about your mother in law in that case? jpr > > Regards, > Jerry Fusselman -- _______________________________________________ Jean-Pierre Rocafort METEO-FRANCE DSI/CM 42 Avenue Gaspard Coriolis 31057 Toulouse CEDEX Tph: 05 61 07 81 02 (33 5 61 07 81 02) Fax: 05 61 07 81 09 (33 5 61 07 81 09) e-mail: jean-pierre.rocafort at meteo.fr Serveur WWW METEO-France: http://www.meteo.fr _______________________________________________ From svenpran at online.no Wed Jan 26 10:24:57 2011 From: svenpran at online.no (Sven Pran) Date: Wed, 26 Jan 2011 10:24:57 +0100 Subject: [BLML] Difficult case from Polish Top Individual Tournament In-Reply-To: <4D3F5FA4.10000@gmail.com> References: <4D3F5FA4.10000@gmail.com> Message-ID: <001501cbbd3a$e674fad0$b35ef070$@no> The reason why Law 84D cannot be applied is the following clause in that law: an irregularity for which these laws provide no rectification Law 24 clearly provides the rectification to be applied. From: blml-bounces at rtflb.org [mailto:blml-bounces at rtflb.org] On Behalf Of Lukasz Kalbarczyk Sent: 26. januar 2011 00:41 To: blml at rtflb.org Subject: Re: [BLML] Difficult case from Polish Top Individual Tournament Welcome all! Sven said: Law 84D does not apply because Law 24 provides a specific rectification. But Law 84D says: D. Director's Option The Director rules any doubtful point in favour of the non-offending side. He seeks to restore equity. If in his judgement it is probable that a non-offending side has been damaged by an irregularity for which these laws provide no rectification he adjusts the score (see Law 12). If _____in his judgement____ ______it is probable_____ that a non-offending side has been damaged by an irregularity. Why 84B doesn't allow to use 84D? ?K, Polish Bridge Union, TD. PS Sorry 4 my english ;) -------------- next part -------------- An HTML attachment was scrubbed... URL: http://lists.rtflb.org/pipermail/blml/attachments/20110126/7953d71b/attachment.html From blml at arcor.de Wed Jan 26 10:57:17 2011 From: blml at arcor.de (Thomas Dehn) Date: Wed, 26 Jan 2011 10:57:17 +0100 (CET) Subject: [BLML] Unsure In-Reply-To: <4D3FDEC2.7050107@skynet.be> References: <4D3FDEC2.7050107@skynet.be> <586818.75337.qm@web28510.mail.ukl.yahoo.com> <4D3704A8.8090006@skynet.be> <4D3852CE.6080908@ulb.ac.be> <4D386625.90105@meteo.fr> <4D3869DD.9030708@ulb.ac.be> <4D3873A6.5070104@meteo.fr> <107026.33279.qm@web28515.mail.ukl.yahoo.com> <4D39BC9E.9040108@nhcc.net> <1820574630.137446.1295675648497.JavaMail.ngmail@webmail11.arcor-online.net> <4D3D7134.4000803@ulb.ac.be> <431431.44115.qm@web28515.mail.ukl.yahoo.com> <4D3D7F51.70805@ulb.ac.be> <395644.68863.qm@web28514.mail.ukl.yahoo.com> <000101cbbbcf$4f8dd270$eea97750$@no> <1868019136.66657.1295900769160.JavaMail.ngmail@webmail08.arcor-online.net> <532598E0-1D81-4AA2-84BA-3EC0BE2D2843@starpower.net> <4D3EE5B3.1030006@skynet.be> <370FDAF7-DB79-4D9B-99A9-ADCF9F66E990@starpower.net> <599DCA0D-F013-4BA0-953E-8BA497B30BB1@starpower.net> Message-ID: <927108203.167866.1296035837950.JavaMail.ngmail@webmail10.arcor-online.net> Herman De Wael wrote: > Eric Landau wrote: > >> It seems to me Eric did, unless there is some difference between > >> "general bridge knowledge" and the "bridge logic" Eric states need not > >> be disclosed. They sound pretty similar to me, but maybe I'm wrong. > > > > If you look up "knowledge" and "logic" in a dictionary, you will > > discover that their definitions are quite dissimilar. > > > > "Bridge knowledge", general or otherwise, is knowledge specific to > > bridge. "Bridge logic" is the application of the principles of logic > > to bridge; the knowledge from which conclusions are derived may or > > may not be "bridge knowledge". I take a very narrow view of GBK, and > > will readily provide my opponents with any "bridge knowledge" I have > > that I think they might find useful. I have never refused to answer > > an opponents question on the grounds of the answer being GBK, and > > probably never will. > > > > If we need an example, "bridge knowledge" is what tells me that when > > partner opens a weak two-bid he almost certainly has a six-card suit, > > "bridge logic" is what tells me in addition that he almost certainly > > has seven cards in the remaining three suits. Would anyone confuse > > these to the point of thinking that I need not disclose the former, > > or that I am obliged to disclose the latter? > > > > Well Eric, maybe there is a difference between the two - but I find only > GBK in the laws. Where do you get the idea that you do not need to > disclose that he has 7 non-hearts when you disclose that he has 6 > hearts? I don't see anywhere in the laws that you do not need to > disclose bridge logic. So unless you call that GBK, you'll have to > disclose it. > > But anyway, I was just using shorthand. Your point was that you did not > need to disclose something because the opponents should be able to work > it out for themselves. I called that using the GBK argument, and I > argued against it. Calling it bridge logic does not change the argument, > or my argument against it. > > What do you do when you claim that you did not tell something because > opponents ought to be able to work it out, when in fact they did not > work it out. I call it obfuscation and misinformation. > Which is why I advise against it. TFLB does not require Eric to teach his opponents how to play bridge. So, for example, if Eric plays weak-twos, he will have to inform opponents whether there are any hands that are too strong for a weak two, but two weak for one of Eric's ultra solid 1M openings. Providing opponents with the description of the 1M and 2M openings is not sufficient. However, if the auction goes 1NT - 5NT (invitation for 7) - 7NT X XX Eric has no obligation whatsoever to alert opponents to the fact that this redouble is for blood, rather than SOS. Thomas From t.kooyman at worldonline.nl Wed Jan 26 11:01:00 2011 From: t.kooyman at worldonline.nl (ton) Date: Wed, 26 Jan 2011 11:01:00 +0100 Subject: [BLML] Difficult case from Polish Top Individual Tournament In-Reply-To: <4D3F5FA4.10000@gmail.com> References: <4D3F5FA4.10000@gmail.com> Message-ID: <002f01cbbd3f$efaf6630$cf0e3290$@kooyman@worldonline.nl> Interesting case. I hope that TD?s agree that it should be possible to adjust the score in this case. When a player has an unfavorable choice which is taken away by an infraction from his partner the result of that choice should be taken into account. Given the facts as described I hesitate to use L23. But it is not impossible to apply it. Make EW vulnerable, give South xxx Jxx xxx Qxxx and South even unconsciously will fear a disaster. But make NS vulnerable and no even unconscious hair on South? head will fear a 4H bid by partner. My advise is to use L12A1 to adjust the score and it might be weighted if the chance to bid 4H is small. By the way South not only led prematurely he also opened the aperture wrongly (if Poland uses the ebl/wbf regulations). ton Van: blml-bounces at rtflb.org [mailto:blml-bounces at rtflb.org] Namens Lukasz Kalbarczyk Verzonden: woensdag 26 januari 2011 0:41 Aan: blml at rtflb.org Onderwerp: Re: [BLML] Difficult case from Polish Top Individual Tournament Welcome all! Sven said: Law 84D does not apply because Law 24 provides a specific rectification. But Law 84D says: D. Director's Option The Director rules any doubtful point in favour of the non-offending side. He seeks to restore equity. If in his judgement it is probable that a non-offending side has been damaged by an irregularity for which these laws provide no rectification he adjusts the score (see Law 12). If _____in his judgement____ ______it is probable_____ that a non-offending side has been damaged by an irregularity. Why 84B doesn't allow to use 84D? ?K, Polish Bridge Union, TD. PS Sorry 4 my english ;) _____ Geen virus gevonden in dit bericht. Gecontroleerd door AVG - www.avg.com Versie: 10.0.1191 / Virusdatabase: 1435/3403 - datum van uitgifte: 01/25/11 -------------- next part -------------- An HTML attachment was scrubbed... URL: http://lists.rtflb.org/pipermail/blml/attachments/20110126/764fbda3/attachment-0001.html From blml at arcor.de Wed Jan 26 11:24:32 2011 From: blml at arcor.de (Thomas Dehn) Date: Wed, 26 Jan 2011 11:24:32 +0100 (CET) Subject: [BLML] Difficult case from Polish Top Individual Tournament In-Reply-To: <000601cbbcd9$394fd750$abef85f0$@no> References: <000601cbbcd9$394fd750$abef85f0$@no> <276CBE3DABAE4CA1950AF97DB7452E1C@coa12> Message-ID: <1782689077.74727.1296037472305.JavaMail.ngmail@webmail16.arcor-online.net> Sven Pran wrote: > > -----Original Message----- > > From: blml-bounces at rtflb.org [mailto:blml-bounces at rtflb.org] On Behalf Of > Maciej > > Bystrzejewski > > Sent: 25. januar 2011 22:25 > > To: blml at rtflb.org > > Subject: [BLML] Difficult case from Polish Top Individual Tournament > > > > Hi all, I'm coming back from a long read-only hiatus. > > > > As could I see, dWS is like Lenin, evergreen :) > > > > To the point, I've got an interesting case for you. > > > > Individual with screens, all players are Premier League level, bidding > goes: > > > > N E S W > > 2D* x 3H pass > > pass 3NT pass pass > > ?** > > > > *Minimulti, only weak two with one major > > ** N starts to think > > > > Now S makes a face-up lead (heart jack) and opens the screen without any > > warning. > > > > First part is obvious - L24B, N has to pass, HJ is a major penalty card > (will be led > > per force) and UI to N. > > > > In the actual deal heart lead beats 3NT by two tricks (it was bid without > heart > > stopper). > > > > So, there was an infraction, there was a rectification, but NOS is left > with a poor > > score, and there was a non-zero chance, that N would bid 4H. In other > words, 4H > > was a LA. > > > > What should the TD do, is he allowed to use specifically: > > > > L23 > > L12A1 > > L84D > > > > Take it as certain, that you, as a TD, strongly believe, that S simply > thought that > > the auction had ended (he is a honest person, it was a friendly (albeit > strong) > > tournament, it was swiss and that table was far from top). > > > > I would heartily welcome all the opinions, especially from Grattan and > Ton, > > because there is no consensus among our top TD staff and each side stands > > strongly on their position. > > I think that first of all you should consult the applicable regulations on > screen use and what they have to say about ending the auction and premature > opening of the screen. > > Absent any specific regulation rule I think that after applying Law 24B the > director should try Law 23, and in my opinion he will normally find that > South "could have known" that silencing his partner at this time could work > to his advantage. What would have been the "normal" result in 4H by South? > This should be used to determine possible damage to NOS. I think S also deserves a PP. Prematurely opening the screen might be an honest mistake. Putting the opening lead face up is not. Thomas From svenpran at online.no Wed Jan 26 11:26:13 2011 From: svenpran at online.no (Sven Pran) Date: Wed, 26 Jan 2011 11:26:13 +0100 Subject: [BLML] Difficult case from Polish Top Individual Tournament In-Reply-To: <002f01cbbd3f$efaf6630$cf0e3290$@kooyman@worldonline.nl> References: <4D3F5FA4.10000@gmail.com> <002f01cbbd3f$efaf6630$cf0e3290$@kooyman@worldonline.nl> Message-ID: <002601cbbd43$75e595f0$61b0c1d0$@no> I agree with Ton that it should be possible to adjust, but the way I read Law 12 B2 this is specifically forbidden unless we can involve (for instance) Law 23. In my opinion a better law 12B2 would have been: The Director may not award an adjusted score on the ground that the rectification provided in these Laws is unduly severe to the offending side or unduly advantageous to the non-offending side. Regards Sven From: blml-bounces at rtflb.org [mailto:blml-bounces at rtflb.org] On Behalf Of ton Sent: 26. januar 2011 11:01 To: 'Bridge Laws Mailing List' Subject: Re: [BLML] Difficult case from Polish Top Individual Tournament Interesting case. I hope that TD?s agree that it should be possible to adjust the score in this case. When a player has an unfavorable choice which is taken away by an infraction from his partner the result of that choice should be taken into account. Given the facts as described I hesitate to use L23. But it is not impossible to apply it. Make EW vulnerable, give South xxx Jxx xxx Qxxx and South even unconsciously will fear a disaster. But make NS vulnerable and no even unconscious hair on South? head will fear a 4H bid by partner. My advise is to use L12A1 to adjust the score and it might be weighted if the chance to bid 4H is small. By the way South not only led prematurely he also opened the aperture wrongly (if Poland uses the ebl/wbf regulations). ton Van: blml-bounces at rtflb.org [mailto:blml-bounces at rtflb.org] Namens Lukasz Kalbarczyk Verzonden: woensdag 26 januari 2011 0:41 Aan: blml at rtflb.org Onderwerp: Re: [BLML] Difficult case from Polish Top Individual Tournament Welcome all! Sven said: Law 84D does not apply because Law 24 provides a specific rectification. But Law 84D says: D. Director's Option The Director rules any doubtful point in favour of the non-offending side. He seeks to restore equity. If in his judgement it is probable that a non-offending side has been damaged by an irregularity for which these laws provide no rectification he adjusts the score (see Law 12). If _____in his judgement____ ______it is probable_____ that a non-offending side has been damaged by an irregularity. Why 84B doesn't allow to use 84D? ?K, Polish Bridge Union, TD. PS Sorry 4 my english ;) _____ Geen virus gevonden in dit bericht. Gecontroleerd door AVG - www.avg.com Versie: 10.0.1191 / Virusdatabase: 1435/3403 - datum van uitgifte: 01/25/11 -------------- next part -------------- An HTML attachment was scrubbed... URL: http://lists.rtflb.org/pipermail/blml/attachments/20110126/d82e5a4c/attachment.html From nigelguthrie at yahoo.co.uk Wed Jan 26 11:49:57 2011 From: nigelguthrie at yahoo.co.uk (Nigel Guthrie) Date: Wed, 26 Jan 2011 10:49:57 +0000 (GMT) Subject: [BLML] Unsure In-Reply-To: References: <586818.75337.qm@web28510.mail.ukl.yahoo.com> <4D3704A8.8090006@skynet.be> <4D3852CE.6080908@ulb.ac.be> <4D386625.90105@meteo.fr> <4D3869DD.9030708@ulb.ac.be> <4D3873A6.5070104@meteo.fr> <107026.33279.qm@web28515.mail.ukl.yahoo.com> <4D39BC9E.9040108@nhcc.net> <1820574630.137446.1295675648497.JavaMail.ngmail@webmail11.arcor-online.net> <4D3D7134.4000803@ulb.ac.be> <431431.44115.qm@web28515.mail.ukl.yahoo.com> <4D3D7F51.70805@ulb.ac.be> <395644.68863.qm@web28514.mail.ukl.yahoo.com> <000101cbbbcf$4f8dd270$eea97750$@no> <1868019136.66657.1295900769160.JavaMail.ngmail@webmail08.arcor-online.net> <532598E0-1D81-4AA2-84BA-3EC0BE2D2843@starpower.net> <4D3EE5B3.1030006@skynet.be> <370FDAF7-DB79-4D9B-99A9-ADCF9F66E990@starpower.net> <102015.57400.qm@web28504.mail.ukl.yahoo.com> Message-ID: <75914.47711.qm@web28504.mail.ukl.yahoo.com> [Jerry] Obviously, on this issue, Nigel is not in the camp of Sven, Herman, and Steve Willner, who argue against saying "unsure" and instead suggest giving a specific answer in most cases, while hiding your own doubt. In contrast, Nigel not only suggests saying "unsure," he says it is required. That sounds rather opposite to me. Now what is supposed to accompany Nigel's required word, "unsure"? Not silence. I thought it was obvious that he would suggest explaining the various options in much the same way that Eric recommends, ideally to give the opponents all of the same relevant information that explainer has, with no personal doubts left unrevealed. Nigel, do you still say that the word "unsure" is required? And in what way does your advice of ideal disclosure when uncertain differ from Eric's? [Nigel] I'm nearer Herman than Sven on this issue. I've explained the *twin problems* as I see them. Some BLMLers sneer "WTP" put address neither. A. What should a player say when asked the systemic meaning of partner's call, if he is uncertain? *I* don't know what the law requires he do. Amusingly, nobody else seems to know either. For example, the *EBU* seems to say that he *must not guess* when unsure. Some BLMLers imply that players must be sure of their agreements but many people are certain of nothing. Most top internationals are uncertain (see appeals, ad nauseam). Especially. for less common calls, the confidence level of the typical player is much lower. *IMO* the law should mandate that aplayer make his *best guess* without qualification. (Even when he has only the vaguest implicit agreement). I feel that he should *not* say "unsure" or express doubt. B. Before you can properly answer question A, you must first answer another. What is a player's partnership understanding? Is it what is in his head, now? Or is it what he previously agreed with partner? *IMO* for legal purposes, your partnership understanding should be *both*: - The player's system-notes and system-card should take precedence. - If not covered there, then explicit discussions are next. - In last place, are implicit agreements, derived from previous experience, analogous contexts, local knowledge, and so on. Another solution to these problems is that the law-book explicitly mandate the protocol, suggested by Richard Hills: When a player says "Undiscussed" or "No agreement" (or -- wash my mouth out with carbolic -- "Unsure"), he must leave the table. Then the bidder himself explains his own call. (I don't really understand why this protocol should need the good offices of a director). Anyway, this protocol would rarely be invoked. Players' memories are likely to undergo an immediate dramatic improvement :) I hope that the 2018 or 2028 law-books will clarify such issues. From nigelguthrie at yahoo.co.uk Wed Jan 26 11:59:44 2011 From: nigelguthrie at yahoo.co.uk (Nigel Guthrie) Date: Wed, 26 Jan 2011 10:59:44 +0000 (GMT) Subject: [BLML] Exam question In-Reply-To: References: <4D3E9ACD.5020602@skynet.be> <1Phfff-2FiDdA0@fwd06.aul.t-online.de> <4D3EE3E8.70308@skynet.be> <74D61212-85D5-42B3-9583-FFE26C078352@starpower.net> <201101260217.p0Q2H3WK019057@mail07.syd.optusnet.com.au> Message-ID: <141014.85780.qm@web28501.mail.ukl.yahoo.com> [Tony Musgrove] I am sure Herman was making the Belgian equivalent of a joke..very funny. However he had better not do it with me directing. I was a playing director in 3rd position. Partner led, and declarer immediiately started abusing partner with the usual .."you should have bid x or y. We are making 6". I naturally refused to play on and looked at declarer's hand, much to his annoyance. "You claimed", said I, "and now I get to adjudicate", [Jerry Fusselman] Herman's was a hypothetical question. I am familiar with this style of directing, but how worthwhile is it to ruin the board to make a point? [Nigel] IMO, the director should nip gross antisocial behaviour in the bud. Tony seems to have dealt with the problem, lawfully, with tact and good humour. From vitoldbr at yandex.ru Wed Jan 26 12:38:04 2011 From: vitoldbr at yandex.ru (vitoldbr) Date: Wed, 26 Jan 2011 14:38:04 +0300 Subject: [BLML] Difficult case from Polish Top Individual Tournament In-Reply-To: <002601cbbd43$75e595f0$61b0c1d0$@no> References: <4D3F5FA4.10000@gmail.com> <002f01cbbd3f$efaf6630$cf0e3290$@kooyman@worldonline.nl> <002601cbbd43$75e595f0$61b0c1d0$@no> Message-ID: <45900886.20110126143804@yandex.ru> An HTML attachment was scrubbed... URL: http://lists.rtflb.org/pipermail/blml/attachments/20110126/5d1b9aa0/attachment.html From sater at xs4all.nl Wed Jan 26 13:09:24 2011 From: sater at xs4all.nl (Hans van Staveren) Date: Wed, 26 Jan 2011 13:09:24 +0100 Subject: [BLML] Exam question In-Reply-To: <141014.85780.qm@web28501.mail.ukl.yahoo.com> References: <4D3E9ACD.5020602@skynet.be> <1Phfff-2FiDdA0@fwd06.aul.t-online.de> <4D3EE3E8.70308@skynet.be> <74D61212-85D5-42B3-9583-FFE26C078352@starpower.net> <201101260217.p0Q2H3WK019057@mail07.syd.optusnet.com.au> <141014.85780.qm@web28501.mail.ukl.yahoo.com> Message-ID: <01ec01cbbd51$df883f40$9e98bdc0$@nl> Some years ago in the Dutch top team competition a player well known for his less than ideal behavior told his screen mate after he played somewhere around trick 8: Had you returned a XXX the contract might have gone down". There was an interesting effect now, the other side of the table, hearing(but not really understanding) the conversation, assumed a claim and showed their hand, asking what was the claim? Director was called. Ruling: this was indeed a claim, furthermore it turned out the claim was invalid: the contract, on careful defense could still go down. Decision: down. The benefit of not opening your mouth unless you have something worthwhile to say seems obvious. Hans -----Original Message----- From: blml-bounces at rtflb.org [mailto:blml-bounces at rtflb.org] On Behalf Of Nigel Guthrie Sent: woensdag 26 januari 2011 12:00 To: Bridge Laws Mailing List Subject: Re: [BLML] Exam question [Tony Musgrove] I am sure Herman was making the Belgian equivalent of a joke..very funny. However he had better not do it with me directing. I was a playing director in 3rd position. Partner led, and declarer immediiately started abusing partner with the usual .."you should have bid x or y. We are making 6". I naturally refused to play on and looked at declarer's hand, much to his annoyance. "You claimed", said I, "and now I get to adjudicate", [Jerry Fusselman] Herman's was a hypothetical question. I am familiar with this style of directing, but how worthwhile is it to ruin the board to make a point? [Nigel] IMO, the director should nip gross antisocial behaviour in the bud. Tony seems to have dealt with the problem, lawfully, with tact and good humour. _______________________________________________ Blml mailing list Blml at rtflb.org http://lists.rtflb.org/mailman/listinfo/blml From Hermandw at skynet.be Wed Jan 26 13:45:13 2011 From: Hermandw at skynet.be (Herman De Wael) Date: Wed, 26 Jan 2011 13:45:13 +0100 Subject: [BLML] Unsure In-Reply-To: <927108203.167866.1296035837950.JavaMail.ngmail@webmail10.arcor-online.net> References: <4D3FDEC2.7050107@skynet.be> <4D3704A8.8090006@skynet.be> <4D3852CE.6080908@ulb.ac.be> <4D386625.90105@meteo.fr> <4D3869DD.9030708@ulb.ac.be> <4D3873A6.5070104@meteo.fr> <107026.33279.qm@web28515.mail.ukl.yahoo.com> <4D39BC9E.9040108@nhcc.net> <1820574630.137446.1295675648497.JavaMail.ngmail@webmail11.arcor-online.net> <4D3D7134.4000803@ulb.ac.be> <431431.44115.qm@web28515.mail.ukl.yahoo.com> <4D3D7F51.70805@ulb.ac.be> <395644.68863.qm@web28514.mail.ukl.yahoo.com> <000101cbbbcf$4f8dd270$eea97750$@no> <1868019136.66657.1295900769160.JavaMail.ngmail@webmail08.arcor-online.net> <532598E0-1D81-4AA2-84BA-3EC0BE2D2843@starpower.net> <4D3EE5B3.1030006@skynet.be> <370FDAF7-DB79-4D9B-99A9-ADCF9F66E990@starpower.net> <599DCA0D-F013-4BA0-953E-8BA497B30BB1@starpower.net> <927108203.167866.1296035837950.JavaMail.ngmail@webmail10.arcor-online.net> Message-ID: <4D401759.8090506@skynet.be> Thomas Dehn wrote: >> >> What do you do when you claim that you did not tell something because >> opponents ought to be able to work it out, when in fact they did not >> work it out. I call it obfuscation and misinformation. >> Which is why I advise against it. > > TFLB does not require Eric to teach his opponents how to play bridge. > > So, for example, if Eric plays weak-twos, he will have to inform > opponents whether there are any hands that are too strong for a weak > two, but two weak for one of Eric's ultra solid 1M openings. Providing > opponents with the description of the 1M and 2M openings is not sufficient. > > However, if the auction goes > 1NT - 5NT (invitation for 7) - 7NT X XX > Eric has no obligation whatsoever to alert opponents to the > fact that this redouble is for blood, rather than SOS. > Thomas, you are absolutely right, but you should try and follow a thread logically, and not just jump in with a comment. Please tell me, in the laws, where it says that Eric does not need to tell his opponents this. If it is anything else than 40B6a, then I apologise profusely. And I call that GBK. OK? Now read back a few posts in this thread. Eric said he did not need to tell his opponents something they should know. I retorted that he was using GBK, and I told him why he was wrong in doing so. Eric told me he was not using GBK, but "Bridge Logic". I told him that such a term does not exist, and that the argumentation is the same anyway (his being wrong and mine telling him why) Now heere you come with a very nice story about things you do not neet to tell opponents. All very true, but what does it prove? Nothing. You also try to tell us how L40B6a works, something about which we do not have a quarrel. The point remains that Eric maintains that there are things he need not tell his opponents (his guesses, mainly). And he supports this by a reference which I translat to L40B6a. Now, if I am mistaken in doing so, again, I apologize, but eric has had three chances to point out such a mistake already. So his Bridge Logic and my GBK are one and the same. And then I ask him again: what does the TD do when a player tells him "I did not tell my opponents my guess, but I gave him all the elements needed to draw the same logical conclusion, it is not my fault if my opponent cannot draw a bridge logic conclusion." I'll tell you what I do when I'm the TD: I rule MI. Which is why I urge Eric -and all players- to tell them your guesses. > > Thomas -- Herman De Wael Wilrijk Antwerpen Belgium From svenpran at online.no Wed Jan 26 14:25:54 2011 From: svenpran at online.no (Sven Pran) Date: Wed, 26 Jan 2011 14:25:54 +0100 Subject: [BLML] Difficult case from Polish Top Individual Tournament In-Reply-To: <45900886.20110126143804@yandex.ru> References: <4D3F5FA4.10000@gmail.com> <002f01cbbd3f$efaf6630$cf0e3290$@kooyman@worldonline.nl> <002601cbbd43$75e595f0$61b0c1d0$@no> <45900886.20110126143804@yandex.ru> Message-ID: <003b01cbbd5c$8f6ff0b0$ae4fd210$@no> Which law infraction (separate from Law 24) prevented partner of possible unhappy decision? And please do not send posts directly to me in addition to blml!!!!! From: vitoldbr [mailto:vitoldbr at yandex.ru] Sent: 26. januar 2011 12:38 To: Sven Pran Cc: 'Bridge Laws Mailing List' Subject: Re[2]: [BLML] Difficult case from Polish Top Individual Tournament Hi all:) For my opinion the infraction consis of two different parts: 1. Outer part - premature lead and opening screen gate: Law 24 and Tournament Regulation 2. Inner (even more important) part - preventing partner of possible unhappy decision: Law 12A1 And then - as Ton suggested about weighted result. Best wishes, Vitold I agree with Ton that it should be possible to adjust, but the way I read Law 12 B2 this is specifically forbidden unless we can involve (for instance) Law 23. In my opinion a better law 12B2 would have been: The Director may not award an adjusted score on the ground that the rectification provided in these Laws is unduly severe to the offending side or unduly advantageous to the non-offending side. Regards Sven From: blml-bounces at rtflb.org [mailto:blml-bounces at rtflb.org] On Behalf Of ton Sent: 26. januar 2011 11:01 To: 'Bridge Laws Mailing List' Subject: Re: [BLML] Difficult case from Polish Top Individual Tournament Interesting case. I hope that TD?s agree that it should be possible to adjust the score in this case. When a player has an unfavorable choice which is taken away by an infraction from his partner the result of that choice should be taken into account. Given the facts as described I hesitate to use L23. But it is not impossible to apply it. Make EW vulnerable, give South xxx Jxx xxx Qxxx and South even unconsciously will fear a disaster. But make NS vulnerable and no even unconscious hair on South? head will fear a 4H bid by partner. My advise is to use L12A1 to adjust the score and it might be weighted if the chance to bid 4H is small. By the way South not only led prematurely he also opened the aperture wrongly (if Poland uses the ebl/wbf regulations). ton Van: blml-bounces at rtflb.org [mailto:blml-bounces at rtflb.org] Namens Lukasz Kalbarczyk Verzonden: woensdag 26 januari 2011 0:41 Aan: blml at rtflb.org Onderwerp: Re: [BLML] Difficult case from Polish Top Individual Tournament Welcome all! Sven said: Law 84D does not apply because Law 24 provides a specific rectification. But Law 84D says: D. Director's Option The Director rules any doubtful point in favour of the non-offending side. He seeks to restore equity. If in his judgement it is probable that a non-offending side has been damaged by an irregularity for which these laws provide no rectification he adjusts the score (see Law 12). If _____in his judgement____ ______it is probable_____ that a non-offending side has been damaged by an irregularity. Why 84B doesn't allow to use 84D? ?K, Polish Bridge Union, TD. PS Sorry 4 my english ;) _____ Geen virus gevonden in dit bericht. Gecontroleerd door AVG - www.avg.com Versie: 10.0.1191 / Virusdatabase: 1435/3403 - datum van uitgifte: 01/25/11 -- Best regards, vitoldbr mailto:vitoldbr at yandex.ru -------------- next part -------------- An HTML attachment was scrubbed... URL: http://lists.rtflb.org/pipermail/blml/attachments/20110126/7f709879/attachment-0001.html From ehaa at starpower.net Wed Jan 26 14:59:08 2011 From: ehaa at starpower.net (Eric Landau) Date: Wed, 26 Jan 2011 08:59:08 -0500 Subject: [BLML] Exam question In-Reply-To: References: Message-ID: <6ABF6AE1-1AAE-487D-A112-452A434EA4B2@starpower.net> On Jan 25, 2011, at 7:15 PM, Jerry Fusselman wrote: > As I said before, I consider the phrase containing demonstrably to > be irrelevant here, on the grounds that he only showed two of his > seven remaining cards. I suppose this issue hinges on whether "his > cards" refers to one or more, two or more, or all. I was assuming > it means all. Has the issue been discussed before? It's not unusual (at least around here) to claim by exposing one's remaining winners with a statement to the effect of, "I'll get these," and fold the rest, conceding them. Sometimes the claimer doesn't bother with the statement, and everyone nevertheless folds their cards, scores it up, and moves on. If called to a table where that had occurred, I would not be sympathetic to the player's argument that he had not intended to claim, he was just making a joke, and had not technically claimed, because he hadn't exposed his entire hand. Eric Landau 1107 Dale Drive Silver Spring MD 20910 ehaa at starpower.net From ehaa at starpower.net Wed Jan 26 15:19:37 2011 From: ehaa at starpower.net (Eric Landau) Date: Wed, 26 Jan 2011 09:19:37 -0500 Subject: [BLML] Exam question In-Reply-To: References: <4D3D3B2F.5090107@skynet.be> <1PhIDl-0dAMts0@fwd00.aul.t-online.de> <4D3E0908.5090009@skynet.be> <82D7A2DF-92A6-495E-ABC1-6C5C27F82B32@starpower.net> <4D3F0245.9010808@skynet.be> <18090296-A6E9-4502-B8D1-2863FD6C7EC5@starpower.net> <4D3FD99C.8070008@skynet.be> Message-ID: On Jan 26, 2011, at 3:29 AM, Harald Skj?ran wrote: > Surely, Herman didn't make a claim, that's pretty obvious from the OP. > What he did was to tell the table that the hand only was about > overtricks. > No claim intended by Herman, no claim anticipated by the opponents and > no claim according to L68A. If Herman's opponents didn't think Herman was claiming, why was there a director call, a ruling, and a post to BLML? If Herman's opponents were to tell me that they didn't think Herman was claiming, I would have absolutely no hesitation at all in accepting their statement as prima facie evidence that Herman "demonstrably did not intend to claim" [L68A]. They were there; I wasn't. Eric Landau 1107 Dale Drive Silver Spring MD 20910 ehaa at starpower.net From B.Schelen at Claranet.NL Wed Jan 26 14:37:38 2011 From: B.Schelen at Claranet.NL (Ben Schelen) Date: Wed, 26 Jan 2011 14:37:38 +0100 Subject: [BLML] Exam question References: <4D3D3B2F.5090107@skynet.be> Message-ID: <169E4D76470342E78418BCA9CA131DC9@benspc> ----- Original Message ----- From: "Herman De Wael" To: "blml" Sent: Monday, January 24, 2011 9:41 AM Subject: [BLML] Exam question > This one may well make it into an exam question for a TD course, if you > ever need one: > No: not suitable. Ben From darkbystry at wp.pl Wed Jan 26 17:16:26 2011 From: darkbystry at wp.pl (Maciej Bystrzejewski) Date: Wed, 26 Jan 2011 17:16:26 +0100 Subject: [BLML] Difficult case from Polish Top Individual Tournament References: <000601cbbcd9$394fd750$abef85f0$@no><276CBE3DABAE4CA1950AF97DB7452E1C@coa12> <1782689077.74727.1296037472305.JavaMail.ngmail@webmail16.arcor-online.net> Message-ID: <58158609E61B4E7ABD907B711830C277@coa12> > I think S also deserves a PP. > Prematurely opening the screen might be an honest mistake. > Putting the opening lead face up is not. > > Thomas Of course, and I'm sure he would get one if it wasn't for the tournament specifity. Regards, Maciej From blml at arcor.de Wed Jan 26 17:26:53 2011 From: blml at arcor.de (Thomas Dehn) Date: Wed, 26 Jan 2011 17:26:53 +0100 (CET) Subject: [BLML] Difficult case from Polish Top Individual Tournament In-Reply-To: <58158609E61B4E7ABD907B711830C277@coa12> References: <58158609E61B4E7ABD907B711830C277@coa12> <000601cbbcd9$394fd750$abef85f0$@no><276CBE3DABAE4CA1950AF97DB7452E1C@coa12> <1782689077.74727.1296037472305.JavaMail.ngmail@webmail16.arcor-online.net> Message-ID: <1610993804.162521.1296059213768.JavaMail.ngmail@webmail07.arcor-online.net> Maciej Bystrzejewski wrote: > > I think S also deserves a PP. > > Prematurely opening the screen might be an honest mistake. > > Putting the opening lead face up is not. > > > > Thomas > > Of course, and I'm sure he would get one if it wasn't for the tournament > specifity. This brings up old incidents where one played put the opening lead face down, and another player put down dummy, before the screen was opened. Only the auction was not over yet. Thomas From swillner at nhcc.net Wed Jan 26 17:34:38 2011 From: swillner at nhcc.net (Steve Willner) Date: Wed, 26 Jan 2011 11:34:38 -0500 Subject: [BLML] Difficult case from Polish Top Individual Tournament In-Reply-To: <000701cbbcde$de194230$9a4bc690$@no> References: <276CBE3DABAE4CA1950AF97DB7452E1C@coa12> <000601cbbcd9$394fd750$abef85f0$@no> <4B4F9B03CE154405B3CC0415A4D55A7C@coa12> <000701cbbcde$de194230$9a4bc690$@no> Message-ID: <4D404D1E.1010905@nhcc.net> On 1/25/2011 5:26 PM, Sven Pran wrote: > Law 84D does not apply because Law 24 provides a specific rectification > > Law 12 does not apply directly because of Law 12B2: I agree with this, too. > So the only way the Director may use Law 12 and award an adjusted score in > your case is in my opinion through Law 23 as I have described previously. L23 is directly referenced by L24B, so there's no doubt it applies. The interesting question is whether it would apply in a L24A case; I think it should despite the lack of specific reference. Some people have written that it would be fairer to adjust even if the conditions of L23 are not met, but I don't agree with that. Imagine South accidentally drops a card on the table and is forced to lead it, and it turns out to be the killing lead, which South never would have found. The rules say South keeps his score for this accident, just as he keeps his score if the forced lead turns out to be a disaster. Both seem fair to me. Usually the accident will favor the NOS, but there's no guarantee. Of course the situation is quite different if L23 does apply -- South "could have known" stopping North from bidding or doubling would be a good idea. In that case, you give an adjusted score, weighted (if the RA allows) according to your judgment of the probability North would have bid. If South violated the screen regulations, he gets a standard PP for that, independent of whether you adjust the score or not. In an individual event, the PP does not apply to North (as it would in a pairs event). From richard.hills at immi.gov.au Wed Jan 26 22:58:18 2011 From: richard.hills at immi.gov.au (richard.hills at immi.gov.au) Date: Thu, 27 Jan 2011 08:58:18 +1100 Subject: [BLML] Ecclesiastes (proposed change in law) [SEC=UNOFFICIAL] In-Reply-To: Message-ID: Ecclesiastes 12:1 "Remember now thy Creator* in the days of thy youth." * This verse is a pun; the Hebrew for "your Creator" sounds like the Hebrew for "your grave". Note that this footnote clarifies the typically pessimistic meaning of this Ecclesiastes verse. I am strongly of the belief that a plethora of clarifying footnotes in the 2018 Lawbook would help grass-roots Director understand the typically optimistic meanings of the Laws. +=+=+=+=+=+=+=+=+=+=+=+=+=+=+=+=+=+=+=+=+=+=+=+=+=+=+=+=+=+ (1) A particular rule of a particular game may be easy to obey, but difficult to fully enforce, thereby often granting a score advantage to rule violators vis-a-vis masochistic rule abiders. (2) If an umpire advises players to break such an easy-to- obey but difficult-to-fully-enforce rule, since rule violating players will often gain score advantages vis- a-vis masochistic rule abiding players, then that umpire's illegal advice means that she is neither a fit nor a proper person to retain her commission as an umpire. (3) If the powers-that-be fail to terminate the commission of such an illegally advising umpire, because her performance as an umpire is admirably effective in all other respects, then those powers-that-be are hetaerae. (4) And if that particular illegally advising umpire then refuses to correct the error of the powers-that-be, by voluntarily resigning her commission as an umpire, then she demonstrates that she is too agamous to act with propriety. (5) If a particular player notes that self-incriminating statements by a particular umpire demonstrate that she is neither a fit nor a proper person to continue as an umpire, then that particular player has NOT perpetrated an "ad hominem" attack upon that particular umpire. Rather, that particular umpire has perpetrated an "ad hominem" attack upon herself. (6) Nevertheless, it is fit and proper for players, umpires and agamous umpires to propose changes in the rules to make the rules even easier to obey, and even easier to enforce. +=+=+=+=+=+=+=+=+=+=+=+=+=+=+=+=+=+=+=+=+=+=+=+=+=+=+=+=+=+ The statements above are principles which apply to any game of skill whatsoever; now let us examine principle (6) as it applies to the 1997, 2007 and 2018 Laws of Duplicate Bridge. In the 1997 Lawbook was a very broad Law whose broadness meant that is was easy to obey, but difficult to enforce, hence often giving a score advantage to rules violating players vis-a-vis masochistic rules abiding players. This 1997 Law was the prohibition on dummy or a defender correcting partner's 90 degree error in pointing a completed trick. Therefore the Drafting Committee carefully assessed the ins and outs of this problem. The Drafting Committee observed that the vast majority of the illegal corrections occurred immediately, on the same trick. Thus the ineffectively broad -- easy-to-obey but difficult to enforce -- 1997 Law was replaced by a more effective narrow -- easier to obey and very easy to enforce -- 2007 Law 65B3: Declarer may require that a card pointed incorrectly is pointed as above. Dummy or either defender may draw attention to a card pointed incorrectly, but for these players the right expires when a lead is made to the following trick. If done later Law 16B may apply. +=+=+=+=+=+=+=+=+=+=+=+=+=+=+=+=+=+=+=+=+=+=+=+=+=+=+=+=+=+ Maciej Bystrzejewski: >Hi all, I'm coming back from a long read-only hiatus. > >As could I see, dWS is like Lenin, evergreen :) > >To the point, I've got an interesting case for you. [snip] Richard Hills: Yes, if the evergreen dWS was merely adopted by Herman De Wael, Alain Gottcheiner and their acolytes, there would not be a serious problem. But the dWS was independently re- invented by a grass-roots EBU player, who thereby wasted the time of an EBU Appeals Committee (to the best of my very imperfect recollection, David Burn was the Chair of that EBU Appeals Committee). So therefore a rules simplification may make the dWS infraction easier for a grass-roots player to avoid, and easier for a grass-roots Director to enforce. +=+=+=+=+=+=+=+=+=+=+=+=+=+=+=+=+=+=+=+=+=+=+=+=+=+=+=+=+=+ 2018 Law 94 - The dWS Infraction Notwithstanding anything else written elsewhere in this 2018 Lawbook, this Law 94 has over-riding power to the extent of any inconsistency. (1) A player's incorrect explanation of partner's previous call(s) and/or play(s) does not become an infraction until her partner's next call or play. (2) If an incorrect explanation is corrected by the explainer after LHO has called or played, but before the explainer's partner has called or played, then LHO may freely change her call or play. (3) LHO's withdrawn call or play is authorised information to RHO, but unauthorised information to the other side. (4) The withdrawn incorrect explanation is unauthorised information to partner, but authorised information (at their own risk) to LHO and RHO. (5) The correct explanation is authorised information to LHO and RHO. (6) The correct explanation may be unauthorised information to partner and therefore may restrict partner's choice of call(s) and/or play(s). See Law 75A. (7) If an incorrect explanation does not receive a timely withdrawal by the explainer, then the dWS infraction has been created. LHO and RHO may well be damaged. (8) Damage to LHO and RHO may include not only a lack of timely correct explanation to their side, but also a lack of timely Law 75A unauthorised information to the misexplainer's partner. The Director should take both of these factors into account if she adjusts the score. +=+=+=+=+=+=+=+=+=+=+=+=+=+=+=+=+=+=+=+=+=+=+=+=+=+=+=+=+=+ Best wishes Richard Hills Recruitment Section Specialist Recruitment Team Level 5 Aqua, workstation W569, 6223 8453 DIAC Social Club movie tickets -------------------------------------------------------------------- Important Notice: If you have received this email by mistake, please advise the sender and delete the message and attachments immediately. This email, including attachments, may contain confidential, sensitive, legally privileged and/or copyright information. Any review, retransmission, dissemination or other use of this information by persons or entities other than the intended recipient is prohibited. DIAC respects your privacy and has obligations under the Privacy Act 1988. The official departmental privacy policy can be viewed on the department's website at www.immi.gov.au. See: http://www.immi.gov.au/functional/privacy.htm --------------------------------------------------------------------- From swillner at nhcc.net Wed Jan 26 22:59:42 2011 From: swillner at nhcc.net (Steve Willner) Date: Wed, 26 Jan 2011 16:59:42 -0500 Subject: [BLML] Exam question In-Reply-To: <4D3EE90C.5020002@t-online.de> References: <4D3D3B2F.5090107@skynet.be> <1PhIDl-0dAMts0@fwd00.aul.t-online.de> <4D3E0908.5090009@skynet.be> <000201cbbc6f$6360ef80$2a22ce80$@no> <4D3E9B17.7050704@skynet.be> <000401cbbc76$6e66ba20$4b342e60$@no> <4D3EE4D1.7010203@skynet.be> <4D3EE90C.5020002@t-online.de> Message-ID: <4D40994E.2030101@nhcc.net> On 1/25/2011 10:15 AM, Matthias Berghaus wrote: > the problem with this kind of question (like many other questions > related to claims and non-claims) is that they are completely impossible > to answer on paper, while no experienced director would get them wrong > at the table. I think this is the right answer. A lot depends on the atmosphere at the table and the exact words and gestures. If the statement -- while briefly flashing two cards of the remaining seven amid a generally light atmosphere at the table -- was "I don't think you're beating this one," I doubt anyone would think it's a claim. On the other hand, if the table is dead serious, and the statement is "I get the six I have plus these two," while folding the other cards, I'd expect unanimity that it is. You have to be there. As I understand Herman's story, the opponents at the table were in no doubt about what was going on. However, any statement about future tricks is at least coming close to being a claim, so if you want to joke about them, you have to be careful. > And even the "natives"... I have found a > website dedicated to the differences between British and American > English. One amusing one is "a rubber" (other than at bridge!). From l.kalbarczyk at gmail.com Wed Jan 26 23:00:38 2011 From: l.kalbarczyk at gmail.com (=?UTF-8?B?xYF1a2FzeiBLYWxiYXJjenlr?=) Date: Wed, 26 Jan 2011 23:00:38 +0100 Subject: [BLML] Difficult case from Polish Top Individual Tournament In-Reply-To: <002601cbbd43$75e595f0$61b0c1d0$@no> References: <4D3F5FA4.10000@gmail.com> <002f01cbbd3f$efaf6630$cf0e3290$@kooyman@worldonline.nl> <002601cbbd43$75e595f0$61b0c1d0$@no> Message-ID: <4D409986.5080207@gmail.com> W dniu 2011-01-26 11:26, Sven Pran pisze: > > I agree with Ton that it should be possible to adjust, but the way I > read Law 12 B2 this is specifically forbidden unless we can involve > (for instance) Law 23. > > In my opinion a better law 12B2 would have been: > > The Director may not award an adjusted score on the ground that the > rectification provided in these Laws is unduly severe to the offending > side or unduly advantageous to the non-offending side. > Rectification was provided only for *Card Prematurely Led, *shown-up during the auction*. *For this case we have an automatic rectification. BUT there was one more irregularity: opening screen gate before end of the auction period, not in accordance with the procedure, additionally. And this was the real reason of damage: mix of these irregularities led to the situation in which we NEED to rectificate the score. There is no rectification for mix of these cases, so we need to use 12A1, 84D, 12B1, 12C and the introduction to the BL, and the general idea of using this rules. Without screens it would be more clear, that player could have known so we could use #23 clearly. Ton said: I hope that TD?s agree that it should be possible to adjust the score in this case. ?K sorry 4 english once more ;) -------------- next part -------------- An HTML attachment was scrubbed... URL: http://lists.rtflb.org/pipermail/blml/attachments/20110126/18d0c952/attachment.html From swillner at nhcc.net Wed Jan 26 23:10:23 2011 From: swillner at nhcc.net (Steve Willner) Date: Wed, 26 Jan 2011 17:10:23 -0500 Subject: [BLML] Difficult case from Polish Top Individual Tournament In-Reply-To: <4D409986.5080207@gmail.com> References: <4D3F5FA4.10000@gmail.com> <002f01cbbd3f$efaf6630$cf0e3290$@kooyman@worldonline.nl> <002601cbbd43$75e595f0$61b0c1d0$@no> <4D409986.5080207@gmail.com> Message-ID: <4D409BCF.2060803@nhcc.net> On 1/26/2011 5:00 PM, ?ukasz Kalbarczyk wrote: > Rectification was provided only for *Card Prematurely Led, *shown-up > during the auction*. > *For this case we have an automatic rectification. BUT there was one > more irregularity: > opening screen gate before end of the auction period, I don't agree with this unless there's something special in the screen regulations. In the context of screens, "card exposed" applies only if the card becomes visible on the other side of the screen. In other words, I'd interpret the L24 infraction -- for which there is specific rectification -- as the combination of exposing the card and making it visible across the screen. With screens or without, I don't see any doubt that L23 needs to be looked at. Whether it applies or not depends on the circumstances and in particular the offender's hand. Could a villain, in the same circumstances, have thought it a good idea to halt the auction? From svenpran at online.no Wed Jan 26 23:18:39 2011 From: svenpran at online.no (Sven Pran) Date: Wed, 26 Jan 2011 23:18:39 +0100 Subject: [BLML] Difficult case from Polish Top Individual Tournament In-Reply-To: <4D404D1E.1010905@nhcc.net> References: <276CBE3DABAE4CA1950AF97DB7452E1C@coa12> <000601cbbcd9$394fd750$abef85f0$@no> <4B4F9B03CE154405B3CC0415A4D55A7C@coa12> <000701cbbcde$de194230$9a4bc690$@no> <4D404D1E.1010905@nhcc.net> Message-ID: <000b01cbbda6$fbbd2950$f3377bf0$@no> On Behalf Of Steve Willner > On 1/25/2011 5:26 PM, Sven Pran wrote: > > Law 84D does not apply because Law 24 provides a specific > > rectification > > > > Law 12 does not apply directly because of Law 12B2: > > I agree with this, too. > > > So the only way the Director may use Law 12 and award an adjusted > > score in your case is in my opinion through Law 23 as I have described > previously. > > L23 is directly referenced by L24B, so there's no doubt it applies. The interesting > question is whether it would apply in a L24A case; I think it should despite the lack > of specific reference. The original Law 23 only concerned forced pass damaging opponents and the title of this law was then "Damaging enforced pass". In 2007 Law 23 was extended to concern any irregularity that damaged opponents even after normal rectification. So I think we should believe that the reason why a reference to Law 23 is not present in Law 24A is that the rectification in 24A does not include a forced pass. This is in my opinion irrelevant; Law 23 as it is written now may kick in after any irregularity and may be used by the Director to adjust the score if he finds that the irregularity has damaged opponents in spite of the normal rectification provided he finds that the offender "could have known". So the important question is whether we rule that South "could have known at the time of his irregularity" that this could damage opponents. In a separate post I have repeated a suggestion I made long time ago: Law 12B2 would have served its purpose much better if it had been written: The Director may not award an adjusted score on the ground that the rectification provided in these Laws is unduly severe to an offending side or unduly advantageous to a non-offending side. That would have opened the door wide open for an adjustment in the present case even without involving Law 23. .....snip From Hermandw at skynet.be Thu Jan 27 00:35:37 2011 From: Hermandw at skynet.be (Herman De Wael) Date: Thu, 27 Jan 2011 00:35:37 +0100 Subject: [BLML] Exam question In-Reply-To: References: <4D3D3B2F.5090107@skynet.be> <1PhIDl-0dAMts0@fwd00.aul.t-online.de> <4D3E0908.5090009@skynet.be> <82D7A2DF-92A6-495E-ABC1-6C5C27F82B32@starpower.net> <4D3F0245.9010808@skynet.be> <18090296-A6E9-4502-B8D1-2863FD6C7EC5@starpower.net> <4D3FD99C.8070008@skynet.be> Message-ID: <4D40AFC9.6040202@skynet.be> Eric Landau wrote: > On Jan 26, 2011, at 3:29 AM, Harald Skj?ran wrote: > >> Surely, Herman didn't make a claim, that's pretty obvious from the OP. >> What he did was to tell the table that the hand only was about >> overtricks. >> No claim intended by Herman, no claim anticipated by the opponents and >> no claim according to L68A. > > If Herman's opponents didn't think Herman was claiming, why was there > a director call, a ruling, and a post to BLML? > There wasn't (a TD call or a ruling). > If Herman's opponents were to tell me that they didn't think Herman > was claiming, I would have absolutely no hesitation at all in > accepting their statement as prima facie evidence that Herman > "demonstrably did not intend to claim" [L68A]. They were there; I > wasn't. > But Eric, you don't get to that part of L68A, since it only applies when I show "my cards", not "any card". > > Eric Landau -- Herman De Wael Wilrijk Antwerpen Belgium From Hermandw at skynet.be Thu Jan 27 00:37:16 2011 From: Hermandw at skynet.be (Herman De Wael) Date: Thu, 27 Jan 2011 00:37:16 +0100 Subject: [BLML] Exam question In-Reply-To: <6ABF6AE1-1AAE-487D-A112-452A434EA4B2@starpower.net> References: <6ABF6AE1-1AAE-487D-A112-452A434EA4B2@starpower.net> Message-ID: <4D40B02C.5010904@skynet.be> Eric Landau wrote: > On Jan 25, 2011, at 7:15 PM, Jerry Fusselman wrote: > >> As I said before, I consider the phrase containing demonstrably to >> be irrelevant here, on the grounds that he only showed two of his >> seven remaining cards. I suppose this issue hinges on whether "his >> cards" refers to one or more, two or more, or all. I was assuming >> it means all. Has the issue been discussed before? > > It's not unusual (at least around here) to claim by exposing one's > remaining winners with a statement to the effect of, "I'll get > these," and fold the rest, conceding them. I did not fold the rest. > Sometimes the claimer > doesn't bother with the statement, and everyone nevertheless folds > their cards, scores it up, and moves on. If called to a table where > that had occurred, I would not be sympathetic to the player's > argument that he had not intended to claim, he was just making a > joke, and had not technically claimed, because he hadn't exposed his > entire hand. > > > Eric Landau -- Herman De Wael Wilrijk Antwerpen Belgium From swillner at nhcc.net Thu Jan 27 04:18:12 2011 From: swillner at nhcc.net (Steve Willner) Date: Wed, 26 Jan 2011 22:18:12 -0500 Subject: [BLML] Difficult case from Polish Top Individual Tournament In-Reply-To: <000b01cbbda6$fbbd2950$f3377bf0$@no> References: <276CBE3DABAE4CA1950AF97DB7452E1C@coa12> <000601cbbcd9$394fd750$abef85f0$@no> <4B4F9B03CE154405B3CC0415A4D55A7C@coa12> <000701cbbcde$de194230$9a4bc690$@no> <4D404D1E.1010905@nhcc.net> <000b01cbbda6$fbbd2950$f3377bf0$@no> Message-ID: <4D40E3F4.2020909@nhcc.net> On 1/26/2011 5:18 PM, Sven Pran wrote: > The original Law 23 only concerned forced pass damaging opponents and the > title of this law was then "Damaging enforced pass". Yes. The original L23 was present in 1987 and I think in 1975 and quite likely earlier still. > In 2007 Law 23 was extended to concern any irregularity that damaged > opponents even after normal rectification. What actually happened in 2007 was that L72B1, which originated in 1997, was merged into L23. The effect of this was as Sven wrote: any time an offender "could have known," there can be an adjusted score. > So I think we should believe that the reason why a reference to Law 23 is > not present in Law 24A is that the rectification in 24A does not include a > forced pass. I bet this is the right explanation. > So the important question is whether we rule that South "could have known at > the time of his irregularity" that this could damage opponents. The exact phrase is "could well damage." This is a little hard to interpret, but it's stronger than "could damage." Anything you do at the table _could_ damage opponents in some bizarre circumstances, but "could well" implies some rational expectation of damage. I don't have the 1997 Laws at hand, but my memory is that L72B1 had a "likely" in it. If so, that would have been a stronger requirement than the current one. > In a separate post I have repeated a suggestion I made long time ago: Law > 12B2 would have served its purpose much better if it had been written: > > The Director may not award an adjusted score on the ground that the > rectification provided in these Laws is unduly severe to an offending side > or unduly advantageous to a non-offending side. > > That would have opened the door wide open for an adjustment in the present > case even without involving Law 23. That would be stronger yet, but opinions will differ on whether it would be an improvement or not. From grandaeval at tiscali.co.uk Thu Jan 27 09:00:01 2011 From: grandaeval at tiscali.co.uk (Grattan) Date: Thu, 27 Jan 2011 08:00:01 -0000 Subject: [BLML] Difficult case from Polish Top Individual Tournament References: <276CBE3DABAE4CA1950AF97DB7452E1C@coa12> Message-ID: <3BA2F7D03BB741AB99288B2676A21E5B@Mildred> Grattan Endicott To: Sent: Tuesday, January 25, 2011 9:25 PM Subject: [BLML] Difficult case from Polish Top Individual Tournament > Hi all, I'm coming back from a long read-only hiatus. > > As could I see, dWS is like Lenin, evergreen :) > > To the point, I've got an interesting case for you. > > Individual with screens, all players are Premier League level, bidding > goes: > > N E S W > 2D* x 3H pass > pass 3NT pass pass > ?** > > *Minimulti, only weak two with one major > ** N starts to think > > Now S makes a face-up lead (heart jack) and opens the screen without any > warning. > > First part is obvious - L24B, N has to pass, HJ is a major penalty card > (will be led per force) and UI to N. > > In the actual deal heart lead beats 3NT by two tricks (it was bid without > heart stopper). > > So, there was an infraction, there was a rectification, but NOS is left > with > a poor score, and there was a non-zero chance, that N would bid 4H. In > other > words, 4H was a LA. > > What should the TD do, is he allowed to use specifically: > > L23 > L12A1 > L84D > > Take it as certain, that you, as a TD, strongly believe, that S simply > thought that the auction had ended (he is a honest person, it was a > friendly > (albeit strong) tournament, it was swiss and that table was far from top). > > I would heartily welcome all the opinions, especially from Grattan and > Ton, > because there is no consensus among our top TD staff and each side stands > strongly on their position. > +=+ As I understand it the tray was still with North and the bidding cards were still on it. So we have (i) a breach of screen regulations and (ii) a faced opening lead during the auction. It appears that West failed to stop North from opening the screen, possibly because the action was too quick for him. It is not stated whether the dummy had been exposed when the Director arrived at the table. Superficially it appears to me that North's enforced Pass damages the non-offending side by removing the possibility that North might bid. So I am wondering why there is no mention above of Law 24B. I have returned from an EBU L&E meeting to find over 100 new messages on my PC, so please excuse me if I have not taken in the facts correctly. ~ Grattan ~ +=+ From vitoldbr at yandex.ru Thu Jan 27 09:59:44 2011 From: vitoldbr at yandex.ru (vitoldbr) Date: Thu, 27 Jan 2011 11:59:44 +0300 Subject: [BLML] Difficult case from Polish Top Individual Tournament In-Reply-To: <003b01cbbd5c$8f6ff0b0$ae4fd210$@no> References: <4D3F5FA4.10000@gmail.com> <002f01cbbd3f$efaf6630$cf0e3290$@kooyman@worldonline.nl> <002601cbbd43$75e595f0$61b0c1d0$@no> <45900886.20110126143804@yandex.ru> <003b01cbbd5c$8f6ff0b0$ae4fd210$@no> Message-ID: <14695064.20110127115944@yandex.ru> An HTML attachment was scrubbed... URL: http://lists.rtflb.org/pipermail/blml/attachments/20110127/6e44c522/attachment.html From t.kooyman at worldonline.nl Thu Jan 27 10:14:23 2011 From: t.kooyman at worldonline.nl (ton) Date: Thu, 27 Jan 2011 10:14:23 +0100 Subject: [BLML] 25A In-Reply-To: <4D409BCF.2060803@nhcc.net> References: <4D3F5FA4.10000@gmail.com> <002f01cbbd3f$efaf6630$cf0e3290$@kooyman@worldonline.nl> <002601cbbd43$75e595f0$61b0c1d0$@no> <4D409986.5080207@gmail.com> <4D409BCF.2060803@nhcc.net> Message-ID: <002d01cbbe02$981535d0$c83fa170$@kooyman@worldonline.nl> I offer you two problems which came up in an internal Dutch discussion. May be they appeared in this forum before, in which case the answers should be clear. 1) North is dealer but West opens the auction by bidding 1H. North calls the TD and starting to explain the relevant law West tells the TD that he never wanted to bid 1S. Let us assume the TD believes this. 2) South dealer opens 1H, West pass and North bids 1NT. Now South says that he made a (mechanical ) mistake and puts the 1S card on top of his bidding cards on the table. What to do in both cases? ton -------------- next part -------------- An HTML attachment was scrubbed... URL: http://lists.rtflb.org/pipermail/blml/attachments/20110127/9ac750c3/attachment.html From gordonrainsford at btinternet.com Thu Jan 27 10:21:10 2011 From: gordonrainsford at btinternet.com (Gordon Rainsford) Date: Thu, 27 Jan 2011 09:21:10 +0000 Subject: [BLML] 25A In-Reply-To: <002d01cbbe02$981535d0$c83fa170$@kooyman@worldonline.nl> References: <4D3F5FA4.10000@gmail.com> <002f01cbbd3f$efaf6630$cf0e3290$@kooyman@worldonline.nl> <002601cbbd43$75e595f0$61b0c1d0$@no> <4D409986.5080207@gmail.com> <4D409BCF.2060803@nhcc.net> <002d01cbbe02$981535d0$c83fa170$@kooyman@worldonline.nl> Message-ID: <2183F2DE-BC07-411A-978D-62098EB03763@btinternet.com> On 27 Jan 2011, at 09:14, ton wrote: > > 1) > > North is dealer but West opens the auction by bidding 1H. North > calls the TD and starting to explain the relevant law West tells > the TD that he never wanted to bid 1S. Let us assume the TD > believes this. > Is there a typo here? -------------- next part -------------- An HTML attachment was scrubbed... URL: http://lists.rtflb.org/pipermail/blml/attachments/20110127/fbd34517/attachment.html From Hermandw at skynet.be Thu Jan 27 10:21:41 2011 From: Hermandw at skynet.be (Herman De Wael) Date: Thu, 27 Jan 2011 10:21:41 +0100 Subject: [BLML] 25A 1) Message-ID: <4D413925.8060608@skynet.be> Ton's post dropped into another thread, and it contained two separate questions. Allow me to repost and separate: I offer you two problems which came up in an internal Dutch discussion. May be they appeared in this forum before, in which case the answers should be clear. 1) North is dealer but West opens the auction by bidding 1H. North calls the TD and starting to explain the relevant law West tells the TD that he never wanted to bid 1S. Let us assume the TD believes this. What to do in both cases? ton -- Herman De Wael Wilrijk Antwerpen Belgium From Hermandw at skynet.be Thu Jan 27 10:22:25 2011 From: Hermandw at skynet.be (Herman De Wael) Date: Thu, 27 Jan 2011 10:22:25 +0100 Subject: [BLML] 25A 2) Message-ID: <4D413951.1080706@skynet.be> Repost in separate thread: I offer you two problems which came up in an internal Dutch discussion. May be they appeared in this forum before, in which case the answers should be clear. 2) South dealer opens 1H, West pass and North bids 1NT. Now South says that he made a (mechanical ) mistake and puts the 1S card on top of his bidding cards on the table. What to do in both cases? ton -- Herman De Wael Wilrijk Antwerpen Belgium From Hermandw at skynet.be Thu Jan 27 10:25:31 2011 From: Hermandw at skynet.be (Herman De Wael) Date: Thu, 27 Jan 2011 10:25:31 +0100 Subject: [BLML] 25A 2) In-Reply-To: <4D413951.1080706@skynet.be> References: <4D413951.1080706@skynet.be> Message-ID: <4D413A0B.6020300@skynet.be> This one seems easy: Herman De Wael wrote: > Repost in separate thread: > > I offer you two problems which came up in an internal Dutch discussion. > May be they appeared in this forum before, in which case the answers > should be clear. > > > > > 2) > > South dealer opens 1H, West pass and North bids 1NT. Now South says that > he made a (mechanical ) mistake and puts the 1S card on top of his > bidding cards on the table. > It is too late to change the call. 1H stands. 1S is UI to North, and AI to East/West. Did I miss something? > > > What to do in both cases? > > > > ton > > > > > > > > > No virus found in this incoming message. > Checked by AVG - www.avg.com > Version: 9.0.872 / Virus Database: 271.1.1/3405 - Release Date: 01/26/11 21:51:00 > -- Herman De Wael Wilrijk Antwerpen Belgium From Hermandw at skynet.be Thu Jan 27 10:29:25 2011 From: Hermandw at skynet.be (Herman De Wael) Date: Thu, 27 Jan 2011 10:29:25 +0100 Subject: [BLML] 25A 1) In-Reply-To: <4D413925.8060608@skynet.be> References: <4D413925.8060608@skynet.be> Message-ID: <4D413AF5.2010703@skynet.be> Herman De Wael wrote: > Ton's post dropped into another thread, and it contained two separate > questions. > > Allow me to repost and separate: > > I offer you two problems which came up in an internal Dutch discussion. > May be they appeared in this forum before, in which case the answers > should be clear. > > > > 1) > > North is dealer but West opens the auction by bidding 1H. North calls > the TD and starting to explain the relevant law West tells the TD that > he never wanted to bid 1S. Let us assume the TD believes this. > I presume there is a typo, and it should say either "he never wanted to bid 1H, but 1S" or "he wanted to bid 1S" > What to do in both cases? > > I think this is a L25A case. L25A does not state that it only applies to calls in turn, so it also applies to calls out of turn. East ("his partner") has not made a subsequent call (whether in turn or not), and if we rule that this change is made without pause for thought, we now simply rule on a 1S opening out of turn. > > ton > > > > > > > No virus found in this incoming message. > Checked by AVG - www.avg.com > Version: 9.0.872 / Virus Database: 271.1.1/3405 - Release Date: 01/26/11 21:51:00 > -- Herman De Wael Wilrijk Antwerpen Belgium From gordonrainsford at btinternet.com Thu Jan 27 10:30:48 2011 From: gordonrainsford at btinternet.com (Gordon Rainsford) Date: Thu, 27 Jan 2011 09:30:48 +0000 Subject: [BLML] 25A 1) In-Reply-To: <4D413925.8060608@skynet.be> References: <4D413925.8060608@skynet.be> Message-ID: <52A02A6D-6B22-4FEB-AD0E-27373FA5D923@btinternet.com> Since it's been reposted identically, I suppose it must be correct and not a typo as I first thought, though it doesn't make much sense to me. Do you mean that West opens 1H out of turn, thinks he has opened 1S and wants 1S to be treated as a mechanical error? Gordon Rainsford On 27 Jan 2011, at 09:21, Herman De Wael wrote: > Ton's post dropped into another thread, and it contained two separate > questions. > > Allow me to repost and separate: > > I offer you two problems which came up in an internal Dutch > discussion. > May be they appeared in this forum before, in which case the answers > should be clear. > > > > 1) > > North is dealer but West opens the auction by bidding 1H. North calls > the TD and starting to explain the relevant law West tells the TD that > he never wanted to bid 1S. Let us assume the TD believes this. > > What to do in both cases? > > > > ton > > > -- > Herman De Wael > Wilrijk Antwerpen Belgium > _______________________________________________ > Blml mailing list > Blml at rtflb.org > http://lists.rtflb.org/mailman/listinfo/blml From Hermandw at skynet.be Thu Jan 27 10:32:33 2011 From: Hermandw at skynet.be (Herman De Wael) Date: Thu, 27 Jan 2011 10:32:33 +0100 Subject: [BLML] quick question regarding national championships Message-ID: <4D413BB1.5090608@skynet.be> Quick question to my Western European neighbours (others may also answer, but this has EU implications). Do you allow non-members of your NBO to compete in your national championships? We must allow all European citizens to compete, but surely a rule requiring membership is allowable. But do you impose susch a rule? We have many national championships that are one-day, and free to all (no pre-qualification required). Should we allow tourists to play? -- Herman De Wael Wilrijk Antwerpen Belgium From harald.skjaran at gmail.com Thu Jan 27 11:10:56 2011 From: harald.skjaran at gmail.com (=?UTF-8?Q?Harald_Skj=C3=A6ran?=) Date: Thu, 27 Jan 2011 11:10:56 +0100 Subject: [BLML] quick question regarding national championships In-Reply-To: <4D413BB1.5090608@skynet.be> References: <4D413BB1.5090608@skynet.be> Message-ID: 2011/1/27 Herman De Wael : > Quick question to my Western European neighbours (others may also > answer, but this has EU implications). > > Do you allow non-members of your NBO to compete in your national > championships? > > We must allow all European citizens to compete, but surely a rule > requiring membership is allowable. But do you impose susch a rule? > > We have many national championships that are one-day, and free to all > (no pre-qualification required). Should we allow tourists to play? We have several national championships. At our annual festival, we have open national championships of different categories and classes. All of these are open to foreigners. No Norwegian Bridge Federation membership is requiered in these championships. At the same festival, we also have our Norwegian Pairs Championship. With 86 pairs competing. 71 of these pairs prequalify in the district federations' district pairs championships, 1 pair is prequalified as reigning champions, and 14 pairs qualify through our national swiss pairs championship just prior to the Nor. Pairs Championships. The swiss pairs ch.ship is open for foreign participation without Norwegian membership. But players who wants to qualify for the Nor. Pairs Championship must acquire a Norwegian membership and licence prior to the start of the swiss pairs. During our season we also have a Norwegian Club Teams Championship (6 rounds K.O. and 8 teams RR final) and our national league. These require membership in the club (Club Teams) or the district federation (the League) one represents, as well as a licence (top 3 divisions (of 4) in the League and from the 2nd K.O. round of the CTC). > > -- > Herman De Wael > Wilrijk Antwerpen Belgium > _______________________________________________ > Blml mailing list > Blml at rtflb.org > http://lists.rtflb.org/mailman/listinfo/blml > -- Kind regards, Harald Skj?ran From t.kooyman at worldonline.nl Thu Jan 27 11:11:42 2011 From: t.kooyman at worldonline.nl (ton) Date: Thu, 27 Jan 2011 11:11:42 +0100 Subject: [BLML] 25A In-Reply-To: <2183F2DE-BC07-411A-978D-62098EB03763@btinternet.com> References: <4D3F5FA4.10000@gmail.com> <002f01cbbd3f$efaf6630$cf0e3290$@kooyman@worldonline.nl> <002601cbbd43$75e595f0$61b0c1d0$@no> <4D409986.5080207@gmail.com> <4D409BCF.2060803@nhcc.net> <002d01cbbe02$981535d0$c83fa170$@kooyman@worldonline.nl> <2183F2DE-BC07-411A-978D-62098EB03763@btinternet.com> Message-ID: <001501cbbe0a$989eb780$c9dc2680$@kooyman@worldonline.nl> Yes: 'S' should be 'H'. He didn't want to bid 1H. ton Van: blml-bounces at rtflb.org [mailto:blml-bounces at rtflb.org] Namens Gordon Rainsford Verzonden: donderdag 27 januari 2011 10:21 Aan: Bridge Laws Mailing List Onderwerp: Re: [BLML] 25A On 27 Jan 2011, at 09:14, ton wrote: 1) North is dealer but West opens the auction by bidding 1H. North calls the TD and starting to explain the relevant law West tells the TD that he never wanted to bid 1S. Let us assume the TD believes this. Is there a typo here? _____ Geen virus gevonden in dit bericht. Gecontroleerd door AVG - www.avg.com Versie: 10.0.1204 / Virusdatabase: 1435/3405 - datum van uitgifte: 01/26/11 -------------- next part -------------- An HTML attachment was scrubbed... URL: http://lists.rtflb.org/pipermail/blml/attachments/20110127/dd3b9cc9/attachment.html From ziffbridge at t-online.de Thu Jan 27 11:11:53 2011 From: ziffbridge at t-online.de (Matthias Berghaus) Date: Thu, 27 Jan 2011 11:11:53 +0100 Subject: [BLML] 25A 2) In-Reply-To: <4D413A0B.6020300@skynet.be> References: <4D413951.1080706@skynet.be> <4D413A0B.6020300@skynet.be> Message-ID: <4D4144E9.6050801@t-online.de> Am 27.01.2011 10:25, schrieb Herman De Wael: > This one seems easy: > > Herman De Wael wrote: >> Repost in separate thread: >> >> I offer you two problems which came up in an internal Dutch discussion. >> May be they appeared in this forum before, in which case the answers >> should be clear. >> >> >> >> >> 2) >> >> South dealer opens 1H, West pass and North bids 1NT. Now South says that >> he made a (mechanical ) mistake and puts the 1S card on top of his >> bidding cards on the table. >> > > It is too late to change the call. 1H stands. > 1S is UI to North, and AI to East/West. > Did I miss something? > Yes. He actually placed 1S on the table. Therefore LHO may accept 1S. L25B1 does not say what to do when such a bid is accepted, so we have to take it from there. Looks like LHO may accept and make any legal bid over 1S. (Is 1NT now a cuebid? :-) ). If he doesn`t 1H stands (and so does 1NT), lots of UI. >> >> >> What to do in both cases? >> >> >> >> ton >> >> >> >> >> >> >> >> >> No virus found in this incoming message. >> Checked by AVG - www.avg.com >> Version: 9.0.872 / Virus Database: 271.1.1/3405 - Release Date: 01/26/11 21:51:00 >> > From ziffbridge at t-online.de Thu Jan 27 11:23:14 2011 From: ziffbridge at t-online.de (Matthias Berghaus) Date: Thu, 27 Jan 2011 11:23:14 +0100 Subject: [BLML] 25A In-Reply-To: <002d01cbbe02$981535d0$c83fa170$@kooyman@worldonline.nl> References: <4D3F5FA4.10000@gmail.com> <002f01cbbd3f$efaf6630$cf0e3290$@kooyman@worldonline.nl> <002601cbbd43$75e595f0$61b0c1d0$@no> <4D409986.5080207@gmail.com> <4D409BCF.2060803@nhcc.net> <002d01cbbe02$981535d0$c83fa170$@kooyman@worldonline.nl> Message-ID: <4D414792.4020602@t-online.de> Am 27.01.2011 10:14, schrieb ton: > > > I offer you two problems which came up in an internal Dutch discussion. May be they appeared in this forum before, in which case the answers should be clear. > > > > 1) > > North is dealer but West opens the auction by bidding 1H. North calls the TD and starting to explain the relevant law West tells the TD that he never wanted to bid 1S. Let us assume the TD believes this. > > Seems to me that it doesn`t matter much whether this is a typo or not. If the TD finds that there has been a mechanical error West gets to change his bid, then North may accept. If West made a mechanical _and_ a mental error it doesn`t matter much either. The TD finds out which bid the player wanted to extract from the box (may get a bit tricky here...), he gets to change it, then North may accept. Rest according to FLB. Stating something about 1S when 1H hit the table is not relevant, since he never meant to bid 1S. L25A does not say that he can correct bids in turn only.... > > 2) > > South dealer opens 1H, West pass and North bids 1NT. Now South says that he made a (mechanical ) mistake and puts the 1S card on top of his bidding cards > > on the table. > > > > What to do in both cases? > > > > ton > > > > > > > > > > > > _______________________________________________ > Blml mailing list > Blml at rtflb.org > http://lists.rtflb.org/mailman/listinfo/blml From henk at ripe.net Thu Jan 27 11:28:52 2011 From: henk at ripe.net (Henk Uijterwaal) Date: Thu, 27 Jan 2011 11:28:52 +0100 Subject: [BLML] quick question regarding national championships In-Reply-To: <4D413BB1.5090608@skynet.be> References: <4D413BB1.5090608@skynet.be> Message-ID: <4D4148E4.6080107@ripe.net> On 27/01/2011 10:32, Herman De Wael wrote: > Quick question to my Western European neighbours (others may also > answer, but this has EU implications). > > Do you allow non-members of your NBO to compete in your national > championships? AFAIK, no, membership is required for all national championships. In practice, this is a non-issue, as it is trivial to become a member regardless of where one lives or which citizenship one has. > We must allow all European citizens to compete, but surely a rule > requiring membership is allowable. But do you impose susch a rule? Is this an EU rule? Henk -- ------------------------------------------------------------------------------ Henk Uijterwaal Email: henk.uijterwaal(at)ripe.net RIPE Network Coordination Centre http://www.xs4all.nl/~henku P.O.Box 10096 Singel 258 Phone: +31.20.5354414 1001 EB Amsterdam 1016 AB Amsterdam Fax: +31.20.5354445 The Netherlands The Netherlands Mobile: +31.6.55861746 ------------------------------------------------------------------------------ I confirm today what I denied yesterday. Anonymous Politician. From jean-pierre.rocafort at meteo.fr Thu Jan 27 11:45:22 2011 From: jean-pierre.rocafort at meteo.fr (Jean-Pierre Rocafort) Date: Thu, 27 Jan 2011 11:45:22 +0100 Subject: [BLML] 25A 1) In-Reply-To: <4D413AF5.2010703@skynet.be> References: <4D413925.8060608@skynet.be> <4D413AF5.2010703@skynet.be> Message-ID: <4D414CC2.9030608@meteo.fr> Herman De Wael a ?crit : > Herman De Wael wrote: >> Ton's post dropped into another thread, and it contained two separate >> questions. >> >> Allow me to repost and separate: >> >> I offer you two problems which came up in an internal Dutch discussion. >> May be they appeared in this forum before, in which case the answers >> should be clear. >> >> >> >> 1) >> >> North is dealer but West opens the auction by bidding 1H. North calls >> the TD and starting to explain the relevant law West tells the TD that >> he never wanted to bid 1S. Let us assume the TD believes this. >> > > I presume there is a typo, and it should say either > > "he never wanted to bid 1H, but 1S" > > or > > "he wanted to bid 1S" maybe also, it was TD and not West who started to explain the relevant law? jpr > >> What to do in both cases? >> >> > > I think this is a L25A case. > L25A does not state that it only applies to calls in turn, so it also > applies to calls out of turn. East ("his partner") has not made a > subsequent call (whether in turn or not), and if we rule that this > change is made without pause for thought, we now simply rule on a 1S > opening out of turn. > > > >> ton >> > -- _______________________________________________ Jean-Pierre Rocafort METEO-FRANCE DSI/CM 42 Avenue Gaspard Coriolis 31057 Toulouse CEDEX Tph: 05 61 07 81 02 (33 5 61 07 81 02) Fax: 05 61 07 81 09 (33 5 61 07 81 09) e-mail: jean-pierre.rocafort at meteo.fr Serveur WWW METEO-France: http://www.meteo.fr _______________________________________________ From PeterEidt at t-online.de Thu Jan 27 12:35:33 2011 From: PeterEidt at t-online.de (Peter Eidt) Date: Thu, 27 Jan 2011 12:35:33 +0100 Subject: [BLML] =?utf-8?q?quick_question_regarding_national_championships?= In-Reply-To: <4D413BB1.5090608@skynet.be> References: <4D413BB1.5090608@skynet.be> Message-ID: <1PiQ8T-1cJH8a0@fwd02.aul.t-online.de> From: Herman De Wael > Quick question to my Western European neighbours (others may also > answer, but this has EU implications). > > Do you allow non-members of your NBO to compete in your national > championships? no In Germany participant inhaerently have to be members of the DBV. In case of foreign members there additional requirements regarding residence and duration of the membership. The board may give special case authirization. > We must allow all European citizens to compete, but surely a rule > requiring membership is allowable. But do you impose susch a rule? > > We have many national championships that are one-day, and free to all > (no pre-qualification required). Should we allow tourists to play? no for tourists there are have open tournaments nationl championships are for national members. From ehaa at starpower.net Thu Jan 27 15:35:06 2011 From: ehaa at starpower.net (Eric Landau) Date: Thu, 27 Jan 2011 09:35:06 -0500 Subject: [BLML] Exam question In-Reply-To: <4D40994E.2030101@nhcc.net> References: <4D3D3B2F.5090107@skynet.be> <1PhIDl-0dAMts0@fwd00.aul.t-online.de> <4D3E0908.5090009@skynet.be> <000201cbbc6f$6360ef80$2a22ce80$@no> <4D3E9B17.7050704@skynet.be> <000401cbbc76$6e66ba20$4b342e60$@no> <4D3EE4D1.7010203@skynet.be> <4D3EE90C.5020002@t-online.de> <4D40994E.2030101@nhcc.net> Message-ID: On Jan 26, 2011, at 4:59 PM, Steve Willner wrote: > On 1/25/2011 10:15 AM, Matthias Berghaus wrote: > >> the problem with this kind of question (like many other questions >> related to claims and non-claims) is that they are completely >> impossible >> to answer on paper, while no experienced director would get them >> wrong >> at the table. > > I think this is the right answer. A lot depends on the atmosphere at > the table and the exact words and gestures. If the statement -- while > briefly flashing two cards of the remaining seven amid a generally > light > atmosphere at the table -- was "I don't think you're beating this > one," > I doubt anyone would think it's a claim. On the other hand, if the > table is dead serious, and the statement is "I get the six I have plus > these two," while folding the other cards, I'd expect unanimity > that it > is. You have to be there. > > As I understand Herman's story, the opponents at the table were in no > doubt about what was going on. However, any statement about future > tricks is at least coming close to being a claim, so if you want to > joke > about them, you have to be careful. I agree with Matthias and Steve. Since Herman posted his original "exam question", we have been pecking at it like a flock of Secretary Birds. Did Herman mention a "specific number" of tricks? Did his action constitute a suggestion that play be curtailed? Did he "show his cards" by exposing two of them? We shouldn't care. It doesn't matter. We need to take a step back and consider what constitutes equity here. There are two possibilities for what actually went down at the table: (Scenario 1) Herman made what appeared to be a joke. His opponents called the TD, pulled out their lawbook, and argued that despite his intent, based on the narrow criteria delineated in L68A, his actions technically constituted a claim, and should therefore must be treated as such. I will find that the fact that Herman's opponents recognized that he did not intend to claim sufficiently demonstrates that he did not intend to claim, and rule in Herman's favor. (Scenario 2) Herman made what appeared to be a claim, and his opponents reacted to it (as by facing or folding their cards, reaching for their scorecards, asking to see Herman's hand, or whatever). Herman called the TD, claimed that he was just joking, pulled out his lawbook, and argued that despite appearances, based on the narrow criteria delineated in L68A, his actions technically did not consitute a claim, and should not be treated as such. I will find that even if Herman did not intend to claim, he could have known at the time of his action that his opponents might be misled into thinking he was claiming (as they did). Since the board will have been compromised, I adjudicate from the point of the irregularity. Not coincidentally, Herman's equity position under L23 is identical to what it would be if he had actually intended to claim, so his intention makes no difference to the adjudicated result. IOW, I rule against him. Herman will need to be careful to accept this ruling without argument. If he insists that he is absolutely 100% certain that at the time of his intended joke he did not step over the very fine line drawn by the narrow criteria delineated in L68A, I shall inquire as to how he can be so sure unless he was aware at the time of precisely where the line was and took pains to avoid crossing it. That would make his action into a deliberate and premeditated attempt to mislead his opponents into thinking he was claiming, a far more grievous violation than anything contemplated above. It has been my experience in 45 years of sitting on ACs that when one side presents a case based on what actually happened at the table, while the other side presents a case based on the precise, exact wording of some law or other, equity invariably lies with the former. Eric Landau 1107 Dale Drive Silver Spring MD 20910 ehaa at starpower.net From agot at ulb.ac.be Thu Jan 27 15:56:16 2011 From: agot at ulb.ac.be (Alain Gottcheiner) Date: Thu, 27 Jan 2011 15:56:16 +0100 Subject: [BLML] Exam question In-Reply-To: References: <4D3D3B2F.5090107@skynet.be> <1PhIDl-0dAMts0@fwd00.aul.t-online.de> <4D3E0908.5090009@skynet.be> <000201cbbc6f$6360ef80$2a22ce80$@no> <4D3E9B17.7050704@skynet.be> <000401cbbc76$6e66ba20$4b342e60$@no> <4D3EE4D1.7010203@skynet.be> <4D3EE90C.5020002@t-online.de> <4D40994E.2030101@nhcc.net> Message-ID: <4D418790.1040107@ulb.ac.be> Le 27/01/2011 15:35, Eric Landau a ?crit : > On Jan 26, 2011, at 4:59 PM, Steve Willner wrote: > >> On 1/25/2011 10:15 AM, Matthias Berghaus wrote: >> >>> the problem with this kind of question (like many other questions >>> related to claims and non-claims) is that they are completely >>> impossible >>> to answer on paper, while no experienced director would get them >>> wrong >>> at the table. >> I think this is the right answer. A lot depends on the atmosphere at >> the table and the exact words and gestures. If the statement -- while >> briefly flashing two cards of the remaining seven amid a generally >> light >> atmosphere at the table -- was "I don't think you're beating this >> one," >> I doubt anyone would think it's a claim. On the other hand, if the >> table is dead serious, and the statement is "I get the six I have plus >> these two," while folding the other cards, I'd expect unanimity >> that it >> is. You have to be there. >> >> As I understand Herman's story, the opponents at the table were in no >> doubt about what was going on. However, any statement about future >> tricks is at least coming close to being a claim, so if you want to >> joke >> about them, you have to be careful. > I agree with Matthias and Steve. Since Herman posted his original > "exam question", we have been pecking at it like a flock of Secretary > Birds. > > Did Herman mention a "specific number" of tricks? Did his action > constitute a suggestion that play be curtailed? Did he "show his > cards" by exposing two of them? > > We shouldn't care. It doesn't matter. We need to take a step back > and consider what constitutes equity here. > > There are two possibilities for what actually went down at the table: > > (Scenario 1) Herman made what appeared to be a joke. His opponents > called the TD, pulled out their lawbook, and argued that despite his > intent, based on the narrow criteria delineated in L68A, his actions > technically constituted a claim, and should therefore must be treated > as such. > > I will find that the fact that Herman's opponents recognized that he > did not intend to claim sufficiently demonstrates that he did not > intend to claim, and rule in Herman's favor. > > (Scenario 2) Herman made what appeared to be a claim, and his > opponents reacted to it (as by facing or folding their cards, > reaching for their scorecards, asking to see Herman's hand, or > whatever). Herman called the TD, claimed that he was just joking, > pulled out his lawbook, and argued that despite appearances, based on > the narrow criteria delineated in L68A, his actions technically did > not consitute a claim, and should not be treated as such. > > I will find that even if Herman did not intend to claim, he could > have known at the time of his action that his opponents might be > misled into thinking he was claiming (as they did). Since the board > will have been compromised, I adjudicate from the point of the > irregularity. Not coincidentally, Herman's equity position under L23 > is identical to what it would be if he had actually intended to > claim, so his intention makes no difference to the adjudicated > result. IOW, I rule against him. > > Herman will need to be careful to accept this ruling without > argument. If he insists that he is absolutely 100% certain that at > the time of his intended joke he did not step over the very fine line > drawn by the narrow criteria delineated in L68A, I shall inquire as > to how he can be so sure unless he was aware at the time of precisely > where the line was and took pains to avoid crossing it. That would > make his action into a deliberate and premeditated attempt to mislead > his opponents into thinking he was claiming, a far more grievous > violation than anything contemplated above. > > It has been my experience in 45 years of sitting on ACs that when one > side presents a case based on what actually happened at the table, > while the other side presents a case based on the precise, exact > wording of some law or other, equity invariably lies with the former. AG : I accept everything as said by Eric, but would like to add one thing : we're all more or less uncomfortable with laws that ask us to guess a player's intent or state of mind. So we shouldn't add to the number of such cases. Here we have a case where a player's acts are undistinguishable from one of the usual ways of claiming, e.g. showing the cards that will take tricks and only those. While not 100% lawful, it's widely used and simple of use, and nonsecreary-birds always accept it. It would need more than a guess to decide that Herman's intentions were different from this. Best regards Alain From darkbystry at wp.pl Thu Jan 27 17:31:07 2011 From: darkbystry at wp.pl (Maciej Bystrzejewski) Date: Thu, 27 Jan 2011 17:31:07 +0100 Subject: [BLML] Difficult case from Polish Top Individual Tournament References: <276CBE3DABAE4CA1950AF97DB7452E1C@coa12> <3BA2F7D03BB741AB99288B2676A21E5B@Mildred> Message-ID: Hi Grattan. ----- Original Message ----- From: "Grattan" To: "Bridge Laws Mailing List" Sent: Thursday, January 27, 2011 9:00 AM Subject: Re: [BLML] Difficult case from Polish Top Individual Tournament > > > Grattan Endicott **************************************************** > Skype directory: grattan.endicott > **************************************************** > "Sir, we are a nest of singing birds." > [Dr. Samuel Johnson, 1730] > ++++++++++++++++++++++++++++++++++ > ----- Original Message ----- > From: "Maciej Bystrzejewski" > To: > Sent: Tuesday, January 25, 2011 9:25 PM > Subject: [BLML] Difficult case from Polish Top Individual Tournament > > >> Hi all, I'm coming back from a long read-only hiatus. >> >> As could I see, dWS is like Lenin, evergreen :) >> >> To the point, I've got an interesting case for you. >> >> Individual with screens, all players are Premier League level, bidding >> goes: >> >> N E S W >> 2D* x 3H pass >> pass 3NT pass pass >> ?** >> >> *Minimulti, only weak two with one major >> ** N starts to think >> >> Now S makes a face-up lead (heart jack) and opens the screen without any >> warning. >> >> First part is obvious - L24B, N has to pass, HJ is a major penalty card >> (will be led per force) and UI to N. Here I mentioned L24B. >> >> In the actual deal heart lead beats 3NT by two tricks (it was bid without >> heart stopper). >> >> So, there was an infraction, there was a rectification, but NOS is left >> with >> a poor score, and there was a non-zero chance, that N would bid 4H. In >> other >> words, 4H was a LA. >> >> What should the TD do, is he allowed to use specifically: >> >> L23 >> L12A1 >> L84D >> >> Take it as certain, that you, as a TD, strongly believe, that S simply >> thought that the auction had ended (he is a honest person, it was a >> friendly >> (albeit strong) tournament, it was swiss and that table was far from >> top). >> >> I would heartily welcome all the opinions, especially from Grattan and >> Ton, >> because there is no consensus among our top TD staff and each side stands >> strongly on their position. >> > +=+ As I understand it the tray was still with North and the bidding cards > were still on it. So we have (i) a breach of screen regulations and (ii) a > faced opening lead during the auction. It appears that West failed to stop > North from opening the screen, possibly because the action was too quick > for him. It is not stated whether the dummy had been exposed when the > Director arrived at the table. > Superficially it appears to me that North's enforced Pass damages the > non-offending side by removing the possibility that North might bid. So I > am wondering why there is no mention above of Law 24B. > I have returned from an EBU L&E meeting to find over 100 new messages > on my PC, so please excuse me if I have not taken in the facts correctly. > ~ Grattan ~ +=+ Yes, the tray was on the NE side and N didn't bid yet. West was surprised by S actions and couldn't stop him from opening the screen. Dummy was not exposed, auction didn't end yet. The TD was called after the screen had been opened. The TD used L24B (look above, I put my comment in the place) - N was forced to pass, S had to lead his prematurely faced card. The problem is what next, cause as you've already noted the enforced pass damaged the NOS. regards, Maciej From swillner at nhcc.net Thu Jan 27 17:37:29 2011 From: swillner at nhcc.net (Steve Willner) Date: Thu, 27 Jan 2011 11:37:29 -0500 Subject: [BLML] quick question regarding national championships In-Reply-To: <4D413BB1.5090608@skynet.be> References: <4D413BB1.5090608@skynet.be> Message-ID: <4D419F49.4050808@nhcc.net> On 1/27/2011 4:32 AM, Herman De Wael wrote: > Do you allow non-members of your NBO to compete in your national > championships? For the ACBL, "yes" except for a very few events, but non-members pay a higher entry fee. Anyone can join the ACBL, of course, regardless of nationality or residence. The higher entry fee was intended to affect "life members of the ACBL" who don't pay dues -- oops, pardon me, "service fees" -- rather than foreign visitors, but it applies to both categories. From PeterEidt at t-online.de Thu Jan 27 19:10:09 2011 From: PeterEidt at t-online.de (Peter Eidt) Date: Thu, 27 Jan 2011 19:10:09 +0100 Subject: [BLML] =?utf-8?q?25A_2=29?= In-Reply-To: <4D4144E9.6050801@t-online.de> References: <4D4144E9.6050801@t-online.de> Message-ID: <1PiWIL-23qdns0@fwd06.aul.t-online.de> [Ton] I offer you two problems which came up in an internal Dutch discussion. May be they appeared in this forum before, in which case the answers should be clear. [Peter] rofl; I vote for the joke of the year ;-))) [Ton] South dealer opens 1H, West pass and North bids 1NT. Now South says that he made a (mechanical ) mistake and puts the 1S card on top of his bidding cards on the table. What to do? [Herman] It is too late to change the call. 1H stands. 1S is UI to North, and AI to East/West. Did I miss something? [Matthias] Yes. He actually placed 1S on the table. Therefore LHO may accept 1S. L25B1 does not say what to do when such a bid is accepted, so we have to take it from there. Looks like LHO may accept and make any legal bid over 1S. (Is 1NT now a cuebid? :-) ). If he doesn`t 1H stands (and so does 1NT), lots of UI. [Peter] I think the problematic parts of Law 25 are: a) Law 25 A2: "2. No substitution of call may be made when his partner has made a subsequent call."; and b) Law 25 B1: "A substituted call not permitted by A may be accepted by the offender?s LHO. (It is accepted if LHO calls intentionally over it.) The first call is then withdrawn, the second call stands and the auction continues." The question is, whether A2 is superior to B1 or vice versa. As B1 only mentions "not permitted by A" and (unfortunately ?) not "not permitted by A1" ISTM that B1 beats A2. If the lawmakers had in mind to "forbid" any "too late" substitutions of a call they easily had the chance to say "A1" instead of "A". So, I agree with Matthias. From PeterEidt at t-online.de Thu Jan 27 19:34:13 2011 From: PeterEidt at t-online.de (Peter Eidt) Date: Thu, 27 Jan 2011 19:34:13 +0100 Subject: [BLML] =?utf-8?q?25A_1=29?= Message-ID: <1PiWfd-1XOJto0@fwd11.aul.t-online.de> [Ton] North is dealer but West opens the auction by bidding 1H. North calls the TD and starting to explain the relevant law West tells the TD that he never wanted to bid 1H. Let us assume the TD believes this. What to do? [Peter] I agree with the previous posts. 1. East has not called yet. 2. OP says TD believes 1H was unintended. 3. If now TD deems that there was no pause for thought before West's statement "I never wanted to bid 1H", all 3 necessary conditions required in Law 25 A1 are fulfilled and the TD may allow 1H to be corrected under Law 25 A1. After that he goes on with / restarts his explanations about Laws 29A ff. From swillner at nhcc.net Thu Jan 27 19:52:02 2011 From: swillner at nhcc.net (Steve Willner) Date: Thu, 27 Jan 2011 13:52:02 -0500 Subject: [BLML] 25A 2) In-Reply-To: <4D4144E9.6050801@t-online.de> References: <4D413951.1080706@skynet.be> <4D413A0B.6020300@skynet.be> <4D4144E9.6050801@t-online.de> Message-ID: <4D41BED2.3070802@nhcc.net> >>South dealer opens 1H, West pass and North bids 1NT. Now South says that >> he made a (mechanical ) mistake and puts the 1S card on top of his >> bidding cards on the table. On 1/27/2011 5:11 AM, Matthias Berghaus wrote: > Yes. He actually placed 1S on the table. Therefore LHO may accept 1S. > L25B1 does not say what to do when such a bid is accepted, If you rule the 1S bid "made," isn't it out of turn (RHO's turn to call)? Also insufficient, of course. I'm not sure L25B is the right place to be looking. I guess what I'm suggesting is that 1S was _either_ a) a try to correct the initial 1H bid _or_ b) a bid out of turn. No doubt South meant it as a), but one of the questions here is whether South's intent matters or not. It seems to me that if you are ruling that intent does not matter, then you're at b). This is a great problem for BLML. From blml at arcor.de Thu Jan 27 19:59:14 2011 From: blml at arcor.de (Thomas Dehn) Date: Thu, 27 Jan 2011 19:59:14 +0100 (CET) Subject: [BLML] 25A 2) In-Reply-To: <4D41BED2.3070802@nhcc.net> References: <4D41BED2.3070802@nhcc.net> <4D413951.1080706@skynet.be> <4D413A0B.6020300@skynet.be> <4D4144E9.6050801@t-online.de> Message-ID: <320558990.286127.1296154754704.JavaMail.ngmail@webmail08.arcor-online.net> Steve Willner wrote: > >>South dealer opens 1H, West pass and North bids 1NT. Now South says that > >> he made a (mechanical ) mistake and puts the 1S card on top of his > >> bidding cards on the table. > > On 1/27/2011 5:11 AM, Matthias Berghaus wrote: > > Yes. He actually placed 1S on the table. Therefore LHO may accept 1S. > > L25B1 does not say what to do when such a bid is accepted, > > If you rule the 1S bid "made," isn't it out of turn (RHO's turn to > call)? Also insufficient, of course. I'm not sure L25B is the right > place to be looking. > > I guess what I'm suggesting is that 1S was _either_ a) a try to correct > the initial 1H bid _or_ b) a bid out of turn. No doubt South meant it > as a), but one of the questions here is whether South's intent matters > or not. It seems to me that if you are ruling that intent does not > matter, then you're at b). > > This is a great problem for BLML. Intent does not matter, as it is simply too late for S to change his call; his partner had already called. Thus this is an insufficient bid out of turn. Thomas From petrus at stift-kremsmuenster.at Thu Jan 27 20:11:35 2011 From: petrus at stift-kremsmuenster.at (Petrus Schuster OSB) Date: Thu, 27 Jan 2011 20:11:35 +0100 Subject: [BLML] quick question regarding national championships In-Reply-To: <4D413BB1.5090608@skynet.be> References: <4D413BB1.5090608@skynet.be> Message-ID: Am 27.01.2011, 10:32 Uhr, schrieb Herman De Wael : > Quick question to my Western European neighbours (others may also > answer, but this has EU implications). > > Do you allow non-members of your NBO to compete in your national > championships? > Open Pairs: no, only members Mixed Pairs: yes Teams: I could not find out - but I'll make sure it will be in this year's CoC, one way or the other. > We must allow all European citizens to compete, but surely a rule > requiring membership is allowable. But do you impose susch a rule? no. Foreign nationals who are members of the Austrian federation may freely play. > > We have many national championships that are one-day, and free to all > (no pre-qualification required). Should we allow tourists to play? > why not - they will pay for it! Regards, Petrus From darkbystry at wp.pl Thu Jan 27 20:22:09 2011 From: darkbystry at wp.pl (Maciej Bystrzejewski) Date: Thu, 27 Jan 2011 20:22:09 +0100 Subject: [BLML] quick question regarding national championships References: <4D413BB1.5090608@skynet.be> Message-ID: <09E233EEDC824F82B52C57EB13BB299B@coa12> ----- Original Message ----- From: "Herman De Wael" To: "blml" Sent: Thursday, January 27, 2011 10:32 AM Subject: [BLML] quick question regarding national championships > Quick question to my Western European neighbours (others may also > answer, but this has EU implications). > > Do you allow non-members of your NBO to compete in your national > championships? > > We must allow all European citizens to compete, but surely a rule > requiring membership is allowable. But do you impose susch a rule? > > We have many national championships that are one-day, and free to all > (no pre-qualification required). Should we allow tourists to play? In Poland you have to be a member of PBU (other players may participate in preeliminaries, but don't count as candidates for promotion), but you may be a foreigner. Top Russsians Gromov and Dubinin are members of our NBO, as well as top Lithuanians and Belarusians. regards, Maciej From grandaeval at tiscali.co.uk Thu Jan 27 23:13:52 2011 From: grandaeval at tiscali.co.uk (Grattan) Date: Thu, 27 Jan 2011 22:13:52 -0000 Subject: [BLML] 25A 2) References: <4D41BED2.3070802@nhcc.net> <4D413951.1080706@skynet.be><4D413A0B.6020300@skynet.be> <4D4144E9.6050801@t-online.de> <320558990.286127.1296154754704.JavaMail.ngmail@webmail08.arcor-online.net> Message-ID: <06A6849013844F45A323890B8F04F0A7@Mildred> Grattan Endicott To: Sent: Thursday, January 27, 2011 6:59 PM Subject: Re: [BLML] 25A 2) ....... > Intent does not matter, as it is simply > too late for S to change his call; his > partner had already called. > Thus this is an insufficient bid out of turn. > +=+ That is rather a bold statement. It may be true, but first we need to examine the bidding box regulations to see if these have anything relevant to say. ~ Grattan ~ +=+ From grandaeval at tiscali.co.uk Fri Jan 28 02:41:31 2011 From: grandaeval at tiscali.co.uk (Grattan) Date: Fri, 28 Jan 2011 01:41:31 -0000 Subject: [BLML] Difficult case from Polish Top Individual Tournament References: <276CBE3DABAE4CA1950AF97DB7452E1C@coa12><3BA2F7D03BB741AB99288B2676A21E5B@Mildred> Message-ID: <0587C1C70D7D47A48E8C23B8212A2ABB@Mildred> Grattan Endicott To: "Bridge Laws Mailing List" Sent: Thursday, January 27, 2011 4:31 PM Subject: Re: [BLML] Difficult case from Polish Top Individual Tournament > Hi Grattan. > > ----- Original Message ----- > From: "Grattan" > To: "Bridge Laws Mailing List" > Sent: Thursday, January 27, 2011 9:00 AM > Subject: Re: [BLML] Difficult case from Polish Top Individual Tournament > > >> >> >> Grattan Endicott> **************************************************** >> Skype directory: grattan.endicott >> **************************************************** >> "Sir, we are a nest of singing birds." >> [Dr. Samuel Johnson, 1730] >> ++++++++++++++++++++++++++++++++++ >> ----- Original Message ----- >> From: "Maciej Bystrzejewski" >> To: >> Sent: Tuesday, January 25, 2011 9:25 PM >> Subject: [BLML] Difficult case from Polish Top Individual Tournament >> >> >>> Hi all, I'm coming back from a long read-only hiatus. >>> >>> As could I see, dWS is like Lenin, evergreen :) >>> >>> To the point, I've got an interesting case for you. >>> >>> Individual with screens, all players are Premier League level, bidding >>> goes: >>> >>> N E S W >>> 2D* x 3H pass >>> pass 3NT pass pass >>> ?** >>> >>> *Minimulti, only weak two with one major >>> ** N starts to think >>> >>> Now S makes a face-up lead (heart jack) and opens the screen without any >>> warning. >>> >>> First part is obvious - L24B, N has to pass, HJ is a major penalty card >>> (will be led per force) and UI to N. > > Here I mentioned L24B. > >>> >>> In the actual deal heart lead beats 3NT by two tricks (it was bid >>> without >>> heart stopper). >>> >>> So, there was an infraction, there was a rectification, but NOS is left >>> with >>> a poor score, and there was a non-zero chance, that N would bid 4H. In >>> other >>> words, 4H was a LA. >>> >>> What should the TD do, is he allowed to use specifically: >>> >>> L23 >>> L12A1 >>> L84D >>> >>> Take it as certain, that you, as a TD, strongly believe, that S simply >>> thought that the auction had ended (he is a honest person, it was a >>> friendly >>> (albeit strong) tournament, it was swiss and that table was far from >>> top). >>> >>> I would heartily welcome all the opinions, especially from Grattan and >>> Ton, >>> because there is no consensus among our top TD staff and each side >>> stands >>> strongly on their position. >>> >> +=+ As I understand it the tray was still with North and the bidding >> cards >> were still on it. So we have (i) a breach of screen regulations and (ii) >> a >> faced opening lead during the auction. It appears that West failed to >> stop >> North from opening the screen, possibly because the action was too quick >> for him. It is not stated whether the dummy had been exposed when the >> Director arrived at the table. >> Superficially it appears to me that North's enforced Pass damages the >> non-offending side by removing the possibility that North might bid. So I >> am wondering why there is no mention above of Law 24B. >> I have returned from an EBU L&E meeting to find over 100 new messages >> on my PC, so please excuse me if I have not taken in the facts correctly. >> ~ Grattan ~ +=+ > > Yes, the tray was on the NE side and N didn't bid yet. West was surprised > by > S actions and couldn't stop him from opening the screen. Dummy was not > exposed, auction didn't end yet. The TD was called after the screen had > been > opened. > > The TD used L24B (look above, I put my comment in the place) - N was > forced > to pass, S had to lead his prematurely faced card. > > The problem is what next, cause as you've already noted the enforced pass > damaged the NOS. << +=+ If you think the NOS damaged (as I do) by the enforced Pass, and if you think Law 24B applies (as I do), then Law 24B refers the Director to Law 23. The intention of my remarks was to draw the attention of those who suggested Law 23 to the way they will get to it. ~ Grattan ~ +=+ p.s. East would be North's screenmate and is the one taken by surprise. From jfusselman at gmail.com Fri Jan 28 02:44:04 2011 From: jfusselman at gmail.com (Jerry Fusselman) Date: Thu, 27 Jan 2011 19:44:04 -0600 Subject: [BLML] Exam question In-Reply-To: <141014.85780.qm@web28501.mail.ukl.yahoo.com> References: <4D3E9ACD.5020602@skynet.be> <1Phfff-2FiDdA0@fwd06.aul.t-online.de> <4D3EE3E8.70308@skynet.be> <74D61212-85D5-42B3-9583-FFE26C078352@starpower.net> <201101260217.p0Q2H3WK019057@mail07.syd.optusnet.com.au> <141014.85780.qm@web28501.mail.ukl.yahoo.com> Message-ID: [Thomas Sowell] The rule of law---"a government of laws and not of men"---implies rules known in advance, applied generally, and constraining the rulers as well as the ruled. On Wed, Jan 26, 2011 at 4:59 AM, Nigel Guthrie wrote: > [Tony Musgrove] > I am sure Herman was making the Belgian equivalent > of a joke..very funny. ?However he had better not do it with me > directing. ?I was a playing director in 3rd position. ?Partner led, > and declarer immediiately started abusing partner with the usual > .."you should have bid x or y. ?We are making 6". ?I naturally > refused to play on and looked at declarer's hand, much to his > annoyance. ?"You claimed", said I, "and now I get to adjudicate", > > [Jerry Fusselman] > I am familiar with this style > of directing, but how worthwhile is it to ruin the board to make a > point? > > [Nigel] > IMO, the director should nip gross antisocial behaviour in > the bud. Tony seems to have dealt with the problem, lawfully, > with tact and good humour. > Please permit me to clarify. Tony broke none of the directors' proprieties, since there are none. If there were any (directors' proprieties), I would wish that they include these: 1. The director should do his best to save the board, and he must not deliberately ruin the board. 2. A playing director's behavior as a player at the table should be of a high standard to serve as a good example. Yes, the director should nip gross antisocial behavior in the bud, but he should do it by a method that players can employ even when a playing director does not happen to be sitting at the table. Players at the table should not be treated like household pets, where the punishment must be inflicted instantly or the animal won't know what the punishment is for. We have a PP mechanism in place; show the players how to use it after gross antisocial behavior. Tony's example was a bad example, for he cannot really want players to look at declarer's hand whenever they feel like it. And please don't ruin the board just because it satisfies your feeling of cosmic justice. There are three other players at the table---the playing director is not the most important. The director, writing that what he did was "natural", gave this example for other directors to emulate; I hope that they will not. And I don't see why Nigel calls the director's actions lawful. What do you think of these two directors' proprieties? If they are bad ideas, please explain why. From blml at arcor.de Fri Jan 28 07:51:17 2011 From: blml at arcor.de (Thomas Dehn) Date: Fri, 28 Jan 2011 07:51:17 +0100 (CET) Subject: [BLML] 25A 2) In-Reply-To: <06A6849013844F45A323890B8F04F0A7@Mildred> References: <06A6849013844F45A323890B8F04F0A7@Mildred> <4D41BED2.3070802@nhcc.net> <4D413951.1080706@skynet.be><4D413A0B.6020300@skynet.be> <4D4144E9.6050801@t-online.de> <320558990.286127.1296154754704.JavaMail.ngmail@webmail08.arcor-online.net> Message-ID: <882930570.304238.1296197477592.JavaMail.ngmail@webmail08.arcor-online.net> Grattan wrote: > ++++++++++++++++++++++++++++++++++ > ----- Original Message ----- > From: "Thomas Dehn" > To: > Sent: Thursday, January 27, 2011 6:59 PM > Subject: Re: [BLML] 25A 2) > ....... > > Intent does not matter, as it is simply > > too late for S to change his call; his > > partner had already called. > > Thus this is an insufficient bid out of turn. > > > +=+ That is rather a bold statement. It may be true, but > first we need to examine the bidding box regulations > to see if these have anything relevant to say. In principle correct, but I have never heard of any regulations where "He puts the 1S card on top of his bidding cards on the table." is not a made 1S bid in classic face to face bridge without screens. Thomas From jean-pierre.rocafort at meteo.fr Fri Jan 28 09:09:06 2011 From: jean-pierre.rocafort at meteo.fr (Jean-Pierre Rocafort) Date: Fri, 28 Jan 2011 09:09:06 +0100 Subject: [BLML] 25A 2) In-Reply-To: <882930570.304238.1296197477592.JavaMail.ngmail@webmail08.arcor-online.net> References: <06A6849013844F45A323890B8F04F0A7@Mildred><4D41BED2.3070802@nhcc .net><4D413951.1080706@skynet.be><4D413A0B.6020300@skynet.be><4D4144E9.6050 801@t-online.de><320558990.286127.1296154754704.JavaMail.ngmail@webmail08.a rcor-online.net> <882930570.304238.1296197477592.JavaMail.ngmail@webmail08.arcor-online.net> Message-ID: <4D4279A2.2040505@meteo.fr> Thomas Dehn a ?crit : > Grattan wrote: >> ++++++++++++++++++++++++++++++++++ >> ----- Original Message ----- >> From: "Thomas Dehn" >> To: >> Sent: Thursday, January 27, 2011 6:59 PM >> Subject: Re: [BLML] 25A 2) >> ....... >>> Intent does not matter, as it is simply >>> too late for S to change his call; his >>> partner had already called. >>> Thus this is an insufficient bid out of turn. >>> >> +=+ That is rather a bold statement. It may be true, but >> first we need to examine the bidding box regulations >> to see if these have anything relevant to say. > > In principle correct, but I have never > heard of any regulations where > "He puts the 1S card on top of his > bidding cards on the table." is not > a made 1S bid in classic face to face bridge > without screens. was the 1S card put just over the 1H card so that it was no more visible? or was it put aside, both visible? jpr > > > Thomas > -- _______________________________________________ Jean-Pierre Rocafort METEO-FRANCE DSI/CM 42 Avenue Gaspard Coriolis 31057 Toulouse CEDEX Tph: 05 61 07 81 02 (33 5 61 07 81 02) Fax: 05 61 07 81 09 (33 5 61 07 81 09) e-mail: jean-pierre.rocafort at meteo.fr Serveur WWW METEO-France: http://www.meteo.fr _______________________________________________ From PeterEidt at t-online.de Fri Jan 28 09:22:46 2011 From: PeterEidt at t-online.de (Peter Eidt) Date: Fri, 28 Jan 2011 09:22:46 +0100 Subject: [BLML] =?utf-8?q?25A_2=29?= In-Reply-To: <4D4279A2.2040505@meteo.fr> References: <4D4279A2.2040505@meteo.fr> Message-ID: <1PijbS-2FZh7A0@fwd09.aul.t-online.de> From: Jean-Pierre Rocafort > Thomas Dehn a ?crit : > > Grattan wrote: > >> ++++++++++++++++++++++++++++++++++ > >> ----- Original Message ----- > >> From: "Thomas Dehn" > >> To: > >> Sent: Thursday, January 27, 2011 6:59 PM > >> Subject: Re: [BLML] 25A 2) > >> ....... > >>> Intent does not matter, as it is simply > >>> too late for S to change his call; his > >>> partner had already called. > >>> Thus this is an insufficient bid out of turn. > >>> > >> +=+ That is rather a bold statement. It may be true, but > >> first we need to examine the bidding box regulations > >> to see if these have anything relevant to say. > > > > In principle correct, but I have never > > heard of any regulations where > > "He puts the 1S card on top of his > > bidding cards on the table." is not > > a made 1S bid in classic face to face bridge > > without screens. > > was the 1S card put just over the 1H card so that it was no more > visible? or was it put aside, both visible? [OP from Ton] [...] and puts the 1S card on top of his bidding cards on the table. From JffEstrsn at aol.com Fri Jan 28 09:35:26 2011 From: JffEstrsn at aol.com (Jeff Easterson) Date: Fri, 28 Jan 2011 09:35:26 +0100 Subject: [BLML] Exam question In-Reply-To: References: <4D3E9ACD.5020602@skynet.be> <1Phfff-2FiDdA0@fwd06.aul.t-online.de> <4D3EE3E8.70308@skynet.be> <74D61212-85D5-42B3-9583-FFE26C078352@starpower.net> <201101260217.p0Q2H3WK019057@mail07.syd.optusnet.com.au> <141014.85780.qm@web28501.mail.ukl.yahoo.com> Message-ID: <4D427FCE.1090005@aol.com> You keep saying that the TD "ruined" the board. I see no evidence for this. He made a decision and the bd. wasnot played. Thus you seem to think that whenever a TD makes a decision and gives an gives an adjusted score the bd, has been "ruined". (Then the laws, which mandate this in some situations, "ruin" the bd.) If so, then the player who created the situation for which the TD was called "ruined" the bd. Ciao, JE Am 28.01.2011 02:44, schrieb Jerry Fusselman: > [Thomas Sowell] The rule of law---"a government of laws and not of > men"---implies rules known in advance, applied generally, and > constraining the rulers as well as the ruled. > > On Wed, Jan 26, 2011 at 4:59 AM, Nigel Guthrie wrote: >> [Tony Musgrove] >> I am sure Herman was making the Belgian equivalent >> of a joke..very funny. However he had better not do it with me >> directing. I was a playing director in 3rd position. Partner led, >> and declarer immediiately started abusing partner with the usual >> .."you should have bid x or y. We are making 6". I naturally >> refused to play on and looked at declarer's hand, much to his >> annoyance. "You claimed", said I, "and now I get to adjudicate", >> >> [Jerry Fusselman] >> I am familiar with this style >> of directing, but how worthwhile is it to ruin the board to make a >> point? >> >> [Nigel] >> IMO, the director should nip gross antisocial behaviour in >> the bud. Tony seems to have dealt with the problem, lawfully, >> with tact and good humour. >> > Please permit me to clarify. Tony broke none of the directors' > proprieties, since there are none. If there were any (directors' > proprieties), I would wish that they include these: > > 1. The director should do his best to save the board, and he must not > deliberately ruin the board. > > 2. A playing director's behavior as a player at the table should be > of a high standard to serve as a good example. > > Yes, the director should nip gross antisocial behavior in the bud, but > he should do it by a method that players can employ even when a > playing director does not happen to be sitting at the table. Players > at the table should not be treated like household pets, where the > punishment must be inflicted instantly or the animal won't know what > the punishment is for. > > We have a PP mechanism in place; show the players how to use it after > gross antisocial behavior. Tony's example was a bad example, for he > cannot really want players to look at declarer's hand whenever they > feel like it. And please don't ruin the board just because it > satisfies your feeling of cosmic justice. There are three other > players at the table---the playing director is not the most important. > > The director, writing that what he did was "natural", gave this > example for other directors to emulate; I hope that they will not. > And I don't see why Nigel calls the director's actions lawful. > > What do you think of these two directors' proprieties? If they are > bad ideas, please explain why. > _______________________________________________ > Blml mailing list > Blml at rtflb.org > http://lists.rtflb.org/mailman/listinfo/blml > From jean-pierre.rocafort at meteo.fr Fri Jan 28 09:47:09 2011 From: jean-pierre.rocafort at meteo.fr (Jean-Pierre Rocafort) Date: Fri, 28 Jan 2011 09:47:09 +0100 Subject: [BLML] 25A 2) In-Reply-To: <1PijbS-2FZh7A0@fwd09.aul.t-online.de> References: <4D4279A2.2040505@meteo.fr> <1PijbS-2FZh7A0@fwd09.aul.t-online.de> Message-ID: <4D42828D.3090401@meteo.fr> Peter Eidt a ?crit : > From: Jean-Pierre Rocafort >> Thomas Dehn a ?crit : >>> Grattan wrote: >>>> ++++++++++++++++++++++++++++++++++ >>>> ----- Original Message ----- >>>> From: "Thomas Dehn" >>>> To: >>>> Sent: Thursday, January 27, 2011 6:59 PM >>>> Subject: Re: [BLML] 25A 2) >>>> ....... >>>>> Intent does not matter, as it is simply >>>>> too late for S to change his call; his >>>>> partner had already called. >>>>> Thus this is an insufficient bid out of turn. >>>>> >>>> +=+ That is rather a bold statement. It may be true, but >>>> first we need to examine the bidding box regulations >>>> to see if these have anything relevant to say. >>> In principle correct, but I have never >>> heard of any regulations where >>> "He puts the 1S card on top of his >>> bidding cards on the table." is not >>> a made 1S bid in classic face to face bridge >>> without screens. >> was the 1S card put just over the 1H card so that it was no more >> visible? or was it put aside, both visible? > > [OP from Ton] > [...] and puts the 1S card on top of his bidding cards on the > table. so there is no doubt the intent was a change of call? jpr -- _______________________________________________ Jean-Pierre Rocafort METEO-FRANCE DSI/CM 42 Avenue Gaspard Coriolis 31057 Toulouse CEDEX Tph: 05 61 07 81 02 (33 5 61 07 81 02) Fax: 05 61 07 81 09 (33 5 61 07 81 09) e-mail: jean-pierre.rocafort at meteo.fr Serveur WWW METEO-France: http://www.meteo.fr _______________________________________________ From PeterEidt at t-online.de Fri Jan 28 09:50:55 2011 From: PeterEidt at t-online.de (Peter Eidt) Date: Fri, 28 Jan 2011 09:50:55 +0100 Subject: [BLML] =?utf-8?q?25A_2=29?= In-Reply-To: <4D42828D.3090401@meteo.fr> References: <4D42828D.3090401@meteo.fr> Message-ID: <1Pik2h-03mYiW0@fwd11.aul.t-online.de> From: Jean-Pierre Rocafort > Peter Eidt a ?crit : > > From: Jean-Pierre Rocafort > >> Thomas Dehn a ?crit : > >>> Grattan wrote: > >>>> ++++++++++++++++++++++++++++++++++ > >>>> ----- Original Message ----- > >>>> From: "Thomas Dehn" > >>>> To: > >>>> Sent: Thursday, January 27, 2011 6:59 PM > >>>> Subject: Re: [BLML] 25A 2) > >>>> ....... > >>>>> Intent does not matter, as it is simply > >>>>> too late for S to change his call; his > >>>>> partner had already called. > >>>>> Thus this is an insufficient bid out of turn. > >>>>> > >>>> +=+ That is rather a bold statement. It may be true, but > >>>> first we need to examine the bidding box regulations > >>>> to see if these have anything relevant to say. > >>> In principle correct, but I have never > >>> heard of any regulations where > >>> "He puts the 1S card on top of his > >>> bidding cards on the table." is not > >>> a made 1S bid in classic face to face bridge > >>> without screens. > >> was the 1S card put just over the 1H card so that it was no more > >> visible? or was it put aside, both visible? > > > > [OP from Ton] > > [...] and puts the 1S card on top of his bidding cards on the > > table. > > so there is no doubt the intent was a change of call? [Peter] not for me ... reading the OP and the subject From Hermandw at skynet.be Fri Jan 28 09:51:51 2011 From: Hermandw at skynet.be (Herman De Wael) Date: Fri, 28 Jan 2011 09:51:51 +0100 Subject: [BLML] Exam question In-Reply-To: References: <4D3D3B2F.5090107@skynet.be> <1PhIDl-0dAMts0@fwd00.aul.t-online.de> <4D3E0908.5090009@skynet.be> <000201cbbc6f$6360ef80$2a22ce80$@no> <4D3E9B17.7050704@skynet.be> <000401cbbc76$6e66ba20$4b342e60$@no> <4D3EE4D1.7010203@skynet.be> <4D3EE90C.5020002@t-online.de> <4D40994E.2030101@nhcc.net> Message-ID: <4D4283A7.3090404@skynet.be> I actually agree with Eric here! Eric Landau wrote: > On Jan 26, 2011, at 4:59 PM, Steve Willner wrote: > >> On 1/25/2011 10:15 AM, Matthias Berghaus wrote: >> >>> the problem with this kind of question (like many other questions >>> related to claims and non-claims) is that they are completely >>> impossible >>> to answer on paper, while no experienced director would get them >>> wrong >>> at the table. >> >> I think this is the right answer. A lot depends on the atmosphere at >> the table and the exact words and gestures. If the statement -- while >> briefly flashing two cards of the remaining seven amid a generally >> light >> atmosphere at the table -- was "I don't think you're beating this >> one," >> I doubt anyone would think it's a claim. On the other hand, if the >> table is dead serious, and the statement is "I get the six I have plus >> these two," while folding the other cards, I'd expect unanimity >> that it >> is. You have to be there. >> >> As I understand Herman's story, the opponents at the table were in no >> doubt about what was going on. However, any statement about future >> tricks is at least coming close to being a claim, so if you want to >> joke >> about them, you have to be careful. > > I agree with Matthias and Steve. Since Herman posted his original > "exam question", we have been pecking at it like a flock of Secretary > Birds. > > Did Herman mention a "specific number" of tricks? Did his action > constitute a suggestion that play be curtailed? Did he "show his > cards" by exposing two of them? > > We shouldn't care. It doesn't matter. We need to take a step back > and consider what constitutes equity here. > Well, let's just say that it does matter, and that we shall use the facts to determine whether Herman did one of the three things above. After all, we should rule under L68A, not under "equity". But let's see what Eric really means: > There are two possibilities for what actually went down at the table: > > (Scenario 1) Herman made what appeared to be a joke. His opponents > called the TD, pulled out their lawbook, and argued that despite his > intent, based on the narrow criteria delineated in L68A, his actions > technically constituted a claim, and should therefore must be treated > as such. > > I will find that the fact that Herman's opponents recognized that he > did not intend to claim sufficiently demonstrates that he did not > intend to claim, and rule in Herman's favor. > Well, you'd be right in doing so, but you'd still need to pass my exam question and argue it within L68A. The answer "he did not mention a specific number of tricks, only a minimum number" should suffice to tell Herman's opponents that you have taken their request for a ruling seriously. > (Scenario 2) Herman made what appeared to be a claim, and his > opponents reacted to it (as by facing or folding their cards, > reaching for their scorecards, asking to see Herman's hand, or > whatever). Herman called the TD, claimed that he was just joking, > pulled out his lawbook, and argued that despite appearances, based on > the narrow criteria delineated in L68A, his actions technically did > not consitute a claim, and should not be treated as such. > Well, you would have the exact same situation. I would not all the TD, WE would. My opponents would be asking for it to be a claim, I would argue against it. The only difference between the two cases is the fact that in scenario one, they recognize I made a joke (but want to see the law applied anyway, which is their good right) while in scenario two, they don't recognize it to be a joke. Most likely, that would be because they failed to hear the "at least" in my statement about 8 tricks. It is of course free to the TD to rule whatever he wants on the basis of the facts as he sees them. > I will find that even if Herman did not intend to claim, he could > have known at the time of his action that his opponents might be > misled into thinking he was claiming (as they did). Since the board > will have been compromised, I adjudicate from the point of the > irregularity. Not coincidentally, Herman's equity position under L23 > is identical to what it would be if he had actually intended to > claim, so his intention makes no difference to the adjudicated > result. IOW, I rule against him. > And you would be right, perhaps, in scenario two. However, my exam question was about a scenario one case. If I did not make that perfectly clear, I apologize. But that is the case. Perhaps I should phrase the question thusly: Playing 2He, declarer has made the first six tricks. He shows AK of hearts and says "I don't think I'll be going down in this one". He plays to trick seven, but the opponents call the Director and ask him if this does not constitute a claim. When asked, the opponents admit that declarer clearly intended to play on and has said nothing about how many overtricks, if any, he is going to make. Give the TD's ruling, with Law reference. I think the ansswer to this question should be: Declarer did not mention a specific number of tricks, he did not show (all) his cards, and he did not suggest play was over, so according to L68A, he did not claim. > Herman will need to be careful to accept this ruling without > argument. If he insists that he is absolutely 100% certain that at > the time of his intended joke he did not step over the very fine line > drawn by the narrow criteria delineated in L68A, I shall inquire as > to how he can be so sure unless he was aware at the time of precisely > where the line was and took pains to avoid crossing it. That would > make his action into a deliberate and premeditated attempt to mislead > his opponents into thinking he was claiming, a far more grievous > violation than anything contemplated above. > > It has been my experience in 45 years of sitting on ACs that when one > side presents a case based on what actually happened at the table, > while the other side presents a case based on the precise, exact > wording of some law or other, equity invariably lies with the former. > So you gladly put away the lawbook when your feelings turn against ruling by the letter? Strange! > > Eric Landau -- Herman De Wael Wilrijk Antwerpen Belgium From Hermandw at skynet.be Fri Jan 28 09:53:44 2011 From: Hermandw at skynet.be (Herman De Wael) Date: Fri, 28 Jan 2011 09:53:44 +0100 Subject: [BLML] Exam question In-Reply-To: <4D418790.1040107@ulb.ac.be> References: <4D3D3B2F.5090107@skynet.be> <1PhIDl-0dAMts0@fwd00.aul.t-online.de> <4D3E0908.5090009@skynet.be> <000201cbbc6f$6360ef80$2a22ce80$@no> <4D3E9B17.7050704@skynet.be> <000401cbbc76$6e66ba20$4b342e60$@no> <4D3EE4D1.7010203@skynet.be> <4D3EE90C.5020002@t-online.de> <4D40994E.2030101@nhcc.net> <4D418790.1040107@ulb.ac.be> Message-ID: <4D428418.3080805@skynet.be> Alain Gottcheiner wrote: > > AG : I accept everything as said by Eric, but would like to add one > thing : we're all more or less uncomfortable with laws that ask us to > guess a player's intent or state of mind. So we shouldn't add to the > number of such cases. > Here we have a case where a player's acts are undistinguishable from one > of the usual ways of claiming, e.g. showing the cards that will take > tricks and only those. While not 100% lawful, it's widely used and > simple of use, and nonsecreary-birds always accept it. It would need > more than a guess to decide that Herman's intentions were different from > this. > Come on Alain, those actions are accompanied by a player putting away his cards, while mine were accompanied by a careful study of dummy, trying to figure out what to play next. The actions are very distinguishable, IMO! > Best regards > > Alain -- Herman De Wael Wilrijk Antwerpen Belgium From agot at ulb.ac.be Fri Jan 28 10:02:43 2011 From: agot at ulb.ac.be (Alain Gottcheiner) Date: Fri, 28 Jan 2011 10:02:43 +0100 Subject: [BLML] Exam question In-Reply-To: <4D4283A7.3090404@skynet.be> References: <4D3D3B2F.5090107@skynet.be> <1PhIDl-0dAMts0@fwd00.aul.t-online.de> <4D3E0908.5090009@skynet.be> <000201cbbc6f$6360ef80$2a22ce80$@no> <4D3E9B17.7050704@skynet.be> <000401cbbc76$6e66ba20$4b342e60$@no> <4D3EE4D1.7010203@skynet.be> <4D3EE90C.5020002@t-online.de> <4D40994E.2030101@nhcc.net> <4D4283A7.3090404@skynet.be> Message-ID: <4D428633.8080108@ulb.ac.be> Le 28/01/2011 9:51, Herman De Wael a ?crit : > I > Perhaps I should phrase the question thusly: > > Playing 2He, declarer has made the first six tricks. He shows AK of > hearts and says "I don't think I'll be going down in this one". He plays > to trick seven, but the opponents call the Director and ask him if this > does not constitute a claim. When asked, the opponents admit that > declarer clearly intended to play on and has said nothing about how many > overtricks, if any, he is going to make. > Give the TD's ruling, with Law reference. > > I think the ansswer to this question should be: > Declarer did not mention a specific number of tricks, he did not show > (all) his cards, and he did not suggest play was over, so according to > L68A, he did not claim. AG : notice that the wording in TFLB ("curtailed") doesn't make it necessary that one suggest that play is over ; it only makes it necessary that declarer suggested accelerating play. Also notice that the interpretation of "showing one's cards" as "showing all of them" is personal to Herman. The sentence "it echoes my thoughts" doesn't mean "all of them". Herman seems to be confusing collective forms with plural forms. He might be excused, as this distinction is very typical of the English language. Of course, based on the twin interpretation of "curtailed" as "over" and "showing one's cards" as "all of them", Herman is right ; simply, he is not right to interpret them in that way. Right deduction from wrom premises causes wrong conclusion, not unsurprisingly. Best regards Alain From agot at ulb.ac.be Fri Jan 28 10:05:19 2011 From: agot at ulb.ac.be (Alain Gottcheiner) Date: Fri, 28 Jan 2011 10:05:19 +0100 Subject: [BLML] Exam question In-Reply-To: <4D428418.3080805@skynet.be> References: <4D3D3B2F.5090107@skynet.be> <1PhIDl-0dAMts0@fwd00.aul.t-online.de> <4D3E0908.5090009@skynet.be> <000201cbbc6f$6360ef80$2a22ce80$@no> <4D3E9B17.7050704@skynet.be> <000401cbbc76$6e66ba20$4b342e60$@no> <4D3EE4D1.7010203@skynet.be> <4D3EE90C.5020002@t-online.de> <4D40994E.2030101@nhcc.net> <4D418790.1040107@ulb.ac.be> <4D428418.3080805@skynet.be> Message-ID: <4D4286CF.10807@ulb.ac.be> Le 28/01/2011 9:53, Herman De Wael a ?crit : > Alain Gottcheiner wrote: >> AG : I accept everything as said by Eric, but would like to add one >> thing : we're all more or less uncomfortable with laws that ask us to >> guess a player's intent or state of mind. So we shouldn't add to the >> number of such cases. >> Here we have a case where a player's acts are undistinguishable from one >> of the usual ways of claiming, e.g. showing the cards that will take >> tricks and only those. While not 100% lawful, it's widely used and >> simple of use, and nonsecreary-birds always accept it. It would need >> more than a guess to decide that Herman's intentions were different from >> this. >> > Come on Alain, those actions are accompanied by a player putting away > his cards, while mine were accompanied by a careful study of dummy, > trying to figure out what to play next. > The actions are very distinguishable, IMO! Indeed. If, as we've all done (but shouldn't have), you had merely said "this contract isn't going down", without showing your cards, while continuing to lstudy dummy, I wouldn't rule that you claimed. From Hermandw at skynet.be Fri Jan 28 11:44:49 2011 From: Hermandw at skynet.be (Herman De Wael) Date: Fri, 28 Jan 2011 11:44:49 +0100 Subject: [BLML] Exam question In-Reply-To: <4D4286CF.10807@ulb.ac.be> References: <4D3D3B2F.5090107@skynet.be> <1PhIDl-0dAMts0@fwd00.aul.t-online.de> <4D3E0908.5090009@skynet.be> <000201cbbc6f$6360ef80$2a22ce80$@no> <4D3E9B17.7050704@skynet.be> <000401cbbc76$6e66ba20$4b342e60$@no> <4D3EE4D1.7010203@skynet.be> <4D3EE90C.5020002@t-online.de> <4D40994E.2030101@nhcc.net> <4D418790.1040107@ulb.ac.be> <4D428418.3080805@skynet.be> <4D4286CF.10807@ulb.ac.be> Message-ID: <4D429E21.8020205@skynet.be> Alain Gottcheiner wrote: > Le 28/01/2011 9:53, Herman De Wael a ?crit : >>> Here we have a case where a player's acts are undistinguishable from one >>> of the usual ways of claiming, e.g. showing the cards that will take >>> tricks and only those. While not 100% lawful, it's widely used and >>> simple of use, and nonsecreary-birds always accept it. It would need >>> more than a guess to decide that Herman's intentions were different from >>> this. >>> >> Come on Alain, those actions are accompanied by a player putting away >> his cards, while mine were accompanied by a careful study of dummy, >> trying to figure out what to play next. >> The actions are very distinguishable, IMO! > Indeed. > > If, as we've all done (but shouldn't have), you had merely said "this > contract isn't going down", without showing your cards, while continuing > to lstudy dummy, I wouldn't rule that you claimed. But Alain, then you are not ruling according to L68A. L68A clearly states "shows his cards". That must mean all of them, because otherwise the lawmakers would have written "shows any card". The fact that you rule "claim" in another case where a player shows 2 cards (and for good reason), should not imply that you should ignore this law here! Maybe your other ruling is also wrong? A player who shows some of his cards, and folds the rest, and puts them into the board has not "shown his cards", but he has "suggested play be curtailed". That is the reason to rule that he has claimed (that and the fact that he tells you he has), not the fact that he's shown two cards. -- Herman De Wael Wilrijk Antwerpen Belgium From Hermandw at skynet.be Fri Jan 28 11:48:57 2011 From: Hermandw at skynet.be (Herman De Wael) Date: Fri, 28 Jan 2011 11:48:57 +0100 Subject: [BLML] Exam question In-Reply-To: <4D428633.8080108@ulb.ac.be> References: <4D3D3B2F.5090107@skynet.be> <1PhIDl-0dAMts0@fwd00.aul.t-online.de> <4D3E0908.5090009@skynet.be> <000201cbbc6f$6360ef80$2a22ce80$@no> <4D3E9B17.7050704@skynet.be> <000401cbbc76$6e66ba20$4b342e60$@no> <4D3EE4D1.7010203@skynet.be> <4D3EE90C.5020002@t-online.de> <4D40994E.2030101@nhcc.net> <4D4283A7.3090404@skynet.be> <4D428633.8080108@ulb.ac.be> Message-ID: <4D429F19.3090305@skynet.be> Alain Gottcheiner wrote: > > Also notice that the interpretation of "showing one's cards" as "showing > all of them" is personal to Herman. The sentence "it echoes my thoughts" > doesn't mean "all of them". Herman seems to be confusing collective > forms with plural forms. He might be excused, as this distinction is > very typical of the English language. > Sorry Alain, but thoughts and cards are not the same thing. Showing that a collective form in one sentence does not imply the full set does not mean that in this sentence it does or it doesn't. Clearly the lawmakers' intent is that all cards must be shown. Otherwise, the sentence "shows a card" would have been used. -- Herman De Wael Wilrijk Antwerpen Belgium From l.kalbarczyk at gmail.com Fri Jan 28 12:15:33 2011 From: l.kalbarczyk at gmail.com (=?UTF-8?B?xYF1a2FzeiBLYWxiYXJjenlr?=) Date: Fri, 28 Jan 2011 12:15:33 +0100 Subject: [BLML] Exam question In-Reply-To: <4D429E21.8020205@skynet.be> References: <4D3D3B2F.5090107@skynet.be> <1PhIDl-0dAMts0@fwd00.aul.t-online.de> <4D3E0908.5090009@skynet.be> <000201cbbc6f$6360ef80$2a22ce80$@no> <4D3E9B17.7050704@skynet.be> <000401cbbc76$6e66ba20$4b342e60$@no> <4D3EE4D1.7010203@skynet.be> <4D3EE90C.5020002@t-online.de> <4D40994E.2030101@nhcc.net> <4D418790.1040107@ulb.ac.be> <4D428418.3080805@skynet.be> <4D4286CF.10807@ulb.ac.be> <4D429E21.8020205@skynet.be> Message-ID: <4D42A555.5010007@gmail.com> W dniu 2011-01-28 11:44, Herman De Wael pisze: > > But Alain, then you are not ruling according to L68A. > L68A clearly states "shows his cards". That must mean all of them, > because otherwise the lawmakers would have written "shows any card". > The fact that you rule "claim" in another case where a player shows 2 > cards (and for good reason), should not imply that you should ignore > this law here! Or /any card/, otherwise the lawmakers would have written "shows all cards". ?K From agot at ulb.ac.be Fri Jan 28 13:46:14 2011 From: agot at ulb.ac.be (Alain Gottcheiner) Date: Fri, 28 Jan 2011 13:46:14 +0100 Subject: [BLML] Exam question In-Reply-To: <4D429F19.3090305@skynet.be> References: <4D3D3B2F.5090107@skynet.be> <1PhIDl-0dAMts0@fwd00.aul.t-online.de> <4D3E0908.5090009@skynet.be> <000201cbbc6f$6360ef80$2a22ce80$@no> <4D3E9B17.7050704@skynet.be> <000401cbbc76$6e66ba20$4b342e60$@no> <4D3EE4D1.7010203@skynet.be> <4D3EE90C.5020002@t-online.de> <4D40994E.2030101@nhcc.net> <4D4283A7.3090404@skynet.be> <4D428633.8080108@ulb.ac.be> <4D429F19.3090305@skynet.be> Message-ID: <4D42BA96.3020300@ulb.ac.be> Le 28/01/2011 11:48, Herman De Wael a ?crit : > Alain Gottcheiner wrote: >> Also notice that the interpretation of "showing one's cards" as "showing >> all of them" is personal to Herman. The sentence "it echoes my thoughts" >> doesn't mean "all of them". Herman seems to be confusing collective >> forms with plural forms. He might be excused, as this distinction is >> very typical of the English language. >> > Sorry Alain, but thoughts and cards are not the same thing. Showing that > a collective form in one sentence does not imply the full set does not > mean that in this sentence it does or it doesn't. Clearly the lawmakers' > intent is that all cards must be shown. Otherwise, the sentence "shows a > card" would have been used. > Want a more practical example ?. "my neighbours drive me mad" ; it could well be that only the right-side neighbours are thusly pointed out. Really, it's a feature of the English language - and if it has been translated too litterally in other languages, too bad. From Hermandw at skynet.be Fri Jan 28 14:13:54 2011 From: Hermandw at skynet.be (Herman De Wael) Date: Fri, 28 Jan 2011 14:13:54 +0100 Subject: [BLML] Exam question In-Reply-To: <4D42A555.5010007@gmail.com> References: <4D3D3B2F.5090107@skynet.be> <1PhIDl-0dAMts0@fwd00.aul.t-online.de> <4D3E0908.5090009@skynet.be> <000201cbbc6f$6360ef80$2a22ce80$@no> <4D3E9B17.7050704@skynet.be> <000401cbbc76$6e66ba20$4b342e60$@no> <4D3EE4D1.7010203@skynet.be> <4D3EE90C.5020002@t-online.de> <4D40994E.2030101@nhcc.net> <4D418790.1040107@ulb.ac.be> <4D428418.3080805@skynet.be> <4D4286CF.10807@ulb.ac.be> <4D429E21.8020205@skynet.be> <4D42A555.5010007@gmail.com> Message-ID: <4D42C112.2010107@skynet.be> ?ukasz Kalbarczyk wrote: > W dniu 2011-01-28 11:44, Herman De Wael pisze: >> >> But Alain, then you are not ruling according to L68A. >> L68A clearly states "shows his cards". That must mean all of them, >> because otherwise the lawmakers would have written "shows any card". >> The fact that you rule "claim" in another case where a player shows 2 >> cards (and for good reason), should not imply that you should ignore >> this law here! > Or /any card/, otherwise the lawmakers would have written "shows all cards". > > ?K > Yes Lucas, valid point, but: If the intent had been to include the case where only some cards are shown, that should also include one card. You cannot read the case for one card into the phrase "shows his cards". It would seem strange to me that the lawmakers would have intended to include the case for AK, but not for A. If I had cashed the King of hearts and then shown the Ace, would that have changed the ruling? Certainly not, whichever rulling one favours. I would like to hear input on this from English speakers: "shows his cards" means all of them? -- Herman De Wael Wilrijk Antwerpen Belgium From svenpran at online.no Fri Jan 28 15:09:53 2011 From: svenpran at online.no (Sven Pran) Date: Fri, 28 Jan 2011 15:09:53 +0100 Subject: [BLML] Exam question In-Reply-To: <4D42C112.2010107@skynet.be> References: <4D3D3B2F.5090107@skynet.be> <1PhIDl-0dAMts0@fwd00.aul.t-online.de> <4D3E0908.5090009@skynet.be> <000201cbbc6f$6360ef80$2a22ce80$@no> <4D3E9B17.7050704@skynet.be> <000401cbbc76$6e66ba20$4b342e60$@no> <4D3EE4D1.7010203@skynet.be> <4D3EE90C.5020002@t-online.de> <4D40994E.2030101@nhcc.net> <4D418790.1040107@ulb.ac.be> <4D428418.3080805@skynet.be> <4D4286CF.10807@ulb.ac.be> <4D429E21.8020205@skynet.be> <4D42A555.5010007@gmail.com> <4D42C112.2010107@skynet.be> Message-ID: <000201cbbef5$098865f0$1c9931d0$@no> On Behalf Of Herman De Wael ............. > Yes Lucas, valid point, but: > If the intent had been to include the case where only some cards are shown, that > should also include one card. You cannot read the case for one card into the > phrase "shows his cards". It would seem strange to me that the lawmakers would > have intended to include the case for AK, but not for A. If I had cashed the King of > hearts and then shown the Ace, would that have changed the ruling? Certainly > not, whichever rulling one favours. Do you assert that you have not claimed just the two tricks (and conceded the rest) if, with say 5 cards left, you show two aces and say words to the effect that you will take these winners, acting in a manner giving the impression that you do not expect to take any further tricks after that? You have not stated any specific number of tricks; you have not shown all your cards and you have not suggested that play should be curtailed. Still, in my experience that is a very frequent way for players to claim some, but not all the remaining tricks. My impression from your OP was that you showed your two winners and claimed the corresponding number of tricks. To me that is a claim unless you "demonstrably did not intend to claim". However, the manner in which you acted is not apparent from your OP; if your opponents did not think that you intended to claim then neither would I rule so. But then I should be extremely surprised if I had been called to the table. From Hermandw at skynet.be Fri Jan 28 15:50:41 2011 From: Hermandw at skynet.be (Herman De Wael) Date: Fri, 28 Jan 2011 15:50:41 +0100 Subject: [BLML] Exam question In-Reply-To: <000201cbbef5$098865f0$1c9931d0$@no> References: <4D3D3B2F.5090107@skynet.be> <1PhIDl-0dAMts0@fwd00.aul.t-online.de> <4D3E0908.5090009@skynet.be> <000201cbbc6f$6360ef80$2a22ce80$@no> <4D3E9B17.7050704@skynet.be> <000401cbbc76$6e66ba20$4b342e60$@no> <4D3EE4D1.7010203@skynet.be> <4D3EE90C.5020002@t-online.de> <4D40994E.2030101@nhcc.net> <4D418790.1040107@ulb.ac.be> <4D428418.3080805@skynet.be> <4D4286CF.10807@ulb.ac.be> <4D429E21.8020205@skynet.be> <4D42A555.5010007@gmail.com> <4D42C112.2010107@skynet.be> <000201cbbef5$098865f0$1c9931d0$@no> Message-ID: <4D42D7C1.8040907@skynet.be> No Sven, that is not what I mean: Sven Pran wrote: > On Behalf Of Herman De Wael ............. >> Yes Lucas, valid point, but: If the intent had been to include the >> case where only some cards are shown, that should also include one >> card. You cannot read the case for one card into the phrase "shows >> his cards". It would seem strange to me that the lawmakers would >> have intended to include the case for AK, but not for A. If I had >> cashed the King of hearts and then shown the Ace, would that have >> changed the ruling? Certainly not, whichever rulling one favours. > > Do you assert that you have not claimed just the two tricks (and > conceded the rest) if, with say 5 cards left, you show two aces and > say words to the effect that you will take these winners, acting in a > manner giving the impression that you do not expect to take any > further tricks after that? No, I do not assert that. For one thing, the difference being that the "manner" which you describe can be judged to "suggest that play be curtailed". Remember that L68A has three parts, one of which is enough to make an action a claim. The showing of the two Aces does not make it a claim, and my assertion that showing AK of trumps does not make it a claim is not contradictory to the fact that what you describe IS a claim. > > You have not stated any specific number of tricks; you have not shown > all your cards and you have not suggested that play should be > curtailed. Still, in my experience that is a very frequent way for > players to claim some, but not all the remaining tricks. > Well, by not playing on, they have suggested that play be curtailed. Anyway, they will not contend that they haven't! > My impression from your OP was that you showed your two winners and > claimed the corresponding number of tricks. To me that is a claim > unless you "demonstrably did not intend to claim". > Which I indeed did not - but that is not the point. My point is that showing two winners is not equal to conceding five losers, and that my actions do not refer to a "specific" number of tricks. No-one has yet answered that one, actually. Do you agree that the word "specific" means that when it is clear that a player has only stated a minimum number of tricks he shall make, no claim has been made? > However, the manner in which you acted is not apparent from your OP; > if your opponents did not think that you intended to claim then > neither would I rule so. But then I should be extremely surprised if > I had been called to the table. > And that is of course the point. When the opponents do not see it as a claim, there will be no director call. But what if, for the same joke, the opponents ask the director to make a ruling? Suppose the Director establishes the facts as being: I showed 2 cards, I stated that I shall make at least 8 tricks, and I did not suggest play be curtailed. Have I claimed? -- Herman De Wael Wilrijk Antwerpen Belgium From svenpran at online.no Fri Jan 28 16:16:06 2011 From: svenpran at online.no (Sven Pran) Date: Fri, 28 Jan 2011 16:16:06 +0100 Subject: [BLML] Exam question In-Reply-To: <4D42D7C1.8040907@skynet.be> References: <4D3D3B2F.5090107@skynet.be> <1PhIDl-0dAMts0@fwd00.aul.t-online.de> <4D3E0908.5090009@skynet.be> <000201cbbc6f$6360ef80$2a22ce80$@no> <4D3E9B17.7050704@skynet.be> <000401cbbc76$6e66ba20$4b342e60$@no> <4D3EE4D1.7010203@skynet.be> <4D3EE90C.5020002@t-online.de> <4D40994E.2030101@nhcc.net> <4D418790.1040107@ulb.ac.be> <4D428418.3080805@skynet.be> <4D4286CF.10807@ulb.ac.be> <4D429E21.8020205@skynet.be> <4D42A555.5010007@gmail.com> <4D42C112.2010107@skynet.be> <000201cbbef5$098865f0$1c9931d0$@no> <4D42D7C1.8040907@skynet.be> Message-ID: <000001cbbefe$498d9fe0$dca8dfa0$@no> On Behalf Of Herman De Wael ............. > No-one has yet answered that one, actually. Do you agree that the word "specific" > means that when it is clear that a player has only stated a minimum number of > tricks he shall make, no claim has been made? My understanding of the word "specific" is that it is contrary to "unspecific" or "undefined". A "specific number of tricks" imply that it must be understood as a precise number, but the actual number need not be directly mentioned. "All remaining tricks", "The remaining tricks less one", "These tricks" (when referring to something that can be counted) are all examples of an expression that satisfy the clause: "Specific number of tricks". If a player states: "I shall make at least two tricks" I shall not consider that a claim, but I shall ask him if he intends to claim and if so do it in a proper way. And if he doesn't intend to claim I shall request him to play on rather than embarrass his opponents with such extraneous remarks. If instead he says "I shall make two tricks" I shall order him to face his cards and complete his claim statement. From ehaa at starpower.net Fri Jan 28 16:19:36 2011 From: ehaa at starpower.net (Eric Landau) Date: Fri, 28 Jan 2011 10:19:36 -0500 Subject: [BLML] Exam question In-Reply-To: <4D42D7C1.8040907@skynet.be> References: <4D3D3B2F.5090107@skynet.be> <1PhIDl-0dAMts0@fwd00.aul.t-online.de> <4D3E0908.5090009@skynet.be> <000201cbbc6f$6360ef80$2a22ce80$@no> <4D3E9B17.7050704@skynet.be> <000401cbbc76$6e66ba20$4b342e60$@no> <4D3EE4D1.7010203@skynet.be> <4D3EE90C.5020002@t-online.de> <4D40994E.2030101@nhcc.net> <4D418790.1040107@ulb.ac.be> <4D428418.3080805@skynet.be> <4D4286CF.10807@ulb.ac.be> <4D429E21.8020205@skynet.be> <4D42A555.5010007@gmail.com> <4D42C112.2010107@skynet.be> <000201cbbef5$098865f0$1c9931d0$@no> <4D42D7C1.8040907@skynet.be> Message-ID: <27C5A3A2-852A-46F5-BA93-681070664AE9@starpower.net> On Jan 28, 2011, at 9:50 AM, Herman De Wael wrote: > No Sven, that is not what I mean: > > Sven Pran wrote: > >> On Behalf Of Herman De Wael ............. >> >>> Yes Lucas, valid point, but: If the intent had been to include the >>> case where only some cards are shown, that should also include one >>> card. You cannot read the case for one card into the phrase "shows >>> his cards". It would seem strange to me that the lawmakers would >>> have intended to include the case for AK, but not for A. If I had >>> cashed the King of hearts and then shown the Ace, would that have >>> changed the ruling? Certainly not, whichever rulling one favours. >> >> Do you assert that you have not claimed just the two tricks (and >> conceded the rest) if, with say 5 cards left, you show two aces and >> say words to the effect that you will take these winners, acting in a >> manner giving the impression that you do not expect to take any >> further tricks after that? > > No, I do not assert that. For one thing, the difference being that the > "manner" which you describe can be judged to "suggest that play be > curtailed". Remember that L68A has three parts, one of which is enough > to make an action a claim. The showing of the two Aces does not > make it > a claim, and my assertion that showing AK of trumps does not make it a > claim is not contradictory to the fact that what you describe IS a > claim. > >> You have not stated any specific number of tricks; you have not shown >> all your cards and you have not suggested that play should be >> curtailed. Still, in my experience that is a very frequent way for >> players to claim some, but not all the remaining tricks. > > Well, by not playing on, they have suggested that play be curtailed. > Anyway, they will not contend that they haven't! > >> My impression from your OP was that you showed your two winners and >> claimed the corresponding number of tricks. To me that is a claim >> unless you "demonstrably did not intend to claim". > > Which I indeed did not - but that is not the point. My point is that > showing two winners is not equal to conceding five losers, and that my > actions do not refer to a "specific" number of tricks. > > No-one has yet answered that one, actually. Do you agree that the word > "specific" means that when it is clear that a player has only stated a > minimum number of tricks he shall make, no claim has been made? > >> However, the manner in which you acted is not apparent from your OP; >> if your opponents did not think that you intended to claim then >> neither would I rule so. But then I should be extremely surprised if >> I had been called to the table. > > And that is of course the point. When the opponents do not see it as a > claim, there will be no director call. But what if, for the same joke, > the opponents ask the director to make a ruling? Suppose the Director > establishes the facts as being: I showed 2 cards, I stated that I > shall > make at least 8 tricks, and I did not suggest play be curtailed. > Have I > claimed? A player claims if he "suggests that play be curtailed". That is a highly subjective judgment, and the player in question does not get to decide for himself whether his actions constituted such a suggestion. The director is responsible for that judgment, but the director wasn't at the table to see what actually occurred. But the opponents were. And if they were under the impression that the player's actions consituted a claim, something or someone perforce "suggested" it to them, and it wasn't a little birdie flying by. Eric Landau 1107 Dale Drive Silver Spring MD 20910 ehaa at starpower.net From Hermandw at skynet.be Fri Jan 28 17:10:01 2011 From: Hermandw at skynet.be (Herman De Wael) Date: Fri, 28 Jan 2011 17:10:01 +0100 Subject: [BLML] Exam question In-Reply-To: <000001cbbefe$498d9fe0$dca8dfa0$@no> References: <4D3D3B2F.5090107@skynet.be> <1PhIDl-0dAMts0@fwd00.aul.t-online.de> <4D3E0908.5090009@skynet.be> <000201cbbc6f$6360ef80$2a22ce80$@no> <4D3E9B17.7050704@skynet.be> <000401cbbc76$6e66ba20$4b342e60$@no> <4D3EE4D1.7010203@skynet.be> <4D3EE90C.5020002@t-online.de> <4D40994E.2030101@nhcc.net> <4D418790.1040107@ulb.ac.be> <4D428418.3080805@skynet.be> <4D4286CF.10807@ulb.ac.be> <4D429E21.8020205@skynet.be> <4D42A555.5010007@gmail.com> <4D42C112.2010107@skynet.be> <000201cbbef5$098865f0$1c9931d0$@no> <4D42D7C1.8040907@skynet.be> <000001cbbefe$498d9fe0$dca8dfa0$@no> Message-ID: <4D42EA59.6070700@skynet.be> Sorry Sven, very good, but not good enough: Sven Pran wrote: > On Behalf Of Herman De Wael > ............. >> No-one has yet answered that one, actually. Do you agree that the word > "specific" >> means that when it is clear that a player has only stated a minimum number > of >> tricks he shall make, no claim has been made? > > My understanding of the word "specific" is that it is contrary to > "unspecific" or "undefined". > A "specific number of tricks" imply that it must be understood as a precise > number, but the actual number need not be directly mentioned. > > "All remaining tricks", "The remaining tricks less one", "These tricks" > (when referring to something that can be counted) are all examples of an > expression that satisfy the clause: "Specific number of tricks". > quite correct. > If a player states: "I shall make at least two tricks" I shall not consider > that a claim, but I shall ask him if he intends to claim and if so do it in > a proper way. And if he doesn't intend to claim I shall request him to play > on rather than embarrass his opponents with such extraneous remarks. If > instead he says "I shall make two tricks" I shall order him to face his > cards and complete his claim statement. > Good, but what if he says "I think I won't go down". Does that mean "at least eight" or "eight and not nine"? It does not help us any when you tell us what your ruling will be in two cases, when the question was really which of the two cases it is. But of course, that is a question for the director at the table, and depends on the intonation. My point being that intonation can make "I'll make 8 tricks" mean "I'll make at least 8", and you should not give simple rules as the one above. But basically, we agree. -- Herman De Wael Wilrijk Antwerpen Belgium From agot at ulb.ac.be Fri Jan 28 17:12:24 2011 From: agot at ulb.ac.be (Alain Gottcheiner) Date: Fri, 28 Jan 2011 17:12:24 +0100 Subject: [BLML] Exam question In-Reply-To: <4D42C112.2010107@skynet.be> References: <4D3D3B2F.5090107@skynet.be> <1PhIDl-0dAMts0@fwd00.aul.t-online.de> <4D3E0908.5090009@skynet.be> <000201cbbc6f$6360ef80$2a22ce80$@no> <4D3E9B17.7050704@skynet.be> <000401cbbc76$6e66ba20$4b342e60$@no> <4D3EE4D1.7010203@skynet.be> <4D3EE90C.5020002@t-online.de> <4D40994E.2030101@nhcc.net> <4D418790.1040107@ulb.ac.be> <4D428418.3080805@skynet.be> <4D4286CF.10807@ulb.ac.be> <4D429E21.8020205@skynet.be> <4D42A555.5010007@gmail.com> <4D42C112.2010107@skynet.be> Message-ID: <4D42EAE8.2050901@ulb.ac.be> Le 28/01/2011 14:13, Herman De Wael a ?crit : > ?ukasz Kalbarczyk wrote: >> W dniu 2011-01-28 11:44, Herman De Wael pisze: >>> But Alain, then you are not ruling according to L68A. >>> L68A clearly states "shows his cards". That must mean all of them, >>> because otherwise the lawmakers would have written "shows any card". >>> The fact that you rule "claim" in another case where a player shows 2 >>> cards (and for good reason), should not imply that you should ignore >>> this law here! >> Or /any card/, otherwise the lawmakers would have written "shows all cards". >> >> ?K >> > Yes Lucas, valid point, but: > If the intent had been to include the case where only some cards are > shown, that should also include one card. You cannot read the case for > one card into the phrase "shows his cards". It would seem strange to me > that the lawmakers would have intended to include the case for AK, but > not for A. AG : why would it be strange ? To show one card might be an intent to play to this trick (perhaps prematurely) ; intentionally showing two or more can't. > If I had cashed the King of hearts and then shown the Ace, > would that have changed the ruling? Certainly not, whichever rulling one > favours. AG : sorry, but in my view it would, precisely because this is the normal way to play. From grandaeval at tiscali.co.uk Fri Jan 28 18:32:21 2011 From: grandaeval at tiscali.co.uk (Grattan) Date: Fri, 28 Jan 2011 17:32:21 -0000 Subject: [BLML] 25A 2) References: <4D42828D.3090401@meteo.fr> <1Pik2h-03mYiW0@fwd11.aul.t-online.de> Message-ID: Grattan Endicott To: "Bridge Laws Mailing List" Sent: Friday, January 28, 2011 8:50 AM Subject: Re: [BLML] 25A 2) > From: Jean-Pierre Rocafort >> Peter Eidt a ??crit : >> > From: Jean-Pierre Rocafort >> >> Thomas Dehn a ??crit : >> >>> Grattan wrote: >> >>>> ++++++++++++++++++++++++++++++++++ >> >>>> ----- Original Message ----- >> >>>> From: "Thomas Dehn" >> >>>> To: >> >>>> Sent: Thursday, January 27, 2011 6:59 PM >> >>>> Subject: Re: [BLML] 25A 2) >> >>>> ....... >> >>>>> Intent does not matter, as it is simply >> >>>>> too late for S to change his call; his >> >>>>> partner had already called. >> >>>>> Thus this is an insufficient bid out of turn. >> >>>>> >> >>>> +=+ That is rather a bold statement. It may be true, but >> >>>> first we need to examine the bidding box regulations >> >>>> to see if these have anything relevant to say. >>>>>>>> >> >>> In principle correct, but I have never >> >>> heard of any regulations where >> >>> "He puts the 1S card on top of his >> >>> bidding cards on the table." is not >> >>> a made 1S bid in classic face to face bridge >> >>> without screens. >> >> was the 1S card put just over the 1H card so that it was no more >> >> visible? or was it put aside, both visible? >> > >> > [OP from Ton] >> > [...] and puts the 1S card on top of his bidding cards on the >> > table. >> >> so there is no doubt the intent was a change of call? > > [Peter] > not for me ... reading the OP and the subject > +=+ Hmmm.... I would be interested to read anything the regulations say about exposure of an extraneous bidding card during the auction. It may be true that the 1S card was placed to signify a wish to change the call from 1H to 1S, but if we have progressed in the auction to a point when a change of call is not permitted then the exposure of the 1S card, for whatever reason, is external to the auction. There may be connotations of UI, I agree, but the disposition of the card and attendant rectification measures will stem from Bidding Box regulations (if any) about exposure during the auction of bidding box cards which are not part of the auction, be it accidentally or purposefully. I doubt that the Director can rule this an insufficient bid out of turn unless the card was placed intentionally as a bid in its own right in the auction at the point reached. If the regulations provide no answer we are in Law 16B territory and the Director decides whether the irregularity has made information available to partner that may suggest a call or play. (Inter alia it could, for example, suggest Pass with a hand that is good for a raise of partner's bid suit.) I am not seeing a difficulty. Is there a difficulty? ~ Grattan ~ +=+ From JffEstrsn at aol.com Fri Jan 28 19:42:34 2011 From: JffEstrsn at aol.com (Jeff Easterson) Date: Fri, 28 Jan 2011 19:42:34 +0100 Subject: [BLML] Exam question In-Reply-To: <4D42C112.2010107@skynet.be> References: <4D3D3B2F.5090107@skynet.be> <1PhIDl-0dAMts0@fwd00.aul.t-online.de> <4D3E0908.5090009@skynet.be> <000201cbbc6f$6360ef80$2a22ce80$@no> <4D3E9B17.7050704@skynet.be> <000401cbbc76$6e66ba20$4b342e60$@no> <4D3EE4D1.7010203@skynet.be> <4D3EE90C.5020002@t-online.de> <4D40994E.2030101@nhcc.net> <4D418790.1040107@ulb.ac.be> <4D428418.3080805@skynet.be> <4D4286CF.10807@ulb.ac.be> <4D429E21.8020205@skynet.be> <4D42A555.5010007@gmail.com> <4D42C112.2010107@skynet.be> Message-ID: <4D430E1A.5070301@aol.com> I'm not certain it is relevant to the case or of any importance (because I have stopped following the discussion closely- see closing note) but Herman asks and I am an "English speaker". (I think he means "native English speaker". All contributors to blml are, more or less, English speakers.) Without giving it great thought (and thus not precluding the possibility that differing arguments might convince me that I am wrong) my gut feeling/first reaction is that "shows his cards" does mean all of them. As mentioned above, I have stopped following this thread closely. First because I dislike endless threads and particularly those with very long postings. So I generally read only the shorter ones. Thus my view is incomplete, to say the least. Secondly because repetition bores me and endless repetition bores me endlessly. Thirdly because I dislike threads in whcih something like 30% of the postings are from one person. So I don't read his unless they are very short. But from what I have read and the latest postings by Herman I have the feeling of shifting sand, perhaps even fighting with windmills. The situation described originally is being modified, more exactly explained. Why wasn't this done originally? The first response was to the first exposition of the "case". When the case is described in more detail there can well be a change in the opinion of those responding. So, once again I ask: why was the case not originally described in all detail and the way it has seemed to have been modified in the last emails? Ciao, JE Am 28.01.2011 14:13, schrieb Herman De Wael: > ?ukasz Kalbarczyk wrote: >> W dniu 2011-01-28 11:44, Herman De Wael pisze: >>> But Alain, then you are not ruling according to L68A. >>> L68A clearly states "shows his cards". That must mean all of them, >>> because otherwise the lawmakers would have written "shows any card". >>> The fact that you rule "claim" in another case where a player shows 2 >>> cards (and for good reason), should not imply that you should ignore >>> this law here! >> Or /any card/, otherwise the lawmakers would have written "shows all cards". >> >> ?K >> > Yes Lucas, valid point, but: > If the intent had been to include the case where only some cards are > shown, that should also include one card. You cannot read the case for > one card into the phrase "shows his cards". It would seem strange to me > that the lawmakers would have intended to include the case for AK, but > not for A. If I had cashed the King of hearts and then shown the Ace, > would that have changed the ruling? Certainly not, whichever rulling one > favours. > > I would like to hear input on this from English speakers: > "shows his cards" means all of them? > From swillner at nhcc.net Sat Jan 29 03:53:08 2011 From: swillner at nhcc.net (Steve Willner) Date: Fri, 28 Jan 2011 21:53:08 -0500 Subject: [BLML] Exam question In-Reply-To: <4D428633.8080108@ulb.ac.be> References: <4D3D3B2F.5090107@skynet.be> <1PhIDl-0dAMts0@fwd00.aul.t-online.de> <4D3E0908.5090009@skynet.be> <000201cbbc6f$6360ef80$2a22ce80$@no> <4D3E9B17.7050704@skynet.be> <000401cbbc76$6e66ba20$4b342e60$@no> <4D3EE4D1.7010203@skynet.be> <4D3EE90C.5020002@t-online.de> <4D40994E.2030101@nhcc.net> <4D4283A7.3090404@skynet.be> <4D428633.8080108@ulb.ac.be> Message-ID: <4D438114.9000200@nhcc.net> On 1/28/2011 4:02 AM, Alain Gottcheiner wrote: > Also notice that the interpretation of "showing one's cards" as "showing > all of them" is personal to Herman. The sentence "it echoes my thoughts" > doesn't mean "all of them". I disagree with the first sentence. While "showing one's cards" doesn't necessarily mean all of them, I think it does mean all the ones that are _important_ for the present purpose in context. This usage may be specific to "showing." For example, "showing one's intentions" or "showing one's interests" has, I believe, the same implication: not all of them on any subject whatsoever, but all of them relevant to the current situation. As Alain wrote, it doesn't generalize to other cases, though it is close for "echoes." That doesn't mean all one's thoughts, but I think it does mean all the ones important for whatever subject is under discussion. English is not a logical language, I'm afraid. > notice that the wording in TFLB ("curtailed") doesn't make it > necessary that one suggest that play is over ; it only makes it > necessary that declarer suggested accelerating play. 'Curtailed' implies incompleteness. It's much closer to "ended" than to "shortened" or "accelerated." In the bridge context, it means no further need for continued card-by-card play. From swillner at nhcc.net Sat Jan 29 03:59:21 2011 From: swillner at nhcc.net (Steve Willner) Date: Fri, 28 Jan 2011 21:59:21 -0500 Subject: [BLML] 25A 2) In-Reply-To: References: <4D42828D.3090401@meteo.fr> <1Pik2h-03mYiW0@fwd11.aul.t-online.de> Message-ID: <4D438289.6040101@nhcc.net> On 1/28/2011 12:32 PM, Grattan wrote: > +=+ Hmmm.... I would be interested to read anything the > regulations say about exposure of an extraneous bidding > card during the auction. ACBL says something to the effect of "placed on the table," closely analogous to play of declarer's card. EBU and others say, I believe, "removed from the box with apparent intent," but it isn't clear whether "intent" means "intent to remove from the box" or "intent to make that call at this moment." > I doubt that the Director can rule this an insufficient bid > out of turn unless the card was placed intentionally as a bid > in its own right in the auction at the point reached. There's nothing about intent in the ACBL regulation. The EBU-type regulation is unclear to me. Like others, I have no doubt that the bidder intended to change his 1H bid to 1S and did not intend to make a new call at this point in the auction (RHO's turn). The question is whether his intent matters or not. From Hermandw at skynet.be Sat Jan 29 08:36:42 2011 From: Hermandw at skynet.be (Herman De Wael) Date: Sat, 29 Jan 2011 08:36:42 +0100 Subject: [BLML] Exam question In-Reply-To: <4D430E1A.5070301@aol.com> References: <4D3D3B2F.5090107@skynet.be> <1PhIDl-0dAMts0@fwd00.aul.t-online.de> <4D3E0908.5090009@skynet.be> <000201cbbc6f$6360ef80$2a22ce80$@no> <4D3E9B17.7050704@skynet.be> <000401cbbc76$6e66ba20$4b342e60$@no> <4D3EE4D1.7010203@skynet.be> <4D3EE90C.5020002@t-online.de> <4D40994E.2030101@nhcc.net> <4D418790.1040107@ulb.ac.be> <4D428418.3080805@skynet.be> <4D4286CF.10807@ulb.ac.be> <4D429E21.8020205@skynet.be> <4D42A555.5010007@gmail.com> <4D42C112.2010107@skynet.be> <4D430E1A.5070301@aol.com> Message-ID: <4D43C38A.2070805@skynet.be> I'll make this short for Jeff. He asked me why the original did not give all the details. Here's the (important line from) the original: I showed AK of hearts, saying "I think I am going to make this contract" If that does not, in one sentence, tell you exactly what happened, then I don't know how to make it more clearer. It is precisely (translated) what I said. The intention seems clear to me. But, as it turned out, that intention was not clear to others. I apologize. Jeff Easterson wrote: > > But from what I have read and the latest postings by Herman I have the > feeling of shifting sand, perhaps even fighting with windmills. The > situation described originally is being modified, more exactly > explained. Why wasn't this done originally? The first response was to > the first exposition of the "case". When the case is described in more > detail there can well be a change in the opinion of those responding. > So, once again I ask: why was the case not originally described in all > detail and the way it has seemed to have been modified in the last emails? > -- Herman De Wael Wilrijk Antwerpen Belgium From JffEstrsn at aol.com Sat Jan 29 15:30:05 2011 From: JffEstrsn at aol.com (Jeff Easterson) Date: Sat, 29 Jan 2011 15:30:05 +0100 Subject: [BLML] judgement Message-ID: <4D44246D.2010506@aol.com> You are playing with a new partner and have few agreements. One is that non vul an opening weak two can be 5-10 points and always(at least) a 6 card suit. He opens 2 hearts. You hold: AK954 4 AKQ87 54 What is your bid? Part 2: If you have agreed to play Ogust and he shows a minimum opening with a bad trump suit does this alter your decision? If so, to what? Thanks, JE From lavaldubreuil at xplornet.com Sat Jan 29 16:49:12 2011 From: lavaldubreuil at xplornet.com (laval dubreuil) Date: Sat, 29 Jan 2011 10:49:12 -0500 Subject: [BLML] judgement In-Reply-To: <4D44246D.2010506@aol.com> References: <4D44246D.2010506@aol.com> Message-ID: <000001cbbfcc$13d7e260$3b87a720$@com> Having agree or not on Ogust, my bid is 2S (I hope my new pd knows it is normally forcing)... Laval Du Breuil Quebec ____________________________________________________________________________ You are playing with a new partner and have few agreements. One is that non vul an opening weak two can be 5-10 points and always(at least) a 6 card suit. He opens 2 hearts. You hold: AK954 4 AKQ87 54 What is your bid? From svenpran at online.no Sat Jan 29 18:14:39 2011 From: svenpran at online.no (Sven Pran) Date: Sat, 29 Jan 2011 18:14:39 +0100 Subject: [BLML] Exam question In-Reply-To: <4D43C38A.2070805@skynet.be> References: <4D3D3B2F.5090107@skynet.be> <1PhIDl-0dAMts0@fwd00.aul.t-online.de> <4D3E0908.5090009@skynet.be> <000201cbbc6f$6360ef80$2a22ce80$@no> <4D3E9B17.7050704@skynet.be> <000401cbbc76$6e66ba20$4b342e60$@no> <4D3EE4D1.7010203@skynet.be> <4D3EE90C.5020002@t-online.de> <4D40994E.2030101@nhcc.net> <4D418790.1040107@ulb.ac.be> <4D428418.3080805@skynet.be> <4D4286CF.10807@ulb.ac.be> <4D429E21.8020205@skynet.be> <4D42A555.5010007@gmail.com> <4D42C112.2010107@skynet.be> <4D430E1A.5070301@aol.com> <4D43C38A.2070805@skynet.be> Message-ID: <000101cbbfd8$035cddd0$0a169970$@no> On Behalf Of Herman De Wael > I'll make this short for Jeff. > > He asked me why the original did not give all the details. > > Here's the (important line from) the original: > > I showed AK of hearts, saying "I think I am going to make this contract" > > If that does not, in one sentence, tell you exactly what happened, then I don't know > how to make it more clearer. And to me that was a very clear statement to the effect that you would make the two (more) necessary tricks for your contract, namely the trump AK; in other words you claimed these specific two tricks, neither of which was the current trick in the play. From jfusselman at gmail.com Sat Jan 29 19:04:08 2011 From: jfusselman at gmail.com (Jerry Fusselman) Date: Sat, 29 Jan 2011 12:04:08 -0600 Subject: [BLML] In the ACBL, is a takeout double conventional? Message-ID: The short answer is yes, it is. According to Mike Flader, "By definition, a takeout double is a convention, so, conventional defenses to them would be allowed, unless disallowed by items such as #1 under disallowed on the GCC." By the way, Wikipedia says "It could be argued that takeout doubles are conventional but their usage is so widespread that it is considered a natural bid." Love the use of the passive voice here to give it an air of objectivity, but who considers it natural? Wikipedia has a breathtakingly high rate of errors, so perhaps I should not have bothered looking it up there. I am not sure it is safe to believe that ACBL directors always allow conventional defenses to takeout doubles. Here's why: It is because no one responded to the question I asked below that I contacted the ACBL tournament department. I explained that I was asking to help clarify for some issues that had come up on BLML. At first, they said that a takeout double is *not* a convention, so a conventional defense to a takeout double is not allowed (under the GCC). I replied, as gently as I could (No doubt other BLMLers could have done it better, for I am afraid that they were offended by my nonimmediate acceptance of their statement), that I thought a penalty double is natural so the opposite meaning, a takeout double, should be conventional. He told me that I would be surprised how vague some players' takeout doubles are as to distribution, being merely a show of points with a request to bid. But he changed his mind slightly, saying he was not totally 100% sure, that he would ask to find out, and get back to me. Exactly one month later, I got Mike Flader's helpful email. Especially this time of year, one cannot expect quick answers from the ACBL rulings department, because are very busy people. Too bad the information is nowhere published and known to so few people. If I or my opponents played a conventional defense to a takeout double, how many ACBL directors would rule it illegal, even though Mike Flader has confirmed to me privately by email that a takeout double is a convention? How can I know ahead of time what will work, when it is not published anywhere? This is reason #101 that I prefer a rule of laws to a rule of men. Jerry Fusselman On Sat, Dec 18, 2010 at 3:27 PM, I wrote: > On Fri, Dec 17, 2010 at 9:05 AM, Eric Landau wrote: >> On Dec 16, 2010, at 9:44 PM, Jerry Fusselman wrote: >> >>> >>> I would guess that most directors would allow 1S after 1H-X- to >>> possibly not show spades, if that was disclosed with adequate >>> precision. ?Do you think I am wrong? >> >> I do. ?I think if you offered it as a two-way conventional method, 1S >> forcing, either natural with spades or weak with heart support and >> spade shortness, you would not be allowed to play it. ?Perhaps "some" >> directors would allow it, but "most" is almost certainlty a serious >> overbid. >> > > Are there any ACBL directors reading this who would tell us how they > would rule (I.e., is the agreement legal under the GCC?) after 1H - (X > takeout) - 1S if 1S shows: > > A. ?Either 4 spades with 6+ points, or a weaker hand with 3 hearts and > spade shortness. > B. ?Either 4 spades with 6+ points, or a 3-card limit raise of hearts. > C. ?Transfer to 1NT a la Lebensohl with one of several hands, but > saying nothing about spades. > D. ?Same as forcing NT (i.e., over a conventional double, 1NT and 1S > are switched so that we can stop in 1NT). > > Personally, I like D. ?It does not deny four spades---I might have > four spades if I have three hearts and a hand worth less than 6 points > in support. > > Each of these are two-way calls, which would you allow, and which would you not? > > Perhaps there are conditions under which you would allow them. ?For > example, would you allow example B if 1H - (X takeout) - 3H was > preemptive and our system had no other way to show a 3-card limit > raise? > From nigelguthrie at yahoo.co.uk Sat Jan 29 20:22:06 2011 From: nigelguthrie at yahoo.co.uk (Nigel Guthrie) Date: Sat, 29 Jan 2011 19:22:06 +0000 (GMT) Subject: [BLML] judgement In-Reply-To: <4D44246D.2010506@aol.com> References: <4D44246D.2010506@aol.com> Message-ID: <749317.39177.qm@web28505.mail.ukl.yahoo.com> [Jeff Easterson] You are playing with a new partner and have few agreements. One is that non vul an opening weak two can be 5-10 points and always(at least) a 6 card suit. He opens 2 hearts. You hold: AK954 4 AKQ87 54 What is your bid? Part 2: If you have agreed to play Ogust and he shows a minimum opening with a bad trump suit does this alter your decision? If so, to what? [Nige1] IMO Part 1: 3D = 10, 2S = 9, P = 8, 2N = 6. After 2H-3D if partner bids 3H, then you can continue econmomically with 3S. Part 2: Over 3C (minimum opening poor trumps), 3H = 10, 3D = 5, 3S = 4. Even 3H seems high. From jfusselman at gmail.com Sat Jan 29 21:39:13 2011 From: jfusselman at gmail.com (Jerry Fusselman) Date: Sat, 29 Jan 2011 14:39:13 -0600 Subject: [BLML] What the GCC allows when competing over an artificial 1C opening Message-ID: The issue here is understanding the ACBL's GCC (general convention chart), which governs which conventions are allowed in most events that are not limited to clubs. (I ignore clubs because they can ignore the GCC: "Clubs have full authority to regulate conventions in games conducted solely at their clubs.") I asked Alain's question from December, and five others that I added, and I give official ACBL responses below. The relevant parts of the GCC are this: By Competitive 7a, a conventional defense to a conventional 1C opening is allowed, unless its primary purpose is to destroy the opponents? methods. This last phrase comes from Disallowed 1. I really want to find out what the following means: "primary purpose is to destroy the opponents? methods." This is an interesting rule. (In some ways, it is like the tenth amendment to the US Constitution.) On Mon, Dec 20, 2010 at 3:13 AM, Alain Gottcheiner wrote: > Le 18/12/2010 20:57, Jerry Fusselman a ?crit : >> >> On Fri, Dec 17, 2010 at 11:32 AM, Eric Landau wrote: >>> >>> Yes, but what does "not designed purely to destroy opponents' >>> methods" mean? ?("Designed"? ?"Purely"?) ?AFAICT there's no extant >>> operational definition, which means that its application in practice >>> is entirely arbitrary. >>> >> About 15 years ago, I called the ACBL office to ask what it means. >> The answer they gave me was immediate: ?The call must say something >> about your hand. ?For example, (1C artificial and forcing) - 1S is >> disallowed if it says nothing about your hand. ?But if 1S shows a 4441 >> hand pattern with the singleton anywhere (I think that was one of the >> examples he gave me), then it is allowed. >> > AG : so if if means "less than opening values and no 7-card suit " it is > allowed ? > In December, on the phone with Butch Campbell, head of the Tournament department, he told me something like this, which I wrote down: "to be allowed, the call must show something of value about the hand, such as suits or high-card strength." If anyone has any refinement on this, I would be totally delighted to hear it. Anyway, Mr. Campbell kindly relayed my questions for BLML to the rulings department. And last weekend, Mike Flader gave the answers to this specific question: Which of these unusual (i.e., a--e say nothing about spades, and f denies five spades) meanings of 1S in direct-seat competition over 1C artificial are allowed? a. (Alain's question) Less than opening values and no 7-card suit; b. Less than 3 points and no 8-card suit; c. Less than opening values; d. Less than 16 points; e. More than 16 points; f. 3 or 4 spades and a balanced hand (at most one doubleton). Can anyone guess what the answers to b--f are using the following official additional hint?: Mike Flader wrote that example a and at least one of the others is not allowed, "because its primary purpose is to disrupt the communication of the opponents. [W]hile they tend to deny something, what they deny is relatively rare and they still say nothing about the hand." The other hint is that at least one of these six is GCC-legal. Now, from that, can you determine which of b--f are allowed under GCC? Which are not allowed? Is it predictable, or not? I found his answer quite surprising, and I want to see if others can guess the pattern from the information I gave. If someone can, it will help me. I really wish to see the pattern so that I know when to call the director and also which methods to invest time in inventing and learning. Is it unethical of me to want to play the game by its rules? Jerry Fusselman From jfusselman at gmail.com Sat Jan 29 22:07:25 2011 From: jfusselman at gmail.com (Jerry Fusselman) Date: Sat, 29 Jan 2011 15:07:25 -0600 Subject: [BLML] Exam question In-Reply-To: <74D61212-85D5-42B3-9583-FFE26C078352@starpower.net> References: <4D3E9ACD.5020602@skynet.be> <1Phfff-2FiDdA0@fwd06.aul.t-online.de> <4D3EE3E8.70308@skynet.be> <74D61212-85D5-42B3-9583-FFE26C078352@starpower.net> Message-ID: On Tue, Jan 25, 2011 at 11:52 AM, Eric Landau wrote: > > There is a TV show in the US called "The McLaughlin Group", a weekly > panel discussion of American politics. ?It is hosted by John > McLaughlin, who appears with four other panelists. ?It is famous for > the following interaction, which occurs with considerable frequency: > Mr. McLaughlin poses a question to the panel; each of the four > panelists, in turn, gives essentially the same answer. ?Mr. > McLaughlin then announces (his signiture phrase), "The answer is...," > and proceeds to expound an opinion diametrically opposite to the > unanimous opinion of his panelists. > > Of course, when Mr. McLaughlin does this, he understands that he is > making a joke. > Interesting post. This is what I want to know: Is it acceptable or unacceptable, in your view, for a BLMLer to say that someone else's view is wrong, or to say that the answer is X when others on BLML have given a different answer? Is it acceptable style, or is it unacceptable bad style? From nigelguthrie at yahoo.co.uk Sun Jan 30 02:18:44 2011 From: nigelguthrie at yahoo.co.uk (Nigel Guthrie) Date: Sun, 30 Jan 2011 01:18:44 +0000 (GMT) Subject: [BLML] Exam question In-Reply-To: References: <4D3E9ACD.5020602@skynet.be> <1Phfff-2FiDdA0@fwd06.aul.t-online.de> <4D3EE3E8.70308@skynet.be> <74D61212-85D5-42B3-9583-FFE26C078352@starpower.net> Message-ID: <451079.57623.qm@web28506.mail.ukl.yahoo.com> [Jerry Fusselman] Interesting post. This is what I want to know: Is it acceptable or unacceptable, in your view, for a BLMLer to say that someone else's view is wrong, or to say that the answer is X when others on BLML have given a different answer? Is it acceptable style, or is it unacceptable bad style? [Nigel] It is acceptable to advance the opinion that a majority are mistaken but insolent to accuse them of lying. Given the human capacity to rationalise, it is unlikely that they are lying anyway. Often you feel that you are the only man in step. Rarely you may even be right. Especially when you are questioning the practices of a semi-professional body from whom it could be a bad career-move to break ranks. Career-prospects for young whistle-blowers are dismal. For example Nurses and Care-workers who report their colleagues' abuse of children or old-people. It's less dangerous if you are about to retire. As far as I remember, Consultant Margaret Cooke retired before hinting that NHS surgeons with private practices might have a vested interest in maintaining a long waiting-list for time-critical life-saving NHS operations. From bpark56 at comcast.net Sun Jan 30 02:26:19 2011 From: bpark56 at comcast.net (Robert Park) Date: Sat, 29 Jan 2011 20:26:19 -0500 Subject: [BLML] In the ACBL, is a takeout double conventional? In-Reply-To: References: Message-ID: <4D44BE3B.9070404@comcast.net> On 1/29/11 1:04 PM, Jerry Fusselman wrote: > The short answer is yes, it is. According to Mike Flader, "By > definition, a takeout double is a convention, so, conventional > defenses to them would be allowed, unless disallowed by items such as > #1 under disallowed on the GCC." > > By the way, Wikipedia says "It could be argued that takeout doubles > are conventional but their usage is so widespread that it is > considered a natural bid." Love the use of the passive voice here to > give it an air of objectivity, but who considers it natural? > Wikipedia has a breathtakingly high rate of errors, so perhaps I > should not have bothered looking it up there. > > I am not sure it is safe to believe that ACBL directors always allow > conventional defenses to takeout doubles. Here's why: > > It is because no one responded to the question I asked below that I > contacted the ACBL tournament department. I explained that I was > asking to help clarify for some issues that had come up on BLML. At > first, they said that a takeout double is *not* a convention, so a > conventional defense to a takeout double is not allowed (under the > GCC). I replied, as gently as I could (No doubt other BLMLers could > have done it better, for I am afraid that they were offended by my > nonimmediate acceptance of their statement), that I thought a penalty > double is natural so the opposite meaning, a takeout double, should be > conventional. He told me that I would be surprised how vague some > players' takeout doubles are as to distribution, being merely a show > of points with a request to bid. But he changed his mind slightly, > saying he was not totally 100% sure, that he would ask to find out, > and get back to me. Exactly one month later, I got Mike Flader's > helpful email. Especially this time of year, one cannot expect quick > answers from the ACBL rulings department, because are very busy > people. Too bad the information is nowhere published and known to so > few people. > > If I or my opponents played a conventional defense to a takeout > double, how many ACBL directors would rule it illegal, even though > Mike Flader has confirmed to me privately by email that a takeout > double is a convention? How can I know ahead of time what will work, > when it is not published anywhere? This is reason #101 that I prefer > a rule of laws to a rule of men. > > Jerry Fusselman > > > My partners & I play Capp/1MX (Cappelletti over 1 of a major doubled), and we have never had a squawk, either from players or directors. (2-level transfers below 2M, with 1NT = relay to 2C) --Bob Park From ardelm at optusnet.com.au Sun Jan 30 05:17:49 2011 From: ardelm at optusnet.com.au (Tony Musgrove) Date: Sun, 30 Jan 2011 15:17:49 +1100 Subject: [BLML] Exam question In-Reply-To: References: <4D3E9ACD.5020602@skynet.be> <1Phfff-2FiDdA0@fwd06.aul.t-online.de> <4D3EE3E8.70308@skynet.be> <74D61212-85D5-42B3-9583-FFE26C078352@starpower.net> Message-ID: <201101300418.p0U4IHeU012808@mail09.syd.optusnet.com.au> At 08:07 AM 30/01/2011, you wrote: >On Tue, Jan 25, 2011 at 11:52 AM, Eric Landau wrote: > > > > There is a TV show in the US called "The McLaughlin Group", a weekly > > panel discussion of American politics. It is hosted by John > > McLaughlin, who appears with four other panelists. It is famous for > > the following interaction, which occurs with considerable frequency: > > Mr. McLaughlin poses a question to the panel; each of the four > > panelists, in turn, gives essentially the same answer. Mr. > > McLaughlin then announces (his signiture phrase), "The answer is...," > > and proceeds to expound an opinion diametrically opposite to the > > unanimous opinion of his panelists. > > > > Of course, when Mr. McLaughlin does this, he understands that he is > > making a joke. > > > >Interesting post. This is what I want to know: Is it acceptable or >unacceptable, in your view, for a BLMLer to say that someone else's >view is wrong, or to say that the answer is X when others on BLML have >given a different answer? Is it acceptable style, or is it >unacceptable bad style? >_______________________________________________ 20 years ago when I first starting taking BLML, it took some time for me to realise who were the gurus, from whose opinions one could learn. I used to try to understand the laws from the book, and from experience and eventually from the Australian publication "Australian Bridge Director's Bulletin". However I can remember giving rulings like "there is a big fight on BLML about this question, so I am not quite sure what the correct ruling is. I will rule thusly and ask the question on BLML". One then had to hope to obtain an answer from one of the gurus.and filter out the garbage. I feel that newbie directors who join this list hoping to learn how the laws are to be applied are lucky to have Grattan's posts "you are wrong" (not in quite those words of course). We used to have David Stevenson, John Probst and quite a few others. Cheers, Tony (Sydney) >Blml mailing list >Blml at rtflb.org >http://lists.rtflb.org/mailman/listinfo/blml From PeterEidt at t-online.de Sun Jan 30 08:44:12 2011 From: PeterEidt at t-online.de (Peter Eidt) Date: Sun, 30 Jan 2011 08:44:12 +0100 Subject: [BLML] =?utf-8?q?Exam_question?= In-Reply-To: References: Message-ID: <1PjRxE-16nFbM0@fwd04.aul.t-online.de> From: Jerry Fusselman > On Tue, Jan 25, 2011 at 11:52 AM, Eric Landau wrote: > > > > There is a TV show in the US called "The McLaughlin Group", a weekly > > panel discussion of American politics. ?It is hosted by John > > McLaughlin, who appears with four other panelists. ?It is famous for > > the following interaction, which occurs with considerable frequency: > > Mr. McLaughlin poses a question to the panel; each of the four > > panelists, in turn, gives essentially the same answer. ?Mr. > > McLaughlin then announces (his signiture phrase), "The answer > > is...," and proceeds to expound an opinion diametrically opposite to > > the unanimous opinion of his panelists. > > > > > > Of course, when Mr. McLaughlin does this, he understands that he is > > making a joke. > > > > > > Interesting post. This is what I want to know: Is it acceptable or > unacceptable, in your view, for a BLMLer to say that someone else's > view is wrong, or to say that the answer is X when others on BLML have > given a different answer? Is it acceptable style, or is it > unacceptable bad style? It depends of course, but in principle it is ok. Say the question is: "2 plus 2 makes ?". Whatever BLMLers give as an answer it is acceptable / necessary / mandatory to tell them they're wrong, if the answer they give is not "4". From Hermandw at skynet.be Sun Jan 30 09:38:39 2011 From: Hermandw at skynet.be (Herman De Wael) Date: Sun, 30 Jan 2011 09:38:39 +0100 Subject: [BLML] Exam question In-Reply-To: <000101cbbfd8$035cddd0$0a169970$@no> References: <4D3D3B2F.5090107@skynet.be> <1PhIDl-0dAMts0@fwd00.aul.t-online.de> <4D3E0908.5090009@skynet.be> <000201cbbc6f$6360ef80$2a22ce80$@no> <4D3E9B17.7050704@skynet.be> <000401cbbc76$6e66ba20$4b342e60$@no> <4D3EE4D1.7010203@skynet.be> <4D3EE90C.5020002@t-online.de> <4D40994E.2030101@nhcc.net> <4D418790.1040107@ulb.ac.be> <4D428418.3080805@skynet.be> <4D4286CF.10807@ulb.ac.be> <4D429E21.8020205@skynet.be> <4D42A555.5010007@gmail.com> <4D42C112.2010107@skynet.be> <4D430E1A.5070301@aol.com> <4D43C38A.2070805@skynet.be> <000101cbbfd8$035cddd0$0a169970$@no> Message-ID: <4D45238F.30100@skynet.be> Sven Pran wrote: > On Behalf Of Herman De Wael >> I'll make this short for Jeff. >> >> He asked me why the original did not give all the details. >> >> Here's the (important line from) the original: >> >> I showed AK of hearts, saying "I think I am going to make this contract" >> >> If that does not, in one sentence, tell you exactly what happened, then I > don't know >> how to make it more clearer. > > And to me that was a very clear statement to the effect that you would make > the two (more) necessary tricks for your contract, namely the trump AK; in > other words you claimed these specific two tricks, neither of which was the > current trick in the play. > You are using the word specific in a sentence that is not in the laws. I did claim those specific two tricks, but I did not mention a _specific number_ of tricks! L68A is very clear on that subject, and I would like to see confirmed that mentioning "at least eight tricks" is _not_ sufficient condition for the definition of it being a claim. We can leave the discussion as to whether I specified "eight" or "at least eight" to the TD at the table (who was never there but that does not matter). -- Herman De Wael Wilrijk Antwerpen Belgium From gordonrainsford at btinternet.com Sun Jan 30 10:00:14 2011 From: gordonrainsford at btinternet.com (Gordon Rainsford) Date: Sun, 30 Jan 2011 09:00:14 +0000 Subject: [BLML] Exam question In-Reply-To: <201101300418.p0U4IHeU012808@mail09.syd.optusnet.com.au> References: <4D3E9ACD.5020602@skynet.be> <1Phfff-2FiDdA0@fwd06.aul.t-online.de> <4D3EE3E8.70308@skynet.be> <74D61212-85D5-42B3-9583-FFE26C078352@starpower.net> <201101300418.p0U4IHeU012808@mail09.syd.optusnet.com.au> Message-ID: On 30 Jan 2011, at 04:17, Tony Musgrove wrote: > 20 years ago when I first starting taking BLML, it took some time > for me to realise who were the gurus, from whose opinions > one could learn. I used to try to understand the laws from > the book, and from experience and eventually from the > Australian publication "Australian Bridge Director's Bulletin". > However I can remember giving rulings like "there is a big > fight on BLML about this question, so I am not quite sure > what the correct ruling is. I will rule thusly and ask > the question on BLML". One then had to hope to obtain > an answer from one of the gurus.and filter out the garbage. > > I feel that newbie directors who join this list hoping to learn > how the laws are to be applied are lucky to have Grattan's > posts "you are wrong" (not in quite those words > of course). We used to have David Stevenson, John Probst > and quite a few others. I suspect that most newbie directors now may well direct their questions to the International Bridge Laws Forum, moderated by David Stevenson, where they are more likely to get a useful and practical answer. Gordon Rainsford From JffEstrsn at aol.com Sun Jan 30 14:20:02 2011 From: JffEstrsn at aol.com (Jeff Easterson) Date: Sun, 30 Jan 2011 14:20:02 +0100 Subject: [BLML] Exam question In-Reply-To: References: <4D3E9ACD.5020602@skynet.be> <1Phfff-2FiDdA0@fwd06.aul.t-online.de> <4D3EE3E8.70308@skynet.be> <74D61212-85D5-42B3-9583-FFE26C078352@starpower.net> Message-ID: <4D456582.6030806@aol.com> To your two questions: as I have written before, I think it is better to say "I disagree" or "in my opinion" even "I think you are wrong". To say "you are wrong" is absolute and I doubt that any of us can seriously consider our opinions really absolute. To the second question which I assume is a joke or rhetorical (or I have misunderstood it). It would seem obvious that disagreements are allowed on blml (and elsewhere). Why are you asking? The problem is when one of the parties assumes absolute knowledge and repeatedly disqualifies the responses of opponents: "you are wrong", "that is silly" "I expected more from you". (Paraphrasing from memory, not necessarily exact quotations.) I don't think the aim of blml is for one participant to show what he believes is his superiority and better (absolute?) knowledge of the rules and procedure. Thus I recommend "my opinion", etc. I also personally dislike constant repetition. Ciao, JE Am 29.01.2011 22:07, schrieb Jerry Fusselman: > On Tue, Jan 25, 2011 at 11:52 AM, Eric Landau wrote: >> There is a TV show in the US called "The McLaughlin Group", a weekly >> panel discussion of American politics. It is hosted by John >> McLaughlin, who appears with four other panelists. It is famous for >> the following interaction, which occurs with considerable frequency: >> Mr. McLaughlin poses a question to the panel; each of the four >> panelists, in turn, gives essentially the same answer. Mr. >> McLaughlin then announces (his signiture phrase), "The answer is...," >> and proceeds to expound an opinion diametrically opposite to the >> unanimous opinion of his panelists. >> >> Of course, when Mr. McLaughlin does this, he understands that he is >> making a joke. >> > Interesting post. This is what I want to know: Is it acceptable or > unacceptable, in your view, for a BLMLer to say that someone else's > view is wrong, or to say that the answer is X when others on BLML have > given a different answer? Is it acceptable style, or is it > unacceptable bad style? > _______________________________________________ > Blml mailing list > Blml at rtflb.org > http://lists.rtflb.org/mailman/listinfo/blml > From blml at arcor.de Sun Jan 30 15:07:20 2011 From: blml at arcor.de (Thomas Dehn) Date: Sun, 30 Jan 2011 15:07:20 +0100 (CET) Subject: [BLML] judgement In-Reply-To: <4D44246D.2010506@aol.com> References: <4D44246D.2010506@aol.com> Message-ID: <1523228225.338386.1296396440651.JavaMail.ngmail@webmail06.arcor-online.net> Jeff Easterson wrote: > You are playing with a new partner and have few agreements. One is that > non vul an opening weak two can be 5-10 points and always(at least) a 6 > card suit. He opens 2 hearts. You hold: > > AK954 > 4 > AKQ87 > 54 > > What is your bid? Pass. I have 16. On average, partner will have approximately 7. 16+7 won't be enough for game on a misfit deal. > Part 2: If you have agreed to play Ogust and he shows a minimum opening > with a bad trump suit does this alter your decision? If so, to what? You got what you asked for. Ogust is not the right bid here. If you decide to make any attempt to reach game, bid 2S. Thomas From adam at tameware.com Sun Jan 30 15:12:23 2011 From: adam at tameware.com (Adam Wildavsky) Date: Sun, 30 Jan 2011 15:12:23 +0100 Subject: [BLML] ACBL New Orleans NABC Cases Posted In-Reply-To: References: Message-ID: On Tue, Nov 16, 2010 at 4:09 PM, I wrote: > http://www.acbl.org/play/casebooks/NewOrleans2010.html > > If you want to discuss a particular case please include the case > number in the Subject: line and indicate whether it's an NABC+ or > Non-NABC+ case. > > I'll post my comments here shortly. Other panelist comments are not > yet available. All panelist comments are now posted on the ACBL site. -- Adam Wildavsky? ? www.tameware.com From richard.willey at gmail.com Sun Jan 30 15:15:34 2011 From: richard.willey at gmail.com (richard willey) Date: Sun, 30 Jan 2011 09:15:34 -0500 Subject: [BLML] What the GCC allows when competing over an artificial 1C opening In-Reply-To: References: Message-ID: On Sat, Jan 29, 2011 at 3:39 PM, Jerry Fusselman wrote: Anyway, Mr. Campbell kindly relayed my questions for BLML to the > rulings department. And last weekend, Mike Flader gave the answers to > this specific question: Which of these unusual (i.e., a--e say > nothing about spades, and f denies five spades) meanings of 1S in > direct-seat competition over 1C artificial are allowed? > > a. (Alain's question) Less than opening values and no 7-card suit; > b. Less than 3 points and no 8-card suit; > c. Less than opening values; > d. Less than 16 points; > e. More than 16 points; > f. 3 or 4 spades and a balanced hand (at most one doubleton). > > Can anyone guess what the answers to b--f are using the following > official additional hint?: Mike Flader wrote that example a and at > least one of the others is not allowed, "because its primary purpose > is to disrupt the communication of the opponents. [W]hile they tend to > deny something, what they deny is relatively rare and they still say > nothing about the hand." The other hint is that at least one of these > six is GCC-legal. > > Now, from that, can you determine which of b--f are allowed under GCC? > Which are not allowed? Is it predictable, or not? > > I found his answer quite surprising, and I want to see if others can > guess the pattern from the information I gave. If someone can, it > will help me. I really wish to see the pattern so that I know when to > call the director and also which methods to invest time in inventing > and learning. Is it unethical of me to want to play the game by its > rules? > Not at all, however, expecting consistent, intelligible, actionable rulings from the brain trust in Memphis is fruitless at best. I wouldn't be at all surprised to see "f" ruled illegal because you are bidding with a balanced hand and preempting with a balanced hand is a big bugaboo... -- I think back to the halcyon dates of my youth, when indeterminate Hessians had something to do with the Revolutionary War, where conjugate priors were monks who had broken their vows, and the expression (X'X)^-1(X'Y) was greek Those were simpler times -------------- next part -------------- An HTML attachment was scrubbed... URL: http://lists.rtflb.org/pipermail/blml/attachments/20110130/36db68f1/attachment.html From bpark56 at comcast.net Sun Jan 30 16:36:00 2011 From: bpark56 at comcast.net (Robert Park) Date: Sun, 30 Jan 2011 10:36:00 -0500 Subject: [BLML] What the GCC allows when competing over an artificial 1C opening In-Reply-To: References: Message-ID: <4D458560.2090603@comcast.net> On 1/30/11 9:15 AM, richard willey wrote: > On Sat, Jan 29, 2011 at 3:39 PM, Jerry Fusselman > wrote: > > Anyway, Mr. Campbell kindly relayed my questions for BLML to the > rulings department. And last weekend, Mike Flader gave the answers to > this specific question: Which of these unusual (i.e., a--e say > nothing about spades, and f denies five spades) meanings of 1S in > direct-seat competition over 1C artificial are allowed? > > a. (Alain's question) Less than opening values and no 7-card suit; > b. Less than 3 points and no 8-card suit; > c. Less than opening values; > d. Less than 16 points; > e. More than 16 points; > f. 3 or 4 spades and a balanced hand (at most one doubleton). > > Can anyone guess what the answers to b--f are using the following > official additional hint?: Mike Flader wrote that example a and at > least one of the others is not allowed, "because its primary purpose > is to disrupt the communication of the opponents. [W]hile they tend to > deny something, what they deny is relatively rare and they still say > nothing about the hand." The other hint is that at least one of these > six is GCC-legal. > > Now, from that, can you determine which of b--f are allowed under GCC? > Which are not allowed? Is it predictable, or not? > > I found his answer quite surprising, and I want to see if others can > guess the pattern from the information I gave. If someone can, it > will help me. I really wish to see the pattern so that I know when to > call the director and also which methods to invest time in inventing > and learning. Is it unethical of me to want to play the game by its > rules? > > > Not at all, however, expecting consistent, intelligible, actionable > rulings from the brain trust in Memphis is fruitless at best. > > I wouldn't be at all surprised to see "f" ruled illegal because you > are bidding with a balanced hand and preempting with a balanced hand > is a big bugaboo... > > In the methods I play wiith my regular partner, 1S over a strong, artificial 1C shows short hearts and at least 3 cards in every other suit (any strength). This has been ruled acceptable by ACBL directors...on more than one occasion. --bp -------------- next part -------------- An HTML attachment was scrubbed... URL: http://lists.rtflb.org/pipermail/blml/attachments/20110130/11c8b9e2/attachment.html From richard.willey at gmail.com Sun Jan 30 16:52:47 2011 From: richard.willey at gmail.com (richard willey) Date: Sun, 30 Jan 2011 10:52:47 -0500 Subject: [BLML] What the GCC allows when competing over an artificial 1C opening In-Reply-To: <4D458560.2090603@comcast.net> References: <4D458560.2090603@comcast.net> Message-ID: On Sun, Jan 30, 2011 at 10:36 AM, Robert Park wrote: > On 1/30/11 9:15 AM, richard willey wrote: > > In the methods I play wiith my regular partner, 1S over a strong, > artificial 1C shows short hearts and at least 3 cards in every other suit > (any strength). This has been ruled acceptable by ACBL directors...on more > than one occasion. > --bp > And I'm quite sure that other people can produce rulings from ACBL directors that claim the converse... The key issue with the ACBL is the complete lack of consistency, precedence, and any organized system of distributing information. -- I think back to the halcyon dates of my youth, when indeterminate Hessians had something to do with the Revolutionary War, where conjugate priors were monks who had broken their vows, and the expression (X'X)^-1(X'Y) was greek Those were simpler times -------------- next part -------------- An HTML attachment was scrubbed... URL: http://lists.rtflb.org/pipermail/blml/attachments/20110130/dc49b574/attachment-0001.html From bpark56 at comcast.net Sun Jan 30 17:49:01 2011 From: bpark56 at comcast.net (Robert Park) Date: Sun, 30 Jan 2011 11:49:01 -0500 Subject: [BLML] What the GCC allows when competing over an artificial 1C opening In-Reply-To: References: <4D458560.2090603@comcast.net> Message-ID: <4D45967D.3000301@comcast.net> On 1/30/11 10:52 AM, richard willey wrote: > > > On Sun, Jan 30, 2011 at 10:36 AM, Robert Park > wrote: > > On 1/30/11 9:15 AM, richard willey wrote: > > > In the methods I play wiith my regular partner, 1S over a strong, > artificial 1C shows short hearts and at least 3 cards in every > other suit (any strength). This has been ruled acceptable by ACBL > directors...on more than one occasion. > --bp > > > And I'm quite sure that other people can produce rulings from ACBL > directors that claim the converse... > > The key issue with the ACBL is the complete lack of > consistency, precedence, and any organized system of distributing > information. > > > I think the key point is that the call must provide useful information to partner that allows him to take appropriate action. In our case, the 1S overcall I cited allows partner to apply the law of total tricks to bid or jump to an appropriate level. Hence it has a purpose other than pure disruption...though it certainly disrupts as well, as their most likely suit is the one skipped over. --bp -------------- next part -------------- An HTML attachment was scrubbed... URL: http://lists.rtflb.org/pipermail/blml/attachments/20110130/7054b9dd/attachment.html From bmeadows666 at gmail.com Sun Jan 30 17:54:28 2011 From: bmeadows666 at gmail.com (Brian) Date: Sun, 30 Jan 2011 11:54:28 -0500 Subject: [BLML] What the GCC allows when competing over an artificial 1C opening In-Reply-To: <4D458560.2090603@comcast.net> References: <4D458560.2090603@comcast.net> Message-ID: <4D4597C4.1000405@gmail.com> On 01/30/2011 10:36 AM, Robert Park wrote: > On 1/30/11 9:15 AM, richard willey wrote: >> On Sat, Jan 29, 2011 at 3:39 PM, Jerry Fusselman >> > wrote: >> >> Anyway, Mr. Campbell kindly relayed my questions for BLML to the >> rulings department. And last weekend, Mike Flader gave the >> answers to >> this specific question: Which of these unusual (i.e., a--e say >> nothing about spades, and f denies five spades) meanings of 1S in >> direct-seat competition over 1C artificial are allowed? >> >> a. (Alain's question) Less than opening values and no 7-card suit; >> b. Less than 3 points and no 8-card suit; >> c. Less than opening values; >> d. Less than 16 points; >> e. More than 16 points; >> f. 3 or 4 spades and a balanced hand (at most one doubleton). >> >> Can anyone guess what the answers to b--f are using the following >> official additional hint?: Mike Flader wrote that example a and at >> least one of the others is not allowed, "because its primary purpose >> is to disrupt the communication of the opponents. [W]hile they >> tend to >> deny something, what they deny is relatively rare and they still say >> nothing about the hand." The other hint is that at least one of >> these >> six is GCC-legal. >> >> Now, from that, can you determine which of b--f are allowed >> under GCC? >> Which are not allowed? Is it predictable, or not? >> >> I found his answer quite surprising, and I want to see if others can >> guess the pattern from the information I gave. If someone can, it >> will help me. I really wish to see the pattern so that I know >> when to >> call the director and also which methods to invest time in inventing >> and learning. Is it unethical of me to want to play the game by its >> rules? >> >> >> Not at all, however, expecting consistent, intelligible, actionable >> rulings from the brain trust in Memphis is fruitless at best. >> >> I wouldn't be at all surprised to see "f" ruled illegal because you >> are bidding with a balanced hand and preempting with a balanced hand >> is a big bugaboo... >> >> > In the methods I play wiith my regular partner, 1S over a strong, > artificial 1C shows short hearts and at least 3 cards in every other > suit (any strength). This has been ruled acceptable by ACBL > directors...on more than one occasion. Yeah, but you've got a qualification there which turns it into at least a semblance of a three suited hand, and they've been allowed for many years, at least under EBU rules (I don't know the history of ACBL systems regulations). I wonder whether you'd get it through if you dropped that qualification, and made the only requirement at least 3 cards in every suit but hearts, so that you could even make the bid on 3-4-3-3 shape. Personally, I'd expect that you'd meet a bit more resistance to your using that version of the bid. Brian. From jfusselman at gmail.com Sun Jan 30 19:04:25 2011 From: jfusselman at gmail.com (Jerry Fusselman) Date: Sun, 30 Jan 2011 12:04:25 -0600 Subject: [BLML] What the GCC allows when competing over an artificial 1C opening In-Reply-To: <4D4597C4.1000405@gmail.com> References: <4D458560.2090603@comcast.net> <4D4597C4.1000405@gmail.com> Message-ID: Thanks for the replies. f is legal, according to Mike Flader. I would expect him to say that Robert Parks's meaning of 1S, "short hearts and at least 3 cards in every other suit (any strength)", is legal too. I would expect both c and d to be deemed illegal for exactly the same reason as a, and they are. The two that surprised me were b and e. His answers so surprise me, that I cannot help but think that his answers were possibly overhasty. But maybe someone else can predict the answers to those two. What is the pattern? Jerry Fusselman From bpark56 at comcast.net Sun Jan 30 20:18:59 2011 From: bpark56 at comcast.net (Robert Park) Date: Sun, 30 Jan 2011 14:18:59 -0500 Subject: [BLML] What the GCC allows when competing over an artificial 1C opening In-Reply-To: <4D4597C4.1000405@gmail.com> References: <4D458560.2090603@comcast.net> <4D4597C4.1000405@gmail.com> Message-ID: <4D45B9A3.30203@comcast.net> On 1/30/11 11:54 AM, Brian wrote: > On 01/30/2011 10:36 AM, Robert Park wrote: >> On 1/30/11 9:15 AM, richard willey wrote: >>> On Sat, Jan 29, 2011 at 3:39 PM, Jerry Fusselman >>> > wrote: >>> >>> Anyway, Mr. Campbell kindly relayed my questions for BLML to the >>> rulings department. And last weekend, Mike Flader gave the >>> answers to >>> this specific question: Which of these unusual (i.e., a--e say >>> nothing about spades, and f denies five spades) meanings of 1S in >>> direct-seat competition over 1C artificial are allowed? >>> >>> a. (Alain's question) Less than opening values and no 7-card suit; >>> b. Less than 3 points and no 8-card suit; >>> c. Less than opening values; >>> d. Less than 16 points; >>> e. More than 16 points; >>> f. 3 or 4 spades and a balanced hand (at most one doubleton). >>> >>> Can anyone guess what the answers to b--f are using the following >>> official additional hint?: Mike Flader wrote that example a and at >>> least one of the others is not allowed, "because its primary purpose >>> is to disrupt the communication of the opponents. [W]hile they >>> tend to >>> deny something, what they deny is relatively rare and they still say >>> nothing about the hand." The other hint is that at least one of >>> these >>> six is GCC-legal. >>> >>> Now, from that, can you determine which of b--f are allowed >>> under GCC? >>> Which are not allowed? Is it predictable, or not? >>> >>> I found his answer quite surprising, and I want to see if others can >>> guess the pattern from the information I gave. If someone can, it >>> will help me. I really wish to see the pattern so that I know >>> when to >>> call the director and also which methods to invest time in inventing >>> and learning. Is it unethical of me to want to play the game by its >>> rules? >>> >>> >>> Not at all, however, expecting consistent, intelligible, actionable >>> rulings from the brain trust in Memphis is fruitless at best. >>> >>> I wouldn't be at all surprised to see "f" ruled illegal because you >>> are bidding with a balanced hand and preempting with a balanced hand >>> is a big bugaboo... >>> >>> >> In the methods I play wiith my regular partner, 1S over a strong, >> artificial 1C shows short hearts and at least 3 cards in every other >> suit (any strength). This has been ruled acceptable by ACBL >> directors...on more than one occasion. > Yeah, but you've got a qualification there which turns it into at > least a semblance of a three suited hand, and they've been allowed for > many years, at least under EBU rules (I don't know the history of ACBL > systems regulations). > > I wonder whether you'd get it through if you dropped that > qualification, and made the only requirement at least 3 cards in every > suit but hearts, so that you could even make the bid on 3-4-3-3 shape. > Personally, I'd expect that you'd meet a bit more resistance to your > using that version of the bid. > > > Brian. The ACBL's position, as I understand it, is that after an opponent makes an artificial call...such as a takeout double or big club bid...almost anything goes, so long is your action has enough useful content to make it not purely disruptive. --bp From jfusselman at gmail.com Sun Jan 30 20:36:46 2011 From: jfusselman at gmail.com (Jerry Fusselman) Date: Sun, 30 Jan 2011 13:36:46 -0600 Subject: [BLML] What the GCC allows when competing over an artificial 1C opening In-Reply-To: <4D45B9A3.30203@comcast.net> References: <4D458560.2090603@comcast.net> <4D4597C4.1000405@gmail.com> <4D45B9A3.30203@comcast.net> Message-ID: On Sun, Jan 30, 2011 at 1:18 PM, Robert Park wrote: > > The ACBL's position, as I understand it, is that after an opponent makes > an artificial call...such as a takeout double or big club bid...almost > anything goes, so long is your action has enough useful content to make > it not purely disruptive. > ? --bp > Sounds logical. What does that mean to these cases from above? b. Less than 3 points and no 8-card suit; e. More than 16 points; By your criteria, would you expect these 1S "overcalls" to be legal or not? From geller at nifty.com Mon Jan 31 00:45:12 2011 From: geller at nifty.com (Robert Geller) Date: Mon, 31 Jan 2011 08:45:12 +0900 Subject: [BLML] pass out of rotation? Message-ID: <4D45F808.803@nifty.com> South is dealer. North places a pass card on the table at exactly the same instant as South places a 1NT card on the table. How should this situation be adjudicated? -- Robert (Bob) Geller, Tokyo, Japan geller at nifty.com From l.kalbarczyk at gmail.com Mon Jan 31 01:09:06 2011 From: l.kalbarczyk at gmail.com (=?UTF-8?B?xYF1a2FzeiBLYWxiYXJjenlr?=) Date: Mon, 31 Jan 2011 01:09:06 +0100 Subject: [BLML] pass out of rotation? In-Reply-To: <4D45FD26.4060502@gmail.com> References: <4D45F808.803@nifty.com> <4D45FD26.4060502@gmail.com> Message-ID: <4D45FDA2.1000608@gmail.com> W dniu 2011-01-31 01:07, ?ukasz Kalbarczyk pisze: > W dniu 2011-01-31 00:45, Robert Geller pisze: >> South is dealer. North places a pass card on the table at exactly the >> same instant as South places a 1NT card on the table. How should this >> situation be adjudicated? >> > > Looks like 30A: > > In the Polish version we have "no player has bid before". > > English version: > > When a player has passed out of rotation __before any player has bid__ > the offender must pass when next it is his turn to call and Law 23 may > apply. > > Looks better than 30B, for me. And it is corresponding (more) with Law 58. -------------- next part -------------- An HTML attachment was scrubbed... URL: http://lists.rtflb.org/pipermail/blml/attachments/20110131/f6d1ccbd/attachment-0001.html From l.kalbarczyk at gmail.com Mon Jan 31 01:07:02 2011 From: l.kalbarczyk at gmail.com (=?UTF-8?B?xYF1a2FzeiBLYWxiYXJjenlr?=) Date: Mon, 31 Jan 2011 01:07:02 +0100 Subject: [BLML] pass out of rotation? In-Reply-To: <4D45F808.803@nifty.com> References: <4D45F808.803@nifty.com> Message-ID: <4D45FD26.4060502@gmail.com> W dniu 2011-01-31 00:45, Robert Geller pisze: > South is dealer. North places a pass card on the table at exactly the > same instant as South places a 1NT card on the table. How should this > situation be adjudicated? > Looks like 30A: In the Polish version we have "no player has bid before". English version: When a player has passed out of rotation __before any player has bid__ the offender must pass when next it is his turn to call and Law 23 may apply. Looks better than 30B, for me. ?K -------------- next part -------------- An HTML attachment was scrubbed... URL: http://lists.rtflb.org/pipermail/blml/attachments/20110131/ff24a74a/attachment.html From richard.hills at immi.gov.au Mon Jan 31 01:36:05 2011 From: richard.hills at immi.gov.au (richard.hills at immi.gov.au) Date: Mon, 31 Jan 2011 11:36:05 +1100 Subject: [BLML] pass out of rotation? [SEC=UNOFFICIAL] In-Reply-To: <4D45F808.803@nifty.com> Message-ID: Robert (Bob) Geller: >South is dealer. North places a pass card on the table at >exactly the same instant as South places a 1NT card on the >table. How should this situation be adjudicated? Law 33 - Simultaneous Calls A call made simultaneously with one made by the player whose turn it was to call is deemed to be a subsequent call. Richard (Wretched) Hills: So South's legal 1NT opening bid is deemed to be the first call and North is deemed to have passed out of turn at West's turn to call. Under Law 29A East may accept North's pass and if so East is next the one due to call. If East does not accept North's pass, then the applicable Law is Law 30B1 - After Any Player Has Bid When a pass out of rotation is made at offender's RHO's turn to call after any player has bid*, offender must pass when next it is his turn to call. Best wishes Richard (Rtflb) Hills * The word "bid" may perhaps be a typo; the intended meaning may perhaps have been "called". -------------------------------------------------------------------- Important Notice: If you have received this email by mistake, please advise the sender and delete the message and attachments immediately. This email, including attachments, may contain confidential, sensitive, legally privileged and/or copyright information. Any review, retransmission, dissemination or other use of this information by persons or entities other than the intended recipient is prohibited. DIAC respects your privacy and has obligations under the Privacy Act 1988. The official departmental privacy policy can be viewed on the department's website at www.immi.gov.au. See: http://www.immi.gov.au/functional/privacy.htm --------------------------------------------------------------------- From yuen.sebastian at gmail.com Mon Jan 31 02:41:57 2011 From: yuen.sebastian at gmail.com (Sebastian) Date: Mon, 31 Jan 2011 12:41:57 +1100 Subject: [BLML] pass out of rotation? In-Reply-To: <4D45F808.803@nifty.com> References: <4D45F808.803@nifty.com> Message-ID: On 31 January 2011 10:45, Robert Geller wrote: > > South is dealer. ?North places a pass card on the table at exactly the > same instant as South places a 1NT card on the table. ?How should this > situation be adjudicated? > L33: "A call made simultaneously with one made by the player whose turn it was to call is deemed to be a subsequent call." So North's pass is deemed to be subsequent to the legal 1NT opening, and L30B1 applies (North must pass at their next turn to call, assuming the pass isn't accepted). Regards, Sebastian. From bpark56 at comcast.net Mon Jan 31 03:45:26 2011 From: bpark56 at comcast.net (Robert Park) Date: Sun, 30 Jan 2011 21:45:26 -0500 Subject: [BLML] What the GCC allows when competing over an artificial 1C opening In-Reply-To: References: <4D458560.2090603@comcast.net> <4D4597C4.1000405@gmail.com> <4D45B9A3.30203@comcast.net> Message-ID: <4D462246.1090608@comcast.net> On 1/30/11 2:36 PM, Jerry Fusselman wrote: > On Sun, Jan 30, 2011 at 1:18 PM, Robert Park wrote: >> The ACBL's position, as I understand it, is that after an opponent makes >> an artificial call...such as a takeout double or big club bid...almost >> anything goes, so long is your action has enough useful content to make >> it not purely disruptive. >> --bp >> > Sounds logical. What does that mean to these cases from above? > > b. Less than 3 points and no 8-card suit; > e. More than 16 points; > > By your criteria, would you expect these 1S "overcalls" to be legal or not? e sounds OK to me, but wouldn't a double be better for that purpose...if you thought your methods needed such a call? Our experience is that it's much more effective to say something about your shape than your strength when coming in over a strong 1C opening. I'm not thrilled by b, but I have no idea what the ACBL would say. I would vote against b. If allowed, I'm intrigued by the thought that you could get more mileage from it if you made it "less than 3 points and no 9-card suit." --bp From jfusselman at gmail.com Mon Jan 31 04:56:34 2011 From: jfusselman at gmail.com (Jerry Fusselman) Date: Sun, 30 Jan 2011 21:56:34 -0600 Subject: [BLML] What the GCC allows when competing over an artificial 1C opening In-Reply-To: <4D462246.1090608@comcast.net> References: <4D458560.2090603@comcast.net> <4D4597C4.1000405@gmail.com> <4D45B9A3.30203@comcast.net> <4D462246.1090608@comcast.net> Message-ID: On Sun, Jan 30, 2011 at 8:45 PM, Robert Park wrote: > On 1/30/11 2:36 PM, Jerry Fusselman wrote: >> On Sun, Jan 30, 2011 at 1:18 PM, Robert Park wrote: >>> The ACBL's position, as I understand it, is that after an opponent makes >>> an artificial call...such as a takeout double or big club bid...almost >>> anything goes, so long is your action has enough useful content to make >>> it not purely disruptive. >>> ? ?--bp >>> >> Sounds logical. ?What does that mean to these cases from above? >> >> b. ?Less than 3 points and no 8-card suit; >> e. ?More than 16 points; >> >> By your criteria, would you expect these 1S "overcalls" to be legal or not? > > e sounds OK to me, but wouldn't a double be better for that purpose...if > you thought your methods needed such a call? Our experience is that it's > much more effective to say something about your shape than your strength > when coming in over a strong 1C opening. > > I'm not thrilled by b, but I have no idea what the ACBL would say. I > would vote against b. If allowed, I'm intrigued by the thought that you > could get more mileage from it if you made it "less than 3 points and no > 9-card suit." > Well, both b and e are illegal, according to Mike Flader's email to me. Writing about why both a and b are disallowed, he wrote, "because its primary purpose is to disrupt the communication of the opponents. [W]hile they tend to deny something, what they deny is relatively rare and they still say nothing about the hand." But this description seems wrong about b. Either he made a mistake, or responses giving just point-range information are often deemed destructive. But there are lots of point-range-only artificial bids out there. Which are illegal due to being disruptive? Jerry Fusselman From svenpran at online.no Mon Jan 31 08:52:18 2011 From: svenpran at online.no (Sven Pran) Date: Mon, 31 Jan 2011 08:52:18 +0100 Subject: [BLML] pass out of rotation? [SEC=UNOFFICIAL] In-Reply-To: References: <4D45F808.803@nifty.com> Message-ID: <000f01cbc11b$c92739e0$5b75ada0$@no> On Behalf Of richard.hills at immi.gov.au > Robert (Bob) Geller: > > >South is dealer. North places a pass card on the table at exactly the > >same instant as South places a 1NT card on the table. How should this > >situation be adjudicated? > > Law 33 - Simultaneous Calls > > A call made simultaneously with one made by > the player whose turn it was to call is deemed > to be a subsequent call. > > Richard (Wretched) Hills: > > So South's legal 1NT opening bid is deemed to be the first call and North is > deemed to have passed out of turn at West's turn to call. > > Under Law 29A East may accept North's pass and if so East is next the one due to > call. If East does not accept North's pass, then the applicable Law is > > Law 30B1 - After Any Player Has Bid > > When a pass out of rotation is made at > offender's RHO's turn to call after any > player has bid*, offender must pass when next > it is his turn to call. > > Best wishes > > Richard (Rtflb) Hills > > * The word "bid" may perhaps be a typo; the intended meaning may perhaps have > been "called". No, it is not a typo: Until an opening bid has been made (i.e. "before any player has bid") the applicable law is 30A and not 30B. In this case the opening bid has been made (by South) and the pass out of turn is at offender's RHO's turn to call so it is indeed Law 30B1 that applies. From Hermandw at skynet.be Mon Jan 31 08:57:57 2011 From: Hermandw at skynet.be (Herman De Wael) Date: Mon, 31 Jan 2011 08:57:57 +0100 Subject: [BLML] pass out of rotation? In-Reply-To: References: <4D45F808.803@nifty.com> Message-ID: <4D466B85.9070407@skynet.be> Sebastian wrote: > On 31 January 2011 10:45, Robert Geller wrote: >> >> South is dealer. North places a pass card on the table at exactly the >> same instant as South places a 1NT card on the table. How should this >> situation be adjudicated? >> > > L33: "A call made simultaneously with one made by the player whose > turn it was to > call is deemed to be a subsequent call." > > So North's pass is deemed to be subsequent to the legal 1NT opening, > and L30B1 applies (North must pass at their next turn to call, > assuming the pass isn't accepted). > There is one extra bit to the ruling: Since North's pass out of turn was not actually done after South's 1NT, it shows (illegally) that he does not have an opening, not that he has nothing to speak over 1NT with. SO he has a maximum of 10 points, not a mere 7. That is UI to South, who will be bound by this not to reopen when opponents interfere. Of course North may then bid again. > Regards, > > Sebastian. -- Herman De Wael Wilrijk Antwerpen Belgium From agot at ulb.ac.be Mon Jan 31 09:53:44 2011 From: agot at ulb.ac.be (Alain Gottcheiner) Date: Mon, 31 Jan 2011 09:53:44 +0100 Subject: [BLML] judgement In-Reply-To: <4D44246D.2010506@aol.com> References: <4D44246D.2010506@aol.com> Message-ID: <4D467898.7010009@ulb.ac.be> Le 29/01/2011 15:30, Jeff Easterson a ?crit : > You are playing with a new partner and have few agreements. One is that > non vul an opening weak two can be 5-10 points and always(at least) a 6 > card suit. He opens 2 hearts. You hold: > > AK954 > 4 > AKQ87 > 54 > > What is your bid? > Even without many agreements, I'm sure he will take 2S as natural. Perhaps it is forcing, or perhaps it isn't, but I'm in-between ... Seriously, I don't want to miss 4S, perhaps with just Qx in partner's hand, and I don't want to commit to hearts, and I consider 3NT a possibility. So ... > Part 2: If you have agreed to play Ogust and he shows a minimum opening > with a bad trump suit does this alter your decision? If so, to what? > Does it alter my decision not to use Ogust ? Nope. To the contrary, it shows that 2S was the right bid. From agot at ulb.ac.be Mon Jan 31 10:03:56 2011 From: agot at ulb.ac.be (Alain Gottcheiner) Date: Mon, 31 Jan 2011 10:03:56 +0100 Subject: [BLML] What the GCC allows when competing over an artificial 1C opening In-Reply-To: <4D458560.2090603@comcast.net> References: <4D458560.2090603@comcast.net> Message-ID: <4D467AFC.9010804@ulb.ac.be> Le 30/01/2011 16:36, Robert Park a ?crit : > On 1/30/11 9:15 AM, richard willey wrote: >> On Sat, Jan 29, 2011 at 3:39 PM, Jerry Fusselman >> > wrote: >> >> Anyway, Mr. Campbell kindly relayed my questions for BLML to the >> rulings department. And last weekend, Mike Flader gave the >> answers to >> this specific question: Which of these unusual (i.e., a--e say >> nothing about spades, and f denies five spades) meanings of 1S in >> direct-seat competition over 1C artificial are allowed? >> >> a. (Alain's question) Less than opening values and no 7-card suit; >> b. Less than 3 points and no 8-card suit; >> c. Less than opening values; >> d. Less than 16 points; >> e. More than 16 points; >> f. 3 or 4 spades and a balanced hand (at most one doubleton). >> >> Can anyone guess what the answers to b--f are using the following >> official additional hint?: Mike Flader wrote that example a and at >> least one of the others is not allowed, "because its primary purpose >> is to disrupt the communication of the opponents. [W]hile they >> tend to >> deny something, what they deny is relatively rare and they still say >> nothing about the hand." The other hint is that at least one of >> these >> six is GCC-legal. >> >> Now, from that, can you determine which of b--f are allowed under >> GCC? >> Which are not allowed? Is it predictable, or not? >> >> I found his answer quite surprising, and I want to see if others can >> guess the pattern from the information I gave. If someone can, it >> will help me. I really wish to see the pattern so that I know >> when to >> call the director and also which methods to invest time in inventing >> and learning. Is it unethical of me to want to play the game by its >> rules? >> >> >> Not at all, however, expecting consistent, intelligible, actionable >> rulings from the brain trust in Memphis is fruitless at best. >> >> I wouldn't be at all surprised to see "f" ruled illegal because you >> are bidding with a balanced hand and preempting with a balanced hand >> is a big bugaboo... >> >> > In the methods I play wiith my regular partner, 1S over a strong, > artificial 1C shows short hearts and at least 3 cards in every other > suit (any strength). This has been ruled acceptable by ACBL > directors...on more than one occasion. This surely is acceptable, as it is a step to finding a contract (it even suggests three strains). Whetehr the 3-4 card spade bid would be allowed is another story, but I think it should : it suggests that whatever suit partner bids will be welcome (especially spades). But the problem is more fundamental : I think that *any* defensive method against over a strong club has as main purpose the destruction of their lines of communication. Whence the definition is impossible to apply efficiently. Best regards Alain -------------- next part -------------- An HTML attachment was scrubbed... URL: http://lists.rtflb.org/pipermail/blml/attachments/20110131/57413812/attachment-0001.html From agot at ulb.ac.be Mon Jan 31 10:08:40 2011 From: agot at ulb.ac.be (Alain Gottcheiner) Date: Mon, 31 Jan 2011 10:08:40 +0100 Subject: [BLML] What the GCC allows when competing over an artificial 1C opening In-Reply-To: <4D462246.1090608@comcast.net> References: <4D458560.2090603@comcast.net> <4D4597C4.1000405@gmail.com> <4D45B9A3.30203@comcast.net> <4D462246.1090608@comcast.net> Message-ID: <4D467C18.6040500@ulb.ac.be> Le 31/01/2011 3:45, Robert Park a ?crit : > On 1/30/11 2:36 PM, Jerry Fusselman wrote: >> On Sun, Jan 30, 2011 at 1:18 PM, Robert Park wrote: >>> The ACBL's position, as I understand it, is that after an opponent makes >>> an artificial call...such as a takeout double or big club bid...almost >>> anything goes, so long is your action has enough useful content to make >>> it not purely disruptive. >>> --bp >>> >> Sounds logical. What does that mean to these cases from above? >> >> b. Less than 3 points and no 8-card suit; >> e. More than 16 points; >> >> By your criteria, would you expect these 1S "overcalls" to be legal or not? > e sounds OK to me, but wouldn't a double be better for that purpose...if > you thought your methods needed such a call? Our experience is that it's > much more effective to say something about your shape than your strength > when coming in over a strong 1C opening. AG : Please notice that efficiency isn't the criterion here. Disallowing conventions doesn't take it into account, and rightly so. Let natural selection act. From agot at ulb.ac.be Mon Jan 31 10:12:13 2011 From: agot at ulb.ac.be (Alain Gottcheiner) Date: Mon, 31 Jan 2011 10:12:13 +0100 Subject: [BLML] pass out of rotation? In-Reply-To: <4D466B85.9070407@skynet.be> References: <4D45F808.803@nifty.com> <4D466B85.9070407@skynet.be> Message-ID: <4D467CED.1030706@ulb.ac.be> Le 31/01/2011 8:57, Herman De Wael a ?crit : > Sebastian wrote: >> On 31 January 2011 10:45, Robert Geller wrote: >>> South is dealer. North places a pass card on the table at exactly the >>> same instant as South places a 1NT card on the table. How should this >>> situation be adjudicated? >>> >> L33: "A call made simultaneously with one made by the player whose >> turn it was to >> call is deemed to be a subsequent call." >> >> So North's pass is deemed to be subsequent to the legal 1NT opening, >> and L30B1 applies (North must pass at their next turn to call, >> assuming the pass isn't accepted). >> > There is one extra bit to the ruling: > > Since North's pass out of turn was not actually done after South's 1NT, > it shows (illegally) that he does not have an opening, not that he has > nothing to speak over 1NT with. SO he has a maximum of 10 points, not a > mere 7. That is UI to South, who will be bound by this not to reopen > when opponents interfere. Of course North may then bid again. > AG : of course this is void if the 1NT opening is weak. From l.kalbarczyk at gmail.com Mon Jan 31 10:25:22 2011 From: l.kalbarczyk at gmail.com (=?UTF-8?B?xYF1a2FzeiBLYWxiYXJjenlr?=) Date: Mon, 31 Jan 2011 10:25:22 +0100 Subject: [BLML] pass out of rotation? In-Reply-To: References: <4D45F808.803@nifty.com> Message-ID: <4D468002.9030007@gmail.com> W dniu 2011-01-31 02:41, Sebastian pisze: > On 31 January 2011 10:45, Robert Geller wrote: >> South is dealer. North places a pass card on the table at exactly the >> same instant as South places a 1NT card on the table. How should this >> situation be adjudicated? >> > L33: "A call made simultaneously with one made by the player whose > turn it was to > call is deemed to be a subsequent call." Y, it's clear - I forgot about it and found 58 only :) (Was I tired?). But N must pass in both cases (30A1, 30B1), same. A why not 30B2? Because we have 1NT and then (#33) pass out of rotation at offender's RHO's turn to call ?K From svenpran at online.no Mon Jan 31 10:36:18 2011 From: svenpran at online.no (Sven Pran) Date: Mon, 31 Jan 2011 10:36:18 +0100 Subject: [BLML] pass out of rotation? In-Reply-To: <4D467CED.1030706@ulb.ac.be> References: <4D45F808.803@nifty.com> <4D466B85.9070407@skynet.be> <4D467CED.1030706@ulb.ac.be> Message-ID: <001a01cbc12a$501cc100$f0564300$@no> On Behalf Of Alain Gottcheiner > Le 31/01/2011 8:57, Herman De Wael a ?crit : > > Sebastian wrote: > >> On 31 January 2011 10:45, Robert Geller wrote: > >>> South is dealer. North places a pass card on the table at exactly > >>> the same instant as South places a 1NT card on the table. How > >>> should this situation be adjudicated? > >>> > >> L33: "A call made simultaneously with one made by the player whose > >> turn it was to call is deemed to be a subsequent call." > >> > >> So North's pass is deemed to be subsequent to the legal 1NT opening, > >> and L30B1 applies (North must pass at their next turn to call, > >> assuming the pass isn't accepted). > >> > > There is one extra bit to the ruling: > > > > Since North's pass out of turn was not actually done after South's > > 1NT, it shows (illegally) that he does not have an opening, not that > > he has nothing to speak over 1NT with. SO he has a maximum of 10 > > points, not a mere 7. That is UI to South, who will be bound by this > > not to reopen when opponents interfere. Of course North may then bid again. > > > AG : of course this is void if the 1NT opening is weak. No, any information from the withdrawn pass out of turn remains UI to South regardless of the quality etc. of his opening bid. (Law 16D) North has not only "denied" regular opening bid strength but also any hand for a weak opening bid. From Hermandw at skynet.be Mon Jan 31 11:07:14 2011 From: Hermandw at skynet.be (Herman De Wael) Date: Mon, 31 Jan 2011 11:07:14 +0100 Subject: [BLML] pass out of rotation? In-Reply-To: <001a01cbc12a$501cc100$f0564300$@no> References: <4D45F808.803@nifty.com> <4D466B85.9070407@skynet.be> <4D467CED.1030706@ulb.ac.be> <001a01cbc12a$501cc100$f0564300$@no> Message-ID: <4D4689D2.9010709@skynet.be> Sven Pran wrote: > On Behalf Of Alain Gottcheiner >> Le 31/01/2011 8:57, Herman De Wael a ?crit : >>> Sebastian wrote: >>>> On 31 January 2011 10:45, Robert Geller wrote: >>>>> South is dealer. North places a pass card on the table at exactly >>>>> the same instant as South places a 1NT card on the table. How >>>>> should this situation be adjudicated? >>>>> >>>> L33: "A call made simultaneously with one made by the player whose >>>> turn it was to call is deemed to be a subsequent call." >>>> >>>> So North's pass is deemed to be subsequent to the legal 1NT opening, >>>> and L30B1 applies (North must pass at their next turn to call, >>>> assuming the pass isn't accepted). >>>> >>> There is one extra bit to the ruling: >>> >>> Since North's pass out of turn was not actually done after South's >>> 1NT, it shows (illegally) that he does not have an opening, not that >>> he has nothing to speak over 1NT with. SO he has a maximum of 10 >>> points, not a mere 7. That is UI to South, who will be bound by this >>> not to reopen when opponents interfere. Of course North may then bid > again. >>> >> AG : of course this is void if the 1NT opening is weak. > > No, any information from the withdrawn pass out of turn remains UI to South > regardless of the quality etc. of his opening bid. (Law 16D) > > North has not only "denied" regular opening bid strength but also any hand > for a weak opening bid. > For once, I prevented myself from answering no to a post that contained an apparent mistake. Apparently Sven could not contain himself. I am quite certain that Alain realizes that the same principle applies when the 1T is weak. What Alain intended was that the hands that don't speak over a weak 1NT and the hands that don't open, are basically the same. So there is also UI in that case, but it is covered by AI with the same meaning (or rather, UI that has been dealt with). -- Herman De Wael Wilrijk Antwerpen Belgium From agot at ulb.ac.be Mon Jan 31 13:08:31 2011 From: agot at ulb.ac.be (Alain Gottcheiner) Date: Mon, 31 Jan 2011 13:08:31 +0100 Subject: [BLML] pass out of rotation? In-Reply-To: <001a01cbc12a$501cc100$f0564300$@no> References: <4D45F808.803@nifty.com> <4D466B85.9070407@skynet.be> <4D467CED.1030706@ulb.ac.be> <001a01cbc12a$501cc100$f0564300$@no> Message-ID: <4D46A63F.40903@ulb.ac.be> Le 31/01/2011 10:36, Sven Pran a ?crit : > On Behalf Of Alain Gottcheiner >> Le 31/01/2011 8:57, Herman De Wael a ?crit : >>> Sebastian wrote: >>>> On 31 January 2011 10:45, Robert Geller wrote: >>>>> South is dealer. North places a pass card on the table at exactly >>>>> the same instant as South places a 1NT card on the table. How >>>>> should this situation be adjudicated? >>>>> >>>> L33: "A call made simultaneously with one made by the player whose >>>> turn it was to call is deemed to be a subsequent call." >>>> >>>> So North's pass is deemed to be subsequent to the legal 1NT opening, >>>> and L30B1 applies (North must pass at their next turn to call, >>>> assuming the pass isn't accepted). >>>> >>> There is one extra bit to the ruling: >>> >>> Since North's pass out of turn was not actually done after South's >>> 1NT, it shows (illegally) that he does not have an opening, not that >>> he has nothing to speak over 1NT with. SO he has a maximum of 10 >>> points, not a mere 7. That is UI to South, who will be bound by this >>> not to reopen when opponents interfere. Of course North may then bid > again. >> AG : of course this is void if the 1NT opening is weak. > No, any information from the withdrawn pass out of turn remains UI to South > regardless of the quality etc. of his opening bid. (Law 16D) > > North has not only "denied" regular opening bid strength but also any hand > for a weak opening bid. > This is true, but since the hands which make such an opening don't pass over 1NT either, so no UI either. From ehaa at starpower.net Mon Jan 31 15:27:14 2011 From: ehaa at starpower.net (Eric Landau) Date: Mon, 31 Jan 2011 09:27:14 -0500 Subject: [BLML] Exam question In-Reply-To: References: <4D3E9ACD.5020602@skynet.be> <1Phfff-2FiDdA0@fwd06.aul.t-online.de> <4D3EE3E8.70308@skynet.be> <74D61212-85D5-42B3-9583-FFE26C078352@starpower.net> Message-ID: <32442BDD-902B-42D8-94FC-CC9CE29A3F7C@starpower.net> On Jan 29, 2011, at 4:07 PM, Jerry Fusselman wrote: > On Tue, Jan 25, 2011 at 11:52 AM, Eric Landau wrote: > >> There is a TV show in the US called "The McLaughlin Group", a weekly >> panel discussion of American politics. It is hosted by John >> McLaughlin, who appears with four other panelists. It is famous for >> the following interaction, which occurs with considerable frequency: >> Mr. McLaughlin poses a question to the panel; each of the four >> panelists, in turn, gives essentially the same answer. Mr. >> McLaughlin then announces (his signiture phrase), "The answer is...," >> and proceeds to expound an opinion diametrically opposite to the >> unanimous opinion of his panelists. >> >> Of course, when Mr. McLaughlin does this, he understands that he is >> making a joke. > > Interesting post. This is what I want to know: Is it acceptable or > unacceptable, in your view, for a BLMLer to say that someone else's > view is wrong, or to say that the answer is X when others on BLML have > given a different answer? Is it acceptable style, or is it > unacceptable bad style? It depends on the question. As Pat Moynihan famously asserted, ?Everyone is entitled to their own opinion, but not their own facts.? If you present a scenario and ask for a ruling, and get a virtual consensus as to how the law is to be applied in that scenario, you are fully entitled to argue until you are blue in the face that the law is stupid, and ought to be changed, even if everyone else thinks otherwise. It is not so acceptable, however, to argue that everyone else in the forum is stupid, and ought to change their minds about what the law says, even though they all think otherwise. Eric Landau 1107 Dale Drive Silver Spring MD 20910 ehaa at starpower.net From ehaa at starpower.net Mon Jan 31 15:42:49 2011 From: ehaa at starpower.net (Eric Landau) Date: Mon, 31 Jan 2011 09:42:49 -0500 Subject: [BLML] pass out of rotation? In-Reply-To: <4D45F808.803@nifty.com> References: <4D45F808.803@nifty.com> Message-ID: <2A51A207-FABF-441A-904A-96885C143373@starpower.net> On Jan 30, 2011, at 6:45 PM, Robert Geller wrote: > South is dealer. North places a pass card on the table at exactly the > same instant as South places a 1NT card on the table. How should this > situation be adjudicated? As though North had passed after South had opened, by L33; L30B applies. Eric Landau 1107 Dale Drive Silver Spring MD 20910 ehaa at starpower.net From ehaa at starpower.net Mon Jan 31 16:00:04 2011 From: ehaa at starpower.net (Eric Landau) Date: Mon, 31 Jan 2011 10:00:04 -0500 Subject: [BLML] What the GCC allows when competing over an artificial 1C opening In-Reply-To: <4D467AFC.9010804@ulb.ac.be> References: <4D458560.2090603@comcast.net> <4D467AFC.9010804@ulb.ac.be> Message-ID: <972CCAE4-ADEE-4531-96E4-D15DDA4CEA68@starpower.net> On Jan 31, 2011, at 4:03 AM, Alain Gottcheiner wrote: > Le 30/01/2011 16:36, Robert Park a ?crit : > >> On 1/30/11 9:15 AM, richard willey wrote: >> >>> Not at all, however, expecting consistent, intelligible, >>> actionable rulings from the brain trust in Memphis is fruitless >>> at best. >>> >>> I wouldn't be at all surprised to see "f" ruled illegal because >>> you are bidding with a balanced hand and preempting with a >>> balanced hand is a big bugaboo... >> >> In the methods I play wiith my regular partner, 1S over a strong, >> artificial 1C shows short hearts and at least 3 cards in every >> other suit (any strength). This has been ruled acceptable by ACBL >> directors...on more than one occasion. > > This surely is acceptable, as it is a step to finding a contract > (it even suggests three strains). > Whetehr the 3-4 card spade bid would be allowed is another story, > but I think it should : it suggests that whatever suit partner bids > will be welcome (especially spades). > > But the problem is more fundamental : I think that *any* defensive > method against over a strong club has as main purpose the > destruction of their lines of communication. > Whence the definition is impossible to apply efficiently. What definition is that? The ACBL prohibits agreements "whose primary purpose is to destroy the opponents? methods", but I have never seen anything like an official explanation of what that's supposed to mean. Has anyone? Eric Landau 1107 Dale Drive Silver Spring MD 20910 ehaa at starpower.net From agot at ulb.ac.be Mon Jan 31 16:24:01 2011 From: agot at ulb.ac.be (Alain Gottcheiner) Date: Mon, 31 Jan 2011 16:24:01 +0100 Subject: [BLML] What the GCC allows when competing over an artificial 1C opening In-Reply-To: <972CCAE4-ADEE-4531-96E4-D15DDA4CEA68@starpower.net> References: <4D458560.2090603@comcast.net> <4D467AFC.9010804@ulb.ac.be> <972CCAE4-ADEE-4531-96E4-D15DDA4CEA68@starpower.net> Message-ID: <4D46D411.4060406@ulb.ac.be> Le 31/01/2011 16:00, Eric Landau a ?crit : > On Jan 31, 2011, at 4:03 AM, Alain Gottcheiner wrote: > >> Le 30/01/2011 16:36, Robert Park a ?crit : >> >>> On 1/30/11 9:15 AM, richard willey wrote: >>> >>>> Not at all, however, expecting consistent, intelligible, >>>> actionable rulings from the brain trust in Memphis is fruitless >>>> at best. >>>> >>>> I wouldn't be at all surprised to see "f" ruled illegal because >>>> you are bidding with a balanced hand and preempting with a >>>> balanced hand is a big bugaboo... >>> In the methods I play wiith my regular partner, 1S over a strong, >>> artificial 1C shows short hearts and at least 3 cards in every >>> other suit (any strength). This has been ruled acceptable by ACBL >>> directors...on more than one occasion. >> This surely is acceptable, as it is a step to finding a contract >> (it even suggests three strains). >> Whetehr the 3-4 card spade bid would be allowed is another story, >> but I think it should : it suggests that whatever suit partner bids >> will be welcome (especially spades). >> >> But the problem is more fundamental : I think that *any* defensive >> method against over a strong club has as main purpose the >> destruction of their lines of communication. >> Whence the definition is impossible to apply efficiently. > What definition is that? The ACBL prohibits agreements "whose > primary purpose is to destroy the opponents? methods", but I have > never seen anything like an official explanation of what that's > supposed to mean. Has anyone? > Yes, that's what I meant. the class of prohibited agreements includes 99% of the agreements used after a strong club. From svenpran at online.no Mon Jan 31 16:36:16 2011 From: svenpran at online.no (Sven Pran) Date: Mon, 31 Jan 2011 16:36:16 +0100 Subject: [BLML] What the GCC allows when competing over an artificial 1C opening In-Reply-To: <972CCAE4-ADEE-4531-96E4-D15DDA4CEA68@starpower.net> References: <4D458560.2090603@comcast.net> <4D467AFC.9010804@ulb.ac.be> <972CCAE4-ADEE-4531-96E4-D15DDA4CEA68@starpower.net> Message-ID: <000001cbc15c$9a2bb9e0$ce832da0$@no> On Behalf Of Eric Landau ............... > What definition is that? The ACBL prohibits agreements "whose primary purpose > is to destroy the opponents' methods", but I have never seen anything like an > official explanation of what that's supposed to mean. Has anyone? There cannot be any such definition for the simple reason that it would be meaningless. Bridge is a game in which the purpose is to obtain an as high a score as possible on each board (where net scores to opponents count as negative scores). Any agreement that aids to achieve this goal is constructive for own side and as a consequence destructive for opposing side. This is the prime purpose with for instance pre-emptive and sacrifice bids, but I would like to see any NBO trying to prohibit me (holding an otherwise valueless hand with say 9 small spades) from bidding 4S over opponents' 1H opening bid in order to make it difficult for them to find their best contgract? The primary purpose of my bid in this case is of course to destroy the opponents' methods. If my undertrick penalty is higher than the score opponents can make in their contract I have lost, otherwise I have won. It is simple as that. What is meaningful is to require for instance that a call cannot legally be made completely at random, i.e. a 1S bid that can be made with any distribution and/or any strength without any restriction should not be legal. From JffEstrsn at aol.com Mon Jan 31 16:45:11 2011 From: JffEstrsn at aol.com (Jeff Easterson) Date: Mon, 31 Jan 2011 16:45:11 +0100 Subject: [BLML] What the GCC allows when competing over an artificial 1C opening In-Reply-To: <000001cbc15c$9a2bb9e0$ce832da0$@no> References: <4D458560.2090603@comcast.net> <4D467AFC.9010804@ulb.ac.be> <972CCAE4-ADEE-4531-96E4-D15DDA4CEA68@starpower.net> <000001cbc15c$9a2bb9e0$ce832da0$@no> Message-ID: <4D46D907.20206@aol.com> About 45 years ago,playing at a club in Berlin (Germany) I made a quite normal weak two opening. This almost "ruined" the game. Great excitement, protest, censure. Considered apparently a purely destructive bid. Only after presenting evidence of players in other countries in which weak twos were nromal was I grudgingly allowed to use such bids and not thrown out of the club. JE Am 31.01.2011 16:36, schrieb Sven Pran: > On Behalf Of Eric Landau > ............... >> What definition is that? The ACBL prohibits agreements "whose primary > purpose >> is to destroy the opponents' methods", but I have never seen anything like > an >> official explanation of what that's supposed to mean. Has anyone? > There cannot be any such definition for the simple reason that it would be > meaningless. > > Bridge is a game in which the purpose is to obtain an as high a score as > possible on each board (where net scores to opponents count as negative > scores). > > Any agreement that aids to achieve this goal is constructive for own side > and as a consequence destructive for opposing side. > > This is the prime purpose with for instance pre-emptive and sacrifice bids, > but I would like to see any NBO trying to prohibit me (holding an otherwise > valueless hand with say 9 small spades) from bidding 4S over opponents' 1H > opening bid in order to make it difficult for them to find their best > contgract? > > The primary purpose of my bid in this case is of course to destroy the > opponents' methods. If my undertrick penalty is higher than the score > opponents can make in their contract I have lost, otherwise I have won. It > is simple as that. > > What is meaningful is to require for instance that a call cannot legally be > made completely at random, i.e. a 1S bid that can be made with any > distribution and/or any strength without any restriction should not be > legal. > > _______________________________________________ > Blml mailing list > Blml at rtflb.org > http://lists.rtflb.org/mailman/listinfo/blml > From adam at irvine.com Mon Jan 31 16:50:09 2011 From: adam at irvine.com (Adam Beneschan) Date: Mon, 31 Jan 2011 07:50:09 -0800 Subject: [BLML] What the GCC allows when competing over an artificial 1C opening In-Reply-To: Your message of "Sun, 30 Jan 2011 12:04:25 CST." Message-ID: <20110131155010.D86A6A8C87D@mailhub.irvine.com> Jerry wrote: > Thanks for the replies. > > f is legal, according to Mike Flader. I would expect him to say that > Robert Parks's meaning of 1S, "short hearts and at least 3 cards in > every other suit (any strength)", is legal too. I would expect both > c and d to be deemed illegal for exactly the same reason as a, and > they are. The two that surprised me were b and e. His answers so > surprise me, that I cannot help but think that his answers were > possibly overhasty. But maybe someone else can predict the answers to > those two. What is the pattern? I won't say anything about (b), but (e) seems like a mistake---an artificial opening bid that shows more than 16 points and says nothing about the hand is clearly legal, so it doesn't make sense that an overcall that shows the same sort of hand would be illegal. My guess is that Mike got into a groove and after answering (d), didn't notice that (e) was fundamentally different from (d) and should have been answered differently. -- Adam From agot at ulb.ac.be Mon Jan 31 17:11:34 2011 From: agot at ulb.ac.be (Alain Gottcheiner) Date: Mon, 31 Jan 2011 17:11:34 +0100 Subject: [BLML] What the GCC allows when competing over an artificial 1C opening In-Reply-To: <20110131155010.D86A6A8C87D@mailhub.irvine.com> References: <20110131155010.D86A6A8C87D@mailhub.irvine.com> Message-ID: <4D46DF36.3020106@ulb.ac.be> Le 31/01/2011 16:50, Adam Beneschan a ?crit : > Jerry wrote: > >> Thanks for the replies. >> >> f is legal, according to Mike Flader. I would expect him to say that >> Robert Parks's meaning of 1S, "short hearts and at least 3 cards in >> every other suit (any strength)", is legal too. I would expect both >> c and d to be deemed illegal for exactly the same reason as a, and >> they are. The two that surprised me were b and e. His answers so >> surprise me, that I cannot help but think that his answers were >> possibly overhasty. But maybe someone else can predict the answers to >> those two. What is the pattern? > I won't say anything about (b), but (e) seems like a mistake---an > artificial opening bid that shows more than 16 points and says nothing > about the hand is clearly legal, so it doesn't make sense that an > overcall that shows the same sort of hand would be illegal. AG : I beg to differ. First, there are many examples of opening bids that are legal, while the corresponding overcall isn't, in the same settings - e.g. Multi 2D overcall. Second, a 1S opening, showing 16+, is disallowed in many competitions - so, why not an overcall ? I think that the whole idea of the proviso "not purely destructive" is that one should not be allowed to make an obstructive, non-descriptive overcall. One could perhaps demand that the conditions for the bid be positive, not exclusive ("shows" rather than "denies"). This wouldn't disallow the semi-3-suited 1S bid, nor the balanced, semi-natural 1S bid, nor CRaSh-type bids, not even the mini-terrorist 1S bid (5+ cards in any suit). But then one should say it that way, because "aiming mainly at disruption" is what overcalls of a strong club*all* are. From nigelguthrie at yahoo.co.uk Mon Jan 31 17:14:38 2011 From: nigelguthrie at yahoo.co.uk (Nigel Guthrie) Date: Mon, 31 Jan 2011 16:14:38 +0000 (GMT) Subject: [BLML] What the GCC allows when competing over an artificial 1C opening In-Reply-To: <20110131155010.D86A6A8C87D@mailhub.irvine.com> References: <20110131155010.D86A6A8C87D@mailhub.irvine.com> Message-ID: <141831.80044.qm@web28514.mail.ukl.yahoo.com> It used to be old-guard club-players, who objected to anything new. Now, it is top US professional teams who have invested time and effort in traditional methods and want to protect themselves and their sponsors from innovation. Bridge is a really great game to be able to survive increasingly incomprehensible laws and heaps of chauvinistic local regulations. From lavaldubreuil at xplornet.com Mon Jan 31 17:22:08 2011 From: lavaldubreuil at xplornet.com (laval dubreuil) Date: Mon, 31 Jan 2011 11:22:08 -0500 Subject: [BLML] Out of Turn Faced-down Opening Lead Message-ID: <000001cbc163$027c3a50$0774aef0$@com> Hi all, Law 54A allows declarer spread his hand after a faced opening lead out of turn, becoming dummy. What is the ruling when declarer spreads his hand after an out of turn faced-down opening lead ? Law 54C requires declarer accept a faced opening lead out of turn if he could have seen any dummy's cards (except...Law 24). What is the ruling when the out of turn opening lead was faced down and dummy then expose some cards ? Law 41A has no such specific provisions. Laval Du Breuil Quebec From adam at irvine.com Mon Jan 31 17:38:43 2011 From: adam at irvine.com (Adam Beneschan) Date: Mon, 31 Jan 2011 08:38:43 -0800 Subject: [BLML] What the GCC allows when competing over an artificial 1C opening In-Reply-To: Your message of "Mon, 31 Jan 2011 17:11:34 +0100." <4D46DF36.3020106@ulb.ac.be> Message-ID: <20110131163844.F3039A8C87D@mailhub.irvine.com> Alain wrote: > > I won't say anything about (b), but (e) seems like a mistake---an > > artificial opening bid that shows more than 16 points and says nothing > > about the hand is clearly legal, so it doesn't make sense that an > > overcall that shows the same sort of hand would be illegal. > AG : I beg to differ. > > First, there are many examples of opening bids that are legal, while the > corresponding overcall isn't, in the same settings - e.g. Multi 2D overcall. > Second, a 1S opening, showing 16+, is disallowed in many competitions - > so, why not an overcall ? Point taken. If there were a list of specific conventions that were or were not allowed over a strong club, and the list said that certain bids that were OK as opening bids were not allowed as overcalls, I might think it was a dumb rule, but at least the organization is within their rights to make rules like that. However, the argument here isn't about whether the GCC's rules are wise or not; the argument is that, given that the GCC rules are what they are, whether an overcall that shows 16+ falls within the rules. And I don't see how a bid that is allowed as an opening bid can be considered a bid whose "primary purpose is to destroy the opponents' methods" when used as an overcall. I can see that I was careless in my wording, in the paragraph you quoted. >From a bridge logic standpoint, I don't see how an overcall like this *could* have destruction as its primary purpose. If righty opens 16+ and you make a bid that shows 17+, you're telling the opponents, "Your side is much less likely to have a game, so there's less need for you to try hard to be accurate in your own bidding". In other words, it doesn't interfere with the opponents' methods---it tells them that they don't need good methods. I think it also tells partner that he shouldn't be intimidated by the opponents' strong club---we may have a game of our own, let's try to find it. So the idea that this bid can fall into the "conventions whose primary purpose is to destroy the opponents' methods" is, well, ludicrous, and I stand by my suggestion that Flader just made an error. -- Adam From PeterEidt at t-online.de Mon Jan 31 17:51:41 2011 From: PeterEidt at t-online.de (Peter Eidt) Date: Mon, 31 Jan 2011 17:51:41 +0100 Subject: [BLML] =?utf-8?q?Out_of_Turn_Faced-down_Opening_Lead?= In-Reply-To: <000001cbc163$027c3a50$0774aef0$@com> References: <000001cbc163$027c3a50$0774aef0$@com> Message-ID: <1Pjwyb-15p1tI0@fwd11.aul.t-online.de> From: "laval dubreuil" > Law 54A allows declarer spread his hand after a faced opening lead out > of turn, becoming dummy. > What is the ruling when declarer spreads his hand after an out of turn > faced-down opening lead ? I would apply Law 24 (see Law 22 B1); and I would consider Law 23 (less probable) and Law 12 A1. > Law 54C requires declarer accept a faced opening lead out of turn if > he could have seen any dummy's cards (except...Law 24). > What is the ruling when the out of turn opening lead was faced down > and dummy then expose some cards ? Then we _are_ in Law 24 and thus in middle of the exception. > Law 41A has no such specific provisions. From agot at ulb.ac.be Mon Jan 31 18:01:33 2011 From: agot at ulb.ac.be (Alain Gottcheiner) Date: Mon, 31 Jan 2011 18:01:33 +0100 Subject: [BLML] What the GCC allows when competing over an artificial 1C opening In-Reply-To: <20110131163844.F3039A8C87D@mailhub.irvine.com> References: <20110131163844.F3039A8C87D@mailhub.irvine.com> Message-ID: <4D46EAED.7060608@ulb.ac.be> Le 31/01/2011 17:38, Adam Beneschan a ?crit : > Alain wrote: > >>> I won't say anything about (b), but (e) seems like a mistake---an >>> artificial opening bid that shows more than 16 points and says nothing >>> about the hand is clearly legal, so it doesn't make sense that an >>> overcall that shows the same sort of hand would be illegal. >> AG : I beg to differ. >> >> First, there are many examples of opening bids that are legal, while the >> corresponding overcall isn't, in the same settings - e.g. Multi 2D overcall. >> Second, a 1S opening, showing 16+, is disallowed in many competitions - >> so, why not an overcall ? > Point taken. If there were a list of specific conventions that were > or were not allowed over a strong club, and the list said that certain > bids that were OK as opening bids were not allowed as overcalls, I > might think it was a dumb rule, but at least the organization is > within their rights to make rules like that. > > However, the argument here isn't about whether the GCC's rules are > wise or not; the argument is that, given that the GCC rules are what > they are, whether an overcall that shows 16+ falls within the rules. > And I don't see how a bid that is allowed as an opening bid can be > considered a bid whose "primary purpose is to destroy the opponents' > methods" when used as an overcall. AG : there is no contradicti?on up to now. Abid might have a destructive purpose and be allowed as an opening bid (e.g. Multi). In that case, it might *both* be allowed as an opening and be destructive as an overcall. From jfusselman at gmail.com Mon Jan 31 19:57:01 2011 From: jfusselman at gmail.com (Jerry Fusselman) Date: Mon, 31 Jan 2011 12:57:01 -0600 Subject: [BLML] What the GCC allows when competing over an artificial 1C opening In-Reply-To: <4D467AFC.9010804@ulb.ac.be> References: <4D458560.2090603@comcast.net> <4D467AFC.9010804@ulb.ac.be> Message-ID: On Mon, Jan 31, 2011 at 3:03 AM, Alain Gottcheiner wrote: > Whetehr the 3-4 card spade bid would be allowed is another story, but I > think it should : it suggests that whatever suit partner bids will be > welcome (especially spades). Yes, the 3-4-card spade bid turns out to be quite constructive, and partner frequently bids. > > But the problem is more fundamental : I think that *any* defensive method > against over a strong club has as main purpose the destruction of their > lines of communication. > Whence the definition is impossible to apply efficiently. > Right, not easy to apply efficiently. Defensive methods against a strong club might 1. hurt their lines communication, 2. help find a good sacrifice, 3. suggest a damaging lead. Seems to me that a system that is helpful in all three areas is likely to be more valuable than one that only obstructs. Jerry From svenpran at online.no Mon Jan 31 22:45:41 2011 From: svenpran at online.no (Sven Pran) Date: Mon, 31 Jan 2011 22:45:41 +0100 Subject: [BLML] Out of Turn Faced-down Opening Lead In-Reply-To: <1Pjwyb-15p1tI0@fwd11.aul.t-online.de> References: <000001cbc163$027c3a50$0774aef0$@com> <1Pjwyb-15p1tI0@fwd11.aul.t-online.de> Message-ID: <000601cbc190$354dbe90$9fe93bb0$@no> On Behalf Of Peter Eidt > From: "laval dubreuil" > > Law 54A allows declarer spread his hand after a faced opening lead out > > of turn, becoming dummy. > > What is the ruling when declarer spreads his hand after an out of turn > > faced-down opening lead ? > > I would apply Law 24 (see Law 22 B1); and I would consider Law 23 (less > probable) and Law 12 A1. Precisely ! > > Law 54C requires declarer accept a faced opening lead out of turn if > > he could have seen any dummy's cards (except...Law 24). > > What is the ruling when the out of turn opening lead was faced down > > and dummy then expose some cards ? > > Then we _are_ in Law 24 and thus in middle of the exception. Again: Precisely ! > > Law 41A has no such specific provisions.